You are on page 1of 524

Corporate

DISHA PUBLICATION
Office
45, 2nd Floor, Maharishi Dayanand Marg,
Corner Market, Malviya Nagar, New Delhi - 110017
Tel : 49842349 / 49842350

© Copyright
No part of this publication may be reproduced in
any form without prior permission of the publisher.

Disha
The author and the publisher do not take any legal
responsibility for any errors or misrepresentations that
might have crept in. We have tried and made our best
efforts to provide accurate up-to-date information in
this book.
All Right Reserved

Disha Experts
Sanjeev Kumar Jha
Govind Thakur

Typeset by Disha DTP Team

www.dishapublication.com www.mylearninggraph.com
Books & Etests
ebooks for
School & for
Competitive Competitive
Exams Exams

Write to us at feedback_disha@aiets.co.in
Contents
1. Physical World, Units and Measurements P-1 – P-13


Topic 1 : Unit of Physical Quantities

Topic 2 : Dimensions of Physical Quantities

Topic 3 : Errors in Measurements

2. Motion in a Straight Line P-14 – P-25


Topic 1 : Distance, Displacement & Uniform Motion

Topic 2 : Non-uniform Motion

Topic 3 : Relative Velocity

Topic 4 : Motion Under Gravity

3. Motion in a Plane P-26 – P-35


Topic 1 : Vectors

Topic 2 : Motion in a Plane with Constant Acceleration

Topic 3 : Projectile Motion

Topic 4 : Relative Velocity in Two Dimensions & Uniform Circular Motion

4. Laws of Motion P-36 – P-53


Topic 1 : Ist, IInd & IIIrd Laws of Motion

Topic 2 : Motion of Connected Bodies, Pulley & Equilibrium of Forces

Topic 3 : Friction

Topic 4 : Circular Motion, Banking of Road

5. Work, Energy and Power P-54 – P-75


Topic 1 : Work

Topic 2 : Energy

Topic 3 : Power

Topic 4 : Collisions

6. System of Particles and Rotational Motion P-76 – P-112


Topic 1 : Centre of Mass, Centre of Gravity & Principle of Moments

Topic 2 : Angular Displacement, Velocity and Acceleration

Topic 3 : Torque, Couple and Angular Momentum

Topic 4 : Moment of Inertia and Rotational K.E.

Topic 5 : Rolling Motion

7. Gravitation P-113 – P-130


Topic 1 : Kepler’s Laws of Planetary Motion

Topic 2 : Newton’s Universal Law of Gravitation

Topic 3 : Acceleration due to Gravity

Topic 4 : Gravitational Field and Potential Energy

Topic 5 : Motion of Satellites, Escape Speed and Orbital Velocity

8. Mechanical Properties of Solids P-131 – P-138


Topic 1 : Hooke’s Law & Young’s Modulus

Topic 2 : Bulk and Rigidity Modulus and Work Done in Stretching a Wire

9. Mechanical Properties of Fluids P-139 – P-154


Topic 1 : Pressure, Density, Pascal’s Law and Archimedes’ Principle

Topic 2 : Fluid Flow, Reynold’s Number and Bernoulli’s Principle

Topic 3 : Viscosity and Terminal Velocity

Topic 4 : Surface Tension, Surface Energy and Capillarity

10. Termal Properties of Matter P-155 – P-168


Topic 1 : Termometer & Termal Expansion

Topic 2 : Calorimetry and Heat Transfer

Topic 3 : Newton’s Law of Cooling

11. Termodynamics P-169 – P-185


Topic 1 : First Law of Termodynamics

Topic 2 : Specifc Heat Capacity and Termodynamical Processes

Topic 3 : Carnot Engine, Refrigerators and Second Law of Termodynamics

12. Kinetic Teory P-186 – P-198


Topic 1 : Kinetic Teory of an Ideal Gas and Gas Laws

Topic 2 : Speed of Gas, Pressure and Kinetic Energy

Topic 3 : Degree of Freedom, Specifc Heat Capacity, and Mean Free Path

13. Oscillations P-199 – P-218


Topic 1 : Displacement, Phase, Velocity and Acceleration in S.H.M.

Topic 2 : Energy in Simple Harmonic Motion

Topic 3 : Time Period, Frequency, Simple Pendulum and Spring Pendulum

Topic 4 : Damped, Forced Oscillations and Resonance

14. Waves P-219 – P-234


Topic 1 : Basic of Mechanical Waves, Progressive and Stationary Waves

Topic 2 : Vibration of String and Organ Pipe

Topic 3 : Beats, Interference and Superposition of Waves

Topic 4 : Musical Sound and Doppler’s Efect

15. Electric Charges and Fields P-235 – P-253


Topic 1 : Electric Charges and Coulomb’s Law

Topic 2 : Electric Field and Electric Field Lines

Topic 3 : Electric Dipole, Electric Flux and Gauss’s Law

16. Electrostatic Potential and Capacitance P-254 – P-280


Topic 1 : Electrostatic Potential and Equipotential Surfaces

Topic 2 : Electric Potential Energy and Work Done in Carrying a Charge

Topic 3 : Capacitors, Grouping of Capacitor and Energy Stored in a Capacitor

17. Current Electricity P-281 – P-311


Topic 1 : Electric Current, Drif of Electrons, Ohm’s Law, Resistance and Resistivity

Topic 2 : Combination of Resistances

Topic 3 : Kirchhof ’s Laws, Cells, Termo e.m.f. Electrolysis

Topic 4 : Heating Efect of Current

Topic 5 : Wheatstone Bridge and Diferent Measuring Instruments

18. Moving Charges and Magnetism P-312 – P-339


Topic 1 : Motion of Charged Particle in Magnetic Field

Topic 2 : Magnetic Field Lines, Biot-Savart’s Law and Ampere’s Circuital Law

Topic 3 : Force and Torque on Current Carrying Conductor

Topic 4 : Galvanometer and its Conversion into Ammeter and Voltmeter

19. Magnetism and Matter P-340 – P-347


Topic 1 : Magnetism, Gauss’s Law, Magnetic Moment, Properties of Magnet

Topic 2 : Te Earth Magnetism, Magnetic Materials and their Properties

Topic 3 : Magnetic Equipment

20. Electromagnetic Induction P-348 – P-360


Topic 1 : Magnetic Flux, Faraday’s and Lenz’s Law

Topic 2 : Motional and Static EMI and Application of EMI

21. Alternating Current P-361 – P-376


Topic 1 : Alternating Current, Voltage and Power

Topic 2 : AC Circuit, LCR Circuit, Quality and Power Factor

Topic 3 : Transformers and LC Oscillations

22. Electromagnetic Waves P-377 – P-388


Topic 1 : Electromagnetic Waves, Conduction and Displacement Current

Topic 2 : Electromagnetic Spectrum

23. Ray Optics and Optical Instruments P-389 – P-414


Topic 1 : Plane Mirror, Spherical Mirror and Refection of Light

Topic 2 : Refraction of Light at Plane Surface and Total Internal Refection

Topic 3 : Refraction at Curved Surface Lenses and Power of Lens

Topic 4 : Prism and Dispersion of Light

Topic 5 : Optical Instruments

24. Wave Optics P-415 – P-432


Topic 1 : Wavefront, Interference of Light, Coherent and Incoherent Sources

Topic 2 : Young’s Double Slit Experiment

Topic 3 : Difraction, Polarisation of Light and Resolving Power

25. Dual Nature of Radiation and Matter P-433 – P-448


Topic 1 : Matter Waves, Cathode and Positive Rays

Topic 2 : Photon, Photoelectric Efect X-rays and Davisson-Germer Experiment

26. Atoms P-449 – P-460


Topic 1 : Atomic Structure and Rutherford’s Nuclear Model

Topic 2 : Bohr’s Model and the Spectra of the Hydrogen Atom

27. Nuclei P-461 – P-472


Topic 1 : Composition and Size of the Nuclei

Topic 2 : Mass-Energy Equivalence and Nuclear Reactions

Topic 3 : Radioactivity

28. Semiconductor Electronics : Materials, Devices and Simple Circuits P-473 – P-493


Topic 1 : Solids, Semiconductors and P-N Junction Diode

Topic 2 : Junction Transistor

Topic 3 : Digital Electronics and Logic Gates

29. Communication Systems P-494 – P-500


Topic 1 : Communication Systems

Mock Test 1 with Solutions MT-1 – MT-8

Mock Test 2 with Solutions MT-9 – MT-16

Opening/ Closing Rank for TOP NITs & List of NITs in India
Opening/ Closing Rank for Top NITS
College Name OR/CR CSE ECE ME EE/EEE
NIT Trichy ORF 2060 5325 4154 5708
CR 5317 8011 12970 10353
NIT Rourkela OR 2253 8571 11662 4084
CR 9420 12009 20304 19168
NIT Surathkal OR 960 3378 6315 3456
CR 3181 5608 11788 6801
NIT Warangal OR 978 2919 4340 5270
CR 2341 2919 10209 8152
NIT Calicut OR 2201 8023 10629 9703
CR 10222 14769 20480 18966
NIT Kurukshetra OR 2268 8320 11195 9454
CR 6170 12067 18115 16273
NIT Durgapur OR 5611 12509 14511 13595
CR 12095 16098 22753 19325
MNIT Allahabad OR 1449 3600 5884 5879
CR 4051 7128 11145 8790
NIT Silchar OR 8699 17899 21851 32579
CR 23882 40841 49215 56958
MNIT Jaipur OR 1148 3881 9277 4119
CR 3831 7868 11426 9179

List of NITs in India


After IITs, NITs form the second layer of topmost engineering Institutes in India
Rank Of (Amongst NITs) Name State NIRF Score NIRF Ranking
1 NIT Trichy Tamil Nadu 61.62 10
2 NIT Rourkela Odisha 57.75 16
3 NIT Karnataka Karnataka 55.25 21
4 NIT Warangal Telangana 53.21 26
5 NIT Calicut Kerala 52.69 28
6 V-NIT Maharashtra 51.27 31
7 NIT Kurukshetra Haryana 47.58 41
7 MN-NIT Uttar Pradesh 47.49 42
8 NIT Durgapur West Bengal 46.47 46
9 NIT Silchar Assam 45.61 51
10 M-NIT Rajasthan 45.20 53
11 SV-NIT Gujarat 41.88 58
12 NIT Hamirpur Himachal Pradesh 41.48 60
13 MA-NIT Madhya Pradesh 40.98 62
14 NITIE Maharashtra 40.48 66
15 NIT Meghalaya Meghalaya 40.32 67
16 NIT Agartala Tripura 39.53 70
17 NIT Raipur Chattisgarh 39.09 74
18 NIT Goa Goa 37.06 87
FAQs - Frequently Asked Questions
QUESTION: Which Colleges other than IITs accept JEE Ad- at the end of each paper of the examination, given to them at the start
vanced scores? of the paper.
ANSWER: QUESTION: During examination can I change my answers?
1. Institute of Science (IISc), Bangalore ANSWER: Candidate will have the option to change previously

2. Indian Institute of Petroleum and Energy (IIPE), Visakhapatnam saved answer of any question, anytime during the entire duration of
the test.

3. Indian Institutes of Science Education and Research (IISER),

Bhopal QUESTION: How can I change a previously saved answer during the
CBT of JEE (Advanced)-2018?
4. Indian Institutes of Science Education and Research (IISER),

Mohali ANSWER: To change the answer of a question that has already been
5. Indian Institutes of Science Education and Research (IISER), answered and saved, first select the corresponding question from
the Question Palette, then click on “Clear Response” to clear the

Kolkata
previously entered answer and subsequently follow the procedure for
6. Indian Institutes of Science Education and Research (IISER),
answering that type of question.

Pune
7. Indian Institutes of Science Education and Research (IISER), QUESTION: Will I be given a printout/hard copy of the questions
papers along with my responses to questions in Paper-I and Paper-II

Thiruvananthapuram
after the completion of the respective papers?
8. Indian Institute of Space Science and Technology (IIST), Thiru-

vananthapuram ANSWER: No.
9. Rajiv Gandhi Institute of Petroleum Technology (RGIPT), Rae QUESTION: How will I be getting a copy of the questions papers

Bareli and my responses to questions in Paper-I and Paper-II?
QUESTION: If I am absent in one of the papers (Paper 1, Paper 2), ANSWER: The responses of all the candidates who have appeared
will my result be declared? for both Paper 1 and Paper 2, recorded during the exam, along with
ANSWER: NO. You will be considered absent in JEE the questions of each paper, will be electronically mailed to their
(Advanced)-2018 and the result will not be prepared/declared. It is registered email ids, by Friday, May 25, 2018, 10:00 IST.
compulsory to appear in both the papers for result preparation. QUESTION: Suppose two candidates have same JEE
QUESTION: Do I have to choose my question paper language at the (Advanced)-2018 aggregate marks. Will the two candidates be given
time of JEE (Advanced)-2018 registration? the same rank?
ANSWER: NO. There is no need to indicate question paper language ANSWER: If the aggregate marks scored by two or more candidates
at the time of JEE (Advanced)-2018 registration. Candidates will are the same, then the following tie-break policy will be used for
have the option to choose their preferred language (English or Hindi), awarding ranks: Step 1: Candidates having higher positive marks
as the default language for viewing the questions, at the start of the will be awarded higher rank. If the tie breaking criterion at Step 1
Computer Based Test (CBT) examination of JEE (Advanced)-2018. fails to break the tie, then the following criterion at Step 2 will be
QUESTION: Can I change the language (from English to Hindi followed. Step 2: Higher rank will be assigned to the candidate who
and vice versa) of viewing the questions during the CBT of JEE has obtained higher marks in Mathematics. If this does not break the
(Advanced)-2018? tie, higher rank will be assigned to the candidate who has obtained
higher marks in Physics. If there is a tie even after this, candidates will
ANSWER: Questions will be displayed on the screen of the Candidate
in the chosen default language (English or Hindi). Further, the be assigned the same rank.
candidate can also switch/toggle between English or Hindi languages, QUESTION: I have read in newspapers that for the academic year
as the viewing language of any question, anytime during the entire 2018-2019, supernumerary seats for female candidates would be there
period of the examination. The candidate will also be having the in IITs. Does this mean that the non-females will get reduced number
option of changing default question viewing language anytime during of seats in IITs in 2018?
the examination.
ANSWER: A decision has been taken at the level of the IIT Council to,
QUESTION: Will I be given rough sheets for my calculations during inter alia, improve the gender balance in the undergraduate programs
the CBT of JEE (Advanced)-2018? at the IITs from the current (approximately) 8% to 14% in 2018-19
ANSWER: Yes, you will be given “Scribble Pad” (containing by creating supernumerary seats specifically for female candidates,
blank sheets, for rough work) at the start of every paper of JEE without any reduction in the number of seats that was made available
(Advanced)-2018. You can do all your calculations inside this to non-female candidates in the previous academic year (i.e. academic
“Scribble Pad”. Candidates MUST submit their signed Scribble Pads year 2017-2018).
1
Physical World, Units and Measurements P-1

Physical World, Units


and Measurements
6. Amount of solar energy received on the earth's surface
per unit area per unit time is defined a solar constant.
TOPIC 1 Unit of Physical Quantities
Dimension of solar constant is : [Sep. 03, 2020 (II)]
(a) ML2T–2 (b) ML0T–3
1. The density of a material in SI unit is 128 kg m–3. In
certain units in which the unit of length is 25 cm and (c) M2L0T–1 (d) MLT–2
the unit of mass is 50 g, the numerical value of density 7. If speed V, area A and force F are chosen as fundamental
of the material is: [10 Jan. 2019 I] units, then the dimension of Young's modulus will be :
(a) 40 (b) 16 (c) 640 (d) 410 [Sep. 02, 2020 (I)]
2. A metal sample carrying a current along X-axis with density Jx is (a) FA V2 –1 (b) FA V2 –3

subjected to a magnetic field Bz (along z-axis). The electric field (c) FA2V–2 (d) FA–1V0
Ey developed along Y-axis is directly proportional to Jx as well 8. If momentum (P), area (A) and time (T) are taken to be the
as Bz. The constant of proportionality has SI unit fundamental quantities then the dimensional formula for
[Online April 25, 2013] energy is : [Sep. 02, 2020 (II)]
m 2
m 3
m2 As (a) [P2AT –2] (b) [PA–1T–2]
(a) (b) (c) (d)
A As As m3 (c) [PA1/ 2T-1 ] (d) [P1/ 2AT-1]
Dimensions of Physical 9. Which of the following combinations has the dimension
TOPIC 2 of electrical resistance (Î0 is the permittivity of vacuum
Quantities
and m0 is the permeability of vacuum)?
1 E 1 [12 April 2019 I]
3. The quantities x = , y= and z = are
m0 e0 B CR m0 m0 e0 e0
defined where C-capacitance, R-Resistance, l-length, (a) e0 (b) e0 (c) m0 (d) m
0
E-Electric field, B-magnetic field and e 0 , m 0 , - free 10. In the formula X = 5YZ2, X and Z have dimensions of
space permittivity and permeability respectively. Then : capacitance and magnetic field, respectively. What are
[Sep. 05, 2020 (II)]
(a) x, y and z have the same dimension. the dimensions of Y in SI units ? [10 April 2019 II]
(b) Only x and z have the same dimension. (a) [M L T A ]
–3 –2 8 4
(b) [M L T A ]
–1 –2 4 2

(c) Only x and y have the same dimension. (c) [M–2 L0 T–4 A–2] (d) [M–2 L–2 T6 A3]
(d) Only y and z have the same dimension.
4. Dimensional formula for thermal conductivity is (here K Î0
11. In SI units, the dimensions of is: [8 April 2019 I]
denotes the temperature : [Sep. 04, 2020 (I)] m0
(a) MLT K –2 (b) MLT K–2
–2
–3
(a) A–1TML3 (b) AT2 M–1L–1
(c) MLT K (d) MLT–3 K–1
(c) AT–3ML3/2 (d) A2T3 M–1L–2
5. A quantity x is given by (IFv2/WL4) in terms of moment of
inertia I, force F, velocity v, work W and Length L. The 12. Let l, r, c and v represent inductance, resistance,
dimensional formula for x is same as that of : capacitance and voltage, respectively. The dimension of
[Sep. 04, 2020 (II)] l
in SI units will be : [12 Jan. 2019 II]
(a) planck’s constant (b) force constant rcv
(a) [LA ]– 2 –1
(b) [A ]
(c) energy density (d) coefficient of viscosity
(c) [LTA] (d) [LT2]
P-2 Physics

13. The force of interaction between two atoms is given by (a) [ M ]=[ T–1 C–2 h ] (b) [ M ]=[ T–1 C2 h ]
æ x2 ö (c) [ M ]=[ T C h ] (d) [ M ]=[ T C–2 h ]
–1 –2 –1

F = ab exp ç - 21. A, B, C and D are four different physical quantities having


ç akT ÷÷ ; where x is the distance, k is the
è ø different dimensions. None of them is dimensionless. But
Boltzmann constant and T is temperature and a and b are we know that the equation AD = C ln (BD) holds true.
two constants. The dimensions of b is: [11 Jan. 2019 I] Then which of the combination is not a meaningful quantity ?
(a) M0L2T–4 (b) M2LT–4 [Online April 10, 2016]
(c) MLT–2 (d) M2 L2 T–2 C AD 2
(a) - (b) A2 –B2C2
14. If speed (V), acceleration (A) and force (F) are considered BD C
as fundamental units, the dimension of Young’s modulus A (A - C)
(c) -C (d)
will be : [11 Jan. 2019 II] B D
22. In the following 'I' refers to current and other symbols
(a) V–2A2F–2 (b) V–2A2F2 have their usual meaning, Choose the option that
(c) V–4 A–2 F (d) V–4A2F corresponds to the dimensions of electrical conductivity :
[Online April 9, 2016]
hc5 (a) M–1 L–3T 3 I (b) M–1 L–3 T3 I2
15. A quantity f is given by f = where c is speed of (c) M–1L3T3 I (d) ML–3 T–3 I2
G
light, G universal gravitational constant and h is the Planck’s 23. If electronic charge e, electron mass m, speed of light in
constant. Dimension of f is that of : [9 Jan. 2019 I] vacuum c and Planck’s constant h are taken as fundamental
(a) area (b) energy quantities, the permeability of vacuum m0 can be expressed
(c) momentum (d) volume in units of : [Online April 11, 2015]
16. Expression for time in terms of G (universal gravitational æ hc ö
constant), h (Planck's constant) and c (speed of light) is æ h ö
(a) ç 2 ÷ (b) ç 2 ÷
proportional to: [9 Jan. 2019 II] è me ø è me ø

hc5 c3 æ h ö æ mc 2 ö
(a) (b) (c) ç 2 ÷ (d) çç 2 ÷÷
G Gh è ce ø è he ø
Gh Gh 24. If the capacitance of a nanocapacitor is measured in terms
(c) 5
(d)
c c3 of a unit ‘u’ made by combining the electric charge ‘e’,
17. The dimensions of stopping potential V0 in photoelectric Bohr radius ‘a0’, Planck’s constant ‘h’ and speed of light
effect in units of Planck’s constant ‘h’, speed of light ‘c’ ‘c’ then: [Online April 10, 2015]
and Gravitational constant ‘G’ and ampere A is:
e2 h hc
[8 Jan. 2019 I] (a) u = (b) u = 2
(a) hl/3G2/3cl/3 A –1 (b) h2/3c5/3G1/3A –1 a0 e a0
(c) h–2/3 e–1/3 G4/3 A–1 (d) h2G3/2C1/3 A–1
B 2 e2c e2 a 0
18. The dimensions of , where B is magnetic field and m0 (c) u = (d) u =
2m 0 ha 0 hc
is the magnetic permeability of vacuum, is: 25. From the following combinations of physical constants
[8 Jan. 2019 II] (expressed through their usual symbols) the only
(a) MLT–2 (b) ML2T–1 combination, that would have the same value in different
(c) ML2T–2 (d) ML–1T–2 systems of units, is: [Online April 12, 2014]
19. The characteristic distance at which quantum gravitational
ch
effects are significant, the Planck length, can be determined (a)
from a suitable combination of the fundamental physical 2peo2
constants G, h and c. Which of the following correctly
gives the Planck length? [Online April 15, 2018] e2
(b) (me = mass of electron)
1 2pe o Gme2
æ Gh ö2 1
(a) G2hc (b) ç 3 ÷ (c) 2 3
2 h 2 c (d) Gh c
èc ø G m o eo G
(c)
20. Time (T), velocity (C) and angular momentum (h) are c2 he 2
chosen as fundamental quantities instead of mass, length
and time. In terms of these, the dimensions of mass would 2p m o eo h
(d)
be : [Online April 8, 2017] ce2 G
Physical World, Units and Measurements P-3

26. In terms of resistance R and time T, the dimensions of ratio -1


(b) éë MLT ùû
-1 -1
m (a) éë ML T ùû
of the permeability m and permittivity e is:
e -1 -2 -2 -2
[Online April 11, 2014] (c) éë ML T ùû (d) éë ML T ùû

(a) [RT–2] (b) [R2T–1] (c) [R2] (d) [R2T2] 35. Dimensions of 1 , where symbols have their usual
mo eo
27. Let [ Î0 ] denote the dimensional formula of the permittivity
meaning, are [2003]
of vacuum. If M = mass, L = length, T = time and A =
electric current, then: [2013] (a) [L-1T] (b) [L-2 T 2 ]
(a) Î0 = [M L T A] (b) Î0 = [M L T A2]
–1 –3 2 –1 –3 4 (c) [L2 T -2 ] (d) [LT -1 ]
(c) Î0 = [M1 L2 T1 A2] (d) Î0 = [M1 L2 T1 A] 36. The physical quantities not having same dimensions are
(a) torque and work [2003]
28. If the time period t of the oscillation of a drop of liquid of
(b) momentum and planck’s constant
density d, radius r, vibrating under surface tension s is given
(c) stress and young’s modulus
by the formula t = r 2b s c d a / 2 . It is observed that the (d) speed and (m o e o ) -1 / 2
37. Identify the pair whose dimensions are equal [2002]
d
time period is directly proportional to . The value of b (a) torque and work (b) stress and energy
s (c) force and stress (d) force and work
should therefore be : [Online April 23, 2013]
3 TOPIC 3 Errors in Measurements
(a) (b) 3
4
3 2 38. A screw gauge has 50 divisions on its circular scale. The
(c) (d) circular scale is 4 units ahead of the pitch scale marking,
2 3
29. The dimensions of angular momentum, latent heat and prior to use. Upon one complete rotation of the circular
capacitance are, respectively. [Online April 22, 2013] scale, a displacement of 0.5 mm is noticed on the pitch
scale. The nature of zero error involved, and the least
(a) ML2 T1A 2 , L2 T -2 , M -1L-2 T 2 count of the screw gauge, are respectively :
(b) ML2 T -2 , L2 T 2 , M -1L-2 T 4 A 2 [Sep. 06, 2020 (I)]
(a) Negative, 2 mm (b) Positive, 10 mm
(c) ML2 T -1, L2 T -2 , ML2 TA 2
(c) Positive, 0.1 mm (d) Positive, 0.1 mm
39. The density of a solid metal sphere is determined by
(d) ML2 T -1 , L2 T -2 , M -1L-2 T 4 A 2 measuring its mass and its diameter. The maximum error in
30. Given that K = energy, V = velocity, T = time. If they are æ x ö
chosen as the fundamental units, then what is dimensional the density of the sphere is çè ÷ %. If the relative errors
100 ø
formula for surface tension? [Online May 7, 2012]
in measuring the mass and the diameter are 6.0% and 1.5%
(a) [KV–2T –2 ] (b) [K2 V2T–2 ] respectively, the value of x is ______.
(c) [K2V–2 T–2 ] (d) [KV2 T2 ] [NA Sep. 06, 2020 (I)]
31. The dimensions of magnetic field in M, L, T and C 40. A student measuring the diameter of a pencil of circular
(coulomb) is given as [2008]
cross-section with the help of a vernier scale records the
(a) [MLT–1 C–1] (b) [MT2 C–2]
following four readings 5.50 mm, 5.55 mm, 5.45 mm,
(c) [MT–1 C–1] (d) [MT–2 C–1]
5.65 mm, The average of these four reading is 5.5375 mm
32. Which of the following units denotes the dimension and the standard deviation of the data is 0.07395 mm. The
average diameter of the pencil should therefore be re-
ML2
, where Q denotes the electric charge? [2006] corded as : [Sep. 06, 2020 (II)]
Q2 (a) (5.5375 ± 0.0739) mm (b) (5.5375 ± 0.0740) mm
(a) Wb/m 2 (b) Henry (H)
(c) (5.538 ± 0.074) mm (d) (5.54 ± 0.07) mm
(c) H/m2 (d) Weber (Wb)
33. Out of the following pair, which one does NOT have 41. A physical quantity z depends on four observables a, b, c
identical dimensions ? [2005] 2
2 3
(a) Impulse and momentum a b
and d, as z = . The percentages of error in the mea-
(b) Angular momentum and planck’s constant cd 3
(c) Work and torque surement of a, b, c and d are 2%, 1.5%, 4% and 2.5% respec-
(d) Moment of inertia and moment of a force
tively. The percentage of error in z is :
34. Which one of the following represents the correct
dimensions of the coefficient of viscosity? [2004] [Sep. 05, 2020 (I)]
P-4 Physics

(a) 12.25% (b) 16.5% (a) 37cm2 (b) 37.030 cm2


(c) 13.5% (d) 14.5% (c) 37.03 cm2 (d) 37.0 cm2
42. Using screw gauge of pitch 0.1 cm and 50 divisions on its 49. In a simple pendulum experiment for determination of
circular scale, the thickness of an object is measured. It acceleration due to gravity (g), time taken for 20
should correctly be recorded as : [Sep. 03, 2020 (I)] oscillations is measured by using a watch of 1 second
(a) 2.121 cm (b) 2.124 cm least count. The mean value of time taken comes out to be
(c) 2.125 cm (d) 2.123 cm 30 s. The length of pendulum is measured by using a
43. The least count of the main scale of a vernier callipers is meter scale of least count 1 mm and the value obtained is
1 mm. Its vernier scale is divided into 10 divisions and
55.0 cm. The percentage error in the determination of g is
coincide with 9 divisions of the main scale. When jaws are
touching each other, the 7th division of vernier scale close to : [8 April 2019 II]
coincides with a division of main scale and the zero of (a) 0.7% (b) 0.2% (c) 3.5% (d) 6.8%
vernier scale is lying right side of the zero of main scale. 50. The least count of the main scale of a screw gauge is 1 mm.
When this vernier is used to measure length of a cylinder The minimum number of divisions on its circular scale
the zero of the vernier scale betwen 3.1 cm and 3.2 cm and required to measure 5 µm diameter of a wire is:
[12 Jan. 2019 I]
4th VSD coincides with a main scale division. The length
of the cylinder is : (VSD is vernier scale division) (a) 50 (b) 200 (c) 100 (d) 500
[Sep. 02, 2020 (I)] 51. The diameter and height of a cylinder are measured by
(a) 3.2 cm (b) 3.21 cm a meter scale to be 12.6 ± 0.1 cm and 34.2 ± 0.1 cm,
(c) 3.07 cm (d) 2.99 cm respectively. What will be the value of its volume in
44. If the screw on a screw-gauge is given six rotations, it moves appropriate significant figures? [10 Jan. 2019 II]
by 3 mm on the main scale. If there are 50 divisions on the 3
circular scale the least count of the screw gauge is: (a) 4264 ± 81 cm (b) 4264.4 ± 81.0 cm3
3
[9 Jan. 2020 I] (c) 4260 ± 80 cm (d) 4300 ± 80 cm3
(a) 0.001 cm (b) 0.02 mm 52. The pitch and the number of divisions, on the circular
(c) 0.01 cm (d) 0.001 mm scale for a given screw gauge are 0.5 mm and 100
45. For the four sets of three measured physical quantities as respectively. When the screw gauge is fully tightened
given below. Which of the following options is correct? without any object, the zero of its circular scale lies 3
[9 Jan. 2020 II] division below the mean line.
(A) A1 = 24.36, B1 = 0.0724, C1 = 256.2 The readings of the main scale and the circular scale, for
(B) A2 = 24.44, B2 = 16.082, C2 = 240.2 a thin sheet, are 5.5 mm and 48 respectively, the
(C) A3 = 25.2, B3 = 19.2812, C3 = 236.183 thickness of the sheet is: [9 Jan. 2019 II]
(D) A4 = 25, B4 = 236.191, C4 = 19.5 (a) 5.755 mm (b) 5.950 mm
(a) A4 + B4 + C4 < A1 + B1 + C1 < A3 + B3 + C3 < A2 + B2 + C2
(c) 5.725 mm (d) 5.740 mm
(b) A1 + B1 + C1 = A2 + B2 + C2 = A3 + B3 + C3 = A4 + B4 + C4 53. The density of a material in the shape of a cube is
(c) A4 + B4 + C4 < A1 + B1 + C1 = A2 + B2 + C2 = A3 + B3 + C3 determined by measuring three sides of the cube and its
(d) A1 + B1 + C1 < A3 + B3 + C3 < A2 + B2 + C2 < A4 + B4 + C4 mass. If the relative errors in measuring the mass and
46. A simple pendulum is being used to determine the value length are respectively 1.5% and 1%, the maximum error
of gravitational acceleration g at a certain place. The length in determining the density is: [2018]
of the pendulum is 25.0 cm and a stop watch with 1 s (a) 2.5% (b) 3.5% (c) 4.5% (d) 6%
resolution measures the time taken for 40 oscillations to 54. The percentage errors in quantities P, Q, R and S are 0.5%,
be 50 s. The accuracy in g is: [8 Jan. 2020 II] 1%, 3% and 1.5% respectively in the measurement of a
(a) 5.40% (b) 3.40% P 3Q 2
(c) 4.40% (d) 2.40% physical quantity A = .
RS
47. In the density measurement of a cube, the mass and edge
The maximum percentage error in the value of A will be
length are measured as (10.00 ± 0.10) kg and (0.10 ± 0.01)
[Online April 16, 2018]
m, respectively. The error in the measurement of density (a) 8.5% (b) 6.0%
is: [9 April 2019 I] (c) 7.5% (d) 6.5%
(a) 0.01 kg/m 3
(b) 0.10 kg/m3 55. The relative uncertainty in the period of a satellite orbiting
around the earth is 10–2. If the relative uncertainty in the
(c) 0.013 kg/m 3
(d) 0.07 kg/m3
radius of the orbit is negligible, the relative uncertainty in
48. The area of a square is 5.29 cm 2. The area of 7 such the mass of the earth is [Online April 16, 2018]
squares taking into account the significant figures is: (a) 3×10–2 (b) 10–2
[9 April 2019 II] (c) 2 × 10–2 (d) 6 × 10–2
Physical World, Units and Measurements P-5

56. The relative error in the determination of the surface area 63. Diameter of a steel ball is measured using a Vernier callipers
of a sphere is a. Then the relative error in the determination which has divisions of 0.1 cm on its main scale (MS) and
of its volume is [Online April 15, 2018] 10 divisions of its vernier scale (VS) match 9 divisions on
2 2 3 the main scale. Three such measurements for a ball are
(a) a (b) a (c) a (d) a given as: [Online April 10, 2015]
3 3 2
57. In a screw gauge, 5 complete rotations of the screw cause S.No. MS(cm) VS divisions
it to move a linear distance of 0.25 cm. There are 100 circular 1. 0.5 8
scale divisions. The thickness of a wire measured by this
2. 0.5 4
screw gauge gives a reading of 4 main scale divisions and
30 circular scale divisions. Assuming negligible zero error, 3. 0.5 6
the thickness of the wire is: [Online April 15, 2018] If the zero error is – 0.03 cm, then mean corrected diameter
(a) 0.0430 cm (b) 0.3150 cm is:
(c) 0.4300 cm (d) 0.2150 cm (a) 0.52 cm (b) 0.59 cm
58. The following observations were taken for determining (c) 0.56 cm (d) 0.53 cm
surface tensiton T of water by capillary method : 64. The current voltage relation of a diode is given by
Diameter of capilary, D = 1.25 × 10–2 m I = ( e1000V T - 1) mA, where the applied voltage V is in
rise of water, h = 1.45 × 10–2 m
Using g = 9.80 m/s2 and the simplified relation volts and the temperature T is in degree kelvin. If a student
makes an error measuring ±0.01 V while measuring the
rhg
T= ´ 10 3 N/m, the possible error in surface tension current of 5 mA at 300 K, what will be the error in the
2
is closest to : [2017] value of current in mA? [2014]
(a) 2. 4 % (b) 10 % (c) 0.15% (d) 1.5% (a) 0.2 mA (b) 0.02 mA (c) 0.5 mA (d) 0.05 mA
59. A physical quantity P is described by the relation 65. A student measured the length of a rod and wrote it as 3.50
P = a1/2 b2 c3 d –4 cm. Which instrument did he use to measure it?
If the relative errors in the measurement of a, b, c and d [2014]
respectively, are 2%, 1%, 3% and 5%, then the relative (a) A meter scale.
error in P will be : [Online April 9, 2017] (b) A vernier calliper where the 10 divisions in vernier
(a) 8% (b) 12% (c) 32% (d) 25% scale matches with 9 division in main scale and main
60. A screw gauge with a pitch of 0.5 mm and a circular scale scale has 10 divisions in 1 cm.
with 50 divisions is used to measure the thickness of a (c) A screw gauge having 100 divisions in the circular
thin sheet of Aluminium. Before starting the measurement, scale and pitch as 1 mm.
it is found that wen the two jaws of the screw gauge are (d) A screw gauge having 50 divisions in the circular scale
brought in contact, the 45th division coincides with the and pitch as 1 mm.
main scale line and the zero of the main scale is barely 66. Match List - I (Event) with List-II (Order of the time interval
visible. What is the thickness of the sheet if the main scale for happening of the event) and select the correct option
reading is 0.5 mm and the 25th division coincides with the from the options given below the lists:
main scale line? [2016] [Online April 19, 2014]
(a) 0.70 mm (b) 0.50 mm List - I List - II
(c) 0.75 mm (d) 0.80 mm
61. A student measures the time period of 100 oscillations of (1) Rotation (i) 10 5 s
a simple pendulum four times. The data set is 90 s, 91 s, 95 period of earth
s, and 92 s. If the minimum division in the measuring clock
is 1 s, then the reported mean time should be: [2016] (2) Revolution (ii) 10 7 s
(a) 92 ± 1.8 s (b) 92 ± 3s period of earth
(c) 92 ± 1.5 s (d) 92 ± 5.0 s
(3) Period of light (iii) 10 –15 s
62. The period of oscillation of a simple pendulum is
wave
L (4) Period of (iv) 10 –3 s
T = 2p . Measured value of L is 20.0 cm known to 1 mm
g sound wave
accuracy and time for 100 oscillations of the pendulum is (a) (1)-(i), (2)-(ii), (3)-(iii), (4)-(iv)
found to be 90 s using a wrist watch of 1s resolution. The
(b) (1)-(ii), (2)-(i), (3)-(iv), (4)-(iii)
accuracy in the determination of g is :
[2015] (c) (1)-(i), (2)-(ii), (3)-(iv), (4)-(iii)
(a) 1% (b) 5% (c) 2% (d) 3% (d) (1)-(ii), (2)-(i), (3)-(iii), (4)-(iv)
P-6 Physics

67. In the experiment of calibration of voltmeter, a standard cell 72. A student measured the diameter of a wire using a screw
of e.m.f. 1.1 volt is balanced against 440 cm of potential wire. gauge with the least count 0.001 cm and listed the
The potential difference across the ends of resistance is measurements. The measured value should be recorded
found to balance against 220 cm of the wire. The as [Online May 12, 2012]
corresponding reading of voltmeter is 0.5 volt. The error in (a) 5.3200 cm (b) 5.3 cm
the reading of volmeter will be: [Online April 12, 2014] (c) 5.32 cm (d) 5.320 cm
73. A screw gauge gives the following reading when used to
(a) – 0. 15 volt (b) 0.15 volt measure the diameter of a wire.
(c) 0.5 volt (d) – 0.05 volt Main scale reading : 0 mm
68. An experiment is performed to obtain the value of Circular scale reading : 52 divisions
acceleration due to gravity g by using a simple pendulum of Given that 1mm on main scale corresponds to 100 divisions
length L. In this experiment time for 100 oscillations is of the circular scale. The diameter of wire from the above
measured by using a watch of 1 second least count and the data is [2011]
value is 90.0 seconds. The length L is measured by using a (a) 0.052 cm (b) 0.026 cm
meter scale of least count 1 mm and the value is 20.0 cm. The
(c) 0.005 cm (d) 0.52 cm
error in the determination of g would be:
74. The respective number of significant figures for the
[Online April 9, 2014]
numbers 23.023, 0.0003 and 2.1 × 10–3 are [2010]
(a) 1.7% (b) 2.7% (c) 4.4% (d) 2.27%
(a) 5, 1, 2 (b) 5, 1, 5
69. Resistance of a given wire is obtained by measuring the
(c) 5, 5, 2 (d) 4, 4, 2
current flowing in it and the voltage difference applied across
it. If the percentage errors in the measurement of the current 75. In an experiment the angles are required to be measured
and the voltage difference are 3% each, then error in the using an instrument, 29 divisions of the main scale exactly
value of resistance of the wire is [2012] coincide with the 30 divisions of the vernier scale. If the
smallest division of the main scale is half- a degree
(a) 6% (b) zero (c) 1% (d) 3%
(= 0.5°), then the least count of the instrument is: [2009]
70. A spectrometer gives the following reading when used to
measure the angle of a prism. (a) half minute (b) one degree
Main scale reading : 58.5 degree (c) half degree (d) one minute
Vernier scale reading : 09 divisions 76. A body of mass m = 3.513 kg is moving along the x-axis
Given that 1 division on main scale corresponds to 0.5 with a speed of 5.00 ms–1. The magnitude of its momentum
degree. Total divisions on the Vernier scale is 30 and match is recorded as [2008]
with 29 divisions of the main scale. The angle of the prism (a) 17.6 kg ms–1 (b) 17.565 kg ms–1
from the above data is [2012] (c) 17.56 kg ms–1 (d) 17.57 kg ms–1
(a) 58.59 degree (b) 58.77 degree
(c) 58.65 degree (d) 59 degree 77. Two full turns of the circular scale of a screw gauge cover a
71. N divisions on the main scale of a vernier calliper coincide distance of 1mm on its main scale. The total number of
with (N + 1) divisions of the vernier scale. If each division of divisions on the circular scale is 50. Further, it is found that
main scale is ‘a’ units, then the least count of the instrument the screw gauge has a zero error of – 0.03 mm. While
is [Online May 19, 2012] measuring the diameter of a thin wire, a student notes the
main scale reading of 3 mm and the number of circular scale
a divisions in line with the main scale as 35. The diameter of
(a) a (b)
N the wire is [2008]
N a (a) 3.32 mm (b) 3.73 mm
(c) ´a (d)
N +1 N +1 (c) 3.67 mm (d) 3.38 mm
Physical World, Units and Measurements P-7

1. (a) Density of material in SI unit, (M1L2 )(M1L1T -2 )(L1T -2 )2


=
128kg (M1L2 T -2 )(L4 )
=
m3 M1L-2 T -2
= = M1L-1T -2 = Energy density
Density of material in new system L3
128 ( 50 g )( 20) 128 Energy
= = ( 20) = 40 units 6. (b) Solar constant =
Time Area
( 25cm ) ( 4)
3 3 64
Dimension of Energy, E = ML2T–2
2. (b) According to question
Dimension of Time = T
E y µ J x BZ Dimension of Area = L2
\ Constant of proportionality \ Dimension of Solar constant
Ey C m3 M1L2 T -2
K= = = = = M1L0 T -3 .
BZ J x J x As TL2
E I stress
[As = C (speed of light) and J = ] 7. (d) Young's modulus, Y =
B Area strain
3. (a) We know that F Dl
ÞY = = FA –1V0
A l0
1
Speed of light, c = =x
m 0 e0 8. (c) Energy, E µ AaT b Pc

E or, E = kAaT b P c ...(i)


Also, c = =y
B where k is a dimensionless constant and a, b and c are the
Time constant, t = Rc = t exponents.

l l Dimension of momentum, P = M 1L1T -1


\z = = = Speed
Rc t Dimension of area, A = L2
Thus, x, y, z will have the same dimension of speed. Dimension of time, T = T 1
dQ dT Putting these value in equation (i), we get
4. (d) From formula, = kA
dt dx
M 1L2T -2 = M c L2a + cT b - c
æ dQ ö by comparison
çè ÷
dt ø c=1
Þk=
æ dT ö 2a + c = 2
Aç ÷
è dx ø b – c = –2
c = 1, a = 1/2, b = –1
[ML2 T -3 ]
[k ] = = [MLT -3K -1 ] \ E = A1/ 2T -1 P1
[L2 ][KL-1 ]
5. (c) Dimension of Force F = M1L1T–2 m0 m 20 æ 1 ö
9. (a) = = m0 c ç = c÷
Dimension of velocity V = L1T–1 e0 e0 m 0 è m0e0 ø
Dimension of work = M1L2T–2 m0c ® MLT–2A–2 × LT–1
Dimension of length = L ML2T–3A–2
Moment of inertia = ML2 Dimensions of resistance
10. (a) X = 5YZ2
IFv 2
\x = X
WL4 ÞY µ 2 ...(i)
Z
P-8 Physics

Q Q2 [ A2T 2 ] b = 1, a + b + c = – 1, –a –2b –2c = – 2


X = Capacitance = = =
V W [ ML2T -2 ] solving above equations we get:
X = [M–1L–2T4A2] a = – 4, b = 1, c = 2
F so [Y] = [V–4FA2] = [V–4A2F]
Z =B= [Q F = ILB]
IL 15. (b) Dimension of [h] = [ML2T–1]
Z = [MT–2A–1] [C] = [LT–1]
[ M -1 L-2T 4 A2 ] [G] = [M–1L3T–2]
Y=
[ MT -2 A-1 ]2 Hence dimension of
Y = [M–3L–2T8A4] (Using (i)) é hC 5 ù é ML2T -1 ù × é L5T -5 ù
é e0 ù e 02 é e0 ù ê ú=ë û ë û
11. (d) ê m ú = m e = ê ú = e C[LT T–1]×[e0] ê G ú é M -1 L3T -2 ù
ë û ë û
ëê 0 úû 0 0 êë m 0 e 0 úû 0

= [ML2T–2] = energy
é 1 ù
êQ = Cú 16. (c) Let t µ Gx hy Cz
êë m0 e0 úû Dimensions of G = [M–1L3T–2],
q 2 h = [ML2T–1] and C = [LT–1]
Q F=
4pe 0 r 2 [T] = [M–1L3T–2]x[ML2T–1]y[LT–1]z
[M0L0T1] = [M–x+y L3x+2y+z T–2x–y–z]
[ AT ]2
Þ [e 0 ] = -2 2
= [ A2 M -1 L-3T 4 ] By comparing the powers of M, L, T both the sides
[ MLT ] ´ [ L ]
–x+ y=0 Þx=y
é e0 ù
\ ê -1 2 -1 -3 4
ú = [ LT ] ´ [ A M L T ] 3x + 2y + z = 0 Þ 5x + z = 0 ..... (i)
m
ëê 0 úû –2x – y –z = 1 Þ 3x + z = –1 ..... (ii)
= [ M -1L-2T 3 A2 ] Solving eqns. (i) and (ii),
12. (b) As we know, Gh
1 5
élù x=y= ,z=- \ tµ
êë r úû = [ T ] and [ cv] = [ AT ] C5
2 2
17. (None)
é l ù é T ù
\ê ú=ê = é A –1 ù
ë rcv û ë AT úû ë û Stopping potential (V0 ) µ h x I yG Z C r
13. (b) Force of interaction between two atoms,
Here,h = Planck’s constant = éë ML2T -1 ùû
æ -x2 ö
ç ÷ I = current = [A]
ç akT ÷
F= abeè ø
G = Gravitational constant = [M–1L3T–2]
Since exponential terms are dimensionless and c = speed of light = [LT–1]
é x2 ù V0 = potential= [ML2T–3A–1]
\ ê akT ú = M0L0T0 \ [ML2T–3A–1]=[ML2T–1]x [A]y[M–1L3T–2]z[LT–1]r
êë úû
Mx – z; L2x+3z+r; T–x–2z–r; Ay
L2 Comparing dimension of M, L, T, A, we get
Þ = M0 L0T 0 y = –1, x = 0, z = – 1 , r = 5
[a ]ML2T-2
Þ [a] = M–1T2 \ V0 µ h0 I –1G –1C 5
[F] = [a] [b] B2
MLT–2 = M–1T2[b] 18. (d) The quantity is the energy density of magnetic
2m0
Þ [b] = M2LT–4 field.
14. (d) Let [Y] = [V]a [F]b [A]c é B 2 ù Energy Force ´ displacement
Þ ê ú= =
[ML–1T–2] = [LT–1]a [MLT–2]b [LT –2]c êë 2m0 úû Volume (displacement)3
[ML–1T–2] = [MbLa+b+c T–a–2b–2c] é ML2T –2 ù –1 –2
Comparing power both side of similar terms we get, =ê 3
ú = ML T
êë L úû
Physical World, Units and Measurements P-9

19. (b) Plank length is a unit of length, lp = 1.616229 × 10–35 m


e0 = é M -1L3T 4 A 2 ù
ë û
hG
lp =
c3 G = é M -1L3T -2 ù and me = é M1L0T 0 ù
ë û ë û
20. (a) Let mass, related as M µ Tx Cy hz
e2
M1L0 T0 = (T')x (L1T–1)y (M1L2T–1)z Now,
M1L0 T0 = Mz Ly + 2z + Tx– y–z 2 pe0 Gm e2
z= 1 2
é M0 L0 T1A1 ù
y + 2z = 0 x – y– z = 0 ë û
= 2
y = –2 x+ 2–1=0 2 p é M -1L-3T 4 A 2 ù é M -1L3T -2 ù é M1L0 T 0 ù
x = –1 ë ûë ûë û
M = [T C h ]
–1 –2 1
éT 2A 2 ù
ë û
21. (d) Dimension of A ¹ dimension of (C) =
Hence A – C is not possible. 2p éM -1-1+ 2 L-3+ 3 T 4 - 2 A 2 ù
ë û
22. (b) We know that resistivity
RA éT 2 A 2 ù
ë û 1
θ< = =
l é 0 0 2 2ù
2p M L T A 2p
1 l ë û
Conductivity = resistivity < RA
1 e2
lI Q is dimensionless thus the combination
< ( Q V = RI) 2p 2 pe 0 Gm e2
VA
[L][I] W W would have the same value in different systems of units.
< QV< <
é [ML2 T,2 ù q it 26. (c) Dimensions of m = [MLT–2A–2]
ê ú ´[L2 ] Dimensions of Î = [M–1L–3T4A2]
ê [I][T] ú
êë úû Dimensions of R = [ML2T–3A–2]
< [M,1L,3T 3 ][I2 ] < [M,1L,3T3I2 ]
Dimensions of m [MLT -2 A -2 ]
23. (c) Let µ0 related with e, m, c and h as follows. \ =
Dimensions of Î [M -1L-3T 4 A 2 ]
m0 = keambcchd = [M2L4T–6A–4 ] = [R2]
[MLT–2A–2] = [AT]a [M]b [LT–1]c [ML2T–1]d 1 q1q 2
27. (b) As we know, F =
= [Mb + d Lc + 2d Ta – c – d Aa] 4 pe0 R 2
On comparing both sides we get q1q2
Þ e0 =
a = – 2 ...(i) 4pFR 2
b+ d = 1 ...(ii)
C2 [AT]2
c + 2d = 1 ...(iii) Hence, e0 = =
N.m2 [MLT -2 ][L2 ]
a – c – d = –2 ...(iv)
= [M–1 L–3 T4 A2]
By equation (i), (ii), (iii) & (iv) we get,
28. (c)
a = – 2, b = 0, c = – 1, d = 1 29. (d) Angular momentum = m × v × r = ML2 T–1
é h ù Q ML2 T -2
\ [m 0 ] = ê 2 ú Latent heat L = = = L2T–2
ë ce û m M
24. (d) Let unit ‘u’ related with e, a0, h and c as follows. Charge
Capacitance C = = M -1L-2 T 4 A 2
[u] = [e]a [a0]b [h]c [C]d P.d.
Using dimensional method, F F l T2
[M–1L–2T+4A+2] = [A1T1]a[L]b[ML2T–1]c[LT–1]d 30. (a) Surface tension, T = = . .
l l l T2
[M–1L–2T+4A+2] = [Mc Lb+2c+d Ta–c–d Aa]
T2 -2
a = 2, b = 1, c = – 1, d = – 1 (As, F.l = K (energy); =V )
l2
e 2 a0 Therefore, surface tension = [KV–2T–2]
\ u=
hc 31. (c) Magnitude of Lorentz formula F = qvB sin q
25. (b) The dimensional formulae of
F MLT -2
e = éM L T A
0 0 1 1ù B= = = [ MT -1C -1]
ë û qv C ´ LT -1
P-10 Physics

f BA æ Dr ö Dm
32. (b) Mutual inductance = = æ DD ö
\%ç ÷ = + 3ç = 6 + 3 ´ 1.5 = 10.5%
I I
è rø m è D ÷ø
[ MT -1Q -1L2 ]
[Henry] = = ML2Q -2 æ Dr ö 1050
[QT -1 ] æ x ö
%ç ÷ = %=ç %
33. (d) Moment of Inertia, I = MR2 è r ø 100 è 100 ÷ø
[I] = [ML2] \ x = 1050.00
r r r 40. (d) Average diameter, dav = 5.5375 mm
Moment of force, t = r ´ F
r Deviation of data, Dd = 0.07395 mm
t = [ L][ MLT -2 ] = [ML2T -2 ]
34. (a) According to, Stokes law, As the measured data are upto two digits after decimal,
therefore answer should be in two digits after decimal.
F
F = 6phrv Þ h = \ d = (5.54 ± 0.07) mm
6pr v
[ MLT –2 ] a 2b 2/ 3
h= –1
Þ h = [ ML-1T -1 ] 41. (d) Given : Z =
[ L][ LT ] cd 3
35. (c) As we know, the velocity of light in free space is Percentage error in Z,
given by
DZ 2 Da 2 Db 1 Dc 3Dd
1 1 = = + + +
c= \ = e 2 = Z12T 2 Z a 3 b 2 c d
mo eo m 0 e0
2 1
1 = 2´ 2+ ´ 1.5 + ´ 4 + 3 ´ 2.5 = 14.5%.
2 2 3 2
mo eo = C [m/s]
42. (a) Thickness = M.S. Reading + Circular Scale Reading
= [LT–1]2 (L.C.)
= [M0L2T–2] Pitch 0.1
36. (b) Momentum, = mv = [MLT–1] Here LC = = = 0.002 cm per
Circular scale division 50
Planck’s constant, division
2 –2 So, correct measurement is measurement of integral
E = [ ML T ] multiple of L.C.
h= = [ML2T–1]
v [T –1 ] 43. (c) L.C. of vernier callipers = 1 MSD – 1 VSD
r r
37. (a) Work W = F × s = Fs cos q æ 9ö
r r = ç1 - ÷ ´ 1 = 0.1 mm = 0.01 cm
Q A × B = AB cos q è 10 ø
= [ MLT -2 ][ L] = [ML2T -2 ] ; Here 7th division of vernier scale coincides with a division
r r r of main scale and the zero of vernier scale is lying right
Torque, t = r ´ F Þ t = rF sin q side of the zero of main scale.
r r
Q A ´ B = AB sin q Zero error = 7 × 0.1 = 0.7 mm = 0.07 cm.
= [ L ] [MLT -2 ] = [ ML2T -2 ] Length of the cylinder = measured value – zero error
38. (b) Given : No. of division on circular scale of screw gauge = 50 = (3.1 + 4 × 0.01) – 0.07 = 3.07 cm.
Pitch = 0.5 mm 44. (d) When screw on a screw-gauge is given six rotations,
Least count of screw gauge it moves by 3mm on the main scale
Pitch 3
= \ Pitch = = 0.5mm
No. of division on circular scale 6
Pitch 0.5 mm
0.5 \ Least count L.C. = =
= mm = 1 ´ 105 m = 10 mm CSD 50
50 1
And nature of zero error is positive. = mm = 0.01 mm = 0.001cm
100
39. (1050) 45. (None)
M M 6 D1 = A1 + B1 + C1 = 24.36 + 0.0724 + 256.2 = 280.6
Density, r = = Þ r = MD -3
V 4 æ Dö
3 p D2 = A2 + B2 + C2 = 24.44 + 16.082 + 240.2 = 280.7
pç ÷ D3 = A3 + B3 + C3 = 25.2 + 19.2812 + 236.183= 280.7
3 è 2ø
Physical World, Units and Measurements P-11

D4 = A4 + B4 + C4 = 25 + 236.191 + 19.5 = 281 53. (c) = 1.5 % + 3 (1%) = 4.5%


None of the option matches. 54. (d) Maximum percentage error in A
46. (c) Given, Length of simple pendulum, l = 25.0 cm = 3(% error in P) + 2(% error in Q)
Time of 40 oscillation, T = 50s 1
Time period of pendulum + (% error in R) + 1(% error in S)
2
l 1
= 3 ´ 0.5 + 2 ´ 1 + ´ 3 + 1 ´ 1.5
T = 2p 2
g
= 1.5 + 2 + 1.5 + 1.5 = 6.5%
4p 2l 4p2 l 55. (c) From Kepler's law, time period of a satellite,
Þ T2 = Þg= 2
g T r3 4p 2 3
Dg Dl 2DT T = 2p T2 = r
= + Gm GM
Þ Fractional error in g =
g l T Relative uncertainty in the mass of the earth
Dg æ 0.1 ö æ 1ö DM DT
Þ =ç ÷ + 2 ç ÷ = 0.044 =2 = 2 ´ 10-2
g è 25.0 ø è 50 ø M T
(Q 4p & G constant and
Dg Dr
\ Percentage error in g = ´ 100 = 4.4% relative uncertainty in radius negligible)
g r
47. (Bonus) d = M = M = Ml -3 Ds Dr
56. (c) Relative error in Surface area, = 2 ´ = a and
V l3 s r
Dv Dr
Dd DM Dl 0.10 æ 0.01ö relative error in volume, = 3´
= +3 = + 3ç
è 0.10 ÷ø = 0.31 kg/m ,
3 v r
d M l 10.00
\ Relative error in volume w.r.t. relative error in area,
48. (d) A = 7 × 5.29 = 37.03 cm2
The result should have three significant figures, so Dv 3
= a
A = 37.0 cm2 v 2
49. (d) We have Value of 1 part on main scale
57. (d) Least count =
l 2 l Number of parts on vernier scale
T = 2p or g = 4p 2
g T 0.25
= cm = 5 × 10–4 cm
Dg DR DT 5×100
´ 100 = ´ 100 + 2 ´ 100 Reading = 4 × 0.05 cm + 30 × 5 × 10–4 cm
g Q T
= (0.2 + 0.0150) cm = 0.2150 cm (Thickness of wire)
0.1 æ 1ö
= ´ 100 + 2 ç ÷ ´ 100 rhg
55 è 30 ø 58. (d) Surface tension, T = ´ 103
2
= 0.18 + 6.67 = 6.8% Relative error in surface tension,
50. (b) Least count of main scale of screw gauge = 1 mm
DT Dr Dh
Least count of screw gauge = + + 0 (Q g, 2 & 103 are constant)
T r h
Pitch Percentage error
=
Number of division on circular scale
DT æ 10 –2 ´ 0.01 10 –2 ´ 0.01ö
100 ´ = ç + ÷ 100
10-3 T è 1.25 ´ 10 –2 1.45 ´ 10 –2 ø
5 ´ 10 -6 =
N = (0.8 + 0.689)
Þ N = 200 = (1.489) = 1.489% @ 1.5%
51. (c) 59. (c) Given, P = a1/2 b2 c2 d–4,
52. (c) Least count of screw gauge, Maximum relative error,
Pitch DP 1 Da Db Dc Dd
= +2 +3 +4
LC = No. of division P 2 a b c d
1
= 0.5 × 10–3 = 0.5 × 10–2 mm + ve error = 3 × 0.5 × 10–2 mm = ´ 2 + 2 ´ 1 + 3 ´ 3 + 4 ´ 5 = 32%
= 1.5 × 10–2 mm = 0.015 mm 2
Reading = MSR + CSR – (+ve error) 0.5
60. (d) L.C. = = 0.01 mm
= 5.5 mm + (48 × 0.5 × 10–2) – 0.015 50
= 5.5 + 0.24 – 0.015 = 5.725 mm Zero error = 5 × 0.01 = 0.05 mm (Negative)
Reading = (0.5 + 25 × 0.01) + 0.05 = 0.80 mm
P-12 Physics

| DT1 | + | DT2 | + | DT3 | + | DT4 | 68. (b) According to the question.


61. (c) DT =
4 Dt 1
t = (90 ± 1) or, =
2 +1+ 3 + 0 t 90
= = 1.5
4
As the resolution of measuring clock is 1.5 therefore Dl 0.1
l = (20 ± 0.1) or, =
the mean time should be 92 ± 1.5 l 20
L Dg
62. (d) As, g = 4p2 %=?
T2 g
Dg DL DT As we know,
So, ´100 = ´100 + 2 ´100
g L T
t = 2p
l 4p 2l
0.1 1 Þ g= 2
= ´100 + 2 ´ ´100 = 2.72 ; 3% g t
20 90
Dg æ Dl Dt ö æ 0.1 1 ö
0.1 or, = ± ç +2 ÷ = ç + 2 ´ ÷ = 0.027
63. (b) Least count = = 0.01 cm g è l t ø è 20 90 ø
10
d1 = 0.5 + 8 × 0.01 + 0.03 = 0.61 cm Dg
d2 = 0.5 + 4 × 0.01 + 0.03 = 0.57 cm \ % = 2.7%
g
d3 = 0.5 + 6 × 0.01 + 0.03 = 0.59 cm
69. (a) According to ohm’s law, V = IR
0.61 + 0.57 + 0.59 V
Mean diameter = R=
3 I
= 0.59 cm Absolute error
64. (a) The current voltage relation of diode is \ Percentage error = ´102
Measurement
I = (e1000 V /T - 1) mA (given) DV DI
where, ´100 = ´ 100 = 3%
V I
When, I = 5mA, e1000 V /T = 6mA
DR DV DI
then, ´ 100 = ´ 102 + ´ 102
1000 R V I
Also, dI = (e1000 V /T
)´ = 3% + 3% = 6%
T
Error = ± 0.01 (By exponential function) 70. (c) Q Reading of Vernier = Main scale reading
+ Vernier scale reading × least count.
1000 Main scale reading = 58.5
= (6 mA) ´ ´ (0.01) = 0.2 mA
300 Vernier scale reading = 09 division
65. (b) Measured length of rod = 3.50 cm least count of Vernier = 0.5°/30
For Vernier Scale with 1 Main Scale Division = 1 mm 0.5°
9 Main Scale Division = 10 Vernier Scale Division, Thus, R = 58.5° + 9 ×
30
Least count = 1 MSD –1 VSD = 0.1 mm R = 58.65°
66. (a) Rotation period of earth is about 24 hrs ; 105 s 71. (d) No. of divisions on main scale = N
Revolution period of earth is about 365 days ; 107 s No. of divisions on vernier scale = N + 1
Speed of light wave C = 3 × 108 m/s size of main scale division = a
Wavelength of visible light of spectrum Let size of vernier scale division be b
l = 4000 – 7800 Å then we have
C = f l æç and T = ö÷
1 aN
è fø aN = b (N + 1) Þ b =
N +1
Therefore period of light wave is 10–15 s (approx)
aN
67. (d) In a voltmeter Least count is a – b = a –
N +1
V µl
V = kl é N +1 - N ù a
= aê ú = N +1
Now, it is given E = 1.1 volt for l1 = 440 cm ë N +1 û
and V = 0.5 volt for l2 = 220 cm 72. (d) The least count (L.C.) of a screw guage is the smallest
Let the error in reading of voltmeter be DV then, length which can be measured accurately with it.
1.1 = 400 K and (0.5 – DV) = 220 K.
1
1.1 0.5 - DV As least count is 0.001 cm = cm
Þ = 1000
440 220 Hence measured value should be recorded upto 3 decimal
\ DV = -0.05 volt places i.e., 5.320 cm
Physical World, Units and Measurements P-13

1 1
73. (a) Least count, L.C. = mm = MSD
100 30
Diameter of wire = MSR + CSR × L.C. 1
= ´ 0.5° = 1 minute.
Q 1 mm = 0.1 cm 30
1 76. (a) Momentum, p = m × v
= 0+ × 52 = 0.52 mm = 0.052 cm
100 Given, mass of a body = 3.513 kg speed of body
74. (a) Number of significant figures in 23.023 = 5 = (3.513) × (5.00) = 17.565 kg m/s
Number of significant figures in 0.0003 = 1 = 17.6 (Rounding off to get three significant figures)
Number of significant figures in 2.1 × 10–3 = 2
77. (d) Least count of screw gauge = 0.01 mm
So, the radiation belongs to X-rays part of the spectrum.
75. (d) 30 Divisions of V.S. coincide with 29 divisions of M.S. 0.5
29 Q mm
50
\ 1 V.S.D = MSD
30 Reading = [M.S.R. + C.S.R. × L.C.] – (zero error)
L.C. = 1 MSD – 1VSD
= [3 + 35 × 0.01] – (–0.03) = 3.38 mm
29
= 1 MSD - MSD
30
2
14 Physics

Motion in a Straight
Line
Distance, Displacement & TOPIC 2 Non-uniform Motion
TOPIC 1
Uniform Motion
1. A particle is moving with speed v = b x along positive 6. The velocity (v) and time (t) graph of a body in a straight
line motion is shown in the figure. The point S is at 4.333
x-axis. Calculate the speed of the particle at time t = t (assume
seconds. The total distance covered by the body in 6 s is:
that the particle is at origin at t = 0). [12 Apr. 2019 II]
[05 Sep. 2020 (II)]
b2 t b2 t b2 t A B
(a) (b) (c) b2 t (d) v (m/s) 4
4 2 2 2
2. All the graphs below are intended to represent the same S D t (in s)
motion. One of them does it incorrectly. Pick it up. 0
1 2 3 4 5 6
[2018] –2
C
distance
velocity 37 49
(a) m (b) 12 m (c) 11 m (d) m
3 4
(a) position (b) time
7. The speed verses time graph for a particle is shown in the
figure. The distance travelled (in m) by the particle during
velocity
the time interval t = 0 to t = 5 s will be __________.
position [NA 4 Sep. 2020 (II)]
(c) time (d) time 10
8
u
–1 6
3. A car covers the first half of the distance between two (ms )
4
places at 40 km/h and other half at 60 km/h. The average
2
speed of the car is [Online May 7, 2012]
(a) 40 km/h (b) 45 km/h 1 2 3 4 5
time
(c) 48 km/h (d) 60 km/h
(s)
4. The velocity of a particle is v = v0 + gt + ft2. If its position
is x = 0 at t = 0, then its displacement after unit time (t = 8. The distance x covered by a particle in one dimensional
1) is [2007] motion varies with time t as x2 = at2 + 2bt + c. If the
acceleration of the particle depends on x as x–n, where n
(a) v0 + g /2 + f (b) v0 + 2g + 3f
is an integer, the value of n is ______. [NA 9 Jan 2020 I]
(c) v0 + g /2 + f/3 (d) v0 + g + f
9. A bullet of mass 20g has an initial speed of 1 ms–1, just
5. A particle located at x = 0 at time t = 0, starts moving before it starts penetrating a mud wall of thickness 20 cm.
along with the positive x-direction with a velocity 'v' that If the wall offers a mean resistance of 2.5×10–2 N, the speed
varies as v = a x . The displacement of the particle of the bullet after emerging from the other side of the wall
varies with time as [2006] is close to : [10 Apr. 2019 II]
(a) t 2 (b) t (c) t 1/2 (d) t3 (a) 0.1 ms–1 (b) 0.7 ms–1
(c) 0.3 ms–1 (d) 0.4 ms–1
Motion in a Straight Line P-15

10. The position of a particle as a function of time t, is given 14. An automobile, travelling at 40 km/h, can be stopped at a
by distance of 40 m by applying brakes. If the same automobile
x(t) = at + bt2 – ct3 is travelling at 80 km/h, the minimum stopping distance, in
where, a, b and c are constants. When the particle attains metres, is (assume no skidding) [Online April 15, 2018]
zero acceleration, then its velocity will be: (a) 75 m (b) 160 m (c) 100 m (d) 150 m
[9 Apr. 2019 II] 15. The velocity-time graphs of a car and a scooter are shown
b2 b2 in the figure. (i) the difference between the distance
(a) a + (b) a + travelled by the car and the scooter in 15 s and (ii) the time
4c 3c
at which the car will catch up with the scooter are,
b2 b2 respectively [Online April 15, 2018]
(c) a + (d) a +
c 2c
A Car B
11. A particle starts from origin O from rest and moves with a 45
uniform acceleration along the positive x-axis. Identify all (a) 337.5m and 25s F

Velocity (ms –1) ®


E Scooter
figures that correctly represents the motion qualitatively 30 G
(a = acceleration, v = velocity, x = displacement, t = time)
[8 Apr. 2019 II] (b) 225.5m and 10s 15

(c) 112.5m and 22.5s O C


D
0 5 10 15 20 25
(d) 11.2.5m and 15s Time in (s) ®
(A) (B) 16. A man in a car at location Q on a straight highway is moving
with speed v. He decides to reach a point P in a field at a
distance d from highway (point M) as shown in the figure.
Speed of the car in the field is half to that on the highway.
What should be the distance RM, so that the time taken to
reach P is minimum? [Online April 15, 2018]
P
(C) (D)
d
(a) (B), (C) (b) (A)
Q
(c) (A), (B), (C) (d) (A), (B), (D) R M
12. A particle starts from the origin at time t = 0 and moves d d
d
along the positive x-axis. The graph of velocity with (a) (b) (c) (d) d
3 2 2
respect to time is shown in figure. What is the position
of the particle at time t = 5s? [10 Jan. 2019 II] 17. Which graph corresponds to an object moving with a
constant negative acceleration and a positive velocity ?
v
(m/s) [Online April 8, 2017]

3 (a) (b)
2 Velocity
Velocity
1
0 Time
1 2 3 4 5 6 7 8 9 10 Time
(a) 10 m (b) 6 m (c) (d)
(c) 3 m (d) 9 m
13. In a car race on straight road, car A takes a time t less
than car B at the finish and passes finishing point with Velocity Velocity

a speed 'v' more than of car B. Both the cars start from
rest and travel with constant acceleration a1 and a2 Distance
Distance
respectively. Then 'v' is equal to: [9 Jan. 2019 II]
18. The distance travelled by a body moving along a line in
2a1 a 2
(a) t (b) 2a1 a 2 t time t is proportional to t3.
a1 + a 2 The acceleration-time (a, t) graph for the motion of the
a1 + a 2 body will be [Online May 12, 2012]
(c) a1 a 2 t (d) t
2
P-16 Physics

(x1 – x2) (x1 – x2)

a
a
(a) (b) (c) O
t (d) O
t

t t 22. A car, starting from rest, accelerates at the rate f through a


distance S, then continues at constant speed for time t
f
a a and then decelerates at the rate to come to rest. If the
2
total distance traversed is 15 S , then [2005]
(c) (d)
1
(a) S = ft 2 (b) S = f t
t t 6
1 2 1 2
19. The graph of an object’s motion (along the x-axis) is shown (c) S = ft (d) S = ft
4 72
in the figure. The instantaneous velocity of the object at 23. A particle is moving eastwards with a velocity of 5 ms–1.
points A and B are vA and vB respectively. Then In 10 seconds the velocity changes to 5 ms–1 northwards.
[Online May 7, 2012] The average acceleration in this time is [2005]
x(m) 1 -2
(a) ms towards north
15 2
1
(b) ms - 2 towards north - east
2
10 1
(c) ms - 2 towards north - west
B 2
(d) zero
5
A 24. The relation between time t and distance x is t = ax2 + bx
Dx = 4 m where a and b are constants. The acceleration is [2005]
Dt = 8
(a) 2bv3 (b) –2abv 2 (c) 2av2 (d) –2av 3
0 10 20 t (s) 25. An automobile travelling with a speed of 60 km/h, can
(a) vA = vB = 0.5 m/s (b) vA = 0.5 m/s < vB brake to stop within a distance of 20m. If the car is going
twice as fast i.e., 120 km/h, the stopping distance will be
(c) vA = 0.5 m/s > vB (d) vA = vB = 2 m/s [2004]
20. An object, moving with a speed of 6.25 m/s, is decelerated (a) 60 m (b) 40 m (c) 20 m (d) 80 m
at a rate given by 26. A car, moving with a speed of 50 km/hr, can be stopped by
dv brakes after at least 6 m. If the same car is moving at a
= -2.5 v where v is the instantaneous speed. The time speed of 100 km/hr, the minimum stopping distance is
dt
taken by the object, to come to rest, would be: [2011] [2003]
(a) 2 s (b) 4 s (c) 8 s (d) 1 s (a) 12 m (b) 18 m (c) 24 m (d) 6 m
21. A body is at rest at x = 0. At t = 0, it starts moving in the 27. If a body looses half of its velocity on penetrating 3 cm in
positive x-direction with a constant acceleration. At the a wooden block, then how much will it penetrate more
same instant another body passes through x = 0 moving before coming to rest? [2002]
in the positive x-direction with a constant speed. The (a) 1 cm (b) 2 cm (c) 3 cm (d) 4 cm.
position of the first body is given by x1(t) after time ‘t’; 28. Speeds of two identical cars are u and 4u at the specific
and that of the second body by x2(t) after the same time instant. The ratio of the respective distances in which the
interval. Which of the following graphs correctly two cars are stopped from that instant is [2002]
describes (x1 – x2) as a function of time ‘t’? [2008] (a) 1 : 1 (b) 1 : 4 (c) 1 : 8 (d) 1 : 16
(x1 – x2) (x1 – x2)

TOPIC 3 Relative Velocity

(a) t (b) t 29. Train A and train B are running on parallel tracks in the
O O
opposite directions with speeds of 36 km/hour and 72
km/hour, respectively. A person is walking in train A in
the direction opposite to its motion with a speed of 1.8
Motion in a Straight Line P-17

km/hour. Speed (in ms–1) of this person as observed from


train B will be close to : (take the distance between the 2 æhö h
(a) t = ç ÷ (b) t = 1.8
tracks as negligible) [2 Sep. 2020 (I)] 3 ègø g
(a) 29.5 ms–1 (b) 28.5 ms–1
(c) 31.5 ms–1q (d) 30.5 ms–1 æhö 2h
30. A passenger train of length 60 m travels at a speed of 80 (c) t = 3.4 ç ÷ (d) t =
km/hr. Another freight train of length 120 m travels at a ègø 3g
speed of 30 km/h. The ratio of times taken by the 36. A Tennis ball is released from a height h and after freely
passenger train to completely cross the freight train when: falling on a wooden floor it rebounds and reaches height
(i) they are moving in same direction, and (ii) in the h
opposite directions is: [12 Jan. 2019 II] . The velocity versus height of the ball during its motion
2
11 5 3 25 may be represented graphically by :
(a) (b) (c) (d) (graph are drawn schematically and on not to scale)
5 2 2 11
31. A person standing on an open ground hears the sound of [4 Sep. 2020 (I)]
a jet aeroplane, coming from north at an angle 60º with
ground level. But he finds the aeroplane right vertically v v
above his position. If v is the speed of sound, speed of the h/2 h/2
plane is: [12 Jan. 2019 II] (a) h(v) (b) h(v)
3 2v v h h
(a) v (b) (c) v (d)
2 3 2
32. A car is standing 200 m behind a bus, which is also at rest.
The two start moving at the same instant but with differ-
v v
ent forward accelerations. The bus has acceleration 2 m/s2
and the car has acceleration 4 m/s2. The car will catch up
with the bus after a time of : h h
(c) h(v) (d) h(v)
[Online April 9, 2017] h/2 h/2
(a) 110 s (b) 120 s
(c) 10 2 s (d) 15 s
37. A ball is dropped from the top of a 100 m high tower on a
33. A person climbs up a stalled escalator in 60 s. If standing
on the same but escalator running with constant velocity 1
planet. In the last s before hitting the ground, it covers a
he takes 40 s. How much time is taken by the person to 2
walk up the moving escalator? [Online April 12, 2014] distance of 19 m. Acceleration due to gravity (in ms–2) near
the surface on that planet is _______.
(a) 37 s (b) 27 s (c) 24 s (d) 45 s
[NA 8 Jan. 2020 II]
34. A goods train accelerating uniformly on a straight railway
38. A body is thrown vertically upwards. Which one of the
track, approaches an electric pole standing on the side of
following graphs correctly represent the velocity vs time?
track. Its engine passes the pole with velocity u and the
[2017]
guard’s room passes with velocity v. The middle wagon of
the train passes the pole with a velocity.
[Online May 19, 2012]
u+v 1 2 (a) (b)
(a) (b) u + v2
2 2
æ u 2 + v2 ö
(c) uv (d) ç ÷
è 2 ø

(c) (d)
TOPIC 4 Motion Under Gravity

35. A helicopter rises from rest on the ground vertically up- 39. Two stones are thrown up simultaneously from the edge
wards with a constant acceleration g. A food packet is of a cliff 240 m high with initial speed of 10 m/s and 40
dropped from the helicopter when it is at a height h. The m/s respectively. Which of the following graph best
time taken by the packet to reach the ground is close to represents the time variation of relative position of the
[g is the accelertion due to gravity] : [5 Sep. 2020 (I)] second stone with respect to the first ?
P-18 Physics

(Assume stones do not rebound after hitting the ground y


and neglect air resistance, take g = 10 m/ s2) [2015] h
(The figures are schematic and not drawn to scale) (c) O t
t1 2t1
t
(a) (y2 – y1) m (b) (y2 – y1) m v y
240 240 v1 h

(d) O t
t

t(s) t(s)
8 12 8 12
42. A parachutist after bailing out falls 50 m without friction. When
(y2 – y1 ) m
240
(y2 – y1) m parachute opens, it decelerates at 2 m/s2 . He reaches the ground
240
(c) (d) with a speed of 3 m/s. At what height, did he bail out ? [2005]
(a) 182 m (b) 91 m
t(s) 12
t(s) (c) 111m (d) 293m
t® 8 12
43. A ball is released from the top of a tower of height h meters.
40. From a tower of height H, a particle is thrown vertically
It takes T seconds to reach the ground. What is the position
upwards with a speed u. The time taken by the particle, to
hit the ground, is n times that taken by it to reach the T
of the ball at second [2004]
highest point of its path. The relation between H, u and n 3
is: [2014] 8h
(a) 2gH = n u 2 2 2
(b) gH = (n – 2) u d 2 (a) meters from the ground
9
(c) 2gH = nu2 (n – 2) (d) gH = (n – 2)u2 7h
41. Consider a rubber ball freely falling from a height h = 4.9 m (b) meters from the ground
9
onto a horizontal elastic plate. Assume that the duration
of collision is negligible and the collision with the plate is h
(c) meters from the ground
totally elastic. 9
Then the velocity as a function of time and the height as
17 h
a function of time will be : [2009] (d) meters from the ground
v y 18
+v1
h
44. From a building two balls A and B are thrown such that A is
(a) O t thrown upwards and B downwards (both vertically). If vA
–v1 and vB are their respective velocities on reaching the
t
v ground, then [2002]
+v1 y (a) vB > vA
h (b) vA = vB
t
(b) O t1 2t1 4t1 t
(c) vA > vB
–v1 t (d) their velocities depend on their masses.
Motion in a Straight Line P-19

1. (b) Given, v = b x x t x
dx é2 x ù
dx ò x = a ò dt ; ê 1 ú = a[t ]t0
or = b x1/2 0 0 ë û0
dt
x t a2 2
-1/2 Þ 2 x = at Þ x = t
or òx dx = ò bdt 4
0 0 6. (a) 4 A B
1/2
x b2t 2 v(m/s) 2
or = 6t or x = O S D t
1/ 2 4 0 1 2 3 4 5 6 (in s)
Differentiating w. r. t. time, we get
–2
dx b2 ´ 2t C
= (t = t)
dt 4 1 13
OS = 4 + =
b t 2 3 3
or v = 1 5
2 SD = 2 - =
2. (b) Graphs in option (c) position-time and option (a) 3 3
velocity-position are corresponding to velocity-time graph Distance covered by the body = area of v-t graph
option (d) and its distance-time graph is as given below. = ar (OABS) + ar (SCD)
Hence distance-time graph option (b) is incorrect. 1 æ 13 ö 1 5 32 5 37
distance = ç + 1÷ ´ 4 + ´ ´ 2 = + = m
2è 3 ø 2 3 3 3 3
u
7. (20)
8 A

time

Total distance travelled x B t


3. (c) Average speed = = O
Total time taken T 5
x Distance travelled = Area of speed-time graph
= = 48 km/h
x x 1
+ = ´ 5 ´ 8 = 20 m
2 ´ 40 2 ´ 60 2
dx
4. (c) We know that, v = 8. (3) Distance X varies with time t as x2 = at2 + 2bt + c
dt
Þ dx = v dt dx
x t Þ 2x = 2at + 2b
dt
Integrating, ò dx = ò v dt
dx dx (at + b)
0 0 Þx = at + b Þ =
t t dt dt x
é 2 gt 2 ft 3 ù
or x = ò (v0 + gt + ft ) dt = êv0 t + + ú d 2 x æ dx ö
2
ëê 2 3 úû Þx +ç ÷ =a
0 0
dt 2 è dt ø
gt 2 ft 3
or, x = v0 t + + 2 2
2 3 æ dx ö æ at + b ö
2 a -ç ÷ a -ç ÷
g f d x è dt ø = è x ø
At t = 1, x = v0 + + . Þ =
2 3 dt 2 x x
5. (a) v = a x ,
2
dx dx ax 2 - ( at + b ) ac - b 2
Þ =a x Þ = a dt = =
dt x x3 x3
Integrating both sides, Þ a µ x–3 Hence, n = 3
P-20 Physics

9. (b) From the third equation of motion a 2t


v2 – u2 = 2aS Þ to =
a1 – a 2
F Putting this value of t0 in equation (i)
But, a =
m
a2t
2 æFö
2 v =( al – a2 ) – a 2t
\v = u - 2ç ÷ S a1 – a 2
èmø
é 2.5 ´ 10-2 ù 20 = ( a1 + a 2 ) a2 t – a 2t = a1a 2 t + a 2 t – a 2 t
Þ v 2 = (1)2 - (2) ê ú
-3
ëê 20 ´10 ûú 100 or, v = a1a 2 t
1 14. (b) According to question, u1 = 40 km/h, v1 = 0 and s1 = 40 m
Þ v2 = 1 – using v2 – u2 = 2as; 02 – 402 = 2a × 40 ...(i)
2
1 Again, 02 – 802 = 2as ...(ii)
Þv= m/s = 0.7m/s From eqn. (i) and (ii)
2
10. (b) x = at + bt2 – ct3 Stopping distance, s = 160 m

dx d v –u 1 2
Velocity, v = = (at + bt 2 + ct 3 ) 15. (c) Using equation, a = and S = ut + at
t 2
dt dt
= a + 2bt – 3ct2 1 (45)
Distance travelled by car in 15 sec = (15)2
2 15
dv d
Acceleration, = (a + 2bt - 3ct 2 ) 675
dt dt = m
2
æb ö Distance travelled by scooter in 15 seconds = 30 × 15 = 450
or 0 = 2b – 3c × 2t \ t =ç ÷
è 3c ø (Q distance = speed × time)
2 æ 2ö Difference between distance travelled by car and scooter
æb ö æ b ö = a+b in 15 sec, 450 – 337.5 = 112.5 m
and v = a + 2b ç ÷ - 3c ç ÷ ç ÷
è 3c ø è 3c ø è 3c ø
Let car catches scooter in time t;
11. (d) For constant acceleration, there is straight line 675
®
+ 45(t –15) = 30t
2
parallel to t-axis on a - t .
337.5 + 45t – 675 = 30t Þ 15t = 337.5
® ®
Inclined straight line on v - t , and parabola on x - t . Þ t = 22.5 sec
12. (d) Position of the particle, 16. (a) Let the car turn of the highway at a distance 'x' from
the point M. So, RM = x
S = area under graph (time t = 0 to 5s)
And if speed of car in field is v, then time taken by the car
1
= ´ 2 ´ 2 + 2 ´ 2 + 3´1= 9m to cover the distance QR = QM – x on the highway,
2
13. (c) Let time taken by A to reach finishing point is t0 QM - x
t1 = .....(i)
\ Time taken by B to reach finishing point = t0 + t 2v
x Time taken to travel the distance 'RP ' in the field

d 2 + x2
t2 = ..... (ii)
v
u=0 vA = a1t0 Total time elapsed to move the car from Q to P
vB = a2(t0 + t) QM - x d 2 + x2
vA – vB = v
t = t1 + t2 = +
2v v
Þ v = a1 t0 – a2 (t0 + t) = (a1 – a2)t0–a2t ...(i) dt
For 't ' to be minimum =0 P
1 1 dx
x B = x A = a1 t 02 = a 2 (t 0 + t) 2
2 2
1é 1 x ù d
Þ a1 t 0 = a 2 ( t 0 + t ) ê- + ú =0
v ëê 2 d 2 + x 2 ûú
( )
Q
Þ a1 – a 2 t 0 = a 2 t d d R M
or x = =
22 - 1 3
Motion in a Straight Line P-21

17. (c) According to question, object is moving with For the body moving with constant speed
constant negative acceleration i.e., a = – constant (C) x2 = vt
1
vdv \ x1 - x2 = at 2 - vt
= -C 2
dx
vdv = – Cdx at t = 0, x1 – x2 = 0
v2 v2 k This equation is of parabola.
= - Cx + k x=- +
2 2C C v
Hence, graph (3) represents correctly. For t < ; the slope is negative
18. (b) Distance along a line i.e., displacement (s) a
= t3 (Q s µ t 3 given) v
For t = ; the slope is zero
By double differentiation of displacement, we get a
acceleration. v
3 2 For t > ; the slope is positive
ds dt dv d 3t a
V= = = 3t 2 and a = = = 6t
dt dt dt dt These characteristics are represented by graph (b).
a = 6t or a µ t 22. (d) Let car starts from A from rest and moves up to point
Hence graph (b) is correct. B with acceleration f.
Dx 1 2
19. (a) Instantaneous velocity v = Distance, AB = S = ft1
Dt 2
Dx A 4 m Distance, BC = (ft1)t
From graph, vA = = = 0.5 m/s
Dt A 8s
Dx 8m u2 ( ft1 )2
and vB = B = = 0.5 m/s Distance, CD = = = ft12 = 2S
Dt B 16s 2a 2( f / 2)
i.e., vA = vB = 0.5 m/s A f B C f /2 D
dv
20. (a) Given, = -2.5 v t1 t 2t 1
dt
dv
Þ = – 2.5 dt 15 S
v Total distance, AD = AB + BC + CD = 15S
Integrating, AD = S + BC + 2S
0 -½ t Þ S + f t1t + 2 S = 15 S
ò6.25 v dv = -2.5ò dt
0
Þ f t1t = 12 S ............. (i)
0
é v +½ ù
= -2.5 [ t ]0
t
Þ ê (½) ú 1 2
êë úû 6.25 f t1 = S ............ (ii)
2
Þ – 2(6.25)½ = – 2.5t t
Þ – 2 × 2.5 = –2.5t Dividing (i) by (ii), we get t1 =
6
Þ t = 25 2
1 ætö f t2
21. (b) For the body starting from rest, distance travelled Þ S= fç ÷ =
2 è 6ø 72
(x1) is given by
23. (c) v2
1
x1 = 0 + at2 N
2
1 2 D v = v 2 + (-v 1 )
Þ x1 = at
2
x1 – x2
90°
W E
- v1 v1

S
v/a
t uur
Initial velocity, v1 = 5iˆ,
P-22 Physics
uur
Final velocity, v2 = 5 ˆj, æ 5ö
2
uur ur ur Þ 02 - ç 50 ´ ÷ = 2 ´ a ´ 6
Change in velocity D v = (v 2 - v 1 ) è 18 ø
2
= v12 + v22 + 2v1v2 cos 90 æ 5ö
Þ - ç 50 ´ ÷ = 2 ´ a ´ 6
è 18 ø
= 5 2 + 52 + 0 = 5 2m/s 250 ´ 250
a=– » = –16 ms–2.
[As | v1 | = | v2 | = 5 m/s] 324 ´ 2 ´ 6
uur Case-2 : Initial velocity, u = 100 km/hr
Dv
Avg. acceleration = 5
t = 100 ´ m/sec
18
5 2 1
= = m / s2 v = 0, s = s, a = a
10 2 As v2 – u2 = 2as
5 2
tan q = = -1 æ 5ö
-5 Þ 02 - ç100 ´ ÷ = 2as
è 18 ø
which means q is in the second quadrant.
2
(towards north-west) æ 5ö
Þ - ç 100 ´ ÷ = 2 × (–16) × 5
24. (d) Given, t = ax2 + bx; è 18 ø
Diff. with respect to time (t) 500 ´ 500
d d dx dx s= = 24m
(t ) = a ( x 2 ) + b = a.2 x + b.v. 324 ´ 32
dt dt dt dt 27. (a) In first case
Þ 1 = 2axv + bv = v(2ax + b)(v = velocity) u
1 u1 = u ; v1 = , s = 3 cm, a1 = ?
2ax + b = . 2 1
v
Using, v12 - u12 = 2a1s1 ...(i)
Again differentiating, we get
dx 1 dv 2
+0= - 2 æ uö 2
2a
dt çè ÷ø - u = 2 × a × 3
v dt 2
dv æ dx ö
Þ a= = –2av3 çèQ = v÷
ø Þ a=
–u2
dt dt
8
25. (d) In first case speed, In second case: Assuming the same retardation
5 50
u = 60 ´ m/s = m/s -u 2
18 3 u2 = u /2 ; v2 = 0 ; s2 = ?; a2 =
d = 20m, 8
Let retardation be a then v22 - u 22 = 2a2 ´ s2 ...(ii)
(0)2 – u2 = –2ad
or u2 = 2ad …(i) u2 æ –u2 ö
\ 0- = 2ç ÷ ´ s2
5 4 è 8 ø
In second case speed, u¢ = 120 ´
18 Þ s2 = 1 cm
100 28. (d) For first car
= m/s
3 u1 = u, v1 = 0, a1 = – a, s1 = s1
and (0)2 – u¢2 = –2ad¢
As v12 - u12 = 2a1s1
or u¢2 = 2ad¢ …(ii)
(ii) divided by (i) gives, Þ –u2 = –2as1
Þ u2 = 2as1
d'
4 = Þ d ' = 4 ´ 20 = 80m u2
d Þ s1 = ...(i)
26. (c) Fir first case : Initial velocity, 2a
5 For second car
u = 50 ´ m / s, u2 = 4u, v1 = 0, a2 = – a, s2 = s2
18
v = 0,s = 6m, a = a \ v22 - u22 = 2a2 s2
Using, v 2 - u 2 = 2as Þ –(4u)2 = 2(–a)s2
Motion in a Straight Line P-23

Þ 16 u2 = 2as2
1 "escalator"
8u 2 33. (c) Person’s speed walking only is
Þ s2 = ...(ii) 60 second
a Standing the escalator without walking the speed is
Dividing (i) and (ii),
1 "escalator"
s1 u 2 a 1
= × 2 = 40 second
s2 2a 8u 16 Walking with the escalator going, the speed add.
29. (a) According to question, train A and B are running on
1 1 15 "escalator"
parallel tracks in the opposite direction. So, the person’s speed is + =
60 40 120 second
36 km/h
1.8 km/h 120
A So, the time to go up the escalator t == 24 second.
5
34 (d) Let 'S' be the distance between two ends 'a' be the
constant acceleration
VA = 36 km/h = 10 m/s As we know v2 – u2 = 2aS
v2 - u2
or, aS =
B 2
72 km/h Let v be velocity at mid point.
2 2 S
Therefore, vc - u = 2a
VB = -72 km/h = –20 m/s 2
VMA = –1.8 km/h = –0.5 m/s vc2 = u 2 + aS
Vman, B = Vman, A + VA, B v2 - u 2
= Vman, A + VA – VB = –0.5 + 10 – (–20) vc2 = u 2 +
2
= – 0.5 + 30 = 29.5 m/s.
30. (a) u 2 + v2
vc =
vP 2
31. (d) Q P
35. (c) For upward motion of helicopter,
o
60
v2 = u 2 + 2 gh Þ v 2 = 0 + 2 gh Þ v = 2gh
v Now, packet will start moving under gravity.
Let 't' be the time taken by the food packet to reach the
ground.
R (Observer)
1
Distance, PQ = vp × t (Distance = speed × time) s = ut + at 2
2
Distance, QR = V.t
1 1
PQ Þ -h = 2 gh t - gt 2 Þ gt 2 - 2 gh t - h = 0
cos 60° = 2 2
QR
g
1 vp ´ t v 2 gh ± 2 gh + 4 ´ ´h
= Þ vp = 2
or, t =
2 V.t 2 g

2
32. (c) 4 m/sec2 2 m/sec2
Car Bus
2 gh 2h
or, t = (1 + 2) Þ t = (1 + 2)
200 m g g
Given, uC = uB = 0, aC = 4 m/s2, aB = 2 m/s2
hence relative acceleration, aCB = 2 m/sec2 h
or, t = 3.4
1 g
Now, we know, s = ut + at 2
2 36. (c) For uniformly accelerated/ deaccelerated motion :
1
200 = ´ 2t 2 Q u = 0 v 2 = u 2 ± 2 gh
2
Hence, the car will catch up with the bus after time As equation is quadratic, so, v-h graph will be a parabola
t = 10 2 second
P-24 Physics
v 39. (b) y1 = 10t – 5t2 ; y2 = 40t – 5t2
for y1 = – 240m, t = 8s
at t= 0, h = d \ y2 – y1 = 30t for t < 8s.
2 for t > 8s,
d 1 ® 2 : V increases downwards
h 2 ® velocity changes its direction 1
3 1 y2 – y1 = 240 – 40t – gt2
2 ® 3 : V decreases upwards 2
40. (c) Speed on reaching ground
u
collision 2
takes 2 v= u + 2 gh
place H
Now, v = u + at
Initially velocity is downwards (–ve) and then after
collision it reverses its direction with lesser magnitude, i.e. Þ u 2 + 2 gh = -u + gt
velocity is upwards (+ve). u
Note that time t = 0 corresponds to the point on the graph Time taken to reach highest point is t = ,
g
where h = d.
Next time collision takes place at 3. u + u 2 + 2 gH nu
Þt = =
37. (08.00) Let the ball takes time t to reach the ground g g
(from question)
1 2
Using, S = ut + gt Þ 2gH = n(n –2)u2
2
1 2 41. (b) For downward motion v = –gt
Þ S = 0´t + gt The velocity of the rubber ball increases in downward
2
Þ 200 = gt2 [Q 2S = 100m] direction and we get a straight line between v and t with a
negative slope.
200 1 2
Þt= …(i) Also applying y - y0 = ut + at
g 2
1 2 1 2
1 æ 1ö We get y - h = - gt Þ y = h - gt
In last s, body travels a distance of 19 m, so in çt – ÷ 2 2
2 è 2ø The graph between y and t is a parabola with y = h at t = 0.
distance travelled = 81 As time increases y decreases.
2 For upward motion.
1 æ 1ö The ball suffer elastic collision with the horizontal elastic
Now, g ç t – ÷ = 81
2 è 2ø plate therefore the direction of velocity is reversed and the
2 magnitude remains the same.
æ 1ö
\ g ç t – ÷ = 81´ 2 Here v = u – gt where u is the velocity just after collision.
è 2ø As t increases, v decreases. We get a straight line between v
æ 1ö 81´ 2 and t with negative slope.
Þ çt – ÷ =
è 2 ø g 1 2
Also y = ut - gt
1 1 2
\ = ( 200 – 81 ´ 2) using (i) All these characteristics are represented by graph (b).
2 g
42. (d) Initial velocity of parachute
Þ g = 2(10 2 – 9 2) after bailing out,
Þ g =2 2 u= 2gh
\ g = 8 m/s2 u = 2 ´ 9.8 ´ 50 = 14 5
38. (a) For a body thrown vertically upwards acceleration
The velocity at ground,
remains constant (a = – g) and velocity at anytime t is 50 m
given by V = u – gt v = 3m/s
v
During rise velocity decreases linearly and during fall v2 - u2 32 - 980
S= = » 243 m a = - 2 m / s2
velocity increases linearly and direction is opposite to
2´2 4
each other. Initially he has fallen 50 m.
Hence graph (a) correctly depicts velocity versus time.
\ Total height from where 3m / s
he bailed out = 243 + 50 = 293 m
Motion in a Straight Line P-25

1 2 44. (b) Ball A is thrown upwards with


43. (a) We have s = ut + gt ,
2 velocity u
1 u
Þ h = 0 × T + gT2 from the building. During its A u
2 downward journey when it comes
1 2 back to the point of throw, its
Þ h = gT h
2 speed is equal to the speed of
T throw
Vertical distance moved in time is B
3 (u). So, for the journey of both
2
1 æTö 1 gT 2 h the balls from point A to B.
h' = g ç ÷ Þ h' = ´ =
2 è 3ø 2 9 9 We can apply v2 – u2 = 2gh.
h As u, g, h are same for both the
8h
\ Position of ball from ground = h - = balls, vA = vB
9 9
3
P-26 Physics

Motion in a Plane
5. Two forces P and Q, of magnitude 2F and 3F, respectively,
are at an angle q with each other. If the force Q is
TOPIC 1 Vectors
doubled, then their resultant also gets doubled. Then, the
® angle q is: [10 Jan. 2019 II]
1. A force F = (i$ + 2 $j + 3k$ ) N acts at a point (4$i + 3 $j - k$ ) m. (a) 120° (b) 60°
Then the magnitude of torque about the point ($i + 2 $j + k$ ) m (c) 90° (d) 30°
will be x N-m. The value of x is ______. ur ur
6. Two vectors A and B have equal magnitudes. The
ur ur ur ur
r r
[NA Sep. 05, 2020 (I)]
r r ( ) (
magnitude of A + B is ‘n’ times the magnitude of A - B . )
2. The sum of two forces P and Q is R such that | R | = ur ur
r The angle between A and B is: [10 Jan. 2019 II]
r
| P | . The angle q (in degrees) that the resultant of 2 P
r r é n 2 - 1ù é n - 1ù
(a) cos -1 ê 2 ú (b) cos -1 ê
and Q will make with Q is _______.
ë n + 1û ë n + 1úû
[NA 7 Jan. 2020 II]
é n2 -1ù é n -1ù
uur uuur uur uuur (c) sin -1 ê 2 ú (d) sin -1 ê
3. Let A1 = 3, A 2 = 5 and A1 + A 2 = 5. The value of ë n + 1 û ë n + 1úû
uur uuur uur uuur r r
(2A1 + 3A 2 · )( )
3A1 - 2A 2 is : [8 April 2020 II] 7. Let A = (iˆ + ˆj) and B = (iˆ - ˆj) . The magnitude of a
r r r r r r r
coplanar vector C such that A.C = B.C = A.B is given
(a) – 106.5 (b) – 99.5
by [Online April 16, 2018]
(c) – 112.5 (d) – 118.5
5 10
4. In the cube of side ‘a’ shown in the figure, the vector (a) (b)
from the central point of the face ABOD to the central 9 9
point of the face BEFO will be: [10 Jan. 2019 I] 20 9
(c) (d)
9 12
ur
8. A vector A is rotated by a small angle Dq radian (Dq << 1)
ur ur ur
to get a new vector B . In that case B - A is :
[Online April 11, 2015]
ur ur ur
(a) A Dq (b) B Dq - A

ur æ Dq2 ö
(c) A çç 1 - ÷ (d) 0
è 2 ÷ø
r r r r
9. If A ´ B = B ´ A, then the angle between A and B is [2004]
(a)
1
2
(
a kˆ - iˆ ) (b)
1 ˆ ˆ
2
(
a i -k ) (a)
p
(b)
p
2 3

(c)
1
2
(
a ˆj - iˆ ) (d)
1
2
(
a ˆj - kˆ ) (c) p (d)
p
4
Motion in a Plane P-27

15. A particle starts from the origin at t = 0 with an initial


Motion in a Plane with
TOPIC 2 velocity of 3.0 iˆ m/s and moves in the x-y plane with a
Constant Acceleration
constant acceleration (6.0 iˆ + 4.0 ˆj) m/ s 2. The x-
10. A balloon is moving up in air vertically above a point A on the coordinate of the particle at the instant when its y-
ground. When it is at a height h1, a girl standing at a distance coordinate is 32 m is D meters. The value of D is:
d (point B) from A (see figure) sees it at an angle 45º with [9 Jan. 2020 II]
(a) 32 (b) 50 (c) 60 (d) 40
respect to the vertical. When the balloon climbs up a further
16. A particle is moving along the x-axis with its coordinate
height h2, it is seen at an angle 60º with respect to the vertical with time ‘t’ given by x(t) = 10 + 8t – 3t2. Another particle is
if the girl moves further by a distance 2.464 d (point C). Then the moving along the y-axis with its coordinate as a function of
height h2 is (given tan 30º = 0.5774): [Sep. 05, 2020 (I)] time given by y(t) = 5 – 8t3. At t = 1 s, the speed of the
second particle as measured in the frame of the first particle
is given as v . Then v (in m/s) is____ [NA 8 Jan. 2020 I]
r
h2 17. A particle moves such that its position vector r (t) = cos
wt iˆ + sin wt ĵ where w is a constant and t is time. Then
r
h1 which of the following statements is true for the velocity v
45° 60° r
(t) and acceleration a (t) of the particle: [8 Jan. 2020 II]
r r r
A d B 2.464d C (a) v is perpendicular to r and a is directed away from
the origin
(a) 1.464 d (b) 0.732 d r r r
(b) v and a both are perpendicular to r
(c) 0.464 d (d) d r r r
(c) v and a both are parallel to r
11. Starting from the origin at time t = 0, with initial velocity r r r
(d) v is perpendicular to r and a is directed towards
5 ˆj ms–1, a particle moves in the x–y plane with a constant the origin
r
18. A particle is moving with velocity n = k ( yiˆ + xjˆ) , where k
acceleration of (10iˆ + 4 ˆj) ms–2. At time t, its coordiantes
is a constant. The general equation for its path is [2010]
are (20 m, y0 m). The values of t and y0 are, respectively : (a) y = x2 + constant (b) y2 = x + constant
[Sep. 04, 2020 (I)] (c) xy = constant (d) y2 = x2 + constant
(a) 2 s and 18 m (b) 4 s and 52 m 19. A particle has an initial velocity of 3iˆ + 4 ˆj and an
(c) 2 s and 24 m (d) 5 s and 25 m acceleration of 0.4iˆ + 0.3 ˆj . Its speed after 10 s is : [2009]
12. The position vector of a particle changes with time
(a) 7 2 units (b) 7 units
r
according to the relation r (t) = 15 t 2 $i + (4 - 20 t 2 ) $j. (c) 8.5 units (d) 10 units
What is the magnitude of the acceleration at t = 1? 20. The co-ordinates of a moving particle at any time ‘t’are
given by x = a t 3 and y = b t 3 . The speed of the particle
[9 April 2019 II]
at time ‘t’ is given by [2003]
(a) 40 (b) 25 (c) 100 (d) 50
(a) 3t a 2 + b2 (b) 3t 2 a 2 + b2
13. ( )
A particle moves from the point 2.0iˆ + 4.0 ˆj m , at t = 0,

( )
with an initial velocity 5.0iˆ + 4.0 ˆj ms -1 . It is acted upon (c) t 2 a 2 + b2 (d) a 2 + b2
by a constant force which produces a constant acceleration
( )
4.0iˆ + 4.0 ˆj ms -2 . What is the distance of the particle TOPIC 3 Projectile Motion
from the origin at time 2s? [11 Jan. 2019 II]
(a) 15 m (b) 20 2m 21. A particle of mass m is projected with a speed u from the
(c) 5 m (d) 10 2m p
ground at an angle q = w.r.t. horizontal (x-axis). When
14. A particle is moving with a velocity vr = K (y iˆ + x ĵ ), 3
it has reached its maximum height, it collides completely
where K is a constant. The general equation for its path
inelastically with another particle of the same mass and
is: [9 Jan. 2019 I]
(a) y = x + constant
2
(b) y = x + constant
2 velocity uiˆ. The horizontal distance covered by the combined
(c) y2 = x2 + constant (d) xy = constant mass before reaching the ground is: [9 Jan. 2020 II]
P-28 Physics

27. Two guns A and B can fire bullets at speeds 1 km/s and
3 3 u2 3 2 u2
(a) (b) 2 km/s respectively. From a point on a horizontal
8 g 4 g ground, they are fired in all possible directions. The
5 u2 u2 ratio of maximum areas covered by the bullets fired by
(c) (d) 2 2 the two guns, on the ground is: [10 Jan. 2019 I]
8 g g
(a) 1 : 16 (b) 1 : 2 (c) 1 : 4 (d) 1 : 8
22. The trajectory of a projectile near the surface of the earth
28. The initial speed of a bullet fired from a rifle is 630 m/s. The
is given as y = 2x – 9x2. If it were launched at an angle q0
rifle is fired at the centre of a target 700 m away at the same
with speed v0 then (g = 10 ms–2): [12 April 2019 I]
level as the target. How far above the centre of the target ?
1 5 [Online April 11, 2014]
(a) q0 = sin –1 and v0 = ms–1
5 3 (a) 1.0 m (b) 4.2 m (c) 6.1 m (d) 9.8 m
æ 2 ö 3 29. The position of a projectile launched from the origin at t =
(b) q0 = cos–1 çè 5 ÷ø and v0 = ms–1 r
5 ( )
0 is given by r = 40iˆ + 50 ˆj m at t = 2s. If the projectile

æ 1 ö was launched at an angle q from the horizontal, then q is


9
(c) q0 = cos–1 çè ÷ø and v0 = ms–1 (take g = 10 ms–2) [Online April 9, 2014]
5 3
-1 2 -1 3
(a) tan (b) tan
æ 2 ö 3 3 2
(d) q0 = sin –1 çè ÷ø and v0 = ms–1
5 5 -1 7 -1 4
(c) tan (d) tan
23. A shell is fired from a fixed artillery gun with an initial 4 5
speed u such that it hits the target on the ground at a 30. A projectile is given an initial velocity of (iˆ + 2 ˆj ) m/s,
distance R from it. If t1 and t2 are the values of the time
where iˆ is along the ground and ĵ is along the vertical.
taken by it to hit the target in two possible ways, the
product t1t2 is : [12 April 2019 I] If g = 10 m/s2 , the equation of its trajectory is : [2013]
(a) R/4g (b) R/g (c) R/2g (d) 2R/g (a) y = x - 5 x 2 (b) y = 2 x - 5 x 2
24. Two particles are projected from the same point with the
(c) 4 y = 2 x - 5 x 2 (d) 4 y = 2 x - 25 x 2
same speed u such that they have the same range R, but
different maximum heights, h1 and h2. Which of the 31. The maximum range of a bullet fired from a toy pistol
following is correct ? [12 April 2019 II] mounted on a car at rest is R0= 40 m. What will be the acute
(a) R = 4 h1h2
2
(b) R =16 h1h2
2 angle of inclination of the pistol for maximum range when
the car is moving in the direction of firing with uniform
(c) R = 2 h1h2
2
(d) R2 = h1h2
velocity v = 20 m/s, on a horizontal surface ? (g = 10 m/s 2)
25. A plane is inclined at an angle a = 30o with respect to the
[Online April 25, 2013]
horizontal. A particle is projected with a speed u = 2 ms–1,
from the base of the plane, as shown in figure. The distance (a) 30° (b) 60° (c) 75° (d) 45°
from the base, at which the particle hits the plane is close to 32. A ball projected from ground at an angle of 45° just clears
a wall in front. If point of projection is 4 m from the foot of
: (Take g=10 ms–2) [10 April 2019 II]
wall and ball strikes the ground at a distance of 6 m on the
other side of the wall, the height of the wall is :
[Online April 22, 2013]
(a) 4.4 m (b) 2.4 m (c) 3.6 m (d) 1.6 m
33. A boy can throw a stone up to a maximum height of 10 m.
The maximum horizontal distance that the boy can throw
(a) 20 cm (b) 18 cm (c) 26 cm (d) 14 cm the same stone up to will be [2012]
26. A body is projected at t = 0 with a velocity 10 ms–1 at an (a) 20 2 m (b) 10 m
angle of 60° with the horizontal. The radius of curvature (c) 10 2 m (d) 20 m
of its trajectory at t = 1s is R. Neglecting air resistance 34. A water fountain on the ground sprinkles water all around
and taking acceleration due to gravity g = 10 ms–2, the it. If the speed of water coming out of the fountain is v, the
value of R is: [11 Jan. 2019 I]
total area around the fountain that gets wet is: [2011]
(a) 10.3 m (b) 2.8 m
v4 p v4 v2 v2
(c) 2.5 m (d) 5.1 m (a) p (b) 2 2 (c) p (d) p
2 2
g g g g
Motion in a Plane P-29

35. A projectile can have the same range ‘R’ for two angles 41. Ship A is sailing towards north-east with velocity km/hr
of projection. If ‘T1’ and ‘T2’ to be time of flights in the where points east and , north. Ship B is at a distance of 80
two cases, then the product of the two time of flights is km east and 150 km north of Ship A and is sailing towards
directly proportional to. [2004] west at 10 km/hr. A will be at minimum distance from B in:
1 1
(a) R (b) (c) 2 (d) R2 [8 April 2019 I]
R R (a) 4.2 hrs. (b) 2.6 hrs.
36. A ball is thrown from a point with a speed ' v0 ' at an (c) 3.2 hrs. (d) 2.2 hrs.
42. Two particles A, B are moving on two concentric circles
elevation angle of q. From the same point and at the same of radii R1 and R2 with equal angular speed w. At t = 0,
' v0 ' their positions and direction of motion are shown in the
instant, a person starts running with a constant speed figure : [12 Jan. 2019 II]
2 Y

to catch the ball. Will the person be able to catch the ball? If
yes, what should be the angle of projection q? [2004] A
(a) No (b) Yes, 30°
(c) Yes, 60° (d) Yes, 45° R1 X
37. A boy playing on the roof of a 10 m high building throws
B
a ball with a speed of 10m/s at an angle of 30º with the R2

horizontal. How far from the throwing point will the ball be
at the height of 10 m from the ground ? [2003]
® ® and t =
p
1 3 The relative velocity vA - vB is given by:
[ g = 10m/s , sin 30 = , cos 30o =
2 o
] 2w
2 2
(a) 5.20m (b) 4.33m (c) 2.60m (d) 8.66m (a) w(R1 + R2) iˆ (b) –w(R1 + R2) iˆ

(c) w(R2 – R1) iˆ (d) w(R1 – R2) iˆ


Relative Velocity in Two
TOPIC 4 Dimensions & Uniform 43. A particle is moving along a circular path with a constant
Circular Motion speed of 10 ms–1. What is the magnitude of the change in
velocity of the particle, when it moves through an angle of
38. A clock has a continuously moving second's hand of 0.1 60° around the centre of the circle?
m length. The average acceleration of the tip of the hand [Online April 10, 2015]
(in units of ms–2) is of the order of: [Sep. 06, 2020 (I)] (a) (b) zero
10 3m/s
(a) 10 –3 (b) 10 –4
(c) 10 2m/s (d) 10 m/s
(c) 10 –2 (d) 10 –1
44. If a body moving in circular path maintains constant speed
39. When a carsit at rest, its driver sees raindrops falling on of 10 ms–1, then which of the following correctly describes
it vertically. When driving the car with speed v, he sees relation between acceleration and radius?
that raindrops are coming at an angle 60º from the hori- [Online April 10, 2015]
zontal. On furter increasing the speed of the car to (1 +
b)v, this angle changes to 45º. The value of b is close to:
[Sep. 06, 2020 (II)] a a
(a) (b)
(a) 0.50 (b) 0.41
(c) 0.37 (d) 0.73
r r
40. The stream of a river is flowing with a speed of 2 km/h.
A swimmer can swim at a speed of 4 km/h. What should
be the direction of the swimmer with respect to the flow a a
of the river to cross the river straight? [9 April 2019 I] (c) (d)
(a) 90° (b) 150°
(c) 120° (d) 60°
r r
P-30 Physics

1. (195) 4 F2 + 36F2 + 24F2 cos q


r = 4 (13F2 +12F2cosq)= 52 F2 + 48 F2 cosq
Given : F = (iˆ + 2 ˆj + 3kˆ) N
r 12F2 1
Þ q = 120o
And, r = [(4iˆ + 3 ˆj - kˆ) - (iˆ + 2 ˆj + kˆ)] = 3iˆ + ˆj - 2kˆ \ cos q = – =–
24F2 2
r r r r
Torque, t = r ´ F = (3iˆ + ˆj - 2kˆ) ´ (iˆ + 2 ˆj + 3kˆ) 6. (a) Let magnitude of two vectors A and B = a
r r
iˆ ˆj kˆ | A + B | = a 2 + a 2 + 2a 2 cos q and
t = 3 1 -2 = 7iˆ - 11 ˆj + 5kˆ r r
| A – B | = a 2 + a 2 – 2a 2 éëcos (180° – q ) ùû
1 2 3
r = a 2 + a 2 – 2a 2 cos q
Magnitude of torque, | t | = 195. and accroding to question,
2. (90) Given, r r r r
| A + B| = n | A–B|
r r r r r 2P + Q
R = P Þ P+Q = P 2P a 2 + a 2 + 2a 2 cos q
or, = n2
P2 + Q2 + 2PQ. cosq = P2 a 2 + a 2 – 2a 2 cos q
Þ Q + 2P cosq = 0 a q
Þ
a 2 (1+ 1 + 2cos q)
n2 Þ
(1+ cos q ) = n 2
Þ cos q = –
Q Q a (1+ 1 – 2cos q)
2
(1– cos q)
..(i)
2P using componendo and dividendo theorem, we get
2 P sin q
tan a = = ¥ (Q 2 P cos q + Q = 0) æ n 2 –1 ö
Q + 2 P cos q q = cos –1 ç 2 ÷
è n +1 ø
Þ a = 90°
r r r r r
3. (d) Using, 7. (a) If C = aiˆ + bjˆ then A.C = A.B
R2 = A12 + A22 + 2A1A2cos q a+b=1 ..... (i)
52 = 32 + 52 + 2 × 3 × 5 cos q rr r r
or cos q = – 0.3 B.C = A.B
2a – b = 1 ..... (ii)
æ ® ®ö æ ® ®ö
çè 2 A1 + 3 A2÷ . ç 3 A1 - 2 A2 ÷ = 2A × 3A Solving equation (i) and (ii) we get
ø è ø 1 1
1 2
+ (3A2) (3A1) cos q – (2A1)(2A2) cos q – 3A2 × 2A2 a= ,b=
3 3
= 6A12 + 9A1A2 cos q – 4A1A2cos q – 6A22 r 1 4 5
\ Magnitude of coplanar vector, C = + =
= 6A12 6A22 + 5A-1A2 cos q 9 9 9
= 6 × 32 – 6 × 52 + 5 × 3 × 5 (– 0.3) 8. (a) Arc length = radius × angle
ur ur ur
= – 118.5 So, | B – A |=| A | D q
4. (c) From figure,
r aˆ a ˆ
rG = i + k
2 2 B A–B
r aˆ a ˆ
rH = j + k q
2 2
r r æ a ˆ a ˆö æ a ˆ a ˆö a ˆ ˆ
\ rH – rG = ç j + k÷ – ç i + k ÷ = j – i
è2 2 ø è2 2 ø 2 ( ) 9.
r r r r
A
r r r r
(c) A ´ B - B ´ A = 0 Þ A ´ B + A ´ B = 0
r r
5. (a) Using, R2 = P2 + Q2 + 2PQcosq \ A´ B = 0
4 F2 + 9F2 + 12F2 cos q = R2 Angle between them is 0, p, or 2 p
When forces Q is doubled, from the given options, q = p
4 F2 + 36F2 + 24F2 cos q = 4R2
Motion in a Plane P-31

10. (d) From figure/ trigonometry, 14. (c) From given equation,
r
h1
= tan 45° \ h1 = d
(
V = K yiˆ + xjˆ )
d
dx dy
= ky and = kx
dt dt
h2 dy
dt = x = dy
h1 Now dx y dx ,Þ ydy = xdx
45° 30° dt
A d B 2.464d C Integrating both side
h1 + h2 y2 = x2 + c
And, = tan 30° 1 2
d + 2.464 d 15. (c) Using S = ut + at
2
Þ (h1 + h2 ) ´ 3 = 3.46d
1
3.46d 3.46d y = u y t + a y t 2 (along y Axis)
Þ (h1 + h2 ) = Þ d + h2 = 2
3 3 1
Þ 32 = 0 ´ t + (4)t 2
\ h2 = d 2
r 1 2
11. (a) Given : u = 5 ˆj m/s Þ ´ 4 ´ t = 32
2
r Þt=4s
Acceleration, a = 10iˆ + 4 ˆj and
1
final coordinate (20, y0) in time t. S x = u xt + a xt 2 (Along x Axis)
2
1 1
S x = ux t + ax t 2 [Q ux = 0] Þ x = 3 ´ 4 + ´ 6 ´ 4 2 = 60
2 2
1 16. (580)
Þ 20 = 0 + ´ 10 ´ t 2 Þ t = 2 s
2 For pariticle ‘A’ For particle ‘B’
1
S y = u y ´ t + ayt 2 XA = –3t + 8t + 10
2
YB = 5 – 8t3
2 r r
1 VA = (8 – 6t )iˆ VB = –24t 2 ˆj
y0 = 5 ´ 2 + ´ 4 ´ 22 = 18 m r r
2 aA = –6iˆ aB = -48tjˆ
® At t = 1 sec
12. (d) r = 15t 2iˆ + (4 - 20t 2 ) ˆj
r r
® VA = (8 – 6t )iˆ = 2iˆ and vB = –24 ˆj
® d r r r r
v = = 30tiˆ - 40tjˆ \ V B / A = – v A + vB = –2iˆ – 24 ˆj
dt
® \ Speed of B w.r.t. A, v = 22 + 242
®
Acceleration, a = d v = 30iˆ - 40 ˆj = 4 + 576 = 580
dt
\ v = 580 (m/s)
\ a = 302 + 402 = 50 m/s 2
17. (d) Given, Position vector,
r r 1r r
13. (b) As S = ut + at 2 r = cos wtiˆ + sin wt ˆj
2 r
r 1 r dr
S = (5iˆ + 4j)2
ˆ + (4iˆ+ 4 ˆj)4 Velocity, v = = w (– sin wtiˆ + cos wt ˆj )
2 dt
Acceleration,
= 10iˆ + 8jˆ + 8iˆ+ 8 ˆj r
r r r dv
rf - ri = 18iˆ + 16jˆ a= = - w 2 (cos wtiˆ + sin wt ˆj )
r r r dt
[as s = change in position = rf - ri ] r r
a = -w 2 r
r
rr = 20iˆ + 20ˆj r
\ a is antiparallel to rr
r
| rr |= 20 2 r r r r
Also v . r = 0 \v ^ r
Thus, the particle is performing uniform circular motion.
P-32 Physics

18. (d) v = k(yi + xj) We have,


v = kyi + kxj 1
dx dy tan q = 2 or cos q =
5
= ky, = kx
dt dt g 10
dy dy dt and = 9 or =9
= ´ 2
2u cos q 2 2u (1/ 5) 2
2
dx dt dx
\ u = 5/3 m/s
dy kx
= 23. (d) R will be same for q and 90° – q.
dx ky
ydy = xdx ...(i) Time of flights:
Integrating equation (i) 2u sin q
t1 = and
g
ò ydy = ò x × dx
2u sin(90° - q) 2u cos q
y2 = x2 + c t2 = =
r r g g
19. (a) Given u = 3iˆ + 4 ˆj , a = 0.4iˆ + 0.3 ˆj , t = 10 s
From 1st equatoin of motion. æ 2u sin q ö æ 2u cos q ö
Now, t1t2 = ç g ÷ ç g ÷ø
v–u è øè
a=
t 2 æ u 2 sin 2q ö 2 R
\ v = at tu = gç ÷=
è g ø g
Þ v = ( 0.4iˆ + 0.3 ˆj ) ´ 10 + ( 3iˆ + 4 ˆj )
24. (b) For same range, the angle of projections are :
Þ 4iˆ + 3 ˆj + 3 ˆj + 4 ˆj q and 90° – q. So,
Þ v = 7iˆ + 7 ˆj u 2sin 2q
r h1 = and
Þ v = 72 + 72 = 7 2 unit. 2g
20. (b) Coordinates of moving particle at time ‘t’ are
u 2sin 2 (90° - q) u 2 cos 2 q
x = at3 and y = bt3 h2 = =
2g 2g
dx dy
vx = = 3at 2 and v y = = 3bt 2 u 2sin 2q
dt dt Also, R =
g
\ v = v2x + v2y = 9a 2t 4 + 9b2t 4
u 2sin 2q u 2 cos2 q
2 2 2 h1 h2 = ×
= 3t a +b 2g 2g
21. (a) Using principal of conservation of linear momentum u 2 u 2 (2sin q cos q) 2
=
for horizontal motion, we have 16 g2
2mvx = mu + mu cos 60°
R2
3u =
vx = 16
4 or R2 = 16 h1 h2
For vertical motion
25. (a) On an inclined plane, time of flight (T) is given by
1 2 2h
h = 0+ gT Þ T = 2u sin q
2 g T=
Let R is the horizontal distance travelled by the body. g cos a
1 Substituting the values, we get
R = v xT + (0)(T ) 2 (For horizontal motion)
2 (2)(2sin15°) 4sin15°
T= =
3u 2h g cos 30° 10 cos30°
R = v xT = ´
1
4 g
Distance, S = (2cos15°)T - g sin 30°(T ) 2
2
3 3u 2
Þ R= x
8g
30
22. (c) Given, y = 2x – 9x2 y 2 m/s gsin
On comparing with, q =15° g gcos30
2 a = 30°
gx
y = x tan q - ,
2 2
2u cos q
Motion in a Plane P-33

4 sin15° æ1
2
ö 16sin 15° 500
= (2cos15°) - ç ´ 10sin 30°÷ = m = 6.1 m
10 10cos 30° è 2 ø 100cos 2 30° 81
Therefore, the rifle must be aimed 6.1 m above the centre
16 3 - 16 of the target to hit the target.
= ; 0.1952m ; 20cm
60 29. (c) From question,
26. (b) Horizontal velocity (initial),
10 m/s 5
g q 40
ux = = 20m/s
o
2
60 gcosq v 1 2
Vertical velocity (initial), 50 = uy t + gt
2
g
1
(10 - 5 3) Þ uy × 2 + (–10) ×4
2
Horizontal component of velocity or, 50 = 2uy – 20
vx = 10cos 60° = 5 m/s 70
or, uy = = 35m / s
vertical component of velocity 2
u y 35 7
vy = 10cos 30° = 5 3 m/s \ tan q = = =
After t = 1 sec. u x 20 4
Horizontal component of velocity vx = 5 m/s 7
Vertical component of velocity Þ Angle q = tan–1
4
( )
vy = | 5 3 –10 | m / s = 10 – 5 3 r ˆ
30. (b) From equation, v = i + 2 ˆj
v2 Þ x=t … (i)
Centripetal, acceleration an =
R 1
y = 2t - (10t 2 ) … (ii)
v +v
2 2
25 +100 + 75 –100 3 2
ÞR= x
=
y
...(i)
an 10cos q From (i) and (ii), y = 2x – 5x2
From figure (using (i)) 31. (b)
10 – 5 3
tan q= = 2 – 3 Þq= 15°
5 P

R=
(
100 2 – 3 ) = 2.8m 32. (b)
45°
wall
10cos15 O 4m A 6m
u 2 sin 2q As ball is projected at an angle 45° to the horizontal
27. (a) As we know, range R =
g therefore Range = 4H
and, area A = p R2 10
or 10 = 4H Þ H = = 2.5 m
\ A µ R2 or, A µ u4 4
(Q Range = 4 m + 6 m = 10m)
4
A1 u14 é 1 ù 1
\ = = = u 2 sin 2 q
A 2 u 42 êë 2 úû 16 Maximum height, H =
2g
28. (c) Let ‘t’ be the time taken by the bullet to hit the target.
H ´ 2g 2.5 ´ 2 ´10
\ 700 m = 630 ms–1 t \ u2 = = = 100
2 2
sin q æ 1 ö
700m 10 ç ÷
Þ t= = sec è 2ø
630ms -1 9
For vertical motion, or, u = 100 = 10 ms -1
Here, u = 0 Height of wall PA

1 2 1 g(OA) 2
\ h = gt = OA tan q -
2 2 u 2 cos2 q
2 1 10 ´ 16
1 æ 10 ö = 4- ´ = 2.4 m
= ´ 10 ´ ç ÷ 2
10 ´ 10 ´
1
´
1
2 è 9ø
2 2
P-34 Physics

u 2 sin2 q u 2 sin 2 q
33. (d) R = ,H=
g 2g
vr vr
Hmax at 2q = 90° vr
u2 60° 45°
Hmax =
2g –vcar = v –vcar = (b + 1)v
v
u2
= 10 Þ u2 = 10 g ´ 2
2g When car is moving with speed v,

u 2 sin 2q vr
u2 tan 60° = ...(i)
R= Þ Rmax = v
g g
10 ´ g ´ 2 When car is moving with speed (1 + b)v ,
Rmax = = 20 metre
g vr
34. (a) Let, total area around fountain tan 45° = ...(ii)
(b + 1)v
2 ...(i)
A = pRmax
Dividing (i) by (ii) we get,
v 2 sin 2q v 2 sin 90° v 2
Where Rmax = = = ...(ii) 3v = (b + 1)v Þ b = 3 - 1 = 0.732.
g g g
From equation (i) and (ii) u 2 1
v4 40. (c) sin q = = =
A=p v 4 2 v
g2 or q = 30°
35. (a) A projectile have same range for two angle q
Let one angle be q, then other is 90° – q with respect to flow, u
2u sin q 2u cos q = 90° + 30° = 120°
T1 = , T2 =
g g
ĵ (North)
4u 2 sin q cos q
then, T1T2 = = 2R
g
B
u 2 sin 2 q 41. (b)
(Q R = )
g rBA
iˆ (East)
Thus, it is proportional to R. (Range) A
36. (c) Yes, Man will catch the ball, if the horizontal r
component of velocity becomes equal to the constant vA = 30iˆ + 50 ˆj km/hr
speed of man. r
vB = (-10iˆ) km/hr
vo
= vo cos q
2 rBA = (80iˆ + 150 ˆj ) km
or q = 60° r r r
37. (d) Horizontal range is required vBA = vB - v A = -10iˆ - 30iˆ - 50iˆ = 40iˆ - 50 ˆj

R=
u 2 sin 2q (10)2 sin(2 ´ 30°)
= = 5 3 = 8.66 m tminimum =
( rrBA )(· vrBA )
( vrBA )
g 10 2
38. (a) Here, R = 0.1 m
2p 2 p (80iˆ + 150 ˆj )( -40iˆ - 50 ˆj )
w= = = 0.105 rad /s
T 60 =
(10 41) 2
Acceleration of the tip of the clock second's hand,
a = w 2 R = (0.105)2 (0.1) = 0.0011 = 1.1 ´ 10 -3 m/s2 10700 107
\t= = = 2.6 hrs.
Hence, average acceleration is of the order of 10–3. 10 41 ´ 10 41 41
39. (d) The given situation is shown in the diagram. Here vr
be the velocity of rain drop.
Motion in a Plane P-35

p p Change in velocity,
42. (c) From, q = wt = w =
2w 2 | Dv | = v12 + v 22 + 2v1 v 2 cos ( p – q )
So, both have completed quater circle q r r
= 2vsin
2
(Q| v1 | = | v 2 |) = v
1
wR1 A = (2 × 10) × sin(30°) = 2 × 10 ×
2
X = 10 m/s
44. (c) Speed, V = constant (from question)
Centripetal acceleration,
wR2 B
V2
a=
Relative velocity, r
( )
v A – v B =wR1 –iˆ - wR 2 ( –i ) =w ( R 2 – R1 ) i ra = constant
Hence graph (c) correctly describes relation between
43. (d) v2 acceleration and radius.

v1
q

v2 v1

(p - q)
- v1
4
P-36 Physics

Laws of Motion
Ist, Ind & IIIrd Laws of 1 æ g ö 1 æ g ö
TOPIC 1 tan -1 ç V0 ÷ sin -1 ç V
Motion (a)
gg ç ÷ (b)
gg ç g 0 ÷÷
è g ø è ø
1. A particle moving in the xy plane experiences a velocity 1 æ g ö 1 æ 2g ö
l n ç1 + V0 ÷ (d) tan -1 ç V
r (c)
g g çè g ÷ø ç g 0 ÷÷
dependent force F = k ( v y i$ + v x $j ) , where vx and vy are x 2g g è ø
r r 5. A ball is thrown vertically up (taken as + z-axis) from the
and y components of its velocity v . if a is the accelera-
ground. The correct momentum-height (p-h) diagram is:
tion of the particle, then which of the following statements
[9 April 2019 I]
is true for the particle? [Sep. 06, 2020 (II)]
r r
(a) quantity v ´ a is constant in time
r
(b) F arises due to a magnetic field (a) (b)
(c) kinetic energy of particle is constant in time
r r
(d) quantity v × a is constant in time
2. A spaceship in space sweeps stationary interplanetary dust.
dM (t ) (c) (d)
As a result, its mass increases at a rate = bv 2 (t ),
dt
where v (t) is its instantaneous velocity. The instantaneous 6. A particle of mass m is moving in a straight line with
acceleration of the satellite is : [Sep. 05, 2020 (II)]
momentum p. Starting at time t = 0, a force F = kt acts in the
bv 3 same direction on the moving particle during time interval
(a) -bv3 (t ) (b) -
M (t ) T so that its momentum changes from p to 3p. Here k is a
2bv 3 bv 3 constant. The value of T is : [11 Jan. 2019 II]
(c) - (d) -
M (t ) 2 M (t ) k p
3. A small ball of mass m is thrown upward with velocity u (a) 2 (b) 2
p k
from the ground. The ball experiences a resistive force
mkv2 where v is its speed. The maximum height attained 2k 2p
by the ball is : [Sep. 04, 2020 (II)] (c) (d)
p k
1 ku 2 1 æ ku 2 ö 7. A particle of mass m is acted upon by a force F given by
(a) tan -1 (b) ln 1 +
2k g k çè 2 g ÷ø R
the empirical law F = 2 v(t). If this law is to be tested
1 ku 2 1 æ ku 2ö t
(c) tan -1 (d) ln 1 +
k 2g 2 k çè g ÷ø experimentally by observing the motion starting from rest,
the best way is to plot : [Online April 10, 2016]
4. A ball is thrown upward with an initial velocity V0 from the
1
surface of the earth. The motion of the ball is affected by a (a) log v(t) against (b) v(t) against t2
drag force equal to mgv2 (where m is mass of the ball, v is t
its instantaneous velocity and g is a constant). Time taken 1
(c) log v(t) against 2 (d) log v(t) against t
by the ball to rise to its zenith is : [10 April 2019 I] t
Laws of Motion P-37

8. A large number (n) of identical beads, each of mass m (a) Statement 1 is true, Statement 2 is true, Statement 2 is
and radius r are strung on a thin smooth rigid horizontal the correct explanation of Statement 1.
rod of length L (L >> r) and are at rest at random (b) Statement 1 is false, Statement 2 is true.
positions. The rod is mounted between two rigid (c) Statement 1 is true, Statement 2 is false.
supports (see figure). If one of the beads is now given (d) Statement 1 is true, Statement 2 is true, Statement 2 is
a speed v, the average force experienced by each support not the correct explanation of Statement 1.
after a long time is (assume all collisions are elastic): 12. Two fixed frictionless inclined planes making an angle 30°
[Online April 11, 2015] and 60° with the vertical are shown in the figure. Two
blocks A and B are placed on the two planes. What is the
relative vertical acceleration of A with respect to B ? [2010]
L A
B

30°
mv 2 mv 2 60°
(a) (b) (a) 4.9 ms–2 in horizontal direction
2(L - nr) L - 2nr
(b) 9.8 ms–2 in vertical direction
mv 2 (c) Zero
(c) (d) zero
L - nr (d) 4.9 ms–2 in vertical direction
9. A body of mass 5 kg under the action of constant force 13. A ball of mass 0.2 kg is thrown vertically upwards by applying
r r
( )
F = Fxˆi + Fy ˆj has velocity at t = 0 s as v = 6iˆ - 2ˆj m/s a force by hand. If the hand moves 0.2 m while applying the
force and the ball goes upto 2 m height further, find the
r r
and at t = 10s as v = +6ˆj m / s . The force F is: magnitude of the force. (Consider g = 10 m/s2). [2006]
[Online April 11, 2014] (a) 4 N (b) 16 N (c) 20 N (d) 22 N
14. A player caught a cricket ball of mass 150 g moving at a
æ 3 ˆ 4 ˆö
(a) ( )
-3jˆ + 4ˆj N (b) ç - i + j ÷ N
è 5 5 ø
rate of 20 m/s. If the catching process is completed in 0.1s,
the force of the blow exerted by the ball on the hand of the
æ 3ˆ 4 ˆö player is equal to [2006]
(
(c) 3iˆ - 4ˆj N ) (d) ç i - j ÷ N
è5 5 ø (a) 150 N (b) 3 N (c) 30 N (d) 300 N
15. A particle of mass 0.3 kg subject to a force F = – kx with
10. A particle of mass m is at rest at the origin at time
k = 15 N/m . What will be its initial acceleration if it is
t = 0. It is subjected to a force F(t) = F0e–bt in the x direction.
Its speed v(t) is depicted by which of the following released from a point 20 cm away from the origin ?[2005]
curves? [2012] (a) 15 m/s2 (b) 3 m/s2 (c) 10 m/s2 (d) 5 m/s2
16. A block is kept on a frictionless inclined surface with angle
F0 F0 of inclination ‘a’. The incline is given an acceleration ‘a’
mb mb to keep the block stationary. Then a is equal to [2005]
(a) v (t ) (b) v (t )
t t
F0 F0
mb mb a
a
(c) v (t ) (d) v (t )
(a) g cosec a (b) g / tan a
t t (c) g tan a (d) g
11. This question has Statement 1 and Statement 2. Of the 17. A rocket with a lift-off mass 3.5 × 104 kg is blasted upwards
four choices given after the Statements, choose the one with an initial acceleration of 10m/s2. Then the initial thrust
that best describes the two Statements. of the blast is [2003]
Statement 1: If you push on a cart being pulled by a horse
(a) 3.5 ´ 10 5 N (b) 7.0 ´ 10 5 N
so that it does not move, the cart pushes you back with an
(c) 14.0 ´ 10 5 N (d) 1.75 ´ 10 5 N
equal and opposite force.
18. Three forces start acting simultaneously on a particle
Statement 2: The cart does not move because the force r
described in statement 1 cancel each other. moving with velocity, v . These forces are represented
[Online May 26, 2012] in magnitude and direction by the three sides of a triangle
ABC. The particle will now move with velocity [2003]
P-38 Physics

C 2g
(a)
3

g R
(b) m
2
A B
r 5g
(a) less than v (c)
r 6
(b) greater than v
r (d) g m
(c) v in the direction of the largest force BC
(d) vr , remaining unchanged 24. Two blocks of mass M1 = 20 kg and M2 = 12 kg are
connected by a metal rod of mass 8 kg. The system is
19. A solid sphere, a hollow sphere and a ring are released
pulled vertically up by applying a force of 480 N as shown.
from top of an inclined plane (frictionless) so that they
The tension at the mid-point of the rod is :
slide down the plane. Then maximum acceleration down
the plane is for (no rolling) [2002] [Online April 22, 2013]
480 N
(a) solid sphere (b) hollow sphere (a) 144 N
(c) ring (d) all same M1

(b) 96 N
Motion of Connected Bodies,
TOPIC 2 Pulley & Equilibrium of (c) 240 N
Forces
20. A mass of 10 kg is suspended by a rope of length 4 m, from (d) 192 N M2
the ceiling. A force F is applied horizontally at the mid-
point of the rope such that the top half of the rope makes 25. A uniform sphere of weight W and radius 5 cm is being
an angle of 45° with the vertical. Then F equals: held by a string as shown in the figure. The tension in the
(Take g = 10 ms–2 and the rope to be massless) string will be : [Online April 9, 2013]
[7 Jan. 2020 II]
(a) 100 N (b) 90 N
(c) 70 N (d) 75 N 8 cm
21. An elevator in a building can carry a maximum of 10
persons, with the average mass of each person being 68
kg. The mass of the elevator itself is 920 kg and it moves
with a constant speed of 3 m/s. The frictional force
opposing the motion is 6000 N. If the elevator is moving
up with its full capacity, the power delivered by the motor
to the elevator (g =10 m/s2) must be at least: W W W W
(a) 12 (b) 5 (c) 13 (d) 13
[7 Jan. 2020 II] 5 12 5 12
(a) 56300 W (b) 62360 W 26. A spring is compressed between two blocks of masses m1
(c) 48000 W (d) 66000 W and m2 placed on a horizontal frictionless surface as shown
22. A mass of 10 kg is suspended vertically by a rope from in the figure. When the blocks are released, they have
initial velocity of v1 and v2 as shown. The blocks travel
the roof. When a horizontal force is applied on the rope
distances x1 and x2 respectively before coming to rest.
at some point, the rope deviated at an angle of 45°at the
roof point. If the suspended mass is at equilibrium, the æx ö
The ratio ç 1 ÷ is [Online May 12, 2012]
magnitude of the force applied is (g = 10 ms–2) è x2 ø
m1 m2
[9 Jan. 2019 II]
(a) 200 N (b) 140 N
v1 v2
(c) 70 N (d) 100 N
23. A mass ‘m’ is supported by a massless string wound around
a uniform hollow cylinder of mass m and radius R. If the
m2 m1 m2 m1
str ing does not slip on the cylinder, with what
(a) m1 (b) m2 (c) m1 (d) m2
acceleration will the mass fall or release? [2014]
Laws of Motion P-39

27. A block of mass m is connected to another block of mass 33. When forces F1, F2, F3 are acting on a particle of mass m
M by a spring (massless) of spring constant k. The block such that F2 and F3 are mutually perpendicular, then the
are kept on a smooth horizontal plane. Initially the blocks particle remains stationary. If the force F1 is now removed
are at rest and the spring is unstretched. Then a constant then the acceleration of the particle is [2002]
force F starts acting on the block of mass M to pull it. (a) F1/m (b) F2F3 /mF1
Find the force of the block of mass m. [2007] (c) (F2 - F3)/m (d) F2 /m.
MF mF 34. Two forces are such that the sum of their magnitudes is
(a) (b)
(m + M ) M 18 N and their resultant is 12 N which is perpendicular
mF to the smaller force. Then the magnitudes of the forces
(c) ( M + m) F (d)
m (m + M ) are [2002]
28. Two masses m1 = 5g and m2 = 4.8 kg tied to a string (a) 12 N, 6 N (b) 13 N, 5 N
are hanging over a light frictionless pulley. What is the (c) 10 N, 8 N (d) 16N, 2N.
acceleration of the masses when left free to move ? 35. A light string passing over a smooth light pulley connects
( g = 9.8m / s 2 ) [2004] two blocks of masses m1 and m2 (vertically). If the
acceleration of the system is g/8, then the ratio of the
masses is [2002]
(a) 8 : 1 (b) 9 : 7 (c) 4 : 3 (d) 5 : 3
36. Three identical blocks of masses m = 2 kg are drawn by a
force F = 10. 2 N with an acceleration of 0. 6 ms-2 on a
frictionless surface, then what is the tension (in N) in
the string between the blocks B and C? [2002]
C B A F
(a) 5 m/s2 (b) 9.8 m/s2
(c) 0.2 m/s2 (d) 4.8 m/s2 (a) 9.2 (b) 3.4 (c) 4 (d) 9.8
29. A spring balance is attached to the ceiling of a lift. A man 37. One end of a massless rope, which passes over a massless
hangs his bag on the spring and the spring reads 49 N, and frictionless pulley P is tied to a hook C while the other
when the lift is stationary. If the lift moves downward with end is free. Maximum tension that the rope can bear is 360
an acceleration of 5 m/s2, the reading of the spring balance N. With what value of maximum safe acceleration (in ms-2)
will be [2003] can a man of 60 kg climb on the rope? [2002]
(a) 24 N (b) 74 N (c) 15 N (d) 49 N P
30. A block of mass M is pulled along a horizontal frictionless
C
surface by a rope of mass m. If a force P is applied at the
free end of the rope, the force exerted by the rope on the
block is [2003]
(a) 16 (b) 6 (c) 4 (d) 8
Pm Pm PM
(a) (b) (c) P (d)
M +m M -m M +m
31. A light spring balance hangs from the hook of the other
TOPIC 3 Friction
light spring balance and a block of mass M kg hangs from
the former one. Then the true statement about the scale 38. An insect is at the bottom of a hemispherical ditch of
reading is [2003] radius 1 m. It crawls up the ditch but starts slipping
(a) both the scales read M kg each after it is at height h from the bottom. If the coefficient
(b) the scale of the lower one reads M kg and of the of friction between the ground and the insect is 0.75,
upper one zero then h is : (g = 10 ms–2) [Sep. 06, 2020 (I)]
(c) the reading of the two scales can be anything but the (a) 0.20 m (b) 0.45 m
sum of the reading will be M kg (c) 0.60 m (d) 0.80 m
(d) both the scales read M/2 kg each 39. A block starts moving up an inclined plane of inclination
32. A lift is moving down with acceleration a. A man in the lift 30° with an initial velocity of v0. It comes back to its
drops a ball inside the lift. The acceleration of the ball as v
observed by the man in the lift and a man standing initial position with velocity 0 . The value of the
2
stationary on the ground are respectively [2002] coefficient of kinetic friction between the block and the
(a) g, g (b) g – a, g – a I
(c) g – a, g (d) a, g inclined plane is close to . The nearest integer to I
1000
is _________. [NA Sep. 03, 2020 (II)]
P-40 Physics

40. A block of mass 5 kg is (i) pushed in case (A) and (ii) pulled 44. Two masses m1 = 5 kg and m2 = 10 kg, connected by an
in case (B), by a force F=20 N, making an angle of 30 o with inextensible string over a frictionless pulley, are moving
the horizontal, as shown in the figures. The coefficient of as shown in the figure. The coefficient of friction of
friction between the block and floor is m = 0.2. The horizontal surface is 0.15. The minimum weight m that
difference between the accelerations of the block, in case should be put on top of m2 to stop the motion is: [2018]
m T
(B) and case (A) will be : (g =10 ms–2) (a) 18.3 kg m
m2
2
[12 April 2019 II]
(b) 27.3 kg
T
(c) 43.3 kg
m1
(d) 10.3 kg
m1g
45. A given object takes n times more time to slide down a 45°
rough inclined plane as it takes to slide down a perfectly
(a) 0.4 ms–2 (b) 3.2 ms–2 smooth 45° incline. The coefficient of kinetic friction
(c) 0.8 ms –2
(d) 0 ms–2
between the object and the incline is :
41. Two blocks A and B masses mA=1 kg and mB = 3 kg are kept
on the table as shown in figure. The coefficient of friction [Online April 15, 2018]
between A and B is 0.2 and between B and the surface of 1 1
the table is also 0.2. The maximum force F that can be (a) 1- (b) 1 - 2
n2 n
applied on B horizontally, so that the block A does not
slide over the block B is : [Take g = 10 m/s2] 1 1
[10 April 2019 II] (c) 2 (d)
2-n 1 - n2
46. A body of mass 2kg slides down with an acceleration of
3m/s2 on a rough inclined plane having a slope of 30°.
The external force required to take the same body up the
(a) 8 N (b) 16 N (c) 40 N (d) 12 N plane with the same acceleration will be: (g = 10m/s2)
42. A block kept on a rough inclined plane, as shown in the [Online April 15, 2018]
figure, remains at rest upto a maximum force 2 N down (a) 4N (b) 14N (c) 6N (d) 20N
the inclined plane. The maximum external force up the 47. A rocket is fired vertically from the earth with an acceleration
inclined plane that does not move the block is 10 N. The of 2g, where g is the gravitational acceleration. On an
coefficient of static friction between the block and the inclined plane inside the rocket, making an angle q with
plane is : [Take g = 10 m/s2] [12 Jan. 2019 II] the horizontal, a point object of mass m is kept. The
10
N minimum coefficient of friction mmin between the mass and
the inclined surface such that the mass does not move is :
[Online April 9, 2016]
2N (a) tan2q (b) tanq
30°
(c) 3 tanq (d) 2 tan q
3 3 48. Given in the figure are two blocks A and B of weight 20 N
(a) (b)
2 4 and 100 N, respectively. These are being pressed against a
1 2 wall by a force F as shown. If the coefficient of friction
(c) (d) between the blocks is 0.1 and between block B and the
2 3
wall is 0.15, the frictional force applied by the wall on
43. A block of mass 10 kg is kept on a rough inclined plane as
block B is: [2015]
shown in the figure. A force of 3 N is applied on the block.
The coefficient of static friction between the plane and
the block is 0.6. What should be the minimum value of F
force P, such that the block doesnot move downward? A B
(take g = 10 ms–2) [9 Jan. 2019 I]
P

k g
10
N

45° (a) 120 N (b) 150 N


3

(a) 32 N (b) 18 N (c) 23 N (d) 25 N (c) 100 N (d) 80 N


Laws of Motion P-41

49. A block of mass m = 10 kg rests on a horizontal table. The A


coefficient of friction between the block and the table is
0.05. When hit by a bullet of mass 50 g moving with speed h
n, that gets embedded in it, the block moves and comes to
stop after moving a distance of 2 m on the table. If a freely
n
falling object were to acquire speed after being dropped
10
B L C
from height H, then neglecting energy losses and taking g
= 10 ms–2, the value of H is close to: 2h vo2 h 2
vo
(a) + (b) +
[Online April 10, 2015] m 2mg m 2mg
(a) 0.05 km (b) 0.02 km
h v2o h v2
(c) 0.03 km (d) 0.04 km (c) + (d) + o
50. A block of mass m is placed on a surface with a vertical 2m mg 2m 2mg

x3 54. A block A of mass 4 kg is placed on another block B of


cross section given by y = . If the coefficient of friction mass 5 kg, and the block B rests on a smooth horizontal
6
table. If the minimum force that can be applied on A so
is 0.5, the maximum height above the ground at which the
that both the blocks move together is 12 N, the maximum
block can be placed without slipping is: [2014]
force that can be applied to B for the blocks to move
1 2 1 1 together will be: [Online April 9, 2014]
(a) m (b) m (c) m (d) m
6 3 3 2 (a) 30 N (b) 25 N (c) 27 N (d) 48 N
51. Consider a cylinder of mass M resting on a rough horizontal 55. A block is placed on a rough horizontal plane. A time
rug that is pulled out from under it with acceleration ‘a’ dependent horizontal force F = kt acts on the block, where
perpendicular to the axis of the cylinder. What is Ffriction k is a positive constant. The acceleration - time graph of
at point P? It is assumed that the cylinder does not slip. the block is : [Online April 25, 2013]
[Online April 19, 2014] a a
w
v
O
(a) (b)
P
O t O t
a
a a

F friction
Ma Ma (c) (d)
(a) Mg (b) Ma (c) (d)
2 3
52. A heavy box is to dragged along a rough horizontal floor. O t O t
To do so, person A pushes it at an angle 30° from the 56. A body starts from rest on a long inclined plane of slope
horizontal and requires a minimum force FA, while person 45°. The coefficient of friction between the body and
B pulls the box at an angle 60° from the horizontal and the plane varies as m = 0.3 x, where x is distance travelled
needs minimum force FB. If the coefficient of friction down the plane. The body will have maximum speed
3 FA (for g = 10 m/s2) when x = [Online April 22, 2013]
between the box and the floor is , the ratio is (a) 9.8 m (b) 27 m (c) 12 m (d) 3.33 m
5 FB
[Online April 19, 2014] 57. A block of weight W rests on a horizontal floor with
coefficient of static friction m. It is desired to make the
5
(a) (b) block move by applying minimum amount of force. The
3 3 angle q from the horizontal at which the force should be
3 2 applied and magnitude of the force F are respectively.
(c) (d) [Online May 19, 2012]
2 3
53. A small ball of mass m starts at a point A with speed vo mW
(a) q = tan -1 ( m) , F =
and moves along a frictionless track AB as shown. The
1 + m2
track BC has coefficient of friction m. The ball comes to
stop at C after travelling a distance L which is: æ 1ö mW
[Online April 11, 2014] (b) q = tan -1 ç ÷ , F =
è mø 1 + m2
P-42 Physics

(c) q = 0, F = mW 64. A block rests on a rough inclined plane making an angle of


30° with the horizontal. The coefficient of static friction
æ m ö mW
(d) q = tan -1 ç ÷ ,F = between the block and the plane is 0.8. If the frictional
è 1 + mø 1+ m force on the block is 10 N, the mass of the block (in kg) is
2
58. An insect crawls up a hemispherical surface very slowly. (take g = 10 m / s ) [2004]
The coefficient of friction between the insect and the (a) 1.6 (b) 4.0 (c) 2.0 (d) 2.5
surface is 1/3. If the line joining the centre of the 65. A horizontal force of 10 N is necessary to just hold a block
hemispherical surface to the insect makes an angle a with stationary against a wall. The coefficient of friction between
the vertical, the maximum possible value of a so that the the block and the wall is 0.2. The weight of the block is
insect does not slip is given by [Online May 12, 2012] [2003]

a
10N
(a) cot a = 3 (b) sec a = 3
(c) cosec a = 3 (d) cos a = 3
59. The minimum force required to start pushing a body up (a) 20 N (b) 50 N (c) 100 N (d) 2 N
rough (frictional coefficient m) inclined plane is F1 while 66. A marble block of mass 2 kg lying on ice when given a
the minimum force needed to prevent it from sliding down velocity of 6 m/s is stopped by friction in 10 s. Then the
is F2. If the inclined plane makes an angle q from the coefficient of friction is [2003]
F1 (a) 0.02 (b) 0.03 (c) 0.04 (d) 0.06
horizontal such that tan q = 2m then the ratio is
F2
Circular Motion, Banking of
(a) 1 (b) 2 [2011 RS] TOPIC 4
(c) 3 (d) 4 Road
60. If a spring of stiffness ‘k’ is cut into parts ‘A’ and ‘B’ of 67. A disc rotates about its axis of symmetry in a hoizontal
length l A : l B = 2 : 3, then the stiffness of spring ‘A’ is plane at a steady rate of 3.5 revolutions per second. A coin
given by [2011 RS] placed at a distance of 1.25cm from the axis of rotation
remains at rest on the disc. The coefficient of friction
3k 2k between the coin and the disc is (g = 10m/s2)
(a) (b)
5 5 [Online April 15, 2018]
5k (a) 0.5 (b) 0.7 (c) 0.3 (d) 0.6
(c) k (d) 68. A conical pendulum of length 1 m makes an angle q = 45°
2
w.r.t. Z-axis and moves in a circle in the XY plane.The
61. A smooth block is released at rest on a 45° incline and then
radius of the circle is 0.4 m and its centre is vertically be-
slides a distance ‘d’. The time taken to slide is ‘n’ times as
low O. The speed of the pendulum, in its circular path, will
much to slide on rough incline than on a smooth incline.
be :
The coefficient of friction is [2005]
(Take g = 10 ms–2) Z
[Online April 9, 2017]
1 1 (a) 0.4 m/s O
(a) mk = 1 – 2 (b) mk = 1- q
n n2 (b) 4 m/s
1 1 (c) 0.2 m/s
(c) ms = 1 - (d) ms = 1- C
n 2
n2
(d) 2 m/s
62. The upper half of an inclined plane with inclination f is 69. A particle is released on a vertical smooth semicircular
perfectly smooth while the lower half is rough. A body track from point X so that OX makes angle q from the
starting from rest at the top will again come to rest at the vertical (see figure). The normal reaction of the track on
bottom if the coefficient of friction for the lower half is the particle vanishes at point Y where OY makes angle f
given by [2005] with the horizontal. Then: [Online April 19, 2014]
(a) 2 cos f (b) 2 sin f (c) tan f (d) 2 tan f X
63. Consider a car moving on a straight road with a speed of Y
100 m/s . The distance at which car can be stopped is q
[ m k = 0.5 ] [2005]
(a) 1000 m (b) 800 m (c) 400 m (d) 100 m f

O
Laws of Motion P-43

1 n2 n2
(a) sin f = cos f (b) sin f = cos q (a) - cos q iˆ + sin q ˆj
2 R R
2 3
(c) sin f = cos q (d) sin f = cos q n2 n2
3 4 (b) - sin q iˆ + cos q ˆj
70. A body of mass ‘m’ is tied to one end of a spring and R R
whirled round in a horizontal plane with a constant angular n2 n2
(c) - cos q iˆ - sin q ˆj
velocity. The elongation in the spring is 1 cm. If the R R
angular velocity is doubled, the elongation in the spring n2 ˆ n2 ˆ
is 5 cm. The original length of the spring is : (d) i+ j
R R
[Online April 23, 2013] 73. An annular ring with inner and outer radii R1 and R2 is
(a) 15 cm (b) 12 cm rolling without slipping with a uniform angular speed. The
(c) 16 cm (d) 10 cm ratio of the forces experienced by the two particles situated
71. A point P moves in counter-clockwise direction on a F1
on the inner and outer parts of the ring , F is [2005]
circular path as shown in the figure. The movement of 'P' is 2
such that it sweeps out a length s = t3 + 5, where s is in 2 R2
æ R1 ö
metres and t is in seconds. The radius of the path is 20 m. (a) çè R ÷ø (b) R1
The acceleration of 'P' when t = 2 s is nearly. [2010] 2
y R1
(c) R2 (d) 1
B 74. Which of the following statements is FALSE for a particle
P(x,y) moving in a circle with a constant angular speed ? [2004]
(a) The acceleration vector points to the centre of the
m circle
20
(b) The acceleration vector is tangent to the circle
x (c) The velocity vector is tangent to the circle
O A (d) The velocity and acceleration vectors are
(a) 13m/s2 (b) 12 m/s2 perpendicular to each other.
(c) 7.2 ms2 (d) 14m/s2 75. The minimum velocity (in ms-1) with which a car driver must
72. For a particle in uniform circular motion, the acceleration traverse a flat curve of radius 150 m and coefficient of friction
r 0.6 to avoid skidding is [2002]
a at a point P(R,q) on the circle of radius R is (Here q is (a) 60 (b) 30
measured from the x-axis) [2010] (c) 15 (d) 25
P-44 Physics

1. (a) Given
r 1 é ku 2 + g ù
F = k ( v y iˆ + vx ˆj ) Þ ln ê ú= h
2k ë g û
\ Fx = kv y iˆ, Fy = kv x ˆj 4. (a) Net acceleration
mdvx dv k dv
= kv y Þ x = v y = a = – (g + gv2)
dt dt m dt
dv y k Let time t required to rise to its zenith (v = 0) so,
Similarly, = vx
dt m 0 t
-dv
dv y vx
= Þ ò v y dvy = ò vx dvx
ò g + gv2
= ò dt [for Hmax, v = 0]
v0 0
dvx v y
1 æ g v0 ö
v 2y = v x2 + C \t = tan -1 ç ÷
gg è g ø
v 2y - v x2 = constant
5. (d) v2 = u2 – 2gh
r r k
v ´ a = (v x iˆ + v y ˆj ) ´ (v y iˆ + vx ˆj ) or v = u 2 - 2 gh
m
2ˆ k k
= (vx k - v y k ) = (vx2 - v 2y ) kˆ = constant

Momentum, P = mv = m u 2 - 2 gh
m m
2. (b) From the Newton's second law, u2
At h = 0, P = mu and at h = ,P = 0
dp d (mv) æ dm ö f
F= = = vç ÷ ...(i) upward direction is positive and downward direction
dt dt è dt ø
is negative.
dM (t ) 6. (b) From Newton’s second law
We have given, = bv 2 (t ) ...(ii)
dt
dp
Thrust on the satellite, = F = kt
dt
æ dm ö 2 3
F = -v ç ÷ = -v (bv ) = -bv [Using (i) and (ii)] Integrating both sides we get,
è dt ø T
3p T é t2 ù
-bv3 òp dp = ò kt dt Þ [ p ]
3p
=k ê ú
Þ F = M (t ) a = -bv3 Þ a = 0 p
êë 2 úû 0
M (t )
3. (d) v = 0 kT 2 p
2
(g+kv ) = a (acceleration) Þ 2p = ÞT =2
H 2 k
u R dv R
7. (a) From F < 2 v(t) Þ m < 2 v(t)
t dt t
r dv Rdt
F = mkv 2 - mg (Q mg and mkv2 act Integrating both sides ò <ò
opposite to each other) dt mt 2
r R
r F In v < ,
a= = -[ kv 2 + g ] mt
m
1
dv æ dv ö [ ln v ×
Þ v× = -[kv 2 + g ] çèQ a = v ÷ø t
dh dh 8. (b) Space between the supports for motion of beads is
0 h
v × dv L–2nr
Þò = ò dh Average force experienced by each support,
u kv 2 + g 0
2mV mV 2
1 0 F= =
Þ ln éëkv 2 + g ùû = - h 2( L – 2nr ) L – 2 nr
2k u
V
Laws of Motion P-45

mv Þ 40 - 0 = 2 (a) 0.2 Þ a = 100 m/s2


L–2nr
\ F = ma = 0.2 ´ 100 = 20 N
mv Þ N - mg = 20 Þ N = 20 + 2 = 22 N
Note :
9. (a) From question, Whand + Wgravity = DK
Mass of body, m = 5 kg Þ F (0.2) + (0.2)(10)(2.2) = 0 Þ F = 22 N
Velocity at t = 0, 14. (c) Given, mass of cricket ball, m = 150 g = 0.15 kg
u = (6iˆ - 2 ˆj) m/s Initial velocity, u = 20 m/s
Velocity at t = 10s, Force,
m(v - u ) 0.15(0 - 20)
v = + 6 ĵ m/s F= = = 30 N
t 0.1
Force, F = ? 15. (c) Mass (m) = 0.3 kg
v -u Force, F = m.a = –kx
Acceleration, a =
t Þ ma = –15x
6 ˆj - (6iˆ - 2 ˆj ) -3iˆ + 4 ˆj Þ 0.3a = –15x
= = m/s2 15 -150
10 5 x= x = - 50 x
Þ a= –
Force, F = ma 0.3 3
( -3iˆ + 4 ˆj ) a = –50 × 0.2 = 10m/s2
= 5´ = ( -3iˆ + 4 ˆj ) N 16. (c) When the incline is given an acceleration a towards
5
10. (c) Given that F(t) = F0e–bt the right, the block receives a reaction ma towards left.
ma
g cos
dv N
Þ m = F0e–bt
dt a a
dv F0 -bt a a
= e mg cosa
dt m + ma sina mg mg sin a
v t
F0 -bt For block to remain stationary, Net force along the incline
ò dv = mò
e dt should be zero.
0 0
mg sin a = ma cos a Þ a = g tan a
t
F é e - bt ù 17. (b) In the absence of air resistance, if
F0 é
( )
Thrust (F)
v= 0ê ú = - e - bt - e -0 ù the rocket moves up with an acceleration
m ëê -b ûú mb ë û
0 a, then thrust
F0 é
Þ v= 1 - e -bt ù F = mg + ma
mb ë û
a
11. (a) According to newton third law of motion i.e. every \ F = m ( g + a) = 3.5 × 104 ( 10 + 10)
action is associated with equal and opposite reaction.
12. (d) mg sin q = ma = 7 × 105 N mg
\ a = g sin q 18. (d) Resultant force is zero, as three forces are represented
\ Vertical component of acceleration by the sides of a triangle taken in the same order. From
= g sin2 q r r
Newton’s second law, Fnet = ma.
\ Relative vertical acceleration of A with respect to B is
Therefore, acceleration is also zero i.e., velocity remains
g (sin 2 60 - sin 2 30] unchanged.
æ3 1ö g 19. (d) This is a case of sliding (if plane is friction less) and
= g ç – ÷ = = 4.9 m/s2
è4 4ø 2 therefore the acceleration of all the bodies is same.
in vertical direction 20. (a) From the free body diagram
13. (d) For the motion of ball, just after the throwing
v = 0, s = 2m, a = –g = –10ms–2
v2 – u2 = 2as for upward journey
Þ -u 2 = 2( -10) ´ 2 Þ u 2 = 40
When the ball is in the hands of the thrower
u = 0, v = 40 ms–1
s = 0.2 m
v2 – u2 = 2as
P-46 Physics

T cos45° = 100 N ...(i) 480


T sin45° = F ...(ii) = = 12 ms -2
20 + 12 + 8
On dividing (i) by (ii) we get Tension at the mid point
T cos 45° 100
= æ Mass of rod ö
T sin 45° F T = ç M2 + ÷a
è 2 ø
Þ F = 100 N
= (12 + 4) × 12 = 192 N
21. (d) Net force on the elevator = force on elevator
25. (d) P
+ frictional force
Þ F = (10 m + M)g + f q
8 cm
where, m = mass of person, M = mass of elevator, T
f = frictional force
Þ F = (10 × 68 + 920) × 9.8 + 600 Q 5 cm O
Þ F = 22000 N
Þ P = FV = 22000 × 3 = 66000 W
w wcosq
22. (d)
PQ = OP 2 + OQ2
o
45
= 132 + 52 = 12
Tension in the string
13
F T = w cos q = W
45
o
12
26. (a)
100 N 27. (d) Writing free body-diagrams for m & M,
At equilibrium, M
m
mg 100 K
tan 45° = = F
F F
N N
\ F = 100 N
a
23. (b) From figure,
T T M
m F

a mg Mg
R we get T = ma and F – T = Ma
where T is force due to spring
T Þ F – ma = Ma or,, F = Ma + ma
\ Acceleration of the system
T
F
m a a= .
mg M +m
Now, force acting on the block of mass m is
Acceleration a = Ra …(i)
and mg – T = ma …(ii) æ F ö = mF
ma = m ç .
From equation (i) and (ii) è M + m ÷ø m + M
æ aö If a is the acceleration along the inclined plane, then
T × R = mR2a = mR2 çè ÷ø 28. (c) Here, m1 = 5kg and m2 = 4.8 kg.
R
If a is the acceleration of the masses,
or T = ma m1a = m1g – T ...(i)
Þ mg – ma = ma m2a = T – m2g ...(ii)
g Solving (i) and (ii) we get
Þ a=
2 æ m - m2 ö
24. (d) Acceleration produced in upward direction a=ç 1 ÷g
è m1 + m2 ø
F
a= (5 - 4.8) ´ 9.8
M1 + M 2 + Mass of metal rod Þa= m / s2 = 0.2 m/s2
(5 + 4.8)
Laws of Motion P-47

29. (a) When lift is stationary, W1 = mg ...(i) 34. (b) Let the two forces be F1 and F2 and let F2 < F1. R is
When the lift descends with acceleration, a the resultant force.
W2 = m(g – a) Given F1 + F2 = 18 ...(i)
49 From the figure F22 + R 2 = F12
W2 = (10 – 5) = 24.5 N
10
F12 - F22 = R 2
\ F12 - F22 = 144 ...(ii)
Only option (b) follows equation (i) and (ii).
T F1
a
mg R
F2
30. (d) Taking the rope and the block as a system
a F1
M m 35. (b) For mass m1
T P m1g – T = m1a ...(i)
For mass m2
we get P = (m + M)a T–m2g = m2a ...(ii)
P
\ Acceleration produced, a =
m+M
Taking the block as a system,
Force on the block, F = Ma
MP T
\ F= T
m+M a m2
31. (a) The Earth exerts a pulling force Mg. The block in turn m1 a
exerts a reaction force Mg on the spring of spring balance m2g
S1 which therefore shows a reading of M kgf. m1g
As both the springs are massless. Therefore, it exerts a Adding the equations we get
force of Mg on the spring of spring balance S2 which (m1 - m2 ) g
a=
shows the reading of M kgf. m1 + m2
g
s2 Mkgf Here a =
8
m1
-1
s1 Mkgf 1 m2 m m m 9
\ = Þ 1 +1 = 8 1 - 8 Þ 1 =
8 m 1 +1 m 2 m2 m2 7
M m2
Mg
36. (b) Force = mass × acceleration
32. (c) Case - I: For the man standing in the lift, the \ F = (m + m + m) × a
acceleration of the ball F = 3m × a
r r r
abm = ab - am Þ abm = g – a F
Case - II: The man standing on the ground, the acceleration a=
3m
of the ball
r r r \a =
10.2
m / s2
abm = ab - am Þ abm = g – 0 = g
6
33. (a) When forces F1, F2 and F3 are acting on the particle,
10.2
it remains in equilibrium. Force F2 and F3 are perpendicular \ T2 = ma = 2 ´ = 3.4N
to each other, 6
2 kg 2 kg 2 kg
F1 = F2 + F3 F
T2
C B A
\ F1 = F22 + F32 T2 T1 T1

The force F1 is now removed, so, resultant of F2 and F3 37. (c) Tension, T = 360 N
will now make the particle move with force equal to F1. Mass of a man m = 60 kg
F mg – T = ma
Then, acceleration, a = 1
m
P-48 Physics

T g mg 3 æg m 3ö
\ a=g- Þ + = 4ç - ÷
m 2 2 ç2 2 ÷ø
è
360
= 10 - = 4m / s 2 Þ 5 + 5 3m = 4(5 - 5 3m) (Substituting, g = 10 m/
60
s2)
38. (a) For balancing, mg sin q = f = mmg cos q
Þ 5 + 5 3m = 20 - 20 3m Þ 25 3m = 15
3
Þ tan q = m = = 0.75 3 346
4 Þm= = 0.346 =
5 1000
f = mgcosq
Rcosq q R I 346
So, =
1000 1000
40. (c) A : N = 5g + 20 sin30°
h h q 1
= 50 + 20 × = 60 N
2
mgsinq N

q R 20 cos 30°
Rcosq 5 f
3
h q 5g
4
20 sin 30°
æ 4ö R F-f 20 cos 30° - µN
h = R - R cos q = R - R ç ÷ = =
è 5ø 5 Accelaration, a1 =
m 5
R
\h = = 0.2 m [Q radius, R = 1m] é ù
5 3
39. (346) ê 20 ´ - 0.2 ´ 60 ú
2
Acceleration of block while moving up an inclined plane, = ê ú = 1.06 m/s2
ê 5 ú
a1 = g sin q + mg cos q êë úû
Þ a1 = g sin 30° + mg cos30°
N 20 sin 30°
g mg 3
= + ..(i) (Q q = 30o)
2 2
20 cos 30°
Using v 2 - u 2 = 2a( s )
f
Þ v02 - 02 = 2a1 ( s) (Q u = 0)
Þ v02 - 2a1 ( s ) = 0 5g
v02
Þs= ...(ii) B : N = 5g – 20 sin 30°
a1
Acceleration while moving down an inclined plane 1
= 50 – 20 × = 40 N
a2 = g sin q - mg cos q 2

Þ a2 = g sin 30° - mg cos 30° F - f é 20cos 30° - 0.2 ´ 40 ù


a2 = =ê ú = 1.86 m/s2
m ë 5 û
g m 3
Þ a2 = - g ...(iii) Now a2 – a1 = 1.86 – 1.06 = 0.8 m/s 2
2 2
41. (b) Taking (A + B) as system
Using again v 2 - u 2 = 2as for downward motion
F – m(M + m)g
2
æv ö v2 = (M + m)a
Þ ç 0 ÷ = 2a2 ( s ) Þ s = 0 ...(iv)
è 2ø 4a2
F – m ( M + m) g
Equating equation (ii) and (iv) Þa= ( M + m)
v02 v2
= 0 Þ a1 = 4a2 F - (0.2)4 ´10 æ F - 8 ö
a1 4a2 a= =ç ÷ ...(i)
4 è 4 ø
Laws of Motion P-49

But, amax = mg = 0.2 × 10 = 2 45. (b) The coefficients of kinetic friction between the object
and the incline
F -8
\ =2
4 æ 1ö 1
m = tan q ç1 - 2 ÷ Þ m = 1 - 2 (Q q = 45°)
Þ F = 16 N è n ø n
46. (d) Equation of motion when the mass slides down
42. (a) From figure, 2 + mg sin 30° = mmg cos 30° and Mg sin q – f = Ma
10 = mg sin 30° + m mg cos 30° Þ 10 – f = 6 (M = 2 kg, a = 3 m/s2, q = 30° given)
A
= 2mmg cos 30° – 2 \ f = 4N f
Þ 6 = mmg cos 30° and kg
4 = mg cos 30° Equation of motion when the block is 2
By dividing above two pushed up Ma
q 30°
3
Let the external force required to take C B
Þ =m´ 3 the block up the plane with same
2 acceleration be F Ma A
f
3 F – Mg sin q – f = Ma g
\ Coefficient of friction, m = k
2
2
Þ F – 10 – 4 = 6 F 30°
q
F = 20 N C B
P
47. (b) Let m be the minimum coefficient of friction
P m3mgcosq
g

µmgcos?
k

µ = 0.6
10

43. (a)
3 + mgsing?
N

45°
3

3mgsinq 3g

100
mg sin 45° = = 50 2
2
At equilibrium, mass does not move so,
[Qm = 10kg, g = 9.8 m s -2
] 3mg sinq = m3mg cosq
[ μ min < tan θ
1
mmg cosq = 0.6 × mg × = 0.6 ´ 50 2
2 f1 f2
3 + mg sinq = P + mmg cosq
F N
3 + 50 2 = P + 30 2 48. (a) A B
\ P = 31.28 = 32 N
44. (b) Given : m1 = 5kg; m2 = 10kg; m = 0.15 f1
20N 100N
FBD for m1, m1g – T = m1a
Þ 50 – T = 5 × a Assuming both the blocks are stationary
and, T – 0.15 (m + 10)g = (10 + m)a N= F
For rest a = 0
f1 = 20 N
or, 50 = 0.15 (m + 10) 10
f2 = 100 + 20 = 120N
N
m f
m2 T
m(m+m2)g (m+m2)g T
m1

m1g = 50N

3
Þ 5= (m + 10) 120N
20
Considering the two blocks as one system and due to
100 equilibrium f = 120N
= m + 10 \ m = 23.3kg; close to option (b)
3
P-50 Physics

49. (d) f = µ(M + m) g R


52. (d) F (Push)
f µ( M + m) g A q = 60°
a= = = µg
M +m ( M + m) 30°
= 0.05 × 10 = 0.5 ms–2
Initial momentum 0.05V f
V0 = =
(M + m) 10.05 mg
m = 50g M = 10 kg mmg
FA =
sin q - m cos q
n
V0 Similarly,
mmg
FB =
sin q + m cos q
mmg
FA sin q - m cos q
\ =
FB mmg
sin q + m cos q
v2 – u2 = 2as
0 – u2 = 2as mmg é 3 ù
u2 = 2as = êm = given ú
3 ë 5 û
2 sin 60° - cos 60°
æ 0.05v ö
ç ÷ = 2 ´ 0.5 ´ 2 5
è 10.05 ø mmg
Solving we get v = 201 2 3
sin 30° + cos 30°
Object falling from height H. 5
V 3
= 2 gH sin 30° + cos30°
10 5
201 2 =
= 2 ´10 ´ H 3
sin 60° - cos 60°
10 5
H = 40 m = 0.04 km
50. (a) At limiting equilibrium, 1 3 3
+ ´
2 5 2
m = tan q =
3 3 1
m - ´
y 2 5 2
q
1æ 3ö 1 8
ç1 + ÷ ´
2è 5ø
= = 2 5
3æ 1ö 3´4
ç1 - ÷
dy x 2 5 è 5ø 10
tanq = m = = (from question)
dx 2 8
Q Coefficient of friction m = 0.5 = 10 =
8
=
2
x2 3´4 3´4 3
\ 0.5 = 10
2
Þ x = ±1 53. (b) Initial speed at point A, u = v0
Speed at point B, v = ?
x3 1 v2 – u2 = 2gh
Now, y = = m
6 6 v2 = v20 + 2gh
51. (d) Force of friction at point P, Let ball travels distance ‘S’ before coming to rest
1 v2 v 2 + 2 gh
Ffriction = Ma sin q S= = 0
3 2mg 2mg
1
= Ma sin 90° [ here q = 90°] v02 2 gh h v2
3 = + = + 0
Ma 2mg 2mg m 2mg
=
3
Laws of Motion P-51

54. (c) Minimum force on A 1


= frictional force between the surfaces cos q =
= 12 N 1 + m2
Hence, Fmin
A 4 kg
mw mw
5 kg = =
B 1 m 2
1 + m2
+
Smooth table 1 + m2 1 + m2
58. (a) O r

F1 a
R y Bowl
Therefore maximum acceleration
A h
12N
amax = = 3m / s 2
4kg mg cos a
mg sin a
Hence maximum force, mg
Fmax = total mass × amax The insect crawls up the bowl upto a certain height h only
= 9 × 3 = 27 N till the component of its weight along the bowl is balanced
55. (b) Graph (b) correctly dipicts the acceleration-time by limiting frictional force.
graph of the block. For limiting condition at point A
56. (d) When the body has maximum speed then R = mg cosa ...(i)
m = 0.3x = tan 45° F1 = mg sina ...(ii)
\ x = 3.33 m Dividing eq. (ii) by (i)
57. (a) Let the force F is applied at an angle q with the 1 F
horizontal. tan a = = 1 = m [ As F1 = mR ]
cot a R
R
1é 1 ù
Þ tan a = m = êQ m = ( Given ) ú
F sinq F 3ë 3 û
F = µR
\ cot a = 3
F cosq 59. (c) N1 F1

w mg sin q
f1 mg cos q
For horizontal equilibrium, q mg
F cos q = µR ...(i)
For vertical equilibrium, N2
2
F

R + F sin q = mg f2
or, R = mg – F sinq ...(ii)
Substituting this value of R in eq. (i), we get
F cosq = µ (mg – F sinq) mg sin q
mg cos q
= µ mg – µ Fsinq q mg
or, F (cosq + µsinq) = µmg
When the body slides up the inclined plane, then
µmg mg sin q + f1 = F1
or, F = ...(iii)
cosq + m sinq or, F1 = mg sin q + mmg cos q
For F to be minimum, the denominator (cosq + µ sinq) When the body slides down the inclined plane, then
should be maximum. mg sin q – f 2 = F2
d
\ (cosq + m sinq ) = 0 or F2 = mg sin q – mmg cos q
dq F1 sin q + m cos q
or, – sinq + µ cosq = 0 \ =
or, tanq = µ F2 sin q - m cos q
or, q = tan–1(µ) F1 tan q + m 2m + m 3m
m Þ F = tan q - m = 2m - m = m = 3
2
Then, sinq = and
1 + m2
P-52 Physics

60. (d) It is given lA : lB = 2 : 3 g sin f = -( g sin f - mg cos f)


2l æ 3l ö Þ m = 2 tan f
lA = , lB = ç ÷
5 è 5ø NOTE
1 According to work-energy theorem, W = DK = 0
\ We know that k µ (Since initial and final speeds are zero)
l
If initial spring constant is k, then \ Workdone by friction + Work done by gravity = 0
l
k l = k Al A = k B l B i.e., -( µ mg cos f ) + mg l sin f = 0
2
æ 2l ö µ
kl = kA ç ÷ or cos f = sin f or µ = 2 tan f
è 5ø 2
5k 63. (a) Given, initial velocity, u = 100m/s.
kA = Final velocity, v = 0.
2 Acceleration, a = mkg = 0.5 × 10
61. (b) a = g sin q - mg cos q
v2 – u2 = 2as or
d q Þ 02 – u2 = 2(–mkg)s
g sin d 1
=
a 45° 45° Þ -1002 = 2 ´ - ´ 10 ´ s
2
smooth rough Þ s = 1000 m
On smooth inclined plane, acceleration of the body = g
sin q. Let d be the distance travelled 64. (c) fs

1 N sq
\ d = ( g sin q)t12 , mg
co
2 30
°
mg
in
2d gs
t1 = , m 30°
g sin q
Since the body is at rest on the inclined plane,
On rough inclined plane,
mg sin 30° = Force of friction
mg sin q – mR
a= Þ m ´ 10 ´ sin 30° = 10
m Þ m ´ 5 = 10 Þ m = 2.0 kg
mg sin q – mmg cos q 65. (d) Horizontal force, N = 10 N.
Þ a=
m Coefficient of friction m = 0.2.
Þ a = g sin q – mkg cos q f = mN
1 ˆ cos q) t 2
\ d = ( g sin q - mkg 2
2
2d 10N 10N 10N
t2 =
ˆ cos q
g sin q - mkg
According to question, t2 = nt1 W
The block will be stationary so long as
2d 2d Force of friction = weight of block
n = ˆ cos q \ mN = W
g sin q g sin q - mkg
Þ 0.2 × 10 = W
Here, m is coefficient of kinetic friction as the block Þ W = 2N
moves over the inclined plane. 66. (d) u = 6 m/s, v = 0, t = 10s, a = ?
\ sin q = (sin q – mk̂ cos q)n2 v–u
Acceleration a =
1 1 t
2
Þ n= Þ n = 0–6
1 - mk 1 - mk Þ a=
10
1 -6
Þ mk = 1 - Þ a= = -0.6m / s2
n2 10
62. (d) For first half
mg
acceleration = g sin f;
For second half f = mN N
acceleration = – ( g sin f - mg cos f) The retardation is due to the frictional force
For the block to come to rest at the bottom, acceleration \ f = ma Þ mN = ma
in I half = retardation in II half.
Laws of Motion P-53

ma At t = 2s, at = 6 ´ 2 = 12 m/s2
Þ mmg = ma Þ m=
mg
9 ´ 16
a 0.6 ac = = 7.2 m/s2
Þm= = = 0.06 20
g 10 \ Resultant acceleration
mv 2 = at2 + ac2
67. (d) Using, mmg = = mrw 2
r
w = 2pn = 2p × 3.5 = 7p rad/sec = (12) 2 + (7.2) 2 = 144 + 51.84
Radius, r = 1.25 cm = 1.25 × 10–2 m = 195.84 = 14 m/s2
Coefficient of friction, µ = ? r
72. (c ) Clearly a = ac cos q(-iˆ) +ac sin q(- ˆj )
m( rw ) 2
mmg = (Q v = rw)
r -v 2 v2
= cos q iˆ - sin q ˆj
R R
Y

P( R, q)
O
1.25 cm
R
Disc q
X
O

µmg = mrw2
2 73. (c)
æ 22 ö
1.25 ´ 10 -2 ´ ç 7 ´ ÷
2
a2
R2
rw è 7ø
Þ m= = v 2 = wR 2
g 10 R1
a1 v1 = wR 1
1.25 ´ 10-2 ´ 222
= = 0.6
10
68. (d) Given, q = 45°, r = 0.4 m, g = 10 m/s2
Let m is the mass of each particle and w is the angular
mv 2
T sin q = ...... (i) speed of the annular ring.
r
T cos q = mg ...... (ii) v 2 w 2 R12
a1 = 1 = = w 2 R1
From equation (i) & (ii) we have, R1 R1
v2 v2
tan q = a2 = 2 = w 2 R2
rg R2
Taking particle masses equal
T q
F1 ma1 mR1w2 R1
= = =
F2 ma2 mR2w2 R2
NOTE :
The force experienced by any particle is only along radial
direction.
v2 = rg Q q = 45° Force experienced by the particle, F = mw2R
Hence, speed of the pendulum in its circular path,
F1 R1
v = rg = 0.4 ´ 10 = 2 m/s \ =
F2 R2
69. (c) 70. (a)
74. (b) Only option (b) is false since acceleration vector is
71. (d) s = t3 + 5 always radial (i.e. towards the center) for uniform circular
ds motion.
Þ velocity, v = = 3t 2
dt 75. (b) The maximum velocity of the car is
dv vmax = mrg
Tangential acceleration at = = 6t
dt Here m = 0.6, r = 150 m, g = 9.8
v 2 9t 4 vmax = 0.6 ≥150 ≥9.8 ; 30m / s
Radial acceleration ac = =
R R
5
P-54 Physics

Work, Energy and


Power
1 3
TOPIC 1 Work (a) 2J (b)J (c) 1J (d) J
2 2
4. A block of mass m is kept on a platform which starts
1. A person pushes a box on a rough horizontal platform from rest with constant acceleration g/2 upward, as
surface. He applies a force of 200 N over a distance of shown in fig. work done by normal reaction on block in
15 m. Thereafter, he gets progressively tired and his applied time t is: [10 Jan. 2019 I]
force reduces linearly with distance to 100 N. The total
distance through which the box has been moved is 30 m.
What is the work done by the person during the total
movement of the box ? [4 Sep. 2020 (II)]
(a) 3280 J (b) 2780 J m g2 t 2 m g2 t2
(a) - (b)
(c) 5690 J (d) 5250 J 8 8
2. B 3m g 2 t 2
C (c) 0 (d)
8
A q 5. A body of mass starts moving from rest along x-axis so
A small block starts slipping down from a point B on an that its velocity varies as v = a s where a is a constant s
inclined plane AB, which is making an angle q with the and is the distance covered by the body. The total work
horizontal section BC is smooth and the remaining section done by all the forces acting on the body in the first second
CA is rough with a coefficient of friction m. It is found that after the start of the motion is: [Online April 16, 2018]
the block comes to rest as it reaches the bottom (point A) 1 4 2
(a) ma t (b) 4ma 4 t 2
of the inclined plane. If BC = 2AC, the coefficient of friction 8
is given by m = k tanq. The value of k is _________. 1
(c) 8ma 4 t 2 (d) ma 4 t 2
4
[NA 2 Sep. 2020 (I)] 6. When a rubber-band is stretched by a distance x, it exerts
r restoring force of magnitude F = ax + bx2 where a and b are
3. Consider a force F = - xiˆ + yjˆ . The work done by this
constants. The work done in stretching the unstretched
force in moving a particle from point A(1, 0) to B(0, 1) rubber-band by L is: [2014]

( )
along the line segment is: (all quantities are in SI units) 1
(a) aL2 + bL3 (b) aL2 + bL3
[9 Jan. 2020 I] 2

aL2 bL3 1 æ aL2 bL3 ö


(c) + (d) 2 çç 2 + 3 ÷÷
2 3 è ø
7. A uniform chain of length 2 m is kept on a table such that
a length of 60 cm hangs freely from the edge of the table.
The total mass of the chain is 4 kg. What is the work done
in pulling the entire chain on the table ? [2004]
(a) 12 J (b) 3.6 J (c) 7.2 J (d) 1200 J
Work, Energy & Power P-55
r r r r
8. A force F = (5i + 3 j + 2k ) N is applied over a particle 14. A spring whose unstretched length is l has a force
which displaces it from its origin to the point constant k. The spring is cut into two pieces of
r r r unstretched lengths 11 and l2 where, l1 = nl2 and n is an
r = (2i - j )m. The work done on the particle in joules is
integer. The ratio k 1/k 2 of the corresponding force
[2004] constants, k1 and k2 will be: [12 April 2019 II]
(a) +10 (b) +7 (c) –7 (d) +13 1 1
9. A spring of spring constant 5 × 103 N/m is stretched initially (a) n (b) 2 (c) (d) n 2
n n
by 5cm from the unstretched position. Then the work 15. A body of mass 1 kg falls freely from a height of 100m, on
required to stretch it further by another 5 cm is [2003] a platform of mass 3 kg which is mounted on a spring
(a) 12.50 N-m (b) 18.75 N-m having spring constant k = 1.25 × 106 N/m. The body sticks
(c) 25.00 N-m (d) 6.25 N-m
to the platform and the spring’s maximum compression is
10. A spring of force constant 800 N/m has an extension of 5 cm.
The work done in extending it from 5 cm to 15 cm is found to be x. Given that g = 10 ms–2, the value of x will be
[2002] close to : [11 April 2019 I]
(a) 16 J (b) 8 J (c) 32 J (d) 24 J (a) 40 cm (b) 4 cm (c) 80 cm (d) 8 cm
16. A uniform cable of mass ‘M’ and length ‘L’ is placed on a
th
TOPIC 2 Energy æ 1ö
horizontal surface such that its ç ÷ part is hanging
è nø
11. A cricket ball of mass 0.15 kg is thrown vertically up by a below the edge of the surface. To lift the hanging part of
bowling machine so that it rises to a maximum height of 20 the cable upto the surface, the work done should be:
m after leaving the machine. If the part pushing the ball [9 April 2019 I]
applies a constant force F on the ball and moves MgL MgL 2MgL
horizontally a distance of 0.2 m while launching the ball, (a) 2 (b) 2 (c) (d) nMgL
2n n n2
the value of F (in N) is (g = 10 ms–2) __________.
17. A wedge of mass M = 4m lies on a frictionless plane. A
[NA 3 Sep. 2020 (I)] particle of mass m approaches the wedge with speed v.
12. A particle (m = l kg) slides down a frictionless track There is no friction between the particle and the plane
(AOC) starting from rest at a point A (height 2 m). After or between the particle and the wedge. The maximum
reaching C, the particle continues to move freely in air height climbed by the particle on the wedge is given by:
as a projectile. When it reaching its highest point P [9 April 2019 II]
(height 1 m), the kinetic energy of the particle (in J) is: v2 2v 2
(Figure drawn is schematic and not to scale; take g = 10 (a) (b)
g 7g
ms–2) ¾¾¾ . [NA 7 Jan. 2020 I]
2v 2 v2
Height (c) (d)
A
P 5g 2g
18. A particle moves in one dimension from rest under the
influence of a force that varies with the distance travelled
2m C
by the particle as shown in the figure. The kinetic energy
of the particle after it has travelled 3 m is :
[8 April 2019 I]
O

13. A particle moves in one dimension from rest under the


influence of a force that varies with the distance travelled
by the particle as shown in the figure. The kinetic energy
of the particle after it has travelled 3 m is :
[7 Jan. 2020 II]

(a) 4 J (b) 2.5 J


(c) 6.5 J (d) 5 J
19. A particle which is experiencing a force, given by
r r r r r
F = 3i - 12 j, undergoes a displacement of d = 4i. If
the particle had a kinetic energy of 3 J at the beginning
of the displacement, what is its kinetic energy at the end
(a) 4 J (b) 2.5 J of the displacement? [10 Jan. 2019 II]
(c) 6.5 J (d) 5 J (a) 9 J (b) 12 J
(c) 10 J (d) 15 J
P-56 Physics

20. A block of mass m, lying on a smooth horizontal surface, v (m/s)


-1
50 ms
is attached to a spring (of negligible mass) of spring
constant k. The other end of the spring is fixed, as shown
in the figure. The block is initally at rest in its equilibrium
position. If now the block is pulled with a constant force
F, the maximum speed of the block is: [9 Jan. 2019 I]
(0,0) 10s t(s)

m (a) – 9300 J (b) 12000 J


F
(c) –4500 J (d) –12000 J
2F F pF F 28. A point particle of mass m, moves long the uniformly
(a) (b) (c) (d) rough track PQR as shown in the figure. The coefficient
mk p mk mk mk of friction, between the particle and the rough track
21. A force acts on a 2 kg object so that its position is given equals m. The particle is released, from rest from the
as a function of time as x = 3t2 + 5. What is the work
done by this force in first 5 seconds? point P and it comes to rest at a point R. The energies,
[9 Jan. 2019 II] lost by the ball, over the parts, PQ and QR, of the track,
(a) 850 J (b) 950 J (c) 875 J (d) 900 J are equal to each other, and no energy is lost when particle
22. A particle is moving in a circular path of radius a under the changes direction from PQ to QR.
k The value of the coefficient of friction m and the distance
action of an attractive potential U = - . Its total energy is: x (= QR), are, respectively close to : [2016]
2r 2
[2018] P
k k
(a) - (b) h=2m
4a 2 2a 2
30° R
3 k
(c) zero (d) - 2 Horizontal Q
2a Surface
23. Two particles of the same mass m are moving in circular (a) 0.29 and 3.5 m (b) 0.29 and 6.5 m
-16 3 (c) 0.2 and 6.5 m (d) 0.2 and 3.5 m
orbits because of force, given by F(r) = -r
29. A person trying to lose weight by burning fat lifts a mass
r
The first particle is at a distance r = 1, and the second, at of 10 kg upto a height of 1 m 1000 times. Assume that the
r = 4. The best estimate for the ratio of kinetic energies potential energy lost each time he lowers the mass is
of the first and the second particle is closest to dissipated. How much fat will he use up considering the
[Online April 16, 2018] work done only when the weight is lifted up? Fat supplies
(a) 10–1 (b) 6 × 10–2 (c) 6 × 102 (d) 3 × 10–3 3.8 × 107 J of energy per kg which is converted to
24. A body of mass m = 10–2 kg is moving in a medium and mechanical energy with a 20% efficiency rate. Take g = 9.8
experiences a frictional force F = –kv2. Its intial speed is v0 = ms–2 : [2016]
1 2 (a) 9.89 × 10–3 kg (b) 12.89 × 10–3 kg
10 ms–1. If, after 10 s, its energy is mv0 , the value of k will
8 (c) 2.45 × 10–3 kg (d) 6.45 × 10–3 kg
be: [2017] 30. A particle is moving in a circle of radius r under the action
(a) 10–4 kg m–1 (b) 10–1 kg m–1 s–1 of a force F = ar2 which is directed towards centre of the
(c) 10–3 kg m–1 (d) 10–3 kg s–1 circle. Total mechanical energy (kinetic energy + potential
25. An object is dropped from a height h from the ground. energy) of the particle is (take potential energy = 0 for r = 0) :
Every time it hits the ground it looses 50% of its kinetic [Online April 11, 2015]
energy. The total distance covered as t ® ¥ is
1 3 5 3 4 3
[Online April 8, 2017] (a) ar (b) ar (c) αr (d) ar3
2 6 3
5 8 31. A block of mass m = 0.1 kg is connected to a spring of
(a) 3h (b) ¥ (c) h (d) h
3 3 unknown spring constant k. It is compressed to a distance
x from its equilibrium position and released from rest. After
26. A time dependent force F = 6t acts on a particle of mass
1 kg. If the particle starts from rest, the work done by the approaching half the distance æç x ö÷ from equilibrium
force during the first 1 second will be [2017] è 2ø
(a) 9 J (b) 18 J (c) 4.5 J (d) 22 J position, it hits another block and comes to rest
momentarily, while the other block moves with a velocity
27. Velocity–time graph for a body of mass 10 kg is shown in 3 ms–1.
figure. Work–done on the body in first two seconds of The total initial energy of the spring is :
the motion is : [Online April 10, 2016] [Online April 10, 2015]
Work, Energy & Power P-57

(a) 0.3 J (b) 0.6 J (a) constant (b) t


(c) 0.8 J (d) 1.5 J
1
th (c) (d) t
æ1ö t
32. A bullet looses ç ÷ of its velocity passing through
ènø 38. The potential energy function for the force between two
one plank. The number of such planks that are required
to stop the bullet can be: [Online April 19, 2014] atoms in a diatomic molecule is approximately given by
a b
n2 2n 2 U(x) = 12 - 6 , where a and b are constants and x is
(a) (b) (c) infinite (d) n x x
2n - 1 n -1
the distance between the atoms. If the dissociation energy
33. A spring of unstretched length l has a mass m with one of the molecule is
end fixed to a rigid support. Assuming spring to be made
of a uniform wire, the kinetic energy possessed by it if its D = éëU ( x = ¥) - U at equilibrium ùû , D is [2010]
free end is pulled with uniform velocity v is:
b2 b2 b2 b2
[Online April 12, 2014] (a) (b) (c) (d)
2a 12a 4a 6a
1 1 2 1 39. An athlete in the olympic games covers a distance of 100
(a) mv 2 (b) mv2 (c) mv (d) mv 2
2 3 6 m in 10 s. His kinetic energy can be estimated to be in the
range [2008]
34. Two springs of force constants 300 N/m 5 5
(Spring A) and 400 N/m (Spring B) are joined together in (a) 200 J - 500 J (b) 2 × 10 J - 3 × 10 J
series. The combination is compressed by 8.75 cm. The (c) 20, 000 J - 50,000 J (d) 2,000 J - 5, 000 J
E E 40. A 2 kg block slides on a horizontal floor with a speed of 4m/
ratio of energy stored in A and B is A . Then A is s. It strikes a uncompressed spring, and compresses it till
EB EB
equal the block is motionless. The kinetic friction force is 15N and
to : [Online April 9, 2013] spring constant is 10,000 N/m. The spring compresses by
[2007]
4 16 3 9 (a) 8.5 cm (b) 5.5 cm
(a) (b) (c) (d)
3 9 4 16 (c) 2.5 cm (d) 11.0 cm
r 41. A particle is projected at 60o to the horizontal with a kinetic
35. The force F = Fiˆ on a particle of mass 2 kg, moving along
energy K. The kinetic energy at the highest point is
the x-axis is given in the figure as a function of its position
x. The particle is moving with a velocity of 5 m/s along the (a) K /2 (b) K [2007]
x-axis at x = 0. What is the kinetic energy of the particle at (c) Zero (d) K /4
x = 8 m? [Online May 26, 2012] 42. A particle of mass 100g is thrown vertically upwards with
a speed of 5 m/s. The work done by the force of gravity
3 during the time the particle goes up is [2006]
(a) –0.5 J (b) –1.25 J
2
(c) 1.25 J (d) 0.5 J
1
43. The potential energy of a 1 kg particle free to move along
F (in N)

0 x (in m) æ x 4 x2 ö
the x-axis is given by V ( x) = ç - ÷ J.
–1
è 4 2ø
–2 The total mechanical energy of the particle is 2 J. Then,
0 1 2 3 4 5 6 78 the maximum speed (in m/s) is [2006]
(a) 34 J (b) 34.5 J (c) 4.5 J (d) 29.4 J 3 1
36. A particle gets displaced by (a) (b) 2 (c) (d) 2
2 2
( )
D r = 2iˆ + 3 ˆj + 4kˆ m under the action of a force 44. A mass of M kg is suspended by a weightless string. The
r
( )
F = 7iˆ + 4 ˆj + 3kˆ . The change in its kinetic energy is
horizontal force that is required to displace it until the
string makes an angle of 45° with the initial vertical
[Online May 7, 2012] direction is [2006]
(a) 38 J (b) 70 J (c) 52.5 J (d) 126 J
(a) Mg ( 2 + 1) (b) Mg 2
37. At time t = 0 a particle starts moving along the x-axis. If
Mg
its kinetic energy increases uniformly with time ‘t’, the (c) (d) Mg ( 2 - 1)
net force acting on it must be proportional to [2011 RS] 2
P-58 Physics

45. A spherical ball of mass 20 kg is stationary at the top of 52. A 60 HP electric motor lifts an elevator having a
a hill of height 100 m. It rolls down a smooth surface to
maximum total load capacity of 2000 kg. If the frictional
the ground, then climbs up another hill of height 30 m and force on the elevator is 4000 N, the speed of the elevator
finally rolls down to a horizontal base at a height of 20 m at full load is close to: (1 HP = 746 W, g = 10 ms–2)
above the ground. The velocity attained by the ball is [7 Jan. 2020 I]
[2005] (a) 1.7 ms–1 (b) 1.9 ms–1
(a) 20 m/s (b) 40 m/s (c) 1.5 ms–1 (d) 2.0 ms–1
(c) 10 30 m/s (d) 10 m/s 53. A particle of mass M is moving in a circle of fixed radius
R in such a way that its centripetal acceleration at time t
46. A particle moves in a straight line with retardation is given by n2 R t2 where n is a constant. The power
proportional to its displacement. Its loss of kinetic energy delivered to the particle by the force acting on it, is :
for any displacement x is proportional to [2004] [Online April 10, 2016]
(a) x (b) e x (c) x2 (d) loge x
47. A particle is acted upon by a force of constant magnitude 1
(a) M n2 R2t2 (b) M n2R2t
which is always perpendicular to the velocity of the particle, 2
the motion of the particles takes place in a plane. It follows (c) M n R2t2 (d) M n R2t
that [2004] 54. A car of weight W is on an inclined road that rises by 100
(a) its kinetic energy is constant m over a distance of 1 Km and applies a constant frictional
(b) its acceleration is constant W
(c) its velocity is constant force on the car. While moving uphill on the road at
20
(d) it moves in a straight line a speed of 10 ms–1, the car needs power P. If it needs
48. A wire suspended vertically from one of its ends is
P
stretched by attaching a weight of 200N to the lower end. power while moving downhill at speed v then value of
The weight stretches the wire by 1 mm. Then the elastic 2
v is: [Online April 9, 2016]
energy stored in the wire is [2003]
(a) 20 ms–1 (b) 5 ms–1 (c) 15 ms–1 (d) 10 ms–1
(a) 0.2 J (b) 10 J (c) 20 J (d) 0.1 J
55. A wind-powered generator converts wind energy into electrical
49. A ball whose kinetic energy is E, is projected at an angle of energy. Assume that the generator converts a fixed fraction
45° to the horizontal. The kinetic energy of the ball at the of the wind energy intercepted by its blades into electrical
highest point of its flight will be [2002] energy. For wind speed n, the electrical power output will be
(a) E (b) E / 2 (c) E/2 (d) zero most likely proportional to
[Online April 25, 2013]
(a) n 4 (b) n 2 (c) n (d) n
TOPIC 3 Power 56. A 70 kg man leaps vertically into the air from a crouching
position. To take the leap the man pushes the ground with
50. A body of mass 2 kg is driven by an engine delivering a a constant force F to raise himself. The center of gravity
constant power of 1 J/s. The body starts from rest and rises by 0.5 m before he leaps. After the leap the c.g. rises
moves in a straight line. After 9 seconds, the body has by another 1 m. The maximum power delivered by the
moved a distance (in m) ________. [5 Sep. 2020 (II)] muscles is : (Take g = 10 ms–2) [Online April 23, 2013]
51. A particle is moving unidirectionally on a horizontal plane (a) 6.26 × 103 Watts at the start
under the action of a constant power supplying energy
source. The displacement (s) - time (t) graph that describes (b) 6.26 × 103 Watts at take off
the motion of the particle is (graphs are drawn schematically (c) 6.26 × 104 Watts at the start
and are not to scale) : [3 Sep. 2020 (II)] (d) 6.26 × 104 Watts at take off
S S 57. A body of mass ‘m’, accelerates uniformly from rest to ‘v1’
in time ‘t1’. The instantaneous power delivered to the body
as a function of time ‘t’ is [2004]
mv1t 2 mv12t
(a) (b) (a) (b)
t1 t12
t t mv1t mv12t
S S (c) t (d)
1 t1
58. A body is moved along a straight line by a machine
delivering a constant power. The distance moved by the
(c) (d) body in time ‘t’ is proportional to [2003]
t t (a) t3/4 (b) t3/2 (c) t1/4 (d) t1/2
Work, Energy & Power P-59

64. A particle of mass m is moving along the x-axis with initial


TOPIC 4 Collisions velocity uiˆ. It collides elastically with a particle of mass
10 m at rest and then moves with half its initial kinetic
59. Two bodies of the same mass are moving with the same energy (see figure). If sin q1 = n sin q2 , then value of n
speed, but in different directions in a plane. They have a is ___________. [NA 2 Sep. 2020 (II)]
completely inelastic collision and move together m
thereafter with a final speed which is half of their initial q1
speed. The angle between the initial velocities of the m 10 m q2
ui$
two bodies (in degree) is ______.[NA 6 Sep. 2020 (I)]
10 m
60. Particle A of mass m moving with velocity ( 3$i + $j ) ms-1
1
65. Two particles of equal mass m have respective initial
collides with another particle B of mass m2 which is at rest æ iˆ + ˆj ö
r r velocities uiˆ and u ç 2 ÷ . They collide completely
initially. Let V1 and V2 be the velocities of particles A and è ø
B after collision respectively. If m1 = 2m2 and after collision inelastically. The energy lost in the process is: [9 Jan. 2020 I]
r r r 1 1
V1 = ($i + 3 $j ) ms -1 , the angle between V1 and V2 is : (a) mu2 (b) mu2
3 8
[6 Sep. 2020 (II)]
3 2
(a) 15º (b) 60º (c) mu2 (d) mu2
4 3
(c) – 45º (d) 105º 66. A body A, of mass m = 0.1 kg has an initial velocity of 3 iˆ ms–1.
61. Blocks of masses m, 2m, 4m and 8m are arranged in a line on It collides elastically with another body, B of the same
a frictionless floor. Another block of mass m, moving with
speed v along the same line (see figure) collides with mass mass which has an initial velocity of 5 ĵ ms–1. After
r
m in perfectly inelastic manner. All the subsequent collisions
are also perfectly inelastic. By the time the last block of
(
collision, A moves with a velocity v = 4 iˆ + ˆj . The )
mass 8m starts moving the total energy loss is p% of the x
energy of B after collision is written as
J. The value of
original energy. Value of ‘p’ is close to : 10
[4 Sep. 2020 (I)] x is _______. [NA 8 Jan. 2020 I]
67. A particle of mass m is dropped from a height h above the
v
ground. At the same time another particle of the same
mass is thrown vertically upwards from the ground with a
m m 2m 4m 8m speed of 2 gh . If they collide head-on completely
(a) 77 (b) 94 inelastically, the time taken for the combined mass to reach
(c) 37 (d) 87
62. A block of mass 1.9 kg is at rest at the edge of a table, of h
height 1 m. A bullet of mass 0.1 kg collides with the block the ground, in units of is: [8 Jan. 2020 II]
g
and sticks to it. If the velocity of the bullet is 20 m/s in
the horizontal direction just before the collision then the 1 3
(a) (b)
kinetic energy just before the combined system strikes 2 4
the floor, is [Take g = 10 m/s2 . Assume there is no 1 3
rotational motion and losss of energy after the collision (c) (d)
is negligiable.] 2 2
68. A man (mass = 50 kg) and his son (mass = 20 kg) are
[3 Sep. 2020 (II)]
standing on a frictionless surface facing each other. The
(a) 20 J (b) 21 J (c) 19 J (d) 23 J
man pushes his son so that he starts moving at a speed of
63. A particle of mass m with an initial velocity u iˆ collides 0.70 ms–1 with respect to the man. The speed of the man
perfectly elastically with a mass 3 m at rest. It moves with respect to the surface is : [12 April 2019 I]
with a velocity v ˆj after collision, then, v is given by : (a) 0.28 ms–1 (b) 0.20 ms–1
[2 Sep. 2020 (I)] (c) 0.47 ms–1 (d) 0.14 ms–1
2 u 69. Two particles, of masses M and 2M, moving, as shown,
(a) v = u (b) v =
3 3 with speeds of 10 m/s and 5 m/s, collide elastically at the
u 1 origin. After the collision, they move along the indicated
(c) v = (d) v = u directions with speeds v1 and v2, respectively. The values
2 6
of v1 and v2 are nearly : [10 April 2019 I]
P-60 Physics

5
C, also perfectly inelastically th of the initial kinetic
6
energy is lost in whole process. What is value of M/m?
[9 Jan. 2019 I]
A B C
m m M

(a) 6.5 m/s and 6.3 m/s (b) 3.2 m/s and 6.3 m/s (a) 5 (b) 2 (c) 4 (d) 3
(c) 6.5 m/s and 3.2 m/s (d) 3.2 m/s and 12.6 m/s 76. In a collinear collision, a particle with an initial speed n0
70. A body of mass 2 kg makes an elastic collision with a strikes a stationary particle of the same mass. If the final
second body at rest and continues to move in the original total kinetic energy is 50% greater than the original
direction but with one fourth of its original speed. What kinetic energy, the magnitude of the relative velocity
is the mass of the second body? [9 April 2019 I] between the two particles, after collision, is: [2018]
(a) 1.0 kg (b) 1.5 kg (c) 1.8 kg (d) 1.2 kg n0
n0 n0
71. A particle of mass ‘m’ is moving with speed ‘2v’ and (a) (b) 2n0 (c) (d)
collides with a mass ‘2m’ moving with speed ‘v’ in the 4 2 2
same direction. After collision, the first mass is stopped 77. The mass of a hydrogen molecule is 3.32×10–27 kg. If 1023
completely while the second one splits into two particles hydrogen molecules strike, per second, a fixed wall of area
each of mass ‘m’, which move at angle 45° with respect to 2 cm2 at an angle of 45° to the normal, and rebound
the original direction. [9 April 2019 II] elastically with a speed of 103 m/s, then the pressure on
The speed of each of the moving particle will be: the wall is nearly: [2018]
(a) 2.35 × 103 N/m2 (b) 4.70 × 103 N/m2
(a) 2 v (b) 2 2 v
(c) 2.35 × 102 N/m2 (d) 4.70 × 102 N/m2
(c) v / (2 2) (d) v/ 2 78. It is found that if a neutron suffers an elastic collinear
72. A body of mass m1 moving with an unknown velocity of collision with deuterium at rest, fractional loss of its energy
is pd; while for its similar collision with carbon nucleus at
v1 iˆ , undergoes a collinear collision with a body of mass
rest, fractional loss of energy is Pc. The values of Pd and
m2 moving with a velocity v2 iˆ . After collision, m1 and m2 Pc are respectively: [2018]
move with velocities of v3 iˆ and v4 iˆ , respectively.. (a) ( ×89, ×28) (b) ( ×28, ×89 ) (c) (0, 0) (d) (0, 1)
If m2 = 0.5 m1 and v3 = 0.5 v1, then v1 is : [8 April 2019 II] 79. A proton of mass m collides elastically with a particle of
v v unknown mass at rest. After the collision, the proton and
(a) v4 – 2 (b) v4 – v2 (c) v4 – 2 (d) v4 + v2 the unknown particle are seen moving at an angle of 90°
2 4
73. An alpha-particle of mass m suffers 1-dimensional elastic with respect to each other. The mass of unknown particle
collision with a nucleus at rest of unknown mass. It is is: [Online April 15, 2018]
scattered directly backwards losing, 64% of its initial m m
kinetic energy. The mass of the nucleus is : (a) (b) (c) 2m (d) m
3 2
[12 Jan. 2019 II] 80. Two particles A and B of equal mass M are moving with
(a) 2 m (b) 3.5 m (c) 1.5 m (d) 4 m the same speed v as shown in the figure. They collide
74. A piece of wood of mass 0.03 kg is dropped from the completely inelastically and move as a single particle C.
top of a 100 m height building. At the same time, a bullet The angle q that the path of C makes with the X-axis is
of mass 0.02 kg is fired vertically upward, with a velocity given by: [Online April 9, 2017]
100 ms–1, from the ground. The bullet gets embedded in
the wood. Then the maximum height to which the 3+ 2 Y
(a) tanq =
combined system reaches above the top of the building 1- 2
before falling below is: (g = 10 ms–2) [10 Jan. 2019 I]
C
(a) 20 m (b) 30 m (c) 40 m (d) 10 m 3- 2
(b) tanq =
75. There block A, B and C are lying on a smooth horizontal 1- 2 q
X
surface, as shown in the figure. A and B have equal
1- 2 A 45°
masses, m while C has mass M. Block A is given an inital (c) tanq = 30°
2(1 + 3) B
speed v towards B due to which it collides with B
perfectly inelastically. The combined mass collides with 1- 3
(d) tanq =
1+ 2
Work, Energy & Power P-61

81. A neutron moving with a speed 'v' makes a head on 86. A projectile of mass M is fired so that the horizontal
collision with a stationary hydrogen atom in ground state. range is 4 km. At the highest point the projectile explodes
The minimum kinetic energy of the neutron for which in two parts of masses M/4 and 3M/4 respectively and
inelastic collision will take place is : the heavier part starts falling down vertically with zero
[Online April 10, 2016] initial speed. The horizontal range (distance from point
(a) 20.4 eV (b) 10.2 eV (c) 12.1 eV (d) 16.8 eV of firing) of the lighter part is :
82. A particle of mass m moving in the x direction with speed [Online April 23, 2013]
2v is hit by another particle of mass 2m moving in the y (a) 16 km (b) 1 km (c) 10 km (d) 2 km
direction with speed v. If the collision is perfectly inelastic, 87. A moving particle of mass m, makes a head on elastic
the percentage loss in the energy during the collision is collision with another particle of mass 2m, which is initially
close to : [2015] at rest. The percentage loss in energy of the colliding
(a) 56% (b) 62% (c) 44% (d) 50% particle on collision, is close to
83. A bullet of mass 4g is fired horizontally with a speed of [Online May 19, 2012]
300 m/s into 0.8 kg block of wood at rest on a table. If the (a) 33% (b) 67% (c) 90% (d) 10%
coefficient of friction between the block and the table is 88. Two bodies A and B of mass m and 2m respectively are
0.3, how far will the block slide approximately? placed on a smooth floor. They are connected by a spring
[Online April 12, 2014] of negligible mass. A third body C of mass m is placed
(a) 0.19 m (b) 0.379 m (c) 0.569 m (d) 0.758 m on the floor. The body C moves with a velocity v0 along
84. Three masses m, 2m and 3m are moving in x-y plane with the line joining A and B and collides elastically with A.
speed 3u, 2u and u respectively as shown in figure. The At a certain time after the collision it is found that the
three masses collide at the same point at P and stick instantaneous velocities of A and B are same and the
together. The velocity of resulting mass will be: compression of the spring is x0. The spring constant k
will be [Online May 12, 2012]
[Online April 12, 2014]
y v02 v0
(a) m (b) m 2 x
2m, 2u x02 0
2
v0 2 æ v0 ö
60° (c) 2m x (d) m
x 0 3 çè x0 ÷ø
m, 3u P 60°
89. A projectile moving vertically upwards with a velocity of
3m, u 200 ms–1 breaks into two equal parts at a height of 490 m.
One part starts moving vertically upwards with a velocity
(a)
u ˆ
12
(
i + 3jˆ ) (b)
u ˆ
12
(
i - 3jˆ ) of 400 ms–1. How much time it will take, after the break
up with the other part to hit the ground?
(c)
u ˆ
12
(
-i + 3jˆ ) (d)
u ˆ
12
(
-i - 3jˆ ) [Online May 12, 2012]

85. This question has statement I and statement II. Of the four (a) 2 10 s (b) 5 s
choices given after the statements, choose the one that
best describes the two statements. (c) 10 s (d) 10 s
Statement - I: Apoint particle of mass m moving with 90. Statement -1: Two particles moving in the same direction
speed u collides with stationary point particle of mass do not lose all their energy in a completely inelastic
M. If the maximum energy loss possible is given as collision.
æ1 ö Statement -2 : Principle of conservation of momentum
f ç mv2 ÷ then f = æç m ö÷ . holds true for all kinds of collisions. [2010]
è2 ø
èM + mø (a) Statement -1 is true, Statement -2 is true ; Statement
Statement - II: Maximum energy loss occurs when the -2 is the correct explanation of Statement -1.
particles get stuck together as a result of the collision. (b) Statement -1 is true, Statement -2 is true; Statement -2
[2013] is not the correct explanation of Statement -1
(a) Statement - I is true, Statment - II is true, Statement (c) Statement -1 is false, Statement -2 is true.
- II is the correct explanation of Statement - I. (d) Statement -1 is true, Statement -2 is false.
(b) Statement-I is true, Statment - II is true, Statement - 91. A block of mass 0.50 kg is moving with a speed of 2.00
II is not the correct explanation of Statement - II. ms–1 on a smooth surface. It strikes another mass of 1.00
(c) Statement - I is true, Statment - II is false. kg and then they move together as a single body. The
(d) Statement - I is false, Statment - II is true. energy loss during the collision is [2008]
(a) 0.16 J (b) 1.00 J (c) 0.67 J (d) 0.34 J
P-62 Physics

(a) 0.16 J (b) 1.00 J (c) 0.67 J (d) 0.34 J


the l st mass moves with velocity
v
92. A bomb of mass 16kg at rest explodes into two pieces of in a direction
3
masses 4 kg and 12 kg. The velolcity of the 12 kg mass is perpendicular to the initial direction of motion. Find the
4 ms–1. The kinetic energy of the other mass is [2006]
speed of the 2 nd mass after collision. [2005]
(a) 144 J (b) 288 J (c) 192 J (d) 96 J
93. The block of mass M moving on the frictionless horizontal m m v
3
surface collides with the spring of spring constant k and A before Aafter
compresses it by length L. The maximum momentum of collision collision
the block after collision is [2005] v 2
(a) 3v (b) v (c) v
(d)
M 3 3
95. Consider the following two statements : [2003]
A. Linear momentum of a system of particles is zero
kL2 ML2 B. Kinetic energy of a system of particles is zero.
(a) (b) Mk L (c) (d) zero Then
2M k
(a) A does not imply B and B does not imply A
94. A mass ‘m’ moves with a velocity ‘v’ and collides
(b) A implies B but B does not imply A
inelastically with another identical mass. After collision
(c) A does not imply B but B implies A
(d) A implies B and B implies A
Work, Energy & Power P-63

1. (d) The given situation can be drawn graphically as


3mg 2 t 2
shown in figure. or, W =
8
Work done = Area under F-x graph
= Area of rectangle ABCD + Area of trapezium BCFE ds
5. (a) From question, v = a s =
F dt
A B a 2t2
200N or, 2 s = at Þ S =
4
100N E a2
F=m´
2
D C F ma 2 a 2 t 2 1 4 2
Work done = ´ = ma t
15m 30m 2 4 8
6. (c) Work done in stretching the rubber-band by a
1
W = (200 ´ 15) + (100 + 200) ´ 15 = 3000 + 2250 distance dx is
2 dW = F dx = (ax + bx2)dx
Þ W = 5250 J Integrating both sides,
2. (3) If AC = l then according to question, BC = 2l and AB = L L
aL2 bL3
3l. W = ò axdx + ò bx 2 dx = +
2 3
0 0
B 7. (b) Mass of over hanging part of the chain

Smooth 4
m¢ = ´ (0.6)kg = 1.2 kg
Rough C 3lsinq 2
m Weight of hanging part of the chain
q = 1.2 × 10 = 12 N
A C.M. of hanging part = 0.3 m below the table
Here, work done by all the forces is zero. Workdone in putting the entire cha in on the table = 12 ×
Wfriction + Wmg = 0 0.30 = 3.6 J.
r
mg (3l )sin q - mmg cos q(l ) = 0 8. (
(b) Given, Force, F = 5iˆ + 3 ˆj + 2kˆ )
Þ mmg cos ql = 3mgl sin q Displacement, x = ( 2iˆ – ˆj )
Þ m = 3tan q = k tan q Work done,
r r
\k = 3 W = F × x = (5iˆ + 3 ˆj + 2kˆ ) × (2iˆ - ˆj )
r uur = 10 – 3 = 7 joules
3. (c) Work done, W = ò F × ds 9. (b) Spring constant, k = 5 × 103 N/m

( )(
= - xi$ + yjˆ × d ´ i$ + dyjˆ ) Let x1 and x2 be the initial and final stretched position of
the spring, then

( )
0 1 1
Þ W = – ò xdx + ò ydy Work done, W = k x22 - x12
2
1
1 0
= ´ 5 ´ 103 é(0.1)2 - (0.05) 2 ù
2 ë û
æ 1ö 1
= ç 0 + ÷ + = 1J 5000
è 2ø 2 = ´ 0.15 ´ 0.05 = 18.75 Nm
2
mg 3mg 10. (b) Small amount of work done in extending the spring
4. (d) Here, N – mg = ma = ÞN=
2 2 by dx is
N = normal reaction dW = k x dx
Now, work done by normal reaction ‘N’ on 0.15
r r æ 3mg ö æ 1
block in time t, W=NS= çè
2

÷ø çè g/ 2 t ÷ø
2
\W= k ò x dx
0.05
P-64 Physics

800 é solving x » 4 cm
= (0.15) 2 - (0.05) 2 ù
2 ë û 16. (a) W = uf – ui
= 400 [(0.15 + 0.05)(0.15 – 0.05)] æ mg L ö MgL .
= 0-ç- ´ =
= 400 × 0.2 × 0.1 = 8 J è n 2n ÷ø 2n 2
11. (150.00) From work energy theorem, 17. (c) mv = (m + M) V’
1 2
W = F × s = DKE = mv mv mv v
2 or v = = =
m + M m + 4m 5
Here V 2 = 2 gh Using conservation of ME, we have
2 1 15 2
\F ×s = F ´ = ´ ´ 2 ´ 10 ´ 20 1 2 1 æ vö
10 2 100 mv = ( m + 4m) ç ÷ + mgh
2 2 è 5ø
\ F = 150 N.
2 v2
12. (10.00) Kinetic energy = change in potential energy of or h =
5 g
the particle, 18. (c) We know area under F-x graph gives the work done
KE = mgDh by the body
Given, m = 1 kg, 1
\W= ´ (3 + 2) ´ (3 - 2) + 2 ´ 2
Dh = h2 – h1 = 2 – 1 = 1m 2
= 2.5 + 4
\ KE = 1 × 10 × 1 = 10 J
13. (c) We know area under F-x graph gives the work done = 6.5 J
by the body Using work energy theorem,
1 D K.E = work done
\W= ´ (3 + 2) ´ (3 - 2) + 2 ´ 2 = 2.5 + 4 = 6.5 J \ D K.E = 6.5 J
2
r r r r r
Using work energy theorem, ( )( )
19. (d) Work done = F.d = 3i –12J . 4i = 12J
D K.E = work done
\ D K.E = 6.5 J From work energy theorem,
wnet = DK.E. = kf – ki
14. (c) l1 + l2 = l and l1 = nl2
Þ 12 = kf – 3
nl l
\ l1 = and l2 = \ Kf = 15J
n +1 n +1
1 20. (d) Maximum speed is at mean position or equilibrium
As k µ ,
l At equilibrium Position
k1 l / (n + 1) 1 F
\ k = (nl ) / (n + 1) = n F = kx Þ x =
2 k
15. (b) Velocity of 1 kg block just before it collides with 3 kg From work-energy theorem,
block = 2gh = 2000 m/s WF + Wsp = DKE
Using principle of conservation of linear momentum just 1 2 1
F(x) – kx = mv 2 - 0
before and just after collision, we get 2 2
2
2000 æ F ö 1 æ Fö 1
1 ´ 2000 = 4v Þ v = m/s 4 kg v F ç ÷ - k ç ÷ = mv2
4 è kø 2 è kø 2
Initial compression of spring 1 F2 1
1.25 × 106 x0 = 30 Þ x0 » 0 Þ = mv 2
2 K 2
using work energy theorem, F
Wg + Wsp = DKE or, v max =
mk
1 21. (d) Position, x = 3t2 + 5
Þ 40 ´ x + ´ 1.25 ´ 106 (02 - x 2 )
2 dx d ( 3t 2 + 5 )
1 \ Velocity, v = Þv =
= 0 - ´ 4 ´ v2 dt dt
2 Þ v= 6t +0
Work, Energy & Power P-65

At t = 0 v= 0 dV
And, at t = 5 sec v = 30 m/s 26. (c) Using, F = ma = m
According to work-energy theorem, w = DKE dt
dV
1 1 6t = 1. [Q m = 1 kg given]
or, W = mv – 0 = (2)(30) = 900J
2 2
dt
2 2
v 1
¶u K ét 2 ù
22. (c) F = -
¶r
rˆ = 3 rˆ ò dV = ò 6t dt V = 6 ê 2 ú = 3 ms –1
r 0 ë û0
Since particle is moving in circular path [Q t = 1 sec given]
2 From work-energy theorem,
mv K K
F= = Þ mv 2 =
r r 3
r2 W = DKE =
1
2
( ) 1
m V 2 - u 2 = ´ 1 ´ 9 = 4.5 J
2
1 K
\ K.E. = mv2 = 2 v – 4 (0 , 50) < ,5 m / s 2
2 2r 27. (c) Acceleration (a) = = (10 , 0)
Total energy = P.E. + K.E. t
K K K u = 50 m/s
= - 2 + 2 = Zero (Q P.E. = - 2 given) \ v = u + at = 50 – 5t
2r 2r 2r Veocity in first two seconds t = 2
23. (b) As the particles moving in circular orbits, So
v(at t <2) < 40 m / s
mv2 16 2
= +r From work-energy theorem,
r r
1 1 1
Kinetic energy, KE0 = mv2 = [16 + r 4 ] ΧK.E. < W < (40 2 , 50 2 ) ´10 < , 4500 J
2 2 2
1 28. (a) Work done by friction at QR = µmgx
For first particle, r = 1, K1 = m(16 + 1)
2 1 2
1 In triangle, sin 30° = =
Similarly, for second particle, r = 4, K 2 = m (16 + 256) 2 PQ
2 Þ PQ = 4m
16 + 1 Work done by friction at PQ = µmg × Cos 30° × 4
K1 2 17
\ = = ; 6 ´ 10-2 3
K 2 16 + 256 272 = µmg × ´ 4 = 2 3 µmg
2 2
24. (a) Let Vf is the final speed of the body. Since work done by friction on parts PQ and QR are equal,
From questions, µmg x = 2 3 umg
1 1 V Þ x = 2 3 @ 3.5m
mV f2 = mV02 Þ V f = 0 = 5 m/s
2 8 2 Using work energy theorem mg sin 30° × 4 = 2 3 µmg + µmgx
æ dV ö 2 dV
F = mç ÷ = -kV \ (10–2) = –kV2 Þ2= 4 3 µ
è dt ø dt Þ µ = 0.29
5 10
dV W mgh ´ 1000 10 ´ 9.8 ´ 1 ´ 1000
ò V2 = -100 K ò dt 29. (b) n =
input
=
input
=
input
10 0
98000
1 1 Input = = 49 × 104J
- = 100K (10) or, K = 10–4 kgm–1 0.2
5 10
25. (a) (K.E.)' = 50% of K.E. after hit i.e., 49 ´ 10 4
Fat used = = 12.89 × 10–3kg.
1 50 1 v 3.8 ´ 107
mv '2 = ´ mv 2 Þ v ' = 30. (b) As we know, dU = F.dr
2 100 2 2
r
1 ar 3
Coefficient of restitution = U = ò ar 2 dr = ...(i)
2 3
0
Now, total distance travelled by object is
mv 2
æ 1ö As, = ar 2
æ 1 + e2 ö 1+ r
ç 2 ÷ = 3h
H = hç 2÷
= hç ÷ m2v2 = mar3
è1- e ø 1
çè 1 - ÷ø
2 1 3
or, 2m(KE) = ar ...(ii)
2
P-66 Physics

Total energy = Potential energy + kinetic energy x B = (8.75 - x) cm


Now, from eqn (i) and (ii)
Total energy = K.E. + P.E. F = 300 ´ x = 400(8.75 - x)
Solving we get, x = 5 cm
ar 3 ar 3 5 3
= + = ar x B = 8.75 - 5 = 3.75cm
3 2 6
31. (b) Applying momentum conservation 1
k (x )2
m1u1 + m2u2 = m1v1 + m2v2
EA 2 A A 300 ´ (5)2 4
= = =
EB 1 2
k B (x B )2 400 ´ (3.75)
0.1u + m(0) = 0.1(0) + m(3) 3
0.1u = 3m 2
1 1 35. (d)
0.1u 2 = m(3) 2 36. (a) According to work-energy theorem,
2 2
Solving we get, u = 3 Change in kinetic energy = work done

1 2 1 æ xö 1
2
kx = K ç ÷ + (0.1)32
® ®
( )(
= F .D r = 7iˆ + 4 ˆj + 3kˆ . 2iˆ + 3 ˆj + 4kˆ )
2 2 è2ø 2 = 14 + 12 + 12 = 38 J
3 2 37. (c) K.E. µ t
Þ kx = 0.9
4 K.E. = ct [Here, c = constant]
3 1 2 1 2
Þ ´ kx = 0.9 mv = ct
2 2 Þ
2
1 2
\ Kx = 0.6 J (total initial energy of the spring) (mv )2
2 Þ = ct
2m
32. (a) Let u be the initial velocity of the bullet of mass m.
After passing through a plank of width x, its velocity p2
Þ = ct (Q p = mv)
decreases to v. 2m
4 4 u(n - 1) Þ p = 2ctm
\ u–v= or, v = u - =
n n n dp d ( 2 ctm )
If F be the retarding force applied by each plank, then Þ F= =
dt dt
using work – energy theorem,
1
1 1 1 1 ( n - 1)2 Þ F = 2 cm ´
Fx = mu2 – mv2 = mu2 – mu2 2 t
2 2 2 2 n2 1
é 2ù Þ Fµ
1 2 ê1 - ( n - 1) ú t
= 2 mu
ê n2 ú – dU ( x )
ë û 38. (d) At equilibrium : F =
dx
1 æ 2n - 1 ö
Fx = mu 2 ç ÷ –d é a


è n2 ø Þ F=
2 dx êë x12 x 6 úû
Let P be the number of planks required to stop the bul-
let. é 12a 6b ù
Þ F = – ê 13 + 7 ú
Total distance travelled by the bullet before coming to ëx x û
rest = Px 1
Using work-energy theorem again, 12a 6b æ 2a ö 6
Þ = 7 Þ x=ç ÷
F ( Px ) =
1
mu 2 - 0 x13 x è b ø
2 a b b2
\ U at equilibrium = - = -
2 ( 2n - 1) ù 1
é1 2
and U(x=¥) = 0
æ 2a ö 4a
or, P ( Fx ) = P ê 2 mu
2 æ 2a ö
2 ú = mu ç ÷ ç ÷
ë n û 2 è b ø è b ø
n2 æ 2 ö b2
\ P= \ D = 0 -ç- b ÷ =
2n - 1 è 4a ø 4a
33. (d) 39. (d) The average speed of the athelete
34. (a) Given : k A = 300 N / m, k B = 400 N / m 5 100
v= = = 10 m / s
Let when the combination of springs is compressed by t 10
force F. Spring A is compressed by x. Therefore compression 1
\ K.E. = mv 2
in spring B 2
Work, Energy & Power P-67

Assuming the mass of athelet to 40 kg his average K.E 44. (d) Work done by tension + Work done by force (applied)
would be + Work done by gravitational force = change in kinetic
1 energy
K.E = ´ 40 ´ (10)2 = 2000J
2 Work done by tension is zero
Assuming mass to 100 kg average kinetic energy
1 O
K.E. = ´ 100 ´ (10) 2 = 5000 J
2
40. (b) Suppose the spring gets compressed by x before 45°
stopping. l
kinetic energy of the block = P.E. stored in the spring + B
work done against friction. A F
1 2 1 2 C F
Þ ´ 2 ´ (4) = ´ 10,000 ´ x + 15 ´ x
2 2 Þ 0 + F ´ AB - Mg ´ AC = 0
Þ 10,000 x2 + 30x – 32 = 0
é 1 ù
Þ 5000 x 2 + 15 x - 16 = 0
æ AC ö ê1 - ú
2
Þ F = Mgç ÷ = Mg ê ú
-15 ± (15)2 - 4 ´ (5000)(-16) è AB ø ê 1 ú
\ x= êë 2 úû
2 ´ 5000
= 0.055m = 5.5cm. l
[Q AB = l sin 45° = and
41. (d) Let u be the velocity with which the particle is thrown 2
and m be the mass of the particle. Then
æ 1 ö
AC = OC - OA = l - l cos 45° = l ç1 - ÷
1 è 2ø
K = mu 2 . ... (1)
2 where l = length of the string.]
At the highest point the velocity is u cos 60° (only the Þ F = Mg ( 2 - 1)
horizontal component remains, the vertical component
being zero at the top-most point). Therefore kinetic energy 45. (b)
at the highest point.
100
1 1 K 30 20
K ' = m(u cos 60°) 2 = mu 2 cos2 60° = [From 1]
2 2 4
mgH 1
42. (b) Given, Mass of the particle, m = 100g mv 2 + mgh
Initial speed of the particle, m = 5 m/s 2
Using conservation of energy,
Final speed of the particle, v = 0
Total energy at 100 m height
Work done by the force of gravity
= Total energy at 20m height
= Loss in kinetic energy of the body.
1 1 100 æ1 2 ö
= m (v2 – u2) = ´ (02 – 52) m (10 × 100) = m çè v + 10 ´ 20÷ø
2 2 1000 2
= –1.25 J 1 2
or v = 800 or v = 1600 = 40 m/s
43. (a) Potential energy 2
Note :
x4 – x2
V(x) = joule Loss in potential energy = gain in kinetic energy
4 2
For maxima of minima 1
m ´ g ´ 80 = mv 2
dV 2
= 0 Þ x 3 - x = 0 Þ x = ±1 1 2
dx 10 ´ 80 = v
2
1 1 1 v2 = 1600 or v = 40 m/s
Þ Min. P.E. = - = - J
4 2 4 46. (c) Given : retardation µ displacement
K.E.(max.) + P.E.(min.) = 2 (Given) i.e., a = –kx [Here, k = constant]
1 9 dv
\ K.E.(max.) = 2 + = But a = v
4 4 dx
1 2 v2 x
K.E.max . = mvmax . vdv
2 \
dx
= - kx Þ ò v dv = - ò kxdx
1 2 9 3 v1 0
Þ ´ 1 ´ vmax . = Þ vmax. =
2 4 2
P-68 Physics

v x v t
é v2 ù 2 é x2 ù P
Þ ò v dv =
m ò0
Þ ê ú = –k ê ú dt
ë 2 û v1 ë 2 û0 0
2 1/ 2
Þ (v22 - v12 ) = - kx2 Þ
v 2 Pt
2
=
m
æ 2 Pt ö
Þv=ç
è m ÷ø
1 æ - kx 2 ö
Þ
1
2
( )
m v22 - v12 = m ç
2 çè 2 ÷ø
÷ Þ
dx
dt
=
2 P 1/ 2
m
t
æ dx ö
çèQ v = ÷ø
dt
\ Loss in kinetic energy, \ DK µ x2 x t
2P 1/ 2
Þ ò dx =
m ò0
47. (a) Work done by such force is always zero since force is t dt
acting in a direction perpendicular to velocity. 0
\ From work-energy theorem = DK = 0
K remains constant. 2 P t 3/ 2 2 P 2 3/ 2
\ Distance, x = = ´ t
48. (d) The elastic potential energy m 3/ 2 m 3
1 2 ´ 1 2 3/ 2 2
= ´ Force ´ extension Þx= ´ ´ 9 = ´ 27 = 18.
2 2 3 3
1 51. (b) We know that
= ´F ´x Power, P = Fv
2
1 dv
= ´ 200 ´ 0.001 = 0.1 J But F = mav = m v
2 dt
49. (c) Let u be the speed with which the ball of mass m is dv
\ P = mv Þ P dt = mv dv
projected. Then the kinetic energy (E) at the point of dt
projection is t v

u
u Integrating both sides ò ò
P dt = m v dv
2 0 0

1 2 Þ v = æ 2P ö t1/ 2
45° P. t = mv çç ÷÷
2 è m ø
u t t
2P 1/ 2 2 P t 3/ 2
1
2
ò
Distance, s = v dt =
m ò
t dt = ×
m 3/2
E= mu 2 ...(i)
0 0
2 8P 3/ 2
When the ball is at the highest point of its flight, the speed Þs= ×t Þ s µ t3/2
9m
u So, graph (b) is correct.
of the ball is (Remember that the horizontal
2 52. (b) Total force required to lift maximum load capacity
component of velocity does not change during a against frictional force = 400 N
projectile motion).
Ftotal = Mg + friction
\ The kinetic energy at the highest point
2
= 2000 × 10 + 4000
1 æ u ö
mç 1 mu 2 E = 20,000 + 4000 = 24000 N
= ÷ = = [From (i)]
2 è 2ø 2 2 2 Using power, P = F × v
50. (18) Given, Mass of the body, m = 2 kg
60 × 746 = 24000 × v
Power delivered by engine, P = 1 J/s
Þ v = 1.86 m/s » 1.9 m/s
Time, t = 9 seconds
Hence speed of the elevator at full load is close to 1.9 ms–1
Power, P = Fv
53. (b) Centripetel acceleration ac = n2Rt2
Þ P = mav [Q F = ma]
v2
dv æ dv ö ac < < n 2 Rt 2
Þm v=P çèQ a = ÷ø R
dt dt
v2 < n2 R 2 t2
P v = nRt
Þ v dv = dt
m dv
ac < < nR
Integrating both sides we get dt
Power = matv = m nR nRt = Mn 2R2t.
Work, Energy & Power P-69

54. (c) While moving downhill power 59. (120)


m
æ wö v0
P = ç w sin q + ÷10
è 20 ø
2m
æ w w ö÷ 3w q
P < çç ∗ ÷÷10 <
èç10 20 ø q v0/2
2
sq
P 3w æç w w ÷ö w co 1
v0
< < ç , ÷÷ V tanq = 10
2 4 çè10 20 ø m
Momentum conservation along x direction,
3 v
= Þ v = 15 m/s v0 1
4 20 2mv0 cos q = 2m Þ cos q = or q = 60°
\ Speed of car while moving downhill v = 15 m/s. 2 2
55. (d) 56. (b) Hence angle between the initial velocities of the two bodies
57. (b) Let a be the acceleration of body = q + q = 60° + 60° = 120°.
Using, v = u + at 60. (d) Before collision,
v
v1 = 0 + at1 Þ a = 1 Velocity of particle A, u1 = ( 3iˆ + ˆj ) m/s
t1
Velocity of the body at instant t, Velocity of particle B, u2 = 0
v = at After collision,
v1t Velocity of particle A, v1 = (iˆ + 3 ˆj)
Þ v=
t1
r r r r Velocity of particle B, v2 = 0
Instantaneous powr, P = F × v = ( ma ) × v Using principal of conservation of angular momentum
2 r r r r
æ mv öæ v t ö æv ö m1u1 + m2 u2 = m1v1 + m2 v2
= ç 1 ÷ç 1 ÷ = m ç 1 ÷ t r
è t1 øè t1 ø è t1 ø Þ 2m2 ( 3iˆ + ˆj ) + m2 ´ 0 = 2m2 (iˆ + 3 ˆj ) + m2 ´ v2
58. (b) Power, P = Fv = ma.v r
mdv Þ 2 3iˆ + 2 ˆj = 2iˆ + 2 3 ˆj + v2
Þ P= v = c = contant r
dt Þ v2 = ( 3 - 1)iˆ - ( 3 - 1) ˆj
æ mdv ö r
çQ F = ma = ÷ Þ v1 = iˆ + 3 ˆj
è dt ø r r
mv0v = cdt For angle between v1 and v2 ,
Integrating both sides, we get r r
v t v ×v 2( 3 - 1)(1 - 3) 1 - 3
cos q = 1r r 2 = =
m ò vdv = c ò dt v1v2 2 ´ 2 2( 3 - 1) 2 2
0 0 Þ q = 105°
1
Þ mv 2 = ct r r
Angle between v1 and v2 is 105°
2
v 2 c.t v1 = $
i+ 3$
j
Þ =
2 m
60°
2c.t
Þ v2 = 45°
m
v2 = ( 3 - 1)$
i - ( 3 - 1) $
j
2c 1 2
Þ v= ´t
m 61. (b) According to the question, all collisions are perfectly
inelastic, so after the final collision, all blocks are moving
dx 2c 12 dx together.
Þ = ´t where v =
dt m dt v
m
x t
2c 1
Þ ò dx = m ò
´ t 2 dt
m 2m 4m 8m
e 0
Let the final velocity be v', using momentum conservation
3
2c 2t 2 v
Þ x= ´ Þ xµt
3
2 mv = 16mv ' Þ v ' =
m 3 16
P-70 Physics

1
Now initial energy Ei = mv 2 2 2 mu 2 mv 2
or, mu = mv + +
2 3 3
1 æ v ö
2
1 mv 2 u
Final energy : E f = ´ 16m ´ ç ÷ = \v =
2 è 16 ø 2 16 2
64. (10.00)
1 2 1 v2 From momentum conservation in perpendicular direction
Energy loss : Ei - E f = mv - m
2 2 16 of initial motion.
1 é 1ù 1 é15 ù mu1 sin q1 = 10mv1 sin q2 ...(i)
Þ mv 2 ê1 - ú Þ mv2 ê ú
2 ë 16 û 2 ë16 û It is given that energy of m reduced by half. If u1 be
The total energy loss is P% of the original energy. velocity of m after collision, then
Energy loss æ 1 2ö 1 1 2
\ %P = ´ 100 çè mu ÷ø = mu1
Original energy 2 2 2
1 2 é15 ù u
mv ê ú Þ u1 =
2 ë16 û ´ 100 = 93.75% 2
=
1 2 If v1 be the velocity of mass 10 m after collision, then
mv
2 1 1 u2 u
Hence, value of P is close to 94. ´ 10 m ´ v12 = m Þ v1 =
2 2 2 20
62. (b) Given, From equation (i), we have
Mass of block, m1 = 1.9 kg
sin q1 = 10 sin q2
Mass of bullet, m2 = 0.1 kg u/2 y
Velocity of bullet, v2 = 20 m/s
Let v be the velocity of the combined system. It is an
65. (b) m u m u/2
inelastic collision. x
Using conservation of linear momentum O
d
m1 ´ 0 + m2 ´ v2 = (m1 + m2 )v x-direction
Þ 0.1 ´ 20 = (0.1 + 1.9) ´ v mu 3u
mu + = 2mv'x Þ Vx' =
Þ v = 1 m/s 2 4
Using work energy theorem mu u
y-direction 0 + = 2mv y Þ v 'y =
'
Work done = Change in Kinetic energy 2 4
Let K be the Kinetic energy of combined system. é 2 2ù
1 1 æ uö æ uö
(m1 + m2 )gh K .E.i = m u 2 + m ê ç ÷ + ç ÷ ú
2 2 ëê è 2 ø è 2ø ú
û
1
= K - (m1 + m2 )V2
2 1 2 mu 2 3mu 2
= mu + =
1 2 4 4
Þ 2 ´ g ´1 = K - ´ 2 ´ 12 Þ K = 21 J
2 1
( ) 1
( )
2 2
63. (c) From conservation of linear momentum K .E . f = ( 2m) vx' + ( 2m) v 'y
ur 2 2
muiˆ + 0 = mvjˆ + 3mv ' 1 é æ 3u ö
2
æ uö ù 5
2

ur u = 2m êç ÷ + ç ÷ ú = mu 2
v
v ' = iˆ - ˆj
2 êëè 4 ø è 4ø ú 8
û
3 3 \ Loss in KE = KEf – KEi
v
y 2
m 3m æ 6 5 ö mu
3m = mu 2 ç - ÷ =
u m è 8 8ø 8
x v' 66. (a) For elastic collision KEi = KEf
Before After 1 1 1 1
collision collision m ´ 25 + ´ m ´ 9 = m ´ 32 + mvB2
From kinetic energy conservation, 2 2 2 2
æ æ u ö2 æ v ö2 ö 34 = 32 + VB2 Þ VB = 2
1 1 1
mu 2 = mv 2 + (3m) ç ç ÷ + ç ÷ ÷ 1 2 1 1
2 2 2 çè 3 ø è 3 ø ÷ KEB = mvB = ´ 0.1 ´ 2 = 0.1J = J
è ø
2 2 10
\x=1
Work, Energy & Power P-71

67. (d) Let t be the time taken by the particle dropped from Also
height h to collide with particle thrown upward. 2M(5sin45°) – M(10sin30°) = 2Mv1sin30° – Mv2sin45°
Using, v
5 2 - 5 = v1 - 2 ....(2)
2
h
4 Solving equation (1) and (2)
S1
( )
3 + 1 v1 = 5 3 + 10 2 - 5 Þ v1 = 6.5m/s

3h v2 = 6.3 m/s
S2 4 æ uö
70. (b) 2u + 0 = 2 ç ÷ + mv2
è 4ø
2
v2 –u2 = 2gh 1 1 æ uö 1
Þ v2 – 02 =2gh and ´ 2 ´ u 2 + 0 = ´ 2 ´ ç ÷ + mv22
2 2 è 4 ø 2
Þ v = 2 gh On solving, we get
Downward distance travelled m = 1.5 kg
71. (b) m (2v) + 2mv = 0 + 2mv’ cos 45°
1 2 1 h h
S1 = gt = g. = or v’ = 2 2v
2 2 2g 4
Distance of collision point from ground 72. (b) m1v1 + m2v2 = m1v2 + m2v1
h 3h or m1v1 + (0.5m1)v2 = m10.5v1 + (0.5m1)v4
s2 = h – =
4 4 On solving, v1 = v4 – v2
Speed of (A) just before collision 73. (d) Using conservation of momentum,
mv0 = mv2 – mv1
gh a V0 M
v1 = gt =
2
And speed of(B) just before collision After collision
a
gh M V2
v2 = 2 gh – V1
2
1 1
Using principle of conservation of linear momentum mv12 = 0.36 ´ mv02
2 2
mv1 + mv2 = 2mvf
Þ v1 = 0.6v0
æ gh ö gh The collision is elastic. So,
Þ v f = m ç 2 gh – –m =0
è 2 ÷ø 2 1 1
MV22 = 0.64´ mv20 [\ M = mass of nucleus]
2m 2 2
After collision, time taken (t1) for combined mass to m
Þ V2 = ´ 0.8V0
reach the ground is M
3h 1 2 mV0 = mM ´ 0.8V0 – m ´ 0.6V0
Þ = gt1
4 2 Þ1.6 m = 0.8 mM
3h Þ 4m 2 = mM
Þ t1 =
2g \ M = 4m
74. (c) 0.03 kg
68. (b) Pi = Pf
or 0 = 20(0.7 – v) = 50v 100 m
or v = 0.2 m/s 100 m/s
69. (a) Apply concervation of linear momentum in X and Y
direction for the system then 0.02 kg
Time taken for the particles to collide,
M (10cos30°) + 2M (5cos45°) = 2M (v1cos30°)
d 100
+ M(v2cos45°) t = V = 100 =1sec
rel

v Speed of wood just before collision = gt = 10 m/s and


5 3 + 5 2 = 3 v1 + 2 ....(1) speed of bullet just before collision = v – gt
2
P-72 Physics

= 100 – 10 = 90 m/s 77. (a) Change in momentum


j^
1
S = 100 × 1 – × 10 × 1 = 95 m
2 P ^J
Now, using conservation of linear momentum just before 2
P
and after the collision P^
45° 45° 2 i
– (0.03) (10) + (0.02) (90) = (0.05)v i^
P ^i
Þ 150 = 5v \ v = 30 m/s – P J^ 2
Max. height reached by body 2
P
30 ´ 30
h = = 2 ´ 10 = 45m P ˆ P ˆ P ˆ P ˆ
DP = J+ J+ i- i
Before After 2 2 2 2
0.03 kg 10 m/s v 2P ˆ
DP = J = IH molecule
2
2P ˆ
90 m/s Þ Iwall = - J
0.02 kg 0.05 kg 2
Pressure, P
\ Height above tower = 40 m
F 2P
75. (c) Kinetic energy of block A = = n (Q n = no.of particles)
A A
1
k1 = mv 20 2 ´ 3.32 ´ 10 -27 ´ 103 ´ 1023
2 = =2.35 × 103N/m2
\ From principle of linear momentum conservation 2 ´ 10 -4
mv0 78. (a) For collision of neutron with deuterium:
mv0 = ( 2m + M ) vf Þ v f = v v1 v2
2m + M
m 2m m 2m
5
According to question, of th the initial kinetic energy Applying conservation of momentum :
6 mv + 0 = mv1 + 2mv2 .....(i)
is lost in whole process. v2 – v1 = v .....(ii)
1 Q Collision is elastic, e = 1
mv02
2 v
k Þ =6 From eqn (i) and eqn (ii) v1 = -
\ i =6 1 æ mv ö
2 3
kf ( 2m + M ) çè 0
1 2 1
2 2m + M ÷ø mv - mv1 2
8
Pd = 2 2 = = 0.89
2m + M M 1 2 9
Þ =6\ =4 mv
m m 2
76. (b) Before Collision After Collision Now, For collision of neutron with carbon nucleus
V0 V1 V2 v v1 v2
m m Þ m m m 12m m 12m
Stationary Applying Conservation of momentum
1 2 1 2 3æ1 ö mv + 0 = mv1+ 12mv2 ....(iii)
mv + mv = ç mv20 ÷ v = v2 – v1 ....(iv)
2 1 2 2 2è2 ø
From eqn (iii) and eqn (iv)
3
Þ v12 + v 22 = v 20 ....(i) v1 = - v
11
2 13
From momentum conservation 2
1 1 æ 11 ö
mv0 = m(v1 + v2) ....(ii) mv2 - m ç v ÷
Squarring both sides, 2 2 è 13 ø 48
Pc = = » 0.28
(v1 + v2)2 = v02 1 169
mv2
Þ v12 + v22 + 2v1v2 = v02 2
v2 79. (d) Apply principle of conservation of momentum along
2v1v2 = - 0 x-direction,
2
mu = mv1 cos 45° + Mv2 cos 45°
3 2 v2
(v1 - v 2 ) 2 = v 21 + v 22 - 2v1v 2 = v0 + 0 1
2 2 mu = (mv1 + Mv2 ) ..... (i)
2
Solving we get relative velocity between the two particles
Along y-direction,
v1 - v2 = 2v0 o = mv1 sin 45° – Mv2 sin 45°
Work, Energy & Power P-73

1
mv 2 < 2´10.2 < 20.4 eV

1
,v
1

m
2
o = (mv1 – Mv2) ..... (ii)
2 90° Y
pf = 3 m V
m, u1 = u M, u2 = 0

M
Proton Unknown mass m pi
2v 45°

,v2
Before collision 82. (a) X
After collision
v2 - v1 cos90
Coefficient of restution e = 1 = v
u cos 45
2m
(Q Collision is elastic)
Initial momentum of the system
v2
Þ =1
u pi = [m(2V) 2 ´ 2m(2V) 2 ]
2 = 2m ´ 2V
Þ u = 2v2 ..... (iii) Final momentum of the system = 3mV
Solving eqs (i), (ii), & (iii), we get mass of unknown By the law of conservation of momentum
particle, M = m
2 2v
80. (a) For particle C, 2 2mv = 3mV Þ = Vcombined
According to law of conservation of linear momentum, 3
verticle component, Loss in energy
2 mv' sin q = mv sin 60° + mv sin 45° 1 1 1
DE = m1V12 + m2V22 - (m1 + m2 )Vcombined
2

mv mv 3 ...... (i) 2 2 2
2mv 'sin q = + 4 5
2 2 DE = 3mv2 - mv 2 = mv 2 = 55.55%
Horizontal component, 3 3
2 mv' cos q = mv sin 60° – mv cos 45° Percentage loss in energy during the collision ; 56%
mv mv 83. (b) Given, m1 = 4g, u1 = 300 m/s
2mv'cos q = + ...... (ii)
2 2 m2 = 0.8 kg = 800 g, u2 = 0 m/s
Y A From law of conservation of momentum,
Y'
B m1u1 + m2u2 = m1v1 + m2v2
v sin 60°

Let the velocity of combined system = v m/s


30° v sin 45° then,
4 × 300 + 800 × 0 = (800 + 4) × v
60° 45°
X X' 1200
v cos 60° – v cos 45° v= = 1.49 m / s
For particle A For particle B 804
Dividing eqn (i) by eqn (ii), Now, m = 0.3 (given)
a = mg
1 3 a = 0.3 × 10 (take g = 10 m/s2)
+
2 2+ 3 = 3 m/s2
tan q = 2 =
1 1
- 1 - 2 then, from v2 = u2 + 2as
2 2 (1.49)2 = 0 + 2 × 3 × s

81. (a) For inelastic collision v ' <


m1
v (1.49 )2
(m1 ∗ m 2 ) s=
6
1 v
< v< 2.22
(1 ∗ 1) 2 s=
6
n ↑ v(H) Before = 0.379 m
v 84. (d) From the law of conservation of momentum we know
(n)(H) ↑ After that,
2
m1u1 + m2u2 + .... = m1v1 + m2v2 + ....
1 1 æ v ö2 1
Loss in K.E. = mv 2 , (2m) çç ÷÷÷ < mv2 Given m1 = m, m2 = 2m and m3 = 3m
2 2 èç 2 ø 4 and u1 = 3u, u2 = 2u and u3 = u
K.E. lost is used to jump from 1st orbit to 2nd orbit ®
DK.E. = 10.2ev Let the velocity when they stick = v
Minimum K.E. of neutron for inelastic collision
P-74 Physics

Then, according to question, Initial momentum of the system block (C)


= mv0. After striking with A, the block C comes to rest
Y
and now both block A and B moves with velocity v when
2m, 2u
compression in spring is x0.
By the law of conservation of linear momentum
j sin 60°
j v0
60° mv0 = (m + 2m) v Þ v =
m, 3u X 3
P 60° i i cos 60° By the law of conservation of energy
(–j) sin 60° K.E. of block C = K.E. of system + P.E. of system
2
–3m, u 1 2 1 æv ö 1
mv0 = ( 3m ) ç 0 ÷ + kx02
() (
m × 3u î + 2m × 2u -ˆi cos 60° - ˆjsin 60° ) 2 2 è 3ø 2
1 2 1 2 1 2
( )
®
+ 3 m × u -ˆi cos 60° + ˆjsin 60° = (m + 2m + 3m) v Þ mv0 = mv0 + kx0
2 6 2
ˆ æ ö ˆ
Þ 3muiˆ - 4mu i - 4mu ç 3 ˆj÷ - 3mu i
2
1 2 1 2 1 2 mv0
2 è 2 ø 2 Þ kx0 = mv0 - mv0 =
2 2 6 3
æ 3 ˆö ®
2
+ 3mu çç j ÷÷ = 6m v 2 æv ö
è 2 ø \ k = mç 0 ÷
® 3 è x0 ø
3 3 ˆ
Þ muiˆ - muiˆ - muj = 6m v 89. (c) Y
2 2 m/2, + v = 400 m/s
1 3 ˆ ®
Þ - muiˆ - muj = 6m v
2 2 Mass before explosion = m
® 490 m and velocity v = 200 m/s (vertically)
Þ v=
u ˆ
12
(
- i - 3jˆ )
P2 P2
85. (d) Maximum energy loss = -
2m 2(m + M) X
O
é P2 1 2 ù Momentum before explosion = Momentum after explosion
êQ K.E. = = mv ú m m
êë 2m 2 úû m ´ 200 ˆj = ´ 400 ˆj + v
2 2

=
P2 é M ù 1
ê ú = mv 2 í
ì M ü
ý
=
m
2
(
ˆ
400 j + v )
2m ë (m + M) û 2 îm + M þ
Statement II is a case of perfectly inelastic collision. Þ 400 ˆj - 400 ˆj = v
By comparing the equation given in statement I with above \ v=0
equation, we get\ i.e., the velocity of the other part of the mass, v = 0
Let time taken to reach the earth by this part be t
æ M ö æ m ö 1
f =ç Applying formula, h = ut + gt 2
è m + M ÷ø instead of çè M + m ÷ø 2
Hence statement I is wrong and statement II is correct. 1
490 = 0 + × 9.8 × t2
86. (c) 2
87. (c) Fractional decrease in kinetic energy of mass m 2 980
2 2
Þ t = = 100
æ m2 - m1 ö æ 2 - 1ö 9.8
= 1- ç = 1- ç
è m2 + m1 ÷ø è 2 + 1÷ø \ t = 100 = 10sec
90. (a) In completely inelastic collision, all initial kinetic
2
æ 1ö 1 8 energy is not lost but loss in kinetic energy 15 as large
= 1 – ç ÷ = 1- =
è 3ø 9 9 as it can be. Linear momentum remain conserved in all
Percentage loss in energy types of collision. Statement -2 explains statement -1
8 correctly because applying the principle of conservation
= ´ 100 ; 90% of momentum, we can get the common velocity and hence
9
C A B the kinetic energy of the combined body.
88. (d) 91. (c) Initial kinetic energy of the system
m m 2m 1 2 1
K.Ei = mu + M (0) 2
2 2
v0
Work, Energy & Power P-75

1 Momentum of the block, = M × v


= ´ 0.5 ´ 2 ´ 2 + 0 = 1J
2 k
Momentum before collision =M× × L = kM × L
M
= Momentum after collision 94. (d) Considering conservation of momentum along x-di-
m1u1 + m2u2 = (m + M) × v rection,
2 mv = mv1 cos q ...(1)
\ 0.5 × 2 + 1 × 0 = (0.5 + 1) × v Þ v = m/s
where v1 is the velocity of second mass
3
Final kinetic energy of the system is In y-direction,
1 mv
K.E f = (m + M )v 2 0= - mv1 sin q
2 3
1 2 2 1 mv
= (0.5 + 1) ´ ´ = J or m1v1 sin q = ...(2)
2 3 3 3 3
\ Energy loss during collision
v/ 3
æ 1ö
= ç1 - ÷ J = 0.67J
è 3ø v
92. (b) Let the velocity and mass of 4 kg piece be v1 and m1 m v
v1 cosq
and that of 12 kg piece be v2 and m2. q
16 kg
v1
Initial momentum
=0 v1 sinq
Situation 1
Squaring and adding eqns. (1) and (2) we get
m2 = 12 kg Final momentum
4 kg = m1
v2 2
v1 v2 = m2v2 – m1v1 v12 = v2 + Þ v1 = v
Situation 2
3 3
Applying conservation of linear momentum 95. (c) Kinetic energy of a system of particle is zero only
16 × 0 = 4 × v1 + 12 × 4 when the speed of each particles is zero. This implies
12 ´ 4 momentum of each particle is zero, thus linear momentum
Þ v1 = – = –12 ms -1 of the system of particle has to be zero.
4 Also if linear momentum of the system is zero it does not
Kinetic energy of 4 kg mass mean linear momentum of each particle is zero. This is because
1 1 linear momentum is a vector quantity. In this case the kinetic
\ K . E. = m1v12 = ´ 4 ´144 = 288 J
2 2 energy of the system of particles will not be zero.
93. (b) When the spring gets compressed by length L. \ A does not imply B but B implies A.
Given, force, F = 200 N extension of wire, x = 1mm.
K.E. lost by mass M = P.E. stored in the compressed spring.
1 1
Mv 2 = k L2
2 2
M
k
Þ v= ×L
M
6
P-76 Physics

System of Particles and


Rotational Motion
Centre of Mass, Centre of
TOPIC 1 Gravity & Principle of
Moments
1. The centre of mass of a solid hemisphere of radius 8 cm is
x cm from the centre of the flat surface. Then value of x is
______. [NA Sep. 06, 2020 (II)]
a
2. A square shaped hole of side l = is carved out at a
2
a (a) (1.25 m, 1.50 m) (b) (0.75 m, 1.75 m)
distance d = from the centre ‘O’ of a uniform circular
2 (c) (0.75 m, 0.75 m) (d) (1 m, 1.75 m)
disk of radius a. If the distance of the centre of mass of the 5. As shown in fig. when a spherical cavity (centred at O) of
a radius 1 is cut out of a uniform sphere of radius R (centred
remaining portion from O is - , value of X (to the nearest
X at C), the centre of mass of remaining (shaded) part of
integer) is ___________. [NA Sep. 02, 2020 (II)] sphere is at G, i.e on the surface of the cavity. R can be
determined by the equation: [8 Jan. 2020 II]
(a) (R2 + R + 1) (2 – R) = 1

a (b) (R2 – R – l) (2 – R) = 1

O
(c) (R2 – R + l) (2 – R) = l
d
l = a/2 (d) (R2 + R – 1) (2 – R) = 1
6. Three point particles of masses 1.0 kg, 1.5 kg and 2.5 kg
are placed at three corners of a right angle triangle of
3. A rod of length L has non-uniform linear mass density
sides 4.0 cm, 3.0 cm and 5.0 cm as shown in the figure.
2 The center of mass of the system is at a point:
æ xö
given by r(x) = a + b ç ÷ , where a and b are constants [7 Jan. 2020 I]
èLø
and 0 £ x £ L. The value of x for the centre of mass of the
rod is at: [9 Jan. 2020 II]
3æ a +b ö 3 æ 2a + b ö
(a) ç ÷L (b) ç ÷L
2 è 2a + b ø 4 è 3a + b ø
4æ a +b ö 3 æ 2a + b ö
(c) ç ÷L (d) ç ÷L
3 è 2a + 3b ø 2 è 3a + b ø (a) 0.6 cm right and 2.0 cm above 1 kg mass
4. The coordinates of centre of mass of a uniform flag shaped (b) 1.5 cm right and 1.2 cm above 1 kg mass
lamina (thin flat plale) of mass 4 kg. (The coordinates of (c) 2.0 cm right and 0.9 cm above 1 kg mass
the same are shown in figure) are: [8 Jan. 2020 I] (d) 0.9 cm right and 2.0 cm above 1 kg mass
System of Particles and Rotational Motion P-77

7. Three particles of masses 50 g, 100 g and 150 g are placed 10. The position vector of the centre of mass r cm of an
at the vertices of an equilateral triangle of side 1 m (as asymmetric uniform bar of negligible area of cross-
shown in the figure). The (x, y) coordinates of the centre of
section as shown in figure is: [12 Jan. 2019 I]
mass will be : [12 Apr. 2019 II]

L 2L 3L
æ 3 5 ö æ 7 3 ö r 13 5 r 5 13
(a) çç 4 m, 12 m ÷÷ (b) çç 12 m, 8 m ÷÷ (a) rcm = L xˆ + L yˆ (b) rcm = L xˆ + L yˆ
è ø è ø 8 8 8 8
æ 7 3 ö æ 3 7 ö r 3 11 r 11 3
(c) çç 12 m, 4 m ÷÷ (d) çç 8 m, 12 m ÷÷ (c) rcm = L xˆ + L yˆ (d) rcm = L xˆ + L yˆ
è ø è ø 8 8 8 8
8. Four particles A, B, C and D with masses mA = m, mB = 11. A force of 40 N acts on a point B at the end of an L-shaped
2m, mC = 3m and mD = 4m are at the corners of a square. object, as shown in the figure. The angle q that will produce
maximum moment of the force about point A is given by:
They have accelerations of equal magnitude with
[Online April 15, 2018]
directions as shown. The acceleration of the centre of 1
A
mass of the particles is : [8 April 2019 I] (a) tan q =
4
(b) tan q = 2
4m
1
(c) tan q = ®
F
2 q
(d) tan q = 4 2m
B

12. In a physical balance working on the principle of moments,


when 5 mg weight is placed on the left pan, the beam
becomes horizontal. Both the empty pans of the balance
(a)
5
( )
a $ $
i– j (b) a
are of equal mass. Which of the following statements is
correct ? [Online April 8, 2017]
(c) Zero (d)
5
( )
a $ $
i+ j (a) Left arm is longer than the right arm
(b) Both the arms are of same length
9. A uniform rectangular thin sheet ABCD of mass M has
(c) Left arm is shorter than the right arm
length a and breadth b, as shown in the figure. If the shaded
portion HBGO is cut-off, the coordinates of the centre of (d) Every object that is weighed using this balance
mass of the remaining portion will be : [8 Apr. 2019 II] appears lighter than its actual weight.
13. In the figure shown ABC is a uniform wire. If centre of
BC
mass of wire lies vertically below point A, then is
AB
close to : [Online April 10, 2016]

æ 3a 3b ö æ 5a 5b ö
(a) ç , ÷ (b) ç , ÷ 60°
è 4 4 ø è 3 3 ø B C
æ 2a 2b ö
(d) æç , ö÷
5a 5b (a) 1.85 (b) 1.5
(c) ç , ÷ (c) 1.37 (d) 3
è 3 3 ø è 12 12 ø
P-78 Physics

14. Distance of the centre of mass of a solid uniform cone 20. Consider a two particle system with particles having masses
from its vertex is z0. If the radius of its base is R and its m1 and m2. If the first particle is pushed towards the centre
height is h then z0 is equal to : [2015]
of mass through a distance d, by what distance should the
5h 3h 2 h2 3h second particle is moved, so as to keep the centre of mass
(a) (b) (c) (d)
8 8R 4R 4 at the same position? [2006]
15. A uniform thin rod AB of length L has linear mass m2 m1
bx (a) m d (b) m + m d
density m (x) = a + , where x is measured from A. If 1 1 2
L
æ 7ö m1
the CM of the rod lies at a distance of ç ÷ L from A, (c) d (d) d
è 12 ø m2
then a and b are related as : [Online April 11, 2015]
21. A body A of mass M while falling vertically downwards
(a) a = 2b (b) 2a = b
1
(c) a = b (d) 3a = 2b under gravity breaks into two parts; a body B of mass
16. A thin bar of length L has a mass per unit length l, that 3
2
increases linearly with distance from one end. If its total M and a body C of mass M. The centre of mass of
3
mass is M and its mass per unit length at the lighter end is bodies B and C taken together shifts compared to that of
lO, then the distance of the centre of mass from the lighter body A towards [2005]
end is: [Online April 11, 2014] (a) does not shift
2
L l o L2 (b) depends on height of breaking
L loL
(a) - (b) + (c) body B
2 4M 3 8M
(d) body C
L l o L2 2L l o L2 22. A ‘T’ shaped object with dimensions shown in the figure,
(c) + (d) -
3 4M 3 6M r
is lying on a smooth floor. A force ‘ F ’ is applied at the
17. A boy of mass 20 kg is standing on a 80 kg free to move
long cart. There is negligible friction between cart and point P parallel to AB, such that the object has only the
ground. Initially, the boy is standing 25 m from a wall. If he translational motion without rotation. Find the location of
walks 10 m on the cart towards the wall, then the final P with respect to C. [2005]
distance of the boy from the wall will be l
A B
[Online April 23, 2013]
(a) 15 m (b) 12.5 m (c) 15.5 m (d) 17 m P
18. A thin rod of length ‘L’ is lying along the x-axis with its F
2l
ends at x = 0 and x = L. Its linear density (mass/length)
n
æ xö
varies with x as k ç ÷ , where n can be zero or any
è Lø C
positive number. If the position xCM of the centre of mass 3 2 4
of the rod is plotted against ‘n’, which of the following (a) l (b) l (c) l (d) l
graphs best approximates the dependence of xCM on n? 2 3 3
[2008] Angular Displacement,
xCM xCM TOPIC 2
Velocity and Aceleration
L 23. A bead of mass m stays at point P(a, b) on a wire bent in
L L
(a) (b) the shape of a parabola y = 4Cx2 and rotating with angular
2 2
n n speed w (see figure). The value of w is (neglect friction) :
O O
[Sep. 02, 2020 (I)]
xCM xCM
y
L L w
L L
(c) 2
(d) 2 P (a, b)
n n
O O
19. A circular disc of radius R is removed from a bigger circular x
O
disc of radius 2R such that the circumferences of the discs
coincide. The centre of mass of the new disc is
(a) 2 2gC (b) 2 gC
a/R form the centre of the bigger disc. The value of a is
[2007] 2gC 2g
(a) 1/4 (b) 1/3 (c) 1/2 (d) 1/6 (c) (d)
ab C
System of Particles and Rotational Motion P-79

24. A cylindrical vessel containing a liquid is rotated about


its axis so that the liquid rises at its sides as shown in the
figure. The radius of vessel is 5 cm and the angular speed
of rotation is w rad s –1. The difference in the height,
h (in cm) of liquid at the centre of vessel and at the side (a)
will be : [Sep. 02, 2020 (I)]
w

(b)
h

10 cm

2w2 5w2 (c)


(a) (b)
25g 2g

25w2 2w2
(c) (d)
2g 5g
25. A spring mass system (mass m, spring constant k and
natural length l) rests in equilibrium on a horizontal disc.
(d)
The free end of the spring is fixed at the centre of the disc.
If the disc together with spring mass system, rotates about
it’s axis with an angular velocity w, (k >> mw2) the relative
28. A smooth wire of length 2pr is bent into a circle and kept
change in the length of the spring is best given by the
in a vertical plane. A bead can slide smoothly on the wire.
option: [9 Jan. 2020 II]
When the circle is rotating with angular speed w about
2 æ mw2 ö 2mw2 the vertical diameter AB, as shown in figure, the bead is
(a) ç ÷ (b)
3 çè k ÷ø k at rest with respect to the circular ring at position P as
shown. Then the value of w2 is equal to :
mw2 mw2 [12 Apr. 2019 II]
(c) (d)
k 3k
26. A particle of mass m is fixed to one end of a light spring
having force constant k and unstretched length l. The other
end is fixed. The system is given an angular speed w about
the fixed end of the spring such that it rotates in a circle in
gravity free space. Then the stretch in the spring is:
[8 Jan. 2020 I]

ml w2 ml w 2
(a) (b) 3g
k - wm k - mw 2 (a) (b) 2 g / (r 3)
2r
ml w 2 ml w 2 (c) ( g 3) / r (d) 2g/r
(c) (d)
k + mw 2
k + mw 29. A long cylindrical vessel is half filled with a liquid. When
the vessel is rotated about its own vertical axis, the liquid
27. A uniform rod of length l is being rotated in a horizontal
rises up near the wall. If the radius of vessel is 5 cm and its
plane with a constant angular speed about an axis passing
rotational speed is 2 rotations per second, then the
through one of its ends. If the tension generated in the rod
difference in the heights between the centre and the sides, in
due to rotation is T(x) at a distance x from the axis, then
cm, will be : [12 Jan. 2019 II]
which of the following graphs depicts it most closely?
(a) 2.0 (b) 0.1
[12 Apr. 2019 II] (c) 0.4 (d) 1.2
P-80 Physics

30. A particle is moving with a uniform speed in a circular 34. A cubical block of side 30 cm is moving with velocity
orbit of radius R in a central force inversely proportional 2 ms–1 on a smooth horizontal surface. The surface has a
to the nth power of R. If the period of rotation of the bump at a point O as shown in figure. The angular velocity
particle is T, then: [2018] (in rad/s) of the block immediately after it hits the bump,
(a) T µ R3/2 for any n. (b) T µ R n /2+1 is : [Online April 9, 2016]
(c) T µ R (n +1)/2 (d) T µ R n /2 a = 30 cm
31. The machine as shown has 2 rods of length 1 m connected
by a pivot at the top. The end of one rod is connected to
the floor by a stationary pivot and the end of the other rod O
has a roller that rolls along the floor in a slot. (a) 13.3 (b) 5.0 (c) 9.4 (d) 6.7
As the roller goes back and forth, a 2 kg weight moves up 35. Two point masses of mass m1 = fM and m2 = (1 – f) M (f
and down. If the roller is moving towards right at a constant < 1) are in outer space (far from gravitational influence of
speed, the weight moves up with a : [Online April 9, 2017] other objects) at a distance R from each other. They move
in circular orbits about their centre of mass with angular
2 kg
velocities w1 for m1 and w2 for m2. In that case
[Online May 19, 2012]
(a) (1 – f) w1 = fw
(b) w1 = w2 and independent of f
(c) fw1 = (1 – f)w2
Fixed pivot
F
x (d) w1 = w2 and depend on f
Movable roller
(a) constant speed Torque, Couple and
(b) decreasing speed TOPIC 3
Angular Momentum
(c) increasing speed
3 36. Four point masses, each of mass m, are fixed at the corners
(d) speed which is th of that of the roller when the
4 of a square of side l. The square is rotating with angular
weight is 0.4 m above the ground frequency w, about an axis passing through one of the
32. A slender uniform rod of mass M and length l is pivoted corners of the square and parallel to its diagonal, as
at one end so that it can rotate in a vertical plane (see shown in the figure. The angular momentum of the square
figure). There is negligible friction at the pivot. The free about this axis is : [Sep. 06, 2020 (I)]
end is held vertically above the pivot and then released.
The angular acceleration of the rod when it makes an angle
q with the vertical is [2017]
is
ax

(a) ml 2w (b) 4 ml2w


(c) 3 ml2w (d) 2 ml2w
3g 2g 37. A thin rod of mass 0.9 kg and length 1 m is suspended, at
(a) cos q (b) cos q
2l 3l rest, from one end so that it can freely oscillate in the
vertical plane. A particle of move 0.1 kg moving in a straight
3g 2g
(c) sin q (d) sin q line with velocity 80 m/s hits the rod at its bottom most
2l 2l point and sticks to it (see figure). The angular speed
33. Concrete mixture is made by mixing cement, stone and (in rad/s) of the rod immediately after the collision will be
sand in a rotating cylindrical drum. If the drum rotates too ______________. [NA Sep. 05, 2020 (II)]
fast, the ingredients remain stuck to the wall of the drum 38. A person of 80 kg mass is standing on the rim of a circular
and proper mixing of ingredients does not take place. The platform of mass 200 kg rotating about its axis at 5
maximum rotational speed of the drum in revolutions per revolutions per minute (rpm). The person now starts
minute (rpm) to ensure proper mixing is close to : moving towards the centre of the platform. What will be
(Take the radius of the drum to be 1.25 m and its axle to the rotational speed (in rpm) of the platform when the
be horizontal): [Online April 10, 2016] person reaches its centre __________.
(a) 27.0 (b) 0.4 (c) 1.3 (d) 8.0
[NA Sep. 03, 2020 (I)]
System of Particles and Rotational Motion P-81

39. A block of mass m = 1 kg slides with velocity v = 6 m/s 42. A uniform cylinder of mass M and radius R is to be pulled
on a frictionless horizontal surface and collides with a over a step of height a (a < R) by applying a force F at its
uniform vertical rod and sticks to it as shown. The rod is centre 'O' perpendicular to the plane through the axes of
pivoted about O and swings as a result of the collision the cylinder on the edge of the step (see figure). The
making angle q before momentarily coming to rest. If minimum value of F required is : [Sep. 02, 2020 (I)]
the rod has mass M = 2 kg, and length l = 1 m, the value F
of q is approximately: (take g = 10 m/s2)
[Sep. 03, 2020 (I)]
O
O R
M, l
a

q 2 2
m v m æ R-aö æ R ö
m (a) Mg 1 - ç ÷ (b) Mg ç ÷ -1
è R ø è R-aø
(a) 63° (b) 55° a a2
(c) 69° (d) 49° (c) Mg (d) Mg 1 - 2
R R
w 43. Consider a uniform rod of mass M = 4m and length l pivoted
40. about its centre. A mass m moving with velocity v making
FV
p
angle q = to the rod’s long axis collides with one end of
FH l 4
the rod and sticks to it. The angular speed of the rod-mass
q system just after the collision is:
[8 Jan. 2020 I]
3 v 3v
(a) (b)
A uniform rod of length 'l' is pivoted at one of its ends on 7 2l 7l
a vertical shaft of negligible radius. When the shaft rotates 3 2v 4v
(c) (d)
at angular speed w the rod makes an angle q with it (see 7 l 7l
figure). To find q equate the rate of change of angular 44. A particle of mass m is moving along a trajectory given by
momentum (direction going into the paper) x = x0 + a cos w1t
ml 2 2 y = y0 + b cos w2 t
w sin q cos q about the centre of mass (CM) to the The torque, acting on the particle about the origin, at t = 0
12 is: [10 Apr. 2019 I]
torque provided by the horizontal and vertical forces FH 2$
2$ +
and FV about the CM. The value of q is then such that : (a) m ( - x0 b + y0 a ) w1 k (b) my 0 w1 k
a
2 2 $
[Sep. 03, 2020 (II)] (c) zero (d) -m( x0 bw2 - y0 aw1 )k
2g g 45. The time dependence of the position of a particle of mass
(a) cos q = (b) cos q = r
3l w2 2l w2 m = 2 is given by r (t ) = 2 t i$ - 3t 2 $j . Its angular momentum,
with respect to the origin, at time t = 2 is :
g 3g [10 Apr. 2019 II]
(c) cos q = (d) cos q =
( )
2
lw 2l w2 $ $
(a) 48 i + j (b) 36k$
41.
(
(c) -34 k$ - $i ) (d) -48k$
0 25 50 75 100 46. A metal coin of mass 5 g and radius 1 cm is fixed to a thin
A B stick AB of negligible mass as shown in the figure The
2m system is initially at rest. The constant torque, that will
Shown in the figure is rigid and uniform one meter long make the system rotate about AB at 25 rotations per second
rod AB held in horizontal position by two strings tied to in 5s, is close to : [10 Apr. 2019 II]
its ends and attached to the ceiling. The rod is of mass 'm'
and has another weight of mass 2 m hung at a distance of
75 cm from A. The tension in the string at A is :
[Sep. 02, 2020 (I)]
(a) 0.5 mg (b) 2 mg
(c) 0.75 mg (d) 1 mg
P-82 Physics

(a) 4.0×10–6 Nm (b) 1.6×10–5 Nm 50. The magnitude of torque on a particle of mass 1 kg is 2.5
(c) 7.9×10–6 Nm (d) 2.0×10–5 Nm Nm about the origin. If the force acting on it is 1 N, and the
47. A rectangular solid box of length 0.3 m is held distance of the particle from the origin is 5m, the angle
horizontally, with one of its sides on the edge of a between the force and the position vector is (in radians):
platform of height 5m. When released, it slips off the [11 Jan. 2019 II]
table in a very short time t = 0.01 s, remaining essentially p p p p
horizontal. The angle by which it would rotate when it (a) (b) (c) (d)
6 3 8 4
hits the ground will be (in radians) close to :
51. To mop-clean a floor, a cleaning machine presses a
[8 Apr. 2019 II] circular mop of radius R vertically down with a total
force F and rotates it with a constant angular speed
about its axis. If the force F is distributed uniformly
over the mop and if coefficient of friction between the
mop and the floor is m, the torque, applied by the
machine on the mop is: [10 Jan. 2019 I]
(a) m FR/3 (b) m FR/6
2
(c) m FR/2 (d) μ mFR
3
(a) 0.5 (b) 0.3 (c) 0.02 (d) 0.28 52. A rigid massless rod of length 3l has two masses attached
48. A particle of mass 20 g is released with an initial velocity at each end as shown in the figure. The rod is pivoted at
5 m/s along the curve from the point A, as shown in the point P on the horizontal axis (see figure). When released
figure. The point A is at height h from point B. The particle from initial horizontal position, its instantaneous angular
acceleration will be: [10 Jan. 2019 II]
slides along the frictionless surface. When the particle
l 2l
reaches point B, its angular momentum about O will be :
(Take g = 10 m/s2) [12 Jan. 2019 II] 5 Mo P 2 Mo
O g g
(a) (b)
13l 3l
a = 10 m
g 7g
(c) (d)
2l 3l
A 53. An L-shaped object, made of thin rods of uniform mass
h = 10 m density, is suspended with a string as shown in figure. If
AB = BC, and the angle made by AB with downward
B vertical is q, then: [9 Jan. 2019 I]
(a) 2 kg-m2/s (b) 8 kg-m2/s
(c) 6 kg-m2/s (d) 3 kg-m2/s
uur uur
49. A slab is subjected to two forces F1 and F2 of same
uur
magnitude F as shown in the figure. Force F2 is in XY-
plane while force F1 acts along z-axis at the point
r r
( )
2i + 3 j . The moment of these forces about point O will
be : [11 Jan. 2019 I]
Z
F1 1 1
(a) tan q = (b) tan q =
F2 2 3 2
O y
30º
2 1
4m
(c) tan q = (d) tan q =
3 3
6m
54. A B
x
(a) ( 3$i - 2 $j + 3k$ ) F (b) ( 3$i - 2 $j – 3k$ ) F x

(c) ( 3$i + 2 $j – 3k$ ) F (d) ( 3$i + 2 $j + 3k$ ) F


System of Particles and Rotational Motion P-83

A uniform rod AB is suspended from a point X, at a variable 58. A bob of mass m attached to an inextensible string of
distance from x from A, as shown. To make the rod length l is suspended from a vertical support. The bob
horizontal, a mass m is suspended from its end A. A set of rotates in a horizontal circle with an angular speed w rad/s
(m, x) values is recorded. The appropriate variable that about the vertical. About the point of suspension: [2014]
give a straight line, when plotted, are: (a) angular momentum is conserved.
[Online April 15, 2018] (b) angular momentum changes in magnitude but not in
1 1 direction.
(a) m, (b) m, 2 (c) m, x (d) m, x2 (c) angular momentum changes in direction but not in
x x
55. A thin uniform bar of length L and mass 8m lies on a smooth magnitude.
horizontal table. Two point masses m and 2m moving in (d) angular momentum changes both in direction and
the same horizontal plane from opposite sides of the bar magnitude.
with speeds 2v and v respectively. The masses stick to the 59. A ball of mass 160 g is th rown up at an an gle
L L of 60° to the horizontal at a speed of 10 ms–1. The angular
bar after collision at a distance and respectively momentum of the ball at the highest point of the trajectory
3 6
from the centre of the bar. If the bar starts rotating about with respect to the point from which the ball is thrown is
its center of mass as a result of collision, the angular speed nearly (g = 10 ms–2) [Online April 19, 2014]
of the bar will be: [Online April 15, 2018] (a) 1.73 kg m2/s (b) 3.0 kg m2/s
L/6 L/3 2v
(c) 3.46 kg m2/s (d) 6.0 kg m2/s
60. A particle is moving in a circular path of radius a, with a
constant velocity v as shown in the figure. The centre of
v
O circle is marked by ‘C’. The angular momentum from the
origin O can be written as: [Online April 12, 2014]
v 6v 3v v y
(a) (b) (c) (d)
6L 5L 5L 5L
a
56. A particle of mass m is moving along the side of a square O q
of side 'a', with a uniform speed v in the x-y plane as shown C x
in the figure : [2016]
y
(a) va (1 + cos 2q) (b) va (1 + cos q)
D a C (c) va cos 2q (d) va
V 61. A particle of mass 2 kg is moving such that at time t, its
a V V a r
A V position, in meter, is given by r (t ) = 5iˆ - 2t 2 ˆj . The angular
a B
V momentum of the particle at t = 2s about the origin in
45° R a
kg m–2 s–1 is : [Online April 23, 2013]
O
Which of the following statements is false for the angular (a) - 80 k$ (b) (10iˆ - 16 ˆj )
ur
momentum L about the origin? (c) -40k$ (d) 40k$
ur éR ù 62. A bullet of mass 10 g and speed 500 m/s is fired into a door
(a) L = mv ê + a ú k$ when the particle is moving from and gets embedded exactly at the centre of the door. The
ë 2 û door is 1.0 m wide and weighs 12 kg. It is hinged at one end
B to C.
and rotates about a vertical axis practically without friction.
ur mv
(b) L = Rk$ when the particle is moving from D to A. The angular speed of the door just after the bullet embeds
2 into it will be : [Online April 9, 2013]
ur mv $ (a) 6.25 rad/sec (b) 0.625 rad/sec
(c) L = – Rk when the particle is moving from A to B. (c) 3.35 rad/sec (d) 0.335 rad/sec
2
63. A stone of mass m, tied to the end of a string, is whirled
ur éR ù
(d) L = mv ê - a ú k$ when the particle is moving from around in a circle on a horizontal frictionless table. The
ë 2 û length of the string is reduced gradually keeping the
C to D. angular momentum of the stone about the centre of the
57. A particle of mass 2 kg is on a smooth horizontal table circle constant. Then, the tension in the string is given by
and moves in a circular path of radius 0.6 m. The height T = Arn, where A is a constant, r is the instantaneous
of the table from the ground is 0.8 m. If the angular speed radius of the circle. The value of n is equal to
of the particle is 12 rad s–1, the magnitude of its angular [Online May 26, 2012]
momentum about a point on the ground right under the (a) – 1 (b) – 2 (c) – 4 (d) – 3
centre of the circle is : [Online April 11, 2015] 64. A thin horizontal circular disc is rotating about a vertical
(a) 14.4 kg m2s–1 (b) 8.64 kg m2s–1 axis passing through its centre. An insect is at rest at a
(c) 20.16 kg m2s–1 (d) 11.52 kg m2s–1
P-84 Physics

point near the rim of the disc. The insect now moves along (a) Angular velocity
a diameter of the disc to reach its other end. During the (b) Angular momentum
journey of the insect, the angular speed of the disc.[2011] (c) Moment of inertia
(a) continuously decreases (d) Rotational
r kinetic energy
(b) continuously increases 70. Let F be the force
r acting on a particle having position
r
(c) first increases and then decreases vector r , and T be the torque of this force about the
(d) remains unchanged origin. Then [2003]
65. A small particle of mass m is projected at an angle q with r r r r
(a) r .Tr = 0 and Fr .Tr ¹ 0
the x-axis with an initial velocity n0 in the x-y plane as r
(b) r .T ¹ 0 and F .T = 0
n sin q r r r r
(c) r .T ¹ 0 and F .T ¹ 0
shown in the figure. At a time t < 0 , the angular r r r r
g (d) r .T = 0 and F .T = 0
momentum of the particle is [2010] 71. A particle of mass m moves along line PC with velocity v
y as shown. What is the angular momentum of the particle
about P? [2002]
v0

C
L
q
x P
l
(a) -mg n0t 2 cos q ˆj (b) mg n 0t cos q kˆ O
(a) mvL (b) mvl (c) mvr (d) zero.
1 1
(c) - mg n 0 t 2 cos q kˆ (d) mgn 0 t 2 cos q iˆ
2 2
Moment of Inertia and
where iˆ, ˆj and k̂ are unit vectors along x, y and z-axis TOPIC 4
Rotational K.E.
respectively.
66. Angular momentum of the particle rotating with a central 72. Shown in the figure is a hollow icecream cone (it is open at
force is constant due to [2007] the top). If its mass is M, radius of its top, R and height, H,
(a) constant torque then its moment of inertia about its axis is :
(b) constant force R [Sep. 06, 2020 (I)]
(c) constant linear momentum
(d) zero torque
67. A thin circular ring of mass m and radius R is rotating H
about its axis with a constant angular velocity w. Two
objects each of mass M are attached gently to the
opposite ends of a diameter of the ring. The ring now
rotates with an angular velocity w' = [2006] MR 2 M (R2 + H 2 )
(a) (b)
w (m + 2 M ) w(m - 2M ) 2 4
(a) (b) MH 2 MR 2
m (m + 2M ) (c) (d)
wm wm 3 3
(c) (d) 73. The linear mass density of a thin rod AB of length L varies
(m + M ) (m + 2M )
æ xö
68. A force of – Fkˆ acts on O, the origin of the coordinate from A to B as l ( x ) = l 0 ç1 + ÷ , where x is the distance
è Lø
system. The torque about the point (1, –1) is [2006] from A. If M is the mass of the rod then its moment of inertia
Z
about an axis passing through A and perpendicular to the
rod is : [Sep. 06, 2020 (II)]
O
Y 5 7
(a) ML2 (b) ML2
12 18
X
2 3
(a) F (iˆ - ˆj ) (b) - F (iˆ + ˆj ) (c) ML2 (d) ML2
5 7
ˆ ˆ
(c) F (i + j ) (d) - F (iˆ - ˆj ) 74. A wheel is rotating freely with an angular speed w on a
69. A solid sphere is rotating in free space. If the radius of shaft. The moment of inertia of the wheel is I and the mo-
the sphere is increased keeping mass same, which one ment of inertia of the shaft is negligible. Another wheel
of the following will not be affected ? [2004]
System of Particles and Rotational Motion P-85

of moment of inertia 3I initially at rest is suddenly coupled 79. Moment of inertia of a cylinder of mass M, length L and
to the same shaft. The resultant fractional loss in the radius R about an axis passing through its centre and
kinetic energy of the system is : [Sep. 05, 2020 (I)] perpendicular to the axis of the cylinder is
5 1 æ R 2 L2 ö
(a) (b) I = M çç + ÷÷ . If such a cylinder is to be made for a
6 4
è 4 12 ø
given mass of a material, the ratio L/R for it to have minimum
3
(c) 0 (d) possible I is : [Sep. 03, 2020 (I)]
4
2 3
75. ABC is a plane lamina of the shape of an equilateral (a) (b)
triangle. D, E are mid points of AB, AC and G is the 3 2
centroid of the lamina. Moment of inertia of the lamina 3 2
about an axis passing through G and perpendicular to the (c) (d)
2 3
plane ABC is I0. If part ADE is removed, the moment of 80. An massless equilateral triangle EFG of side 'a' (As shown
NI0 in figure) has three particles of mass m situated at its
inertia of the remaining part about the same axis is vertices. The moment of inertia of the system about the
16
where N is an integer. Value of N is _______. N
line EX perpendicular to EG in the plane of EFG is ma 2
[NA Sep. 04, 2020 (I)] 20
where N is an integer. The value of N is _________.
A [Sep. 03, 2020 (II)]
X
E F
D
G

B C
76. A circular disc of mass M and radius R is rotating about E a G
its axis with angular speed w1. If another stationary disc 81. Two uniform circular discs are rotating independently in
R the same direction around their common axis passing
having radius and same mass M is dropped co-axially through their centres. The moment of inertia and angular
2
on to the rotating disc. Gradually both discs attain velocity of the first disc are 0.1 kg-m2 and 10 rad s–1
constant angular speed w2. The energy lost in the process respectively while those for the second one are 0.2 kg-m 2
is p% of the initial energy. Value of p is ___________. and 5 rad s–1 respectively. At some instant they get stuck
together and start rotating as a single system about their
[NA Sep. 04, 2020 (I)]
common axis with some angular speed. The kinetic energy
77. Consider two uniform discs of the same thickness and of the combined system is : [Sep. 02, 2020 (II)]
different radii R1 = R and R2 = aR made of the same material.
10 20
If the ratio of their moments of inertia I1 and I2, respectively, (a) J (b) J
about their axes is I1 : I2 = 1 : 16 then the value of a is : 3 3
[Sep. 04, 2020 (II)] 5 2
(c) J (d) J
(a) 2 2 (b) 2 3 3
(c) 2 (d) 4 82. Three solid spheres each of mass m and diameter d are
stuck together such that the lines connecting the centres
78. y O' form an equilateral triangle of side of length d. The ratio
I0
of moment of inertia I0 of the system about an axis
IA
O passing the centroid and about center of any of the spheres
80 cm
IA and perpendicular to the plane of the triangle is:
[9 Jan. 2020 I]
x 13
60 cm (a)
23
For a uniform rectangular sheet shown in the figure, the
ratio of moments of inertia about the axes perpendicular to 15
(b)
the sheet and passing through O (the centre of mass) and 13
O' (corner point) is : [Sep. 04, 2020 (II)] 23
(c)
(a) 2/3 (b) 1/4 13
(c) 1/8 (d) 1/2 13
(d)
15
P-86 Physics

83. One end of a straight uniform 1 m long bar is pivoted on 87. Mass per unit area of a circular disc of radius a depends
horizontal table. It is released from rest when it makes on the distance r from its centre as s(r) = A + Br. The
an angle 30° from the horizontal (see figure). Its angular moment of inertia of the disc about the axis, perpendicular
to the plane and passing through its centre is:
speed when it hits the table is given as ns -1 , where n is
[7 Jan. 2020 II]
an integer. The value of n is ________ . [9 Jan. 2020 I]
4æ A aB ö 4 æ aA Bö
(a) 2pa çè + ÷ø (b) 2pa çè + ÷
4 5 4 5ø
4 æ A aB ö 4æA Bö
(c) pa çè + ÷ø (d) 2pa çè + ÷ø
4 5 4 5
88. A circular disc of radius b has a hole of radius a at its centre
84. A uniformly thick wheel with moment of inertia I and
(see figure). If the mass per unit area of the disc varies as
radius R is free to rotate about its centre of mass (see
fig). A massless string is wrapped over its rim and two æ s0 ö
blocks of masses m1 and m2 (m1 > m2) are attached to the ç ÷ , then the radius of gyration of the disc about its axis
è r ø
ends of the string. The system is released from rest. The
passing through the centre is : [12 Apr. 2019 I]
angular speed of the wheel when m1 descents by a distance
h is: [9 Jan. 2020 II]
1/ 2
a2 + b2 + ab a+b
é 2(m1 - m2 ) gh ù (a) (b)
2 2
(a) ê 2 ú
ëê (m1 + m2 )R + 1 ûú
1/ 2 a2 + b2 + ab a+b
é 2(m1 + m2 ) gh ù (c) (d)
3 3
(b) ê 2 ú
êë (m1 + m2 )R + 1 úû
I1
1/2 89. Two coaxial discs, having moments of inertia I1 and , are
é (m1 - m2 ) ù 2
(c) ê 2 ú gh
ëê (m1 + m2 )R + 1 ûú w1
rotating with respective angular velocities w1 and ,
1/2 2
é m1 + m2 ù
(d) ê ú gh about their common axis. They are brought in contact with
2
êë (m1 + m2 )R + 1 úû each other and thereafter they rotate with a common angular
85. As shown in the figure, a bob of mass m is tied by a velocity. If Ef and Ei are the final and initial total energies,
massless string whose other end portion is wound on a then (Ef – Ei) is : [10 Apr. 2019 I]
fly wheel (disc) of radius r and mass m. When released
from rest the bob starts falling vertically. When it has I1w12 I w2 3 2 I w2
(a) - (b) 1 1 (c) I1w1 (d) - 1 1
covered a distance of h, the angular speed of the wheel 12 6 8 24
will be: [7 Jan. 2020 I] 90. A thin disc of mass M and radius R has mass per unit area
s(r) = kr2 where r is the distance from its centre. Its moment
of inertia about an axis going through its centre of mass
and perpendicular to its plane is : [10 Apr. 2019 I]
MR 2 2MR 2
(a) (b)
3 3
1 4 gh 3 MR 2 MR 2
(a) (b) r (c) (d)
r 3 2 gh 6 2
1 2 gh 3 91. A solid sphere of mass M and radius R is divided into two
r
(c) (d) 4 gh
r 3 7M
unequal parts. The first part has a mass of and is
86. The radius of gyration of a uniform rod of length l, about 8
l converted into a uniform disc of radius 2R. The second
an axis passing through a point away from the centre
4 part is converted into a uniform solid sphere. Let I1 be
of the rod, and perpendicular to it, is: [7 Jan. 2020 I] the moment of inertia of the new sphere about its axis.
1 1 7 3 The ratio I1/I2 is given by : [10 Apr. 2019 II]
(a) l (b) l (c) l (d) l
4 8 48 8 (a) 185 (b) 140 (c) 285 (d) 65
System of Particles and Rotational Motion P-87

92. A stationary horizontal disc is free to rotate about its axis. 98. An equilateral triangle ABC is cut from a thin solid sheet
When a torque is applied on it, its kinetic energy as a of wood. (See figure) D, E and F are the mid-points of its
function of q, where q is the angle by which it has rotated, sides as shown and G is the centre of the triangle. The
is given as kq2. If its moment of inertia is I then the angular moment of inertia of the triangle about an axis passing
acceleration of the disc is: [9 April 2019 I] through G and perpendicular to the plane of the triangle is
k k k 2k I0. If the smaller triangle DEF is removed from ABC, the
(a) q (b) q (c) 2 I q (d) q moment of inertia of the remaining figure about the same
4I I I
axis is I. Then : [11 Jan. 2019 I]
93. Moment of inertia of a body about a given axis is 1.5 kg A
m2. Initially the body is at rest. In order to produce a
rotational kinetic energy of 1200 J, the angular
acceleration of 20 rad/s2 must be applied about the axis D E
for a duration of: [9 Apr. 2019 II] G
(a) 2.5s (b) 2s (c) 5s (d) 3s
94. A thin smooth rod of length L and mass M is rotating B F C
freely with angular speed w0 about an axis perpendicular 15 3
(a) I= I0 (b) I = I0
to the rod and passing through its center. Two beads of 16 4
mass m and negligible size are at the center of the rod 9 I0
initially. The beads are free to slide along the rod. The (c) I = I 0 (d) I =
16 4
angular speed of the system, when the beads reach the 99. a string is wound around a hollow cylinder of mass 5 kg
opposite ends of the rod, will be: [9 Apr. 2019 II] and radius 0.5 m. If the string is now pulled with a
M w0 M w0 horizontal force of 40 N, and the cylinder is rolling
(a) (b) without slipping on a horizontal surface (see figure), then
M +m M + 3m
M w0 M w0 the angular acceleration of the cylinder will be (Neglect
(c) (d) the mass and thickness of the string) [11 Jan. 2019 II]
M + 6m M + 2m
40 N
95. A thin circular plate of mass M and radius R has its density
varying as r(r) = r0 r with r0 as constant and r is the
distance from its center. The moment of Inertia of the (a) 20 rad/s2 (b) 16 rad/s2
circular plate about an axis perpendicular to the plate and (c) 12 rad/s 2 (d) 10 rad/s2
passing through its edge is I = a MR2. The value of the 100. A circular disc D1 of mass M and radius R has two
coefficient a is: [8 April 2019 I] identical discs D2 and D3 of the same mass M and radius
(a) 1 2 (b) 3 5 (c) 8 5 (d) 3 R attached rigidly at its opposite ends (see figure). The
2 moment of inertia of the system about the axis OO’,
96. Let the moment of inertia of a hollow cylinder of length 30 passing through the centre of D1, as shown in the figure,
cm (inner radius 10 cm and outer radius 20 cm), about its will be : [11 Jan. 2019 II]
axis be 1. The radius of a thin cylinder of the same mass O'
such that its moment of inertia about its axis is also I, is:
[12 Jan. 2019 I]
(a) 12 cm (b) 16 cm
O
(c) 14 cm (d) 18 cm D2 D3

97. The moment of inertia of a solid sphere, about an axis D1

parallel to its diameter and at a distance of x from it, is ‘I(x)’. (a) MR2 (b) 3MR2
Which one of the graphs represents the variation of I(x) 4 2
with x correctly? [12 Jan. 2019 II] (c) MR 2 (d) MR 2
5 3
I(x) 101. Two identical spherical balls of mass M and radius R
I(x)
each are stuck on two ends of a rod of length 2R and
(a) (b) mass M (see figure). The moment of inertia of the
system about the axis passing perpendicularly through
the centre of the rod is: [10 Jan. 2019 II]
O x O x
I(x) I(x)

(c) (d)

O x O x
P-88 Physics

137 17 105. From a uniform circular disc of radius R and mass 9 M, a


(a) MR 2 (b) MR 2 R
15 15 small disc of radius is removed as shown in the figure.
3
209 152
(c) MR 2 (d) MR 2 The moment of inertia of the remaining disc about an
15 15 axis perpendicular to the plane of the disc and passing,
m through centre of disc is : [2018]
102. Two masses m and are connected at the two ends of
2
a massless rigid rod of length l. The rod is suspended by 2R
a thin wire of torsional constant k at the centre of mass 3
of the rod-mass system (see figure). Because of R
torsional constant k, the restoring toruque is t= kq for
angular displacement q. If the rod is rotated by q0 and
released, the tension in it when it passes through its mean 40 37
position will be: [9 Jan. 2019 I] (a) 4 MR2 (b) MR2 (c) 10 MR2 (d) MR2
9 9
106. A thin circular disk is in the xy plane as shown in the
figure. The ratio of its moment of inertia about z and z¢ axes
will be [Online April 16, 2018]
z z¢

O y

3k q0 2 2k q0 2 k q0 2 k q0 2
(a) (b) (c) (d) x
l l l 2l (a) 1 : 2 (b) 1 : 4 (c) 1 : 3 (d) 1 : 5
103. A rod of length 50 cm is pivoted at one end. It is raised 107. A thin rod MN, free to rotate in the vertical plane about the
such that if makes an angle of 30° from the horizontal as fixed end N, is held horizontal. When the end M is released
shown and released from rest. Its angular speed when the speed of this end, when the rod makes an angle a with
it passes through the horizontal (in rads–1) will be the horizontal, will be proportional to: (see figure)
(g = 10 ms–2) [9 Jan. 2019 II] [Online April 15, 2018]
M N
a

30°
(a) cos a (b) cosa
(c) sin a (d) sin a
(a)
30
(b) 108. The moment of inertia of a uniform cylinder of length l and
30 radius R about its perpendicular bisector is I. What is the
7
ratio l/R such that the moment of inertia is minimum ?
20 30 [2017]
(c) (d)
3 2 3 3 3
(a) 1 (b) (c) (d)
104. Seven identical circular planar disks, each of mass M and 2 2 2
radius R are welded symmetrically as shown. The moment 109. Moment of inertia of an equilateral triangular lamina ABC,
of inertia of the arrangement about the axis normal to the about the axis passing thr ough its centre O and
plane and passing through the point P is: [2018] perpendicular to its plane is Io as shown in the figure. A
cavity DEF is cut out from the lamina, where D, E, F are
the mid points of the sides. Moment of inertia of the
P
remaining part of lamina about the same axis is :
O [Online April 8, 2017]
C
7 15
(a) Io (b) Io
8 16
F E
19 55 73 181 3Io 31I o
(a) MR 2 (b) MR 2 (c) MR 2 (d) MR2 (c) (d)
O
2 2 2 2 4 32
A D B
System of Particles and Rotational Motion P-89

R (c) Statement 1 is true, Statement 2 is true, Statement


110. A circular hole of radius is made in a thin uniform 2 is correct explanation of the Statement 1.
4
disc having mass M and radius R, as shown in figure. The (d) Statement 1 is true, Statement 2 is false.
moment of inertia of the remaining portion of the disc about 115. A solid sphere having mass m and radius r rolls down an
an axis passing through the point O and perpendicular to inclined plane. Then its kinetic energy is
the plane of the disc is : [Online April 9, 2017] [Online May 7, 2012]
219 MR 2 5 2
(a) (a) rotational and translational
256 7 7
2 5
(b) rotational and translational
237 MR 2 R 7 7
(b) R/4 2 3
512 O o' (c) rotational and translational
19 MR 2 5 5
(c) 3R/4 1 1
512 (d) rotational and translational
2 2
197 MR 2 116. A circular hole of diameter R is cut from a disc of mass M
(d)
256 and radius R; the circumference of the cut passes through
111. From a solid sphere of mass M and radius R a cube of the centre of the disc. The moment of inertia of the
maximum possible volume is cut. Moment of inertia of cube remaining portion of the disc about an axis perpendicular
about an axis passing through its center and perpendicular to the disc and passing through its centre is
to one of its faces is : [2015] [Online May 7, 2012]
4MR 2 4MR 2
(a) (b) æ 15 ö 2 æ 1ö 2
9 3p 3 3p (a) çè ÷ø MR (b) çè ÷ø MR
2
32 8
MR MR 2
(c) (d) æ 3ö 2 æ 13 ö 2
32 2p 16 2p (c) çè ÷ø MR (d) çè ÷ø MR
8 32
112. Consider a thin uniform square sheet made of a rigid
material. If its side is ‘a’ mass m and moment of inertia I 117. A mass m hangs with the help of a string wrapped around
about one of its diagonals, then :[Online April 10, 2015] a pulley on a frictionless bearing. The pulley has mass m
and radius R. Assuming pulley to be a perfect uniform
ma 2 ma 2 ma 2 circular disc, the acceleration of the mass m, if the string
(a) I> (b) <I<
12 24 12 does not slip on the pulley, is: [2011]
ma 2 ma 2 2
g
g 3
(c) I= (d) I = (a) g (b) (c) (d) g
24 12 3 3 2
113. A ring of mass M and radius R is rotating about its axis 118. A pulley of radius 2 m is rotated about its axis by a force
with angular velocity w. Two identical bodies each of mass F = (20t – 5t2) newton (where t is measured in seconds)
m are now gently attached at the two ends of a diameter of applied tangentially. If the moment of inertia of the pulley
the ring. Because of this, the kinetic energy loss will be: about its axis of rotation is 10 kg-m2 the number of rotations
[Online April 25, 2013] made by the pulley before its direction of motion is
reversed, is: [2011]
m( M + 2m) 2 2 Mm
(a) w R (b) w2 R2 (a) more than 3 but less than 6
M ( M + m) (b) more than 6 but less than 9
Mm ( M + m) M 2 2 (c) more than 9
(c) w2 R2 (d) w R
( M + 2 m) (M + 2m) (d) less than 3
114. This question has Statement 1and Statement 2. Of the four 119. A thin uniform rod of length l and mass m is swinging
choices given after the Statements, choose the one that freely about a horizontal axis passing through its end. Its
best describes the two Statements. maximum angular speed is w. Its centre of mass rises to
Statement 1: When moment of inertia I of a body rotating a maximum height of [2009]
about an axis with angular speed w increases, its angular 1 lw 1l w2 2
momentum L is unchanged but the kinetic energy K (a) (b)
6 g 2 g
increases if there is no torque applied on it.
1l w2 2
1 l 2w2
Statement 2: L = Iw, kinetic en ergy of rotation (c) (d)
1 2 6 g 3 g
= Iw [Online May 12, 2012] 120. Consider a uniform square plate of side ‘a’ and mass ‘M’.
2
(a) Statement 1 is true, Statement 2 is true, Statement The moment of inertia of this plate about an axis
2 is not the correct explanation of Statement 1. perpendicular to its plane and passing through one of its
(b) Statement 1 is false, Statement 2 is true. corners is [2008]
P-90 Physics

126. A particle performing uniform circular motion has angular


5 1
(a) Ma 2 (b) Ma 2 frequency is doubled & its kinetic energy halved, then the
6 12 new angular momentum is [2003]
7 2 L
(c) Ma 2 (d) Ma 2 (a) (b) 2 L
12 3 4
L
121. For the given uniform square lamina ABCD, whose centre (c) 4 L (d)
2
is O, [2007] 127. Moment of inertia of a circular wire of mass M and radius
R about its diameter is [2002]
D F C (a) MR2/2 (b) MR 2 (c) 2MR 2 (d) MR2/4
128. Initial angular velocity of a circular disc of mass M is
w 1. Then two small spheres of mass m are attached gently
O to diametrically opposite points on the edge of the disc.
What is the final angular velocity of the disc? [2002]
A B æ M + mö æ M + mö
E (a) çè ÷w (b) çè ÷w
M ø 1 m ø 1
(a) I AC = 2 I EF (b) 2 I AC = I EF æ M ö æ M ö
(c) çè ÷w (d) çè ÷ w1
M + 4m ø 1 M + 2mø
(c) I AD = 3I EF (d) I AC = I EF
122. Four point masses, each of value m, are placed at the
corners of a square ABCD of side l. The moment of inertia TOPIC 5 Rolling Motion
of this system about an axis passing through A and parallel
to BD is [2006]
129.A uniform sphere of mass 500 g rolls without slipping
(a) 2ml 2 (b) 3ml2 on a plane horizontal surface with its centre moving at a
speed of 5.00 cm/s. Its kinetic energy is:
(c) 3ml 2 (d) ml 2
[8 Jan. 2020 II]
123. The moment of inertia of a uniform semicircular disc of
(a) 8.75 × 10–4 J (b) 8.75 × 10–3 J
mass M and radius r about a line perpendicular to the
(c) 6.25 × 10–4 J (d) 1.13 × 10–3 J
plane of the disc through the centre is [2005]
2 1 130.
(a) Mr 2 ` (b) Mr
5 4
1
(c) Mr 2 (d) Mr 2
2
124. One solid sphere A and another hollow sphere B are of
same mass and same outer radii. Their moment of inertia Consider a uniform cubical box of side a on a rough floor
about their diameters are respectively IA and IB Such that that is to be moved by applying minimum possible force F
[2004] at a point b above its centre of mass (see figure). If the
(a) IA < IB (b) IA > IB coefficient of friction is m = 0.4, the maximum possible value
IA dA b
(c) IA = IB (d) = of 100 × for box not to topple before moving is
I B dB a
________. [NA 7 Jan. 2020 II]
where dA and dB are their densities. 131.A solid sphere and solid cylinder of identical radii approach
125. A circular disc X of radius R is made from an iron plate of an incline with the same linear velocity (see figure). Both
roll without slipping all throughout. The two climb
thickness t, and another disc Y of radius 4R is made from an maximum heights hsph and hcyl on the incline. The ratio
t hsph
iron plate of thickness . Then the relation between the is given by : [8 Apr. 2019 II]
4
moment of inertia IX and II is [2003] hcyl

(a) ΙY = 32 Ι X (b) ΙY = 16 Ι X

(c) Ι Y = Ι X (d) ΙY = 64 Ι X
2 14 4
(a) (b) 1 (c) (d)
5 15 5
System of Particles and Rotational Motion P-91

132.The following bodies are made to roll up (without Figure). If they roll on the incline without slipping such
slipping) the same inclined plane from a horizontal plane: sin qc
that their accelerations are the same, then the ratio
R sin q s
(i) a ring of radius R, (ii) a solid cylinder of radius is: [Online April 9, 2014]
2
MC
A
R
and (iii) a solid sphere of radius . If, in each case, the
4
speed of the center of mass at the bottom of the incline M
is same, the ratio of the maximum heights they climb is: B S

[9 April 2019 I]
qC
(a) 4 : 3 : 2 (b) 10 : 15 : 7
qS
(c) 14 : 15 : 20 (d) 2 : 3 : 4 C
D
133. A homogeneous solid cylindrical roller of radius R and
mass M is pulled on a cricket pitch by a horizontal 8 15
(a) (b)
force. Assuming rolling without slipping, angular 7 14
acceleration of the cylinder is: [10 Jan. 2019 I] 8 15
(c) (d)
3F F 7 14
(a) (b) 137. A loop of radius r and mass m rotating with an angular
2mR 3m R velocity w0 is placed on a rough horizontal surface.
The initial velocity of the centre of the hoop is zero.What
F 2F
(c) (d) will be the velocity of the centre of the hoop when it ceases
2mR 3m R to slip ? [2013]
134. A roller is made by joining together two cones at their rw0 rw0
vertices O. It is kept on two rails AB and CD, which are (a) (b)
4 3
placed asymmetrically (see figure), with its axis
perpendicular to CD and its centre O at the centre of line rw0
(c) (d) rw0
joining AB and Cd (see figure). It is given a light push so 2
that it starts rolling with its centre O moving parallel to 138. A tennis ball (treated as hollow spherical shell) starting
CD in the direction shown. As it moves, the roller will from O rolls down a hill. At point A the ball becomes air
tend to: [2016] borne leaving at an angle of 30° with the horizontal. The
B ball strikes the ground at B. What is the value of the
D
distance AB ?
(Moment of inertia of a spherical shell of mass m and radius
2
R about its diameter = mR 2 )
3
O [Online April 22, 2013]
O

C
A
(a) go straight. 2.0 m
(b) turn left and right alternately.
(c) turn left. 30°
(d) turn right. 0.2 m A B
135. A uniform solid cylindrical roller of mass ‘m’ is being
pulled on a horizontal surface with force F parallel to the (a) 1.87 m (b) 2.08 m
surface and applied at its centre. If the acceleration of the (c) 1.57 m (d) 1.77 m
cylinder is ‘a’ and it is rolling without slipping then the 139. A thick-walled hollow sphere has outside radius R0. It rolls
value of ‘F’ is: [Online April 10, 2015] down an incline without slipping and its speed at the bottom
is v0. Now the incline is waxed, so that it is practically
5
(a) ma (b) ma frictionless and the sphere is observed to slide down
3 (without any rolling). Its speed at the bottom is observed
3 to be 5v0/4. The radius of gyration of the hollow sphere
(c) ma (d) 2 ma
2 about an axis through its centre is [Online May 26, 2012]
136. A cylinder of mass Mc and sphere of mass Ms are placed (a) 3R0/2 (b) 3R0/4
at points A and B of two inclines, respectively (See (c) 9R0 /16 (d) 3R0
P-92 Physics

140. A solid sphere is rolling on a surface as shown in figure, 7


with a translational velocity v ms–1. If it is to climb the (a) 2gh (b) gh
inclined surface continuing to roll without slipping, then 5
minimum velocity for this to happen is 7 10
(c) gh (d) gh
[Online May 12, 2012] 2 7
141. A round uniform body of radius R, mass M and moment of
inertia I rolls down (without slipping) an inclined plane
making an angle q with the horizontal. Then its
h acceleration is [2007]
g sin q g sin q
(a) (b)
2
1 + I / MR 2
v
1 - MR / I
g sin q g sin q
(c) (d)
1 + MR 2 / I 1 - I / MR 2
System of Particles and Rotational Motion P-93

1. (3) Centre of mass of solid hemisphere of radius R lies at L Læ


bx 3 ö æ aL2 bL2 ö
3R ò xdm = òç
ç ax + ÷
L2 ÷ø
dx = ç
ç 2
+ ÷
4 ÷ø
a distance above the centre of flat side of hemisphere. 0 0è è
8
æ aL2 bL2 ö
3R 3 ´ 8 çç + ÷
\ hcm = = = 3 cm 2 4 ÷ø
8 8 \ X CM = è
2. (23.00) Let s be the mass density of circular disc. bL
aL +
3
Original mass of the disc, m0 = pa 2 s
3L æ 2a + b ö
a2 Þ X CM = ç ÷
Removed mass, m = s 4 è 3a + b ø
4 4. (b)
æ a2 ö æ 4p - 1ö
Remaining, mass, m ' = ç pa 2 - ÷ s = a 2 ç s
è 4 ø è 4 ÷ø
Y

a
2
For given Lamina
X
a x y
1
2 m1 = 1, C1 = (1.5, 2.5)
m2 =3, C2 = (0.5, 1.5)
m x + m2 x2 1.5 + 1.5
New position of centre of mass X cm = 1 1 = = 0.75
m1 + m2 4
2 a2 a
m1 y1 + m2 y2 2.5 + 4.5
m0 x0 - mx pa ´ 0 - 4 ´ 2 Ycm = = = 1.75
X CM = = m1 + m2 4
m0 - m a2
pa 2 - \ Coordinate of centre of mass of flag shaped lamina
4
(0.75, 1.75)
-a 3 / 8 -a -a a 5. (a) Mass of sphere = volume of sphere x density of
= = = =-
æ 1 ö 2 2(4p - 1) 8p - 2 23 sphere
çè p - ÷ø a
4 4 3
= pR r
\ x = 23 3
3. (b) Given,
4
æxö
2 Mass of cavity M cavity = p(1)3 r
Linear mass density, r( x ) = a + b ç ÷ 3
èLø Mass of remaining

X CM =
ò xdm 4 4
M (Remaining) = pR3r – p(1)3 r
ò dm 3 3
Centre of mass of remaining part,
L
M r + M 2 r2
ò dm = ò r( x)dx X COM = 1 1
0 M1 + M 2
Lé 2ù
æxö bL é4 3 ù é4 3 ù
= ò êa + b ç ÷ ú dx = aL + êë 3 pR rúû 0 + êë 3 p(1) (– r) úû [ R –1]
0 êë
èLø úû 3 Þ –(2 – R ) =
4 3 4
pR r + p(1)3 (–r)
3 3
P-94 Physics

( R – 1) M a
Þ = 2– R M ´0- ´
3
( R –1) = 4 4 =- a
( R –1) M 12
M-
Þ =2–R 4
( R –1)( R 2 + R + 1)
Þ (R2 + R + 1) (2 – R) = 1 b
and yCM = -
12
6. (d)
So CM coordinates one
a a 5a
x0 = - =
2 12 12
b b 5b
and y0 = - =
2 12 12

m1 x1 + m2 x2 + m3 x3 2m (L,L)
Xcm = 10. (a)
m1 + m2 + m3
æ Lö
mç 2L, ÷ æ 5L ö
1 ´ 0 + 1.5 ´ 3 + 2.5 ´ 0 1.5 ´ 3 è 2ø
X cm = = = 0.9cm ç ,0 ÷
1 + 1.5 + 2.5 5 è 2 ø
2L m 3L
m y + m2 y2 + m3 y3 Y
Ycm = 1 1
m1 + m2 + m3
1 ´ 0 + 1.5 ´ 0 + 2.5 ´ 4 2.5 ´ 4
Ycm = = = 2cm X
1 + 1.5 + 2.5 5
x-coordinate of centre of mass is
Hence, centre of mass of system is at point (0.9, 2)
50 ´ 0 + 100 ´1 + 150 ´ 0.5 7 5mL
2mL + 2mL +
7. (c) x cm =
50 + 100 + 150
= m
Xcm = 2 = 13 L
12
4m 8
y-coordinate of centre of mass is
æ Lö
2m ´ L + m ´ ç ÷ + m ´ 0
è 2ø 5L
Ycm = =
4m 8
11. (c) To produce maximum moment of force line of action
of force must be perpendicular to line AB.
A

3 q
50 ´ 0 + 100 ´ 0 + 150 ´ 3 4m
y cm = 2 = m
50 + 100 + 150 4 q
2m B
8. (a) Acceleration of centre of mass (acm) is given by
r r 2 1
r m a + m2 a2 + ........ \ tan q = =
\ acm = 1 1 4 2
m1 + m2 + ........ 12. (c) According to principle of moments when a system is
(2m)ajˆ + 3m ´ aiˆ + ma (-iˆ) + 4m ´ a(- ˆj ) stable or balance, the anti-clockwise moment is equal to
= clockwise moment.
2m + 3m + 4m + m
i.e., load × load arm = effort × effort arm
2aiˆ - 2ajˆ a ˆ ˆ When 5 mg weight is placed, load arm shifts to left side,
= = (i - j )
10 5 hence left arm becomes shorter than right arm.
9. (d) With respect to point q, the CM of the cut-off portion
æ x ö1
(rx ) ç ÷ + ry y /2
æa bö x è2ø2
ç , ÷ . 13. (c) Centre of mass xcm =
è 4 4ø 2 r( x + y )
MX - mx 1 y y2
Using, xCM = Þ + =
M -m 2 x x2
System of Particles and Rotational Motion P-95

A x
£1 Here
L
With increase in the value of n, the centre of mass shift
x towards the end x = L This is satisfied by only option (a).
L L L n
æ xö
ò xdm ò x (l dx) ò k çè L÷ø .xdx
y
C xCM = 0 = 0
= 0
B (0,0) L L L n
æxö
Bc y 1 + 3 ò dm ò l dx ò k çè L ÷ø dx
\ = = 1.37 0 0 0
AB x 2
L
14. (d) Let density of cone = r. é x n+ 2 ù
kê nú
Centre of mass, ycm =
ò ydm =
êë (n + 2) L úû 0
=
L(n + 1)
ò dm é kx n +1 ù
L n+2
h ê nú
ëê (n + 1) L ûú 0
ò ypr
2
dyr h 2

= 0
=
ò0 r ydy
...(i) L
1 2 1 2 For n = 0 , xCM = ; n = 1,
pR hr R h 2
3 3
2L 3L
A xCM = ; n = 2, xCM = ;....
3 4
y For n ® ¥xcm = L
a
dy h Moment of inertia of a square plate about an axis through
r its centre and perpendicular to its plane is.
19. (b) Let s be the mass per unit area of the disc.
B C
R Then the mass of the complete disc
For a cone, we know that = s(p(2R)2 )
r y y 2R
= \r= R
R n n
h h
3 é y4 ù O R
ò 3 y dy
3ê ú
ë 4 û0 3
ycm = 0 3 = 3
= h
h h 4
15. (b) Centre of mass of the rod is given by: The mass of the removed disc = s (pR 2 ) = psR 2
L
bx2 Let us consider the above situation to be a complete disc
ò (ax + L
) dx of radius 2R on which a disc of radius R of negative mass
xcm = 0 is superimposed. Let O be the origin. Then the above
L
bx figure can be redrawn keeping in mind the concept of
ò (a + L )dx centre of mass as :
0
2 R
aL bL2 L æ a + b ö
2 4ps R
+ ç ÷ –ps R
2
= 2 3 = è 2 3ø O

aL +
bL
2
a+
b
2 xc.m =
( 6p( 2R) ) ´ 0 + ( -6( pR )) R
2 2

a b 4psR2 - psR2
+
Now
7L 2 3
= -psR2 ´ R
b \ xc.m =
12 3psR2
a+
2 R 1
On solving we get, b = 2a \ xc.m =- = aR Þ a =
16. (c) 17. (d) 3 3
n
æ xö 20. (c) Initially,
18. (a) The linear mass density l = k ç ÷
è Lø
P-96 Physics

m1 ( - x1 ) + m2 x2 24. (c) Here, rdr w2 r = rgdh


0= Þ m1 x1 = m2 x2 ...(1)
m1 + m2 R h
Let the particles is displaced through distanced away from Þ w2 ò rdr = g ò dh
centre of mass 0 0
x1– d x2– d ¢
d d¢ w
m1 O m2

m1 (d - x1 ) + m2 ( x2 - d ') dh
\0 = dr
m1 + m2
Þ 0 = m1d - m1 x1 + m2 x2 - m2 d '
m1
Þd'= d [From (1).]
m2
21. (a) The centre of mass of bodies B and C taken together
does not shift as no external force acts. The centre of mass w2 R 2
Þ = gh (Given R = 5 cm)
of the system continues its original path. It is only the 2
internal forces which comes into play while breaking.
w2 R 2 25w2
l \h = =
22. (d) 2g 2g
A B
y1 25. (c) Free body diagram in the frame of disc
kx
® mw2 (l0 + x )
¬¾¾ m ¾¾
F P y2
2l \ mw2 (l 0 + x ) = kx

y ml 0 w2
Þ x=
k – mw2
(0, 0) For k >> mw2
C
To have translational motion without rotation, the force x mw2
Þ =
uur l0 k
F has to be applied at centre of mass. i.e. the point ‘P’has
to be at the centre of mass 26. (b) At elongated position (x),
Taking point C at the origin position, positions of y, and y2 mv 2
are r1 = 2l, r2 = l and ml = m and m2 = 2m Fradial = = mr w2
r
m y + m2 y2 m ´ 2 l + 2m ´ l 4l
y= 1 1 = = \ kx = m(l + x)w2
m1 + m2 3m 3
(Q r = l + x here)
dy
23. (a) y = 4Cx 2 Þ = tan q = 8Cx kx = mlw2 + mxw2
dx
At P, tan q = 8Ca mlw2
\x=
y k – mw2
w 2
mw acosq
T x
2
N 27. (d) ò (- dT ) = ò (dm)w x
q 2 0 l
mw a
a P(a, b)
mg
x
mgsinq q mgcosq
x
For steady circular motion æm ö 2
– T = ò ç l dx ÷w x
2
mw a cos q = mg sin q l
è ø

g tan q
Þw= mw2 2
a or T = (l - x 2 )
l
g ´ 8aC It is a parabola between T and x.
\w = = 2 2 gC
a
System of Particles and Rotational Motion P-97

28. (b) N sin q = mw2 (r/2) ...(i) The rotational speed of the drum
v g 10
Þω< < <
R R 1.25
r/2 1 The maximum rotational speed of the drum in revolutions
sin q = = Þ q = 30° per minute
r 2
60 10
ω(rpm) < < 27 .
and N cos q = mg ...(ii) 2p 1.25
w r 2 34. (b) Angular momentum, mvr = Iw
or tan q = Moment of Inertia (I) of cubical block is given by
2g R
m.2
w2 r æ 2 æ ö2 ö÷ \ w= 2
çR
∗ çç ÷÷÷ ÷÷÷
R
I < m çç
or tan 30° = é R2 æ R ö 2 ù
2g
çç 6 èç 2 ø ÷÷ mê + ç ÷ ú
è ø
1 w2 r êë 6 è 2 ø úû
or =
3 2g 12 3 10
Þw= = = = 5 rad / s .
2g 8R 2 ´ 0.3 2
\ w2 = .
r 3 35. (b) Angular velocity is the angular displacement per
29. (a) Using v2 = u2 + 2gy [\u = 0 at (0,0)] Dq
unit time i.e., w =
v2 = 2gy [\v = wx] Dt
Y Here w1 = w2 and independent of f.
w 36. (c) Angular momentum, L = I w
Þy
X
(0,0)
l
w2 x 2 (2 ´ 2p) 2 ´ (0.05) 2 Axis
Þ y= = ; 2cm m
2g 20

2 1 mv2
30. (c) mw R = Force µ n (Force = )
R R
1 1 m
Þ w2 µ Þ wµ l/ 2 m
n +1 n +1
R
R 2
2p l/ 2
Time period T =
w
2
n +1 æ l ö
I = m(0) 2 + m ç ´ 2 + m( 2l ) 2
Time period, T µ R 2 è 2 ÷ø
31. (b) decreasing speed
32. (c) Torque at angle q 2ml 2
w,a
= + 2ml 2 = 3ml 2
l
2
t = Mg sin q. Angular momentum L = I w = 3ml 2 w
2 Q
Also t = la Q 37. (20)
l
\ l a = Mg sin q w
2
(M, L)
Ml 2 l é Ml 2 ù
.a = Mg sin q êQ I rod = ú
3 2 ë 3 û
la sin q 3g sin q m v
Þ =g \ a=
3 2 2l Before collision After collision
33. (a) For just complete rotation
Using principal of conservation of angular momentum we
v < Rg at top point have
P-98 Physics
r r
Li = L f Þ mvL = I w 1 æ 5 2 ö 9v 2
Þ ç ml ÷ø = 2mgl (1 - cos q )
2è3 25l 2
æ ML2 ö
Þ mvL = ç + mL2 ÷ w 3
è 3 ø Þ mv 2 = 2mgl (1 - cos q)
5´ 2
æ 0.9 ´ 12 ö 3 36 27
Þ 0.1 ´ 80 ´ 1 = ç + 0.1 ´ 12 ÷ w ´ = 1 - cos q Þ 1 - = cos q
è 3 ø 10 2 ´ 10 50
æ 3 1ö 4 23
Þ8=ç + ÷wÞ8= w or, cos q = \q ; 63°.
è 10 10 ø 10 50
40. (d) Vertical force = mg
Þ w = 20 rad/sec.
38. (9.00) 2 l
Horizontal force = Centripetal force = mw sin q
Here M0 = 200 kg, m = 80 kg 2
Using conservation of angular momentum, Li = Lf l
Torque due to vertical force = mg sin q
2
M0
2 l l
Torque due to horizontal force = mw sin q cos q
2 2
m
FV

I1w1 = I 2 w 2
FH
æ M R2 ö
I1 = ( I M + I m ) = ç 0 + mR 2 ÷
è 2 ø
1 l
I2 = M 0 R 2 and w1 = 5 rpm w2 sin q
2 2
æ M R2 ö 5
\w2 = ç 0 + mR 2 ÷ ´ mg
ç 2 ÷ M R2
è ø 0
2 Net Torque = Angular momentum
5R 2
(80 + 100)
= 2 ´ = 9 rpm. l l l ml 2 2
R 100 mg sin q - mw2 sin q cos q = w sin q cos q
2 2 2 12
39. (a) Using conservation of angular momentum
3 g
æ 2 ml 2 ö Þ cos q =
5 3v 2 w 2l
mvl = ç ml 2 + w Þ mvl = ml 2 w Þ w =
è 3 ÷ø 3 5l 41. (d) Net torque, tnet about B is zero at equilibrium
3 ´ 6 18 \ TA ´ 100 - mg ´ 50 - 2mg ´ 25 = 0
or, w = = rad/s
5 ´1 5 Þ TA ´ 100 = 100mg
w Þ TA = 1 mg (Tension in the string at A)
M = 2 kg

TA TB
50 cm
25 cm
A B
m = 1 kg
Now using energy conservation, after collision mg
1 2 l
I w = 2mg (1 - cos q) + mgl (1 - cos q)
2 2 2 mg
System of Particles and Rotational Motion P-99

42. (a) 46. (d) Angular acceleration,


F
w - w0 25 ´ 2p - 0
a= = = 10 p rad/s2
t 5
R t = Ia
R–a N æ5 ö æ5ö
mR 2 ÷ a » ç ÷ (5 ´10-3 )(10-4 )10p
Mg x f a Þ t = çè 4 ø è4ø

For step up, F ´ R ³ Mg ´ x = 2.0 × 10–5 Nm


47. (a)
x = R 2 - ( R - a) 2 from figure 48. (c) According to work-energy theorem
2 1 1
Mg æ R-aö mgh = mv 2B – mv 2A
Fmin = ´ R 2 - ( R - a )2 = Mg 1 - ç ÷ 2 2
R è R ø
2gh = v2B – v 2A
43. (c)
2 × 10 × 10 = v2B – 52
Þ vB =15 m/s
Angular momentum about O,
LO = mvr
= 20 × 10–3 × 20
LO = 6 kg.m2/s
About point O angular momentum r F F 3 ˆ
Linitial = Lfinal 49. (a) Given, F1 = ( -ˆi) + (- j)
2 2
mV 1 é 4mL2 mL2 ù r
Þ ´ =ê + ú´ w r = 0iˆ + 6jˆ
1
2 2 ë 12 4 û
Torque due to F1 force
6V 3 2V
\w= = r r r æF F 3 ˆ ö
7 2L 7L t F1 = r1 ´ F1 = 6ˆj ´ ç ( -ˆi) + ˆ
( - j) ÷ = 3F(k)
ç2 2 ÷
44. (b) Given that, x = x0 + a cos w1t è ø
y = y0 + b sin w2t Torque due to F2 force
r
dx t F2 = (2iˆ + 3j)
ˆ ´ Fkˆ = 3Fiˆ + 2F( - ˆj)
= Vx
dt r r r
t = t + t = 3Fiˆ + 2F(-ˆj) + 3F(k)
net F1 F2
ˆ
dy
Þ vx = – aw1 sin (w1t), and = v = bw cos(w t) = (3iˆ - 2jˆ + 3k)F
ˆ
dt y 2 2
r r r
50. (a) Torque about the origin = t = r ´ F
dv x dv y
= a = – aw2 cos (w t), = ay = – bw22 sin (w2t) = r F sin q Þ 2.5 = 1 × 5 sin q
dt x 1 1
dt 1
At t = 0, x = x0 + a, y = y0 sin q= 0.5 =
2
ax = – aw21, ay = 0
p
r r r r r Þ q=
Now, t = r ´ F = m(r´ a) 6
51. (d) Consider a strip of radius x and thickness dx,
= [(x 0 + a) iˆ+ y 0 ˆj] ´ m( - a w12 ˆi) = + my0 a w12 kˆ
Torque due to friction on this strip
r Net torque = å Torque on ring
45. (d) We have given r = 2tiˆ - 3t 2 ˆj
r R
mF.2 pxdx
r (at t = 2) = 4iˆ - 12 ˆj ò dt = ò
r 0 pR 2
r dr
Velocity, v = = 2iˆ - 6tjˆ
dt 2µF R 3
Þ t= ·
r R2 3
v (at t = 2) = 2iˆ - 12 ˆj
r 2µFR
r r t=
L = mvr sin qnˆ = m(r ´ v ) 3
52. (a) Applying torque equation about point P.
= 2(4iˆ - 12 ˆj ) ´ (2iˆ - 12 ˆj ) = -48kˆ
t = I a = [2M0(2l)2 + 5M0l2]a
P-100 Physics

Þ 5M0gl – 4 M0gl = [2M0(2l)2 + 5 M0l2]a In none of the cases, the perpendicular


Þ M0gl = (13M0gl2)a æ R ö
distance r^ is ç + a÷
g è 2 ø
\a=
13l
57. (a) Angular momentum,
53. (d) Given that, the rod is of uniform mass density and
L0 = mvr sin 90°
AB = BC
= 2 × 0.6 × 12 × 1 × 1
[As V = rw, Sin 90° = 1]
So, L0 = 14.4 kgm2/s
o 0.6m

0.8m 1m

O
58. (c) Torque working on the bob of mass m is, t = mg × l sin
q. (Direction parallel to plane of rotation of particle)

q
l l

Let mass of one rod is m. m mg


Balancing torque about hinge point. r
mg (C1P) = mg (C2N) As t is perpendicular to L , direction of L changes but
magnitude remains same.
æL ö æL ö
mg ç sin q ÷ = mg ç cos q - L sin q ÷ 59. (c) Given : m = 0.160 kg
è2 ø è2 ø
q = 60°
3 mgL v = 10 m/s
Þ mgL sin q = cos q
2 2 ® ®
sin q 1 1 Angular momentum L = r ´ m v
Þ = or, tan q = = H mv cos q
cos q 3 3
54. (a) Balancing torque w.r.t. point of suspension v2 sin 2 q é v2 sin 2 q ù
= cos q êH = ú
æl ö 2g êë 2g úû
mg x = Mg ç – x ÷
è2 ø
X l/2-X
l A B 102 ´ sin 2 60°´ cos 60°
Þ mx = M – Mx =
2 2 ´10
Mg
æ lö1 = 3.46 kg m2/s
m = çM ÷ – M 60. (a)
è 2ø x m 61. (a) Angular momentum L = m (v × r)
1
y= a –C Straight line equation. æ dr ö
x = 2 kg ç ´ r÷ = 2 kg(4t j ´ 5i - 2t 2 ˆj)
55. (a) è dt ø
56. (a) We know that |L| = mvr^ = 2 kg (–20 t k̂ ) = 2 kg × –20 ×2 m–2 s–1 k̂
y
= –80 k̂
D V C 62. (b) 63. (d)
a
V
a 64. (c) Angular momentum, L = Iw Þ L = mr2w
a a V As insect moves along a diameter, the effective mass and
a hence moment of inertia (I) first decreases then increases
A V B so from principle of conservation of angular momentum,
R/ 2 angular speed w first increases then decreases.
r r r
65. (c ) L = m(r ´ v )
O a X r
L = m éêv0 cos qt iˆ + (v0 sin qt - gt 2 ) ˆj ùú
1
R
2
ë 2 û
System of Particles and Rotational Motion P-101

´ éë v0 cos q iˆ + (v0 sin q - gt ) ˆj ùû R

é 1 ˆ ù
= mv0 cos qt ê - gt ú k
ë 2 û
1 H r
= - mgv0 t cos qkˆ
2
2
uur q
uur d L
66. (d) We know Torque t c = c
dt
uur From diagram,
where Lc = Angular momentum about the center of mass
of the body. Central forces act along the center of mass. r R R
Therefore torque about center of mass is zero. = tan q = or r = h ...(ii)
h H H
dL uur
\t = =0 Þ Lc = constt. Mass of element, dm = r(pr 2 )dh ...(iii)
dt
67. (d) Applying conservation of angular momentum I¢w¢ = Iw From eq. (i), (ii) and (iii),
(mR2 + 2MR2)w¢ = mR2w
Þ (m + 2m)R2w¢ = mR2w dh
Area of element, dA = 2prdl = 2pr
cos q
é m ù
Þ w' = wê
ë m + 2 M úû 2 Mh tan dh
ur ur uur Mass of element, dm =
68. (c) Torque t = r ´ F = (iˆ - ˆj ) ´ ( - Fkˆ) R R 2 + H 2 cos q
(here, r = h tan q )
= F [- iˆ ´ kˆ + ˆj ´ kˆ] = F ( ˆj + iˆ) = F ( iˆ + ˆj)
H H 2
éSince kˆ ´ iˆ = ˆj and ˆj ´ kˆ = iˆù æR ö
ë û I = ò dI = ò dm(r ) =
2
ò r(pr )dh çè H × h÷ø
2

69. (b) Angular momentum will remain the same since 0 0

no external torque act in free space. H æ æR ö


r r r
70. (d) We know that t = r ´ F
= ò r çè p çè H × h÷ø ÷ dh
ø
0

MR 2
Solving we get, I =
F

2
t r
73. (b) dm
A B
x
dx
Mass of the small element of the rod
dm = l × dx
r r r
Vector t is perpendicular to both r and F . We also Moment of inertia of small element,
know that the dot product of two vectors which have an
æ xö
angle of 90° between them is zero. dI = dm × x 2 = l 0 ç1 + ÷ × x 2 dx
è Lø
r r r r
\ r × T = 0 and F × T = 0 Moment of inertia of the complete rod can be obtained by
71. (d) Angular momentum (L) integration
= (linear momentum) × (perpendicular distance of the line
L
of action of momentum from the axis of rotation) æ x3 ö
As the particle moves with velocity V along line PC, the line I = l 0 ò ç x 2 + ÷ dx


of motion passes through P.
\ L = mv × r L
x3 x4 é L3 L3 ù
= mv × 0 = l0 + = l0 ê + ú
=0 3 4L 0 ë3 4û
72. (d) Hollow ice-cream cone can be assume as several parts
of discs having different radius, so 7l 0 L3
ÞI = ...(i)
12
I = ò dI = ò dm(r ) 2
...(i) Mass of the thin rod,
P-102 Physics

L L
3l L Let I ADE = I BDF = I EFC = I 2
æ xö
M = ò l dx = ò l 0 ç1 + ÷ dx = 0 I0 5I
è Lø 2
0 0 = I0 Þ I 2 = 0
\ 3 I 2 + I1 = I 0 Þ 3 I 2 +
16 16
2M Hence, moment of inertia of DECB i.e., after removal part
\ l0 =
3L ADE
7 æ 2M ö 3 7 æ 5 I ö æ I ö 11I 0 NI 0
\I = 2 = 2 I 2 + I1 = 2 ç 0 ÷ + ç 0 ÷ = =
ç ÷ L Þ I = ML è 16 ø è 16 ø 16 16
12 è 3L ø 18
Therefore value of N = 11.
74. (d) By angular momentum conservation, Lc = L f
1 2
76. (20) As we know moment of inertia disc, I disc = MR
w 2
wI + 3 I ´ 0 = 4 I w ' Þ w ' =
4 w1 w2 w
M M R1(M)
3I
I1 I2 R2(M)
w I R2=R/2
R1=R
Using angular momentum conservation
1 2
I1w1 + I 2w2 = ( I1 + I 2 ) ´ w f
( KE )i = Iw
2 MR 2 æ MR 2 MR 2 ö 4
´ w + 0 = çç + ÷÷ w f Þ w f = w
1 2 è 2 8 ø 5
( KE ) f = (3I + I )w '2
2
1 2 1 æ MR 2 ö 2 MR 2 w2
æ wö
2
I w2 Initial K.E., K i = I w = çç ÷w =
1
= ´ (4 I ) ´ ç ÷ = 2 2 è 2 ÷ø 4
2 è 4ø 8
1 1 3 1 æ MR 2 MR 2 ö 16 2 MR 2 w2
DKE = I w 2 - I w 2 = I w 2 Final K.E., K f = çç + ÷ w =
2 8 8 2è 2 8 ÷ø 25 5
Percentage loss in kinetic energy % loss
3 2
Iw
DKE 8 3 MR 2 w2 MR 2 w2
\ Fractional loss in K.E. = = = . -
KEli 1 2 4 4 5
Iw = ´ 100 = 20% = P%
2 MR 2 w2
75. (11) Let mass of triangular lamina = m, and length of side
4
= l, then moment of inertia of lamina about an axis passing
through centroid G perpendicular to the plane. Hence, value of P = 20.
77. (c) Let p be the density of the discs and t is the thickness
I 0 µ ml 2 of discs.
I 0 = kml 2 Moment of inertia of disc is given by
A MR 2 [r(pR 2 )t ]R 2
I= =
2 2
I µR 4 (As r and t are same)
G 4
I 2 æ R2 ö 16
=ç ÷ Þ = a4 Þ a = 2
B I1 è R1 ø 1
Let moment of inertia of DEF = I1 about G
2
78. (b) Moment of inertia of rectangular sheet about an axis
æ m öæ l ö ml 2 I0 passing through O,
So, I1 µ ç ÷ç ÷ µ or I1 =
4
è øè ø 2 16 16 M 2 M
IO = (a + b 2 ) = [(80) 2 + (60) 2 ]
A 12 12
y
O'

D l/2 E
O
G 80 cm
cm
0

m/4
10

m/4
B C x
F 60 cm
System of Particles and Rotational Motion P-103

From the parallel axis theorem, moment of inertia about O ', 81. (b) Initial angular momentum = I1w1 + I 2 w 2
IO ' = IO + M (50)2 Let w be angular speed of the combined system.
M Final angular momentum = I1w + I 2 w
IO (802 + 602 )
12 1 According to conservation of angular momentum
= =
IO ' M 4
(802 + 602 ) + M (50) 2 ( I1 + I 2 )w = I1w1 + I 2 w 2
12
79. (c) Let there be a cylinder of mass m length L and radius I1w1 + I 2 w 2 0.1 ´ 10 + 0.2 ´ 5 20
Þw= = =
R. Now, take elementary disc of radius R and thickness dx I1 + I 2 0.1 + 0.2 3
at a distance of x from axis OO' then moment of inertia
Final rotational kinetic energy
about OO' of this element.
2
O 1 1 1 æ 20 ö
Kf = I1w 2 + I 2 w 2 = (0.1 + 0.2) ´ ç ÷
2 2 2 è 3ø
R
20
Þ Kf = J
3
dx 82. (a) Moment of inertia,
O'
2
dmR
dI = + dmx 2
4
n =- L / 2
dmR 2 M
Þ I = ò dI = ò + ò dx ´ x 2
4 n=L / 2
L

MR 2 ML2
Given : I = +
4 12
M V ML2 MV ML2 2 æd ö
2
ÞI = ´ + ÞI= + I1 = m ç ÷ + m ( AO )
2
4 pL 12 4pL 12 5 è2ø
dI mV M ´ 2L
=- + =0 d
dL 4pL2
12 and AO =
3
2 3 2
ÞV = pL Þ pR 2 L = pL3 Moment of inertia about ‘O’
3 3
é 2 æ d ö2 æ d ö ù
2
L 3 I 0 = 3I1 = 3 ê m ç ÷ + m ç
\ = ÷ ú
R 2 êë 5 è 2 ø è 3 ø úû
80. (25)
Moment of inertia of the system about axis XE. 13
Þ I0 = Md 2
X F 10
rF
é 2 æ d ö2 ù 2 æ d ö2
And I A = 2 ê M ç ÷ + Md 2 ú + M ç ÷
a a êë 5 è 2 ø úû 5 è 2 ø
23
Þ IA = Md 2
60° 10
E a G 13
rG Md 2
IO 10 13
\ = =
I = I E + I F + IG I A 23 23
Md 2
10
Þ I = m(rE )2 + m(rF )2 + m(rG )2
83. (15) Here, length of bar, l = 1 m
2
æaö 5 25 angle, q = 30°
Þ I = m ´ 02 + m ç ÷ + ma 2 = ma 2 = ma 2
2
è ø 4 20 1 2
\ N = 25. DPE = DKE or mgh = Iw
2
P-104 Physics

l 1 æ ml 2 ö 2 b
Þ (mg) sin 30° = ç ÷w 2
2 2 çè 3 ÷ø 88. (c) I = ò (dm)r
a
l 1 1 æ ml 2 ö 2 b
Þ mg ´ = ç ÷w æ s0 ö 2 2 ps0 3 b
2 2 2 çè 3 ÷ø = ò ç r ´ 2pr dr ÷ r = | r |a
a è ø 3
Þ w = 15 rad/s 2ps0 3
= (b - a 3 )
84. (a) Using principal of conservation of energy 3
1 1 Mass of the disc,
(m1 – m2 ) gh = (m1 + m2 )v 2 + I w2
2 2 b
s0
1 2 1 2 m= ò r
´ 2pr dr = 2ps (b – a)
Þ (m1 – m2 ) gh = (m1 + m2 )(wR ) + I w a
0

2 2
(Q v = wR ) Radius of gyration,

w2 é I
Þ (m1 – m2 ) gh = (m1 + m2 ) R 2 + I ù k=
m
2 ë û

Þ w=
2(m1 – m2 ) gh (2ps0 / 3)(b3 - a3 ) a 2 + b2 + ab
2 = =
(m1 + m2 ) R + I 2ps0 (b - a ) 3
85. (a) When the bob covered a distance ‘h’ 89. (d) As no external torque is acting so angular momen-
1 2 1 2 tum should be conserved
Using mgh = mv + I w (I1 + I2) w =I1w1 + I2w2 [wc = common angular velocity
2 2
of the system, when discs are in contact]
1 1 mr 2 Iw
= m(w r) 2 + ´ ´ w2 (Q v = wr no slipping ) I1w1 + 1 1
2 2 2 4 æ 5 ´ 2 öw
wc =
3 2 2 I1 çè 4 3 ÷ø 1
Þ mgh = mw r I1 +
4 2
5w1
4 gh1 4 gh wc =
Þw= = 6
2r 3
3r 1 1 1
86. (c) Moment inertia of the rod passing through a point E f - Ei = ( I1 + I2 ) wc2 - I1w12 - I2 w22
2 2 2
l 5w1
away from the centre of the rod Put I2 = I1/2 and wc = 5w1/6
4 6
I = Ig + ml2 We get :
æ I 2 ö 7 MI 2 I w2
MI 2 E f – Ei = – 1 1
Þ I= + M ´ç ÷ = 24
12 ç 16 ÷ 48
è ø 90. (b) As from the question density (s) = kr 2
2 7MI 2 R
R 4 pkR 4
Using I = MK = (K = radius of gyration) Mass of disc M = ò (kr )2prdr = 2 pk
2
=
48 4 2
0
7
Þ K= I 2M
48 Þk= ....(i)
pR 4
87. (a) Given, \ Moment of inertia about the axis of the disc.
mass per unit area of circular disc, s = A + Br
Area of the ring = 2 prdr l = ò dl = ò ( dm ) r 2 = ò sdAr 2
Mass of the ring, dm = s2prdr
= ò (kr 2 )(2prdr)r 2
Moment of inertia,
I = ò dmr 2 = ò s 2prdr.r 2 æ 2M ö
p´ ç ´ R6
R
pkR 4÷6
2
a = ò 2pk r dr =
5
= è p R ø = MR 2
é Aa 4 Ba5 ù
Þ I = 2 p ò ( A + Br )r 3 dr = 2p ê + ú 0
3 3 3
0 êë 4 5 úû
[putting value of k from eqn ....(i)]
é A Ba ù
Þ I = 2pa 4 ê + ú
ë4 5 û
System of Particles and Rotational Motion P-105

91. (b) Using parallel axis theorem


5
æ 1 1 ö 16pr0 R
\ I = IC + MR 2 = 2pr0 R 5 ç + ÷ =
è3 5ø 15
8 é2 ù 8
= ê pr0 R 3 ú R 2 = MR 2
5 ë3 û 5
96. (b)
97. (d) According to parallel axes theorem
2
I = mR 2 + mx 2
5
æ 7M ö 21 æ 7 ö 14 Hence graph (d) correctly depicts I vs x.
I1 = ç ÷ (2 R) = ç ´ 4 ÷ MR2 = mR 2
è 8 ø 2 è 16 ø 8 98. (a) Let mass of the larger triangle = M
2æM ö Side of larger triangle = l
I2 = ç ÷ r 2
5è 8 ø Moment of inertia of larger triangle = ma2
öæ R ö MR 2
2 M
2æ M
Þ I2 = ç ÷ çç ÷= Mass of smaller triangle = 4
5è 8 øè 4 ÷ 80
ø
l
é4 3 14 3 ù Length of smaller triangle =
ê 3 pr r = 8 3 pR ´ r ú 2
ê ú Moment of inertia of removed triangle
êëÞ r = R /2 úû 2
Mæaö
I1 14 ´ 80 = 4ç ÷
= = 140 è2ø 2
I2 8 M æaö
I ç ÷
2
1 2 \ removed = 4 . è ø2
92. (d) I w = kQ 2 Ioriginal M (a)
2
I0
æ 2k ö I removed =
or w = çç ÷÷ Q 16
è I ø I 15I0
So, I = I0 - 0 =
dw 2 K æ dQ ö æ 2k ö 16 16
or a = = ç ÷ =ç ÷w 99. (b)
p I è dt ø çè I ÷ø 40
æ 2k öæ 2k ö a =Ra
=ç ÷ç q 2k q O
ç I ÷ç I ÷÷ = f
è øè ø I
P
93. (b) w = at = 20t From newton’s second law
1 2 40 + f = m (Ra) .....(i)
Given, I w = 1200 Taking torque about 0 we get
2
1 40 × R – f × R = Ia
2
or ´ 1.5 ´ (20t ) = 1200 40 × R – f × R = mR2 a
2
or t = 2 s 40 – f = mR a ...(ii)
Solving equation (i) and (ii)
94. (c) Iiwi = If wf
40
æ ML2 a= =16rad / s 2
æ Lö ö
æ ML2 ö 2
ç mR
or çç 12 w
÷ 0
÷
=
ç 12
+ 2 m ç ÷ ÷÷ w f
è ø è è2ø ø
MR 2
100. (b) Moment of inertia of disc D1 about OO¢ = I1 =
æ M w0 ö 2
\w f = ç ÷
è M + 6m ø M.O.I of D2 about OO¢
95. (c) Taking a circular ring of radius r and thickness dr as a
1 æ MR 2 ö MR 2
= I2 = 2 ç 2 ÷ + MR = 4 + MR
2 2
mass element, so total mass,
R è ø
2pr0 R 3
M = ò r0 r ´ 2prdr = M.O.I of D3 about OO¢
0
3
1 æ MR 2 ö MR 2
÷ + MR = + MR 2
2
R
2pr0 R 5 = I3 = ç
2è 2 ø 4
IC = ò r0 r ´ 2prdr ´ r 2 =
0
5
P-106 Physics

so, resultant M.O.I about OO¢ is I = I1 + I2 + I3 104. (d) Using parallel axes theorem, moment of inertia about ‘O’
Io = Icm + md2
MR 2 æ MR 2 ö 2R
ÞI = + 2ç + MR 2 ÷ 2R
2 è 4 ø 7MR 2 55MR 2 2R
2 2
= + 6(M ´ (2R)2 ) = O
MR MR 2 2 2
= + + 2MR 2 = 3 MR 2R
2 2 Again, moment of inertia about 2R
101. (a) For Ball
point P, Ip = Io + md2 2R
using parallel axes theorem, for ball moment of inertaia,
2 22 55MR 2 181
Iball = MR + M ( 2R ) = MR
2 2 2 = + 7M(3R) 2 = MR 2
5 5 2 2
22 105. (a) Let s be the mass per unit area.
For two balls Iballs= 2× MR2 =and,
5 R/ 3
M ( 2R ) MR
2 2 O'
Irod = =
12 3
2R/3
Isystem = Iballs + Irod
O
44 MR 2 137
= MR 2 + = MR 2
5 3 15
k
102. (c) As we know, w =
I
The total mass of the disc
3k é 1 2ù = s × p R2 = 9M
w= êëQ I rod = ml ú The mass of the circular disc cut
ml 2 3 û
2
æ Rö p R2
= s´pç ÷ = s ´ =M
è 3ø 9
Let us consider the above system as a complete disc
of mass 9M and a negative mass M super imposed
Tension when it passes through the mean position, on it.
Moment of inertia (I1 ) of the complete disc =
l 3k l 2
1
= mw2 q02 = m 2 q02 = kq0 9MR 2 about an axis passing through O and
3 ml 3 l 2
perpendicular to the plane of the disc.
M.I. of the cut out portion about an axis passing
103. (d) through O' and perpendicular to the plane of disc
Initial position 2
= 50 cm 1 æ Rö
= ´M ´ç ÷
2 è 3ø
\ M.I. (I2) of the cut out portion about an axis
o
30 passing through O and perpendicular to the plane of
disc
Final position
é1 æ Rö
2
æ 2R ö ù
2

By the low of conservation of energy, = ê ´M ´ç ÷ + M ´ç ÷ ú


êë 2 è 3ø è 3 ø ú
P.E. of rod = Rotational K.E. û
[Using perpendicular axis theorem]
l 1 \ The total M.I. of the system about an axis passing
mg Sin q = Iw2 through O and perpendicular to the plane of the disc
2 2
is
I = I1 + I2
l 1 ml 2 2 l 1 1 ml 2 2
Þ mg Sin 30° = ω Þ mg ´ = w 1 é1 æ Rö
2
æ 2R ö ù
2
2 2 3 2 2 2 3 2
= 9MR - ê ´ M ´ ç ÷ + M ´ ç ÷ ú
2 êë 2 è 3ø è 3 ø ú
For complete length of rod, û
9MR 2 9MR 2 (9 - 1)MR 2
30 = - = = 4 MR 2
w = 3g 2 ( 2l ) = rods –1 2 18 2
2
System of Particles and Rotational Motion P-107

106. (c) As we know, moment of inertia of a disc about an axis From equation (i),
passing through C.G. and perpendicular to its plane, I0
IDEF =
mR 2 16
Iz = Moment of inertia of remaining part
2
Moment of inertia of a disc about a tangential axis I0 15I 0
Iremain = I0 - =
perpendicular to its own plane, 16 16
110. (b) Moment of Inertia of complete disc about 'O' point
3 q q
Iz' = mR 2 MR 2
2 Itotal =
r 2
Radius of removed disc = R/4
mR 2 3mR 2
\ Iz Iz' = =1 3 \ Mass of removed disc = M/16
2 2 [As M µ R2]
107. (a) When the rod makes an angle a M.I of removed disc about its own axis (O')
2
l
Displacement of centre of mass = cos a 1 Mæ Rö MR 2
2 = çè ÷ø =
2 16 4 512
l l M.I of removed disc about O
mg cos a = I w 2
2 2 Iremoved disc = Icm + mx2
2
l ml 2 2 MR 2 M æ 3R ö 19 MR 2
mg cos a = w (Q M.I. of thin uniform rod = + ç ÷ =
2 6 512 16 è 4 ø 512
about an axis passing through its centre of mass and M.I of remaining disc
2
ml 2 Iremaining = MR - 19 MR 2 =
237
perpendicular to the rod I = ) MR 2
12 2 512 512
3 g cos a 2
Þ w= 111. (a) Here a = R
l 3
4 3
Speed of end = w ´ l = 3g cos al pR
M
Now, =3 3
i.e., Speed of end, w µ cos a M¢ a
108. (c) As we know, moment of inertia of a solid cylinder
about an axis which is perpendicular bisector 4 3
pR
3
mR 2 ml 2 = 3 3
= p. a
I= + æ 2 ö 2
4 12 ç R÷
é è 3 ø
m l2 ù
I = ê R2 + ú l
4ë 3û 2M
M¢=
3p
m é V l2 ù dl m é -V 2l ù
= ê + ú Þ = + =0 Moment of inertia of the cube about the given axis,
4 ë pl 3 û dl 4 êë pl 2 3 úû
M ¢a 2
I=
V 2l 2 pl 3 6
2
= ÞV =
pl 3 3 2
2M æ 2 ö
3 2 ´ç R÷ 2
2pl l 3 l 3 3p è 3 ø = 4 MR
pR 2 l = Þ 2 = or, = =
3 R 2 R 2 6 9 3p
109. (b) According to theorem of perpendicular axes, moment 112. (d) For a thin uniform square sheet
of inertia of triangle (ABC) ma 2
I0 = kml2 ..... (i) I1 = I2 = I3 =
12
BC = 1
Moment of inertia of a cavity DEF
2
mæ lö
IDEF = K ç ÷ I1
4 è 2ø
k
= ml 2 I2
16 I3
P-108 Physics

1 2 M.I. of complete disc can also be written as


113. (c) Kinetic energy (rotational) KR = Iw ITotal = Iremoved hole + Iremaining disc
2
1 3MR 2
Kinetic energy (translational) KT = Mv 2 (v = Rw) ITotal = + Iremaining disc ...(ii)
2 32
M.I.(initial) Iring = MR2; winitial = w From eq. (i) and (ii),
M.I.(new) I¢(system) = MR 2 + 2mR 2 1 3MR 2
MR 2 = + I remaining disc
2 32
Mw Þ Iremaining disc
w¢(system) =
M + 2m
MR 2 3MR 2 æ 13 ö
Solving we get loss in K.E. = - = ç ÷ MR2
2 32 è 32 ø
Mm
= w2 R 2 117. (b) For translational motion,
(M + 2m) mg – T = ma .....(1)
114. (b) As L = Iw so L increases with increase in w. For rotational motion,
Kinetic energy(rotational) depends on an angular velocity T.R = I a
‘w’ and moment of inertia of the body I. m R

1 2
i.e., K .E. ( rotational) = Iw T
2
1 2 m
115. (b) K .Erotational = Iw
2 mg
12 2 2 æ 2 2ö 1 2
= wr d çèQ ISolid sphere = mr ÷ø Þ T.R = mR a
2
25 5
Also, acceleration, a = Ra
1 2
K .Etranslational = mv 1 1
2 \ T = mRa = ma
2 2
K .Erotational 2
\ K .E = Substituting the value of T is equation (1) we get mg -
translational 5
1 2
Hence option (b) is correct ma = ma Þ a = g
2 3
116. (d) M.I. of complete disc about its centre O.
118. (a) Given,
1 Force, F = (20t – 5t2)
ITotal = MR2 ...(i) Radius, r = 2m
2
Torque, T = r f = Ia
R/2 Þ 2(20t – 5t2) = 10a
O¢ Circular hole of \a = 4t – t2
diameter R (radius = R/2) w t
R O
Disc mass = M
Þ
dw
dt
= 4t - t 2 Þ ò d w = ò (4t - t ) dt2

0 0
radius = R
t3
Þ w = 2t 2 - (as w = 0 at t = 0, 6s)
3
q 6 æ
Mass of circular hole (removed) t3 ö
ò d q = ò ç 2t 2 - ÷ dt
=
M
4
(
As M = pR 2t \ M µ R 2 ) 0 0 è

36
M.I. of removed hole about its own axis Þ q = 36 rad Þ 2 p n = 36 Þ n = <6
2p
2 O
1 æ M ö æ Rö 1 119. (c)
= ç ÷ç ÷ = MR2
2 è 4 ø è 2ø 32
M.I. of removed hole about O¢
Iremoved hole = Icm + mx2 h
C. M
2 C. M Reference
MR 2 M æ Rö level for P.E.
= + çè ÷ø
32 4 2
B
MR 2 MR 2 3MR 2
= + = A
32 16 32
System of Particles and Rotational Motion P-109

1 2 Again, by the same theorem I = IAC + IBD = 2 IAC


The moment of inertia of the rod about O is ml . The (\ IAC = IBD by symmetry of the figure)
3
1 2 I
kinetic energy of the rod at position A = I w where I is \ I AC = ...(ii)
2 2
the moment of inertia of the rod about O. When the rod is in From (i) and (ii), we get, IEF = IAC.
position B, its angular velocity is zero. In this case, the 122. (c)
D l C
energy of the rod is mgh where h is the maximum height to n
which the centre of mass (C.M) rises O
2
Gain in potential energy = Loss in kinetic energy l/
A B
1 2 1 æ 1 2ö 2
\ mgh = I w = 2 çè 3 ml ÷ø w
2
n'
Þ h=
l 2 w2
6g Inn' = M.I due to the point mass at B +
M.I due to the point mass at D +
1 Ma 2 M.I due to the point mass at C.
120. (d) Inn' = M (a 2 + a 2 ) =
12 6 2
n m æ l ö
Inn' = m ç
è 2 ÷ø
A 2
D æ l ö
+m ç
è 2 ÷ø

( 2l)
2
O +m
2
æ l ö
Þ I nn ' = 2 ´ m ç + m( 2l) 2
B è 2 ÷ø
C = ml2 + 2ml2 = 3ml2
123. (c) The disc may be assumed as combination of two semi
n m 1

circular parts. Therefore, circular disc will have twice


DB 2a a the mass of semicircular disc.
Also, DO = = =
2 2 2 1
By parallel axes the orem, moment of inertia of plate about Moment of inertia of disc = (2m)r2 = Mr2
an axis through one of its corners. 2
2 Let I be the moment of inertia of the uniform semicircular
æ a ö Ma 2 Ma 2 disc
Imm ' = I nn ' + M ç ÷ = +
è 2ø 6 2
Mr 2
2 2 Þ 2 I = 2Mr 2 Þ I =
Ma + 3Ma 2 2
= = Ma 2 124. (a) The moment of inertia of solid sphere A about its
6 3
121. (d) By the theorem of perpendicular axes, 2
I = IEF + IGH diameter I A = MR 2 .
5
Here, I is the moment of inertia of square lamina about an
axis through O and perpendicular to its plane. The moment of inertia of a hollow sphere B about its
\ IEF = IGH (By Symmetry of Figure) 2 2
diameter I B = MR .
Z 3
Y
D F
\ I A < IB
C
mass( M )
125. (d) We know that density (d ) =
volume(V )
X
O \ M = d ´ V = d ´ ( pR 2 ´ t ) .
1
The moment of inertia of a disc is given by I = MR 2
A B
2
E 1 1
\ Ix = MxRx2 = (d ´ pR 2 ´ t ) R 2
I 2 2
\ I EF = ...(i)
2
P-110 Physics

pd 129. (a) K.E of the sphere = translational K.E + rotational K.E


= t ´ R4
2 1 1
= mv 2 + I w2
Iy =
1
M R2 =

2 y y 2 êë
( )
æ 1ö ù
p 4R 2 ç ÷ d ú ´ ( 4 R )
è 4ø û
2
2 2
Where, I = moment of inertia,
I X t X RX4 t ´ R4 1 w = Angular, velocity of rotation
\ = = = m = mass of the sphere
IY 4
tY RY t 4 64
´ (4 R ) v = linear velocity of centre of mass of sphere
4
1 2 2
126. (a) Rotational kinetic energy = I w , Q Moment of inertia of sphere I = mR 2
2 5
L 1 2 1 2
Angular momentum, L = Iw Þ I = \ K .E = mv + ´ mR 2 ´ w2
w 2 2 5
1L 1 2
\ K .E. = ´ w 2 = Lw 1 2 1 2 ævö æ v ö
2w 2 Þ K .E = mv + ´ mR 2 ´ ç ÷ çQw = ÷
2 2 5 è Rø è Rø
2K.E
L' = 1æ2 öæ v ö
2
w Þ KE = ç mR2 + mR 2 ÷ç ÷
When w is doubled and K.E is haled. 2è5 øè R ø
New angular momentum,
1 7 v2 7 1 25
2K.E Þ KE = mR 2 ´ ´ = ´ ´
2 5 R 2 10 2 104
L' = 2 35
2w Þ KE = ´ 10 –4 joule
4
L
Þ \ L' = Þ KE = 8.75 × 10–4 joule
4
130. (50) For the box to be slide
127. (a) M. I of a circular wire about an axis nn' passing through
F = mmg = 0.4 mg
the centre of the circle and perpendicular to the plane of the
circle = MR2 Z For no toppling
n Y
æa ö a
F ç + b ÷ £ mg
è2 ø 2
X
æa ö a
Þ 0.4 mg ç + b ÷ £ mg
n' è2 ø 2
As shown in the figure, X-axis and Y-axis lie along diameter
of the ring . Using perpendicular axis theorem Þ 0.2 a + 0.4 b £ 0.5 a
IX + IY = IZ
Here, IX and IY are the moment of inertia about the diameter. b 3
Þ £
Þ 2 IX = MR2 [Q IX = IY (by symmetry) and IZ = MR2] a 4
1 i.e. b £ 0.75 a but this is not possible.
\ IX = MR 2 As the maximum value of b can be equal to 0.5a.
2
128. (c) Moment of inertia of circular disc 100b
1 Þ = 50
I1 = MR 2 a
2 131. (c) For sphere,
When two small sphere are attached on the edge of the
disc, the moment of inertia becomes 1 2 1
1 mv + Iw2 = mgh
I 2 = MR 2 + 2mR 2 2 2
2
When two small spheres of mass m are attached gently, 1 2 1 æ2 2öv
2 7v 2
the external torque, about the axis of rotation, is zero and or mv + ç mR ÷ 2 = mgh or h =
2 2 è5 øR 10 g
therefore the angular momentum about the axis of rotation
is constant. For cylinder
I
\ I1w1 = I 2 w 2 Þ w 2 = 1 w1 1 2 1 æ mR 2 ö
I2 mv + ç ÷ = mgh '
1
MR 2
M
2 2 çè 2 ÷ø
2
\ w2 = 1 ´ w1 = w1
M + 4m
MR 2 + 2mR 2
2
System of Particles and Rotational Motion P-111

3
3v 2 F = mRa
or h' = 2
4g
2F
a=
h 7v 2 / 10 g 14 3mR
\ = = 134. (c) As shown in the diagram, the normal reaction of AB
h ' 3v 2 / 4 g 15 on roller will shift towards O.
This will lead to tending of the system of cones to turn
1 2 1 2 left.
132.(Bonus) mgh = mvcm + I cm w
2 2 B D
2
1 2 1 æv ö
= mvcm + I cm ç cm ÷
2 2 è R ø

1æ I ö 2 O
= ç m + cm ÷ vcm
2è R2 ø

1æ mR 2 ö 2 A C
For ring : mgh = 2 çç m + 2 ÷÷ vcm 135. (c) From figure,
è R ø
ma = F – f ....(i)
2
vcm a
\h = .
g
F
Mass = m
1æ mR 2 ö 2 O
For solid cylinder, mgh = 2 çç m + ÷ vcm
÷
è 2R2 ø
f
3v 2
\ h = cm And, torque t = Ia
4g
mR 2
1æ 2 mR 2 ö 2 a = fR
= ç
For sphere, mgh 2 ç m + ÷ vcm 2
5 R2 ÷
è ø mR 2 a é aù
2 = fR êQ a = ú
7vcm 2 R ë R û
\h =
10 g ma
= f ...(ii)
3 7 2
Ratio of heights 1: : Þ 20 :15 :14 Put this value in equation (i),
4 10
ma 3ma
133. (d) ma = F – or F =
2 2
136. (d) As we know,
mg sin q
Acceleration, a =
I
m+
r2
M c .g. sin qc M c .g. sin qc
For cylinder, a c = 2 =
1 McR McR 2
F – fr = ma ...(i) Mc + Mc +
2 R2 2R 2
2
mR 2
frR = Ia = a ...(ii) or, ac =g sin qc
2 3
for pure rolling For sphere,
a = aR ...(iii)
from (1) (2) and (3) Ms g sin qs = Ms g sin qs
as =
I 2 MR 2
mRa M s + s2 Ms +
F– = maR r 5 R2
2
P-112 Physics
2
5 1 2 1 2 V0
or, as = g sin qs = mV0 + mk …(i)
7 2 2 R02
given, ac = as é V ù
êQ I = mk , w =
2
ú
2 5 ë R0 û
i.e., g sin qc = g sin qs
3 7 When body is sliding down then body has only
5 translatory motion.
g
sin qc 7 15 \ P.E. = K.Etrans
\ = =
sin qs 2 14 2
g 1 æ5 ö
3 = m v0 ...(ii)
2 çè 4 ÷ø
r
137. (c) o o Dividing (i) by (ii) we get

From conservation of angular momentum about any fix 1 2 é K2 ù


point on the surface, mv ê1 + ú
P.E. 2 0 êë R02 úû K2 K2
mr2w0 = 2mr2w =
25 9
= = 1+ 2 Þ 2 =
w0 r P.E. 1 25 16 16
´ ´ mV02 R0 R0
Þ w = w0/2 Þ v =
2
[Q v = rw ] 2 16
138. (b) Velocity of the tennis ball on the surface of the earth 3
or ground or, K = R0 .
4
2gh 140. (d) Minimum velocity for a body rolling without slipping
v= ( where k = radius of gyration of spherical
k2
1+ 2 gh
R2 v=
K2
shell =
2 1+
R) R2
3
v2 sin 2q
Horizontal range AB = K2 2
g For solid sphere, =
2 5
R
2
æ 2gh ö
çç ÷÷ sin(2 ´ 30°) 2 gh
1+ k2 / R2 10
=è ø
= 2.08 m \ v=
2
=
7
gh
g K
1+
139. (b) When body rolls dawn on inclined plane with R2
velocity V0 at bottom then body has both rotational 141. (b) Acceleration of the body rolling down an inclined
and translational kinetic energy. plane is given by.
Therefore, by law of conservation of energy,
g sin q
P.E. = K.Etrans + K.Erotational a=
I
1 1 1+
= mV02 + I w 2 MR 2
2 2
7
Gravitation P-113

Gravitation
(a) 500 days (b) 320 days
TOPIC 1 Kepler's Laws of Planetary (c) 260 days (d) 220 days
Motion 4. The time period of a satellite of earth is 5 hours. If the
1. If the angular momentum of a planet of mass m, moving separation between the earth and the satellite is increased
around the Sun in a circular orbit is L, about the center of to 4 times the previous value, the new time period will
the Sun, its areal velocity is: [9 Jan. 2019 I] become [2003]
(a) 10 hours (b) 80 hours
L 4L L 2L
(a) (b) (c) (d) (c) 40 hours (d) 20 hours
m m 2m m
2. Figure shows elliptical path abcd of a planet around the
TOPIC 2 Newton's Universal Law of
1 Gravitation
sun S such that the area of triangle csa is the area of the
4 5. A straight rod of length L extends from x = a to x = L + a.
ellipse. (See figure) With db as the semimajor axis, and ca The gravitational force it exerts on point mass ‘m’ at x = 0,
as the semiminor axis. If t1 is the time taken for planet to go if the mass per unit length of the rod is A + Bx2, is given
over path abc and t2 for path taken over cda then: by: [12 Jan. 2019 I]
[Online April 9, 2016]
c é æ 1 1ö ù
(a) Gm ê A ç - ÷ - BL ú
ë èa+L aø û
d b
S é æ1 1 ö ù
(b) Gm ê A ç - ÷ - BL ú
ë è a a + L ø û
a
(a) t1 = 4t2 (b) t1 = 2t2 é æ 1 1ö ù
(c) t1 = 3t2 (d) t1 = t2 (c) Gm ê A ç - ÷ + BL ú
ë èa+L aø û
3. India’s Mangalyan was sent to the Mars by launching it
into a transfer orbit EOM around the sun. It leaves the é æ1 1 ö ù
earth at E and meets Mars at M. If the semi-major axis of (d) Gm ê A ç - ÷ + BL ú
ë è a a + L ø û
Earth’s orbit is ae = 1.5 × 1011 m, that of Mars orbit am =
2.28 × 1011 m, taken Kepler’s laws give the estimate of time 6. Take the mean distance of the moon and the sun from the
for Mangalyan to reach Mars from Earth to be close to: earth to be 0.4 × 106 km and 150 × 106 km respectively.
[Online April 9, 2014] Their masses are 8 × 1022 kg and 2 × 1030 kg respectively.
Mars orbit
The radius of the earth is 6400 km. Let DF1 be the difference
in the forces exerted by the moon at the nearest and farthest
O
points on the earth and DF2 be the difference in the force
exerted by the sun at the nearest and farthest points on
am ae
E
DF1
M
Sun the earth. Then, the number closest to is:
DF2

Earth’s orbit [Online April 15, 2018]


(a) 2 (b) 6 (c) 10–2 (d) 0.6
P-114 Physics

7. Four particles, each of mass M and equidistant from each 12. The acceleration due to gravity on the earth’s surface at
other, move along a circle of radius R under the action of the poles is g and angular velocity of the earth about the
their mutual gravitational attraction. The speed of each axis passing through the pole is w. An object is weighed at
particle is: [2014] the equator and at a height h above the poles by using a
spring balance. If the weights are found to be same, then h
GM GM is : (h<<R, where R is the radius of the earth)
(a) (b) 2 2
R R
R 2w 2 R 2w 2
(a) (b) [5 Sep. 2020 (II)]
(c)
GM
R
(
1+ 2 2 ) (d)
1 GM
2 R
(
1+ 2 2 ) 2g g

R 2w 2 R 2w 2
8. From a sphere of mass M and radius R, a smaller sphere of (c) (d)
4g 8g
R
radius is carved out such that the cavity made in the 13. The height 'h' at which the weight of a body will be the
2
original sphere is between its centre and the periphery same as that at the same depth 'h' from the surface of the
(See figure). For the configuration in the figure where the earth is (Radius of the earth is R and effect of the rotation
of the earth is neglected) : [2 Sep. 2020 (II)]
distance between the centre of the original sphere and the
removed sphere is 3R, the gravitational force between the 5 R
two sphere is: [Online April 11, 2014] (a) R-R (b)
2 2
5R - R 3R - R
(c) (d)
2 2
14. A box weighs 196 N on a spring balance at the north pole.
Its weight recorded on the same balance if it is shifted to
the equator is close to (Take g = 10 ms –2 at the north pole
3R and the radius of the earth = 6400 km): [7 Jan. 2020 II]
41 GM 2 41 GM 2 59 GM 2 GM 2 (a) 195.66 N (b) 194.32 N
(a) (b) (c) (d) (c) 194.66 N (d) 195.32 N
3600 R 2 450 R 2 450 R 2 225 R 2
15. The ratio of the weights of a body on the Earth’s surface to
9. Two particles of equal mass ‘m’ go around a circle of radius that on the surface of a planet is 9:4. The mass of the
R under the action of their mutual gravitational attraction.
The speed of each particle with respect to their centre of 1
planet is th of that of the Earth. If ‘R’ is the radius of the
mass is [2011 RS] 9
Gm Gm Gm Gm Earth, what is the radius of the planet ? (Take the planets
(a) (b) (c) (d) to have the same mass density). [12 April 2019 II]
4R 3R 2R R
10. Two spherical bodies of mass M and 5M & radii R & 2R R R R R
respectively are released in free space with initial separation (a) (b) (c) (d)
3 4 9 2
between their centres equal to 12 R. If they attract each
16. The value of acceleration due to gravity at Earth’s surface
other due to gravitational force only, then the distance
is 9.8 ms– 2. The altitude above its surface at which the
covered by the smaller body just before collision is [2003]
acceleration due to gravity decreases to 4.9 ms– 2, is close
(a) 2.5 R (b) 4.5 R (c) 7.5 R (d) 1.5 R
to : (Radius of earth = 6.4 × 106 m) [10 April 2019 I]
(a) 2.6×10 m6 6
(b) 6.4×10 m
TOPIC 3 Acceleration due to Gravity (c) 9.0×106 m (d) 1.6×106 m
17. Suppose that the angular velocity of rotation of earth is
11. The value of acceleration due to gravity is g1 at a height increased. Then, as a consequence.
[Online April 16, 2018]
R
h= (R = radius of the earth) from the surface of the (a) There will be no change in weight anywhere on the
2
earth
earth. It is again equal to g1 and a depth d below the sur- (b) Weight of the object, everywhere on the earth, wild
ædö decrease
face of the earth. The ratio ç ÷ equals : [5 Sep. 2020 (I)] (c) Weight of the object, everywhere on the earth, will
è Rø
increase
4 5 1 7 (d) Except at poles, weight of the object on the earth will
(a) (b) (c) (d)
9 9 3 9 decrease
Gravitation P-115

18. The variation of acceleration due to gravity g with distance 22. Assuming the earth to be a sphere of uniform density, the
d from centre of the earth is best represented by (R = acceleration due to gravity inside the earth at a distance of
Earth's radius): [2017, Online May 7, 2012] r from the centre is proportional to[Online May 12, 2012]
g (a) r (b) r–1 (c) r2 (d) r–2
g
23. The height at which the acceleration due to gravity
g
(a) (b) becomes (where g = the acceleration due to gravity on
d d 9
O R O R the surface of the earth) in terms of R, the radius of the
g earth, is [2009]
g
R
(a) (b) R / 2 (c) 2R (d) 2 R
2
(c) d (d)
d 24. The change in the value of ‘g’ at a height ‘h’ above the
O O R surface of the earth is the same as at a depth ‘d’ below the
19. The mass density of a spherical body is given by r (r) = surface of earth. When both ‘d’ and ‘h’ are much smaller
k than the radius of earth, then which one of the following is
for r < R and r (r) = 0 for r > R, correct? [2005]
r
where r is the distance from the centre. 3h h
The correct graph that describes qualitatively the accel- (a) d = (b) d =
2 2
eration, a, of a test particle as a function of r is : (c) d = h (d) d =2 h
[Online April 9, 2017] 25. Average density of the earth [2005]
(a) is a complex function of g
a a (b) does not depend on g
(c) is inversely proportional to g
(a) (b) (d) is directly proportional to g

R r R r Gravitational Field and


TOPIC 4
Potential Energy
a a
26. Two planets have masses M and 16 M and their radii are a
(c) (d) and 2a, respectively. The separation between the centres
of the planets is 10a. A body of mass m is fired from the
r surface of the larger planet towards the smaller planet along
R R r
the line joining their centres. For the body to be able to
20. If the Earth has no rotational motion, the weight of a person reach the surface of smaller planet, the minimum firing
on the equator is W. Determine the speed with which the speed needed is : [6 Sep. 2020 (II)]
earth would have to rotate about its axis so that the person
GM GM
3 (a) 2 (b) 4
at the equator will weight W . Radius of the Earth is a a
4
2
6400 km and g =10 m/s . [Online April 8, 2017] GM 2 3 5GM
(c) (d)
ma 2 a
(a) 1.1×10–3 rad/s (b) 0.83×10–3 rad/s
27. On the x-axis and at a distance x from the origin, the
(c) 0.63 × 10–3 rad/s (d) 0.28×10–3 rad/s gravitational field due to a mass distribution is given by
21. The change in the value of acceleration of earth towards Ax
sun, when the moon comes from the position of solar in the x-direction. The magnitude of
( x + a 2 )3/2
2
eclipse to the position on the other side of earth in line
with sun is: gravitational potential on the x-axis at a distance x, taking
its value to be zero at infinity, is : [4 Sep. 2020 (I)]
(mass of the moon = 7.36 × 1022 kg, radius of the moon’s
orbit = 3.8 × 108 m). [Online April 22, 2013] A A
(a) 1
(b)
6.73 × 10–5 m/s2 6.73 × 10–3 m/s2
3
(a) (b) ( x2 + a2 ) 2
( x2 + a2 ) 2

(c) 6.73 × 10–2 m/s2 (d) 6.73 × 10–4 m/s2 3


1 2 2
(c) A( x 2 + a 2 ) (d) A( x + a )
2 2
P-116 Physics

28. The mass density of a planet of radius R varies with the


GM GM
æ r2 ö (a) 1.35 (b) 1.16
distance r from its centre as r(r ) = r0 çç1 - 2 ÷÷ . Then the a a
è R ø
GM GM
gravitational field is maximum at : [3 Sep. 2020 (II)] (c) 1.21 (d) 1.41
a a
(a) r = 3 R (b) r = R 33. A test particle is moving in circular orbit in the gravitational
4
K
field produced by a mass density r (r ) = . Identify the
1 5 r2
(c) r = R R (d) r =
3 9 correct relation between the radius R of the particle’s orbit
29. Consider two solid spheres of radii R1 = 1m, R2=2m and and its period T: [8 April 2019 II]
(a) T/R is a constant 2 3
(b) T /R is a constant
masses M1 and M2, respectively. The gravitational field
m1 (c) T/R2 is a constant (d) TR is a constant
due to sphere 1 and 2 are shown. The value of m is: 34. A body of mass m is moving in a circular orbit of radius R
2
[8 Jan. 2020 I] about a planet of mass M. At some instant, it splits into
two equal masses. The first mass moves in a circular orbit
4
R
Gravitational field E

of radius , and the other mass, in a circular orbit of


3 2
2 3R
2 radius . The difference between the final and initial
1 2
1 total energies is: [Online April 15, 2018]
GMm GMm GMm GMm
0 (a) - (b) + (c) - (d)
1 2 3 4 5 2R 6 R 6 R 2R
radius R 35. From a solid sphere of mass M and radius R, a spherical
2 1 1 1 portion of radius R/2 is removed, as shown in the figure.
(a) (b) (c) (d) Taking gravitational potential V = 0 at r = ¥, the potential at
3 6 2 3
the centre of the cavity thus formed is :
30. An asteroid is moving directly towards the centre of the
(G = gravitational constant) [2015]
earth. When at a distance of 10 R (R is the radius of the
earth) from the earths centre, it has a speed of 12 km/s.
Neglecting the effect of earths atmosphere, what will be
the speed of the asteroid when it hits the surface of the
earth (escape velocity from the earth is 11.2 km/ s)? Give
your answer to the nearest integer in kilometer/s _____.
[NA 8 Jan. 2020 II]
-2GM -2GM -GM -GM
31. A solid sphere of mass ‘M’ and radius ‘a’ is surrounded by (a) (b) (c) (d)
a uniform concentric spherical shell of thickness 2a and 3R R 2R R
mass 2M. The gravitational field at distance ‘3a’ from the 36. Which of the following most closely depicts the correct
centre will be: [9 April 2019 I] variation of the gravitational potential V(r) due to a large
planet of radius R and uniform mass density ? (figures
2GM GM GM 2GM are not drawn to scale) [Online April 11, 2015]
(a) 2 (b) 2 (c) 2 (d)
9a 9a 3a 3a 2
32. Four identical particles of mass M are located at the corners V(r)
of a square of side ‘a’. What should be their speed if each V(r)
of them revolves under the influence of others’
gravitational field in a circular orbit circumscribing the (a) r (b) O r
O
square ? [8 April 2019 I]
V(r)
r V(r) r
(c) O (d)
O
Gravitation P-117

37. The gravitational field in a region is given by


® (a)
2Gm
r
( 2 -1 ) (b)
Gm
r
g = 5N / kgiˆ + 12N / kgjˆ . The change in the gravitational
potential energy of a particle of mass 1 kg when it is taken 2Gm æ 1 ö 2Gm
1-
r çè ÷
from the origin to a point (7 m, – 3 m) is: (c) (d)
2ø r
[Online April 19, 2014]
42. Two bodies of masses m and 4 m are placed at a distance r.
(a) 71 J (b) 13 58J (c) – 71 J (d) 1 J The gravitational potential at a point on the line joining
m1 m2 them where the gravitational field is zero is: [2011]
38.
v1 v2
4Gm 6Gm 9Gm
(a) - (b) - (c) - (d) zero
r r r
d
43. This question contains Statement-1 and Statement-2. Of
Two hypothetical planets of masses m1 and m2 are at rest
the four choices given after the statements, choose the
when they are infinite distance apart. Because of the
one that best describes the two statements. [2008]
gravitational force they move towards each other along
the line joining their centres. What is their speed when Statement-1 : For a mass M kept at the centre of a cube of
their separation is ‘d’? [Online April 12, 2014] side ‘a’, the flux of gravitational field passing through its
sides 4 p GM. and
(Speed of m1 is v1 and that of m2 is v2)
(a) v1 = v2 Statement-2: If the direction of a field due to a point source
is radial and its dependence on the distance ‘r’ from the
2G 2G
(b) v1 = m 2 v 2 = m1 1
d ( m1 + m 2 ) d ( m1 + m 2 ) source is given as , its flux through a closed surface
r2
2G 2G depends only on the strength of the source enclosed by
(c) v1 = m1 v 2 = m2
d ( m1 + m 2 ) d ( m1 + m 2 ) the surface and not on the size or shape of the surface.
(a) Statement -1 is false, Statement-2 is true
2G 2G
(d) v1 = m 2 v2 = m 2 (b) Statement -1 is true, Statement-2 is true; Statement -2
m1 m2
is a correct explanation for Statement-1
39. The gravitational field, due to the 'left over part' of a uniform (c) Statement -1 is true, Statement-2 is true; Statement -
sphere (from which a part as shown, has been 'removed 2 is not a correct explanation for Statement-1
out'), at a very far off point, P, located as shown, would be
(nearly) : [Online April 9, 2013] (d) Statement -1 is true, Statement-2 is false
44. A particle of mass 10 g is kept on the surface of a uniform
Removed Mass of complete sphere of mass 100 kg and radius 10 cm. Find the work to
Part sphere = M be done against the gravitational force between them to
P
R R take the particle far away from the sphere

(you may take G = 6.67× 10 -11 Nm 2 / kg 2 ) [2005]


x
(a) 3.33 × 10 -10 J (b) 13.34 × 10 -10 J
5 GM 8 GM 7 GM 6 GM
(a) (b) (c) (d)
6 x2 9 x2 8 x2 7 x2 (c) 6.67 × 10 -10 J (d) 6.67 × 10 -9 J
40. The mass of a spaceship is 1000 kg. It is to be launched 45. If ‘g’ is the acceleration due to gravity on the earth’s
from the earth's surface out into free space. The value of g surface, the gain in the potential energy of an object of
and R (radius of earth) are 10 m/s2 and 6400 km respectively. mass ‘m’ raised from the surface of the earth to a height
The required energy for this work will be [2012] equal to the radius ‘R' of the earth is [2004]
11
(a) 6.4 × 10 Joules 8
(b) 6.4 × 10 Joules
1 1
(c) 6.4 × 109 Joules (d) 6.4 × 1010 Joules (a) mgR (b) mgR (c) 2 mgR (d) mgR
4 2
41. A point particle is held on the axis of a ring of mass m and
radius r at a distance r from its centre C. When released, it 46. Energy required to move a body of mass m from an orbit of
reaches C under the gravitational attraction of the ring. Its radius 2R to 3R is [2002]
speed at C will be [Online May 26, 2012] (a) GMm/12R2 (b) GMm/3R2
(c) GMm/8R (d) GMm/6R.
P-118 Physics

a circular orbit at this height is E2. The value of h for


Motion of Satellites, Escape which E1 and E2 are equal, is: [9 Jan. 2019 II]
TOPIC 5
Speed and Orbital Velocity 3
(a) 1.6 × 10 km (b) 3.2 × 103 km
3
(c) 6.4 × 10 km (d) 28 × 104 km
47. A satellite is in an elliptical orbit around a planet P. It is
observed that the velocity of the satellite when it is farthest 53. Planet A has mass M and radius R. Planet B has half the
from the planet is 6 times less than that when it is closest mass and half the radius of Planet A. If the escape velocities
to the planet. The ratio of distances between the satellite from the Planets A and B are vA and vB, respectively, then
and the planet at closest and farthest points is :
vA n
[NA 6 Sep. 2020 (I)] = .
vB 4 The value of n is : [9 Jan. 2020 II]
(a) 1 : 6 (b) 1 : 3 (c) 1 : 2 (d) 3 : 4
48. A body is moving in a low circular orbit about a planet of (a) 4 (b) 1 (c) 2 (d) 3
mass M and radius R. The radius of the orbit can be taken 54. A satellite of mass m is launched vertically upwards
to be R itself. Then the ratio of the speed of this body in with an initial speed u from the surface of the earth.
the orbit to the escape velocity from the planet is : After it reaches height R (R = radius of the earth), it
1
(a) (b) 2 [4 Sep. 2020 (II)] m
2 ejects a rocket of mass so that subsequently the
10
(c) 1 (d) 2 satellite moves in a circular orbit. The kinetic energy of
49. A satellite is moving in a low nearly circular orbit around the rocket is (G is the gravitational constant; M is the
the earth. Its radius is roughly equal to that of the earth’s mass of the earth): [7 Jan. 2020 I]
radius Re. By firing rockets attached to it, its speed is
instantaneously increased in the direction of its motion so m æ 2 113 GM ö æ 2 119 GM ö
(a) çu + ÷ (b) 5m çè u - ÷
20 è 200 R ø 200 R ø
3
that it become times larger. Due to this the farthest
2 2
3m æ 5GM ö
2
mæ 2GM ö
distance from the centre of the earth that the satellite (c) ç u+ (d) 20 çè u - 3 R ÷ø
reaches is R. Value of R is : [3 Sep. 2020 (I)] 8 è 6 R ÷ø
(a) 4Re (b) 2.5Re (c) 3Re (d) 2Re 55. A spaceship orbits around a planet at a height of 20 km
K from its surface. Assuming that only gravitational field of
50. The mass density of a spherical galaxy varies as over the planet acts on the spaceship, what will be the number
r
a large distance 'r' from its centre. In that region, a small of complete revolutions made by the spaceship in 24 hours
star is in a circular orbit of radius R. Then the period of around the planet ? [Given : Mass of Planet = 8×10 22 kg,
revolution, T depends on R as : [2 Sep. 2020 (I)] Radius of planet = 2×10 6 m, Gravitational constant
G = 6.67×10–11Nm2/kg2] [10 April 2019 II]
1
(a) T 2 µ R (b) T 2 µ R3 (c) T2 µ (d) T µ R (a) 9 (b) 17 (c) 13 (d) 11
R3
56. A rocket has to be launched from earth in such a way that
51. A body A of mass m is moving in a circular orbit of radius it never returns. If E is the minimum energy delivered by
m the rocket launcher, what should be the minimum energy
R about a planet. Another body B of mass collides with that the launcher should have if the same rocket is to be
2
r launched from the surface of the moon? Assume that the
æ vö density of the earth and the moon are equal and that the
A with a velocity which is half çè ÷ø the instantaneous earth’s volume is 64 times the volume of the moon.
2
r
velocity v or A. The collision is completely inelastic. [8 April 2019 II]
Then, the combined body: [9 Jan. 2020 I]
E E E E
(a) continues to move in a circular orbit (a) (b) (c) (d)
64 32 4 16
(b) Escapes from the Planet’s Gravitational field
57. A satellite of mass M is in a circular orbit of radius R about
(c) Falls vertically downwards towards the planet the centre of the earth. A meteorite of the same mass, falling
(d) starts moving in an elliptical orbit around the planet towards the earth collides with the satellite completely in
52. The energy required to take a satellite to a height 'h' elastically. The speeds of the satellite and the meteorite are
above Earth surface (radius of Eareth = 6.4 × 10 3 km) is the same, Just before the collision. The subsequent motion
E1 and kinetic energy required for the satellite to be in of the combined body will be [12 Jan. 2019 I]
Gravitation P-119

(a) such that it escape to infinity 63. An astronaut of mass m is working on a satellite orbiting
(b) In an elliptical orbit the earth at a distance h from the earth's surface. The radius
of the earth is R, while its mass is M. The gravitational pull
(c) in the same circular orbit of radius R
FG on the astronaut is : [Online April 10, 2016]
(d) in a circular orbit of a different radius
(a) Zero since astronaut feels weightless
58. Two satellites, A and B, have masses m and 2m respectively.
A is in a circular orbit of radius R, and B is in a circular orbit GMm GMm
(b) < FG <
of radius 2R around the earth. The ratio of their kinetic (R + h) 2
R2
energies, TA/TB, is : [12 Jan. 2019 II]
1 GMm
(a) (b) 1 (c) FG =
2 (R + h) 2
1
(c) 2 (d) GMm
2 (d) 0 < FG <
R2
59. A satellite is revolving in a circular orbit at a height h from
the earth surface, such that h << R where R is the radius of 64. A very long (length L) cylindrical galaxy is made of
the earth. Assuming that the effect of earth’s atmosphere uniformly distributed mass and has radius R(R < < L). A
can be neglected the minimum increase in the speed star outside the galaxy is orbiting the galaxy in a plane
required so that the satellite could escape from the perpendicular to the galaxy and passing through its centre.
gravitational field of earth is: [11 Jan. 2019 I] If the time period of star is T and its distance from the
galaxy’s axis is r, then : [Online April 10, 2015]
(a) 2gR (b) gR
(a) T µ r (b) Tµ r

(c)
gR
2
(d) gR ( )
2 -1
(c) T µ r2 (d) T2 µ r3
65. What is the minimum energy required to launch a satellite
60. A satellite is moving with a constant speed v in circular of mass m from the surface of a planet of mass M and
orbit around the earth. An object of mass ‘m’ is ejected radius R in a circular orbit at an altitude of 2R? [2013]
from the satellite such that it just escapes from the
gravitational pull of the earth. At the time of ejection, 5GmM 2GmM GmM GmM
(a) (b) (c) (d)
the kinetic energy of the object is: [10 Jan. 2019 I] 6R 3R 2R 2R
(a) 2 m v 2 (b) m v 2 66. A planet in a distant solar system is 10 times more massive
than the earth and its radius is 10 times smaller. Given that
1 3 the escape velocity from the earth is 11 km s–1, the escape
(c) m v2 (d) m v2
2 2 velocity from the surface of the planet would be [2008]
61. Two stars of masses 3 × 1031 kg each, and at distance (a) 1.1 km s–1 (b) 11 km s–1
2 × 1011 m rotate in a plane about their common centre (c) 110 km s–1 (d) 0.11 km s–1
of mass O. A meteorite passes through O moving 67. Suppose the gravitational force varies inversely as the nth
perpen-dicular to the star’s rotation plane. In order to power of distance. Then the time period of a planet in circular
escape from the gravitational field of this double star, orbit of radius ‘R’ around the sun will be proportional to
the minimum speed that meteorite should have at O is:
(Take Gravitational constant G = 66 × 10–11 Nm2 kg–2) æ n -1ö
çè ÷
(a) R n
(b) 2 ø [2004]
R
[10 Jan. 2019 II]
(a) 2.4 × 10 m/s4 (b) 1.4 × 105 m/s æ n +1ö æ n- 2ö
çè ÷ø ç ÷
(c) 3.8 × 104 m/s (d) 2.8 × 105 m/s (c) R 2 (d) Rè 2 ø
62. A satellite is revolving in a circular orbit at a height 'h' from 68. The time period of an earth satellite in circular orbit is
the earth's surface (radius of earth R; h < < R). The minimum independent of [2004]
increase in its orbital velocity required, so that the satellite (a) both the mass and radius of the orbit
could escape from the earth's gravitational field, is close (b) radius of its orbit
to : (Neglect the effect of atmosphere.) [2016]
(c) the mass of the satellite
(a) gR / 2 (b) gR ( 2-1) (d) neither the mass of the satellite nor the radius of its
orbit.
(c) 2gR (d) gR
P-120 Physics

69. A satellite of mass m revolves around the earth of radius R 71. The kinetic energy needed to project a body of mass m from
at a height x from its surface. If g is the acceleration due to the earth surface (radius R) to infinity is [2002]
gravity on the surface of the earth, the orbital speed of the (a) mgR/2 (b) 2mgR (c) mgR (d) mgR/4.
satellite is [2004]
72. If suddenly the gravitational force of attraction between
Earth and a satellite revolving around it becomes zero,
gR 2 gR æ gR2 ö 1/ 2
then the satellite will [2002]
(a) (b) (c) gx (d) ç ÷
R+ x R-x è R + xø (a) continue to move in its orbit with same velocity
70. The escape velocity for a body projected vertically (b) move tangentially to the original orbit in the same
upwards from the surface of earth is 11 km/s. If the body is velocity
projected at an angle of 45°with the vertical, the escape (c) become stationary in its orbit
velocity will be [2003] (d) move towards the earth
(a) 11 2 km / s (b) 22 km/s 73. The escape velocity of a body depends upon mass as
[2002]
11
km / s
(a) m0 (b) m1 (c) m2 (d) m3
(c) 11 km/s (d)
2
Gravitation P-121

dA a + L Gm
1. (c) Areal velocity; Þ F=ò 2
(A + Bx 2 )dx
a x
dt
a+L
1 2 é A ù
dA = r dq = Gm ê - + Bx ú
2 ë x ûa
dA 1 2 dq 1 2 é æ1 1 ö ù
Þ = r = r w = Gm ê A ç -
dt 2 dt 2 è ÷ø + BL ú
ë a a + L û
Also, L = mvr = mr2w
6. (a) As we know, Gravitational force of attraction,
dA 1 L
\ = GMm
dt 2 m F=
2. (c) Let area of ellipse abcd = x R2
x x GM e m GM e M s
Area of SabcS = + (i .e., ar of abca + SacS) F1 = 2
and F2 =
2 4 r1 r22
(Area of half ellipse + Area of triangle)
2GM e m GM e M s
3x DF1 = Dr1 and DF2 = Dr2
= r13 r23
4
c
DF1 mDr1 r23 æ m ö æ r23 ö æ Dr1 ö
= 3 =ç ÷ç ÷ç ÷
DF2 r1 Ms Dr2 è Ms ø è r13 ø è Dr2 ø
d b
S Using Dr1 = Dr2 = 2 Rearth; m = 8 × 1022 kg;
Ms = 2 × 1030 kg
a r1 = 0.4 × 106 km and r2 = 150 × 106 km
3x x 3
Area of SadcS = x - = DF1 æ 8 ´ 1022 ö æ 150 ´ 106 ö
4 4 =ç ÷ç ÷ ´1 @ 2
Area of SabcS 3x / 4 = t1 DF2 è 2 ´ 1030 ø è 0.4 ´ 106 ø
=
Area of SadcS x / 4 t2
Mv 2
t1 7. (d) 2 F cos 45° + F ¢ = (From figure)
= 3 or, t = 3t R
t2 1 2
GM 2 GM 2
3. (b) Where F = and F ¢ =
4. (c) According to Kepler’s law of periods T2 µ R3 ( 2 R) 2 4R2
2 3
æ T2 ö æ R2 ö F
\ ç ÷ = çR ÷ M M
è T1 ø è 1ø F'
3 3 F
æR ö 2 é 4R ù 2 R
Þ T2 = T1 ç 2 ÷ = 5´ ê ú
è R1 ø ë R û o
= 5 × 23 = 40 hours
5. (d) Given l = (A + Bx2),
Taking small element dm of length dx at a distance x M M
from x = 0
dx 2 ´ GM 2 GM 2 Mv 2
x=0 Þ + =
2( R 2)2 4 R2 R

m dF GM 2 é 1 1 ù
so, dm = l dx Þ ê + ú = Mv 2
dm = (A + Bx2)dx R ë4 2û
Gmdm GM æ 2 + 4 ö 1 GM
dF = \ v= ç ÷= (1 + 2 2 )
x 2 R çè 4 2 ÷ø 2 R
P-122 Physics

8. (a) Volume of removed sphere 1


3
S = ut + at 2
4 æRö 4 æ 1ö 2
Vremo = p ç ÷ = pR3 ç ÷ 1
3 è2ø 3 è 8ø \ x5 M = a5 M t 2 ....(ii)
Volume of the sphere (remaining) 2
For mass M
4 3 4 3æ1ö 4 æ7ö u = 0, s = xM, t = t, a = aM
Vremain = pR - pR ç ÷ = pR 3 ç ÷
3 3 è8ø 3 è8ø 1 2
Therefore mass of sphere carved and remaining sphere \ s = ut + at
2
1 7
are at respectively M and M. 1 2
8 8 Þ xM = aM t … (iii)
2
Therefore, gravitational force between these two sphere, Dividing (ii) by (iii)
7M 1 1
GM m G 8 ´ 8 M 7 GM 2 x5 M a5 M t 2 a
1
F= = = = 2 = 5M =
r2 (3R )2 64 ´ 9 R 2 xM 1 aM 5 [From (i)]
aM t 2
2
41 GM 2
; \ 5x5M = xM ....(iv)
3600 R 2
9. (a) As two masses revolve about the common centre of From the figure it is clear that
mass O. x5M + xM = 9R ....(v)
\ Mutual gravitational attraction = centripetal force Where O is the point where the two spheres collide.
From (iv) and (v)
Gm 2 R xM
= mw 2 R + xM = 9 R
( 2R) 2
m m
5
Gm 2
O \ 6xM = 45R
Þ =w
4 R3 45
\ xM = R = 7.5R
Gm 6
Þ w=
4 R3 11. (b) According to question, gh = g d = g1
If the velocity of the two particles with respect to the centre
of gravity is v then
v = wR h = R/2

Gm Gm d
v= 3
´R =
4R 4R
(R-d)
10. (c) We know that
Force = mass × acceleration.
GM GM ( R - d )
9R gh = and g d =
æ Rö
2 R3
xM x5M çè R + ÷ø
R 2R 2
12R
GM GM ( R - d ) 4 (R - d )
The gravitational force acting on both the masses is the 2
= 3
Þ =
æ 3R ö R 9 R
same. çè ÷ø
F1 = F2 2
ma1 = ma2
Þ 4 R = 9 R - 9d Þ 5 R = 9 d
9M 5M
Þ = =5 d 5
95M M \ =
9M 1 R 9
Þ =
95M 5 12. (b) Value of g at equator, g A = g × - Rw2
Let t be the time taken for the two masses to collide and Value of g at height h above the pole,
x5M, xM be the distance travelled by the mass 5M and M
æ 2h ö
respectively. gB = g × ç1 - ÷
For mass 5M è Rø
u = 0, As object is weighed equally at the equator and poles, it
means g is same at these places.
Gravitation P-123

g A = gB -2 -2
æ h ö æ h ö
g h = g ç1 + Þ 4.9 = 9.8 ç 1 +
æ 2h ö
Þ g - Rw2 = g ç1 - ÷ è R e ÷ø è R e ÷ø
è Rø
1 æ h ö
2 gh R2 w2 = ç1 -
2 è R e ÷ø
Þ Rw = 2 Þ h = [as h <<< Re]
R 2g
13. (c) The acceleration due to gravity at a height h is given
by
h = Re ( 2 -1 )
h = 6400 × 0.414 km = 2.6 × 106 m
GM 17. (d) With rotation of earth or latitude, acceleration due to
g=
( R + h)2 gravity vary as g' = g – w2R cos2 f
Where f is latitude, there will be no change in gravity
Here, G = gravitation constant at poles as f = 90°
M = mass of earth At all other points as w increases g' will decreases
The acceleration due to gravity at depth h is hence, weight, W = mg decreases.
GM æ hö 18. (b) Variation of acceleration due to gravity, g with
g' = ç1 - ÷ distance 'd ' from centre of the earth
R2 è Rø
Gm
Given, g = g' If d < R, g = .d i.e., g µ d (straight line)
R2
GM GM æ hö Gm
\ = 2 ç1 - ÷ If d = R, gs =
( R + h)2 R è Rø R2
\ R3 = ( R + h)2 ( R - h) = ( R 2 + h2 + 2hR )( R - h) Gm 1
If d > R, g = 2 i.e., g µ
Þ R 3 = R3 + h2 R + 2hR 2 - R 2 h - h3 - 2h 2 R d d2
Þ h3 + h 2 (2 R - R ) - R 2 h = 0 19. (b) Given that, mass density æç
mass ö
of a spherical
Þ h3 + h2 R - R 2 h = 0 è volume ÷ø
Þ h 2 + hR - R 2 = 0 body r(r) =
k
r
- R ± R2 + 4(1) R2 M k
Þh= = for inside r £ R
2 V r
- R + 5R ( 5 - 1) kv
= = R M= ..... (i)
2 2 r
Inside the surface of sphere Intensity
14. (d) Weight at pole, w = mg = 196 N
Þ m = 19.6 kg GMr F
I= 3 Q I=
Weight at equator, w’ = mg’ = m(g – w2R) R m
é 2p ö
2
ù GMr mg
æ 3 ginside = or I = =g
= 19.6 ê10 – ç ÷ø ´ 6400 ´ 10 ú N 3 m
êë è 24 ´ 3600 úû R
2p ö G kv
æ = . .r = constant From eq. (i),
çèQ w = ÷ø R3 r
T
= 19.6 [10 – 0.034] = 195.33 N
Similarly, gout = GM
We mge 9 ge 9 r2
15. (d) W = mg = 4 or g = 4 Hence, option (2) is correct graph.
p p p
20. (c) We know, g' = g – w2R cos2 q
3g
GM / R 2 9 = g - w2R
or = 4
G (M / 9) / R 2p 4
Given, g ' = g
3
4
g
R w 2R =
\ Rp = 4
2
16. (a) Given g 10
w= =
Acceleration due to gravity at a height h from earth’s 4R 4 ´ 6400 ´ 103
surface is 1
= = 0.6 ´ 10 -3 rad/s
2 ´ 8 ´ 100
P-124 Physics

21. (a) Let A be the point where gravitation field of both planets
22. (a) Acceleration due to gravity at depth d from the cancel each other i.e. zero.
surface of the earth or at a distance r from the centre ‘O’ of
GM G (16 M )
4 2
=
the earth g¢ = prGr x (10a - x )2
g
3
d 1 4
Hence g ' µ r g¢ Þ = Þ 4 x = 10a - x Þ x = 2a ... (i)
x (10a - x)
R r
Using conservation of energy, we have
O r = ( R – d) GMm G (16M )m GMm G (16 M ) m
- - + KE = - -
8a 2a 2a 8a
é 1 16 1 16 ù
KE = GMm ê + - -
GM ë 8a 2a 2a 8a úû
23. (d) On earth’s surface g =
R2 é1 + 64 - 4 - 16 ù
At height above earth’s surface Þ KE = GMm ê úû
ë 8a
GM
gh = 1 2 é 45 ù 90GM
( R + h )2 Þ mv = GMm ê ú Þ v =
2 ë 8a û 8a
gn R2 3 5GM
\ =
g ( R + h)2 Þv=
2 a
2
g /9 é R ù 27. (a) Given : Gravitational field,
Þ =ê
g ë R + h úû Ax
EG = , V¥ = 0
R 1 ( x + a 2 )3/ 2
2
Þ =
R+h 3
\ h = 2R
Vx x
r r
24. (d) Value of g with altitude is, ò dV = - ò EG × d x
V¥ ¥
é 2h ù
gh = g ê1 - ú ; x
ë Rû Ax
Þ Vx - V¥ = - ò dx
Value of g at depth d below earth’s surface, ( x + a 2 )3/ 2
2
¥
é dù
gd = g ê1 - ú A A
ë Rû \ Vx = 2 2 1/ 2
-0=
Equating gh and gd, we get d = 2h (x + a ) ( x + a 2 )1/ 2
2

25. (d) Value of g on earth’s surface,


GM Gr ´ V 28. (d)
g= 2 =
R R2 r
4 dx
G ´ r ´ pR 3 x
3
Þ g=
R2
4
g = rpG. R where r ® average density
3
Mass of small element of planet of radius x and thickness dx.
æ 3g ö
r= ç æ x2 ö
è 4pGR ÷ø dm = r ´ 4px 2 dx = r0 ç1 - 2 ÷ ´ 4px 2 dx
Þ r is directly proportional to g. è R ø
Mass of the planet
26. (d)
r
æ x4 ö
M a A 2a
16M

ò
M = 4pr0 çç x 2 - 2 ÷÷ dx
R ø

x (10a – x) r3 r5
Þ M = 4pr0 - 2
3 5R
10a
Gravitation P-125

Gravitational field,
GM 2 GM 2
ær Þ F = ( 2) +
GM G 3
r ö 5
a2 2a 2
E= 2
= 2
´ 4pr0 ç - 2 ÷ Y
r r è 3 5R ø
D
æ r r3 ö A
Þ E = 4pGr0 ç - 2 ÷ r
ç 3 5R ÷
è ø
O a
dE
E is maximum when =0
dr 45°
æ 1 3r 2 ö B X
dE a C
Þ = 4pGr0 ç - 2 ÷ = 0
dr ç 3 5R ÷
è ø Mv 2
= Resultant force towards centre
5 r
Þr= R
3 Mv 2 GM 2 æ 1ö
\ = 2 ç 2+ ÷
29. (b) Gravitation field at the surface æ a ö a è 2ø
çè ÷ø
Gm 2
E=
r2 GM æ 1 ö
Þ v2 = çè1 + ÷
Gm1 Gm2 a 2 2ø
\ E1 = and E2 =
r12 r22 GM æ 1 ö GM
From the diagram given in question, Þ v= ç 1+ ÷ = 1.16
a è 2 2 ø a
E1 2
= GMm r(dV )m
E2 3 (r1 = 1m, R2 = 2m given) 33. (a) F = =òa
2
r r2
2
E1 æ r2 ö æ m1 ö 2 æ 2ö æ m ö R
\ =ç ÷ ç ÷ Þ =ç ÷ ç 1÷ k 4 pr 2 dr
E2 è r1 ø è m2 ø 3 è 1 ø è m2 ø = mG ò 2
0r r2
æ m1 ö 1
Þ çè m ÷ø = 6
R
æ 1ö
2 = - 4pkGm ç ÷
è rø0
30. (16.00)
Using law of conservation of energy 4pkGm
=-
Total energy at height 10 R = total energy at earth R
GM E m 1 GM E m 1 Using Newton’s second law, we have
– + mV02 = - + mV 2
10 R 2 R 2
mv02 4pkGm
é GMm ù =
êëQ Gravitational potential energy = – r úû R R
or v0 = C (const.)
GM E æ 1 ö V02 V 2
Þ çè 1– ÷ø + = 2pR 2pR
R 10 2 2 Time period, T = v = C
0
9
Þ V 2 = V02 + gR T
5 or = = constant.
R
9
Þ V = V02 + gR » 16 km / s GMm
5 34. (c) Initial gravitational potential energy, Ei = –
2R
[Q V0 = 12 km/s given] Final gravitational potential energy,
GM G (2 M ) GM GMm / 2 GMm / 2 GMm GMm
Eg = +
31. (c) 2 2 = Ef = – – = – –
(3a ) (3a ) 3a 2 æRö æ 3R ö 2R 6R
2ç ÷ 2ç ÷
AC a 2 a è ø
2 è 2 ø
32. (b) AC = a 2 Q r== =
2 2 2 4GMm 2GMm
= – =-
Resultant force on the body 6R 3R
GM 2 ˆ GM 2 ˆ GM 2 \ Difference between initial and final energy,
B= i + 2 j+ (cos 45°iˆ + sin 45° ˆj ) GMm æ 2 1 ö GMm
a2 a (a 2)2 Ef – Ei = ç– + ÷ = –
R è 3 2ø 6R
P-126 Physics

35. (d) Due to complete solid sphere, potential at point P


By conservation of linear momentum
-GM é 2 æ R ö

Vsphere = ê3R - ç ÷ ú v1 m m
m1v1 + m2 v 2 = 0 or = - 2 Þ v2 = – 1 v1
2R 3 êë è2ø úû v2 m1 m2
Putting value of v2 in equation (1), we get
-GM æ 11R 2 ö GM
= ç ÷ = -11 2
3 ç 4 ÷ 8R æ m v ö 2Gm1m 2
2R è ø m1v12 + m 2 çç - 1 1 ÷÷ =
Solid è m 2 ø d
sphere 2 2 2
m1m 2 v1 + m1 v1 2Gm1m 2
=
m2 d
2Gm 22 2G
P v1 = = m2
d (m1 + m 2 ) d (m1 + m 2 )
Cavity
2G
Similarly v 2 = - m1
d ( m1 + m 2 )
Due to cavity part potential at point P 39. (c) Let mass of smaller sphere (which has to be removed) is m
GM
R
3 8 3GM Radius = (from figure)
Vcavity = - =- 2
2 R 8R
2 M m
So potential at the centre of cavity =
4 3 4 æ R ö3
= Vsphere - Vcavity pR pç ÷
3 3 è2ø
11GM æ 3 GM ö -GM
=- -ç- ÷= M
8R è 8 R ø R Þm=
8
GM 2 2 Mass of the left over part of the sphere
36. (c) As, V = – 3 (3R – r )
2R M 7
Graph (c) most closely depicts the correct variation of v(r). M' = M - = M
8 8
37. (
(d) Gravitational field, I = 5iˆ + 12ˆj N/kg ) Therefore gravitational field due to the left over part of the
sphere
dv
I =- GM ' 7 GM
dr = 2 =
x 8 x2
éx y ù 40. (d) The work done to launch the spaceship
v = - ê ò I x dx + ò I y dy ú ¥ ur uur ¥
êë 0 úû GMm
0 W = - ò F × dr = - ò 2 dr
r
= – éë I x .x + I y .y ùû R R
GMm
= – éë5 ( 7 - 0 ) + 12 ( -3 - 0 ) ùû W =+
R
= - éë35 + ( -36 ) ùû = 1 J / kg … (i)
The force of attraction of the earth on the spaceship, when
i.e., change in gravitational potential 1 J/kg. it was on the earth's surface
Hence change in gravitational potential energy 1 J
38. (b) We choose reference point, infinity, where total GMm
F=
energy of the system is zero. R2
So, initial energy of the system = 0 GMm GM
Þ mg = g= 2
1 1 Gm 1m 2 2 Þ R
… (ii)
Final energy = m 1 v12 + m 2 v 22 - R
2 2 d Substituting the value of g in (i) we get
From conservation of energy, gR 2m
Initial energy = Final energy W=
R
1 1 Gm 1 m 2 Þ W = mgR
\0= m 1 v12 + m 2 v 2 2 -
2 2 d Þ W = 1000 × 10 × 6400 × 103
1 1 Gm 1 m 2 = 6.4 × 1010 Joule
or m 1 v12 + m 1 v 22 = ...(1)
2 2 d
Gravitation P-127

41. (c) Let 'M' be the mass of the particle 46. (d) Gravitational potential energy of mass m in an orbit
Now, Einitial = Efinal of radius R
GMm GM m 1 GMm
i.e. +0= + MV 2 u= –
2r r 2 R
Energy required = potential energy at 3R – potential energy
1 2 GMm é 1 ù a 2R
or, 2 MV = r ê1 - ú
ë 2û -GMm æ -GMm ö

1 2 Gm é 1 ù =
3R è 2R ÷ø
Þ 2 V = r ê1 - ú
ë 2û
-GMm GMm
= +
2Gm æ 1 ö 3R 2R
or, V = ç 1- ÷
r è 2ø -2GMm + 3GMm GMm
= =
42. (c) Let P be the point where gravitational field is zero. 6R 6R
Gm 4Gm 47. (a) By angular momentum conservation
\ x 2 = (r - x )2
rmin vmax = mrmax vmin
1 2 r
Þ = Þ r – x = 2x Þ x=
x r-x 3 vmin
m P 4m
x rmin rmax
r
Gravitational potential at P, planet
V =-
Gm 4Gm
- =-
9Gm vmax
r 2r r
3 3 vmax
GM Given, vmin =
43. (b) Gravitational field, E = – 6
r2
uuur r 2 \
rmin vmin 1
= =
Flux, f = ò E g × dS =| E × 4p r |= -4p GM rmax vmax 6
where, M = mass enclosed in the closed surface 48. (a) Orbital speed of the body when it revolves very close
r 1 to the surface of planet
This relationship is valid when | E g | µ 2 .
r GM
GMm V0 = ...(i)
44. (c) Initial P.E. Ui = – R
R Here, G = gravitational constant
When the particle is far away from the sphere, the P.E. of Escape speed from the surface of planet
the system is zero.
\ Uf = 0 2GM
Ve = ...(ii)
é -GMm ù R
W = DU = U f - U i = 0 - ê
ë R úû
Dividing (i) by (ii), we have

6.67 ´10 -11 ´ 100 10 GM


W= ´ = 6.67 × 10–10 J V0 R 1
0.1 1000 = =
45. (b) On earth’s surface potential energy, Ve 2GM 2
R
GmM
U= 49. (c) 3
R hv = V
2
At a height R from the earth's surface, P.E. of system =
GmM Re
-
2R
-GmM GmM
\ DU = + ;
2R R
GmM
Þ DU =
2R Rmax
GM GM 1
Now 2 = g ; \ = gR \ DU = mgR
R R 2 Vmin.
P-128 Physics

GM GMe m GMe m
Orbital velocity, V0 = – + E1 = –
Re Re ( Re + h )
From energy conversation, æ 1 1 ö GMe m h
Þ E1 = GMe m ç – Þ E1 = ´
2 è R e R e + h ÷ø ( Re + h) Re
GMm 1 æ 3 ö GMm 1 2 Gravitational attraction
- + mç V÷ = + mVmin ...(1)
Re 2 è 2 ø Rmax 2
mv 2 GM em
From angular momentum conversation FG = mac = =
( R e + h) ( Re + h )2
3
VRe = Vmin Rmax ...(2) GM e m
2 mv 2 =
(Re + h)
Solving equation (1) and (2) we get,
Rmax = 3Re mv2 GMe m
E2 = =
50. (a) According to question, mass density of a spherical 2 2( Re + h )
E1 = E2
k
galaxy varies as .
r Clearly, h = 1 Þ h = R e = 3200km
Re 2 2
Mass, M = ò rdV
2GM A
r = R0 53. (a) Escape velocity of the planet A is VA = RA
k
ÞM = ò r
4pr 2 dr
where MA and RA be the mass and radius of the planet
0
A.
R0 According to given problem
Þ M = 4pk ò r dr M
m MA R
0
R MB = , RB = A
2 2
4pkR02 2GM A
or, M = = 2pkR2 MA
2 2G VA RA n
\ VB = 2 \ = = =1
GMm RA VB 2GM A 2 4
FG = = mw02 R (= FC ) 2 RA 2
R02
Þ n=4
4pkR 2
G æ 2p ö M M
2pKG
çQ w =
Þ 2 = w2 R Þ w = R
0 0 ÷ 54. (b) R u Þ R v
R2 R è T ø
m
2p 2p R 2pR 2pR 1 – GMm 1 2 –GMm
\T = = = Þ T2 = mu 2 + = mv +
w0 2pKG KG KG 2 R 2 2R
Q 2p, K and G are constants 1 – GMm
Þ m( v 2 – u 2 ) =
\ T 2 µ R. 2 2R
r r GM
51. (d) From law of conservation of momentum, pi = p f Þ V = V = u2 – ...(i)
R
m1u1 + m2u2 = MVf
GM m´v
æ mv ö v0 = \ vrad = = 10 v
çè mv + ÷ 2R æ mö
4 ø 5v çè ÷ø
Þ vf = = 10
3m 6
2 m
Clearly, vf < vi \ Path will be elliptical Ejecting a rocket of mass
10
52. (b) K.E. of satellite is zero at earth surface and at height
9m GM m GM
h from energy conservation \ ´ = ´ vt Þ Vt2 = 81
Usurface + E1 = Uh 10 2 R 10 2R
Gravitation P-129

Kinetic energy of rocket,


E
KE rocket =
1M 2
2 10
(
VT + Vr2 ) or E’ =
16
r
57. (b) mviˆ + mvjˆ = 2mv
1 m æ 2 GM GM ö
= ´ ´ ç (u – )100 + 81 ÷ r v v
2 10 è R R ø Þ v = ˆi + ˆj
2 2
m æ GM 81 GM ö
= ´ 100 ç u 2 – + ÷ r æ vö æ vö
2
v
2
20 è R 200 R ø Þ v= ç ÷ +ç ÷ =
è 2ø è 2ø 2
vT
1 GM
M M = ´
2 R
10 vR
2R GM
9M 58. (b) Orbital, velocity, v =
10 r
Kinetic energy of satellite A,
æ 119 GM ö
= 5m ç u 2 – ÷ 1 2
è 200 R ø TA = m A VA
2
55. (d) Time period of revolution of satellite, Kinetic energy of satellite B,
2 pr 1 2
T= TB = m BVB
v 2
GM GM
v= m´
TA R
r Þ T = GM = 1
B 2m ´
2R
r r3
\ T = 2p r = 2p 59. (d) For a satellite orbiting close to the earth, orbital
GM GM
velocity is given by
Substituting the values, we get
v0 = g(R + h) » gR
(202)3 ´ 1012 Escape velocity (ve) is
T = 2p sec
6.67 ´ 10 -11 ´ 8 ´ 1022
ve = 2g(R + h) » 2gR [Q h <<R]
T = 7812.2 s
T ; 2.17 hr Þ 11 revolutions. Dv = ve - v0 = ( 2 - 1) gR
56. (d) Escape velocity, 60. (b) At height r from center of earth, orbital velocity
2GM 2GrV
vc = = GM
R R v=
r
2GS ´ 4pR3 8 By principle of energy conservation
= = prGR 2
R 3 æ GMm ö
KE of ‘m’ + çè – ÷ =0+0
r ø
' 8 2 (Q At infinity, PE = KE = 0)
For moon, vc = prGRm
3 2
GMm æ GM ö
or KE of ‘m’ = =ç m = mv2
4 3 4 3 R r è r ÷ø
Given, pR = 64 ´ pRm or Rm = 61. (d) Let M is mass of star m is mass of meteroite
3 3 4
By energy convervation between 0 and ¥.
2
' 8 æ Rö v
\ ve = prG ç ÷ = c GMm –GMm 1
3 è 4ø 4 – + + m Vese2 = 0 + 0
r r 2
1 2
mve v2 v
E 4 ´ 6.67 ´ 10 –11 ´ 3 ´ 1031
= 2 = 'e2 = e = 16 \v=
4GM
=
E' 1 æ ve ö
mv 'e2 vc çè 4 ÷ø
r 1011
2
; 2.8 ´105 m / s
P-130 Physics

67. (c) Gravitational force, F = KR–n


62. (b) For h << R, the orbital velocity is gR
This force provides the centripetal force MRw2 to the
Escape velocity = 2gR planet at height h above earth’s surface.
\ The minimum increase in its orbital velocity \ F = KR–n = MRw2
Þ w2 = KR–(n+1)
= 2gR – gR = gR ( 2 – 1) - ( n +1)

63. (c) According to universal law of Gravitation, Þ w= KR 2


- (n +1)
GMm 2p
Gravitational force F = µR 2
(R + h)2 T
Astronaut + ( n +1)
\T µR 2
68. (c) Time period of satellite is given by
h
( R + h )3
T = 2p
GM
Where R + h = radius of orbit of satellite
R M = mass of earth.
Time period is independent of mass of satellite.
Earth 69. (d) Gravitational force provides the necessary centripetal
force.
\ Centripetal force on a satellite = Gravitational force
2GM mv 2 2GM
64. (a) F= m or, = m
Lr r Lr mv 2 GmM GM
\ = also g = 2
2 ( R + x) ( R + x) 2
R
æ 2p ö 2GMm é 2p ù
mr ç ÷ = êQv = r w and w = T ú
è øT Lr ë û mv 2 æ GM ö R 2 n!
\ = mç
Þ T µr ( R + x) è R 2 ÷ø ( R + x )2 r !( n - r ) !
65. (a) As we know,
-GMm mv 2 R2
Gravitational potential energy = \ = mg
r ( R + x) ( R + x) 2
and orbital velocity, v0 = GM / R + h 1/2
gR 2
2
æ gR 2 ö
1
Ef = mv02 -
GMm 1 GM GMm
= m -
\v = Þ v = çç ÷÷
2 3R 2 3R 3R R+ x èR+xø
GMm æ 1 ö - GMm 70. (c) ve = 2 gR
= ç - 1÷ =
3R è 2 ø 6R Clearly escape velocity does not depend on the angle at
which the body is projected.
-GMm
Ei = +K 1 2
R 71. (c) K.E = m ve
Ei = Ef 2
5GMm Here ve = escape velocity is independent of mass of the
Therefore minimum required energy, K = body
6R
66. (c) Escape velocity on earth, Escape velocity, ve = 2gR
Substituting value of ve in above equation we get
2GMe
ve = = 11 km s–1 1
Re K.E = m ´ 2 gR = mgR
2
2GM p 72. (b) Due to inertia of motion it will move tangentially to
Rp the original orbit with the same velocity.
(ve ) p Mp Re
\ = = ´ = 2GM
(ve )e 2GM e Me Rp 73. (a) Escape velocity, ve = 2 gR =
Re R
Þ ve µ m0
10M e Re Where M, R are the mass and radius of the planet
= ´ = 10
Me R e /10 respectively. Clearly, escape velocity is independent of
mass of the body
\ (ve ) p = 10 ´ (ve )e = 10 ´ 11 = 110 km / s
8
Mechanical Properties of Solids P-131

Mechanical Properties
of Solids
5. The elastic limit of brass is 379 MPa. What should be the
TOPIC 1 Hooke's Law & Young's Modulus minimum diameter of a brass rod if it is to support a 400
N load without exceeding its elastic limit?
1. If the potential energy between two molecules is given by [10 April 2019 II]
A B (a) 1.00 mm (b) 1.16 mm
U = - 6 + 12 , then at equilibrium, separation between
r r
molecules, and the potential energy are: [Sep. 06, 2020 (I)] (c) 0.90 mm (d) 1.36 mm
1 1
6. A steel wire having a radius of 2.0 mm, carrying a load of
A2
(a) æç ÷ , -
ö6
(b) æç ÷ , 0
B Bö 6
è 2 Aø è Aø 4kg, is hanging from a ceiling. Given that g = 3.1 À ms–2,
2B
1 1 what will be the tensile stress that would be developed in
æ 2B ö 6 A2 æ 2B ö 6 A2 the wire? [9 April 2019 I]
(c) ç ÷ , - (d) ç ÷ , -
è Aø 4B è Aø 2B (a) 6.2 × 106 Nm–2 (b) 5.2 × 106 Nm–2
2. A body of mass m = 10 kg is attached to one end of a wire of (c) 3.1 × 106 Nm–2 (d) 4.8 × 106 Nm–2
length 0.3 m. The maximum angular speed (in rad s–1) with 7. A steel wire having a radius of 2.0 mm, carrying a load of
which it can be rotated about its other end in space station is 4kg, is hanging from a ceiling. Given that g = 3.1 À ms–2,
(Breaking stress of wire = 4.8 × 107 Nm–2 and area of cross- what will be the tensile stress that would be developed in
section of the wire = 10–2 cm2) is _______ . the wire? [8 April 2019 I]
[9 Jan 2020 I] 6
(a) 6.2 × 10 Nm –2 (b) 5.2 × 10 Nm–2
6

3. A uniform cylindrical rod of length L and radius r, is made 6


(c) 3.1 × 10 Nm –2 (d) 4.8 × 106 Nm–2
from a material whose Young’s modulus of Elasticity 8. Young’s moduli of two wires A and B are in the ratio 7 : 4.
equals Y. When this rod is heated by temperature T and
Wire A is 2 m long and has radius R. Wire B is 1.5 m long
simultaneously subjected to a net longitudinal
compressional force F, its length remains unchanged. The and has radius 2 mm. If the two wires stretch by the same
coefficient of volume expansion, of the material of the length for a given load, then the value of R is close to :
rod, is (nearly) equal to : [12 April 2019 II] [8 April 2019 II]
(a) 9F/(pr 2YT) (b) 6F/(pr 2YT (a) 1.5 mm (b) 1.9 mm (c) 1.7 mm (d) 1.3 mm
(c) 3F/(pr 2YT) (d) F/(3pr 2YT) 9. As shown in the figure, forces of 105N each are applied in
4. In an environment, brass and steel wires of length 1 m opposite directions, on the upper and lower faces of a
each with areas of cross section 1 mm2 are used. The cube of side 10cm, shifting the upper face parallel to itself
wires are connected in series and one end of the combined by 0.5cm. If the side of another cube of the same material
wire is connected to a rigid support and other end is is, 20cm, then under similar conditions as above, the
subjected to elongation. The stress required to produce a displacement will be: [Online April 15, 2018]
net elongation of 0.2 mm is, F
[Given, the Young’s modulus for steel and brass are,
respectively, 120×109N/m2 and 60×109N/m2]
[10 April 2019 II] F
(a) 1.2×106 N/m 2 (b) 4.0×106 N/m 2
(a) 1.00cm (b) 0.25cm
(c) 1.8×106 N/m2 (d) 0.2×106 N/m2 (c) 0.37cm (d) 0.75cm
P-132 Physics

10. A thin 1 m long rod has a radius of 5 mm. A force of 50 pkN 14. A copper wire of length 1.0 m and a steel wire of length
is applied at one end to determine its Young's modulus. 0.5 m having equal cross-sectional areas are joined end to
Assume that the force is exactly known. If the least count end. The composite wire is stretched by a certain load
in the measurement of all lengths is 0.01 mm, which of the which stretches the copper wire by 1 mm. If the Young’s
following statements is false ? [Online April 10, 2016] modulii of copper and steel are respectively 1.0 × 1011 Nm–
2 and 2.0 × 1011 Nm–2, the total extension of the composite
(a) The maximum value of Y that can be determined is
2 × 1014N/m2. wire is : [Online April 23, 2013]
DY (a) 1.75 mm (b) 2.0 mm (c) 1.50 mm (d) 1.25 mm
(b) gets minimum contribution from the uncertainty 15. A uniform wire (Young’s modulus 2 × 1011 Nm–2) is
Y
in the length subjected to longitudinal tensile stress of 5 × 107 Nm–2. If
the overall volume change in the wire is 0.02%, the
DY
(c) gets its maximum contribution from the fractional decrease in the radius of the wire is close to :
Y [Online April 22, 2013]
uncertainty in strain
(a) 1.0 × 10–4 (b) 1.5 × 10–4
(d) The figure of merit is the largest for the length of the
(c) 0.25 × 10–4 (d) 5 × 10–4
rod.
16. If the ratio of lengths, radii and Young's moduli of steel
11. A uniformly tapering conical wire is made from a material
and brass wires in the figure are a, b and c respectively,
of Young's modulus Y and has a normal, unextended length
then the corresponding ratio of increase in their lengths is :
L. The radii, at the upper and lower ends of this conical
[Online April 9, 2013]
wire, have values R and 3R, respectively. The upper end of
the wire is fixed to a rigid support and a mass M is
suspended from its lower end. The equilibrium extended Steel
length, of this wire, would equal : [Online April 9, 2016]
M
æ 2 Mg ö æ 1 Mg ö
L ç1 + L ç1 +
(a) è 9 pYR 2 ÷ø (b) è 9 pYR 2 ÷ø Brass

æ 1 Mg ö æ 2 Mg ö
L ç1 + L ç1 + 2M
(c) è 3 pYR 2 ÷ø (d) è 3 pYR 2 ÷ø
12. The pressure that has to be applied to the ends of a steel 3c 2a 2 c 3a 2ac
(a) (b) (c) 2
(d)
wire of length 10 cm to keep its length constant when its 2ab2 b 2b c b2
temperature is raised by 100ºC is: 17. A steel wire can sustain 100 kg weight without breaking. If
the wire is cut into two equal parts, each part can sustain
(For steel Young’s modulus is 2 ´ 1011 Nm -2 and a weight of [Online May 19, 2012]
coefficient of thermal expansion is 1.1 ´ 10-5 K -1 ) [2014] (a) 50 kg (b) 400 kg (c) 100 kg (d) 200 kg
18. A structural steel rod has a radius of 10 mm and length of
(a) 2.2 ´ 108 Pa (b) 2.2 ´ 10 9 Pa 1.0 m. A 100 kN force stretches it along its length. Young’s
modulus of structural steel is 2 × 1011 Nm–2. The percentage
(c) 2.2 ´ 10 7 Pa (d) 2.2 ´ 106 Pa
strain is about [Online May 7, 2012]
13. Two blocks of masses m and M are connected by means (a) 0.16% (b) 0.32% (c) 0.08% (d) 0.24%
of a metal wire of cross-sectional area A passing over a 19. The load versus elongation graphs for four wires of same
frictionless fixed pulley as shown in the figure. The system length and made of the same material are shown in the
is then released. If M = 2 m, then the stress produced in figure. The thinnest wire is represented by the line
the wire is : [Online April 25, 2013] Load D [Online May 7, 2012]
C
B
A

T
O Elongation
m (a) OA (b) OC (c) OD (d) OB
T
20. Two wires are made of the same material and have the
same volume. However wire 1 has cross-sectional area A
M
and wire 2 has cross-sectional area 3A. If the length of wire
1 increases by Dx on applying force F, how much force is
2mg 4mg mg 3mg needed to stretch wire 2 by the same amount? [2009]
(a) (b) (c) (d) (a) 4 F (b) 6 F (c) 9 F (d) F
3A 3A A 4A
Mechanical Properties of Solids P-133

21. A wire elongates by l mm when a load W is hanged from it. If a


the wire goes over a pulley and two weights W each are hung
at the two ends, the elongation of the wire will be (in mm) b
[2006]
(a) l (b) 2l (c) zero (d) l/2

Bulk and Rigidity Modulus and


TOPIC 2
Work Done in Stretching a Wire
(a) (pmB b) a (b) (2pmBb) Da
22. Two steel wires having same length are suspended from a
(c) (pmB b) Da (d) (4 pmB b) Da
ceiling under the same load. If the ratio of their energy 26. Steel ruptures when a shear of 3.5 × 108 N m–2 is applied.
stored per unit volume is 1 : 4, the ratio of their diameters The force needed to punch a 1 cm diameter hole in a steel
is: [9 Jan 2020 II] sheet 0.3 cm thick is nearly: [Online April 12, 2014]
(a) 2 :1 (b) 1 : 2 (a) 1.4 × 104 N (b) 2.7 × 104 N
(c) 3.3 × 104 N (d) 1.1 × 104 N
(c) 2 : 1 (d) 1 : 2 27. The bulk moduli of ethanol, mercury and water are given
23. A boy’s catapult is made of rubber cord which is 42 cm as 0.9, 25 and 2.2 respectively in units of 109 Nm–2. For a
long, with 6 mm diameter of cross-section and of negligible given value of pressure, the fractional compression in
mass. The boy keeps a stone weighing 0.02 kg on it and DV
stretches the cord by 20 cm by applying a constant force. volume is . Which of the following statements about
V
When released, the stone flies off with a velocity of 20 ms–
1. Neglect the change in the area of cross-section of the DV
for these three liquids is correct ?[Online April 11, 2014]
cord while stretched. The Young’s modulus of rubber is V
closest to : [8 April 2019 I] (a) Ethanol > Water > Mercury
(a) 106 N/m–2 (b) 104 N/m–2 (b) Water > Ethanol > Mercury
(c) 108 N/m–2 (d) 103 N/m–2 (c) Mercury > Ethanol > Water
24. A solid sphere of radius r made of a soft material of bulk (d) Ethanol > Mercury > Water
modulus K is surrounded by a liquid in a cylindrical 28. In materials like aluminium and copper, the correct order of
container. A massless piston of area a floats on the surface magnitude of various elastic modului is:
of the liquid, covering entire cross-section of cylindrical [Online April 9, 2014]
container. When a mass m is placed on the surface of the (a) Young’s modulus < shear modulus < bulk modulus.
piston to compress the liquid, the fractional decrement in
(b) Bulk modulus < shear modulus < Young’s modulus
æ dr ö (c) Shear modulus < Young’s modulus < bulk modulus.
the radius of the sphere çè ÷ø ,is : [2018]
r (d) Bulk modulus < Young’s modulus < shear modulus.
Ka Ka 29. If ‘S’ is stress and ‘Y’ is young’s modulus of material of a
mg mg
(a) (b) (c) (d) wire, the energy stored in the wire per unit volume is [2005]
mg 3mg 3Ka Ka
2 2Y
25. A bottle has an opening of radius a and length b. A cork of S S
(a) (b) 2S 2Y (c) (d)
length b and radius (a + Da) where (Da < < a) is compressed 2Y 2Y S2
to fit into the opening completely (see figure). If the bulk 30. A wire fixed at the upper end stretches by length l by
modulus of cork is B and frictional coefficient between the applying a force F. The work done in stretching is [2004]
bottle and cork is m then the force needed to push the cork
F Fl
into the bottle is : [Online April 10, 2016] (a) 2Fl (b) Fl (c) (d)
2l 2
P-134 Physics

-A B Stress
1. (c) Given : U = 6
+ 12 Young modulus, Y =
r r æ Dl ö
çè ÷ø
For equilibrium, L
dU Let s be the stress
F= = -( A( -6r -7 )) + B ( -12r -13 ) = 0
dr
sL1 sL2
6 A 12 B
Þ 0 = 7 - 13 Þ
6A
= 6
1 Total elongation Dlnet = Y + Y
1 2
r r 12 B r
1/ 6
æ 2B ö é1 1ù
\ Separation between molecules, r = ç ÷ Dlnet = s ê + ú [Q L1 = L2 = 1m]
è Aø
ë Y1 Y2 û
Potential energy,
æ æ YY ö
æ 2B ö ö
1/ 6
A B s = Dl ç 1 2 ÷
U çr = ç ÷ ÷ = - + 2 2 è Y1 + Y2 ø
è è A ø ø 2 B / A 4B / A
- A2 A2 - A2 æ 120 ´ 60 ö N
+= = = 0.2 ´ 10-3 ´ ç ´ 109 = 8 ´ 106 2
2B 4B 4B è 180 ÷ø m
2. (4) Given : Wire length, l = 0.3 m
F 400 ´ 4
Mass of the body, m = 10 kg 5. (b) Stress = = 2
= 379 ´106 N/m 2
A pd
Breaking stress, s = 4.8 × 107 Nm–2
400 ´ 4
Area of cross-section, a = 10–2 cm2 Þ d2 =
Maximum angular speed w = ? 379 ´ 106 p
d = 1.15 mm
T = Mlw2
6. (c) Given,
T ml w 2 Radius of wire, r = 2 mm
s= = Mass of the load m = 4 kg
A A
F mg
Stress = A =
ml w 2 7 2
£ 48 ´ 10 Þ w £
48 ´ 107 A ( ) p(r ) 2
A ml 4 ´ 3.1p
= = 3.1 ´ 106 N/m2
p ´ (2 ´ 10-3 )2

Þ w2 £
( 48 ´ 10 )(10 ) = 16 Þ w
7 -6
= 4 rad/s
7. (c) Given,
10 ´ 3
max Radius of wire, r = 2 mm
Mass of the load m = 4 kg
3. (c) Dtemp = Dforce
F mg 4 ´ 3.1p
FL FL F Stress = = = = 3.1 ´ 106 N/m2
or La (DT) = \ a= = 2 A p (r ) 2 p ´ (2 ´ 10 -3 ) 2
AY AYT pr YT
8. (c) D1 = D2
Coefficient of volume expression
Fl1 Fl2 2 1.5
3F or = or = 2
r = 3a = . pr12 y1 pr22 y2 2
R ´7 2 ´4
pr 2YT
4. (Bonus) \ R = 1.75 mm
9. (b) For same material the ratio of stress to strain is same
For first cube
Brass Steel F force1 105
Stress1 = =
area1 (0.12 )
Mechanical Properties of Solids P-135

At equilibrium change in length of the wire


change in length1 0.5 ´ 10-2
Strain1 = = 1
original length1 0.1 Mg dx
For second block,
ò dL < ò é 2R ù2
0 pê x ∗ Rú y
êë L úû
force2 105
stress2 = =
area 2 (0.22 ) Taking limit from 0 to L

é ù
strain2 =
change in length 2
=
x ê ú
ê ú
original length 2 0.2 Mg ê 1 L ú MgL
ΧL < ê, ≥ ú<
x is the displacement for second block. p y ê é 2Rx ù L 2R ú 3p R 2 y
ê ê ∗ Rú ú
ê êë L úû 0 ú
stress1 stress2 ë û
For same material, =
strain1 strain 2
The equilibrium extended length of wire = L + DL
MgL æ 1 Mg ÷ö
10.5 105 < L∗ < L çç1 ∗ ÷
3p R 2 Y èç 3 p YR 2 ÷ø
(0.1)2 (0.2)2
or, =
0.5 ´ 10 -2 x stress
0.2 12. (a) Young's modulus Y =
0.1 strain
Solving we get, x = 0.25 cm stress = Y ´ strain
Stress in steel wire = Applied pressure
F Dl Pressure = stress = Y × strain
10. (a) Young's modulus Y = /
A l
DL
Strain = = α DT
Fl L
Y<
pr 2Χl (As length is constant)
Given, radius r = 5mm, force F = 50pk N, = 2 × 1011 × 1.1 × 10–5 × 100 = 2.2 × 108 Pa
l æ 2mM ö
< 0.01 mm 13. (b) Tension in the wire, T = ç ÷g
Χl èm+Mø
Force / Tension 2mM
Fl Stress = = g
\ Y= = 2 × 1014 N / m2. Area A(m + M)
pr lD 2
2(m ´ 2m)g
11. (c) Consider a small element dx of radius r, = (M = 2 m given)
A(m + 2m)
2R
r< x∗R 4m 2 4mg
L = g=
3mA 3A
14. (d) Yc ´ ( DLc / Lc ) = Ys ´ (DLs / Ls )
R

æ 1´ 10-3 ö 11 æ DL ö
Þ 1´1011 ´ ç ÷ = 2 ´10 ´ ç s ÷
x ç 1 ÷ è 0.5 ø
è ø
r
0.5 ´10-3
dx
\ DLs = = 0.25 mm
2
Therefore, total extension of the composite wire
L
= DLc + DLs
= 1 mm + 0.25 m = 1.25 m

3R
Mg
P-136 Physics

19. (a) From the graph, it is clear that for the same value of
15. (c) Given , y = 2 ´ 1011 Nm -2
load, elongation is maximum for wire OA. Hence OA is the
æ Fö thinnest wire among the four wires.
Stress ç ÷ = 5 ´ 107 Nm -2
è Aø l
DV = 0.02% = 2 × 10–4 m3 20. (c) A Y
Dr Wire (1)
=?
r

stress æ Dl ö g
g= Þ strain ç ÷ = … (i) 3A Y
strain è l 0 ø stress

DV = 2prl 0 Dr - pr 2 Dl … (ii) l/3


Wire (2)
From eqns (i) and (ii) putting the value of Dl , l 0 and DV
For wire 1
and solving we get Length, L1 = l
Dr Area, A1 = A
= 0.25 ´10-4 For wire 2
r
16. (c) According to questions, l
Length, L2 =
3
ls r y Dls
= a, s = b, s = c, =? Area, A2 = 3A
lb rb yb Dl b
As the wires are made of same material, so they will have
Fl Fl same young’s modulus.
As, y = ADl Þ Dl = Ay For wire 1,
F/A
Y= ...(i)
3mgl s D x/l
Dl s = [Q Fs = (M + 2M)g] For wire 2 ,
prs2 .ys
F '/ 3 A
2Mgl b Y= ...(ii)
Dl b = Dx /( l / 3)
[Q Fb = 2Mg]
prb2 .y b From (i) and (ii) we get,
3Mgl s F l F' l
´ = ´ Þ F¢ = 9F
Dl s pr 2 .y 3a A Dx 3 A 3Dx
= s s = 2
\ Dl b 2Mg.l b 2b C 21. (a) Case (i)
prb2 .y b

17. (c) Breaking force a area of cross section of wire


Load hold by wire is independent of length of the wire.
18. (a) Given: F = 100 kN = 105 N
T
Y = 2 × 1011 Nm–2
l0 = 1.0 m
T T
radius r = 10 mm = 10– 2 m
W
Stress
From formula, Y =
Strain
W W
Stress F
Þ Strain = =
Y AY At equilibrium, T = W

105 105 1 W/A


= Young’s modules, Y = .....(1)
= -4 11 = 628 l/L
pr 2Y 3.14 ´ 10 ´ 2 ´ 10
1 W L
Therefore % strain = ´100 = 0.16% Elongation, l = ´
628 A Y
Mechanical Properties of Solids P-137

Case (ii) At equilibrium T = W dV 3dr


Fractional change in volume = ...(ii)
W/A V r
\ Young’s moduls, Y =
l/2 3dr mg
L/2 Using eq. (i) & (ii) =
r Ka
W/A W L dr mg
Þ Y= Þ l= ´
\ = (fractional decrement in radius)
l/ L A Y r 3Ka
Þ Elongation is the same.
Normal force N N
25. (d) Stress < < <
22. (a) If force F acts along the length L of the wire of cross- Area A (2 p a)b
section A, then energy stored in unit volume of wire
Stress = B×strain
is given by
N 2paΧa ≥ b
1 <B
Energy density = stress × strain (2pa)b pa 2 b
2
(2pa)2 Χab 2
1 F F æ F X ö ÞN<B
= ´ ´ çèQ stress = and strain = ÷ pa 2 b
2 A AY A AY ø
Force needed to push the cork.
1 F2 1 F 2 ´ 16 1 F 2 ´ 16 f < m N < m 4pbΧaB = (4pmBb)Da
= = =
2 A2Y 2 (pd 2 )2 Y 2 pd 4Y
26. (c) D
If u1 and u2 are the densities of two wires, then
4
u1 æ d 2 ö d1 d
= ( 4) Þ 1 = 2 :1
14 F
=ç ÷ Þ
u2 è d1 ø d2 d2

23. (a) When a catapult is stretched up to length l, then the


stored energy in it = Dk. E Þ h

1 æ YA ö 1 mv 2 L
. ç ÷ ( DI )2 = mv 2 Þy=
2 è Lø 2 D (Dl ) 2
Shearing strain is created along the side surface of the
m = 0.02 kg
punched disk. Note that the forces exerted on the disk are
v = 20 ms–1
exerted along the circumference of the disk, and the total
L = 0.42 m
force exerted on its center only.
A = (p d2)/(4)
Let us assume that the shearing stress along the side
d = 6 × 10–3 m
surface of the disk is uniform, then
Dl = 0.2 m
0.02 ´ 400 ´ 0.42 ´ 4 F= ò dFmax = ò s max dA = s max ò dA
y= = 2.3 × 106 N/m2 surface surface surface
p ´ 36 ´ 10 –6 ´ 0.04
æDö
So, order is 106. = ò s max .A = smax .2p ç ÷ h
è2ø
volumetric stress
24. (c) Bulk modulus, K = 8 æ1 -2 ö -2
volumetric strain = 3.5 ´10 ´ ç ´ 10 ÷ ´ 0.3 ´10 ´ 2p
è2 ø
mg = 3.297 ´ 104 ; 3.3 ´ 10 4 N
K=
æ dV ö
aç ÷ 1
èV ø 27. (a) Compressibility =
Bulk modulus
dV mg As bulk modulus is least for ethanol (0.9) and maximum for
Þ = ....(i)
V Ka mercury (25) among ehtanol, mercury and water. Hence
4 3 DV
volume of sphere, V = pR compression in volume
3 V
Ethanol > Water > Mercury
P-138 Physics

lateral strain ( b ) stress


28. (c) Poisson’s ratio, s = Þ strain =
longitudinal strain ( a ) Y
On substituting the expression of strain in equation (i) we
For material like copper, s = 0.33
get
And, Y = 3k (1 – 2 s)
1 stress 1 S 2
9 1 3 E= ´ stress ´ = ×
Also, = + 2 Y 2 Y
Y k h
30. (d) Let A and L be the area and length of the wire.
Y = 2h (1+ s)
Work done by constant force in displacing the wire by a
Hence, h < Y < k
distance l.
29. (a) Energy stored in the wire per unit volume,
= change in potential energy
1
E= ´ stress ´ strain ...(i) 1
2 = × stress × strain × volume
2
We know that,
1 F l Fl
stress = ´ ´ ´ A´ L =
Y= 2 A L 2
strain
9
Mechanical Properties of Fluids P-139

Mechanical Properties
of Fluids
6. M
Pressure, Density, Pascal's Law
TOPIC 1 and Archimedes' Principle
5m
1. A hollow spherical shell at outer radius R floats just
N
submerged under the water surface. The inner radius of
the shell is r. If the specific gravity of the shell material is
5m
27
w.r.t water, the value of r is : [5 Sep. 2020 (I)]
8
O
8 4 2 1 Two liquids of densities r1 and r2(r2 = 2r1) are filled up
(a) R (b) R (c) R (d) R
9 9 3 3 behind a square wall of side 10 m as shown in figure. Each
2. An air bubble of radius 1 cm in water has an upward liquid has a height of 5 m. The ratio of the forces due to these
acceleration 9.8 cm s–2. The density of water is 1 gm liquids exerted on upper part MN to that at the lower part NO
cm–3 and water offers negligible drag force on the bubble. The is (Assume that the liquids are not mixing): [8 Jan 2020 (II)]
mass of the bubble is (g = 980 cm/s2) [4 Sep. 2020 (I)] (a) 1/3 (b) 2/3 (c) 1/2 (d) 1/4
(a) 4.51 gm (b) 3.15 gm (c) 4.15 gm (d) 1.52 gm 7. A cubical block of side 0.5 m floats on water with 30% of
3. Two identical cylindrical vessels are kept on the ground its volume under water. What is the maximum weight that
and each contain the same liquid of density d. The area of can be put on the block without fully submerging it under
the base of both vessels is S but the height of liquid in one water? [Take, density of water = 103 kg/m3]
vessel is x1 and in the other, x2. When both cylinders are [10 April 2019 (II)]
connected through a pipe of negligible volume very close (a) 46.3 kg (b) 87.5 kg (c) 65.4 kg (d) 30.1 kg
to the bottom, the liquid flows from one vessel to the other 8. A submarine experiences a pressure of 5.05×106 Pa at depth
until it comes to equilibrium at a new height. The change in
of d1 in a sea. When it goes further to a depth of d2, it
energy of the system in the process is : [4 Sep. 2020 (II)]
experiences a pressure of 8.08×106 Pa. Then d1– d1 is
(a) gdS ( x22 + x12 ) (b) gdS ( x2 + x1 )2 approximately (density of water=103 kg/m3 and acceleration
due to gravity = 10 ms–2): [10 April 2019 (II)]
3 1 (a) 300 m (b) 400 m (c) 600 m (d) 500 m
(c) gdS ( x2 - x1 )2 (d) gdS ( x2 - x1 )2
4 4 4
4. A leak proof cylinder of length 1 m, made of a metal which 9. A wooden block floating in a bucket of water has of its
5
has very low coefficient of expansion is floating vertically volume submerged. When certain amount of an oil poured
in water at 0°C such that its height above the water surface into the bucket, it is found that the block is just under the
is 20 cm. When the temperature of water is increased to oil surface with half of its volume under water and half in
4°C, the height of the cylinder above the water surface oil. The density of oil relative to that of water is:
becomes 21 cm. The density of water at T = 4°C, relative to [9 April 2019 (II)]
the density at T = 0°C is close to: [8 Jan 2020 (I)] (a) 0.5 (b) 0.8 (c) 0.6 (c) 0.7
(a) 1.26 (b) 1.04 (c) 1.01 (d) 1.03 10. A load of mass M kg is suspended from a steel wire of
5. Consider a solid sphere of radius R and mass density length 2 m and radius 1.0 mm in Searle’s apparatus
æ r2 ö experiment. The increase in length produced in the wire is
r(r) = r0 ç 1 - 2 ÷ , 0 < r £ R. The minimum density of a 4.0 mm. Now the load is fully immersed in a liquid of relative
è R ø density 2. The relative density of the material of load is 8.
liquid in which it will float is: [8 Jan 2020 (I)] The new value of increase in length of the steel wire is :
r0 r0 2r0 2r0 [12 Jan. 2019 (II)]
(a) (b) (c) (d)
3 5 5 3 (a) 3.0 mm (b) 4.0 mm (c) 5.0 mm (d) Zero
P-140 Physics

11. A soap bubble, blown by a mechanical pump at the mouth 1 + sin a 1 + cos a
of a tube, increases in volume, with time, at a constant rate. (a) (b)
The graph that correctly depicts the time dependence of 1 - sin a 1 - cos a
pressure inside the bubble is given by: [12 Jan. 2019 (II)] 1 + tan a 1 + sin a
(c) (d)
1 - tan a 1 - cos a
P P 15. A uniform cylinder of length L and mass M having cross-
sectional area A is suspended, with its length vertical, from
(a) (b) a fixed point by a massless spring such that it is half
submerged in a liquid of density s at equilibrium position.
1 log(t) The extension x0 of the spring when it is in equilibrium
t is: [2013]

P P Mg Mg æ LAs ö
(a) (b) ç1 – ÷
(c) (d) k k è M ø

Mg æ LAs ö Mg æ LAs ö
13 (c) ç1 – ÷ (d) ç1 + ÷
t t k è 2M ø k è M ø
12. A liquid of density r is coming out of a hose pipe of radius
a with horizontal speed v and hits a mesh. 50% of the liquid 16. A ball is made of a material of density r where roil < r < rwater
passes through the mesh unaffected. 25% looses all of its with roil and rwater represe-nting the densities of oil and
momentum and 25% comes back with the same speed. The water, respectively. The oil and water are immiscible. If the
resultant pressure on the mesh will be: [11 Jan. 2019 (I)] above ball is in equilibrium in a mixture of this oil and water,
1 2 3 2 1 2 which of the following pictures represents its equilibrium
(a) ρv (b) ρv (c) ρv (d) ρv 2 position ? [2010]
4 4 2
13. A thin uniform tube is bent into a circle of radius r in the
vertical plane. Equal volumes of two immiscible liquids, whose
densities are r1 and r1 (r1 > r2) fill half the circle. The
angle q between the radius vector passing through the common water oil
interface and the vertical is [Online April 15, 2018]
-1 é p æ r - r ö ù (a) (b)
(a) q = tan ê ç 1 2 ÷ ú oil
ë 2 è r1 + r2 ø û water
-1 p æ r1 - r2 ö
(b) q = tan ç ÷
2 è r1 + r2 ø
ær ö
(c) q = tan -1 p ç 1 ÷
è r2 ø
(d) None of above water oil
14. There is a circular tube in a vertical plane. Two liquids
which do not mix and of densities d1 and d2 are filled in the
(c) (d)
tube. Each liquid subtends 90º angle at centre. Radius
oil
joining their interface makes an angle a with vertical. Ratio water
d1
is: [2014]
d2 17. Two identical charged spheres are suspended by strings
of equal lengths. The strings make an angle of 30°
with each other. When suspended in a liquid of density
0.8g cm–3, the angle remains the same. If density of the
material of the sphere is 1.6 g cm–3 , the dielectric constant
of the liquid is [2010]
a d2 (a) 4 (b) 3 (c) 2 (d) 1
18. A jar is filled with two non-mixing liquids 1 and 2 having
densities r1 and, r2 respectively. A solid ball, made of a
material of density r3 , is dropped in the jar. It comes to
d1 equilibrium in the position shown in the figure.Which of
the following is true for r1, r2and r3? [2008]
Mechanical Properties of Fluids P-141

24. Water flows into a large tank with flat bottom at the rate
r1
of 10–4 m3 s–(1) Water is also leaking out of a hole of area
1 cm2 at its bottom. If the height of the water in the tank
r3 remains steady, then this height is: [10 Jan. 2019 I]
(a) 5.1 cm (b) 7 cm (c) 4 cm (d) 9 cm
25. The top of a water tank is open to air and its water lavel
is maintained. It is giving out 0.74m3 water per minute
through a circular opening of 2 cm radius in its wall. The
(a) r3 < r1 < r2 (b) r1 > r3 > r2
depth of the centre of the opening from the level of water
(c) r1 < r2 < r3 (d) r1 < r3 < r2 in the tank is close to: [9 Jan. 2019 (II)]
(a) 6.0 m (b) 4.8 m (c) 9.6 m (d) 2.9 m
Fluid Flow, Reynold's Number 26. When an air bubble of radius r rises from the bottom to the
TOPIC 2 and Bernoulli's Principle
5r
surface of a lake, its radius becomes . Taking the
19. A fluid is flowing through a horizontal pipe of varying cross- 4
section, with speed v ms–1 at a point where the pressure is atmospheric pressure to be equal to 10m height of water
P column, the depth of the lake would approximately be
P Pascal. At another point where pressure is Pascal its (ignore the surface tension and the effect of temperature):
2 [Online April 15, 2018]
speed is V ms . If the density of the fluid is r kg m–3 and the
–1
(a) 10.5m (b) 8.7m (c) 11.2m (d) 9.5m
flow is streamline, then V is equal to : [6 Sep. 2020 (II)] 27. Two tubes of radii r1 and r2, and lengths l1 and l2, respec-
P 2P 2 tively, are connected in series and a liquid flows through
(a) +v (b) +v
r r each of them in streamline conditions. P1 and P2 are pressure

(c)
P
+ v2 (d)
P 2
+v differences across the two tubes. If P2 is 4P1 and l2 is l1 ,
2r r 4
20. Water flows in a horizontal tube (see figure). The pressure then the radius r2 will be equal to : [Online April 9, 2017]
of water changes by 700 Nm–2 between A and B where the r1
(a) r1 (b) 2r1 (c) 4r1 (d)
area of cross section are 40 cm2 and 20 cm2, respectively. 2
Find the rate of flow of water through the tube. 28. Consider a water jar of radius R that has water filled up to
(density of water = 1000 kgm–3) [9 Jan. 2020 (I)] height H and is kept on a stand of height h (see figure).
A Through a hole of radius r (r << R) at its bottom, the water
leaks out and the stream of water coming down towards
B the ground has a shape like a funnel as shown in the figure.
If the radius of the cross–section of water stream when it
hits the ground is x. Then : [Online April 9, 2016]
(a) 3020 cm3/s (b) 2720 cm3/s
R
(c) 2420 cm3/s (d) 1810 cm3/s
21. An ideal fluid flows (laminar flow) through a pipe of non- H
uniform diameter. The maximum and minimum diameters of
the pipes are 6.4 cm and 4.8 cm, respectively. The ratio of 2r
the minimum and the maximum velocities of fluid in this
pipe is: [7 Jan. 2020 (II)]
9 3 3 81 h
(a) (b) (c) (d) 256
16 2 4
22. Water from a tap emerges vertically downwards with an
initial speed of 1.0 ms–1. The cross-sectional area of the 2x
1
tap is 10–4 m 2. Assume that the pressure is constant æ H ö
æ H ö4
throughout the stream of water and that the flow is streamlined. (a) x = r ç (b) x = r çè ÷
The cross-sectional area of the stream, 0.15 m below the tap è H + h ÷ø H + hø
2 1
would be : [Take g = 10 ms–2) [10 April 2019 (II)] æ H ö æ H ö2
(c) x = r ç (d) x = r ç
(a) 2×10–5 m2 (b) 5×10–5 m2 è H + h ÷ø è H + h ÷ø
(c) 5×10–4 m2 (d) 1×10–5 m2
29. If it takes 5 minutes to fill a 15 litre bucket from a water tap
23. Water from a pipe is coming at a rate of 100 liters per
minute. If the radius of the pipe is 5 cm, the Reynolds 2
of diameter cm then the Reynolds number for the
number for the flow is of the order of : (density of water = p
1000 kg/m3, coefficient of viscosity of water = 1 mPa s) flow is (density of water = 103 kg/m3) and viscosity of
[8 April 2019 I] water = 10–3 Pa.s) close to : [Online April 10, 2015]
(a) 10 3 (b) 10 4 (c) 10 2 (d) 10 6 (a) 1100 (b) 11,000 (c) 550 (d) 5500
P-142 Physics

30. An open glass tube is immersed in mercury in such a way (a) Depends on H (b) 1 : 1
that a length of 8 cm extends above the mercury level. The (c) 2 : 2 (d) 1 : 2
open end of the tube is then closed and sealed and the tube 36. Water is flowing through a horizontal tube having cross-
is raised vertically up by additional 46 cm. What will be sectional areas of its two ends being A and A¢ such that the
length of the air column above mercury in the tube now? ratio A/A¢ is 5. If the pressure difference of water between
(Atmospheric pressure = 76 cm of Hg) [2014] the two ends is 3 × 105 N m–2, the velocity of water with
(a) 16 cm (b) 22 cm (c) 38 cm (d) 6 cm which it enters the tube will be (neglect gravity effects)
31. In the diagram shown, the difference in the two tubes of [Online May 12, 2012]
the manometer is 5 cm, the cross section of the tube at A (a) 5 m s–1 (b) 10 m s–1
and B is 6 mm2 and 10 mm2 respectively. The rate at which (c) 25 m s–1 (d) 50 10 m s–1
water flows through the tube is (g = 10 ms–2)
37. A square hole of side length l is made at a depth of h and
[Online April 19, 2014]
a circular hole of radius r is made at a depth of 4h from the
surface of water in a water tank kept on a horizontal surface.
If l << h, r << h and the rate of water flow from the holes is
5 cm the same, then r is equal to [May 7, 2012]

A B
h
4h
A v1
(a) 7.5 cc/s (b) 8.0 cc/s (c) 10.0 cc/s (d) 12.5 cc/s
32. A cylindrical vessel of cross-section A contains water to a
height h. There is a hole in the bottom of radius ‘a’. The B v2
time in which it will be emptied is: [Online April 12, 2014]
2A h 2A h
(a) 2 g (b)
pa pa 2 g
2 2A h A h l l l l
(c) (d) (a) (b) (c) (d)
pa 2 g 2pa 2 g 2p 3p 3p 2p
33. Water is flowing at a speed of 1.5 ms–1 through horizontal 38. Water is flowing continuously from a tap having an internal
tube of cross-sectional area 10–2 m2 and you are trying to diameter 8 × 10–3 m. The water velocity as it leaves the tap
stop the flow by your palm. Assuming that the water stops is 0.4 ms–1. The diameter of the water stream at a distance 2
immediately after hitting the palm, the minimum force × 10–1 m below the tap is close to: [2011]
that you must exert should be (a) 7.5 × 10–3 m (b) 9.6 × 10–3 m
(density of water = 103 kgm–3) [Online April 9, 2014] (c) 3.6 × 10–3 m (d) 5.0 × 10–3 m
(a) 15 N (b) 22.5 N (c) 33.7 N (d) 45 N 39. A cylinder of height 20 m is completely filled with water.
34. Air of density 1.2 kg m–3 is blowing across the horizontal The velocity of efflux of water (in ms-1) through a small hole
wings of an aeroplane in such a way that its speeds above on the side wall of the cylinder near its bottom is [2002]
and below the wings are 150 ms –1 and 100 ms –1 , (a) 10 (b) 20 (c) 25.5 (d) 5
respectively. The pressure difference between the upper
and lower sides of the wings, is : [Online April 22, 2013] TOPIC 3 Viscosity and Terminal Velocity
(a) 60 Nm–2 (b) 180 Nm–2
(c) 7500 Nm–2 (d) 12500 Nm–2 40. In an experiment to verify Stokes law, a small spherical ball of
35. In a cylindrical water tank, there are two small holes A and radius r and density r falls under gravity through a distance h
B on the wall at a depth of h1, from the surface of water and at in air before entering a tank of water. If the terminal velocity of
a height of h2 from the bottom of water tank. Surface of water the ball inside water is same as its velocity just before entering
is at heigh H from the bottom of water tank. Water coming out the water surface, then the value of h is proportional to :
from both holes strikes the ground at the same point S. Find (ignore viscosity of air) [5 Sep. 2020 (II)]
the ratio of h1 and h2 [Online May 26, 2012] (a) r 4 (b) r 3
(c) r (d) r2
h1
41. A solid sphere, of radius R acquires a terminal velocity
v1 when falling (due to gravity) through a viscous fluid
A having a coefficient of viscosity h. The sphere is broken
H into 27 identical solid spheres. If each of these spheres
acquires a terminal velocity, v2, when falling through the
B
same fluid, the ratio (v1/v2) equals: [12 April 2019 (II)]
h2 (a) 9 (b) 1/27 (c) 1/9 (d) 27

S
Mechanical Properties of Fluids P-143

42. Which of the following option correctly describes the 47. The terminal velocity of a small sphere of radius a in a
variation of the speed v and acceleration 'a' of a point mass viscous liquid is proportional to [Online May 26, 2012]
falling vertically in a viscous medium that applies a force (a) a 2 (b) a 3 (c) a (d) a–1
F = –kv, where 'k' is a constant, on the body? (Graphs are 48. If a ball of steel (density r = 7.8 g cm ) attains a terminal
–3

schematic and not drawn to scale) velocity of 10 cm s–1 when falling in water (Coefficient
[Online April 9, 2016] of viscosity hwater = 8.5 × 10–4 Pa.s), then, its terminal
velocity in glycerine (r = 1.2 g cm–3, h = 13.2 Pa.s) would
a
be, nearly [2011 RS]
(a) 6.25 × 10–4 cm s–1 (b) 6.45 × 10–4 cm s–1
(c) 1.5 × 10–5 cm s–1 (d) 1.6 × 10–5 cm s–1
(a) 49. A spherical solid ball of volume V is made of a material
v of density r1. It is falling through a liquid of density
t r1 (r2< r1). Assume that the liquid applies a viscous force
v on the ball that is proportional to the square of its speed
v, i.e., Fviscous = –kv2 (k > 0). The terminal speed of the ball
is [2008]
(b)
Vg (r1 – r2 ) Vg r1
a (a) (b)
t k k
v Vgr1 Vg (r1 – r2 )
(c) (d)
k k
50. If the terminal speed of a sphere of gold (density
(c) a = 19.5 kg/m3) is 0.2 m/s in a viscous liquid (density = 1.5
kg/m 3), find the terminal speed of a sphere of silver
t (density = 10.5 kg/m3) of the same size in the same liquid
v [2006]
(a) 0.4 m/s (b) 0.133 m/s
(c) 0.1 m/s (d) 0.2 m/s
(d)
a 51. Spherical balls of radius ‘R’ are falling in a viscous fluid
t of viscosity ‘h’ with a velocity ‘v’. The retarding viscous
43. The velocity of water in a river is 18 km/hr near the surface. force acting on the spherical ball is [2004]
If the river is 5 m deep, find the shearing stress between the (a) inversely proportional to both radius ‘R’ and velocity ‘v’
horizontal layers of water. The co-efficient of viscosity of (b) directly proportional to both radius ‘R’ and velocity ‘v’
water = 10–2 poise. [Online April 19, 2014]
(c) directly proportional to ‘R’ but inversely proportional
(a) 10–1 N/m2 (b) 10–2 N/m2
(c) 10–3 N/m2 (d) 10–4 N/m2 to ‘v’
44. The average mass of rain drops is 3.0 × 10–5 kg and their (d) inversely proportional to ‘R’ but directly proportional
avarage terminal velocity is 9 m/s. Calculate the energy to velocity ‘v’
transferred by rain to each square metre of the surface at a
place which receives 100 cm of rain in a year. Surface Tension, Surface
TOPIC 4
[Online April 11, 2014] Energy and Capillarity
(a) 3.5 × 105 J (b) 4.05 × 104 J
(c) 3.0 × 105 J (d) 9.0 × 104 J 52. When a long glass capillary tube of radius 0.015 cm is
45. A tank with a small hole at the bottom has been filled with dipped in a liquid, the liquid rises to a height of 15 cm
water and kerosene (specific gravity 0.8). The height of within it. If the contact angle between the liquid and glass
water is 3m and that of kerosene 2m. When the hole is to close to 0°, the surface tension of the liquid, in
opened the velocity of fluid coming out from it is nearly: milliNewton m–1, is [r(liquid) = 900 kgm–3, g = 10 ms–2]
(take g = 10 ms–2 and density of water = 103 kg m–3) (Give answer in closest integer) __________.
[Online April 11, 2014] [NA 3 Sep. 2020 (I)]
(a) 10.7 ms–1 (b) 9.6 ms–1 53. Pressure inside two soap bubbles are 1.01 and 1.02
(c) 8.5 ms–1 (d) 7.6 ms–1 atmosphere, respectively. The ratio of their volumes is :
46. In an experiment, a small steel ball falls through a liquid at [3 Sep. 2020 (I)]
a constant speed of 10 cm/s. If the steel ball is pulled upward (a) 4 : 1 (b) 0.8 : 1 (c) 8 : 1 (d) 2 : 1
with a force equal to twice its effective weight, how fast 54. A capillary tube made of glass of radius 0.15 mm is dipped
will it move upward ? [Online April 25, 2013] vertically in a beaker filled with methylene iodide (surface
(a) 5 cm/s (b) Zero (c) 10 cm/s (d) 20 cm/s tension = 0.05 Nm–1, density = 667 kg m–3) which rises
P-144 Physics

to height h in the tube. It is observed that the two tangents


drawn from liquid-glass interfaces (from opp. sides of
the capillary) make an angle of 60° with one another. Then
h is close to (g = 10 ms–2). [2 Sep. 2020 (II)]
(a) 0.049 m (b) 0.087 m
(c) 0.137 m (d) 0.172 m
55. A small spherical droplet of density d is floating exactly R
half immersed in a liquid of density r and surface tension
T. The radius of the droplet is (take note that the surface
2r
tension applies an upward force on the droplet):
2 2 rwg 2 rw g
[9 Jan. 2020 (II)] (a) R (b) R
3T 6T
2T T
(a) r = (b) r = 2 rw g 2 3r w g
3(d + r) g (d - r) g (c) R (d) R
T T
T 3T 60. A large number of liquid drops each of radius r coalesce
(c) r = (d) r = to from a single drop of radius R. The energy released in
(d + r) g (2d - r) g
the process is converted into kinetic energy of the big
56. The ratio of surface tensions of mercury and water is drop so formed. The speed of the big drop is (given,
given to be 7.5 while the ratio of their densities is 13.6. surface tension of liquid T, density r)
Their contact angles, with glass, are close to 135o and 0o, [Online April 19, 2014, 2012]
respectively. It is observed that mercury gets depressed T æ1 1 ö 2T æ 1 1 ö
by an amount h in a capillary tube of radius r1, while water - -
r çè r R ÷ø r çè r R ÷ø
(a) (b)
rises by the same amount h in a capillary tube of radius
r2. The ratio, (r1/r2), is then close to : 4T æ 1 1 ö 6T æ 1 1 ö
ç - ÷ -
r çè r R ÷ø
[10 April 2019 (I)] (c) (d)
r èr Rø
(a) 4/5 (b) 2/5 (c) 3/5 (d) 2/3
61. Two soap bubbles coalesce to form a single bubble. If V
57. If ‘M’ is the mass of water that rises in a capillary tube of
is the subsequent change in volume of contained air and S
radius ‘r’, then mass of water which will rise in a capillary
change in total surface area, T is the surface tension and P
tube of radius ‘2r’ is : [9 April 2019 I]
atmospheric pressure, then which of the following relation
M is correct? [Online April 12, 2014]
(a) M (b) (c) 4 M (d) 2M
2 (a) 4PV + 3ST = 0 (b) 3PV + 4ST = 0
58. If two glass plates have water between them and are (c) 2PV + 3ST = 0 (d) 3PV + 2ST = 0
separated by very small distance (see figure), it is very 62. An air bubble of radius 0.1 cm is in a liquid having surface
difficult to pull them apart. It is because the water in tension 0.06 N/m and density 103 kg/m3. The pressure inside
between forms cylindrical surface on the side that gives the bubble is 1100 Nm–2 greater than the atmospheric
rise to lower pressure in the water in comparison to pressure. At what depth is the bubble below the surface of
atmosphere. If the radius of the cylindrical surface is R the liquid? (g = 9.8 ms–2) [Online April 11, 2014]
and surface tension of water is T then the pressure in water (a) 0.1 m (b) 0.15 m (c) 0.20 m (d) 0.25 m
between the plates is lower by : [Online April 10, 2015] 63. A capillary tube is immersed vertically in water and the height
of the water column is x. When this arrangement is taken into
a mine of depth d, the height of the water column is y. If R is
Cylindrical surface
of water x
the radius of earth, the ratio y is: [Online April 9, 2014]
2T 4T T T
(a) (b) (c) (d)
R R 4R R æ dö æ 2d ö
(a) ç 1 - ÷ (b) ç 1 - ÷
59. On heating water, bubbles being formed at the bottom of è R ø è Rø
the vessel detach and rise. Take the bubbles to be spheres æ R -d ö æR+dö
of radius R and making a circular contact of radius r with (c) ç ÷ (d) ç ÷
èR+dø è R -d ø
the bottom of the vessel. If r << R and the surface tension
64. Wax is coated on the inner wall of a capillary tube and the
of water is T, value of r just before bubbles detach is:
tube is then dipped in water. Then, compared to the unwaxed
(density of water is rw) [2014]
capillary, the angle of contact q and the height h upto which
water rises change. These changes are :
[Online April 23, 2013]
Mechanical Properties of Fluids P-145

(a) q increases and h also increases 69. Two mercury drops (each of radius ‘r’) merge to form
(b) q decreases and h also decreases bigger drop. The surface energy of the bigger drop, if T is
(c) q increases and h decreases the surface tension, is : [2011 RS]
(d) q decreases and h increases
(a) 4pr 2T (b) 2pr 2T
65. A thin tube sealed at both ends is 100 cm long. It lies
horizontally, the middle 20 cm containing mercury and two (c) 28 / 3 pr 2T (d) 25/ 3 pr 2T
equal ends containing air at standard atmospheric pressure. If
the tube is now turned to a vertical position, by what amount 70. A capillary tube (A) is dipped in water. Another identical
will the mercury be displaced ? [Online April 23, 2013] tube (B) is dipped in a soap-water solution. Which of the
following shows the relative nature of the liquid columns
in the two tubes? [2008]

A B
l0 l0
20 cm
100 cm
(Given : cross-section of the tube can be assumed to be (a)
uniform)
(a) 2.95 cm (b) 5.18 cm (c) 8.65 cm (d) 0.0 cm
66. This question has Statement-1 and Statement-2. Of the four A B
choices given after the Statements, choose the one that
best describes the two Statetnents.
Statement-1: A capillary is dipped in a liquid and liquid
rises to a height h in it. As the temperature of the liquid is (b)
raised, the height h increases (if the density of the liquid
and the angle of contact remain the same).
Statement-2: Surface tension of a liquid decreases with
A B
the rise in its temperature. [Online April 9, 2013]
(a) Statement-1 is true, Statement-2 is true; Statement-2 is
not the correct explanation for Statement-1.
(c)
(b) Statement-1 is false, Statement-2 is true.
(c) Statement-1 is true, Statement-2 is false.
(d) Statement-1 is true, Statement-2 is true; Statement-2 is
the correct explanation for Statement-1. A B
67. A thin liquid film formed between a U-shaped wire and a
light slider supports a weight of 1.5 × 10–2 N (see figure).
The length of the slider is 30 cm and its weight is negligible. (d)
The surface tension of the liquid film is [2012]

71. A 20 cm long capillary tube is dipped in water. The water


FILM rises up to 8 cm. If the entire arrangement is put in a freely
falling elevator the length of water column in the capillary
z tube will be [2005]
(a) 10 cm (b) 8 cm (c) 20 cm (d) 4 cm
72. If two soap bubbles of different radii are connected by a
W tube [2004]
(a) 0.0125 Nm–1 (b) 0.1 Nm–1 (a) air flows from the smaller bubble to the bigger
(c) 0.05 Nm–1 (d) 0.025 Nm–1 (b) air flows from bigger bubble to the smaller bubble till
68. Work done in increasing the size of a soap bubble from a the sizes are interchanged
radius of 3 cm to 5 cm is nearly (Surface tension of soap (c) air flows from the bigger bubble to the smaller bubble
solution = 0.03 Nm–1) [2011] till the sizes become equal
(a) 0.2 p mJ (b) 2p mJ (c) 0.4p mJ (d) 4p mJ (d) there is no flow of air.
P-146 Physics

1. (a) In equilibrium, mg = Fe By volume conservation,


FB = V r0 g and mass = volume × density Sx1 + Sx2 = S (2 x f )
4 4
p( R 3 - r 3 )r0 g = pR3rw g x1 + x2
3 3 xf =
2
r0 27 When valve is opened loss in potentail energy occur till
Given, relative density, =
rw 8 water level become same.
é æ r ö 3 ù 27 DU = U i - U f
Þ ê1 - ç ÷ ú rw = rw
è Rø ú 8
ëê û éæ x 2 x 2 ö ù
9 r3 1 r3 2 r3 DU = rSg êç 1 + 2 ÷ - x 2f ú
Þ 1- 3 = Þ 1- = 3 Þ = 3 ëêè 2 2 ø ûú
R 27 3 R 3 R
r æ 2ö
1/3
r3 8 é x2 x2 æ x + x ö 2 ù
Þ =ç ÷ Þ1- = = rSg ê 1 + 2 - ç 1 2 ÷ ú
R è 3ø R3 27 êë 2 2 è 2 ø úû
r3 8 19
Þ = 1- = rSg é x12 x22 ù
R 3 27 27 = ê + - x1 x2 ú = rSg ( x1 - x2 )2
8
2 êë 2 2 úû 4
\ r = 0.89 R = R.
9 4. (c) When cylinder is floating in water at 0°C
2. (c) Given : Net thrust = A(h2 – h1 )r0°c g
Radius of air bubble = 1 cm,
= A(100 – 80)r0°c g
Upward acceleration of bubble, a = 9.8 cm/s2,
rwater = 1 g cm–3 20 cm
Fbuoyant
4p 3 4p 100 cm
Volume V = r = ´ (1)3 = 4.19 cm3 0°C
3 3 a
When cylinder is floating in water at 4° C
Fbuoyant mg Net thrust = A(h2 – h1 )r4°c g
Fbuoyant - mg = ma Þ m =
g +a = A(100 – 21)r4°c g
(V rw g ) V rw (4.19) ´ 1 4.19 21 cm
\m = = = = = 4.15g
g+a a 9.8 1.01
1+ 1+
g 980 4°C

3. (d)
r4°c 80
\ = = 1.01
r0°c 79
xf xf
x1 5. (c) For minimum density of liquid, solid sphere has to
x2 float (completely immersed) in the liquid.
mg = FB (also Vimmersed = Vtotal)
4 3
or ò rdV = pR r
Initial potential energy, 3 l

x1 x é æ r2 ö ù
U1 = (rSx1 ) g × + (rSx2 ) g × 2 ê r ( r ) = r 0ç 1 – 0 < r £ R given ú
2 2 2 ÷
ëê è R ø úû
Final potential energy,
xf
R æ r2 ö 4
U f = (rSx f ) g × ´2 Þ ò r0 4 p ç 1 – 2 ÷ . r 2 dr = pR 3r l
2 0 è R ø 3
Mechanical Properties of Fluids P-147

R T' fB
é r3 r5 ù 4
Þ 4pr0 ê – 2 ú = pR3rl T
ëê 3 5R ûú 0 3

4pr0 R3 2 4 3
´ = pR rl Mg
3 5 3 Mg
2r0 Þ Dl µ F ...(i)
\ rl = T = Mg
5
M
M T = Mg – fB = Mg – r ×rl × g
6. (d) P1 = 0 b
r F1 æ r ö
N l æ 2ö
P2=5rg = ç1 – r ÷ Mg = ç1 – ÷ Mg
è bø è 8 ø
2r F2
O P2 = 15 rg 3
T= Mg
Let P1, P2 and P3 be the pressure at points M, N and O 4
respectively. From eqn (i)
Pressure is given by P = rgh Dl¢ T¢ 3
Now, P1 = 0 (Q h = 0) = = [Given: Dl = 4 mm]
Dl T 4
P2 = rg(5)
3 3
P3 = rg(15) \ Dl¢ = × Dl = ´ 4 = 3 mm
= 15 rg 4 4
11. (d)
(P + P )
Force on upper part, F1 = 1 2 A 4
V = ct or, pr 3 = ct
2 3
( P2 + P3 ) 1
Force on lower part, F2 = A Þ r = kt 3
2
4T
F1 5rg 5 1 P = P0 + k t1/3
\ = = =
F2 20rg 20 4
7. (b) When a body floats then the weight of the body = upthrust æ 1 ö
P = P0 + c ç 1/3 ÷
30 èt ø
\ (50)3 ´ ´ (1) ´ g = M cube g ...(i)
100 12. (b) Mass per unit time of the liquid = rav
Let m mass should be placed, then Momentum per second carried by liquid
(50)3 × (1) × g = (Mcube + m)g ...(ii) = rav × v
Subtracting equation (i) from equation (ii), we get Net force due to bounced back liquid,
Þ mg = (50)3 × g (1 – 0.3) = 125 × 0.7 × 103 g
Þ m = 87.5 kg é1 ù
F1 = 2 ´ ê rav 2 ú
8. (a) P1 = P0 + rgd1 ë4 û
P2 = P0 + rgd2 1
Net force due to stopped liquid, F2 = rav
2
DP = P2 – P1 = rgDd
3.03 × 106 = 103 × 10 × Dd 4
Þ Dd ; 300 m Total force,
1 1 3
æ 4V ö F = F1 + F2 = rav + rav = rav
2 2 2
9. (c) Mg = ç ÷ rw g 2 4 4
è 5 ø
3 2
æ M ö 4 rw 4rw Net pressure = rv
or çè ÷ø = or r = 4
V 5 5 13. (d) Pressure at interface A must be same from both the
When block floats fully in water and oil, then sides to be in equilibrium.
Mg = Fb1 + Fb2
æV ö V
( rV ) g = ç ÷ roil g + rw g R d2 Rsinq
è 2ø 2 q
3 R
or roil = rw = 0.6rw q Rcosq
5 Rsina – Rsin q
A
F Dl d1
10. (a) Using = Y×
A l
\ (R cos q + R sin q)r2 g = (R cos q - R sin q)r1g
P-148 Physics

d1 cos q + sin q 1 + tan q Fe ' = T 'sin 30° ...(A)


Þ d = cos q - sin q = 1 - tan q
2 mg = FB + T 'cos 30°
Þ r1 – r1 tan q = r2 + r2 tan q But FB = Buoyant force
Þ (r1 + r2) tan q = r1 – r2
ær –r ö 30°
\ q = tan –1 ç 1 2 ÷ FB T¢
è r1 + r2 ø
T¢ cos 30°
14. (c) Pressure at interface A must be same from both the
sides to be in equilibrium. F¢ e
T¢ sin 30°

R d2 Rsina
mg
a

a Rcosa R = V (d - r) g = V (1.6 - 0.8) g = 0.8 Vg


Rsina – Rsin a
A m 0.8 mg mg
d1
= 0.8 g= =
\ ( R cos a + R sin a )d2 g = ( R cos a - R sin a )d1 g d 1.6 2
mg
d1 cos a + sin a 1 + tan a \ mg = + T ' cos 30°
Þ d = cos a - sin a = 1 - tan a 2
2 mg
Þ = T 'cos 30° ...(B)
15. (c) From figure, kx 0 + FB = Mg 2
2 Fe'
From (A) and (B), tan 30° =
KX0 mg
From (1) and (2)
FB
Fe 2 Fe'
= (2)
Mg mg mg

L Þ Fe = 2Fe'
kx 0 + s Ag = Mg If K be the dielectric constant, then
2
[Q mass = density × volume] Fe
Fe' =
L K
Þ kx 0 = Mg - s Ag
2 2 Fe
\ Fe = ÞK=2
sLAg K
Mg - 18. (d) As liquid 1 floats over liquid 2. The lighter liquid
Þ x0 = 2 = Mg æ1 - LAs ö
ç ÷ floats over heavier liquid. So, r1 < r2
k k è 2M ø
Hence, extension of the spring when it is in equilibrium is, Also r3 < r2 because the ball of density r3 does not sink
to the bottom of the jar.
Mg æ LAs ö
x0 = ç1 - ÷ Also r3 > r1 otherwise the ball would have floated in
k è 2M ø liquid 1. we conclude that
16. (b) Oil will float on water so, (b) or (d) is the correct option. r1 < r3 < r2.
But density of ball is more than that of oil,, hence it will
19. (d) Using Bernoulli's equation
sink in oil.
1 1
17. (c) P1 + ru12 + rgh1 = P2 + ru 22 + rgh2
T 30° 2 2
T cos 30° For horizontal pipe, h1 = 0 and h2 = 0 and taking
P
Fe P1 = P, P2 = , we get
T sin 30° 2
mg 1 2 P 1
Þ P + ru = + rV 2
2 2 2
Fe = T sin 30°
P 1 2 1
mg = T cos30° Þ + ru = rV 2
2 2 2
Fe
Þ tan 30° = ...(1)
Þ V = u2 +
P
mg
r
In liquid ,
Mechanical Properties of Fluids P-149

20. (b) According to question, area of cross-section at A,


aA = 40 cm2 and at B, aB = 20 cm2 24. (a)
Let velocity of liquid flow at A, = VA and at B, = VB
Using equation of continuity aAVA = aBVB
40VA = 20VB Q in
Þ 2VA = VB
Now, using Bernoulli’s equation
1
2
1
2
1
PA + rVA2 = PB + rVB2 Þ PA - PB = r VB2 - VA2
2
( )
h
1 æ V 2ö
3V 2
Þ DP = 1000 ç VB2 - B ÷ Þ DP = 500 ´ B
2 ç 4 ÷ø 4
è
( DP ) ´ 4 ( 700) ´ 4 Qout
Þ VB = = m/s = 1.37 ´ 102 cm/s
1500 1500
Volume flow rate Q = aB × vB Since height of water column is constant therefore, water
= 20 × 100 × VB = 2732 cm3/s » 2720 cm3/s inflow rate (Qin)
21. (a) From the equation of continuity = water outflow rate
A1v1= A2v2 Qin = 10–4 m3s–1
Here, v1 and v2 are the velocities at two ends of pipe. Qout = Au = 10–4 × 2gh
A1 and A2 are the area of pipe at two ends
\ 10 = 10 ×
–4 –4
20 ´ h
v1 A2 p(4.8) 2 9
Þ = = = 1
v2 A1 p(6.4) 2 16 \h = m = 5 cm
20
22. (b) Using Bernoullie’s equation
1 25. (b) Here, volume tric flow rate
P + (v12 - v22 ) + rgh = P
0.74
2 = = p r 2 v = ( p´ 4 ´10 –4 ) ´ 2gh
Þ v22 = v12 + 2 gh 60
74 ´100 74 0
Þ 2gh = Þ 2gh =
Þ v2 = v12 + 2 gh 240 p 24p
Equation of continuity
740 ´ 740
A1v1 = A2v2 Þ 2gh =
24 ´ 24 ´10
(Qp2 =10)
æ 2 15 ö
(1 cm2) (1 m/s) = (A2) ç (1) + 2 ´ 10 ´ ÷ 74 ´ 74
è 100 ø Þh = » 4.8m
2 ´ 24 ´ 24
10–4 × 1 = A2 × 2
i.e., The depth of the centre of the opening from the
10-4 level of water in the tank is close to 4.8 m
\ A2 = = 5 × 10–5 m2 26. (d) Using P1V1 = P2V2
2
23. (b) Rate of flow of water (V) = 100 lit/min 4 4 125r 3
-3 ( P1 ) pr 3 = ( P2 ) p
100 ´ 10 5 3 3 64
= ´ ´ 10 -3 m3
60 3 rg (10) + rgh 125
=
V 5 ´ 10 -3 rg (10) 64
\ Velocity of flow of water (v) = = 640 + 64 h = 1250
A 3 ´ ´(5 ´ 10-2 )2
On solving we get h = 9.5 m
10 2 27. (d) The volume of liquid flowing through both the tubes
= = m/s i.e., rate of flow of liquid is same.
15p 3p
= 0.2 m/s Therefore, V = V1= V2
Dvr pP1r14 pP2 r24
\ Reynold number (NR) = i.e., =
h 8hl1 8hl2
2 P1r14 P2r24
(10 ´ 10-2 ) ´ ´ 1000 =
= 3p = 2 ´ 104
or
l1 l2
1
Order of NR = 104 Q P2 = 4 P1 and l2 = l1/4
P-150 Physics

P1r14 4P1r24 r4 \ v 22 - v12 = 2gh ...(1)


= Þ r24 = 1
l1 l1 4 16 According to the equation of continuty
r2 = r1 2 A1v1 = A 2 v2 ...(2)
28. (a) According to Bernoulli's Principle,
A1 6 mm2
1 2 1 =
r v1 ∗ r gh < r v22 A 2 10 mm 2
2 2
v12 ∗ 2gh < v22 A1 v 2 6
= =
From equation (2),
A 2 v1 10
2gH ∗ 2gh < v22 ...(i)
a1v1 = a2v2 6
or, v 2 = v1
10
pr 2 2gh < p x 2 v 2
Putting this value of v2 in equation (1)
r2 2
2gh < v 2 æ6 ö
ç v1 ÷ - ( v1 ) = 2 ´10 ´ 5
2 3
2
x è 10 ø
Substituting the value of v2 in equation (i)
1 éQ g = 10m / s 2 = 103 cm / s 2 ù
r 4
é ù4 ê ú
2gH ∗ 2gh < 2gh or, x < r ê H ú ëêand h = 5 cm ûú
x4 êë H ∗ h úû
10
2 Solving we get v1 =
29. (d) Given: Diameter of water tap = cm 8
p Therefore the rate at which water flows through the
1 6 ´ 10
\ Radius, r = ´10-2 m tube = A1v1 = A 2 v 2 = = 7.5cc / s
p 8
dm dh
= rAV
V 32. (b) Let the rate of falling water level be -
dt dt
Initially at t = 0 ; h = h
2
15 æ 1 ö -4 t=t;h=0 A
= 10 3 ´ p ç ÷ ´ 10 V
5 ´ 60 è pø æ dh ö
Þ V = 0.05 m/s Then, A ç - ÷ = pa 2 .v
è dt ø
r Vr
Reynold’s number, Re = A
n dt = - 2
dh h
pa 2 gh
2 a
103 ´ 0.5 ´ 10-2 [Q velocity of efflux of
p
= @ 5500 liquid v = 2 gh ]
10-3
Integrating both sides
30. (a) (54–x) t
A 0
-1 2
54 cm P
ò dt = - òh dh
8 cm x 0 2 g pa 2 h
0
A é h1 2 ù
[ t ]t0 = - . ê ú
Hg 2 g pa 2 ëê 1 2 ûú h

2A h
Length of the air column above mercury in the tube is, t=
P + x = P0 pa 2 g
Þ P = (76 – x) 33. (a) For 1 m length of horizontal tube
Þ 8 × A × 76 = (76 – x) × A × (54 – x) Mass of water M = density × volume
\ x = 38 = 103 × area × length
Thus, length of air column = 103 × 10–2 × 1 = 10 kg
= 54 – 38 = 16 cm. Dp
31. (a) According to Bernoulli’s theorem, Therefore minimum force = (rate of change of
Dt
momentum)
1 1
P1 + rv12 = P2 + rv 22 = 10 × 1.5 = 15 N
2 2
Mechanical Properties of Fluids P-151

34. (c) Pressure difference 37. (a) As A1v1 = A2v2 (Principle of continuity)
1
P2 - P1 = r v22 - v12
2
( 1
)
= ´1.2 (150)2 - (100) 2
2
( ) or, l 2 2 gh = pr 2 2 g ´ 4h

1 (Efflux velocity = 2gh )


= ´ 1.2(22500 - 10000)
2
= 7500 Nm–2 l2 l2 l
\ r2 = or r = =
2p 2p 2p
35. (b)
38. (c) Using Bernoulli's theorem, for horizontal flow
h1
1 1
A V1 P0 + rv12 + rgh = P0 + rv22 + 0
2 2
H
B V2 v2 = v12 + 2 gh = 0.16 + 2 ´ 10 ´ 0.2 =2.03m/s
h2 According to equation of continuity
G A2v2 = A1v1
D22 D2
i.e. R1 = R2 = R p ´ v2 = p 1 v1
or, v1 t1 = v2 t2 ... (i) 4 4

Where v1 = velocity of efflux at A = (2 gh1 ) and v1


Þ D2 = D1 = 3.55 × 10–3 m
v2
v2 = velocity of efflux at B = (2 g (H - h2 ) 39. (b) Given, Height of cylinder, h=20 cm Acceleration due
t1 = time of fall water stream through A to gravity, g=10 ms–2
Velocity of efflux
2(H - h1
= ( ) v = 2gh
g
Where h is the height of the free surface of liquid from th e
2h2 hole
t2 = time of fall of the water stream through B = Þ v = 2 ´10 ´ 20 = 20 m / s
g
Putting these values is eqn (i) we get 40. (a) Using, v2 – u2 = 2gh
(H – h1)h1 = (H – h2)h2 Þ v 2 - 02 = 2 gh Þ v = 2 gh
or [H – (h1 + h2)][h1 – h2] = 0 Terminal velocity,
Here, H = h1 + h2 is irrelevant because the holes are at
2 r 2 (r - s ) g
h1 VT =
9 h
two different heights. Hence h1 = h2 or, h = 1
2 After falling through h the velocity should be equal to
36. (a) According to Bernoulli’s theorem terminal velocity
1 1
P1 + rv12 = P2 + rv22 ...(i) 2 r 2 (r - s ) g
2 2 \ 2 gh =
From question, 9 h
A 4 r 4 g 2 (r - s)2
P1 - P2 = 3 ´ 105 , 1 = 5 Þ 2 gh =
A2 81 h2
According to equation of continuity
A1 v1 = A2 v2 2r 4 g (r - s)2
Þh= Þ h a r4
A1 v2 81h2
or, A = v = 5
2 1 4 3 4 3
Þ v2 = 5v1 41. (a) 27 × pr = pr
3 3
From equation (i)
R
1
P1 - P2 = r v22 - v12
2
( ) or r =
3
.
Terminal velocity, v µ r2
1
2
2 2
or 3 × 105 = ´ 1000 5v1 - v1 ( ) v1 r12
\ v = 2
Þ 600 = 6v1 ´ 4v1 2 r2
Þ v12 = 25 æ r2 ö
2
æ R / 3ö
2
1
\ v1 = 5 m/s or v2 = ç ÷ v1 = çè ÷ø v1 =
è r1 ø R 9
P-152 Physics

v1 W–T –F=0
or v2 = 9. 2 r 2 (r - s)g
And terminal velocity v =
42. (c) When a point mass is falling vertically in a viscous 9 h
medium, the medium or viscous fluid exerts drag force on As in case of upward motion upward force is twice its
the body to oppose its motion and at one stage body effective weight, therefore, it will move with same speed
falling with constant terminal velocity. 10 cm/s
43. (b) h = 10–2 poise 47. (a) Terminal velocity in a viscous medium is given by:
18000 2 a 2 (r - s ) g
v = 18 km/h = = 5 m/s VT =
3600 9h
l =5m \ VT µ a 2
v 48. (a) When the ball attains terminal velocity
Strain rate =
l Weight of the ball = viscous force + buoyant force
shearing stress \V rg = 6phrv + V rl g
Coefficient of viscosity, h =
strain rate Þ Vg ( r - rl ) = 6phrv
\ Shearing stress = h × strain rate
-2 5
'
( )
Also Vg r- ri = 6ph ' rv '
= 10 ´ = 10–2 Nm–2
5 '
44. (b) Total volume of rain drops, received 100 cm in a year \ v ' h ' = (r - r l ) ´ v h
(r - r l )
by area 1 m2
100 (r - rl' ) vh
2
= 1m ´ m = 1 m3 Þ v' = ´
100 (r - rl ) h '
As we know, density of water, (7.8 - 1.2) 10 ´ 8.5 ´ 10-4
d = 103 kg/m3 = ´
Therefore, mass of this volume of water (7.8 - 1) 13.2
M = d × v = 103 × 1 = 103 kg \ v ' = 6.25 ´ 10-4 cm/s
Average terminal velocity of rain drop 49. (a) When the ball attains terminal velocity
v = 9 m/s (given) Weight of the ball = Buoyant force + Viscous force
Therefore, energy transferred by rain, B=Vr2 g
Fv
1
E= mv2
2
1
= × 103 × (9)2
2
1
= × 103 × 81 = 4.05 × 104 J W=V r1g
2
45. (b) According to Toricelli’s theorem, \ V r1 g = V r2 g + kvt2 Þ Vg ( r1 – r2 ) g = kvt2
Velocity of efflex,
Vg (r1 - r 2 )
Veff = 2 gh = 2 ´ 9.8 ´ 5 @ 9.8 ms -1 Þ vt =
k
46. (c) T (upthrust) 50. (c) Given,
F Density of gold, rG = 19.5 kg/m3
(viscous r Density of silver, r5 = 10.5kg/m3
force) Density of liquid, s = 1.5kg/m3
2 r 2 (r - s ) g
W(weight) Terminal velocity, vT =
9h
Weight of the body vT (10.5 - 1.5) 9
\ 2
= Þ vT = 0.2 ´
4 3 0.2 (19.5 - 1.5) 2 18
W = mg = pr rg
3 \ vT = 0.1 m/s
2
4 3 51. (b) From Stoke's law, force of viscosity acting on a
T = pr sg
3 spherical body is
and F = 6phvr
F = 6phrv
When the body attains terminal velocity net force acting
on the body is zero. i.e., hence F is directly proportional to radius & velocity.
Mechanical Properties of Fluids P-153

52. (101) 56. (b) As we know that


Given : Radius of capillary tube, 2T cos q
r = 0.015 cm = 15 × 10–5 mm =Rh
rrg
h = 15 cm = 15 × 10–2 mm
THg
2T cos q = 7.5
Using, h = [cos q = cos0° = 1] TWater
rgr
Surface tension, rHg cos qHg cos135° 1
= 13.6 & = =
rhrg 15 ´ 10-5 ´ 15 ´ 10-2 ´ 900 ´ 10 rW cos qW cos 0° 2
T= = = 101 m i l l i
2 2 R Hg æ THg ö æ rW ö æ cos qHg ö
newton m–1 =ç ÷ç ÷ç ÷
53. (c) According to question, pressure inside, 1st soap R Water è TW ø è rHg ø è cos qW ø
bubble,
1 1 2
4T = 7.5 ´ ´ = 0.4 =
DP1 = P1 - P0 = 0.01 = ...(i) 13.6 2 5
R1
2T cos q
4T 57. (d) We have, h =
And DP2 = P2 - P0 = 0.02 = ...(ii) r rg
R2 Mass of the water in the capillary
Dividing, equation (ii) by (i),
2 2T cos q
1 R2 m = rV = r × pr2h = r ´ pr ´
= Þ R1 = 2R2 rrg
2 R1 Þm ar
4 m r
Volume V = pR 3 \ 1 =
3 m2 2r
V1 R13 8R23 8 or m2 = 2m1 = 2m.
\ = 3 = 3 =
V2 R2 R2 1 T T
54. (b) Given, 58. (d) Here excess pressure, Pexcess = r + r
1 2
Angle of contact q = 30°
Surface tension, T = 0.05 Nm–1 T ær = R ö
Pexcess = Qç1 ÷
Radius of capillary tube, r = 0.15 mm = 0.15 × 10–3m R è r2 = O ø
Density of methylene iodide, r = 667 kg m–3 59. (a) When the bubble gets detached,
Buoyant force = force due to surface tension

R
30° q
60°
rq
T×dl
Force due to excess pressure = upthrust
2T cos q
Capillary rise, h = 2T
rgr Access pressure in air bubble =
R
3
2 ´ 0.05 ´ 2T 4p R 3
2 (p r 2 ) = rw g
= = 0.087 m R 3T
667 ´ 10 ´ 0.15 ´ 10-3
55. (d) For the drops to be in equilibrium upward force on 2 2 R 4 rw g 2 2rw g
Þr = Þ r=R
drop = downward force on drop 3T 3T
4 3 2 60. (d) When drops combine to form a single drop of radius R.
T .2pR =pR dg – pR 3rg
3 3 é1 1 ù
Then energy released, E = 4pTR 3 ê - ú
2 3 ër R û
Þ T(2pR) = pR (2d – r) g If this energy is converted into kinetic energy then
3
1 2 é1 1 ù
2 3T mv = 4pR 3T ê - ú
R 2 ër R û
Þ T= (2d – r) g Þ R=
3 (2d – r) g
P-154 Physics

1 é4 3 ù 2 66. (b) Surface tension of a liquid decreases with the rise in


é1 1 ù
´ ê pR rú v = 4pR 3T ê - ú temperture. At the boiling point of liquid, surface tension
2 ë3 û ër R û is zero.
6T é 1 1 ù
v2 = - 2T cos q
r êë r R úû Capillary rise h = rdg
6T é 1 1 ù As surface tension T decreases with rise in temperature
v= -
r êë r R úû hence capillary rise also decreases.
61. (b) 67. (d) Let T is the force due to surface tension per unit length,
62. (a) Given: Radius of air bubble, then
r = 0.1 cm = 10–3 m F = 2lT
Surface tension of liquid, l = length of the slider.
S = 0.06 N/m = 6 × 10–2 N/m At equilibrium, F = W
Density of liquid, r = 103 kg/m3 \ 2Tl = mg
Excess pressure inside the bubble, mg 1.5 ´ 10 -2 1.5
Þ T= = = = 0.025 Nm–1
rexe = 1100 Nm–2 2l 2 ´ 30 ´ 10-2 60
Depth of bubble below the liquid surface, 68. (c) Work done = increase in surface area × surface tension
h=? Þ W = 2T4p[(52) – (3)2] × 10–4
As we know, = 2 × 0.03 × 4p [25 – 9] × 10–4 J
2s = 0.4p × 10–3 J = 0.4p mJ
rExcess = hrg + 69. (c) As volume remains constant
r
\Sum of volumes of 2 smaller drops
2 ´ 6 ´ 10-2
Þ 1100 = h × 103 × 9.8 + = Volume of the bigger drop
10-3
4 4
Þ 1100 = 9800 h + 120 2. pr 3 = pR 3 Þ R = 21/ 3 r
Þ 9800h = 1100 – 120 3 3
980 2
Þ h= = 0.1 m Surface energy = Surface tension × Surface area = T .4 pR
9800
= T 4p 22 / 3 r 2 = T .28 / 3 pr 2 .
63. (a) Acceleration due to gravity changes with the depth,
70. (c) In case of water, the meniscus shape is concave
æ dö
g¢ = g ç1 - ÷ 2s cos q
è R ø upwards. From ascent formula h =
Pressure, P = rgh r rg
x æ dö The surface tension (s) of soap solution is less than water.
Hence ratio, is ç1 - ÷ Therefore height of capillary rise for soap solution should
y è Rø
be less as compared to water. As in the case of water, the
64. (c) Angle of contact q meniscus shape of soap solution is also concave upwards.
TSA - TSL 71. (c) Water fills the tube entirely in gravityless condition
cos q = i.e., 20 cm.
TLA
72. (a) Let pressure outside be P0 and r and R be the radius
when water is on a waxy or oily surface
of smaller bubble and bigger bubble respectively.
TSA < TSL cos q is negative i.e.,
2T
90° < q < 180° \ Pressure P1 For smaller bubble = P0 +
r
i.e., angle of contact q increases
2T
And for q > 90° liquid level in capillary tube fall. P2 For bigger bubble = P0 + ( R > r)
R
i.e., h decreases
\ P1 > P2
65. (b)
hence air moves from smaller bubble to bigger bubble.
10
Thermal Properties
of Matter
T°C, then the new lengths are the same. If the ratio of the
TOPIC 1 Thermometer & Thermal coefficients of linear expansion of A and B is 4 : 3, then
Expansion the value of T is : [11 Jan. 2019 II]
(a) 230°C (b) 270°C
1. Two different wires having lengths L1 and L2 , and (c) 200°C (d) 250°C
respective temperature coefficient of linear expansion a1 7. A thermometer graduated according to a linear scale reads
and a 2 , are joined end-to-end. Then the effective a value x0 when in contact with boiling water, and x0/3
temperature coefficient of linear expansion is : when in contact with ice. What is the temperature of an
[Sep. 05, 2020 (II)] object in °C, if this thermometer in the contact with the
a1 L1 + a 2 L2 object reads x0/2? [11 Jan. 2019 II]
(a) (b) 2 a1a 2 (a) 25 (b) 60 (c) 40 (d) 35
L1 + L2
8. A rod, of length L at room temperature and uniform area
a + a2 aa L2 L1 of cross section A, is made of a metal having coefficient
(c) 1 (d) 4 1 2
2 a1 + a 2 ( L2 + L1 ) 2 of linear expansion a/°C. It is observed that an external
compressive force F, is applied on each of its ends,
2. A bakelite beaker has volume capacity of 500 cc at 30°C.
prevents any change in the length of the rod, when its
When it is partially filled with Vm volume (at 30°C) of
mercury, it is found that the unfilled volume of the beaker temperature rises by DT K. Young’s modulus, Y, for this
remains constant as temperature is varied. If g(beaker) = 6 × metal is: [9 Jan. 2019 I]
10–6 °C–1 and g(mercury) = 1.5 × 10–4 °C–1, where g is the F F
coefficient of volume expansion, then Vm (in cc) is close to (a) (b)
AaDT A a ( D T - 273)
__________. [NA Sep. 03, 2020 (I)]
F 2F
3. When the temperature of a metal wire is increased from (c) (d)
0°C to 10°C, its length increased by 0.02%. The percentage 2A a D T AaDT
change in its mass density will be closest to : 9. An external pressure P is applied on a cube at 0oC so that
[Sep. 02, 2020 (II)] it is equally compressed from all sides. K is the bulk
(a) 0.06 (b) 2.3 modulus of the material of the cube and a is its
(c) 0.008 (d) 0.8 coefficient of linear expansion. Suppose we want to bring
4. A non-isotropic solid metal cube has coefficients of linear the cube to its original size by heating. The temperature
expansion as: 5 ´ l0–5/°C along the x-axis and 5 ´ 10–6/°C should be raised by : [2017]
along the y and the z-axis. If the coefficient of volume
expansion of the solid is C ´ 10–6/°C then the value of C is 3a
(a) (b) 3PKa
¾¾¾ . [NA 7 Jan. 2020 I] PK
5. At 40 C, a brass wire of 1 mm radius is hung from the
o P P
(c) (d)
ceiling. A small mass, M is hung from the free end of the 3aK aK
wire. When the wire is cooled down from 40oC to 20oC it 10. A steel rail of length 5 m and area of cross-section 40
regains its original length of 0.2 m. The value of M is close cm2 is prevented from expanding along its length while
to: [12 April 2019 I] the temperature rises by 10°C. If coefficient of linear
(Coefficient of linear expansion and Young’s modulus of expansion and Young’s modulus of steel are 1.2 × 10–5 K–1
brass are 10–5/oC and 1011 N/m2, respectively; g = 10 ms–2) and 2 × 1011 Nm–2 respectively, the force developed in
(a) 9 kg (b) 0.5 kg (c) 1.5 kg (d) 0.9 kg the rail is approximately: [Online April 9, 2017]
6. Two rods A and B of identical dimensions are at (a) 2 × 107 N (b) 1 × 105 N
temperature 30°C. If A is heated upto 180°C and B upto (c) 2 × 109 N (d) 3 × 10–5 N
P-156 Physics

11. A compressive force, F is applied at the two ends of a K1 K2 K3


long thin steel rod. It is heated, simultaneously, such that
100°C 0°C
its temperature increases by DT. The net change in its
length is zero. Let l be the length of the rod, A its area of 70°C 20°C
cross–section, Y its Youn g’s modulus, and a its
(a) K1 : K3 = 2 : 3, K1 < K3 = 2 : 5
coefficient of linear expansion. Then, F is equal to :
(b) K1 < K2 < K3
[Online April 8, 2017]
(c) K1 : K2 = 5 : 2, K1 : K3 = 3 : 5
(a) l2 Ya DT (b) lA Ya DT
(d) K1 > K2 > K3
AY 16. A bullet of mass 5 g, travelling with a speed of 210 m/s,
(c) A Ya DT (d) strikes a fixed wooden target. One half of its kinetics en-
aDT
ergy is converted into heat in the bullet while the other
12. The ratio of the coefficient of volume expansion of a glass half is converted into heat in the wood. The rise of tem-
container to that of a viscous liquid kept inside the perature of the bullet if the specific heat of its material is
container is 1 : 4. What fraction of the inner volume of the 0.030 cal/(g – ºC) (1 cal = 4.2 × 107 ergs) close to :
container should the liquid occupy so that the volume of [Sep. 05, 2020 (I)]
the remaining vacant space will be same at all temperatures ? (a) 87.5ºC (b) 83.3ºC
[Online April 23, 2013] (c) 119.2ºC (d) 38.4ºC
(a) 2 : 5 (b) 1 : 4 (c) 1 : 64 (d) 1 : 8 17. The specific heat of water = 4200 J kg–1 K–1 and the latent
13. On a linear temperature scale Y, water freezes at – 160° Y heat of ice = 3.4 × 105 J kg–1. 100 grams of ice at 0°C is
and boils at – 50° Y. On this Y scale, a temperature of 340 K placed in 200 g of water at 25°C. The amount of ice that will
would be read as : (water freezes at 273 K and boils at 373 K) melt as the temperature of water reaches 0°C is close to (in
grams) : [Sep. 04, 2020 (I)]
[Online April 9, 2013] (a) 61.7 (b) 63.8
(a) – 73.7° Y (b) – 233.7° Y (c) 69.3 (d) 64.6
(c) – 86.3° Y (d) – 106.3° Y 18. A calorimter of water equivalent 20 g contains 180 g of
14. A wooden wheel of radius R is made of two semicircular water at 25°C. 'm' grams of steam at 100°C is mixed in it till
part (see figure). The two parts are held together by a ring the temperature of the mixture is 31°C. The value of 'm' is
made of a metal strip of cross sectional area S and length close to (Latent heat of water = 540 cal g–1, specific heat of
L. L is slightly less than 2pR. To fit the ring on the wheel, it water = 1 cal g–1 °C–1) [Sep. 03, 2020 (II)]
is heated so that its temperature rises by DT and it just (a) 2 (b) 4
steps over the wheel. As it cools down to surrounding (c) 3.2 (d) 2.6
temperature, it presses the semicircular parts together. If 19. Three containers C1, C2and C3 have water at different
the coefficient of linear expansion of the metal is a, and its temperatures. The table below shows the final temperature
T when different amounts of water (given in liters) are taken
Young's modulus is Y, the force that one part of the wheel
from each container and mixed (assume no loss of heat
applies on the other part is : [2012] during the process) [8 Jan. 2020 II]
(a) 2pSY aDT
R C1 C2 C3 T
(b) SY aDT 1l 2l — 60°C

(c) – 1l 2l 30°C
pSY aDT
2l — 1l 60°C
(d) 2SY aDT 1l 1l 1l q
The value of q (in °C to the nearest integer) is______.
TOPIC 2 Calorimetry and Heat
Transfer 20. M grams of steam at 100°C is mixed with 200 g of ice at its
melting point in a thermally insulated container. If it
15. Three rods of identical cross-section and lengths are made produces liquid water at 40°C [heat of vaporization of water
of three different materials of thermal conductivity K1, K2 is 540 cal/ g and heat of fusion of ice is 80 cal/g], the value
and K3, respectively. They are joined together at their ends
of M is ________ [NA 7 Jan. 2020 II]
to make a long rod (see figure). One end of the long rod is
maintained at 100ºC and the other at 0ºC (see figure). If the 21. When M1 gram of ice at –10 C (Specific heat = 0.5 cal g–1 oC–1)
o

joints of the rod are at 70ºC and 20ºC in steady state and is added to M2 gram of water at 50oC, finally no ice is
there is no loss of energy from the surface of the rod, left and the water is at 0oC. The value of latent heat of
the correct relationship between K1, K2 and K3 is :
ice, in cal g–1 is: [12 April 2019 I]
[Sep. 06, 2020 (II)]
Thermal Properties of Matter P-157

50 M 2 5M1 27. A metal ball of mass 0.1 kg is heated upto 500°C and
(a) -5 (b) M - 50 dropped into a vessel of heat capacity 800 JK–1 and
M1 2 containing 0.5 kg water. The initial temperature of water
50M 2 5 M2 and vessel is 30°C. What is the approximate percentage
(c) (d) -5 increment in the temperature of the water? [Specific Heat
M1 M1
Capacities of water and metal are, respectively, 4200 Jkg–
22. A massless spring (K = 800 N/m), attached with a mass 1K–1 and 400 Jkg–1 K–1 ] [11 Jan. 2019 II]
(500 g) is completely immersed in 1kg of water. The spring (a) 15% (b) 30%
is stretched by 2cm and released so that it starts vibrating.
(c) 25% (d) 20%
What would be the order of magnitude of the change in
28. A heat source at T = 103 K is connected to another heat
the temperature of water when the vibrations stop
reservoir at T = 102 K by a copper slab which is 1 m
completely? (Assume that the water container and spring
thick. Given that the thermal conductivity of copper is
receive negligible heat and specific heat of mass = 400 J/kg K,
0.1 WK–1 m–1, the energy flux through it in the steady
specific heat of water = 4184 J/kg K) [9 April 2019 II]
state is: [10 Jan. 2019 I]
(a) 10–4 K (b) 10–5 K (c) 10–1 K (d) 10–3 K
(a) 90 Wm–2 (b) 120 Wm–2
23. Two materials having coefficients of thermal conductivity (c) 65 Wm–2 (d) 200 Wm–2
‘3K’ and ‘K’ and thickness ‘d’ and ‘3d’, respectively, are
29. An unknown metal of mass 192 g heated to a temperature
joined to form a slab as shown in the figure. The
temperatures of the outer surfaces are ‘q2’ and ‘q1’ of 100°C was immersed into a brass calorimeter of mass
respectively, (q2 > q1). The temperature at the interface is: 128 g containing 240 g of water at a temperature of 8.4°C.
[9 April 2019 II] Calculate the specific heat of the unknown metal if water
temperature stablizes at 21.5°C. (Specific heat of brass is
394 J kg–1 K–1) [10 Jan. 2019 II]
q1 9q 2 q 2 + q1 (a) 458 J kg K
–1 –1
(b) 1232 J kg–1 K–1
(a) + (b) (c) 916 J kg–1 K–1 (d) 654 J kg–1 K–1
10 10 2
q1 5q2 q 2q 30. Temperature difference of 120°C is maintained between
(c) + (d) 1 + 2 two ends of a uniform rod AB of length 2L. Another bent
6 6 3 3
24. A cylinder of radius R is surrounded by a cylindrical shell 3L
of inner radius R and outer radius 2R. The thermal rod PQ, of same cross-section as AB and length , is
conductivity of the material of the inner cylinder is K1 and 2
that of the outer cylinder is K2. Assuming no loss of connected across AB (See figure). In steady state,
heat, the effective thermal conductivity of the system for temperature difference between P and Q will be close to:
heat flowing along the length of the cylinder is: [9 Jan. 2019 I]
[12 Jan. 2019 I]
L
K1 + K 2 4
(a) (b) K1 + K 2
2 A B
2K1 + 3K 2 K1 + 3K 2 L
P L Q
(c) (d)
5 4 2
25. Ice at –20°C is added to 50 g of water at 40°C, When the
(a) 45°C (b) 75°C (c) 60°C (d) 35°C
temperature of the mixture reaches 0°C, it is found that 20
g of ice is still unmelted. The amount of ice added to the 31. A copper ball of mass 100 gm is at a temperature T. It is
water was close to [11 Jan. 2019 I] dropped in a copper calorimeter of mass 100 gm, filled with
(Specific heat of water = 4.2J/g/°C 170 gm of water at room temperature. Subsequently, the
Specific heat of Ice = 2.1 J/g/°C temperature of the system is found to be 75°C. T is given
Heat of fusion of water at 0°C = 334J/g) by (Given : room temperature = 30° C, specific heat of
(a) 50g (b) 100 g copper = 0.1 cal/gm°C [2017]
(c) 60 g (d) 40 g (a) 1250°C (b) 825°C (c) 800°C (d) 885° C
26. When 100 g of a liquid A at 100°C is added to 50 g of a 32. In an experiment a sphere of aluminium of mass 0.20 kg
is heated upto 150°C. Immediately, it is put into water of
liquid B at temperature 75°C, the temperature of the mixture
volume 150 cc at 27°C kept in a calorimeter of water
becomes 90°C. The temperature of the mixture, if 100 g of equivalent to 0.025 kg. Final temperature of the system
liquid A at 100°C is added to 50 g of liquid B at 50°C, will is 40°C. The specific heat of aluminium is :
be : [11 Jan. 2019 II] (take 4.2 Joule=1 calorie) [Online April 8, 2017]
(a) 85°C (b) 60°C (a) 378 J/kg – °C (b) 315 J/kg – °C
(c) 80°C (d) 70°C (c) 476 J/kg – °C (d) 434 J/kg – °C
P-158 Physics

33. An experiment takes 10 minutes to raise the temperature 38. A mass of 50g of water in a closed vessel, with
of water in a container from 0ºC to 100ºC and another 55 surroundings at a constant temperature takes 2 minutes to
minutes to convert it totally into steam by a heater cool from 30°C to 25°C. A mass of 100g of another liquid in
supplying heat at a uniform rate. Neglecting the specific an identical vessel with identical surroundings takes the
heat of the container and taking specific heat of water same time to cool from 30° C to 25° C. The specific heat of
to be 1 cal / g ºC, the heat of vapourization according the liquid is :
to this experiment will come out to be : (The water equivalent of the vessel is 30g.)
[Online April 11, 2015] [Online April 25, 2013]
(a) 560 cal/ g (b) 550 cal/ g (a) 2.0 kcal/kg (b) 7 kcal/kg
(c) 540 cal/ g (d) 530 cal/ g (c) 3 kcal/kg (d) 0.5 kcal/kg
39. 500 g of water and 100 g of ice at 0°C are in a calorimeter
34. Three rods of Copper, Brass and Steel are welded together
whose water equivalent is 40 g. 10 g of steam at 100°C is
to form a Y shaped structure. Area of cross - section of
added to it. Then water in the calorimeter is : (Latent heat
each rod = 4 cm2. End of copper rod is maintained at 100ºC
of ice = 80 cal/g, Latent heat of steam = 540 cal/ g)
where as ends of brass and steel are kept at 0ºC. Lengths
[Online April 23, 2013]
of the copper, brass and steel rods are 46, 13 and 12 cms
(a) 580 g (b) 590 g (c) 600 g (d) 610 g
respectively. The rods are thermally insulated from
surroundings excepts at ends. Thermal conductivities of 40. Given that 1 g of water in liquid phase has volume 1 cm 3
copper, brass and steel are 0.92, 0.26 and 0.12 CGS units and in vapour phase 1671 cm3 at atmospheric pressure
and the latent heat of vaporization of water is 2256 J/g; the
respectively. Rate of heat flow through copper rod is:[2014]
change in the internal energy in joules for 1 g of water at
(a) 1.2 cal/s (b) 2.4 cal/s 373 K when it changes from liquid phase to vapour phase
(c) 4.8 cal/s (d) 6.0 cal/s at the same temperature is : [Online April 22, 2013]
35. A black coloured solid sphere of radius R and mass M is (a) 2256 (b) 167 (c) 2089 (d) 1
inside a cavity with vacuum inside. The walls of the cavity 41. A large cylindrical rod of length L is made by joining two
are maintained at temperature T0. The initial temperature
identical rods of copper and steel of length æç ö÷ each.
L
of the sphere is 3T0. If the specific heat of the material of
the sphere varies as aT3 per unit mass with the temperature è 2ø
T of the sphere, where a is a constant, then the time taken The rods are completely insulated from the surroundings.
for the sphere to cool down to temperature 2T0 will be (s is If the free end of copper rod is maintained at 100°C and
that of steel at 0°C then the temperature of junction is
Stefan Boltzmann constant) [Online April 19, 2014] (Thermal conductivity of copper is 9 times that of steel)
Ma Ma [Online May 19, 2012]
æ3ö æ 16 ö
(a) In ç ÷
2 (b) In ç ÷
2
(a) 90°C (b) 50°C (c) 10°C (d) 67°C
4pR s è 2 ø 4pR s è 3 ø 42. The heat radiated per unit area in 1 hour by a furnace
whose temperature is 3000 K is (s = 5.7 × 10–8 W m–2 K–4)
Ma æ 16 ö Ma æ3ö
(c) In ç ÷
2 (d) In ç ÷
2 [Online May 7, 2012]
16pR s è 3 ø 16pR s è 2 ø (a) 1.7 × 1010 J (b) 1.1 × 1012 J
36. Water of volume 2 L in a closed container is heated with a (c) 2.8 × 108 J (d) 4.6 × 106 J
coil of 1 kW. While water is heated, the container loses 43. 100g of water is heated from 30°C to 50°C. Ignoring the
energy at a rate of 160 J/s. In how much time will the slight expansion of the water, the change in its internal
temperature of water rise from 27°C to 77°C? (Specific heat energy is (specific heat of water is 4184 J/kg/K): [2011]
of water is 4.2 kJ/kg and that of the container is negligible). (a) 8.4 kJ (b) 84 kJ (c) 2.1 kJ (d) 4.2 kJ
[Online April 9, 2014] 44. The specific heat capacity of a metal at low temperature
(a) 8 min 20 s (b) 6 min 2 s (T) is given as
(c) 7 min (d) 14 min 3
æ T ö
37. Assume that a drop of liquid evaporates by decrease in its C p (kJK -1kg -1 ) = 32 ç
è 400 ÷ø
surface energy, so that its temperature remains
A 100 gram vessel of this metal is to be cooled from 20ºK
unchanged.What should be the minimum radius of the
to 4ºK by a special refrigerator operating at room
drop for this to be possible? The surface tension is T, temperature (27°C). The amount of work required to
density of liquid is r and L is its latent heat of cool the vessel is [2011 RS]
vaporization. [2013] (a) greater than 0.148 kJ
(a) rL/T (b) T / rL (c) T/rL (d) 2T/rL (b) between 0.148 kJ and 0.028 kJ
(c) less than 0.028 kJ
(d) equal to 0.002 kJ
Thermal Properties of Matter P-159

45. A long metallic bar is carrying heat from one of its ends 49. The figure shows a system of two concentric spheres of
to the other end under steady–state. The variation of radii r1 and r2 are kept at temperatures T1 and T2,
temperature q along the length x of the bar from its hot end respectively. The radial rate of flow of heat in a substance
is best described by which of the following figures?[2009] between the two concentric spheres is proportional to
[2005]
q q

(a) (b) r1
T1
x x
r2 T2
q q
ær ö (r2 - r1 )
(a) ln ç 2 ÷ (b)
è r1 ø (r1 r2 )
(c) (d)
x x r1 r2
(c) ( r2 - r1 ) (d)
(r2 - r1 )
46. One end of a thermally insulated rod is kept at a temperature
50. If the temperature of the sun were to increase from T to
T1 and the other at T2. The rod is composed of two sections
of length l1 and l2 and thermal conductivities K1 and K2 2T and its radius from R to 2R, then the ratio of the radiant
respectively. The temperature at the interface of the two energy received on earth to what it was previously will
section is [2007] be [2004]
(a) 32 (b) 16 (c) 4 (d) 64
T1 l1 l2 T2
51. The temperature of the two outer surfaces of a composite
slab, consisting of two materials having coefficients of
thermal conductivity K and 2K and thickness x and 4x,
K1 K2
respectively, are T2 and T1 (T2 > T1 ) . The rate of heat
( K1l1T1 + K 2l2T2 ) ( K 2l2T1 + K1l1T2 )
(a) (b) transfer through the slab, in a steady state is
( K1l1 + K 2l2 ) ( K1l1 + K 2 l2 )
æ A(T2 - T1 ) K ö
( K 2l1T1 + K1l2T2 ) ( K1l2T1 + K 2l1T2 ) çè ÷ø f , with f equal to [2004]
(c) (d) x
( K 2 l1 + K1l2 ) ( K1l2 + K 2 l1 )
47. Assuming the Sun to be a spherical body of radius R at a x 4x
temperature of TK, evaluate the total radiant powerd
incident of Earth at a distance r from the Sun [2006]

T4 T4 2K T1
(a) 4pr02 R 2s 2 (b) pr02 R2 s 2
K
r r
4 4
T T
(c) r02 R2 s 2 (d) R2 s
4pr r2
where r0 is the radius of the Earth and s is Stefan's constant. 2 1 1
(a) (b) (c) 1 (d)
48. Two rigid boxes containing different ideal gases are placed 3 2 3
on a table. Box A contains one mole of nitrogen at 52. The earth radiates in the infra-red region of the spectrum.
The spectrum is correctly given by [2003]
temperature T0, while Box contains one mole of helium at (a) Rayleigh Jeans law
æ 7ö (b) Planck’s law of radiation
temperature çè ÷ø T0 . The boxes are then put into thermal
3 (c) Stefan’s law of radiation
contact with each other, and heat flows between them until
the gases reach a common final temperature (ignore the (d) Wien’s law
heat capacity of boxes). Then, the final temperature of 53. Heat given to a body which raises its temperature by 1°C
the gases, Tf in terms of T0 is [2006] is [2002]
(a) water equivalent
3 7
(a) T f = T0 (b) T f = T0 (b) thermal capacity
7 3
(c) specific heat
3 5
(c) T f = T0 (d) T f = T0 (d) temperature gradient
2 2
P-160 Physics

54. Infrared radiation is detected by [2002] 62. A hot body, obeying Newton’s law of cooling is cooling
(a) spectrometer (b) pyrometer down from its peak value 80°C to an ambient temperature
(c) nanometer (d) photometer of 30°C. It takes 5 minutes in cooling down from 80°C to
55. Which of the following is more close to a black body? 40°C. How much time will it take to cool down from 62°C to
[2002] 32°C?
(a) black board paint (b) green leaves
(Given In 2 = 0.693, In 5 = 1.609) [Online April 11, 2014]
(c) black holes (d) red roses
(a) 3.75 minutes (b) 8.6 minutes
56. If mass-energy equivalence is taken into account, when
water is cooled to form ice, the mass of water should[2002] (c) 9.6 minutes (d) 6.5 minutes
(a) increase 63. If a piece of metal is heated to temperature q and then
(b) remain unchanged allowed to cool in a room which is at temperature q0, the
(c) decrease graph between the temperature T of the metal and time t
(d) first increase then decrease will be closest to [2013]
57. Two spheres of the same material have radii 1 m and 4 m
and temperatures 4000 K and 2000 K respectively. The
T T
ratio of the energy radiated per second by the first sphere
to that by the second is [2002] q0
(a) (b)
(a) 1 : 1 (b) 16 : 1
(c) 4 : 1 (d) 1 : 9. O t O t

TOPIC 3 Newton's Law of Cooling


T T
q0 q0
58. A metallic sphere cools from 50°C to 40°C in 300 s. If (c) (d)
atmospheric temperature around is 20°C, then the sphere's
temperature after the next 5 minutes will be close to : O t O t
[Sep. 03, 2020 (II)] 64. A liquid in a beaker has temperature q(t) at time t and q0
(a) 31°C (b) 33°C (c) 28°C (d) 35°C is temperature of surroundings, then according to
59. Two identical beakers A and B contain equal volumes of Newton's law of cooling the correct graph between
two different liquids at 60°C each and left to cool down. loge(q – q0) and t is : [2012]
Liquid in A has density of 8 × 102 kg/m3 and specific heat of
2000 J kg–1 K–1 while liquid in B has density of 103 kg m–3
loge (q – q0)

loge (q – q0)
and specific heat of 4000 J kg–1 K–1. Which of the following
best describes their temperature versus time graph
(a) (b)
schematically ? (assume the emissivity of both the beakers
to be the same) [8 April 2019 I]
0 0
t t

(a) (b)
loge (q – q0)
loge (q – q0)

(c) (d)

(c) (d)
0 0
t t

60. A body takes 10 minutes to cool from 60°C to 50°C. The 65. According to Newton’s law of cooling, the rate of cooling
temperature of surroundings is constant at 25°C. Then, of a body is proportional to (Dq)n , where Dq is the
the temperature of the body after next 10 minutes will difference of the temperature of the body and the
be approximately [Online April 15, 2018] surroundings, and n is equal to [2003]
(a) 43°C (b) 47°C (c) 41°C (d) 45°C (a) two (b) three
61. Hot water cools from 60°C to 50°C in the first 10 minutes
(c) four (d) one
and to 42°C in the next 10 minutes. The temperature of
the surroundings is: [Online April 12, 2014]
(a) 25°C (b) 10°C (c) 15°C (d) 20°C
Thermal Properties of Matter P-161

1. (a) Let L'1 and L'2 be the lengths of the wire when Volume increase by 0.06% therefore density decrease by
0.06%.
temperature is changed by DT °C .
4. (60.00) Volume, V = Ibh
At T °C,
Leq = L1 + L2 DV Dl Db Dh
\ g= = + +
V l b h
At T + D°C
(g = coefficient of volume expansion)
L'eq = L'1 + L'2
Þ g = 5 × 10–5 + 5 × 10–6 + 5 × 10–6
\ Leq (1 + a eq DT ) = L1 (1 + a1DT ) + L2 (1 + a 2 DT ) = 60 × 10–6/°C
\ Value of C = 60.00
[Q L ' = L (1 + a DT )]
5. (Bonus) Dtemp = Dload and A = pr2 = p(10–3)2 = p × 10–6
Þ ( L1 + L2 )(1 + a eq DT ) = L1 + L2 + L1a1DT + L2a 2 DT
FL
L a DT =
a1 L1 + a 2 L2 AY
Þ a eq =
L1 + L2
F ´ 0.2
2. (20.00) or 0.2 × 10–5 × 20 =
( p ´ 10 -6 ) ´ 1011
Volume capacity of beaker, V0 = 500 cc
f
Vb = V0 + V0 g beaker DT \ F = 20p N \ m = = 2p = 6.28 kg
g
When beaker is partially filled with Vm volume of mercury,
6. (a) Change in length in both rods are same i.e.
Vb1 = Vm + Vm g m DT Dl1 =D l 2
Unfilled volume (V0 - Vm ) = (Vb - Vm1 ) la1D q1 = la 2 Dq 2

Þ V0 g beaker = Vm g M a1 Dq 2 é a1 4 ù
= êQ = ú
a 2 Dq1 ë a2 3 û
V0 g beaker
\Vm = 4 q – 30
gM =
3 180 – 30
500 ´ 6 ´ 10 -6
or, Vm = = 20 cc. q = 230°C
1.5 ´ 10-4
7. (a) Let required temperature = T°C
3. (a) Change in length of the metal wire (Dl) when its
temperature is changed by DT is given by M.P. B.P.
o o o
Dl = l aDT 0C TC 100 C
Here, a = Coefficient of linear expansion
Here, Dl = 0.02%, DT = 10ºC x0 x0
3
Dl 0.02
\a = = x0
l DT 100 ´ 10 2
Þ a = 2 ´ 10-5 x0
Volume coefficient of expansion, g = 3a = 6 ´ 10 -5 6
x 0 x 0 x0
M Þ T° C = – =
Qr = 2 3 6
V æ x ö
& ç x 0 – 0 ÷ = (100 – 0°C)
DV -5 -2 è 3 ø
´ 100 = gDT = (6 ´ 10 ´ 10 ´ 100) = 6 ´ 10
V
P-162 Physics

2x 0 300 340 - 273 °Y - ( -160)


Þ =100 Þ x 0 = =
3 2 373 - 273 -50 - ( -160)
x 0 150
Þ T° C = = = 25° C 67 y + 160
6 6 Þ =
100 110
stress F/ A
8. (a) Young’s modulus Y = = A Dl / l \ Y = – 86.3° Y
strain ( )
Using, coefficient of linear expansion, 14. (d) The Young modulus is given as
Dl Dl stress F/S
a= Þ =aDT Y= =
lD T l strain DL / L
Here, DL = 2p DR L = 2p R
F
\Y = F
A ( aDT ) Y= ´ 2 pR
S2 pDR
9. (c) As we know, Bulk modulus
FR
DV P Þ Y= …(i)
DP = S.DR
K= Þ
æ -DV ö V K DR
çè ÷ The coefficient of linear expansion a =
V ø g=3 a R DT
V = V0 (1 + gDt) DR R 1
DV Þ = a.DT Þ = … (ii)
= gDt R DR aDT
V0 From equation (i) and (ii)
P P P F
\ = gDt Þ Dt = = Y = Þ F = Y .S.aDT
K gK 3aK S .aDT
Thermal stress F A \ The ring is pressing the wheel from both sides, Thus
10. (b) Young's modulus = =
Strain DL L Fnet = 2F = 2YSaDT
15. (a) As the rods are identical, so they have same length (l)
F æ DL ö
Y= çèQ = a D q÷ and area of cross-section (A). They are connected in series.
A. a. Dq L ø
So, heat current will be same for all rods.
Force developed in the rail F = YAaDt
= 2 × 1011 × 40 × 10–4 × 1.2 × 10–5 × 10 æ DQ ö æ DQ ö æ DQ ö
Heat current = ç ÷ =ç ÷ =ç ÷
= 9.6 × 104 = 1 × 105 N è D t ø AB è D t ø BC è Dt øCD
11. (c) Due to thermal exp., change in length (Dl) = l a DT ... (100 - 70) K1 A (70 - 20) K 2 A (20 - 0) K 3 A
(i) Þ = =
Normal stress l l l
Young’s modulus (Y) =
Longitudinal strain Þ K1 (100 - 70) = K 2 (70 - 20) = K3 (20 - 0)
FA Dl F
Y= Þ = Þ K1 (30) = K 2 (50) = K3 (20)
Dl l l AY
Fl K1 K 2 K3
Dl = Þ = =
AY 10 6 15
Fl Þ K1 : K 2 : K3 = 10 : 6 :15
From eqn (i), = l a DT
AY
Þ K1 : K3 = 2 : 3.
F = AY a DT
12. (b) When there is no change in liquid level in vessel 16. (a) According to question, one half of its kinetic energy
then g¢real = g¢ vessel is converted into heat in the wood.
Change in volume in liquid relative to vessel 1 2 1
mv ´ = ms DT
DVapp = Vg 'app Dq = V(g 'real - g 'vessel ) 2 2
v2 210 ´ 210
Reading on any scale – LFP Þ DT = = = 87.5°C
13. (c) 4 ´ s 4 ´ 4.2 ´ 0.3 ´ 1000
UFP - LFP 17. (a) Here ice melts due to water.
= constant for all scales Let the amount of ice melts = mice
mw sw Dq = mice Lice
Thermal Properties of Matter P-163

mw sw Dq 1 2
\ mice = 22. (b) .kx = mC (DT ) + mw Cw DT
Lice 2
0.2 ´ 4200 ´ 25 1 2
= = 0.0617 kg = 61.7 g or ´ 800 ´ 0.02 = 0.5 ´ 400 ´ DT + 1 ´ 4184 ´ DT
3.4 ´ 105 2
18. (a) Heat given by water = mw Cw (Tmix - Tw ) \ DT = 1 ´ 10 -5 K
d q 3d
= 200 ´1´ (31 - 25) 23. (a) H1 = H2 q 2 3k k q1
Heat taken by steam = m Lstem + m Cw (Ts – Tmix)
= m × 540 + m (1) × (100 –31) æ q2 - q ö æ q - q1 ö
or (3k ) A ç ÷ = kA ç
= m × 540 + m (1) × (69) è d ø è 3d ÷ø
From the principal of calorimeter, æ q + 9q 2 ö
or q = ç 1
Heat lost = Heat gained è 10 ÷ø
\ (200)(31 - 25) = m ´ 540 + m(1)(69) 24. (d) Effective thermal conductivity of system
Þ 1200 = m(609) Þ m » 2.
K1A1 + K 2 A 2 A2
19. (50.00) K eq =
A1 + A 2
Let Q1, Q2, Q3 be the temperatures of container C1,C2 and
C3 respectively. K1A1
K1pR 2 + K 2 [p (2R)2 - pR 2 ]
Using principle of calorimetry in container C1, we have =
p(2R)2 K2
(q1 – 60) = 2 ms(60 – q)
Þ q1 – 60 = 120 – 2q K1(pR 2 ) + K 2 (3pR 2 )
K1 + 3K 2
= =
Þ q1 = 180 – 2q ...(i) 4pR 2 4
For container C2 25. (d) Let m gram of ice is added.
ms (q2 – 30) = 2ms (30 – q) From principal of calorimeter
Þ q2= 90 – 2q3 ...(ii) heat gained (by ice) = heat lost (by water)
For container C3 \ 20 × 2.1 × m + (m – 20) × 334
2ms (q1 – 60) = ms (60 – q) = 50 × 4.2 × 40
Þ 2q1–120 = 60 – q 376 m = 8400 + 6680
Þ 2q1 + q = 180 ...(iii) m = 40.1
Also, q1 + q2 + q3 = 3q ...(iv) 26. (c) Heat loss = Heat gain = mSDq
Adding (i), (ii) and (iii) So, mASADqA= mBSBDqB
3q1 + 3q2 + 3q3 = 450 Þ 100 × SA × (100 – 90) = 50 × SB × (90 – 75)
Þ q1 + q2 + q3 = 150
3
Þ 3q = 150 Þ q = 50 ºC 2SA = 1.5SB Þ SA = S
4 B
20. (40) Using the principal of calorimetry Now, 100 × SA × (100 – q) = 50× SB × (q – 50)
Mice Lf + mice (40 – 0) Cw æ 3ö
2 × çè ÷ø × (100 – q) = (q – 50)
= mstream Lv + mstream (100 – 40) Cw 4
Þ M (540) + M × 1 × (100 – 40) 300 – 3q = 2q – 100
= 200 × 80 + 200 × 1 × 40 400 = 5q Þ q = 80°C
27. (d) Assume final temperature = T°C
Þ 600 M = 24000
Heat lass = Heat gain = msDT
Þ M = 40g ÞmB sB DTB = mw swDTw
21. (a) M1Cice × (10) + M1L = M2Cw (50) 0.1 × 400 × (500 – T)
= 0.5 × 4200 × (T – 30) + 800 (T – 30)
or M1 × Cice (=0.5) × 10 + M1L = M2 × 1 × 50
Þ 40 (500 – T) = (T – 30) (2100 + 800)
50M 2 Þ 20000 – 40T = 2900 T– 30 × 2900
Þ L = M -5 Þ 20000 + 30 × 2900 = T(2940)
1
T = 30.4°C
P-164 Physics

DT 6.4 and P × 55 × 60 = mL ...(ii)


´100 = ´100 = 21%, Dividing equation (i) by (ii) we get
T 30
so the closest answer is 20%. 10 C ´100
=
Temp. of Temp. of 55 L
heat source heat reservoir \ L = 550 cal./g.
28. (a) 3 1m 2 34. (c) Rate of heat flow is given by,
10 K 10 K
KA(q1 - q 2 )
æ dQ ö kADT Q=
çè ÷ø = l
dt l
Where, K = coefficient of thermal conductivity
1 æ dQ ö kD T l = length of rod and A = area of cross-section of rod
Energy flux, çè ÷ø =
A dt l 100°C
( 0.1)( 900)
= = 90 W/m 2 Copper
1
29. (c) Let specific heat of unknown metal be ‘s’ According T
to principle of calorimetry, Heat lost B Brass
Steel
= Heat gain m × sDq = m1sbrass (Dq1 + m2 swater + Dq2) 0°C
0°C
Þ 192 × S × (100 – 21.5)
If the junction temperature is T, then
= 128 × 394 × (21.5 – 8.4)
QCopper = QBrass + QSteel
Solving we get,+ 240 × 4200 × (21.5 – 8.4)
S = 916 Jkg–1k–1 0.92 ´ 4(100 - T )
46
DTAB 120 120 ´ 5
= =
30. (a) R AB 8 8R 0.26 ´ 4 ´ (T - 0) 0.12 ´ 4 ´ (T - 0)
R = +
5 13 12
L Þ 200 – 2T = 2T + T
Þ T = 40°C
L/4 R R/4
R/4 L/4 0.92 ´ 4 ´ 60
120 O \ QCopper = = 4.8 cal/s
46
A R/2 P Q R/2 B 35. (c) In the given problem, fall in temperature of sphere,

In steady state temperature difference between P and dT = ( 3T0 - 2T0 ) = T0


Q, Temperature of surrounding, Tsurr = T0
120 ´ 5 3 360 Initial temperature of sphere, Tinitial = 3T0
DTPQ = × R= = 45°C
8R 5 8 Specific heat of the material of the sphere varies as,
31. (d) According to principle of calorimetry,
Heat lost = Heat gain c = aT 3 per unit mass (a = a constant)
100 × 0.1(T – 75) = 100 × 0.1 × 45 + 170 × 1 × 45 Applying formula,
10 T– 750 = 450 + 7650 = 8100
Þ T – 75 = 810
dT sA 4
=
dt McJ
4
(
T - Tsurr )
T = 885°C
T0 s 4pR 2 é
32. (d) According to principle of calorimetry, Þ = ( 3T )4 - ( T0 )4 ùûú
dt Ma ( 3T )3 J ëê 0
Qgiven = Qused 0
0.2 × S × (150 – 40) = 150 × 1 × (40 – 27) + 25 × (40 – 27) Ma 27T04 J
0.2 × S × 110 = 150 × 13 + 25 × 13 Þ dt =
s 4 pR 2 ´ 80T04
Specific heat of aluminium
Solving we get,
13 ´ 25 ´ 7 Time taken for the sphere to cool down temperature 2T 0,
S= = 434 J/kg-°C
0.2 ´ 110
Ma æ 16 ö
33. (b) As Pt = mCDT t= ln ç ÷
So, P × 10 × 60 = mC 100 ...(i) 16pR s è 3 ø
2
Thermal Properties of Matter P-165

36. (a) From question, From formula temperature of junction;


In 1 sec heat gained by water K copper qcopper lsteel + K steel qsteel lcopper
= 1 KW – 160 J/s q=
K copper lsteel + Ksteel lcopper
= 1000 J/s – 160 J/s
= 840 J/s L L
9k ´ 100 ´ + k ´ 0´
Total heat required to raise the temperature of water (vol- 2 2
= L L
ume 2L) from 27°c to 77°c 9k ´ + k ´
= mwater ×sp. ht × Dq 2 2
= 2 × 103 × 4.2 × 50 [Q mass = density × volume] 900
kL
And, 840 × t = 2 × 103 × 4.2 × 50
= 2 = 90°C
10kL
2 ´103 ´ 4.2 ´ 50 2
or, t =
840 42. (a) According to Stefan’s law
= 500 s = 8 min 20s E = s T4
37. (d) When radius is decrease by DR, Heat radiated per unit area in 1 hour (3600s) is
4pR 2 DRrL = 4pT[R 2 - (R - DR) 2 ] = 5.7× 10–8 × (300)4 × 3600 = 1.7 × 1010 J
43. (a) DU = DQ = mcDT
Þ rR2 DRL = T[R 2 - R 2 + 2RDR - DR 2 ]
Þ rR 2 DRL = T2RDR [ DR is very small] 100
=× 4184 (50 – 30) » 8.4 kJ
1000
2T
ÞR= 44. (d) Required work = energy released
rL
38. (d) As the surrounding is identical, vessel is identical
time taken to cool both water and liquid (from 30°C to 25°C)
Here, Q = mc dT ò
is same 2 minutes, therefore 4
4
æ T3 ö 3.2 3
æ dQ ö æ dQ ö
=ç ÷ = ò 0.1 ´ 32 ´ ç 3÷
dT = ò 64 ´ 106 T dT
çè ÷ø è 400 ø
dt water è dt ø liquid 20 20

4
(mw C w + W)DT (ml Cl + W) D T
or, = = 5 ´ 10 –8 òT
3
dT = 0.002kJ
t t
20
(W = water equivalent of the vessel)
Therefore, required work = 0.002 kJ
or , m w C w = m l C l
45. (a) Let Q be the temperature at a distance x from hot end
mWCW of bar. Let Q is the temperature of hot end.
\ Specific heat of liquid , Cl =
ml The heat flow rate is given by
50 ´ 1 dQ kA(q1 - q)
= = 0.5 kcal / kg =
100 dt x
39. (b) As 1g of steam at 100°C melts 8g of ice at 0°C. x dQ
10 g of steam will melt 8× 10 g of ice at 0°C Þ q - q = x dQ Þ q = q1 -
1 kA dt
kA dt
Water in calorimeter = 500 + 80 + 10g = 590g
Thus, the graph of Q versus x is a straight line with a positive
40. (c) intercept and a negative slope.
41. (a) L The above equation can be graphically represented by
option (a).
100°C Copper Steel 0°C 46. (d) Let T be the temperature of the interface.
In the steady state, Q1 = Q2
L/2 L/2 T1 T2
1 2
Let conductivity of steel Ksteel = k then from question
Conductivity of copper Kcopper = 9k
qcopper = 100°C K1 K2
qsteel = 0°C
K1 A(T1 - T ) K 2 A(T - T2 )
L \ = ,
lsteel = lcopper = l1 l2
2
P-166 Physics

where A is the area of cross-section. The radial rate of flow of heat through this elementary
shell will be
Þ K1 A(T1 - T )l 2 = K 2 A(T - T2 )l1
dQ KA[(T - dT ) - T ] - KAdT
Þ K1T1l 2 - K1T l 2 = K 2T l1 - K 2T2 l1 = =
dt dr dr
Þ ( K 2 l1 + K1l 2 )T = K1T1l 2 + K2T2 l1
2 dT
= -4pKr (Q A = 4pr 2 )
K1T1l 2 + K 2T2 l1 dr
Þ T = K 2 l1 + K1l 2 Since the area of the surface through which heat will
flow is not constant. Integrating both sides between the
K1l 2T1 + K 2 l1T2 limits of radii and temperatures of the two shells, we get
= .
K1l 2 + K 2 l1
r2 T2
47. (b) From stefan's law, total power radiated by Sun, E = sT4 × æ dQ ö 1
4pR2 çè ÷
dt ø ò r 2 dr = -4pK ò dT
r1 T1
The intensity of power Per unit area incident on earth's r2 T2
surface æ dQ ö -2
4 2
çè ÷
dt ø ò r dr = -4pK ò dT
sT ´ 4pR r1 T1
= 2
4pr
Total power received by Earth dQ é 1 1 ù
ê - ú = -4pK [T2 - T 1 ]
E dt ë r1 r2 û
E' = × Cross – Section area of earth facing the
4pr 2 dQ -4pKr1r2 (T2 - T1 )
4 2 or =
sT R dt (r2 - r1 )
sun = 2
(pr02 )
r
dQ r r
48. (c) When two gases are mixed to gether then \ µ 1 2
dt (r2 - r1 )
Heat lost by He gas = Heat gained by N2 gas
50. (d) From stefan's law, energy radiated by sun per second
n1Cv1 DT1 = n2Cv2 DT2
E = sAT 4 ;
3 é7 ù 5
R T0 - T f ú = R éëT f - T0 ùû
2 ëê 3 û 2 \ A µ R2

7T0 - 3T f = 5T f - 5T0 \ E µ R 2T 4

12 E2 R22 T24
Þ 12T0 = 8T f Þ T f = T0 \ =
8 E1 R12 T14
3 put R2 = 2R, R1 = R ; T2 = 2T, T1 = T
Þ Tf = T0 ..
2
49. (d) E (2R )2 (2T ) 4
Þ 2 = = 64
T - dT E1 R 2T 4
dr 51. (d) The thermal resistance is given by
·
T1 x 4x x 2 x 3x
r1
r + = + =
KA 2KA KA KA KA
Amount of heat flow per second,
T2 dQ D T (T - T1 ) KA
r2 = = 2
dt 3 x 3x
KA
Consider a thin concentric shell of thickness (dr) and of
radius (r) and let the temperature of inner and outer 1 ì A(T2 - T1 ) K ü 1
surfaces of this shell be T and (T – dT) respectively. = í ý \f =
3î x þ 3
Thermal Properties of Matter P-167

52. (d) Wein’s law correctly explains the spectrum


dT es ´ A ´ T 4 dT 1
53. (b) Heat required for raising the temperature of a body - = Þ- µ
through 1ºC is called its thermal capacity. dt r ´ Vol. ´ S dt rS
54. (b) Pyrometer is used to detect infra-red radiation.
æ dT ö
55. (a) Black body is one which absorb all incident radiation. çè - ÷ø
Black board paint is quite approximately equal to black dt A rB S B 103 4000
= ´ = ´
bodies. æ dT ö r A S A 8 ´ 102 2000
56. (c) When water is cooled at 0°C to form ice, energy is çè - ÷ø
dt B
released from water in the form of heat. As energy is
equivalent to mass, therefore, when water is cooled to æ dT ö æ dT ö
ice, its mass decreases. Þ ç- ÷ > ç- ÷
è dt ø A è dt ø B
57. (a) From stefan's law, the energy radiated per second is
given by E = esT4 A So, A cools down at faster rate.
Here, T = temperature of the body 60. (a) According to Newton’s law of cooling,
A = surface area of the body æ q1 - q2 ö æ q1 + q2 ö
For same material e is same. s is stefan's constant çè ÷ø = K çè - q0 ÷
ø
t 2
Let T1 and T2 be the temperature of two spheres. A1 and
A2 be the area of two spheres. æ 60 - 50 ö æ 60 + 50 ö
çè ÷ø = K çè - 25÷
ø
..... (i)
10 2
E1 T14 A1 T14 4pr12
\ = = æ 50 - q ö æ 50 + q ö
E2 T24 A2 T24 4pr22 and, çè ÷ø = K çè - 25÷ ..... (ii)
10 2 ø
Dividing eq. (i) by (ii),
(4000)4 ´12 1
= =
(2000)4 ´ 42 1 10 60
= Þ q = 42.85°C @ 43°C
58. (b) From Newton's Law of cooling, (50 - q) q
61. (b) By Newton’s law of cooling
T1 - T2 éT + T ù
= K ê 1 2 - T0 ú
t ë 2 û q1 - q2 é q + q2 ù
= -K ê 1 - q0 ú
Here, T1 = 50°C, T2 = 40°C t ë 2 û
an d To = 20°C, t = 600S = 5 minutes where q0 is the temperature of surrounding.
50 - 40 æ 50 + 40 ö Now, hot water cools from 60°C to 50°C in 10 minutes,
Þ = Kç - 20 ÷ ...(i)
5 Min è 2 ø 60 - 50 é 60 + 50 ù
= -K ê - q0 ú ...(i)
Let T be the temperature of sphere after next 5 minutes. 10 ë 2 û
Then
Again, it cools from 50°C to 42°C in next 10 minutes.
40 - T æ 40 + T ö
= Kç - 20 ÷ ...(ii) 50 - 42 é 50 + 42 ù
5 è 2 ø = -K ê - q0 ú ...(ii)
10 ë 2 û
Dividing eqn. (ii) by (i), we get
Dividing equations (i) by (ii) we get
40 - T 40 + T - 40 T
= = 1 55 - q0
10 50 + 40 - 40 50 =
0.8 46 - q0
T
Þ 40 - T = Þ 200 - 5T = T 10 55 - q 0
5 =
8 46 - q 0
200 460 - 10q0 = 440 - 8q0
\T = = 33.3°C
6
2q0 = 20
æ dT ö 4
59. (b) Rate of Heat loss = mS çè ÷ø = esAT q0 = 10°C
dt
P-168 Physics

62. (b) From Newton’s law of cooling, 63. (c) According to Newton’s law of cooling, the
temperature goes on decreasing with time non-linearly.
1 æ q - q0 ö
t= log e ç 2 ÷ 64. (a) According to newton's law of cooling
k è q1 - q0 ø
dq
From question and above equation, = - k(q - q0 )
dt
1 (40 - 30) dq
5= log e ...(1) Þ = - kdt
k (80 - 30) (q - q0 )
q t
1 (32 - 30) dq
And, t =
k
log e
(62 - 30)
...(2) Þ ò (q - q 0 )
= - k ò dt
q0 q
Dividing equation (2) by (1),
Þ log(q - q0 ) = -kt + c
1 (32 - 30)
log e Which represents an equation of straight line.
t k (62 - 30)
= Thus the option (a) is correct.
5 1 (40 - 30)
log e
k (80 - 30) dQ
65. (d) From Newton’s law of cooling - µ (Dq)
On solving we get, time taken to cool down from 62°C dt
to 32°C, t = 8.6 minutes.
11
Thermodynamics P-169

Thermodynamics
1 1T2 + P2V2T1
PV 1 1 + P2V2 )
T1T2 ( PV
TOPIC 1 First Law of Thermodynamics (c)
1 1 + P2V2
PV (d)
1 1 1 + P2V2T2
PV T

1. A gas can be taken from A to B via two different processes 5. When a system is taken from state i to state f along the
ACB and ADB. path iaf, it is found that Q =50 cal and W = 20 cal. Along the
path ibf Q = 36 cal. W along the path ibf is [2007]
a f

i b
(a) 14 cal (b) 6 cal (c) 16 cal (d) 66 cal
6. A system goes from A to B via two processes I and II as
shown in figure. If DU1 and DU2 are the changes in internal
When path ACB is used 60 J of heat flows into the system energies in the processes I and II respectively, then [2005]
and 30J of work is done by the system. If path ADB is p II
used work done by the system is 10 J. The heat Flow into
the system in path ADB is : [9 Jan. 2019 I] A B
(a) 40 J (b) 80 J (c) 100 J (d) 20 J I
2. 200g water is heated from 40°C to 60°C. Ignoring the slight
expansion of water, the change in its internal energy is v
close to (Given specific heat of water = 4184 J/kgK): (a) relation between DU1 and DU 2 can not be determined
[Online April 9, 2016]
(a) 167.4 kJ (b) 8.4 kJ (c) 4.2 kJ (d) 16.7 kJ (b) DU1 = DU 2
3. A gas is compressed from a volume of 2m3 to a volume of (c) DU 2 < DU1
1m3 at a constant pressure of 100 N/m2. Then it is heated at (d) DU 2 > DU1
constant volume by supplying 150 J of energy. As a result,
7. Which of the following is incorrect regarding the first law
the internal energy of the gas: [Online April 19, 2014]
of thermodynamics? [2005]
(a) increases by 250 J (b) decreases by 250 J
(a) It is a restatement of the principle of conservation of
(c) increases by 50 J (d) decreases by 50 J energy
4. An insulated container of gas has two chambers separated (b) It is not applicable to any cyclic process
by an insulating partition. One of the chambers has volume (c) It does not introduces the concept of the entropy
V1 and contains ideal gas at pressure P1 and temperature
(d) It introduces the concept of the internal energy
T1. The other chamber has volume V2 and contains
ideal gas at pressure P2 and temperature T2. If the partition
Specific Heat Capacity and
is removed without doing any work on the gas, the TOPIC 2
final equilibrium temperature of the gas in the container
Thermodynamical Processes
will be [2008] 8. Three different processes that can occur in an ideal
monoatomic gas are shown in the P vs V diagram. The
1 1 + P2V2 )
T1T2 ( PV 1 1T1 + P2V2T2
PV
(a) (b) paths are lebelled as A ® B, A ® C and A ® D. The change
1 1T2 + P2V2T1
PV 1 1 + P2V2
PV
P-170 Physics

in internal energies during these process are taken as EAB,


EAC and EAD and the workdone as WAB, WAC and WAD.
The correct relation between these parameters are :
[5 Sep. 2020 (I)]
D T1>T2
C
P
B (a) (b)
A T1
T2
V

(a) EAB = EAC < EAD, WAB > 0, WAC = 0, WAD < 0
(b) EAB = EAC = EAD, WAB > 0, WAC = 0, WAD > 0 (c) (d)
(c) EAB < EAC < EAD, WAB > 0, WAC > WAD
(d) EAB > EAC > EAD, WAB < WAC < WAD
9. In an adiabatic process, the density of a diatomic gas
becomes 32 times its initial value. The final pressure of the
gas is found to be n times the initial pressure. The value of
n is : [5 Sep. 2020 (II)] 14. Starting at temperature 300 K, one mole of an ideal diatomic
1 gas (g = 1.4) is first compressed adiabatically from volume
(a) 32 (b) 326 (c) 128 (d)
32 V1
V1 to V2 = . It is then allowed to expand isobarically to
10. Match the thermodynamic processes taking place in a 16
system with the correct conditions. In the table : DQ is the volume 2V2. If all the processes are the quasi-static then
heat supplied, DW is the work done and DU is change in the final temperature of the gas (in °K) is (to the nearest
internal energy of the system. [4 Sep. 2020 (II)] integer) ______. [9 Jan. 2020 II]
Process Condition
15. A thermodynamic cycle xyzx is shown on a V-T diagram.
(I) Adiabatic (A) DW = 0
(II) Isothermal (B) DQ = 0
(III) Isochoric (C) DU ¹ 0, DW ¹ 0,
DQ ¹ 0
(IV) Isobaric (D) DU = 0
(a) (I)-(A), (II)-(B), (III)-(D), (IV)-(D)
(b) (I)-(B), (II)-(A), (III)-(D), (IV)-(C) The P-V diagram that best describes this cycle is:
(c) (I)-(A), (II)-(A), (III)-(B), (IV)-(C) (Diagrams are schematic and not to scale)
(d) (I)-(B), (II)-(D), (III)-(A), (IV)-(C) [8 Jan. 2020 I]
11. A balloon filled with helium (32°C and 1.7 atm.) bursts.
Immediately afterwards the expansion of helium can be
considered as : [3 Sep. 2020 (I)]
(a) irreversible isothermal (b) irreversible adiabatic (a) (b)
(c) reversible adiabatic (d) reversible isotherm7al
12. An engine takes in 5 mole of air at 20°C and 1 atm, and
compresses it adiabaticaly to 1/10th of the original
volume. Assuming air to be a diatomic ideal gas made up
of rigid molecules, the change in its internal energy during
this process comes out to be X kJ. The value of X to the (c) (d)
nearest integer is ________. [NA 2 Sep. 2020 (I)]
13. Which of the following is an equivalent cyclic process
16. A litre of dry air at STP expands adiabatically to a volume
corresponding to the thermodynamic cyclic given in the
of 3 litres. If g = 1.40, the work done by air is:
figure?
(31.4 = 4.6555) [Take air to be an ideal gas]
where, 1 ® 2 is adiabatic.
(Graphs are schematic and are not to scale) [9 Jan. 2020 I] [7 Jan. 2020 I]
(a) 60.7 J (b) 90.5 J (c) 100.8 J (d) 48 J
Thermodynamics P-171

17. Under an adiabatic process, the volume of an ideal gas gets (a) DQA < DQB, DUA < DUB
doubled. Consequently the mean collision time between
(b) DQA > DQB, DUA > DUB
Cp
the gas molecule changes from t1 to t2. If = g for this (c) DQA > DQB, DUA = DUB
Cv
(d) DQA = DQB; DUA = DUB
t2
gas then a good estimate for t is given by: 23. A thermally insulted vessel contains 150 g of water at 0°C.
1
Then the air from the vessel is pumped out adiabatically. A
[7 Jan. 2020 I] fraction of water turns into ice and the rest evaporates at
g +1
g 0°C itself. The mass of evaporated water will be closed to:
1 æ 1ö æ 1ö 2
(a) 2 (b) (c) çè ÷ø (d) çè ÷ø (Latent heat of vaporization of water = 2.10 × 106 J kg–1 and
2 2 2 Latent heat of Fusion of water = 3.36 × 10 5 J kg–1)
18. A sample of an ideal gas is taken through the cyclic process [8 April 2019 I]
abca as shown in the figure. The change in the internal (a) 150 g (b) 20 g (c) 130 g (d) 35 g
energy of the gas along the path ca is – 180 J, The gas 24. The given diagram shows four processes i.e., isochoric,
absorbs 250 J of heat along the path ab and 60 J along the isobaric, isothermal and adiabatic. The correct assignment
path bc. The work down by the gas along the path abc is: of the processes, in the same order is given by :
[12 Apr. 2019 I] [8 Apr. 2019 II]

(a) 120 J (b) 130 J (c) 100 J (d) 140 J


19. A cylinder with fixed capacity of 67.2 lit contains helium (a) a d b c (b) d a c b (c) a d c b (d) d a b c
gas at STP. The amount of heat needed to raise the 25. For the given cyclic process CAB as shown for gas, the
temperature of the gas by 20oC is : [Given that R = 8.31 J work done is: [12 Jan. 2019 I]
mol – 1 K – 1] [10 Apr. 2019 I] 6.0 C
A
(a) 350 J (b) 374 J (c) 748 J (d) 700 J
5
20. n moles of an ideal gas with constant volume heat capacity
CV undergo an isobaric expansion by certain volume. The 4
ratio of the work done in the process, to the heat supplied
p(Pa) 3
is: [10 Apr. 2019 I]
nR nR 2
(a) (b) C - nR
CV + nR V 1 B

4nR 4nR
(c) (d) C + nR 1 2 3 4 5
CV - nR V V(m3)
(a) 30 J (b) 10 J (c) 1 J (d) 5 J
21. One mole of an ideal gas passes through a process where
26. A rigid diatomic ideal gas undergoes an adiabatic process
é 1 æ V ö2 ù at room temperature. The relation between temperature and
pressure and volume obey the relation P = P0 ê1 - ç 0 ÷ ú . volume for this process is TVx = constant, then x is:
êë 2 è V ø úû
[11 Jan. 2019 I]
Here Po and Vo are constants. Calculate the charge in the 3 2 2 5
temperature of the gas if its volume changes from Vo to 2Vo. (a) (b) (c) (d)
5 5 3 3
[10 Apr. 2019 II] 27. Half mole of an ideal monoatomic gas is heated at constant
1 Po Vo 5 Po Vo 3 Po Vo 1 Po Vo pressure of 1 atm from 20°C to 90°C. Work done by gas is
(a) (b) (c) (d) close to: (Gas constant R = 8.31 J/mol-K) [10 Jan. 2019 II]
2 R 4 R 4 R 4 R
22. Following figure shows two processes A and B for a gas. (a) 581 J (b) 291 J (c) 146 J (d) 73 J
If DQA and DQB are the amount of heat absorbed by the 28. One mole of an ideal monoatomic gas is taken along the
system in two cases, and DUA and DUB are changes in path ABCA as shown in the PV diagram. The maximum
internal energies, respectively, then: [9 April 2019 I] temperature attained by the gas along the path BC is given
by [Online April 16, 2018]
P-172 Physics

P V
B
c
3P0
b
P0 A C
V d
V0 2V0 a
T
(a) 25 P0 V0 (b) 25 P0 V0 (c) 25 P0 V0 (d) 5 P0V0 The corresponding P - V diagram for the process is (all
8 R 4 R 16 R 8 R figures are schematic and not drawn to scale) :
29. One mole of an ideal monoatomic gas is compressed
[Online April 10, 2015]
isothermally in a rigid vessel to double its pressure at room
temperature, 27°C. The work done on the gas will be: P P
[Online April 15, 2018] d c a b
(a) 300R ln 6 (b) 300R (a) (b)
(c) 300R ln 7 (d) 300R ln 2 a b d c
30. 'n' moles of an ideal gas undergoes a process A ® B as
shown in the figure. The maximum temperature of the gas V V
during the process will be : [2016]
P P
P d c a b
A
2P0 (c) a b (d) d c

P0 B V V
35. One mole of a diatomic ideal gas undergoes a cyclic process
V0 2V0 V ABC as shown in figure. The process BC is adiabatic. The
temperatures at A, B and C are 400 K, 800 K and 600 K
9P0 V0 9P0 V0 9P0 V0 3P0 V0
(a) (b) (c) (d) respectively. Choose the correct statement:
2nR nR 4nR 2nR
[2014]
31. The ratio of work done by an ideal monoatomic gas to the
heat supplied to it in an isobaric process is :
[Online April 9, 2016] B
2 3 3 2 800 K
(a) (b) (c) (d)
5 2 5 3
32. Consider an ideal gas confined in an isolated closed P
chamber. As the gas undergoes an adiabatic expansion,
the average time of collision between molecules increases 600 k
as Vq, where V is the volume of the gas. The value of q A C
400 K
æ Cp ö
is : ç g = ÷ [2015]
è Cv ø V
g +1 g -1 3g + 5 3g - 5 (a) The change in internal energy in whole cyclic process
(a) (b) (c) (d) is 250 R.
2 2 6 6
33. Consider a spherical shell of radius R at temperature T. The (b) The change in internal energy in the process CA is 700 R.
black body radiation inside it can be considered as an ideal
(c) The change in internal energy in the process AB is -
gas of photons with internal energy per unit volume u =
350 R.
U 1æUö
µ T 4 and pressure p = ç ÷ . If the shell now (d) The change in internal energy in the process BC is –
V 3è V ø
undergoes an adiabatic expansion the relation between T 500 R.
and R is : [2015] 36. An ideal monoatomic gas is confined in a cylinder by a spring
loaded piston of cross section 8.0 × 10–3 m2. Initially the gas
1 1
(a) T µ (b) Tµ (c) T µ e–R (d) T µ e–3R is at 300 K and occupies a volume of 2.4 × 10–3 m3 and the
R R3 spring is in its relaxed state as shown in figure. The gas is
34. An ideal gas goes through a reversible cycle a®b®c®d heated by a small heater until the piston moves out slowly
has the V - T diagram shown below. Process d®a and by 0.1 m. The force constant of the spring is 8000 N/m and
b®c are adiabatic.
the atmospheric pressure is 1.0 × 105 N/m2. The cylinder and
Thermodynamics P-173

the piston are thermally insulated. The piston and the spring 41. Helium gas goes through a cycle ABCDA (consisting of
are massless and there is no friction between the piston and two isochoric and isobaric lines) as shown in figure. The
the cylinder. The final temperature of the gas will be: efficiency of this cycle is nearly : (Assume the gas to be
close to ideal gas) [2012]
(Neglect the heat loss through the lead wires of the heater.
B C
The heat capacity of the heater coil is also negligible). (a) 15.4 % 2P0
[Online April 11, 2014]
(b) 9.1 %
P0 D
A
(c) 10.5%
(d) 12.5 % V0 2V0
42. An ideal monatomic gas with pressure P, volume V and
temperature T is expanded isothermally to a volume 2V
and a final pressure Pi. If the same gas is expanded
adiabatically to a volume 2V, the final pressure is Pa. The
(a) 300 K (b) 800 K (c) 500 K (d) 1000 K P
ratio a is [Online May 26, 2012]
37. During an adiabatic compression, 830 J of work is done on Pi
2 moles of a diatomic ideal gas to reduce its volume by (a) 2–1/3 (b) 21/3 (c) 22/3 (d) 2–2/3
50%. The change in its temperature is nearly: 43. The pressure of an ideal gas varies with volume as P = aV,
(R = 8.3 JK–1 mol–1) [Online April 11, 2014] where a is a constant. One mole of the gas is allowed to
(a) 40 K (b) 33 K (c) 20 K (d) 14 K undergo expansion such that its volume becomes ‘m’ times
38. The equation of state for a gas is given by PV = nRT + aV, its initial volume. The work done by the gas in the process
where n is the number of moles and a is a positive constant. is [Online May 19, 2012]
The initial temperature and pressure of one mole of the gas
contained in a cylinder are To and Po respectively. The
work done by the gas when its temperature doubles
(a)
aV 2
2
(
m -1 ) (b)
a 2V 2 2
2
m -1 ( )
( ) aV 2 2
( )
isobarically will be: [Online April 9, 2014] a 2
(c) m -1 (d) m -1
Po To R Po To R 2 2
(a) (b) 44. n moles of an ideal gas undergo a process A ® B as shown
Po - a Po + a
in the figure. Maximum temperature of the gas during the
(c) PoToRIn 2 (d) PoToR process is [Online May 12, 2012]
39. A certain amount of gas is taken through a cyclic process
(A B C D A) that has two isobars, one isochore and one A
isothermal. The cycle can be represented on a P-V indicator 2P0
diagram as : [Online April 22, 2013]
P0 B
B C B
P P C

P V0 2V
0
(a) (b) D
V
A D A
9P0V0 3P0V0 9 P0V0 9 P0V0
V V (a) (b) (c) (d)
nR 2 nR 2 nR 4 nR
B C
45. This question has Statement 1 and Statement 2. Of the four
P B choices given after the Statements, choose the one that
P C
best describes the two Statements.
(c) (d) Statement 1: In an adiabatic process, change in internal
A D
A D energy of a gas is equal to work done on/by the gas in the
V process.
V
Statement 2: The temperature of a gas remains constant
40. An ideal gas at atmospheric pressure is adiabatically
compressed so that its density becomes 32 times of its in an adiabatic process. [Online May 7, 2012]
initial value. If the final pressure of gas is 128 atmospheres, (a) Statement 1 is true, Statement 2 is true, Statement 2 is a
the value of ‘g’of the gas is : correct explanation of Statement 1.
[Online April 22, 2013] (b) Statement 1 is true, Statement 2 is false.
(a) 1.5 (b) 1.4 (c) 1.3 (d) 1.6
P-174 Physics

(c) Statement 1 is false, Statement 2 is true. 53. If minimum possible work is done by a refrigerator in
(d) Statement 1 is false, Statement 2 is true, Statement 2 is converting 100 grams of water at 0°C to ice, how much heat
not a correct explanation of Statement 1. (in calories) is released to the surroundings at temperature
46. A container with insulating walls is divided into equal parts 27°C (Latent heat of ice = 80 Cal/gram) to the nearest integer?
by a partition fitted with a valve. One part is filled [NA 3 Sep. 2020 (II)]
with an ideal gas at a pressure P and temperature T, whereas 54. A heat engine is involved with exchange of heat of 1915 J,
the other part is completly evacuated. If the valve is – 40 J, +125 J and – Q J, during one cycle achieving an
suddenly opened, the pressure and temperature of the gas efficiency of 50.0%. The value of Q is :
will be : [2011 RS] [2 Sep. 2020 (II)]
(a) 640 J (b) 40 J (c) 980 J (d) 400 J
P T T P 1
(a) , (b) P, T (c) P, (d) ,T 55. A Carnot engine having an efficiency of is being used
2 2 2 2 10
Directions for questions 47 to 49: Questions are based on as a refrigerator. If the work done on the refrigerator is 10
the following paragraph. J, the amount of heat absorbed from the reservoir at lower
Two moles of helium gas are taken over the cycle ABCDA, as temperature is: [8 Jan. 2020 II]
shown in the P-T diagram. [2009] (a) 99 J (b) 100 J (c) 1 J (d) 90 J
5 A B 56. A Carnot engine operates between two reservoirs of
2 × 10 temperatures 900 K and 300 K. The engine performs 1200 J
of work per cycle. The heat energy (in J) delivered by the
P (Pa)
engine to the low temperature reservoir, in a cycle, is
1 × 10
5
C
_______. [NA 7 Jan. 2020 I]
D
T 57. Two ideal Carnot engines operate in cascade (all heat
300K 500K
given up by one engine is used by the other engine to
47. Assuming the gas to be ideal the work done on the gas in produce work) between temperatures, T1 and T2. The
taking it from A to B is temperature of the hot reservoir of the first engine is T1
(a) 300 R (b) 400 R (c) 500 R (d) 200 R and the temperature of the cold reservoir of the second
48. The work done on the gas in taking it from D to A is engine is T2. T is temperature of the sink of first engine
(a) + 414 R (b) – 690 R (c) + 690 R (d) – 414 R which is also the source for the second engine. How is
49. The net work done on the gas in the cycle ABCDA is T related to T1 and T2, if both the engines perform equal
amount of work ? [7 Jan. 2020 II]
(a) 279 R (b) 1076 R (c) 1904 R (d) zero
2T1T2 T1 + T2
50. The work of 146 kJ is performed in order to compress one (a) T = T + T (b) T =
kilo mole of gas adiabatically and in this process the 1 2 2
temperature of the gas increases by 7°C. The gas is [2006] (c) T = T1T2 (d) T = 0
(R = 8.3 J mol–1 K–1)
58. A Carnot engine has an efficiency of 1/6. When the
(a) diatomic
temperature of the sink is reduced by 62oC, its efficiency
(b) triatomic
is doubled. The temperatures of the source and the sink
(c) a mixture of monoatomic and diatomic
are, respectively. [12 Apr. 2019 II]
(d) monoatomic
51. Which of the following parameters does not characterize (a) 62 C, 124 C
o o
(b) 99 C, 37oC
o

the thermodynamic state of matter? [2003] (c) 124 C, 62 C


o o
(d) 37oC, 99oC
(a) Temperature (b) Pressure 59. Three Carnot engines operate in series between a heat
source at a temperature T1 and a heat sink at temperature
(c) Work (d) Volume T4 (see figure). There are two other reservoirs at temperature
T2 and T3, as shown, with T1 > T2 > T3 > T(4) The three
Carnot Engine, Refrigerators engines are equally efficient if: [10 Jan. 2019 I]
TOPIC 3 and Second Law of
Thermodynamics
52. An engine operates by taking a monatomic ideal gas through
the cycle shown in the figure. The percentage efficiency of
the engine is close is ______. [NA 6 Sep. 2020 (II)]

3 PO B C

2 PO

PO
A D

VO 2VO
Thermodynamics P-175

( ) (ii) Sequentially keeping in contact with 8 reservoirs such


1/3
(a) T2 = ( T1 T4 )
1/2
; T3 = T12 T4
that each reservoir supplies same amount of heat.
= (T T ) ; T = (T T )
1/3 1/3
(b) T2 2
1 4 3
2
1 4
In both the cases body is brought from initial temperature
100°C to final temperature 200°C. Entropy change of the
= (T T ) ; T = (T T )
1/3 1/3
2 2
(c) T2 1 4 3 1 4 body in the two cases respectively is :
(a) ln2, 2ln2 (b) 2ln2, 8ln2
= ( T T ) ; T = (T T )
1/4 1/4
3 3
(d) T2 1 4 3 1 4
(c) ln2, 4ln2 (d) ln2, ln2
60. Two Carnot engines A and B are operated in series. The
66. A Carnot engine absorbs 1000 J of heat energy from a
first one, A receives heat at T1 (= 600 K) and rejects to a
reservoir at 127°C and rejects 600 J of heat energy during
reservoir at temperature T2. The second engine B receives each cycle. The efficiency of engine and temperature of
heat rejected by the first engine and in turn, rejects to a sink will be: [Online April 12, 2014]
heat reservoir at T3 (= 400 K). Calculate the temperature (a) 20% and – 43°C (b) 40% and – 33°C
T2 if the work outputs of the two engines are equal: (c) 50% and – 20°C (d) 70% and – 10°C
[9 Jan. 2019 II]
(a) 600 K (b) 400 K (c) 300 K (d) 500 K 67. p
61. A Carnot's engine works as a refrigerator between 250 K A B
2p0
and 300 K. It receives 500 cal heat from the reservoir at the
lower temperature.The amount of work done in each cycle
p0
to operate the refrigerator is: [Online April 15, 2018] D C
(a) 420 J (b) 2100 J (c) 772 J (d) 2520 J
62. Two Carnot engines A and B are operated in series. Engine v0 2v0 v
A receives heat from a reservoir at 600K and rejects heat to
The above p-v diagramrepresents the thermodynamic cycle
a reservoir at temperature T. Engine B receives heat rejected
of an engine, operating with an ideal monatomic gas. The
by engine A and in turn rejects it to a reservoir at 100K. If
amount of heat, extracted from the source in a single cycle
the efficiencies of the two engines A and B are represented
is [2013]
h
by hA and hB respectively, then what is the value of A
hB æ 13 ö
(a) p 0 v 0 (b) ç ÷ p0 v0
[Online April 15, 2018] è2ø
12 12 5 7
(a) (b) (c) (d) æ 11 ö
7 5 12 12 (c) ç ÷ p0 v0 (d) 4p0v0
è2ø
63. An engine operates by taking n moles of an ideal gas
through the cycle ABCDA shown in figure. The thermal 68. A Carnot engine, whose efficiency is 40%, takes in heat
efficiency of the engine is : (Take Cv =1.5 R, where R is gas from a source maintained at a temperature of 500K. It is
constant) [Online April 8, 2017] desired to have an engine of efficiency 60%. Then, the
B C intake temperature for the same exhaust (sink) temperature
(a) 0.24 2P0 must be : [2012]
(a) efficiency of Carnot engine cannot be made larger than 50%
P
(b) 0.15 P0 (b) 1200 K
D
A
(c) 750 K
(c) 0.32
V0 2V0 (d) 600 K
V 69. The door of a working refrigerator is left open in a
(d) 0.08
well insulated room. The temperature of air in the room
64. A Carnot freezer takes heat from water at 0°C inside it and will [Online May 26, 2012]
rejects it to the room at a temperature of 27°C. The latent (a) decrease
heat of ice is 336 × 103 J kg–1. If 5 kg of water at 0°C is (b) increase in winters and decrease in summers
converted into ice at 0°C by the freezer, then the energy (c) remain the same
consumed by the freezer is close to : (d) increase
[Online April 10, 2016] 70. This question has Statement 1 and Statement 2. Of the four
(a) 1.51 × 105 J (b) 1.68 × 106 J choices given after the Statements, choose the one that
(c) 1.71 × 107 J (d) 1.67 × 105 J best describes the two Statements.
65. A solid body of constant heat capacity 1 J/°C is being heated Statement 1: An inventor claims to have constructed an
by keeping it in contact with reservoirs in two ways : [2015] engine that has an efficiency of 30% when operated
(i) Sequentially keeping in contact with 2 reservoirs such between the boiling and freezing points of water. This is
that each reservoir supplies same amount of heat. not possible.
P-176 Physics

Statement 2: The efficiency of a real engine is always 1 1 2 1


less than the efficiency of a Carnot engine operating (a) (b) (c) (d)
4 2 3 3
between the same two temperatures.
75. Which of the following statements is correct for any
[Online May 19, 2012]
thermodynamic system ? [2004]
(a) Statement 1 is true, Statement 2 is true, Statement 2 is
(a) The change in entropy can never be zero
not the correct explanation of Statement 1.
(b) Internal energy and entropy are state functions
(b) Statement 1 is true, Statement 2 is false.
(c) The internal energy changes in all processes
(c) Statement 1 is false, Statement 2 is true.
(d) The work done in an adiabatic process is always zero.
(d) Statement 1 is true, Statement 2 is true, Statement 2 is
76. “Heat cannot by itself flow from a body at lower temperature
the correct explanation of Statement 1.
to a body at higher temperature” is a statement or
71. A Carnot engine operating between temperatures T1 and T2
consequence of [2003]
1
has efficiency . When T2 is lowered by 62 K its efficiency (a) second law of thermodynamics
6
(b) conservation of momentum
1 (c) conservation of mass
increases to . Then T1 and T2 are, respectively: [2011]
3 (d) first law of thermodynamics
(a) 372 K and 310 K (b) 330 K and 268 K
77. A Carnot engine takes 3 × 106 cal of heat from a reservoir at
(c) 310 K and 248 K (d) 372 K and 310 K 627°C, and gives it to a sink at 27°C. The work done by the
72. A diatomic ideal gas is used in a Carnot engine as the engine is [2003]
working substance. If during the adiabatic expansion part
(a) 4.2 × 106 J (b) 8.4 × 106 J
of the cycle the volume of the gas increases from V to 32
(c) 16.8 × 106 J (d) zero
V, the efficiency of the engine is [2010]
78. Which statement is incorrect? [2002]
(a) 0.5 (b) 0.75 (c) 0.99 (d) 0.25
(a) All reversible cycles have same efficiency
73. A Carnot engine, having an efficiency of h = 1/10 as heat (b) Reversible cycle has more efficiency than an
engine, is used as a refrigerator. If the work done on the irreversible one
system is 10 J, the amount of energy absorbed from the (c) Carnot cycle is a reversible one
reservoir at lower temperature is [2007]
(d) Carnot cycle has the maximum efficiency in all cycles
(a) 100 J (b) 99 J (c) 90 J (d) 1 J
79. Even Carnot engine cannot give 100% efficiency because
74. The temperature-entropy diagram of a reversible engine we cannot [2002]
cycle is given in the figure. Its efficiency is [2005]
(a) prevent radiation
T (b) find ideal sources
(c) reach absolute zero temperature
2T0 (d) eliminate friction
T0

S
S0 2S0
Thermodynamics P-177

5. (b) For path iaf,


1. (a)
Q1 = 50 cal, W1 = 20 cal
By first law of thermodynamics, a f
DU = Q1 – W1 = 50 – 20 = 30 cal.
For path ibf
Q2 = 36 cal i b
W2 = ?
DU remains same for both paths ACB and ADB DUibf = Q2 – W2
DQACB = DWACB + DUACB Since, the change in internal energy does not depend on the
Þ 60 J = 30 J + DUACB path, therefore DUiaf = DUibf
Þ UACB = 30 J DUiaf = DUibf
\ DUADB = DUACB = 30 J Þ 30 = Q2 – W2
DQADB = DUADB + DWADB Þ W2 = 36 – 30 = 6 cal.
= 10 J + 30 J = 40 J 6. (b) Change in internal energy is independent of path taken
2. (d) Volume of water does not change, no work is done on by the process. It only depends on initial and final states i.e.,
or by the system (W = 0) DU1 = DU2
According to first law of thermodynamics 7. (b, c) First law is applicable to a cyclic process. Concept of
Q < ΧU ∗ W entropy is introduced by the second law of thermodynamics.
8. (b) Temperature change DT is same for all three processes
For Isochoric process Q < ΧU
A ® B; A ® C and A ® D
DU = mcdT = 2 × 4184 × 20 = 16.7 kJ.
3. (a) As we know, DU = nCv DT = same
DQ = D u + D w (Ist law of thermodynamics) E AB = E AC = E AD
Work done, W = P ´ DV
Þ DQ = D u + P D v
AB ® volume is increasing Þ WAB > 0
or 150 = Du + 100 (1 - 2 )
AD ® volume is decreasing Þ WAD < 0
= Du - 100
AC ® volume is constant Þ WAC = 0
\ Du = 150 + 100 = 250J 9. (c) In adiabatic process
Thus the internal energy of the gas increases by 250 J
PV g = constant
4. (a) Here Q = 0 and W = 0. Therefore from first law of
g
thermodynamics DU = Q + W = 0 æ mö æ mö
Internal energy of first vessle + Internal energy of second \ P ç ÷ = constant çèQ V = r ø÷
è rø
vessel = Internal energy of combined vessel
As mass is constant
n1Cv T1 + n2 Cv T2 = (n1 + n2 )Cv T
\ P µ rg
n1T1 + n2 T2 If Pi and Pf be the initial and final pressure of the gas and
\T =
n1 + n2
ri and r f be the initial and final density of the gas. Then
PV
1 1 and for second vessle g
For first vessel n1 = Pf ærf ö
RT1 = ç ÷ = (32)7 / 5
PV Pi è ri ø
n2 = 2 2
RT2 nP
Þ i = (25 )7 /5 = 27
PV
1 1 ´ T + P2V2 ´ T Pi
1 2
RT1 RT2 Þ n = 27 = 128.
\T=
PV
1 1 + P2V2 10. (d)
RT1 RT2 (I) Adiabatic process : No exchange of heat takes
place with surroundings.
1 1 + P2V2 )
T1T2 ( PV
= Þ DQ = 0
1 1 2 + P2V2T1
PV T
(II) Isothermal process : Temperature remains constant
P-178 Physics

f 14. (1818) For an adiabatic process,


\ DT = 0 Þ DU = nR DT Þ DU = 0 TVg–1 = constant
2
No change in internal energy [DU = 0]. \ T1V1g –1 = T2V2g –1
(III) Isochoric process volume remains constant 1.4 -1
æ ö
çV ÷
DV = 0 Þ W = ò P × dV = 0 Þ T2 = (300) ´ ç 1 ÷
Hence work done is zero. çç V1 ÷÷
è 16 ø
(IV) In isobaric process pressure remains constant.
Þ T2=300×(16) 0.4
W = P × DV ¹ 0 Ideal gas equation, PV = nRT
f f nRT
DU = nR DT = [ P DV ] ¹ 0 \ V=
2 2 P
Þ V = kT (since pressure is constant for isobaric
\ DQ = nC p DT ¹ 0 process)
11. (b) Bursting of helium balloon is irreversible and in this So, during isobaric process
V2 = kT2 ...(i)
process DQ = 0 , so adiabatic.
2V2 = kTf ...(ii)
12. (46)
Dividing (i) by (ii)
For adiabatic process, TV g -1 = constant 1 T2
=
or, T1V1g -1 = T2V2g -1 2 Tf

V1 7 Tf = 2T2 = 300 × 2 × (16)0.4 =1818 K


T1 = 20°C + 273 = 293 K , V2 = and g = 15. (a) From the corresponding V-T graph given in question,
10 5
g -1
Process xy ® Isobaric expansion,
æV ö Process yz ® Isochoric (Pressure decreases)
T1 (V1 ) g -1 = T2 ç 1 ÷
è 10 ø Process zx ® Isothermal compression
Therefore, corresponding PV graph is as shown in figure
2/5
æ 1ö
Þ 293 = T2 ç ÷ Þ T2 = 293(10) 2/ 5 ; 736 K
è 10 ø
DT = 736 - 293 = 443 K
During the process, change in internal energy
5
DU = NCV DT = 5 ´ ´ 8.3 ´ 443 ; 46 ´ 103 J = X kJ 16. (b) Given, V1 = 1 litre, P1 = 1 atm
2
V2 = 3 litre, g = 1.40,
\ X = 46 .
g g
Using, PVr = constant Þ PV
1 1 = P2V2
13. (c) For process 3 ® 1 volume is constant
\ Graph given in option (d) is wrong. 1.4
æ1ö 1
And process 1 ® 2 is adiabatic \ graph in option (1) is Þ P2 = P1 ´ ç ÷ = atm
è 3ø 4.6555
wrong
PV – P V
Q v = constant \ Work done, W = 1 1 2 2
P ­, T ­ g –1
For Process 2 ® 3 Pressure constant i.e., P = constant æ 1 ö
\ V¯T¯ ç1´1 – ´ 3 ÷1.01325 ´ 105 ´ 10 –3
è 4.6555 ø
Hence graph (c) is the correct V – T graph of given = = 90.1 J
0.4
P – V graph
Closest value of W = 90.5 J
V 17. (Bonus) We know that Relaxation time,
2
V
Tµ ...(i)
3 T
1
Equation of adiabatic process is
TVg–1 = constant

T 1
Þ T µ g-1
V
Thermodynamics P-179

g –1 24. (d) a ® Isobasic, b ® Isothermal, c ® Adiabatic,


Þ T µ V 1+ 2 using (i) d ® Isochoric
25. (b) Total work done by the gas during the cycle is equal
1+g to area of triangle ABC.
Þ T µV 2 1
\ DW = ´ 4 ´ 5 = 10 J
2
1+g
Tf 1+g 26. (b) Equation of adiabatic change is
æ 2V ö 2
TVg-1 = constant
Þ =ç ÷ = (2) 2
Ti è V ø 7
7
Put g = , we get: g - 1 = - 1
18. (b) DUac = – (DUca) = – (– 180) = 180 J 5 5
2
Q = 250 + 60 = 310 J \x =
Now Q = DU + W 5
27. (b) Work done,
or 310 = 180 + W
1
or W = 130 J W = PDV = nRDT = ´ 8.31´ 70 ; 291J
2
19. (c) As the process is isochoric so, 28. (a) Equation of the BC
67.2 3R 2P
Q = nc v DT = ´ ´ 20 = 90R = 90 ´ 8.31 ; 748 j. P = P0 - 0 (V - 2V0 )
22.4 2 V0
20. (a) At constant volume using PV = nRT
Work done (W) = nRDT 2P V 2
P0 V - 0 + 4P0 V
Heat given Q = CvDT + nRDT V0
Temperature, T =
W nRDT nR
1´ R
So, \ = = (Q n = 1 mole given)
Q Cv DT + nRDT C V + nR
21. (b) We have given, P é 2V 2 ù
T = 0 ê5V - ú
F ëê V0 ûú
é 1 æ V ö2 ù
P = P0 ê1 - ç 0 ÷ ú dT 4V 5
êë 2 è V ø úû =0Þ5- = 0 Þ V = V0
dV V0 4
When V1 = V0 P é 5V 2 25 2 ù 25 P0 V0
T = 0 ê5 ´ 0 - ´ V0 ú =
é 1 ù P0 Rë 4 V0 16 û 8 R
Þ P1 = P0 ê1 - ú =
ë 2û 2 æ pf ö
When V2 = 2V0 29. (d) Work done on gas = nRT ln çç ÷÷ = R(300) ln(2)
è p1 ø
é 1 æ 1 ö ù æ 7 P0 ö æ Pf ö
Þ P2 = P0 ê1 - ç ÷ ú = ç ÷ = 300 Rln2 ççQ = 2 given ÷÷
ë 2 è 4 øû è 8 ø
è pi ø
PV P V é PV ù 30. (c) The equation for the line is
DT = T2 - T1 = 1 1 - 2 2 êQ T = P
nR nR ë nR úû
3Po
æ 1 ö æ 1 ö æ P0V0 7 P0V0 ö
DT = ç 1 1 - P2V2 ) = ç
÷ ( PV ÷ç -
è nR ø è nR ø è 2 4 ÷ø c
2Po q
5P V 5P V Po
= 0 0 = 0 0 (Q n = 1) q
4 nR 4R Po
Vo
22. (c) Internal energy depends only on initial and final state V
So, DUA = DUB Vo 2Vo
Also DQ = DU + W - P0 -P0
As WA > WB Þ DQA > DQB P = V V + 3P [slope = V , c = 3P0]
0 0
23. (b) Suppose amount of water evaporated be M gram. PV0 + P0V = 3P0V0 ...(i)
Then (150 – M) gram water converted into ice. But pV = nRT
so, heat consumed in evoporation = Heat released in fusion nRT
M × Lv = (150 – M) × Ls \P= ...(ii)
V
M × 2.1 × 106 = (150 – M) × 3.36 × 105
nRT
Þ M – 20 g From (i) & (ii) V0 + P0V = 3P0V0
V
P-180 Physics

nRT 12. (d) In cyclic process, change in total internal energy is zero.
From (i) & (ii) V0 + P0V = 3P0V0 DUcyclic = 0
V
\ nRT V0 + P0V = 3P0V0V
2 ...(iii) 5R
dT DUBC = nCv DT = 1 ´ DT
For temperature to be maximum =0 2
dV Where, Cv = molar specific heat at constant volume.
Differentiating e.q. (iii) by ‘V’ we get For BC, DT = –200 K
dT \ DUBC = –500R
nRV0 + P0(2V) = 3P0V0 13. (c)
dV
dT 14. (c) Given : work done, W = 830 J
\ nRV0 = 3P0V0 – 2 P0V No. of moles of gas, m = 2
dV For diatomic gas g = 1.4
dT 3P0 V0 - 2P0 V Work done during an adiabatic change
= =0
dV nRV0 mR (T1 - T2 )
3V0 W=
3P g -1
V= \ P= 0 [From (i)]
2 2
9P0 V0 2 ´ 8.3( DT ) 2 ´ 8.3(DT )
Þ 830 = =
\ Tmax = [From (iii)] 1.4 - 1 0.4
4nR
8. (a) Efficiency of heat engine is given by 830 ´ 0.4
w C R R 2 Þ DT = = 20 K
h = = 1- V = = = 2 ´ 8.3
Q CP Cp 5R 5 15. (a)
2 16. (c) P-V indicator diagram for isobaric
(Q Cp – Cv = R) P
5
For monoatomic gas C P = R . slope
dP
=0
2 dV
1
9. (a) t =
æ N ö 3RT
2pd2 ç ÷ V
èVø M P-V indicator diagram for isochoric process
V P
t µ slope
T
dP
As, TVg–1 = K =¥
dV
So, t µ Vg + 1/2
g+ 1 V
Therefore, q = P-V indicator diagram for isothermal process
2
P
1æU ö slope
10. (a) As, P = ç ÷
3èV ø dP -P
= =
U dV V
But = KT 4
V
V
1 4 17. (b) Volume of the gas
So, P = KT
3 m
uRT 1 v= and
or = KT 4 [As PV = u RT] d
V 3 Using PV g = constant
4 3 3
p R T = constant P' V æ d'ö
g
3 = =ç ÷
1 P V' è d ø
Therefore, Tµ or 128 = (32)g
R
11. (b) In VT graph 7
\ g = = 1.4
ab-process : Isobaric, temperature increases. 5
bc process : Adiabatic, pressure decreases. 18. (a) The efficiency
cd process : Isobaric, volume decreases.
output work
da process : Adiabatic, pressure increases. h=
The above processes correctly represented in P-V diagram (b). heat given to the system
Thermodynamics P-181

3 3 3 46. (d)
= n RDT = V0 DP = P0V0 P, T
2 2 2 Vacuum
n n
Wi = ( P0V0 ) + (2 P0V0 ) + 2 P0V0
2 2
It is the free expansion
Heat given in going B to C = nCpDT
\ So, T remains constant
æ5 ö 5 Þ PV
= n ç R ÷ DT = (2 P0 )DV 1 1 = P2V2
è2 ø 2
V
= 5P0V0 Þ P = P2 (V )
2
and W0 = area under PV diagram P0V0
æ Pö
W PV 2 P2 = ç ÷
h= = 0 0 = è 2ø
Q 13 47. (b) The process A ® B is isobaric.
P0V0 13
2 \ work done WAB = nR(T2 – T1)
Efficiency in % = 2R (500 - 300) = 400 R
2 200 48. (a) The process D to A is isothermal as temperature is
h= ´ 100 = ; 15.4%
13 13 constant.
42. (d) For isothermal process : P
Work done, WDA = 2.303nRT log10 D
PV = Pi .2V PA
P = 2Pi ...(i) = 2.303 ´ 2 R ´ 300
For adiabatic process
PVg = Pa (2V)g 1 ´ 105
log10 – 414R.
(Q for monatomic gas g= 5 3 ) 2 ´ 105
Therefore, work done on the gas is +414 R.
5 5 49. (a) The net work in the cycle ABCDA is
or, 2Pi V 3 = Pa (2V) 3 [From (i)]
W = W AB + WBC + WCD + WDA
Pa 2 PB
Þ = 5 = 400R + 2.303nRT log + (-400R) - 414R
Pi PC
2 3-2
Pa 2 ´ 105
Þ =2 3 = 2.303 ´ 2R ´ 500log - 414 R
Pi 1 ´ 105
43. (d) Given P = aV = 693.2 R – 414 R = 279.2 R
mV 50. (a) Work done in adiabatic compression is given by
Work done, w =
ò PdV W=
nRDT
1- g
V
mV 1000 ´ 8.3 ´ 7
aV 2 Þ -146000 =
= ò aVdV =
2
(m 2 - 1) .
58.1
1- g
58.1
V or 1 - g = - Þ g = 1+ = 1.4
44. (b) Work done during the process A ® B 146 146
Hence the gas is diatomic.
= Area of trapezium (= area bounded by indicator diagram
51. (c) Work is not a state function. The remaining three
with V-axis)
parameters are state function.
1 3
=
2
( 2 P0 + P0 ) ( 2V0 - V0 ) = P0V0
2
52. (19) P
B C
Ideal gas eqn : PV = nRT 3P0

PV 3P0V0
Þ T= =
nR 2nR
45. (b) In an adiabatic process, dH = 0
P0 D
And according to first law of thermodynamics A
dH = dU + W
\ W = – dU V0
V
2V0
P-182 Physics

From the figure,


Q1 – Q2 w
Work, W = 2 P0V0 Also, =
Q1 Q1
Heat given, Qin = WAB + WBC = n × CV DTAB + nCP DTBC
1 w
3R n5 R Þ =
=n (TB - TA ) + (TC - TB ) 10 Q1
2 2
Þ Q1 = w × 10 = 100 J
æ 3R 5R ö So, Q1 – Q2 = w
çèQ Cv = and CP = ÷
2 2ø Þ Q2 = Q1– w
3 5 Þ 100 – 10 = Q2 = 90 J
= ( PBVB - PAV A ) + ( PCVC - PBVB )
2 2
56. (600.00) Given; T1 = 900 K, T2 = 300K, W = 1200 J
3 5
= ´ [3P0V0 - P0V0 ] + [6 P0V0 - 3P0V0 ]
2 2 T2 W
Using, 1 – =
15 21 T1 Q1
= 3 P0V0 + P0V0 = P0V0
2 2 300 1200
Þ 1– =
W 2 P0V0 4 900 Q1
Efficiency, h = = =
Qin 21 21 2 1200
P0V0 Þ = Þ Q1 = 1800
400 2 3 Q1
h% = » 19.
21 Therefore heat energy delivered by the engine to the low
53. (8791) temperature reservoir, Q2 = Q1 – W = 1800 – 1200 =
Given, 600.00 J
Heat absorbed, Q2 = mL = 80 × 100 = 8000 Cal 57. (b) Let QH = Heat taken by first engine
Temperature of ice, T2 = 273 K QL = Heat rejected by first engine
Temperature of surrounding, Q2 = Heat rejected by second engine
T1 = 273 + 27 = 300 K Work done by 1st engine = work done by 2nd engine
W = QH – QL = QL – Q2 Þ 2QL = QH + Q2
w Q1 - Q2 T1 - T2 300 - 273
Efficiency = = = = qH q2
Q2 Q2 T2 273 2= +
qL qL
Q1 - 8000 27 Let T be the temperature of cold reservoir of first engine.
Þ = Þ Q1 = 8791 Cal
8000 273 Then in carnot engine.
Work done W QH T1 Q T
54. (c) Efficiency, h = = = and L =
Heat absorbed SQ QL T Q2 T2
T1 T2
Q1 + Q2 + Q3 + Q4 Þ 2= + using (i)
= = 0.5 T T
Q1 + Q3
T1 + T2
Here, Q1 = 1915 J, Q2 = – 40 J and Q3 = 125 J Þ 2T = T1 + T2 Þ T=
2
1915 - 40 + 125 + Q4 T2
\ = 0.5
1915 + 125 58. (b) Using, n = 1 -
T1
Þ 1915 - 40 + 125 + Q4 = 1020 T2
1
n= = 1- T
Þ Q4 = 1020 - 2000 6 1

Þ Q4 = -Q = -980 J T T2 - 62
and 3 = 1 - T
Þ Q = 980 J 1
On solving, we get
55. (d) For carnot refrigerator T1 = 99°C and T2 = 37°C
Q1 – Q2 59. (b) According to question, h1 = h2 = h3
Efficiency =
Q1 T2 T T
\1– = 1– 3 =1– 4
Where, T1 T2 T3
Q1 = heat lost from sorrounding [Q Three engines are equally efficient]
Q2 = heat absorbed from reservoir at low temperature.
Thermodynamics P-183

T2 T3 T4 W 2
Þ = = Thermal efficiency of engine (h) = = = 0.15
T1 T2 T3 Q given 13
64. (d) DH = mL = 5 × 336 × 103 = Qsink
Þ T2 = T1T3 ...(i) Qsink T
< sink
T3 = T2 T4 ...(ii) Qsource Tsource
From (i) and (ii) Tsource
[ Qsource < ´ Qsink
1 Tsink
T2 = (T12 T4 ) 3
Energy consumed by freezer
1
2
T3 = (T1 T4 ) 3 æ Tsource ö÷
ç ,1÷÷
[ w output < Qsource , Qsink < Qsink çç
T1 – T2 w A çè Tsink ø÷
60. (d) hA = =
Tl Q1
Given: Tsource < 27°C ∗ 273 < 300K,
T –T W
and, hB = 2 3 = B Tsink < 0°C ∗ 273 < 273 k
T2 Q2
3 æ 300 ö
Woutput = 5´336´10 ççç ,1÷÷ < 1.67 ´105 J
According to question,
WA = WB è 273 ø÷
Q T T - T3 T1 65. (d) The entropy change of the body in the two cases is
\ 1= 1´ 2 =
Q 2 T2 T1 - T2 T2 same as entropy is a state function.
66. (b) Given : Q1 = 1000 J
Tl + T3
\T2 = Q2 = 600 J
2 T1 = 127°C = 400 K
600 + 400 T2 = ?
=
2 h=?
= 500K Efficiency of carnot engine,
61. (a) Given: Temperature of cold body, T 2 = 250 K W
temperature of hot body; T1 = 300 K h= ´100%
Q1
Heat received, Q2 = 500 cal work done, W = ?
T2 W 250 W Q2 - Q1
Efficiency = 1 – = Þ 1– = or, h = ´100%
T1 Q2 + W 300 Q2 + W Q1

Q2 500 ´ 4.2 1000 - 600


W= = J = 420 J or, h = ´ 100%
5 5 1000
T1 - T2 h = 40%
62. (d) Efficiency of engine A, nA =
T1 Q 2 T2
Now, for carnot cycle Q = T
T2 - T3 T +T 1 1
and nB = ; T2 = 1 3 = 350 K
T2 2 600 T
= 2
600 - 350 1000 400
nA 600 7 600 ´ 400
or = = T2 =
nB 350 - 100 12 1000
350 = 240 K
63. (b) Work-done (W) = P0V0 = 240 – 273
According to principle of calorimetry
\ T2 = -33°C
Heat given = QAB = QBC
= nCVdTAB + nCPdTBC 67. (b) Heat is extracted from the source in path DA and AB is
3 5 3 æ P0V0 ö 5 æ 2 P0V0 ö
= (nRTB - nRTA ) + (nRTC - nRTB ) DQ = R + R
2 2 2 çè R ÷ø 2 çè R ÷ø
3 5
= (2P0 V0 - P0 V0 ) + (4P0 V0 - 2 P0V) 3 5 æ 13ö
2 2 Þ P0V0 + 2 P0V0 = ç ÷ P0V0
13 2 2 è 2ø
= P0 V0
2
P-184 Physics

68. (c) The efficiency of the carnot’s heat engine is given as


For adiabatic expansion T1V1g-1 = T2V2g-1
æ T ö
h = ç 1 - 2 ÷ ´100 Þ T1V g - 1 = T2 (32V )g - 1
è T1 ø
T1
When efficiency is 40%, Þ = (32)g–1
T1 = 500 K; h = 40 T2
æ T2 ö 7
40 = ç 1 - ÷ ´ 100 For diatomic gas, g =
è 500 ø 5
40 T 2
Þ = 1- 2 \ g -1 =
100 500 5
T2 60
Þ = Þ T2 = 300 K T
2
500 100 \ 1 = (32) 5 Þ T1 = 4T2
When efficiency is 60%, then T2
60 æ 300 ö 300 40 T2
= ç1 - ÷Þ = Now, efficiency = 1 -
100 è T2 ø T2 100 T1
100 ´ 300 T2 1 3
Þ T2 = Þ T2 = 750 K = 1- = 1 - = = 0.75.
40 4T2 4 4
69. (d) In a refrigerator, the heat dissipated in the
atmosphere is more than that taken from the cooling 73. (c) The efficiency (h) of a Carnot engine and the
chamber, therefore the room is heated. If the door of coefficient of performance (b) of a refrigerator are related
a refrigerator is kept open. as
70. (d) According to Carnot's theorem - no heat engine working 1- h
between two given temperatures of source and sink can be b=
h
more efficient than a perfectly reversible engine i.e. Carnot
engine working between the same two temperatures. Q2
Also, b =
T2 W
Efficiency of Carnot's engine, n = 1 –
T1 1 – n Q2
where, T1 = temperature of source \b= =
n W
T2 = temperature of sink
71. (d) Efficiency of engine 1
1-
10 Q
T 1 \ b= = 2.
h1 = 1 - 2 = æ 1ö W
T1 6 ç ÷
è 10 ø
T2 5
Þ = ....(i) is independent of path taken by the process.
T1 6
Q2
When T2 is lowered by 62K, then Þ 9=
T2 - 62 10
Again, h2 = 1 - Þ Q2 = 90 J.
T1
T 62 1 1
=1– 2 + = 74. (d) Q1 = area under BC = T0 S0 + T0 S0
T1 T1 3 ....(ii) 2
Solving (i) and (ii), we get, Q2 = area under AC = T0(2S0 – S0) = T0S0
and Q3 = 0
5
T1 = 372 K and T2 = × 372 = 310 K W Q1 - Q2
6
Efficiency, h = =
Q1 Q1
72. (b) P T
T1
(V, T1)

2T0 B
(32 V, T2) Q1
Q3
T2 T0 C
A Q2
V
S0 2S0
Thermodynamics P-185

Q2 TS 1 W 2 2 2
= 1- = 1- 0 0 = \ = Þ W = ´ Q = ´ 3 ´ 106
Q1 3 3 Q 3 3 3
T S
2 0 0 = 2 × 106 cal
75. (b) Internal energy and entropy are state function, they are = 2 × 106 × 4.2 J = 8.4 × 106 J
independent of path taken. 78. (a) All reversible engines have same efficiencies if they
76. (a) This is a consequence of second law of are working for the same temperature of source and sink.
thermodynamics If the temperatures are different, the efficiency is
77. (b) Here, T1 = 627 + 273 = 900 K different.
T2 = 27 + 273 = 300 K 79. (c) In Carnot’s cycle we assume frictionless piston,
T2 absolute insulation and ideal source and sink (reservoirs).
Efficiency, h = 1 -
T1 T2
The efficiency of carnot’s cycle h = 1 -
300 1 2 T1
= 1- = 1- =
900 3 3 The efficiency of carnot engine will be 100% when its
sink (T2) is at 0 K.
W The temperature of 0 K (absolute zero) cannot be realised
But h =
Q in practice so, efficiency is never 100%.
12
P-186 Physics

Kinetic Theory
Kinetic Theory of an Ideal nRT é 1 1 ù nRT é l1 - l2 ù
TOPIC 1 (c) ê + ú (d) ê ú
Gas and Gas Laws g ë l2 l1 û g ë l1 l2 û

1. Initially a gas of diatomic molecules is contained in a 5. The temperature of an open room of volume 30 m3 increases
cylinder of volume V1 at a pressure P1 and temperature from 17°C to 27°C due to sunshine. The atmospheric pressure
250 K. Assuming that 25% of the molecules get in the room remains 1 × 105 Pa. If ni and nf are the number of
dissociated causing a change in number of moles. The molecules in the room before and after heating, then nf – ni
pressure of the resulting gas at temperature 2000 K, will be : [2017]
when contained in a volume 2V1 is given by P2. The ratio (a) 2.5 × 1025 (b) –2.5 × 1025
P2/P1 is ______. [NA Sep. 06, 2020 (I)] (c) –1.61 × 1023 (d) 1.38 × 1023
6. For the P-V diagram given for an ideal gas,
2. The change in the magnitude of the volume of an ideal gas
when a small additional pressure DP is applied at a constant 1

temperature, is the same as the change when the P


Constant
P=
temperature is reduced by a small quantity DT at constant V
pressure. The initial temperature and pressure of the gas
were 300 K and 2 atm. respectively. If | DT |= C | DP | ,
then value of C in (K/atm.) is __________. 2
[NA Sep. 04, 2020 (II)] V
3. The number density of molecules of a gas depends on out of the following which one correctly represents the
their distance r from the origin as , n(r) = n0e–ar4. Then T-P diagram ? [Online April 9, 2017]
the total number of molecules is proportional to : 2
[12 April 2019 II] 2
T
(a) n0a –3/4 (b) n0 a1/2 (a) T (b)
(c) n0 a1/4 (d) n0 a–3 1 1
4. A vertical closed cylinder is separated into two parts by a P
P
frictionless piston of mass m and of negligible thickness.
The piston is free to move along the length of the cylinder. T T
The length of the cylinder above the piston is l1, and that 2 1
1 2
below the piston is l2, such that l1 > l2. Each part of the
cylinder contains n moles of an ideal gas at equal temperature (c) (d)
T. If the piston is stationary, its mass, m, will be given by:
P P
(R is universal gas constant and g is the acceleration due to
7. Chamber I Chamber II
gravity) [12 Jan. 2019 II]
ideal real
RT é l1 - 3l2 ù RT é 2l1 + l2 ù gas gas
(a) ê ú (b) g ê l I ú
ng ë l1 I 2 û ë 1 2 û 1 2 3 4
Kinetic Theory P-187

There are two identical chambers, completely thermally (a) 104 N/m2 (b) 108 N/m2
insulated from surroundings. Both chambers have a 3
(c) 10 N/m 2 (d) 1016 N/m2
partition wall dividing the chambers in two compartments. 13. The temperature, at which the root mean square velocity
Compartment 1 is filled with an ideal gas and of hydrogen molecules equals their escape velocity from
Compartment 3 is filled with a real gas. Compartments 2 the earth, is closest to : [8 April 2019 II]
and 4 are vacuum. A small hole (orifice) is made in the [Boltzmann Constant kB = 1.38 × 10 J/K–23
partition walls and the gases are allowed to expand in
vacuum. Avogadro Number NA = 6.02 × 1026 /kg
Statement-1: No change in the temperature of the gas Radius of Earth : 6.4 × 106 m
takes place when ideal gas expands in vacuum. However, Gravitational acceleration on Earth = 10 ms–2]
the temperature of real gas goes down (cooling) when it (a) 800 K (b) 3 × 105 K
expands in vacuum. 4
Statement-2: The internal energy of an ideal gas is only (c) 10 K (d) 650 K
kinetic. The internal energy of a real gas is kinetic as 14. A mixture of 2 moles of helium gas (atomic mass = 4u), and
well as potential. [Online April 9, 2013] 1 mole of argon gas (atomic mass = 40u) is kept at 300 K in
(a) Statement-1 is false and Statement-2 is true. a container. The ratio of their rms speeds
(b) Statement-1 and Statement-2 both are true.
Statement-2 is the correct explanation of Statement-1. é Vrms ( helium ) ù
ê ú is close to : [9 Jan. 2019 I]
(c) Statement-1 is true and Statement-2 is false. ë Vrms ( argon ) û
(d) Statement-1 and Statement-2 both are true.
(a) 3.16 (b) 0.32
Statement-2 is not correct explanation of Statement-1. (c) 0.45 (d) 2.24
8. Cooking gas containers are kept in a lorry moving with 15. N moles of a diatomic gas in a cylinder are at a temperature
uniform speed. The temperature of the gas molecules T. Heat is supplied to the cylinder such that the tempera-
inside will [2002] ture remains constant but n moles of the diatomic gas get
(a) increase converted into monoatomic gas. What is the change in
(b) decrease the total kinetic energy of the gas ?
(c) remain same [Online April 9, 2017]
(d) decrease for some, while increase for others
1
(a) nRT (b) 0
2
Speed of Gas, Pressure
TOPIC 2 3 5
and Kinetic Energy (c) nRT (d) nRT
2 2
16. In an ideal gas at temperature T, the average force that a
9. Number of molecules in a volume of 4 cm 3 of a perfect molecule applies on the walls of a closed container
monoatomic gas at some temperature T and at a pressure depends on T as Tq. A good estimate for q is:
of 2 cm of mercury is close to? (Given, mean kinetic en- [Online April 10, 2015]
ergy of a molecule (at T) is 4 × 10–14 erg, g = 980 cm/s2,
density of mercury = 13.6 g/cm3) [Sep. 05, 2020 (I)] 1
(a) (b) 2
(a) 4.0 × 1018 (b) 4.0 × 1016 2
16 1
(c) 5.8 × 10 (d) 5.8 × 1018 (c) 1 (d)
10. Nitrogen gas is at 300°C temperature. The temperature 4
17. A gas molecule of mass M at the surface of the Earth has
(in K) at which the rms speed of a H2 molecule would be
kinetic energy equivalent to 0°C. If it were to go up
equal to the rms speed of a nitrogen molecule, is
straight without colliding with any other molecules, how
_____________. (Molar mass of N2 gas 28 g);
high it would rise? Assume that the height attained is much
[NA Sep. 05, 2020 (II)]
less than radius of the earth. (kB is Boltzmann constant).
11. For a given gas at 1 atm pressure, rms speed of the [Online April 19, 2014]
molecules is 200 m/s at 127°C. At 2 atm pressure and at
273k B
227°C, the rms speed of the molecules will be: (a) 0 (b)
[9 April 2019 I] 2Mg
(a) 100 m/s (b) 80 5 m/s 546k B 819k B
(c) (d)
(c) 100 5 m/s (d) 80 m/s 3Mg 2Mg
12. If 1022 gas molecules each of mass 10–26 kg collide with a 18. At room temperature a diatomic gas is found to have an
surface (perpendicular to it) elastically per second over r.m.s. speed of 1930 ms–1. The gas is:
an area 1 m2 with a speed 104 m/s, the pressure exerted [Online April 12, 2014]
by the gas molecules will be of the order of : (a) H2 (b) Cl2
[8 April 2019 I] (c) O2 (d) F2
P-188 Physics

19. In the isothermal expansion of 10g of gas from volume The total internal energy, U of a mole of this gas, and the
V to 2V the work done by the gas is 575J. What is the æ Cp ö
root mean square speed of the molecules of the gas at value of g ç = ÷ are given, respectively, by:
è Cv ø
that temperature? [Online April 25, 2013] [Sep. 06, 2020 (I)]
(a) 398m/s (b) 520m/s
(c) 499m/s (d) 532m/s 5 6 7
20. A perfect gas at 27°C is heated at constant pressure so as (a) U = RT and g = (b) U = 5RT and g =
2 5 5
to double its volume. The final temperature of the gas will
be, close to [Online May 7, 2012] 5 7 6
(c) U = RT and g = (d) U = 5RT and g =
(a) 327°C (b) 200°C 2 5 5
(c) 54°C (d) 300°C 27. In a dilute gas at pressure P and temperature T, the mean
21. A thermally insulated vessel contains an ideal gas of time between successive collisions of a molecule varies
molecular mass M and ratio of specific heats g. It is
moving with speed v and it's suddenly brought to rest. with T is : [Sep. 06, 2020 (II)]
Assuming no heat is lost to the surroundings, its 1
temperature increases by: [2011] (a) T (b)
T
( g - 1) Mv 2 K gM 2v
(a) (b) K
2 gR 2R 1
(c) (d) T
( g - 1) ( g - 1) 2 T
(c) Mv 2 K (d) 2( g + 1) R Mv K
2R 28. Match the Cp/Cv ratio for ideal gases with different type
22. Three perfect gases at absolute temperatures T1, T2 and T3 of molecules : [Sep. 04, 2020 (I)]
are mixed. The masses of molecules are m1, m2 and m3 and Column-I Column-II
the number of molecules are n1, n2 and n3 respectively. Molecule Type Cp/Cv
Assuming no loss of energy, the final temperature of the (A) Monatomic (I) 7/5
mixture is : [2011] (B) Diatomic rigid molecules (II) 9/7
n1T1 + n2T2 + n3T3 n1T12 + n2T22 + n3T32 (C) Diatomic non-rigid molecules(III) 4/3
(a) (b)
n1 + n2 + n3 n1T1 + n2T2 + n3T3 (D) Triatomic rigid molecules (IV) 5/3
n12T12 + n22T22 + n32T32 (T1 + T2 + T3 ) (a) (A)-(IV), (B)-(II), (C)-(I), (D)-(III)
(c) (d) (b) (A)-(III), (B)-(IV), (C)-(II), (D)-(I)
n1T1 + n2T2 + n3T3 3
23. One kg of a diatomic gas is at a pressure of (c) (A)-(IV), (B)-(I), (C)-(II), (D)-(III)
8 × 104N/m2. The density of the gas is 4kg/m3. What is (d) (A)-(II), (B)-(III), (C)-(I), (D)-(IV)
the energy of the gas due to its thermal motion?[2009] 29. A closed vessel contains 0.1 mole of a monatomic ideal
(a) 5 × 104 J (b) 6 × 104 J gas at 200 K. If 0.05 mole of the same gas at 400 K is
4
(c) 7 × 10 J (d) 3 × 104 J added to it, the final equilibrium temperature (in K) of
24. The speed of sound in oxygen (O2) at a certain temperature the gas in the vessel will be close to _________.
is 460 ms–1. The speed of sound in helium (He) at the same [NA Sep. 04, 2020 (I)]
temperature will be (assume both gases to be ideal)
[2008] 30.
(a) 1421 ms –1 (b) 500 ms –1

(c) 650 ms–1 (d) 330 ms–1


25. At what temperature is the r.m.s velocity of a hydrogen
molecule equal to that of an oxygen molecule at 47°C? Consider a gas of triatomic molecules. The molecules are
[2002] assumed to be triangular and made of massless rigid rods
whose vertices are occupied by atoms. The internal energy
(a) 80 K (b) –73 K
(c) 3 K (d) 20 K of a mole of the gas at temperature T is :
[Sep. 03, 2020 (I)]
Degree of Freedom, Specific 5 3
TOPIC 3 Heat Capacity, and Mean (a) RT (b) RT
2 2
Free Path
9
26. Molecules of an ideal gas are known to have three (c) RT (d) 3RT
2
translational degrees of freedom and two rotational 31. To raise the temperature of a certain mass of gas by 50°C
degrees of freedom. The gas is maintained at a at a constant pressure, 160 calories of heat is required.
temperature of T. When the same mass of gas is cooled by 100°C at constant
Kinetic Theory P-189

volume, 240 calories of heat is released. How many 37. Consider a mixture of n moles of helium gas and 2n
degrees of freedom does each molecule of this gas have moles of oxygen gas (molecules taken to be rigid) as an
(assume gas to be ideal)? [Sep. 03, 2020 (II)] ideal gas. Its CP/CV value will be: [8 Jan. 2020 II]
(a) 5 (b) 6 (a) 19/13 (b) 67/45
(c) 3 (d) 7 (c) 40/27 (d) 23/15
32. A gas mixture consists of 3 moles of oxygen and 5 moles of Cp 5
argon at temperature T. Assuming the gases to be ideal 38. Two moles of an ideal gas with C = are mixed with 3
V 3
and the oxygen bond to be rigid, the total internal energy
(in units of RT) of the mixture is : [Sep. 02, 2020 (I)] Cp 4
(a) 15 (b) 13 moles of another ideal gas with C = . The value of
V 3
(c) 20 (d) 11 Cp
33. An ideal gas in a closed container is slowly heated. As its for the mixture is: [7 Jan. 2020 I]
temperature increases, which of the following statements CV
are true? [Sep. 02, 2020 (II)] (a) 1. 45 (b) 1.50
(1) The mean free path of the molecules decreases (c) 1.47 (d) 1.42
(2) The mean collision time between the molecules 39. Two moles of helium gas is mixed with three moles of
decreases hydrogen molecules (taken to be rigid). What is the molar
(3) The mean free path remains unchanged specific heat of mixture at constant volume?
(4) The mean collision time remains unchanged (R = 8.3 J/mol K) [12 April 2019 I]
(a) (2) and (3) (b) (1) and (2) (a) 19.7 J/mol L (b) 15.7 J/mol K
(c) (3) and (4) (d) (1) and (4) (c) 17.4 J/mol K (d) 21.6 J/mol K
34. Consider two ideal diatomic gases A and B at some 40. A diatomic gas with rigid molecules does 10 J of work
temperature T. Molecules of the gas A are rigid, and have when expanded at constant pressure. What would be the
a mass m. Molecules of the gas B have an additional heat energy absorbed by the gas, in this process ?
m [12 April 2019 II]
vibrational mode, and have a mass . The ratio of the (a) 25 J (b) 35 J
4
(c) 30 J (d) 40 J
specific heats (CA B
V and CV ) of gas A and B, respectively 41. A 25×10 – 3 m3 volume cylinder is filled with 1 mol of O2
is: [9 Jan 2020 I]
gas at room temperature (300 K) . The molecular diameter
(a) 7 : 9 (b) 5 : 9
of O2, and its root mean square speed, are found to be 0.3
(c) 3 : 5 (d) 5 : 7
nm and 200 m/s, respectively. What is the average collision
35. Two gases-argon (atomic radius 0.07 nm, atomic weight
rate (per second) for an O2 molecule?
40) and xenon (atomic radius 0.1 nm, atomic weight 140)
[10 April 2019 I]
have the same number density and are at the same
(a) ~1012 (b) ~1011
temperature. The ratio of their respective mean free
(c) ~1010 (d) ~1013
times is closest to: [9 Jan 2020 II]
42. When heat Q is supplied to a diatomic gas of rigid
(a) 3.67 (b) 1.83
molecules, at constant volume its temperature increases
(c) 2.3 (d) 4.67
by DT. The heat required to produce the same change in
36. The plot that depicts the behavior of the mean free time t
temperature, at a constant pressure is :
(time between two successive collisions) for the molecules
[10 April 2019 II]
of an ideal gas, as a function of temperature (T),
qualitatively, is: (Graphs are schematic and not drawn to 2 5
(a) Q (b) Q
scale) [8 Jan. 2020 I] 3 3
7 3
(c) Q (d) Q
5 2
t t 43. An HCl molecule has rotational, translational and
(a) (b) vibrational motions. If the rms velocity of HCl molecules
in its gaseous phase is v , m is its mass and k B is
1
T T Boltzmann constant, then its temperature will be:
[9 April 2019 I]
mv 2 mv 2
t (a) (b)
t 6kB 3k B
(c) (d)
mv 2 mv 2
1 (c) (d)
T T 7k B 5k B
P-190 Physics

44. The specific heats, C p and Cv of a gas of diatomic 51. Two moles of an ideal monoatomic gas occupies a volume
molecules, A, are given (in units of J mol–1 k–1) by 29 and V at 27°C. The gas expands adiabatically to a volume 2 V.
22, respectively. Another gas of diatomic molecules, B, Calculate (1) the final temperature of the gas and (2) change
has the corresponding values 30 and 21. If they are treated in its internal energy. [2018]
as ideal gases, then: [9 April 2019 II] (a) (1) 189 K (2) 2.7 kJ
(a) A is rigid but B has a vibrational mode. (b) (1) 195 K (2) –2.7 kJ
(b) A has a vibrational mode but B has none. (c) (1) 189 K (2) –2.7 kJ
(c) A has one vibrational mode and B has two. (d) (1) 195 K (2) 2.7 kJ
(d) Both A and B have a vibrational mode each. 52. Two moles of helium are mixed with n with moles of
45. An ideal gas occupies a volume of 2 m 3 at a pressure of 3 C 3
× 106 Pa. The energy of the gas: [12 Jan. 2019 I] hydrogen. If P = for the mixture, then the value of n
CV 2
(a) 9 × 106 J (b) 6 × 104 J is [Online April 16, 2018]
(c) 108 J (d) 3 × 102 J (a) 3/2 (b) 2
46. An ideal gas is enclosed in a cylinder at pressure of 2 atm (c) 1 (d) 3
and temperature, 300 K. The mean time between two 53. Cp and Cv are specific heats at constant pressure and
successive collisions is 6 × 10–8 s. If the pressure is doubled constant volume respectively. It is observed that
and temperature is increased to 500 K, the mean time Cp – Cv = a for hydrogen gas
between two successive collisions wiil be close to: Cp – Cv = b for nitrogen gas
[12 Jan. 2019 II] The correct relation between a and b is : [2017]
–7
(a) 2 × 10 s (b) 4 × 10 s –8 (a) a = 14 b (b) a = 28 b
(c) 0.5 × 10–8 s (d) 3 × 10–6 s 1
(c) a = b (d) a = b
14
47. A gas mixture consists of 3 moles of oxygen and 5 moles 54. An ideal gas has molecules with 5 degrees of freedom.
of argon at temperature T. Considering only translational The ratio of specific heats at constant pressure (C p) and
and rotational modes, the total internal energy of the at constant volume (Cv) is : [Online April 8, 2017]
system is : [11 Jan. 2019 I] 7
(a) 6 (b)
(a) 15 RT (b) 12 RT 2
(c) 4 RT (d) 20 RT 5 7
(c) (d)
48. In a process, temperature and volume of one mole of an 2 5
ideal monoatomic gas are varied according to the relation 55. An ideal gas undergoes a quasi static, reversible process
VT = K, where K is a constant. In this process the in which its molar heat capacity C remains constant. If
temperature of the gas is increased by DT. The amount of during this process the relation of pressure P and volume
heat absorbed by gas is (R is gas constant) : V is given by PVn = constant, then n is given by (Here CP
[11 Jan. 2019 II] and CV are molar specific heat at constant pressure and
constant volume, respectively) : [2016]
1 1
(a) RΔT (b) KRΔT CP - C C - CV
2 2 (a) n = (b) n =
2K C - CV C - CP
3
(c) RΔT (d) ΔT CP C – CP
2 3 (c) n = (d) n =
49. Two kg of a monoatomic gas is at a pressure of 4 × 104 CV C – CV
N/m2. The density of the gas is 8 kg/m3. What is the 56. Using equipartition of energy, the specific heat
order of energy of the gas due to its thermal motion? (in J kg–1 K–1) of aluminium at room temperature can
be estimated to be (atomic weight of aluminium = 27)
[10 Jan 2019 II] [Online April 11, 2015]
3 5
(a) 10 J (b) 10 J (a) 410 (b) 25
(c) 104 J (d) 106 J (c) 1850 (d) 925
50. A 15 g mass of nitrogen gas is enclosed in a vessel at 57. Modern vacuum pumps can evacuate a vessel down to a
a temperature 27°C. Amount of heat transferred to the pressure of 4.0 × 10–15 atm. at room temperature (300
K). Taking R = 8.0 JK–1 mole–1, 1 atm = 105 Pa and
gas, so that rms velocity of molecules is doubled, is NAvogadro = 6 × 1023 mole–1, the mean distance between
about: [Take R = 8.3 J/K mole] [9 Jan. 2019 II] molecules of gas in an evacuated vessel will be of the
(a) 0.9 kJ (b) 6 kJ order of: [Online April 9, 2014]
(c) 10 kJ (d) 14 kJ (a) 0.2 mm (b) 0.2 mm
(c) 0.2 cm (d) 0.2 nm
Kinetic Theory P-191

58. Figure shows the variation in temperature (DT) with the 60. If CP and CV denote the specific heats of nitrogen per unit
amount of heat supplied (Q) in an isobaric process mass at constant pressure and constant volume
corresponding to a monoatomic (M), diatomic (D) and a respectively, then [2007]
polyatomic (P) gas. The initial state of all the gases are the (a) CP – CV = 28R (b) CP – CV = R/28
same and the scales for the two axes coincide. Ignoring (c) CP – CV = R/14 (d) CP – CV = R
vibrational degrees of freedom, the lines a, b and c 61. A gaseous mixture consists of 16 g of helium and 16 g of
respectively correspond to : [Online April 9, 2013]
Cp
a oxygen. The ratio of the mixture is [2005]
Cv
Q b (a) 1.62 (b) 1.59
(c) 1.54 (d) 1.4
c
62. One mole of ideal monatomic gas (g = 5/3) is mixed with
one mole of diatomic gas (g = 7/5). What is g for the
DT mixture? g Denotes the ratio of specific heat at constant
(a) P, M and D (b) M, D and P pressure, to that at constant volume [2004]
(a) 35/23 (b) 23/15
(c) P, D and M (d) D, M and P
(c) 3/2 (d) 4/3
Cp 63. During an adiabatic process, the pressure of a gas is found
59. A given ideal gas with g = = 1.5 at a temperature T. If
Cv to be proportional to the cube of its absolute temperature.
the gas is compressed adiabatically to one-fourth of its The ratio CP/CV for the gas is [2003]
initial volume, the final temperature will be 4
(a) (b) 2
[Online May 12, 2012] 3
(a) 2 2T (b) 4 T 5 3
(c) (d)
(c) 2 T (d) 8 T 3 2
P-192 Physics

1. (5) Using ideal gas equation, PV = nRT æ 1 1ö


Þ nRT ç l – l ÷ = mg
Þ PV
1 1 = nR ´ 250 [Q T1 = 250 K] ...(i) è 2 1ø

5n nRT æ l1 – l 2 ö
P2 (2V1 = R ´ 2000 [Q T2 = 2000 K] ...(ii) \ m = g ç l ×l ÷
4 è 1 2 ø
Dividing eq. (i) by (ii), 5. (b) Given: Temperature Ti = 17 + 273 = 290 K
P1 4 ´ 250 P 1 Temperature Tf = 27 + 273 = 300 K
= Þ 1 =
2 P2 5 ´ 2000 P2 5 Atmospheric pressure, P0 = 1 × 105 Pa
P2 Volume of room, V0 = 30 m3
\ = 5. Difference in number of molecules, nf – ni = ?
P1
2. (150) In first case, Using ideal gas equation, PV = nRT(N0),
From ideal gas equation N0 = Avogadro's number
PV = nRT PV
Þ n= (N )
P D V + V DP = 0 (As temperature is constant) RT 0

DV = -
DP
V
P0V0 æ 1 1 ö
...(i) \ nf – ni = ç - ÷N
P R è T f Ti ø 0
In second case, using ideal gas equation again
P DV = -nR DT 1 ´ 105 ´ 30 æ 1 1 ö
= ´ 6.023 ´ 10 23 ç - ÷
nR DT 8.314 è 300 290 ø
DV = - ...(ii) = – 2.5 × 1025
P
Equating (i) and (ii), we get 6. (c) From P-V graph,

nR DT DP V 1
=- V Þ DT = DP Pµ , T = constant and Pressure is increasing from 2
P P nR V
to 1 so option (3) represents correct T-P graph.
Comparing the above equation with | DT | = C | DP | , we 7. (a) In ideal gases the molecules are considered as point
have particles and for point particles, there is no internal
V DT 300 K excitation, no vibration and no rotation. For an ideal gas
C= = = = 150 K/atm the internal energy can only be translational kinetic energy
nR DP 2 atm
and for real gas both kinetic as well as potential energy.
8. (c) The centre of mass of gas molecules also moves with
3. (a) N = ò r(dv)
lorry with uniform speed. As there is no relative motion of
r
4
r
4 gas molecule. So, kinetic energy and hence temperature
-ar
= ò n0 e ´ 4pr 2 dr = 4p n òr
2
(e -ar )dr remain same.
0
0 0
3
µ n0 a–3/4 9. (c) Given : K.E.mean = kT = 4 ´ 10 -14
4. (d) Clearly from figure, 2
P2A = P1A + mg P = 2 cm of Hg, V = 4 cm3
nRT × A nRT × A PV PrgV 2 ´ 13.6 ´ 980 ´ 4
or, + mg ; 4 ´ 1018
Al 2 = Al1 N= =
8
KT KT
´ 10 -14
3
n
l1 T P1A 10. (41) Room mean square speed is given by

3RT
vrms =
l2 n P2A mg M
T
Kinetic Theory P-193

Here, M = Molar mass of gas molecule Now, change in total kinetic energy of the gas
T = temperature of the gas molecule 1
DU = Q = nRT
We have given vN2 = vH2 2
1 mN 2
3RTN2 3RTH 2 16 . (c) Pressure, P = V rm s
3 V
\ =
M N2 M H2 (mN )T
or, P =
V
TH2 573 If the gas mass and temperature are constant then
Þ = Þ TH2 = 41 K
2 28 P µ (Vrms)2 µ T
So, force µ (Vrms)2 µ T
3RT i.e., Value of q = 1
11. (c) Vrms =
M 17. (d) Kinetic energy of each molecule,
v1 T1 (273 + 127) 400 4 2 3
= = = = = K.E. = K B T
v2 T2 (273 + 237) 500 5 5 2
In the given problem,
5 5 Temperature, T = 0°C = 273 K
\ v2 = v1 = ´ 200 = 100 5 m/s. Height attained by the gas molecule, h = ?
2 2
12. (Bouns) Rate of change of momentum during collision 3 819K B
K.E. = K B ( 273) =
mv – (– mv ) 2mv 2 2
= = N K.E. = P.E.
Dt Dt
819K B
Þ = Mgh
so pressure P = N ´ (2mv) 2
Dt ´ A 819K B
or h =
1022 ´ 2 ´ 10–26 ´ 104 2Mg
= = 2 N / m2
1´ 1 3RT
18. (a) Q C=
13. (c) vrms = ve M
3 ´ 8.314 ´ 300
3RT
= 11.2 ´ 103 (1930 ) =
2

M M
3 ´ 8.314 ´ 300
M= » 2 ´10 -3 kg
3kT 1930 ´1930
or = 11.2 ´ 103
m The gas is H2.
3rv
3 ´ 1.38 ´ 10-23 T 3 19. (c) v rms =
or = 11.2 ´ 10 \ v = 104 K mass of the gas
2 ´ 10-3 20. (a) Given, V1 = V
V1rms M2 V2 = 2V
14. (a) Using V = M1 T1 = 27° + 273 = 300 K
2rms
T2 = ?
Vrms ( He ) M Ar 40
= From charle’s law
Vrms ( Ar ) M He = 4 = 3.16
V1 V2
=
T1 T2 (
15. (a) Energy associated with N moles of diatomic gas, Q Pressure is constant )
5
Ui = N RT V 2V
2 or, 300 = T
Energy associated with n moles of monoatomic gas 2
3 \ T2 = 600 K = 600 – 273 = 327°C
= n RT 21. (c) As, work done is zero.
2
Total energy when n moles of diatomic gas converted into So, loss in kinetic energy = heat gain by the gas
1 2 R
3 5 mv = nCv DT = n DT
monoatomic (Uf) = 2n RT + (N - n) RT 2 g -1
2 2
1 2 m R
1 5 mv = DT
= nRT + NRT 2 M g -1
2 2
P-194 Physics

Mv 2 ( g - 1) Cp 2 2 7
\ DT = K And g = = 1+ = 1+ =
2R Cv f 5 5
n1
22. (a) Number of moles of first gas = N 1
A
n2 27. (b) Mean free path, l =
Number of moles of second gas = N 2pnd 2
A where, d = diameter of the molecule
n3
Number of moles of third gas = N n = number of molecules per unit volume
A
If there is no loss of energy then l
But, mean time of collision, t =
P1V1 + P2V2 + P3V3 = PV vrms
n1 n n
RT1 + 2 RT2 + 3 RT3 3kT
NA NA NA But vrms =
R
n1 + n2 + n3
= RTmix
NA l 1
\t = Þtµ
n1T1 + n2T2 + n3T3 3kT T
Tmix = n1 + n2 + n3 m
23. (a) Given, mass = 1 kg 28. (c) As we know,
Density = 4 kg m–3
Cp 2
mass 1 g= = 1+ , where f = degree of freedom
Volume = = m3 Cv f
density 4
Internal energy of the diatomic gas (A) Monatomic, f = 3
5 5 1 2 5
= PV = ´ 8 ´ 104 ´ = 5 ´ 10 4 J \ g = 1+ =
2 2 4 3 3
Alternatively: (B) Diatomic rigid molecules, f = 5
5 5m 5 m PM 2 7
K.E = nRT = RT = ´ [Q PM = dRT ] \ g = 1+ =
2 2M 2M d 5 5
4
5 mP 5 1 ´ 8 ´ 10 (C) Diatomic non-rigid molecules, f = 7
= = ´ = 5 ´ 104 J
2 d 2 4 2 9
\ g = 1+ =
gRT 7 7
24. (a) The speed of sound in a gas is given by v =
M (D) Triatomic rigid molecules, f = 6
g 2 4
\ vµ [As R and T is constant] \ g = 1+ =
M 6 3
vO2 g O2 M He 29. (266.67) Here work done on gas and heat supplied to the
\ = ´ gas are zero.
vHe M O2 g He
Let T be the final equilibrium temperature of the gas in the
1.4 4 vessel.
= ´ = 0.3237 Total internal energy of gases remain same.
32 1.67
vO 2 460 i.e., u1 + u2 = u '1 + u '2
\ vHe = = = 1421 m / s
0.3237 0.3237 or, n1Cv DT1 + n2Cv DT2 = (n1 + n2 )CvT
25. (d) RMS velocity of a gas molecule is given by Þ (0.1)Cv (200) + (0.05)Cv (400) = (0.15)CvT
3RT 800
Vrms = \T = = 266.67 K
M 3
Let T be the temperature at which the velocity of hydrogen
30. (d) Here degree of freedom, f = 3 + 3 = 6 for triatomic non-
molecule is equal to the velocity of oxygen molecule.
linear molecule.
3RT 3R ´ (273 + 47) Internal energy of a mole of the gas at temperature T,
\ =
2 32 f 6
Þ T = 20K U= nRT = RT = 3RT
2 2
26. (c) Total degree of freedom f = 3 + 2 = 5 31. (b) Let Cp and Cv be the specific heat capacity of the gas
Total energy, U =
nfRT 5RT at constant pressure and volume.
=
2 2
Kinetic Theory P-195

At constant pressure, heat required


5
DQ1 = nC p DT R
CvA
5
B 7 = 2 =
Þ 160 = nC p × 50 \ Cv = R Hence C B 7 7
...(i) 2 v R
2
At constant volume, heat required
35. (Bonus) Mean free path of a gas molecule is given by
DQ2 = nCv DT
1
l=
Þ 240 = nCv ×100 ...(ii) 2pd 2 n
Dividing (i) by (ii), we get Here, n = number of collisions per unit volume
d = diameter of the molecule
160 C p 50 Cp 4
= × Þ = If average speed of molecule is v then
240 Cv 100 Cv 3
l
Cp 4 2 Mean free time, t =
g= = = 1+ (Here, f = degree of freedom) v
Cv 3 f 1 1 M
Þ t= =
Þ f = 6. 2
2pnd v 2pnd 2 3RT
32. (a) Total energy of the gas mixture, æ 3RT ö
f1n1 RT1 f 2 n2 RT2 çQ v = M ÷
Emix = + è ø
2 2 2
5 5 M \ t1 = M1 ´ d2
= 3 ´ RT + ´ 3RT = 15RT \ tµ
d2 t2 d12 M2
2 2
33. (a) As we know mean free path 40 æ 0.1 ö
2
= ´ç ÷ = 1.09
1 140 è 0.07 ø
l=
æ Nö mean free path
2 ç ÷ pd 2 1
èV ø 36. (c) Relaxation time (t ) µ Þ tµ
speed v
Here, N = no. of molecule and, v µ T
V = volume of container
1
d = diameter of molecule \tµ
T
But PV = nRT = nNKT
1
N P Hence graph between t v/s is a straight line which
Þ = =n T
V KT is correctly depicted by graph shown in option (c).
1 KT 37. (a) Helium is a monoatomic gas and Oxygen is a diatomic
l= gas.
2 pd 2 P
For constant volume and hence constant number density 3 5
For helium, CV1 = R and CP1 = R
2 2
P
n of gas molecules is constant.
T 5 7
For oxygen, CV2 = R and CP = R
So mean free path remains same. 2 2 2
As temperature increases no. of collision increases so N1CP1 + N 2 CP2
relaxation time decreases. g=
34. (d) Specific heat of gas at constant volume N1CV1 + N 2 CV2

1 5 7
Cv = fR; f = degree of freedom n. R + 2n. R
2 g= 2 2 = 19nR ´ 2
For gas A (diatomic) Þ 3 5 2(13nR )
n. R + 2n. R
f = 5 (3 translational + 2 rotational) 2 2
5 æC ö 19
\C A = R \ç P ÷ =
v 2 è V ø mixture 13
C
For gas B (diatomic) in addition to (3 translational + 2
rotational) 2 vibrational degree of freedom.
P-196 Physics

n1C p + n2C p 1
1 2 43. (a) mv 2 = 3k BT
38. (d) Using, gmixture= 2
n1Cv + n2Cv
1 2
mv 2
n1 n n +n or T =
Þ + 2 = 1 2 6k B
g 1 –1 g 2 –1 g m –1 CP 29
3 2 5 44. (b) gA = = = 1.32 < 1.4 (diatomic)
Þ + = Cv 22
4 5 g m –1
–1 –1 30 10
3 3 and g B = = = 1.43 > 1.4
21 7
9 2´3 5 5 Gas A has more than 5-degrees of freedom.
Þ + = Þ g m –1 =
1 2 g m –1 12 45. (a) Energy of the gas, E
17 f f
Þ gm = = 1.42 = nRT = PV
12 2 2
n1[Cv1 ] + n2 [Cv2 ] f
= (3 ´ 106 )(2) = f ´ 3 ´ 106
39. (c) [Cv]min = 2
n1 + n2
Considering gas is monoatomic i.e., f = 3
é 3R 5R ù Energy, E = 9 × 106 J
ê2´ 2 + 3´ 2 ú 1
46. (b) Using, t=
= êê 2+3
ú
ú 2npd 2 Vavg
ë û
= 2.1 R = 2.1 × 8.3 = 17.4 J/mol–k T é no.of molecules ù
\t µ êë\n = úû
P Volume
Cv 1 1 5
40. (b) F = C = r = (7 / 5) = t1 500 P
p 7 or, = ´ » 4´10–8
6 ´10 –8
2P 300
W 5 2
or = 1- = f1 f
Q 7 7 47. (a) U = n1RT + 2 n 2RT
2 2
7 7 ´ 10 Considering translational and rotational modes, degrees
or Q = W = = 35 J
2 2 of freedom f1 = 5 and f2 = 3
41. (c) V = 25 × 10–3 m3, N = 1 mole of O2
T = 300 K 5 3
\ u = (3RT) + ´ 5RT
Vrms = 200 m/s 2 2
1 U = 15RT
\ l= 48. (a) According to question VT = K
2Npr 2
1 <V> we also know that PV = nRT
Average time = = 200.Npr 2 . 2
t l æ PV ö
ÞT = ç
23 è nR ÷ø
2 ´ 200 ´ 6.023 ´ 10
= .p ´ 10-18 ´ 0.09
25 ´ 10 -3 æ PV ö
ÞVç ÷ = k Þ PV = K
2

The closest value in the given option is = 1010 è nR ø


42. (c) Amount of heat required (Q) to raise the temperature R
at constant volume Q C= + CV (For polytropic process)
1– x
Q = nCvDT ...(i)
R 3R R
Amount of heat required (Q1) at constant pressure C= + =
Q1 = nCPDT ...(ii) 1– 2 2 2
Dividing equation (ii) by (i), we get \DQ = nC DT
Q1 C p R
\ = = ´DT [here, n = 1 mole]
Q Cv 2
49. (c) Thermal energy of N molecule
æ 7ö æ C p 7ö
æ3 ö
Þ Q1 = (Q) ç ÷
è 5ø çQ g = C = 5 ÷ = N ç kT ÷
è v ø è2 ø
Kinetic Theory P-197

N 3 3 3 54. (d) The ratio of specific heats at constant pressure (C p)


= RT = ( nRT ) = PV and constant volume (Cv)
NA 2 2 2
Cp æ 2ö
3 æmö 3 æ 2ö = g = ç1 + ÷
= Pç ÷= Pç ÷ Cv è fø
2 è r ø 2 è8ø
where f is degree of freedom
3 2 Cp æ 2 ö 7
= ´ 4´10 ´ = 1.5 ´10 J
4 4

2 8 = ç1 + ÷ =
Cv è 5 ø 5
therefore, order = 104 J
55. (d) For a polytropic process
50. (c) Heat transferred,
R R
Q = nCv DT as gas in closed vessel C = Cv + \ C - Cv =
To double the rms speed, temperature should be 4 times 1- n 1- n
R R
i.e., T' = 4T as vrms = 3RT / M \ 1- n = \ 1- =n
C - Cv C - Cv
15 5 ´ R C - C v - R C - C v - Cp + C v
\Q= ´ ´ ( 4T - T ) \ n= =
28 2 C - Cv C - Cv
é CP 7 ù C - Cp
êë\ CV = γ diatomic = 5 & C p - Cv = R úû = (Q C p - C v = R )
C - Cv
or, Q = 10000 J = 10 kJ
56. (d) Using equipartition of energy, we have
51. (c) In an adiabatic process
TVg–1 = Constant or, T1V1g–1 = T2V2g–1 6
KT = mCT
5 2
For monoatomic gas g =
3 3 ´1.38 ´10 –23 ´ 6.02 ´1023
C=
300 27 ´10–3
(300)V2/3 = T2(2V)2/3 Þ T2 = \ C = 925 J/kgK
(2) 2/3
T2 = 189 K (final temperature) 57. (b)
f 58. (b) On giving same amount of heat at constant pressure,
Change in internal energy DU = n R DT there is no change in temperature for mono, dia and
2
polyatomic.
æ 3 öæ 25 ö
= 2 ç ÷ç ÷ (-111) = -2.7 kJ æ No. of molecules ö
è 2 øè 3 ø ( DQ) P = mC p DT ç m =
52. (b) Using formula, è Avogedro 's no. ÷ø
n1g 1 n 2 g 2 1
+ or DT µ
æ Cp ö g1 - 1 g 2 - 1 no. of molecules
g mixture = ç ÷ =
è Cv ø mix n1 n 59. (c) TV g -1 = constant
+ 2
g1 - 1 g 2 - 1 g -1
T1V1g -1 = T2V2
æ Cp ö 3
Putting the value of n1 = 2, n2 = n, ç ÷ = 1
è C v ø mix 2 1 æV ö 2
Þ T (V ) 2 =T2 ç ÷
è 4ø
5 7
g 1 = , g 2 = and solving we get, n = 2 é Vù
3 5
êëQ g = 1.5, T1 = T ,V1 = V and V2 = 4 úû
53. (a) As we know, Cp – Cv = R where Cp and Cv are molar
specific heat capacities 1
æ 4V ö 2
R \ T2 = ç ÷ T = 2T
or, Cp – Cv = èV ø
M
60. (b) According to Mayer's relationship
R
For hydrogen (M = 2) Cp – Cv = a = CP – CV = R, as per the question (CP – CV) M = R
2
Þ CP – CV = R/28
R
For nitrogen (M = 28) Cp – Cv = b = Here M = 28 = mass of 1 unit of N2
28
61. (a) For mixture of gas specific heat at constant volume
a
\ = 14 or, a = 14b n1Cv1 + n2Cv2
b
Cv =
n1 + n2
P-198 Physics

No. of moles of helium, Cp 47 R 18


mHe 16 Þ
\ = ´ = 1.62
n1 = M = =4 Cv 18 29 R
He 4
Number of moles of oxygen, 5 7
62. (c) g 1 = g =
16 1 3 2 5
n2 = = n1 = 1, n2 = 1
32 2
n1 + n2 n n
3 1 5 5 = 1 + 2
4 ´ R + ´ R 6R + R g -1 g1 - 1 g 2 - 1
= 2 2 2 = 4
\ Cv æ 1ö 9 1 +1 1 1 3 5
çè 4 + ÷ø Þ = + = + =4
2 2 g - 1 5 - 1 7 -1 2 2
3 5
29 R ´ 2 29 R 2 3
= = and \ =4 Þ g=
9´ 4 18
g -1 2
Specific heat at constant pressure
63. (d) P µ T 3 Þ PT -3 = constant ....(i)
5R 1 7 R
n1C p1 + n2C p2 4´ + ´ But for an adiabatic process, the pressure temperature
Cp = = 2 2 2 relationship is given by
(n1 + n2 ) æ 1ö
çè 4 + ÷ø P1-g T g = constant
2
g
7 Þ PT 1-g
= constt. ....(ii)
10 R + R
= 4 = 47 R g 3
9 18 From (i) and (ii) = -3 Þ g = -3 + 3g Þ g =
1- g 2
2
13
Oscillations P-199

Oscillations
Displacement, Phase, Velocity 5. Two particles are performing simple harmonic motion in a
TOPIC 1 straight line about the same equilibrium point. The
and Acceleration in S.H.M.
amplitude and time period for both particles are same
1. The position co-ordinates of a particle moving in a 3-D and equal to A and T, respectively. At time t = 0 one particle
coordinate system is given by [9 Jan 2019, II] has displacement A while the other one has displacement
x = a coswt -A
y = a sinwt and they are moving towards each other. If they cross
2
and z = awt
The speed of the particle is: each other at time t, then t is:
[Online April 9, 2016]
(a) 2 aw (b) aw (c) 3 aw (d) 2aw
2. Two simple harmonic motions, as shown, are at right angles. 5T T T T
(a) (b) (c) (d)
They are combined to form Lissajous figures. 6 3 4 6
x(t) = A sin (at + d) 6. A simple harmonic oscillator of angular frequency 2 rad
y(t) = B sin (bt) s–1 is acted upon by an external force F = sin t N. If the
Identify the correct match below oscillator is at rest in its equilibrium position at t = 0, its
[Online April 15, 2018] position at later times is proportional to :
p [Online April 10, 2015]
(a) Parameters: A = B, a = 2b; d = ; Curve: Circle
2 1 1
p (a) sin t + cos 2t (b) cos t - sin 2t
(b) Parameters: A = B, a = b; d = ; Curve: Line 2 2
2 1 1
p (c) sin t - sin 2t (d) sin t + sin 2t
(c) Parameters: A ¹ B, a = b; d = ; Curve: Ellipse 2 2
2 7. x and y displacements of a particle are given as x(t) = a sin
(d) Parameters: A ¹ B, a = b; d = 0; Curve: Parabola
wt and y (t) = a sin 2wt. Its trajectory will look like :
3. The ratio of maximum acceleration to maximum velocity in [Online April 10, 2015]
a simple harmonic motion is 10 s–1. At, t = 0 the displacement
y y
is 5 m. What is the maximum acceleration ? The initial phase
is p [Online April 8, 2017]
4 x x
(a) (b)
(a) 500 m/s2 (b) 500 2 m/s2
(c) 750 m/s2 (d) 750 2 m/s2
4. A particle performs simple harmonic mition with amplitude y
y
A. Its speed is trebled at the instant that it is at a distance
2A
from equilibrium position. The new amplitude of the x x
3 (c) (d)
motion is : [2016]
7A A
(a) A 3 (b) (c) 41 (d) 3A
3 3
P-200 Physics

8. A body is in simple harmonic motion with time period half 12. Two particles are executing simple harmonic motion of
second (T = 0.5 s) and amplitude one cm (A = 1 cm). Find the same amplitude A and frequency w along the x-axis.
the average velocity in the interval in which it moves form Their mean position is separated by distance X0(X0 > A).
equilibrium position to half of its amplitude. If the maximum separation between them is (X0 + A), the
[Online April 19, 2014] phase difference between their motion is: [2011]
p p p p
(a) 4 cm/s (b) 6 cm/s (c) 12 cm/s (d) 16 cm/s (a) (b) (c) (d)
3 4 6 2
9. Which of the following expressions corresponds to simple
13. A mass M, attached to a horizontal spring, executes S.H.M.
harmonic motion along a straight line, where x is the
with amplitude A1. When the mass M passes through its
displacement and a, b, c are positive constants?
mean position then a smaller mass m is placed over it and
[Online April 12, 2014]
both of them move together with amplitude A2. The ratio
(a) a + bx – cx 2 (b) bx2
æ A1 ö
(c) a – bx + cx2 (d) – bx of ç A ÷ is: [2011]
10. A particle which is simultaneously subjected to two è 2ø
perpendicular simple harmonic motions represented by; 1
x = a1 cos wt and y = a2 cos 2 wt traces a curve given by: M +m æ M ö2
(a) (b) çè ÷
[Online April 9, 2014]
M M + mø
1
y y æ M + mö 2 M
(c) çè ÷ (d)
M ø M +m
14. A point mass oscillates along the x-axis according to the
a2 a2
a1
law x = x0 cos(wt - p / 4) . If the acceleration of the particle
(a) O
x (b) O a1
x
is written as a = A cos(wt + d ) ,then [2007]

(a) A = x0 w 2 , d = 3p / 4 (b) A = x0, d = -p / 4


y y 2
(c) A = x0 w 2 , d = p / 4 (d) A = x0 w , d = -p / 4
15. A coin is placed on a horizontal platform which undergoes
a2 a2 vertical simple harmonic motion of angular frequency w.
a1 a1 The amplitude of oscillation is gradually increased. The
(c) x (d) x
O O coin will leave contact with the platform for the first time
(a) at the mean position of the platform [2006]
g
11. The displacement y(t) = A sin (wt + f) of a pendulum for (b) for an amplitude of 2
w
2p
f= is correctly represented by g2
3 (c) for an amplitude of
[Online May 19, 2012] w2
y y (d) at the highest position of the platform
16. The maximum velocity of a particle, executing simple
harmonic motion with an amplitude 7 mm, is 4.4 m/s. The
period of oscillation is [2006]
t (a) 0.01 s (b) 10 s (c) 0.1 s (d) 100 s
(a) (b) t
17. The function sin 2 (wt ) represents [2005]
(a) a periodic, but not simple harmonic motion with a
y y p
period
w
(b) a periodic, but not simple harmonic motion with a
2p
period
w
(c) t (d) t
p
(c) a simple harmonic motion with a period
w
2p
(d) a simple harmonic motion with a period
w
Oscillations P-201

18. Two simple harmonic motions are represented by the 23. A pendulum is executing simple harmonic motion and its
æ pö maximum kinetic energy is K1. If the length of the
equations y1 = 0.1 sin ç100pt + ÷ and y 2 = 0.1 cos pt . pendulum is doubled and it performs simple harmonic
è 3ø
motion with the same amplitude as in the first case, its
The phase difference of the velocity of particle 1 with maximum kinetic energy is K2. [11 Jan 2019, II]
respect to the velocity of particle 2 is [2005]
K1
p -p p -p (a) K2 = 2K1 (b) K2 =
(a) (b) (c) (d) 2
3 6 6 3
K1
19. Two particles A and B of equal masses are suspended from (c) K2 = (d) K2 = K1
two massless springs of spring constants k 1 and k 2, 4
respectively. If the maximum velocities, during 24. A particle is executing simple harmonic motion (SHM) of
oscillation, are equal, the ratio of amplitude of A and B amplitude A, along the x-axis, about x = 0. When its
is [2003] potential Energy (PE) equals kinetic energy (KE), the
position of the particle will be: [9 Jan 2019, II]
k1 k2 k2 k1
(a) (b) k (c) (d) k A A
k2 1 k1 2 (a) (b)
2 2 2
20. The displacement of a particle varies according to the
relation x = 4(cos pt + sin pt ). The amplitude of the A
(c) (d) A
particle is [2003] 2
25. A particle is executing simple harmonic motion with a time
(a) – 4 (b) 4 (c) 4 2 (d) 8 period T. At time t = 0, it is at its position of equilibrium.
The kinetic energy-time graph of the particle will look like:
Energy in Simple Harmonic [2017]
TOPIC 2
Motion
21. The displacement time graph of a particle executing S.H.M. (a) (b)
is given in figure : (sketch is schematic and not to scale)
displacement

2T
4 (c) (d)
O time (s)
T 3T T 5T
4 4 4 26. A block of mass 0.1 kg is connected to an elastic spring of
spring constant 640 Nm–1 and oscillates in a medium of
Which of the following statements is/are true for this constant 10–2 kg s–1. The system dissipates its energy
motion? [Sep. 02, 2020 (II)] gradually. The time taken for its mechanical energy of
vibration to drop to half of its initial value, is closest to :
3T [Online April 9, 2017]
(1) The force is zero at t =
4 (a) 2 s (b) 3.5 s (c) 5 s (d) 7 s
(2) The acceleration is maximum at t = T 27. For a simple pendulum, a graph is plotted between its
T kinetic energy (KE) and potential energy (PE) against its
(3) The speed is maximum at t =
4 displacement d. Which one of the following represents these
T correctly? (graphs are schematic and not drawn to scale)
(4) The P.E. is equal to K.E. of the oscillation at t = [2015]
2
E KE
(a) (1), (2) and (4) (b) (2), (3) and (4) E
PE
(c) (1), (2) and (3) (d) (1) and (4)
22. A particle undergoing simple harmonic motion has time (a) d (b)
KE

pt
dependent displacement given by x ( t ) = A sin . The PE
90
E E
ratio of kinetic to potential energy of this particle at KE
PE
t = 210s will be : [11 Jan 2019, I]
(c) PE (d)
KE
1 1 d d
(a) (b) 1 (c) 2 (d)
9 3
P-202 Physics

28. A pendulum with time period of 1s is losing energy. At


certain time its energy is 45 J. If after completing 15
Time Period, Frequency,
oscillations, its energy has become 15 J, its damping TOPIC 3 Simple Pendulum and Spring
constant (in s–1) is : [Online April 11, 2015] Pendulum
1 1 1 35. An object of mass m is suspended at the end of a massless
(a) (b) ln3 (c) 2 (d) ln3
2 30 15 wire of length L and area of cross-section, A. Young
29. This question has Statement 1 and Statement 2. Of the modulus of the material of the wire is Y. If the mass is
four choices given after the Statements, choose the one pulled down slightly its frequency of oscillation along the
that best describes the two Statements. vertical direction is: [Sep. 06, 2020 (I)]
If two springs S1 and S2 of force constants k1 and k 2
respectively, are stretched by the same force, it is found 1 mL 1 YA
(a) f = (b) f =
that more work is done on spring S1 than on spring S2. 2p YA 2p mL
Statement 1 : If stretched by the same amount work done
1 mA 1 YL
on S1 (c) f = (d) f =
Statement 2 : k1 < k2 [2012] 2p YL 2p mA
(a) Statement 1 is false, Statement 2 is true. 36. When a particle of mass m is attached to a vertical spring
(b) Statement 1 is true, Statement 2 is false. of spring constant k and released, its motion is described
(c) Statement 1 is true, Statement 2 is true, Statement 2 is by y (t) = y0 sin2wt, where ‘y’ is measured from the lower
the correct explanation for Statement 1 end of unstretched spring. Then w is :
(d) Statement 1 is true, Statement 2 is true, Statement 2 is [Sep. 06, 2020 (II)]
not the correct explanation for Statement 1
30. A particle of mass m executes simple harmonic motion with 1 g g
(a) (b)
amplitude a and frequency n. The average kinetic energy 2 y0 y0
during its motion from the position of equilibrium to the
end is [2007] g 2g
(c) (d)
2
(a) 2p ma n 2 2 2
(b) p ma n 2 2 2 y0 y0
1 37. A block of mass m attached to a massless spring is
2 2
(c) ma n (d) 4p 2 ma 2 n2 performing oscillatory motion of amplitude 'A' on a
4
31. Starting from the origin a body oscillates simple frictionless horizontal plane. If half of the mass of the block
harmonically with a period of 2 s. After what time will its breaks off when it is passing through its equilibrium point,
kinetic energy be 75% of the total energy? [2006] the amplitude of oscillation for the remaining system
become fA. The value of f is : [Sep. 03, 2020 (II)]
1 1 1 1
(a) s (b) s (c) s (d) s 1 1
6 4 3 12 (a) (b) 1 (c) (d) 2
2 2
32. The total energy of a particle, executing simple harmonic
motion is [2004] 38. A person of mass M is, sitting on a swing of length L and
swinging with an angular amplitude q0. If the person stands
(a) independent of x (b) µ x2
up when the swing passes through its lowest point, the
(c) µ x (d) µ x1/2 work done by him, assuming that his centre of mass
where x is the displacement from the mean position, hence moves by a distance l (l<<L), is close to :
total energy is independent of x. [12 April 2019, II]
33. A body executes simple harmonic motion. The potential (a) mgl (1– q02) (b) mgl (1+q02)
energy (P.E), the kinetic energy (K.E) and total energy (T.E)
æ q0 2 ö
are measured as a function of displacement x. Which of
(c) mgl (d) Mgl çç 1 + 2 ÷
÷
the following statements is true ? [2003] è ø
(a) K.E. is maximum when x = 0 39. A simple pendulum oscillating in air has period T. The bob
(b) T.E is zero when x = 0
of the pendulum is completely immersed in a non-viscous
(c) K.E is maximum when x is maximum
(d) P.E is maximum when x = 0 1
liquid. The density of the liquid is th of the material of
34. In a simple harmonic oscillator, at the mean position 16
[2002] the bob. If the bob is inside liquid all the time, its period of
(a) kinetic energy is minimum, potential energy is maximum oscillation in this liquid is : [9 April 2019 I]
(b) both kinetic and potential energies are maximum
1 1 1 1
(c) kinetic energy is maximum, potential energy is minimum (a) 2T (b) 2T (c) 4T (d) 4T
(d) both kinetic and potential energies are minimum 10 14 15 14
Oscillations P-203

40. Two light identical springs of spring constant k are attached 3 2


horizontally at the two ends of a uniform horizontal rod AB of (a) s (b) s
2 3
length l and mass m. The rod is pivoted at its centre ‘O’ and
can rotate frreely in horizontal plane. The other ends of two 3
(c) s (d) 2 3s
springs are fixed to rigid supports as shown in figure. The rod 2
is gently pushed through a small angle and released. The 45. A particle executes simple harmonic motion with an
frequency of resulting oscillation is: [12 Jan 2019, I] amplitude of 5 cm. When the particle is at 4 cm from the
A mean position, the magnitude of its velocity in SI units
is equal to that of its acceleration. Then, its periodic time
in seconds is: [10 Jan 2019, II]

y 4p 3
(a) (b) p
3 8
O
8p 7
(c) (d) p
x 3 3
46. A cylindrical plastic bottle of negligible mass is filled
with 310 ml of water and left floating in a pond with still
water. If pressed downward slightly and released, it
B starts performing simple harmonic motion at angular
1 3k 1 2k 1 6k 1 k frequency w. If the radius of the bottle is 2.5 cm then w
(a) (b) (c) (d) is close to: (density of water = 103 kg/m3)
2π m 2π m 2π m 2π m
41. A simple pendulum, made of a string of length l and a bob [10 Jan 2019, II]
–1
of mass m, is released from a small angle q0. It strikes a (a) 3.75 rad s (b) 1.25 rad s–1
block of mass M, kept on a horizontal surface at its lowest (c) 2.50 rad s –1
(d) 5.00 rad s–1
point of oscillations, elastically. It bounces back and goes 47. A rod of mass 'M' and length '2L' is suspended at its middle
up to an angle q1. The M is given by : [12 Jan 2019, I] by a wire. It exhibits torsional oscillations; If two masses
m æ θ0 + θ1 ö æ θ -θ ö each of 'm' are attached at distance 'L/2' from its centre on
(a) ç ÷ (b) mç 0 1 ÷
2 è θ0 - θ1 ø è θ0 + θ1 ø
both sides, it reduces the oscillation frequency by 20%.
The value of ratio m/M is close to : [9 Jan 2019, II]
æθ +θ ö m æ θ0 - θ1 ö (a) 0.77 (b) 0.57
(c) mç 0 1 ÷ (d) ç ÷
è θ0 - θ1 ø 2 è θ0 + θ1 ø (c) 0.37 (d) 0.17
42. A simple harmonic motion is represented by : 48. A silver atom in a solid oscillates in simple harmonic motion
y = 5(sin3pt + 3 cos3pt) cm in some direction with a frequency of 1012/sec. What is the
The amplitude and time period of the motion are : force constant of the bonds connecting one atom with the
other? (Mole wt. of silver = 108 and Avagadro number
[12 Jan 2019, II]
= 6.02 ×1023 gm mole –1) [2018]
2 3 (a) 6.4 N/m (b) 7.1 N/m (c) 2.2 N/m (d) 5.5 N/m
(a) 10 cm, s (b) 10 cm, s
3 2
49. A particle executes simple harmonic motion and is located
3 2 at x = a, b and c at times t0, 2t0 and 3t0 respectively. The
(c) 5 cm, s (d) 5 cm, s
2 3 frequency of the oscillation is [Online April 16, 2018]
43. A simple pendulum of length 1 m is oscillating with an
1 æa+bö 1 æa+bö
angular frequency 10 rad/s. The support of the pendulum (a) cos -1 ç ÷ (b) cos -1 ç ÷
starts oscillating up and down with a small angular 2pt 0 è 2c ø 2pt 0 è 3c ø
frequency of 1 rad/s and an amplitude of 10–2 m. The
1 æ 2a + 3c ö 1 æa +cö
relative change in the angular frequency of the pendulum (c) cos -1 ç ÷ (d) cos-1 ç ÷
is best given by : [11 Jan 2019, II] 2 pt 0 è b ø 2 pt 0 è 2b ø
(a) 10–3 rad/s (b) 1 rad/s 50. In an experiment to determine the period of a simple pen-
(c) 10–1 rad/s (d) 10–5 rad/s dulum of length 1 m, it is attached to different spherical
44. The mass and the diameter of a planet are three times the bobs of radii r1 and r2. The two spherical bobs have uni-
respective values for the Earth. The period of oscillation form mass distribution. If the relative difference in the pe-
of a simple pendulum on the Earth is 2s. The period of riods, is found to be 5 × 10–4 s, the difference in radii, |r1– r2|
oscillation of the same pendulum on the planet would be: is best given by: [Online April 9, 2017]
[11 Jan 2019, II] (a) 1 cm (b) 0.1 cm (c) 0.5 cm (d) 0.01 cm
P-204 Physics

51. A 1 kg block attached to a spring vibrates with a frequency

T (in s )
2
of 1 Hz on a frictionless horizontal table. Two springs
identical to the original spring are attached in parallel to an

2
8.0
8 kg block placed on the same table. So, the frequency of
vibration of the 8 kg block is : [Online April 8, 2017] 6.0
1 1 1
(a) Hz (b) Hz (c) Hz (d) 2 Hz 4.0
4 2 2 2
52. A pendulum clock loses 12 s a day if the temperature is
2.0
40°C and gains 4 s a day if the temperature is 20° C. The
temperature at which the clock will show correct time, and
the co-efficient of linear expansion (a) of the metal of the 0.5 1.0 1.5 2.0 L (in m)
pendulum shaft are respectively : [2016]
What is the value of g at the place?
(a) 30°C; a = 1.85 × 10–3/°C (a) 9.81 m/s2 (b) 9.87 m/s2
(b) 55°C; a = 1.85 × 10–2/°C (c) 9.91 m/s 2 (d) 10.0 m/s2
(c) 25°C; a = 1.85 × 10–5/°C 57. The amplitude of a simple pendulum, oscillating in air with
(d) 60°C; a = 1.85 × 10–4/°C a small spherical bob, decreases from 10 cm to 8 cm in 40
seconds. Assuming that Stokes law is valid, and ratio of
53. In an engine the piston undergoes vertical simple harmonic
the coefficient of viscosity of air to that of carbon dioxide
motion with amplitude 7 cm. A washer rests on top of the
piston and moves with it. The motor speed is slowly is 1.3. The time in which amplitude of this pendulum will
increased. The frequency of the piston at which the washer reduce from 10 cm to 5 cm in carbon dioxide will be close to
no longer stays in contact with the piston, is close to : (In 5 = 1.601, In 2 = 0.693). [Online April 9, 2014]
[Online April 10, 2016] (a) 231 s (b) 208 s (c) 161 s (d) 142 s
58. Two bodies of masses 1 kg and 4 kg are connected to a
(a) 0.7 Hz (b) 1.9 Hz (c) 1.2 Hz (d) 0.1 Hz
vertical spring, as shown in the figure. The smaller mass
54. A pendulum made of a uniform wire of cross sectional area executes simple harmonic motion of angular frequency
A has time period T. When an additional mass M is added 25 rad/s, and amplitude 1.6 cm while the bigger mass
to its bob, the time period changes to TM. If the Young's remains stationary on the ground. The maximum force
1 exerted by the system on the floor is (take g = 10 ms–2)
modulus of the material of the wire is Y then is equal
Y [Online April 9, 2014]
to: 1 kg
(g = gravitational acceleration) [2015]
é æ T ö2 ù A é æ T ö2 ù A
(a) ê1 - ç M ÷ ú (b) ê1 - ç T ÷ ú Mg
ë è T ø û Mg ëê è M ø úû
éæ T ö2 ù A éæ T ö
2 ù Mg 4 kg
(c) êç M ÷ - 1ú (d) êç M ÷ - 1ú
ëè T ø û Mg ëè T ø û A
55. A particle moves with simple harmonic motion in a straight (a) 20 N (b) 10 N (c) 60 N (d) 40 N
line. In first t s, after starting from rest it travels a distance 59. An ideal gas enclosed in a vertical cylindrical container
supports a freely moving piston of mass M. The piston
a, and in next t s it travels 2a, in same direction, then:
and the cylinder have equal cross sectional area A. When
(a) amplitude of motion is 3a [2014] the piston is in equilibrium, the volume of the gas is V0
(b) time period of oscillations is 8t and its pressure is P0. The piston is slightly displaced
(c) amplitude of motion is 4a from the equilibrium position and released. Assuming that
(d) time period of oscillations is 6t the system is completely isolated from its surrounding,
the piston executes a simple harmonic motion with
56. In an experiment for determining the gravitational
frequency [2013]
acceleration g of a place with the help of a simple
pendulum, the measured time period square is plotted 1 AgP0 V
1 0 0MP
(a) (b)
against the string length of the pendulum in the figure. 2p V0 M 2p A 2 g
[Online April 19, 2014]
1 A 2 gP0 1 MV0
(c) (d)
2p MV0 2p AgP0
Oscillations P-205

60. A mass m = 1.0 kg is put on a flat pan attached to a vertical ld lr


spring fixed on the ground. The mass of the spring and the (a) 2p (b) 2p
rg dg
pan is negligible. When pressed slightly and released, the
mass executes simple harmonic motion. The spring ld lr
constant is 500 N/m. What is the amplitude A of the (c) 2p (d) 2p (r - d ) g
motion, so that the mass m tends to get detached from
(r - d ) g
the pan ? 65. If x, v and a denote the displacement, the velocity and
(Take g = 10 m/s2). the acceleration of a particle executing simple harmonic
motion of time period T, then, which of the following
The spring is stiff enough so that it does not get distorted
does not change with time? [2009]
during the motion. [Online April 22, 2013]
(a) aT/x (b) aT + 2pv
m (c) aT/v (d) a2T2 + 4p2v2
66. Two springs, of force constants k1 and k2 are connected
to a mass m as shown. The frequency of oscillation of
the mass is f. If both k1 and k2 are made four times their
k original values, the frequency of oscillation becomes
[2007]
m
(a) A > 2.0 cm (b) A = 2.0 cm k1 k2
(c) A < 2.0 cm (d) A = 1.5 cm
61. Two simple pendulums of length 1 m and 4 m respectively (a) 2 f (b) f /2 (c) f /4 (d) 4 f
are both given small displacement in the same direction 67. The displacement of an object attached to a spring and
at the same instant. They will be again in phase after the executing simple harmonic motion is given by x = 2 × 10–2
shorter pendulum has completed number of oscillations cos pt metre.The time at which the maximum speed first
equal to : [Online April 9, 2013] occurs is [2007]
(a) 2 (b) 7 (c) 5 (d) 3
(a) 0.25 s (b) 0.5 s (c) 0.75 s (d) 0.125 s
62. If a simple pendulum has significant amplitude (up to a
68. The bob of a simple pendulum is a spherical hollow ball
factor of 1/e of original) only in the period between t =
filled with water. A plugged hole near the bottom of the
0s to t = t s, then t may be called the average life of the
oscillating bob gets suddenly unplugged. During
pendulum. When the spherical bob of the pendulum
observation, till water is coming out, the time period of
suffers a retardation (due to viscous drag) proportional oscillation would [2005]
to its velocity with b as the constant of proportionality,
the average life time of the pendulum in second is (a) first decrease and then increase to the original value
(assuming damping is small) [2012] (b) first increase and then decrease to the original value
0.693 1 2 (c) increase towards a saturation value
(a) (b) b (c) (d)
b b b (d) remain unchanged
63. A ring is suspended from a point S on its rim as shown in 69. If a simple harmonic motion is represented by
the figure. When displaced from equilibrium, it oscillates 2
with time period of 1 second. The radius of the ring is d x
+ ax = 0 , its time period is [2005]
(take g = p2) [Online May 19, 2012] dt 2
S 2p 2p
(a) (b) (c) 2p a (d) 2pa
a a
70. The bob of a simple pendulum executes simple harmonic
motion in water with a period t, while the period of oscillation
of the bob is t0 in air. Neglecting frictional force of water and
(a) 0.15 m (b) 1.5 m (c) 1.0 m (d) 0.5 m given that the density of the bob is (4 / 3) ´ 1000 kg/m 3 .
64. A wooden cube (density of wood ‘d’) of side ‘l’ floats in a Which relationship between t and t0 is true? [2004]
liquid of density ‘r’ with its upper and lower surfaces (a) t = 2t 0 (b) t = t 0 / 2
horizontal. If the cube is pushed slightly down and
released, it performs simple harmonic motion of period (c) t = t0 (d) t = 4t 0
‘T’ [2011 RS]
P-206 Physics

71. A particle at the end of a spring executes S.H.M with a 78. An oscillator of mass M is at rest in its equilibrium position
period t1. while the corresponding period for another spring
1 2
is t2. If the period of oscillation with the two springs in in a potential V = k(x - X) . A particle of mass m comes
series is T then [2004] 2
from right with speed u and collides completely inelastically
-1 -1 -1
(a) T = t1 + t 2 (b) T 2 = t12 + t 22 with M and sticks to it. This process repeats every time
the oscillator crosses its equilibrium position. The
(c) T = t1 + t2 (d) T -2 = t1-2 + t 2-2
amplitude of oscillations after 13 collisions is:
72. A mass M is suspended from a spring of negligible mass. (M = 10, m = 5, u = 1, k = 1). [Online April 16, 2018]
The spring is pulled a little and then released so that the 1
1
mass executes SHM of time period T. If the mass is (a) (b)
2 3
5T
increased by m, the time period becomes . Then the
3 2 3
m (c) (d)
ratio of is [2003] 3 5
M
3 25 16 5 79. The angular frequency of the damped oscillator is given
(a) (b) (c) (d)
5 9 9 3
æk r2 ö
73. The length of a simple pendulum executing simple harmonic
by, w = çç m - ÷ where k is the spring constant, m
motion is increased by 21%. The percentage increase in è 4m 2 ÷ø
the time period of the pendulum of increased length is is the mass of the oscillator and r is the damping constant.
[2003]
r2
(a) 11% (b) 21% (c) 42% (d) 10% If the ratio is 8%, the change in time period
mk
74. If a spring has time period T, and is cut into n equal parts, compared to the undamped oscillator is approximately
then the time period of each part will be [2002] as follows: [Online April 11, 2014]
(a) T n (b) T / n (c) nT (d) T (a) increases by 1% (b) increases by 8%
75. A child swinging on a swing in sitting position, stands up, (c) decreases by 1% (d) decreases by 8%
then the time period if the swing will [2002] 80. The amplitude of a damped oscillator decreases to 0.9 times
(a) increase its original magnitude in 5s. In another 10s it will decrease
to a times its original magnitude, where a equals [2013]
(b) decrease
(a) 0.7 (b) 0.81 (c) 0.729 (d) 0.6
(c) remains same
81. A uniform cylinder of length L and mass M having cross-
(d) increases if the child is long and decreases if the
sectional area A is suspended, with its length vertical, from
child is short
a fixed point by a massless spring, such that it is half
submerged in a liquid of density s at equilibrium position.
Damped, Forced
TOPIC 4 When the cylinder is given a downward push and released,
Oscillations and Resonance it starts oscillating vertically with a small amplitude. The
time period T of the oscillations of the cylinder will be :
76. A damped harmonic oscillator has a frequency of 5
[Online April 25, 2013]
oscillations per second. The amplitude drops to half its
1
value for every 10 oscillations. The time it will take to drop é M ù 2
(a) Smaller than 2p ê ú
to
1
of the original amplitude is close to : ë (k + Asg ) û
1000
M
[8 April 2019, II] (b) 2p
k
(a) 50s (b) 100s (c) 20s (d) 10s 1
77. The displacement of a damped harmonic oscillator is given é M ù 2
(c) Larger than 2p ê ú
by x(t) = e–0.1t. cos(10pt + j). Here t is in seconds. ë (k + Asg ) û
1
The time taken for its amplitude of vibration to drop to half é M ù 2
of its initial value is close to : [9 Jan 2019, II] (d) 2p ê ú
ë ( k + Asg ) û
(a) 4s (b) 7s (c) 13s (d) 27s
Oscillations P-207

82. Bob of a simple pendulum of length l is made of iron. 83. In forced oscillation of a particle the amplitude is
The pendulum is oscillating over a horizontal coil maximum for a frequency w1 of the force while the energy
carrying direct current. If the time period of the pendulum is maximum for a frequency w2 of the force; then
is T then : [Online April 23, 2013] [2004]
(a) w1 < w2 when damping is small and w1 > w2 when
l damping is large
(a) T < 2p and damping is smaller than in air alone.
g (b) w1 > w2
(c) w1 = w2
l (d) w1 < w2
(b) T = 2p and damping is larger than in air alone.
g 84. A particle of mass m is attached to a spring (of spring
constant k) and has a natural angular frequency w0. An
l
(c) T > 2p and damping is smaller than in air alone. external force F(t) proportional to cos wt (w ¹ w 0 ) is
g
applied to the oscillator. The displacement of the oscillator
will be proportional to [2004]
l
(d) T < 2p and damping is larger than in air alone. 1 1
g (a) 2 2 (b) m (w 2 - w 2 )
m (w 0 + w ) 0
m m
(c) 2 2 (d) 2 2
w0 - w (w 0 + w )
P-208 Physics

1. (a) Here, vx = – a w sin wt, vy = a w cos wt and vz = aw Equilibrium point A


Þ v = v 2x + v 2y + v z2 5. (d)
A O
( –aw sin wt ) + ( aw cos wt ) + ( aw )
2 2 2
Þv= 2
(at time t = 0)
v = 2aw
2. (c) From the two mutually perpendicular S.H.M.’s, the
general equation of Lissajous figure,
12 0°

x2 y2 2 xy
2
+ 2- cos d = sin 2 d 60°
A B AB
x = A sin (at + d)
y = B sin (bt + r) p
Angle covered to meet q = 60° = rad.
Clearly A ¹ B hence ellipse. 3
3. (b) Maximum velocity in SHM, vmax = aw If they cross each other at time t then
Maximum acceleration in SHM, Amax = aw2 q p T
where a and w are maximum amplitude and angular t< < T<
2p 3≥2p 6
frequency. 6. (c) As we know,
A F = ma Þ a µ F
Given that, max = 10
v max or, a µ sin t
i.e., w = 10 s–1 Þ
dv
µ sin t
Displacement is given by dt
x = a sin (wt + p/4) 0 t
at t = 0, x = 5
5 = a sin p/4
Þ ò dV µ ò sin t dt
0 0
5 = a sin 45° Þ a = 5 2 V µ – cos t + 1
x t
Maximum acceleration Amax = aw2 = 500 2 m/s2
2 2
ò dx = ò (- cos t + 1) dt
4. (b) We know that V = w A - x 0 0
2 1
2 æ 2A ö x = sin t – sin 2t
Initially V = w A - ç ÷ 2
è 3 ø 7. (c) At t = 0, x (t) = 0 ; y (t) = 0
2 x (t) is a sinusoidal function
2 æ 2A ö
Finally 3V = w A' - ç ÷ p
è 3 ø At t = ; x (t) = a and y (t) = 0
2A 2w
Where A'= final amplitude (Given at x = , velocity to Hence trajectory of particle will look like as (c).
3
trebled) 8. (c) Given: Time period, T = 0.5 sec
On dividing we get Amplitude, A = 1 cm
Average velocity in the interval in which body moves
2 from equilibrium to half of its amplitude, v = ?
æ 2A ö
A' 2 - ç ÷
3 è 3 ø S
=
1 æ 2A ö
2
A2 - ç ÷
è 3 ø
é 2 4A 2 ù 4A 2 7A
9 ê A - 9 ú = A'2 – \ A' =
ëê ûú 9 3 O
Oscillations P-209

Time taken to a displacement A/2 where A is the


k k
T MA1 = ( M + m) A2
amplitude of oscillation from the mean position ‘O’ is M m+M
12
0.5 æ k ö
Therefore, time, t = sec \ çV = A M ÷
12 è ø
A 1 Þ A1 M = A2 M + m
Displacement, s = = cm
2 2
A 1 1 A m+M
Þ A = M
\ Average velocity, v = 2 = 2 = 12 cm / s 2
t 0.5 14. (a) Given,
12
9. (d) In linear S.H.M., the restoring force acting on particle Displacement, x = x0 cos (wt – p / 4 )
should always be proportional to the displacement of the dx æ pö
particle and directed towards the equilibrium position. \ Velocity, v = = - x0 w sin ç wt - ÷
dt è 4ø
i.e., F µ x
Acceleration,
or F = - bx where b is a positive constant.
dv 2 æ pö
10. (b) Two perpendicular S.H.Ms are a= = - x0w cos ç wt - ÷
dt è 4ø
x = a1 cos wt ....(1)
and y = a2 2 cos wt ....(2) 2 é æ pö ù
= x0w cos ê p + ç wt - ÷ ú
From eqn (1) ë è 4øû
x
= cos wt æ 3p ö
a1 = x0 w 2 cos ç wt + ÷ ...(1)
è 4ø
and from eqn (2)
Acceleration, a = A cos (wt + d) ...(2)
y Comparing the two equations, we get
= 2cos wt
a2
3p
A = x0w2 and d = .
a 4
\ y= 2 2 x
a1 15. (b) For block A to move in SHM.
N
11. (a) Displacement y (t) = A sin (wt + f)[Given]
2p A
For f =
3
2p
at t = 0; y = A sin f = A sin mg x
mean
3
= A sin 120° = 0.87 A [Q sin 120° ; 0.866] position
Graph (a) depicts y = 0.87A at t = 0 mg – N = mw2x
12. (a) Let, x1 = A sin w t and x2 = A sin (w t + f) where x is the distance from mean position
For block to leave contact N = 0
æ fö f
x2 – x1 = 2A cos çè wt + 2 ÷ø sin 2
g
Þ mg = mw 2 x Þ x =
f w2
The above equation is SHM with amplitude 2A sin
2 16. (a) Maximum velocity,
f
\ 2 A sin = A vmax = a w
2
f 1 p Here, a = amplitude of SHM
Þ sin = Þf=
2 2 3 w = angular velocity of SHM
13. (c) At mean position, F net = 0 2p æ 2p ö
Therefore, by principal of conseruation of linear
vmax = a ´ \ çèQ w = ÷ø
T T
momentum.
-3
\ Mv1 = (M + m)v2 2pa 2 ´ 3.14 ´ 7 ´ 10
ÞT = = » 0.01 s
M w, a, = (M + m) w2 a2 vmax 4.4
P-210 Physics

p x = 4 2 sin(p t + 45°)
17. (a) Clearly sin 2wt is a periodic function with period
w On comparing it with standard equation x = A sin(wt + f)
we get A = 4 2
21. (c) From graph equation of SHM
X = A cos wt
0 p/w 2p/w 3p/w
2 3T
d y (1) At particle is at mean position.
For SHM 2
µ -y 4
dt
\ Acceleration = 0, Force = 0
1 – cos 2wt
y = sin2 wt = (2) At T particle again at extreme position so acceleration
2 is maximum.
1 1
= – cos 2wt T
2 2 (3) At t = , particle is at mean position so velocity is
4
dy 1
v= = ´ 2w sin 2wt = 2w sin wt cos wt maximum.
dt 2
Acceleration = 0
= w sin 2wt
(4) When KE = PE
2
d y 2 1 1
Acceleration, a = = 2w cos 2wt which is not Þ k ( A2 - x 2 ) = kx 2
dt 2 2 2
proportional to –y. Hence, it is not in SHM. Here, A = amplitude of SHM
18. (b) Velocity of particle 1, x = displacement from mean position
dy1 æ pö +A
v1 = = 0.1 ´ 100p cos ç100pt + ÷ Þ A2 = 2 x 2 Þ x =
dt è 3ø 2
Velocity of particle 2,
A T
dy æ pö Þ = A cos wt Þ t =
v2 = 2 = - 0.1p sin pt = 0.1p cos ç pt + ÷ 2 2
dt è 2ø \ x = – A which is not possible
\ Phase difference of velocity of particle 1 with respect to \ 1, 2 and 3 are correct.
the velocity of particle 2 is
1
p p 2 p - 3p p 22. (d) Kinetic energy, k = mw 2 A 2 cos 2 wt
= f1 - f 2 = - = =– 2
3 2 6 6
1
19. (c) Maximum velocity during SHM, Vmax = Aw Potential energy, U = mw 2 A 2 sin 2 wt
But k = mw2 2
k 2 2 p 1
k = cot wt = cot (210) =
\ w= U 90 3
m
1 2 2
\ Maximum velocity of the body in SHM 23. (a) K = mw x
2
k
=A 1
m Þ K max = mw 2 A 2
2
As maximum velocities are equal
A = Lq
k1 k
\ A1 = A2 2 g
m m w=
L
A1 k2
Þ A1 k 1 = A 2 k2 Þ = 1 g 2 2
A2 k1 Þ K= m. .L q
2 L
20. (c) Displacement, x = 4(cos pt + sin pt ) 1
= mgLq 2
æ sin pt cos pt ö 2
= 2 ´ 4ç + ÷
è 2 2 ø K L 1
\ 1 = = Þ K 2 = 2K1
K 2 2L 2
= 4 2(sin p t cos 45° + cos p t sin 45°)
Oscillations P-211

K1 L 1 Taking log on both sides


\ = = Þ K 2 = 2K1 b 1
K 2 2L 2 = ln3
m 15
1 2
24. (c) Potential energy (U) = kx 1 2
2 29. (b) Work done, w = kx
2
1 2 1 2
Kinetic energy (K) = kA - kx 1
2 2 Work done by spring S1, w1 = k1x 2
2
According to the question, U = k
1
1 1 1 Work done by spring S2, w2 = k2 x 2
\ kx 2 = kA2 - kx 2 2
2 2 2
Since w1 > w2 Thus (k1 > k2)
A 30. (b) The kinetic energy of a particle executing S.H.M. at
Þ x2 = A2 or, x =±
2 any instant t is given by
25. (b) For a particle executing SHM
1 2 2 2
At mean position; t = 0, wt = 0, y = 0, V = Vmax = aw K= ma w sin wt
2
1
\ K.E. = KEmax = mw2a2 where, m = mass of particle
2 a = amplitude
T p w = angular frequency
At extreme position : t = , wt = , y = A, V = Vmin = 0
4 2 t = time
\ K.E. = KEmin = 0
1
1 The average value of sin 2wt over a cycle is .
Kinetic energy in SHM, KE = mw2(a2 – y2) 2
2
1 1 æ 1ö æ 2 1ö
= mw2a2cos2wt \KE = mw2a2 çè ÷ø çè Q < sin q > = ÷ø
2 2 2 2
Hence graph (b) correctly depicts kinetic energy time graph. 1 2 2 1
26. (b) Since system dissipates its energy gradually, and = mw a = ma2 (2pn)2 (Q w = 2pn)
4 4
hence amplitude will also decreases with time according to
a = a0 e–bt/m ....... (i) 2 2 2
or, < K > = p ma n
Q Energy of vibration drop to half of its initial value 31. (a) K.E. of a body undergoing SHM is given by,
(E0), as E µ a2 Þ a µ E 1 2 2 2
K .E. = ma w cos wt
a -2 2
a = 0 Þ bt = 10 t = t
2 m 0.1 10 Here, a = amplitude of SHM
w = angular velocity of SHM
From eqn (i),
a0 1 2 2
= a 0e - t 10 Total energy in S.H.M = ma w
2 2
1
t Given K.E. = 75% T.E.
= e - t 10 or 2 = e10 1 75 1
2 ma 2 w 2 cos 2 w t = ´ ma 2 w 2
t 2 100 2
ln 2 = \ t = 3.5 seconds
10 2 p
Þ 0.75 = cos wt Þ wt =
1 2 2 6
27. (d) K.E = k ( A - d )
2 p p´2 1
1 2 Þt= Þt= Þt= s
and P.E. = kd 6´w 6 ´ 2p 6
2 32. (a) At any instant the total energy in SHM is
At mean position d = 0. At extreme positions d = A
bt 1 2
– kA = constant,
28. (d) As we know, E = E0 e m 2 0
b15
where A0 = amplitude
– k = spring constant
15 = 45e m
hence total energy is independent of x.
[As no. of oscillations = 15 so t = 15sec] 33. (a) K.E. of simple harmonic motion
b15
1 – 1
=e m = mw2 (a 2 - x 2 )
3 2
P-212 Physics

34. (c) The kinetic energy (K. E.) of particle in SHM is given by,
l
1
K.E = k ( A2 - x 2 ) ; 39. (c) T = 2p
g
2
1 2 When immersed non viscous liquid
Potential energy of particle in SHM is U = kx
2 æ g ö 15 g
Where A = amplitude and k = mw2 amt = ç g - ÷ =
è 16 ø 16
x = displacement from the mean position
At the mean position x = 0
1 2 l l 4
T ¢ = 2p = 2p = T
\ K.E. = kA = Maximum 0net 15 g 15
2 Now
and U = 0 16
35. (b) An elastic wire can be treated as a spring and its spring
40. (c) Net torque due to spring force:
constant.
l
YA é F Dl ù t = -2Kx cos q
k= êQ Y = A l0 úû
2
L ë l
Frequency of oscillation, x=
2
1 k 1 YA
f = =
2p m 2p mL
q
36. (c) y = y0 sin 2 wt
y0 æ 1 - cos 2wt ö
çèQ sin wt =
2
Þy= (1 - cos 2wt ) ÷ø
2 2
y0 - y0 q
Þ y- = cos 2wt
2 2
Þ y = A cos 2wt Kx
æ Kl 2 ö é Kl 2 ù
y0 Þ t=ç ÷ q = -Cq êlet C = ú
\ Amplitude = è 2 ø 2 ûú
2 ëê
Angular velocity = 2w Þ So, frequency of resulting oscillations
ky0 k 2g Kl2
For equilibrium of mass, = mg Þ =
2 m y0 1 C 1 2 = 1 6K
f= =
2 2p I 2p Ml 2 2p M
Also, spring constant k = m(2w)
12
k 2g 1 2g g 41. (b)
Þ 2w = = Þw= =
m y0 2 y0 2 y0
q0 l
1 2
37. (a) Potential energy of spring = kx
2
Here, x = distance of block from mean position,
mM
k = spring constant
Velocity before colision v = 2gl(1 - cos q0 )
1
At mean position, potential energy = kA2 Velocity after colision
2
At equilibrium position, half of the mass of block breaks v1 = 2gl(1 - cos q1)
off, so its potential energy becomes half. Using momentum conservation
mv = MVm – mV1
1 æ 1 2ö 1 2
Remaining energy = ç kA ÷ø = kA ' m 2gl(1 - cos q0 ) = MVm - m 2gl(1 - cos q)
2è2 2
Here, A' = New distance of block from mean position Þ m 2gl { 1 - cos q0 + 1 - cos q1 = MVm}
A
Þ A' = Vm + 2gl(1 - cos q1)
2 and e = 1 =
38. (b) 2gl(1 - cos q0 )
Oscillations P-213

2gl ( 1 - cos q0 - 1 - cos q1 = Vm ) ... (i) Þ w A 2 – x2 = w2 x


Þ A 2 – x 2 = w2 x 2
m 2gl ( 1 - cos q 0 + 1 - cos q1 ) = MVM ... (ii)
Þ 5 2 – 4 2 = w2 ( 4 2 )
Dividing (ii) by (i) we get
3
( 1 - cos q0 + 1 - cos q1 )=M 3 = w × 4 Þ w=
4
( 1 - cos q0 - 1 - cos q1 ) m
\ T = 2p/w =
2 p 8p
=
3/ 4 3
By componendo and dividendo rule 46. (Bonus)
æq ö B0– B )
sin ç 1 ÷
m-M 1 - cos q1 è 2ø
= =
m+M 1 - cos q0 æ q0 ö
sin ç ÷
è 2ø
x
M q0 - q1 q -q a at
Þ = ÞM=m 0 1 equilibriu
m q0 + q1 q0 + q1 x0 B0 = mg

42. (a) Given : y = 5 éësin(3pt) + 3 cos(3pt) ùû


æ pö mg
Þ y = 10sin ç 3pt + 3 ÷
è ø Extra boyant force = rAxg
\ Amplitude = 10 cm B0 + B = mg + ma
2p 2p 2 \ B = ma = rAxg = (pr2rg)x
Time period, T = = = s
w 3p 3
a=
( pr rg ) x
2

g m
43. (a) Angular frequency of pendulum w =
l using, a = w2x
\ relative change in angular frequency
pr 2rg
Þw=
Dw 1 Dg [as length remains constant] m
=
w 2 g W ;7.95 rads–1
47. (c)
D g = 2Aws2 [ws = angular frequency of support and, A =
amplitude]
L/2 L/2
Dw 1 2Aws2
= ´ m m
w 2 g

1 2 ´12 ´10 –2 M
Dw = ´ = 10–3 rad/sec.
2 10 L– X–L
GM 1 C
44. (d) Acceleration due to gravity g = 2 f1 = ...(i)
R 2p 1
2
gp Mp æ R e ö 2
æ1ö 1 1 3C
= ç ÷÷ = 3 ç ÷ = =
g e M e çè R p ø è3ø 3 2 ML2

1 Tp g 1 C
Also T µ Þ = e= 3 f2 =
2p æM M ö
g Te gp L2 ç + ÷ ...(ii)
è 3 2 ø
Þ Tp = 2 3 s As frequency reduces by 80%
f2
45. (c) Velocity, v = w A 2 – x 2 ...(i) \ f2 = 0.8 f1 Þ f = 0.8 ...(iii)
1
acceleration, a = –w2x ...(ii)
Solving equations (i), (ii) & (iii)
and according to question,
m
| v | =| a | Ratio = 0.37
M
P-214 Physics

As frequency reduces by 80%


1 k´2 1
f2 F= = Hz
\ f2 = 0.8 f1 Þ f = 0.8 ...(iii) 2p g 2
1 1
Solving equations (i), (ii) & (iii) 52. (c) Time lost/gained per day = µ Dq ´ 86400 second
2
m 1
Ratio= 0.37 12 = a (40 – q) ´ 86400 .... (i)
M 2
48. (b) As we know, frequency in SHM 1
4 = a (q – 20) ´ 86400 ....(ii)
1 k 2
f= = 1012
2p m 40 – q
where m = mass of one atom On dividing we get, 3 =
q – 20
108 3q – 60 = 40 – q
´10-3 kg
(
Mass of one atom of silver, = 6.02 ´ 1023
) 4q = 100 Þ q = 25°C
53. (b) Washer contact with piston Þ N = 0
1 k Given Amplitude A = 7 cm = 0.07 m.
´ 6.02 ´ 1023 = 1012
2p 108 ´ 10-3 amax = g = w2A
Solving we get, spring constant, The frequency of piston
K = 7.1N/m ω g 1 1000 1
49. (d) Using y = A sin wt f< < < < 1.9 Hz.
2π A 2π 7 2p
a = A sin wt0
b = A sin 2wt0 l
c = A sin 3wt0 54. (c) As we know, time period, T = 2p
g
a + c = A[sin wt0 + A sin 3wt0] = 2A sin 2wt0 cos wt0
When additional mass M is added then
a+c
= 2cos wt 0 l + Dl
b TM = 2p
1 æ a + cö 1 æ a + cö g
Þ w = cos -1 ç Þf = cos -1 ç
t0 è 2b ÷ø 2 pt 0 è 2b ÷ø TM l + Dl
=
T l
50. (b) As we know, Time-period of simple pendulum,
2
Tµ l æ TM ö l + Dl
Þç ÷ =
è T ø l
DT 1 Dl
differentiating both side, =
T 2 l æ TM ö
2
Mg é Mgl ù
Q change in length Dl = r1 – r2 or, ç
è T ø÷ = 1+
AY
êëQ Dl = AY úû
1 r1 - r2
5 ´ 10-4 = 1 éæ TM ö
2 ù A
2 1 \ = êç ÷ - 1 ú
Y êè T ø úû Mg
r1 – r2 = 10 × 10–4 ë
10–3 m = 10–1 cm = 0.1 cm 55. (d) In simple harmonic motion, starting from rest,
At t = 0 , x = A
1 k x = Acoswt ..... (i)
51. (c) Frequency of spring (f ) = = 1 Hz
2p m When t = t , x = A – a
When t = 2 t , x = A –3a
k m = 1 kg From equation (i)
Þ 4p 2 =
m A – a = Acosw t ......(ii)
A – 3a = A cos2w t ....(iii)
If block of mass m = 1 kg is attached then, As cos2w t = 2 cos2 w t – 1...(iv)
From equation (ii), (iii) and (iv)
k = 4p2
2
Now, identical springs are attached in parallel with mass m A - 3a æ A-a ö
= 2ç ÷ -1
= 8 kg. Hence, A è A ø
keq = 2k k
8 kg A - 3a 2 A2 + 2a 2 - 4 Aa - A2
Þ =
A A2
Oscillations P-215

Þ A2 – 3aA = A2 + 2a2 – 4Aa = 4 × 10 + 625 × 1.6 × 10–2 + 1 × 10


Þ 2a2 = aA = 40 + 10 + 10
Þ A = 2a = 60 N
a 1 Mg
Þ = 59. (c) = P0
A 2 A
Now, A – a = A coswt P0V0 g = PV g
A- a Mg = P0A … (1)
Þ cos wt = Let piston is displaced by distance x
A
P0 Ax0 g = PA( x0 - x ) g
1 2p p
Þ cos wt = or t= P0 x0
g
2 T 3 P=
Þ T = 6t ( x0 - x )g
56. (b) From graph it is clear that when Piston
L = 1m, T2 = 4s2
As we know,
x
L
T = 2p
g x0 Cylinder
containing
4 p2 L ideal gas
Þ g=
T2 æ P xg ö
Mg - ç 0 0 g ÷ A = Frestoring
2
æ 22 ö 1 æ 22 ö
2 ç ( x - x) ÷
= 4´ç ÷ ´ = ç ÷ è 0 ø
è 7 ø 4 è 7 ø æ x0g ö
484 P0 A ç1 - ÷ = Frestoring [ x - x » x ]
\ g= = 9.87m / s 2 ç ( x - x) ÷ g 0 0
49 è 0 ø
57. (d) As we know, gP Ax
F=- 0
x = x0 e–bt/2m x0
From question, \ Frequency with which piston executes SHM.
40b
- ....(i) 1 gP0 A 1 gP0 A2
8 = 10e 2m f = =
2p x0 M 2p MV0
bt
-
Similarly, 5 = 10e 2m ....(ii) 60. (c) As F = -kx
Solving eqns (i) and (ii) we get 61. (a) Let T1 and T2 be the time period of the two pendulums
t @ 142 s 1 4
T1 = 2p and T2 = 2p
58. (c) Mass of bigger body M = 4 kg g g
Mass of smaller body m = 1 kg
Smaller mass (m = 1 kg) executes S.H.M of angular As l1 < l 2 therefore T1 < T2
frequency w = 25 rad/s Let at t = 0 they start swinging together. Since their time
Amplitude x = 1.6 cm = 1.6 × 10–2 periods are different, the swinging will not be in unison
As we know, always. Only when number of completed oscillations
differ by an integer, the two pendulums will again begin
m to swing together
T = 2p
K Let longer length pendulum complete n oscillation and
shorter length pendulum complete (n + 1) oscillation. For
2p m unison swinging
or, = 2p
w K (n + 1)T1 = nT2
1 1
or, = [Q m = 1kg; w = 25 rad / s ] (n + 1) ´ 2p
l
= (n) ´ 2p
4
25 K
g g
or, K = 625 Nm–1.
The maximum force exerted by the system on the floor Þn=1
= Mg + Kx + mg \n+1=1+1=2
P-216 Physics

62. (d) The equation of motion for the pendulum, for rg ld


damped harmonic motion \ w= Þ T = 2p
dl rg
F = – kx - bv
65. (a) For an SHM, the acceleration
Þ ma + kx + bv = 0
a = -w2 x where w 2 is a constant.
Þ d 2x dx
m 2 + kx + b =0 –4p 2 x aT –4p2
dt dt a= Þ =
d2 x k b dx
T2 x T
Þ + x+ =0 aT
dt 2 m m dt The time period T is also constant. Therefore, is a
x
constant.
d2 x b dx k
Þ + + x =0 … (1) 66. (a) The two springs are in parallel.
2 m dt m
dt \ Effective spring constant,
Let x = elt is the solution of the equation (1) k = k1 + k2
Initial frequency of oscillation is given by
dx d2 x
= lelt Þ = l 2 elt
dt dt 2 1 k1 + k 2
v = ....(i)
Substituting in the equation (1) 2p m
b k When both k1 and k2 are made four times their original
l 2 elt + l elt + elt = 0
m m values, the new frequency is given by
b k
l2 + l + = 0 1 4 k1 + 4 k 2
m m v' =
2p m
b b2 k 4(k1 + 4k 2 ) æ 1 k1 + k2 ö
- ± -4 1
m m 2 m -b ± b2 - 4km = = 2ç ÷÷ = 2v
l= = 2p m ç 2p m
è ø
2 2m
67. (b) Here, Displacement, x = 2 × 10–2 cos p t
Solving the equation (1) for x, we have
-b
Velocity is given by
t
x= e m
2 dx
v= = 2 × 10–2 p sin p t
k +b dt
w = w0 2 - l 2 where w0 = , l =
m 2 For the first time, the when velocity becomes maximum,
1 2 sin p t = 1
The average life = =
l b Þ sin p t = sin p
63. (a) 2
64. (a) Let the cube be at a depth x from the equilibrium p 1
position. Þ pt = or,, t = = 0.5 sec.
2 2
Force acting on the cube = up thrust on the portion of
length x. 68. (b) When plugged hole near the bottom of the oscillating
bob gets suddenly unplugged, centre of mass of
F = – rl 2 xg [\ mass density X volume ] ....(i) combination of liquid and hollow portion (at position l ),
Clearly F µ – x, Hence it is a SHM. first goes down ( to l + D l) and when total water is drained
Equation of SHM is F = –kx ....(ii)
Comparing equation (i) and (ii) we have out, centre of mass regain its original position (to l ),
k = rl2g l
m Time period, T = 2p
g
Now, Time period, T = 2p
k \ ‘T ’ first increases and then decreases to original value.
3
l d
Þ T = 2p
rl 2 g
ld
= 2p
rg
c
Comparing the above equation with
a = –w2x, we get
Oscillations P-217

l m 25
70. (a) Time period, t = 2p ; Þ 1+ =
g eff M 9
l m 25 16
In air, t 0 = 2p Þ = -1 =
g M 9 9
Buoyant l
force 1000 Vg 73. (d) Time period, T = 2p
g

New length, l ' = l + 2 1 % o f l


4
´ 1000 Vg l' = l + 0.21 l
3
Weight Þ l' = 1.21 l
æ4 ö 1000
Net force = ç - 1 ÷ ´ 1000 Vg = Vg 1.21l
è3 ø 3 T ' = 2p
g
1000 Vg g
geff = =
4 T '- T
3 ´ ´1000V 4 % increase in length = ´100
3 T
l l
\ t = 2p = 2 ´ 2p 1.21l - l
g /4 g =
l
´ 100 = ( )
1.21 - 1 ´ 100
t = 2t0
= (1.1 - 1) ´ 100 = 10%
m
71. (b) Time period for first spring, t1 = 2p , 74. (b) Let k be the spring constant of the original spring.
k1
m
m Time period T = 2p where m is the mas s of oscillating
Time period for second spring, t 2 = 2p k
k2 body.
k1k 2 When the spring is cut into n equal parts, the spring
Force constant of the series combination keff =
kl + k 2 constant of one part becomes nk. Therefore the new time
\ Time period of oscillation for series combination period,
m T
m( k l + k 2 ) T ' = 2p =
T = 2p nk n
k1k 2
l
75. (b) The time period T = 2p where l = distance
2 2 g
m m t2 t1
\ T = 2p + = 2p + between the point of suspension and the centre of mass of
k 2 k1 2 2
(2p) (2p) the child.
As the child stands up, her centre of mass is raised. The
Þ T 2 = t12 + t 22 distance between point of suspension and centre of mass
where x is the displacement from the mean position decreases ie length l decreases.
72. (c) With mass M, the time period of the spring. \ l¢ < l
M
T = 2p \ T ¢ < T i.e., the period decreases.
k
With mass M + m, the time period becomes, point of suspension

M + m 5T
T ' = 2p = l'
k 3 l
M +m 5 M
\ 2p = ´ 2p
k 3 k
25 Case (ii) child standing Case (i) child sitting
Þ M +m = ´M
9
P-218 Physics

Putting value of M, m, u and K we get amplitude


point of suspension
1 75 1
A= =
15 1 3
l' 79. (b) The change in time period compared to the undamped
l
oscillator increases by 8%.
bt
-
2m
80. (c) Q A = A 0e
Case (ii) child standing Case (i) child sitting
(where, A0 = maximum amplitude)
76. (c) Time of half the amplitude is = 2s According to the questions, after 5 second,
Using, A = A0e–kt b(5)
-
0.9A 0 = A 0 e 2m … (i)
A0
= Ae e - k ´2 ...(i) After 10 more second,
2
b(15)
-
A = A0 e 2m …(ii)
A0
and = Ae e- kt ...(ii)
1000 From eqns (i) and (ii)
Dividing (i) by (ii) and solving, we get A = 0.729 A0
\ a = 0.729
t = 20 s
81. (a)
77. (b) Amplitude of vibration at time t = 0 is given by
82. (d) When the pendulum is oscillating over a current
A = A0e –0.1× 0 = 1 × A0 = A0 carrying coil, and when the direction of oscillating
pendulum bob is opposite to the direction of current. Its
instantaneous acceleration increases.
A0
also at t = t, if A =
2 l
Hence time period T < 2p
g
1 and damping is larger than in air alone due energy dissipa-
Þ = e –0.1t tion.
2
83. (c) As energy µ ( Amplitude)2, the maximum for both
of them occurs at the same frequency and this is only
t = 10 ln 2 ;7s possible in case of resonance.
78. (b) In first collision mu momentum will be imparted to In resonance state w1 = w 2
system, in second collision when momentum of (M + m) is in 84. (b) Equation of displacement in forced oscillation is given
opposite direction mu momentum of particle will make its by
momentum zero.
F0
On 13th collision, m ® M + 12 ; M + 13m ® V y= 2 2 2
m (w 0 - w )
mu u
mu = (M + 13m)v Þ v = = F0
M + 13m 15 =
u K m (w 0 2 - w 2 )
v = wA Þ = ´A Here damping effect is considered to be zero
15 M - 13m
1
\x µ
2 2
m (w 0 - w )
14
Waves
4. A small speaker delivers 2 W of audio output. At what
Basic of Mechanical Waves, distance from the speaker will one detect 120 dB intensity
TOPIC 1 Progressive and Stationary sound ? [Given reference intensity of sound as 10–12W/m2]
Waves [12 April 2019 II]
(a) 40 cm (b) 20 cm (c) 10 cm (d) 30 cm
1. Assume that the displacement (s) of air is proportional 5. The pressure wave,
P = 0.01 sin[1000t – 3x] Nm–2, corresponds to the sound
to the pressure difference (Dp) created by a sound wave.
produced by a vibrating blade on a day when atmospheric
Displacement (s) further depends on the speed of sound temperature is 0°C. On some other day when temperature
(v), density of air (r) and the frequency ( f ). If Dp ~ is T, the speed of sound produced by the same blade and
10Pa, v ~ 300 m/s, r ~ 1 kg/m3 and f ~ 1000 Hz, then s at the same frequency is found to be 336 ms–1. Approximate
will be of the order of (take the multiplicative constant value of T is : [9 April 2019 I]
(a) 4°C (b) 11°C (c) 12°C (d) 15°C
to be 1) [Sep. 05, 2020 (I)]
6. A travelling harmonic wave is represented by the equa-
3 tion y(x, t)=10–3 sin(50t+2x), where x and y are in meter and
(a) mm (b) 10 mm t is in seconds. Which of the following is a correct state-
100
1 ment about the wave? [12 Jan. 2019 I]
(c) mm (d) 1 mm (a) The wave is propagating along the negative x-axis
10 with speed 25 ms–1.
2. For a transverse wave travelling along a straight line, the
distance between two peaks (crests) is 5 m, while the (b) The wave is propagating along the positive x-axis with
distance between one crest and one trough is 1.5 m. The speed 100 ms–1.
possible wavelengths (in m) of the waves are : (c) The wave is propagating along the positive x-axis with
speed 25 ms–1.
[Sep. 04, 2020 (I)] (d) The wave is propagating along the negative x-axis
1 1 1 with speed 100 ms–1.
(a) 1, 3, 5, ..... (b) , , , .......
1 3 5 7. A transverse wave is represented by
1 1 1 10 æ 2p 2p ö
(c) 1, 2, 3, ..... (d) , , , ....... y = sin ç t - x÷
2 4 6 p è T l ø
3. A progressive wave travelling along the positive x-direction For what value of the wavelength the wave velocity is
is represented by y(x,t) = Asin (kx – wt + f). Its snapshot at twice the maximum particle velocity?
t = 0 is given in the figure. [12 April 2019 I] [Online April 9, 2014]
(a) 40 cm (b) 20 cm (c) 10 cm (d) 60 cm
8. In a transverse wave the distance between a crest and
neighbouring trough at the same instant is 4.0 cm and the
distance between a crest and trough at the same place is
1.0 cm. The next crest appears at the same place after a
time interval of 0.4s. The maximum speed of the vibrating
particles in the medium is : [Online April 25, 2013]
3p 5p
For this wave, the phase f is : (a) cm/s (b) cm/s
2 2
p p p
(a) - (b) p (c) 0 (d) (c) cm/s (d) 2p cm/s
2 2 2
P-220 Physics

9. When two sound waves travel in the same direction in a 16. When temperature increases, the frequency of a tuning
medium, the displacements of a particle located at 'x' at fork [2002]
time ‘t’ is given by : (a) increases
y1 = 0.05 cos (0.50 px – 100 pt) (b) decreases
y2 = 0.05 cos (0.46 px – 92 pt) (c) remains same
where y1, y2 and x are in meters and t in seconds. The (d) increases or decreases depending on the material
speed of sound in the medium is : [Online April 9, 2013]
TOPIC 2 Vibration of String and Organ
(a) 92 m/s (b) 200 m/s (c) 100 m/s (d) 332 m/s
10. The disturbance y (x, t) of a wave propagating in the Pipe
1
positive x-direction is given by y = at time t = 0
1 + x2 17. In a resonance tube experiment when the tube is filled
1 with water up to a height of 17.0 cm from bottom, it reso-
and by y = at t = 2 s, where x and y are in
é1 + ( x - 1) 2 ù nates with a given tuning fork. When the water level is
ë û raised the next resonance with the same tuning fork oc-
meters. The shape of the wave disturbance does not change curs at a height of 24.5 cm. If the velocity of sound in air
during the propagation. The velocity of wave in m/s is is 330 m/s, the tuning fork frequency is :
[Online May 26, 2012] [Sep. 05, 2020 (I)]
(a) 2.0 (b) 4.0 (c) 0.5 (d) 1.0 (a) 2200 Hz (b) 550 Hz
11. The transverse displacement y (x, t) of a wave is given by (c) 1100 Hz (d) 3300 Hz
y( x, t ) = e
(
- ax 2 + bt 2 + 2 ab ) xt ). 18. A uniform thin rope of length 12 m and mass 6 kg hangs
This represents a: [2011] vertically from a rigid support and a block of mass 2 kg
is attached to its free end. A transverse short wave-train
b
(a) wave moving in – x direction with speed of wavelength 6 cm is produced at the lower end of the
a rope. What is the wavelength of the wavetrain (in cm)
(b) standing wave of frequency b when it reaches the top of the rope ?[Sep. 03, 2020 (I)]
1
(c) standing wave of frequency (a) 3 (b) 6
b (c) 12 (d) 9
a 19. Two identical strings X and Z made of same material have
(d) wave moving in + x direction speed
b tension T X an d T Z in them. If their fundamental
12. A wave travelling along the x-axis is described by the frequencies are 450 Hz and 300 Hz, respectively, then
equation y(x, t) = 0.005 cos (a x – bt). If the wavelength the ratio TX/TZ is: [Sep. 02, 2020 (I)]
and the time period of the wave are 0.08 m and 2.0s, (a) 2.25 (b) 0.44
respectively, then a and b in appropriate units are [2008] (c) 1.25 (d) 1.5
0.08 2.0 20. A wire of density 9 × 10–3 kg cm–3 is stretched between
(a) a = 25.00 p , b = p (b) a = ,b =
two clamps 1 m apart. The resulting strain in the wire is
p p
0.04 1.0 p 4.9 × 10–4. The lowest frequency of the transverse
(c) a = ,b = (d) a = 12.50p, b = vibrations in the wire is
p p 2.0
13. A sound absorber attenuates the sound level by 20 dB. (Young’s modulus of wire Y = 9 × 1010 Nm–2), (to the nearest
The intensity decreases by a factor of [2007] integer), ___________. [Sep. 02, 2020 (II)]
(a) 100 (b) 1000 (c) 10000 (d) 10 21. A one metre long (both ends open) organ pipe is kept in a
14. The displacement y of a particle in a medium can be gas that has double the density of air at STP. Assuming
the speed of sound in air at STP is 300 m/s, the frequency
-6 æ pö difference between the fundamental and second harmonic
expressed as, y = 10 sin ç100t + 20 x + ÷ m where t is
è 4ø of this pipe is ______ Hz. [NA 8 Jan. 2020 (I)]
22. A transverse wave travels on a taut steel wire with a
in second and x in meter. The speed of the wave is [2004]
velocity of v when tension in it is 2.06 ´ l04 N. When
(a) 20 m/s (b) 5 m/s
the tension is changed to T, the velocity changed to v/2.
(c) 2000 m/s (d) 5p m/s
The value of T is close to: [8 Jan. 2020 (II)]
15. The displacement y of a wave travelling in the x -direction
(a) 2.50 ´ l04 N (b) 5.15 ´ l03 N
æ pö (c) 30.5 ´ l04 N (d) 10.2 ´ l02 N
is given by y = 10 - 4 sin ç 600 t - 2 x + ÷ metres
è 3ø 23. Speed of a transverse wave on a straight wire (mass 6.0
g, length 60 cm and area of cross-section 1.0 mm2) is 90
where x is expressed in metres and t in seconds. The speed ms–1. If the Young’s modulus of wire is 16 ´ l011 Nm–2 the
of the wave - motion, in ms–1 , is [2003] extension of wire over its natural length is:
(a) 300 (b) 600 (c) 1200 (d) 200 [7 Jan. 2020 (I)]
Waves P-221

(a) 0.03 mm (b) 0.02 mm 32. A string of length 1 m and mass 5 g is fixed at both
(c) 0.04 mm (d) 0.01 mm ends. The tension in the string is 8.0 N. The string is
set into vibration using an external vibrator of frequency
24. Equation of travelling wave on a stretched string of linear
100 Hz. The separation between successive nodes on
density 5 g/m is y = 0.03 sin (450 t – 9x) where distance and
the string is close to: [10 Jan. 2019 (I)]
time are measured in SI units. The tension in the string is: (a) 10.0 cm (b) 33.3 cm (c) 16.6 cm (d) 20.0 cm
[11 Jan 2019 (I)] 33. A granite rod of 60 cm length is clamped at its middle point
(a) 10 N (b) 7.5 N (c) 12.5 N (d) 5 N and is set into longitudinal vibrations. The density of
25. A heavy ball of mass M is suspended from the ceiling of granite is 2.7 × 103 kg/m3 and its Young's modulus is
a car by a light string of mass m (m<<M). When the car is 9.27×1010 Pa.
at rest, the speeed of transverse waves in the string is 60 What will be the fundamental frequency of the longitudinal
ms–1. when the car has acceleration a, the wave-speed vibrations? [2018]
increases to 60.5 ms–1. The value of a, in terms of (a) 5 kHz (b) 2.5 kHz (c) 10 kHz (d) 7.5 kHz
gravitational acceleration g, is closest to: [9 Jan. 2019 (I)] 34. The end correction of a resonance column is 1cm. If the
shortest length resonating with the tuning fork is 10cm,
g g g g the next resonating length should be
(a) (b) (c) (d)
30 5 10 20 [Online April 16, 2018]
26. A wire of length L and mass per unit length 6.0 × 10–3 kgm–1 (a) 32cm (b) 40cm (c) 28cm (d) 36cm
is put under tension of 540 N. Two consecutive 35. Two wires W1 and W2 have the same radius r and
frequencies that it resonates at are: 420 Hz and 490 Hz. respective densities r1 and r2 such that r2 = 4r1. They
Then L in meters is: [9 Jan. 2020 (II)] are joined together at the point O, as shown in the figure.
(a) 2.1 m (b) 1.1 m The combination is used as a sonometer wire and kept
(c) 8.1 m (d) 5.1 m under tension T. The point O is midway between the two
27. A tuning fork of frequency 480 Hz is used in an experiment bridges. When a stationary waves is set up in the
for measuring speed of sound (v) in air by resonance composite wire, the joint is found to be a node. The ratio
tube method. Resonance is observed to occur at two of the number of antinodes formed in W1 to W2 is :
successive lengths of the air column, l1 = 30 cm and l2 = 70 [Online April 8, 2017]
cm. Then, v is equal to : [12 April 2019 (II)]
r1 r2
(a) 332 ms–1 (b) 384 ms–1
(c) 338 ms–1 (d) 379 ms–1 W1 O W 2
28. A string 2.0 m long and fixed at its ends is driven by a 240
Hz vibrator. The string vibrates in its third harmonic mode. (a) 1 : 1 (b) 1 : 2 (c) 1 : 3 (d) 4 : 1
The speed of the wave and its fundamental frequency is: 36. A uniform string of length 20 m is suspended from a rigid
[9 April 2019 (II)] support. A short wave pulse is introduced at its lowest
(a) 180 m/s, 80 Hz (b) 320 m/s, 80 Hz end. It starts moving up the string. The time taken to reach
(c) 320 m/s, 120 Hz (d) 180 m/s, 120 Hz the supports is : [2016]
29. A string is clamped at both the ends and it is vibrating in (take g = 10 ms–2)
its 4th harmonic. The equation of the stationary wave is Y
= 0.3 sin(0.157x) cos(200At). The length of the string is: (a) 2 2s (b) 2 s (c) 2p 2 s (d)2 s
(All quantities are in SI units.) [9 April 2019 (I)] 37. A pipe open at both ends has a fundamental frequency f
(a) 20 m (b) 80 m (c) 40 m (d) 60 m in air. The pipe is dipped vertically in water so that half of
30. A wire of length 2L, is made by joining two wires A and B it is in water. The fundamental frequency of the air column
of same length but different radii r and 2r and made of the is now : [2016]
same material. It is vibrating at a frequency such that the f 3f
(a) 2f (b) f (c) (d)
joint of the two wires forms a node. If the number of 2 4
antinodes in wire A is p and that in B is q then the ratio 38. A pipe of length 85 cm is closed from one end. Find the
p : q is : [8 April 2019 (I)] number of possible natural oscillations of air column in
the pipe whose frequencies lie below 1250 Hz. The velocity
of sound in air is 340 m/s. [2014]
(a) 12 (b) 8 (c) 6 (d) 4
(a) 3 : 5 (b) 4 : 9 (c) 1 : 2 (d) 1: 4
31. A closed organ pipe has a fundamental frequency of 1.5 39. The total length of a sonometer wire between fixed ends is
kHz. The number of overtones that can be distinctly 110 cm. Two bridges are placed to divide the length of wire
heard by a person with this organ pipe will be: (Assume in ratio 6 : 3 : 2. The tension in the wire is 400 N and the
that the highest frequency a person can hear is 20,000 Hz) mass per unit length is 0.01 kg/m. What is the minimum
[10 Jan. 2019 (I)]
(a) 6 (b) 4 common frequency with which three parts can vibrate?
(c) 7 (d) 5 [Online April 19, 2014]
P-222 Physics

(a) 1100 Hz (b) 1000 Hz (a) 105 Hz (b) 1.05 Hz


(c) 166 Hz (d) 100 Hz (c) 1050 Hz (d) 10.5 Hz
40. A sonometer wire of length 1.5 m is made of steel. The 48. Tube A has both ends open while tube B has one end
tension in it produces an elastic strain of 1%. What is the closed, otherwise they are identical. The ratio of
fundamental frequency of steel if density and elasticity of fundamental frequency of tube A and B is [2002]
steel are 7.7 × 103 kg/m3 and 2.2 × 1011 N/m2 respectively? (a) 1 : 2 (b) 1 : 4 (c) 2 : 1 (d) 4 : 1
(a) 188.5 Hz (b) 178.2 Hz [2013] 49. A wave y = a sin(wt–kx) on a string meets with another
(c) 200.5 Hz (d) 770 Hz wave producing a node at x = 0. Then the equation of the
41. A sonometer wire of length 114 cm is fixed at both the ends. unknown wave is [2002]
Where should the two bridges be placed so as to divide the (a) y = a sin( w t + kx)
wire into three segments whose fundamental frequencies (b) y = –a sin( w t + kx)
are in the ratio 1 : 3 : 4 ? [Online April 23, 2013] (c) y = a sin( w t – kx)
(a) At 36 cm and 84 cm from one end (d) y = –a sin( w t – kx)
(b) At 24 cm and 72 cm from one end
(c) At 48 cm and 96 cm from one end Beats, Interference and
(d) At 72 cm and 96 cm from one end TOPIC 3
Superposition of Waves
42. A cylindrical tube, open at both ends, has a fundamental
frequency f in air. The tube is dipped vertically in water so 50. There harmonic waves having equal frequency n and same
that half of it is in water. The fundamental frequency of the
air-column is now : [2012] p p
and - respectively..
intensity I0, have phase angles 0,
(a) f (b) f / 2 (c) 3 f /4 (d) 2 f 4 4
43. An air column in a pipe, which is closed at one end, will When they are superimposed the intensity of the resultant
be in resonance wtih a vibrating tuning fork of frequency wave is close to: [9 Jan. 2020 I]
264 Hz if the length of the column in cm is (velocity of (a) 5.8 I0 (b) 0.2 I0 (c) 3 I0 (d) I0
sound = 330 m/s) [Online May 26, 2012] 51. The correct figure that shows, schematically, the wave
(a) 125.00 (b) 93.75 (c) 62.50 (d) 187.50 pattern produced by superposition of two waves of
44. A uniform tube of length 60.5 cm is held vertically with frequencies 9 Hz and 11 Hz is : [10 April 2019 II]
its lower end dipped in water. A sound source of frequency
500 Hz sends sound waves into the tube. When the length
of tube above water is 16 cm and again when it is 50 cm,
(a)
the tube resonates with the source of sound. Two lowest
frequencies (in Hz), to which tube will resonate when it
is taken out of water, are (approximately).
[Online May 19, 2012]
(a) 281, 562 (b) 281, 843
(c) 276, 552 (d) 272, 544 (b)
45. The equation of a wave on a string of linear mass density
0.04 kg m–1 is given by
é æ t x öù
y = 0.02(m) sin ê2p ç - ÷ú .
ë è 0.04(s ) 0.50(m) ø û
(c)
The tension in the string is [2010]
(a) 4.0 N (b) 12.5 N (c) 0.5 N (d) 6.25 N
46. While measuring the speed of sound by performing a
resonance column experiment, a student gets the first
resonance condition at a column length of 18 cm during
winter. Repeating the same experiment during summer, (d)
she measures the column length to be x cm for the second
resonance. Then [2008]
(a) 18 > x (b) x > 54 52. A resonance tube is old and has jagged end. It is still used
(c) 54 > x > 36 (d) 36 > x > 18 in the laboratory to determine velocity of sound in air. A
47. A string is stretched between fixed points separated by tuning fork of frequency 512 Hz produces first resonance
75.0 cm. It is observed to have resonant frequencies of when the tube is filled with water to a mark 11 cm below
420 Hz and 315 Hz. There are no other resonant a reference mark, near the open end of the tube. The
frequencies between these two. Then, the lowest resonant experiment is repeated with another fork of frequency
frequency for this string is [2006] 256 Hz which produces first resonance when water
Waves P-223

reaches a mark 27 cm below the reference mark. The 58. A travelling wave represented by
velocity of sound in air, obtained in the experiment, is y = A sin (wt – kx) is superimposed on another wave
close to: [12 Jan. 2019 II] represented by y = A sin (wt + kx). The resultant is
(a) 322 ms–1 (b) 341 ms–1 (a) A wave travelling along + x direction [2011 RS]
(c) 335 ms–1 (d) 328 ms–1
(b) A wave travelling along – x direction
53. A tuning fork vibrates with frequency 256 Hz and gives
one beat per second with the third normal mode of (c) A standing wave having nodes at
vibration of an open pipe. What is the length of the pipe? nl
(Speed of sound of air is 340 ms–1) x= , n = 0,1, 2....
2
[Online April 15, 2018] (d) A standing wave having nodes at
(a) 190 cm (b) 180 cm æ 1ö l
x = ç n + ÷ ; n = 0,1, 2....
(c) 220 cm (d) 200 cm è 2ø 2
54. 5 beats/ second are heard when a turning fork is sounded 59. Statement - 1 : Two longitudinal waves given by
with a sonometer wire under tension, when the length of equations : y1(x, t) = 2a sin (wt – kx) and y2(x, t) = a
the sonometer wire is either 0.95m or 1m . The frequency sin (2wt - 2kx) will have equal intensity.. [2011 RS]
of the fork will be: [Online April 15, 2018] Statement - 2 : Intensity of waves of given frequency in
(a) 195Hz (b) 251Hz (c) 150Hz (d) 300Hz same medium is proportional to square of amplitude only.
55. A standing wave is formed by the superposition of two (a) Statement-1 is true, statement-2 is false.
waves travelling in opposite directions. The transverse (b) Statement-1 is true, statement-2 is true, statement-
2 is the correct explanation of statement-1
displacement is given by
(c) Statement-1 is true, statement-2 is true, statement-
æ 5p ö 2 is not the correct explanation of statement-1
y(x, t) = 0.5 sin ç x ÷ cos(200 pt). (d) Statement-1 is false, statement-2 is true.
è 4 ø 60. Three sound waves of equal amplitudes have frequencies
What is the speed of the travelling wave moving in the (n –1), n, (n + 1). They superpose to give beats. The number
positive x direction ? of beats produced per second will be : [2009]
(x and t are in meter and second, respectively.) (a) 3 (b) 2 (c) 1 (d) 4
[Online April 9, 2017] 61. When two tuning forks (fork 1 and fork 2) are sounded
simultaneously, 4 beats per second are heard. Now, some
(a) 160 m/s (b) 90 m/s (c) 180 m/s (d) 120 m/s
tape is attached on the prong of the fork 2. When the
56. A wave represented by the equation y1 = acos (kx – wt) is tuning forks are sounded again, 6 beats per second are
superimposed with another wave to form a stationary wave heard. If the frequency of fork 1 is 200 Hz, then what was
such that the point x – 0 is node. The equation for the the original frequency of fork 2? [2005]
other wave is [Online May 12, 2012] (a) 202 Hz (b) 200 Hz (c) 204 Hz (d) 196 Hz
(a) a cos (kx – wt + p) (b) a cos (kx + wt + p) 62. A tuning fork of known frequency 256 Hz makes 5 beats
per second with the vibrating string of a piano. The beat
æ pö æ pö frequency decreases to 2 beats per second when the
(c) a cos ç kx + wt + ÷ (d) a cos ç kx - wt + ÷
è 2ø è 2ø tension in the piano string is slightly increased. The
57. Following are expressions for four plane simple harmonic frequency of the piano string before increasing the tension
waves [Online May 7, 2012] was [2003]
æ x ö (a) (256 + 2) Hz (b) (256 – 2) Hz
(i) y1 = A cos 2p ç n1t + ÷ (c) (256 – 5) Hz (d) (256 + 5) Hz
è l1 ø
63. A tuning fork arrangement (pair) produces 4 beats/sec with
æ x ö one fork of frequency 288 cps. A little wax is placed on the
(ii) y2 = A cos 2p ç n1t + + p÷
è l1 ø unknown fork and it then produces 2 beats/sec. The
frequency of the unknown fork is [2002]
æ xö
(iii) y3 = A cos 2p ç n2t + ÷ (a) 286 cps (b) 292 cps (c) 294 cps (d) 288 cps
è l2 ø
æ xö Musical Sound and Doppler's
(iv) y4 = A cos 2p ç n2t - ÷
TOPIC 4
è l2 ø Effect
The pairs of waves which will produce destructive 64. A sound source S is moving along a straight track with
interference and stationary waves respectively in a speed v, and is emitting sound of frequency vo (see
medium, are figure). An observer is standing at a finite distance, at the
(a) (iii, iv), (i, ii) (b) (i, iii), (ii, iv) point O, from the track. The time variation of frequency
(c) (i, iv), (ii, iii) (d) (i, ii), (iii, iv) heard by the observer is best represented by:
[Sep. 06, 2020 (I)]
P-224 Physics

(t0 represents the instant when the distance between the 10 beats/s. The velocity of sound is 330 m/s. Then u
source and observer is minimum) equals: [12 April 2019 II]
v
v (a) 5.5 m/s (b) 15.0 m/s
vo
(c) 2.5 m/s (d) 10.0 m/s
(a) vo (b) 71. A stationary source emits sounds waves of frequency
500 Hz. Two observers moving along a line passing
through the source detect sound to be of frequencies
t0 t
t0 t 4801 Hz and 530 Hz. Their respective speeds are, in
v v
ms–1,
vo
(Given speed of sound = 300 m/s) [10 April 2019 I]
vo (a) 12, 16 (b) 12, 18 (c) 16, 14 (d) 8, 18
(c) (d) 72. A source of sound S is moving with a velocity of 50 m/s
towards a stationary observer. The observer measures
t0 t t0 t the frequency of the source as 1000 Hz. What will be the
65. A driver in a car, approaching a vertical wall notices that apparent frequency of the source when it is moving away
the frequency of his car horn, has changed from 440 Hz to from the observer after crossing him? (Take velocity of
480 Hz, when it gets reflected from the wall. If the speed of sound in air 350 m/s) [10 April 2019 II]
sound in air is 345 m/s, then the speed of the car is : (a) 750 Hz (b) 857 Hz (c) 1143 Hz (d) 807 Hz
[Sep. 05, 2020 (II)] 73. Two cars A and B are moving away from each other in
(a) 54 km/hr (b) 36 km/hr opposite directions. Both the cars are moving with a speed
(c) 18 km/hr (d) 24 km/hr of 20 ms–1 with respect to the ground. If an observer in car A
66. The driver of a bus approaching a big wall notices that the detects a frequency 2000 Hz of the sound coming from car B,
frequency of his bus’s horn changes from 420 Hz to 490 Hz what is the natural frequency of the sound source in car B?
when he hears it after it gets reflected from the wall. Find (speed of sound in air = 340 ms–1) [9 April 2019 II]
the speed of the bus if speed of the sound is 330 ms–1. (a) 2250 Hz (b) 2060 Hz
[Sep. 04, 2020 (II)] (c) 2300 Hz (d) 2150 Hz
(a) 91 kmh–1 (b) 81 kmh–1 74. A train moves towards a stationary observer with speed
(c) 61 kmh–1 (d) 71 kmh–1 34 m/s. The train sounds a whistle and its frequency
67. Magnetic materials used for making permanent magnets registered by the observer is f1 If the speed of the train
(P) and magnets in a transformer (T) have different is reduced to 17 m/s, the frequency registered is f2 If
properties of the following, which property best matches speed of sound is 340 m/s, then the ratio f1/f2 is:
for the type of magnet required? [Sep. 02, 2020 (I)] [10 Jan. 2019 I]
(a) T : Large retentivity, small coercivity (a) 18/17 (b) 19/18 (c) 20/19 (d) 21/20
(b) P : Small retentivity, large coercivity 75. A musician using an open flute of length 50 cm
(c) T : Large retentivity, large coercivity produces second harmonic sound waves. A person runs
(d) P : Large retentivity, large coercivity towards the musician from another end of a hall at a
68. A stationary observer receives sound from two identical speed of 10 km/h. If the wave speed is 330 m/s, the
tuning forks, one of which approaches and the other one frequency heard by the running person shall be close to:
recedes with the same speed (much less than the speed [9 Jan. 2019 II]
of sound). The observer hears 2 beats/sec. The oscillation (a) 666 Hz (b) 753 Hz
frequency of each tuning fork is v0 = 1400 Hz and the (c) 500 Hz (d) 333 Hz
velocity of sound in air is 350 m/s. The speed of each 76. Two sitar strings, A and B, playing the note 'Dha' are slightly
tuning fork is close to: [7 Jan. 2020 I] out of tune and produce beats and frequency 5 Hz. The
tension of the string B is slightly increased and the beat
1 1 1 frequency is found to decrease by 3 Hz . If the frequency
(a) m/s (b) 1m/s (c) m/s (d) 8 m/s
2 4 of A is 425 Hz, the original frequency of B is
69. A submarine (A) travelling at 18 km/hr is being chased [Online April 16, 2018]
along the line of its velocity by another submarine (B) (a) 430 Hz (b) 428 Hz (c) 422 Hz (d) 420 Hz
travelling at 27 km/hr. B sends a sonar signal of 500 Hz to 77. A toy–car, blowing its horm, is moving with a steady speed
detect A and receives a reflected sound of frequency v. of 5 m/s, away from a wall. An observer, towards whom the
The value of v is close to : [12 April 2019 I] toy car is moving, is able to hear 5 beats per second. If the
(Speed of sound in water = 1500 ms–1) velocity of sound in air is 340 m/s, the frequency of the
(a) 504 Hz (b) 507 Hz horn of the toy car is close to : [Online April 10, 2016]
(c) 499 Hz (d) 502 Hz (a) 680 Hz (b) 510 Hz (c) 340 Hz (d) 170 Hz
70. Two sources of sound S1 and S2 produce sound waves of 78. Two engines pass each other moving in opposite
same frequency 660 Hz. A listener is moving from source directions with uniform speed of 30 m/s. One of them is
S1 towards S2 with a constant speed u m/s and he hears blowing a whistle of frequency 540 Hz. Calculate the
Waves P-225

frequency heard by driver of second engine before they (a) 2 (b) 4


pass each other. Speed of sound is 330 m/sec: (c) 8 (d) 10
[Online April 9, 2016] 84. A and B are two sources generating sound waves. A
(a) 450 Hz (b) 540 Hz (c) 270 Hz (d) 648 Hz listener is situated at C. The frequency of the source at A
79. A train is moving on a straight track with speed 20 ms–1. is 500 Hz. A, now, moves towards C with a speed 4 m/s.
It is blowing its whistle at the frequency of 1000 Hz. The The number of beats heard at C is 6. When A moves away
percentage change in the frequency heard by a person from C with speed 4 m/s, the number of beats heard at C
standing near the track as the train passes him is (speed is 18. The speed of sound is 340 m/s. The frequency of
of sound = 320 ms–1) close to : [2015] the source at B is : [Online April 22, 2013]
(a) 18% (b) 24% (c) 6% (d) 12% A C B
80. A source of sound emits sound waves at frequency f0. It
is moving towards an observer with fixed speed
(a) 500 Hz (b) 506 Hz (c) 512 Hz (d) 494 Hz
vs (vs < v, where v is the speed of sound in air). If the
85. An engine approaches a hill with a constant speed. When
observer were to move towards the source with speed
it is at a distance of 0.9 km, it blows a whistle whose echo
v0, one of the following two graphs (A and B) will given
is heard by the driver after 5 seconds. If the speed of
the correct variation of the frequency f heard by the
sound in air is 330 m/s, then the speed of the engine is :
observer as v0 is changed.
[Online April 9, 2013]
(a) 32 m/s (b) 27.5 m/s (c) 60 m/s (d) 30 m/s
(A) (B) 86. This question has Statement 1 and Statement 2. Of the
f f four choices given after the Statements, choose the one
that best describes the two Statements.
v0 1/v0 Statement 1: Bats emitting ultrasonic waves can detect
The variation of f with v0 is given correctly by : the location of a prey by hearing the waves reflected from it.
[Online April 11, 2015] Statement 2: When the source and the detector are
f0 moving, the frequency of reflected waves is changed.
(a) graph A with slope = [Online May 12, 2012]
(v + vs ) (a) Statement 1 is false, Statement 2 is true.
f0 (b) Statement 1 is true, Statement 2 is false.
(b) graph B with slope =
(v – vs ) (c) Statement 1 is true, Statement 2 is true, Statement
f0 2 is not the correct explanation of Statement 1.
(c) graph A with slope =
(v – vs ) (d) Statement 1 is true, Statement 2 is true, Statement
f0 2 is the correct explanation of Statement 1.
(d) graph B with slope = 87. A motor cycle starts from rest and accelerates along a
(v + vs )
straight path at 2m/s2. At the starting point of the motor
81. A bat moving at 10 ms–1 towards a wall sends a sound
cycle there is a stationary electric siren. How far has the
signal of 8000 Hz towards it. On reflection it hears a
motor cycle gone when the driver hears the frequency of
sound of frequency f. The value of f in Hz is close to
the siren at 94% of its value when the motor cycle was at
(speed of sound = 320 ms–1)
rest? (Speed of sound = 330 ms–1) [2009]
[Online April 10, 2015]
(a) 8516 (b) 8258 (c) 8424 (d) 8000 (a) 98 m (b) 147 m (c) 196 m (d) 49 m
82. A source of sound A emitting waves of frequency 1800 88. A whistle producing sound waves of frequencies 9500
Hz is falling towards ground with a terminal speed v. The HZ and above is approaching a stationary person with
observer B on the ground directly beneath the source speed v ms–1. The velocity of sound in air is 300 ms–1. If
receives waves of frequency 2150 Hz. The source A the person can hear frequencies upto a maximum of
receives waves, reflected from ground of frequency 10,000 HZ, the maximum value of v upto which he can
nearly: (Speed of sound = 343 m/s) hear whistle is [2006]
[Online April 12, 2014]
15
(a) 2150 Hz (b) 2500 Hz (a) 15 2 ms -1 (b) ms -1
(c) 1800 Hz (d) 2400 Hz 2
83. Two factories are sounding their sirens at 800 Hz. A man (c) 15 ms–1 (d) 30 ms–1
goes from one factory to other at a speed of 2m/s. The 89. An observer moves towards a stationary source of sound,
velocity of sound is 320 m/s. The number of beats heard with a velocity one-fifth of the velocity of sound. What is
by the person in one second will be: the percentage increase in the apparent frequency ?[2005]
[Online April 11, 2014] (a) 0.5% (b) zero (c) 20 % (d) 5 %
P-226 Physics

1. (a) As we know, 6. (a) Comparing the given equation


y = 10–3sin(50t + 2x) with standard equation,
w y = a sin(wt – kx)
Pressure amplitude, DP0 = aKB = S 0 KB = S 0 ´ ´ rV 2
V Þ wave is moving along –ve x-axis with speed
é w Bù w 50
êQ K = , V = ú v= Þv=
K 2
= 25 m/sec.
ë V rû
7. (a) Given, amplitude a = 10 cm
DP0 10 1 3 wave velocity = 2 × maximum particle velocity
Þ S0 = » m= mm » mm
rV w 1 ´ 300 ´ 1000 30 100 i.e, wl = 2 aw
2. (b) Given : Distance between one crest and one trough 2p p
or, l = 4a = 4 × 10 = 40 cm
= 1.5 m 8. (b)
l 9. (b) Standard equation
= (2n1 + 1)
2 æw ö
y(x, t) = A cos ç x - wt ÷
èV ø
Distance between two crests = 5 m = n2 l
From any of the displacement equation
1.5 (2n1 + 1) Say y1
= Þ 3n2 = 10n1 + 5 w
5 2n2 = 0.50 p and w = 100 p
V
Here n1 and n2 are integer.
100p
If n1 = 1, n2 = 5 \l = 1 \ = 0.5p
V
n1 = 4, n2 = 15 \l = 1/ 3 100p
\ V= = 200 m/s
n1 = 7, n2 = 25 \l = 1/ 5 0.5p
10. (c) The equation of wave at any time is obtained by
1 1 1 putting X = x – vt
Hence possible wavelengths , , metre. 1 1
1 3 5 y= 2 = 1 + ( x - vt ) 2 ...(i)
3. (b) At t = 0, x = 0, y = 0 1+ x
f = p rad We know at t = 2 sec,
4. (a) Using, b = 10 1
y= ...(ii)
1 + ( x - 1) 2
æ I ö On comparing (i) and (ii) we get
or 120 = 10 log10 çè -12 ÷ø ...(i)
10 vt = 1
1
P 2 V=
Also I = 2
= ...(ii) t
4pr 4pr 2 As t = 2 sec
On solving above equations, we get 1
\V= =0.5 m/s.
r = 40 cm. 2
5. (a) On comparing with P = P0 sin (wt – kx), we have 11. (a) Given
w = 1000 rad/s, K = 3 m–1
y (x, t) = e (-ax2 +bt2 +2 ab xt )
w 1000 -[( ax )2 +( b t )2 + 2 a x . b t ]
\ v0 = = = 333.3m/s = e
k 3
- ( a x + bt )2
v1 T = e
= 1 2
v2 T2 æ ö
b ÷
-ç x + t
e è a ø
=
333.3 273 + 0 It is a function of type y = f (x + vt)
or =
336 273 + t \ y (x, t) represents wave travelling along –ve x direction
\ t = 4°C w b
Þ Speed of wave = =
k a
Waves P-227

12. (a) Given, l = 2(l2 - l1 ) = 2 ´ (24.5 - 17) = 15 cm


Wavelength, l = 0.08m
Time period, T = 2.05 Now, from v = f l Þ 330 = l ´ 15 ´ 10-2
y(x, t) = 0.005 cos (ax - bt) (Given)
Comparing it with the standard equation of wave 330 1100 ´ 100
\l = ´ 100 = = 2200 Hz
y(x, t) = a cos (kx - wt) we get 15 5
2p 2p 18. (c) Using, V = f l
k=a = and w = b =
l T
V1 V2 V
2p 2p = Þ l 2 = 2 l1
\ a= = 25p and b = =p l1 l 2 V1
0.08 2
æI ö
13. (a) Loudness of sound. L1 = 10 log ç 1 ÷ ; T2
è I0 ø
12 m, 6 kg
æI ö
L2 = 10log ç 2 ÷
è I0 ø l = 6 cm T1
æI ö æ I2 ö 2 kg
\ L1 – L2 = 10 log ç 1 ÷ - 10 log ç ÷
è I0 ø è I0 ø Again using,
æI I ö V T T
or, DL = 10 log ç 1 ´ 0 ÷ n= = l 2 = 2 l1 T2 = 8g (Top)
è I0 I 2 ø l M T1
æI ö
or, DL = 10 log ç 1 ÷ 8g
è I2 ø = l1 = 2l1 = 12 cmT = 2g (Bottom)
2g 1
The sound level attenuated by 20 dB ie
L1 – L2 = 20 dB
1 T
æI ö æI ö 19. (a) Using f = ,
or, 20 = 10 log ç 1 ÷ or, 2 = log ç 1 ÷ 2l m
è I2 ø è I2 ø
I1 I mass
or, = 102 or, I2 = 1 . where, T = tension and m =
I2 100 length
Þ Intensity decreases by a factor 100.
1 Tx 1 Tz
æ pö fx = and f z =
14. (b) Given, y = 10–6 sin çè100t + 20 x + ÷ø m 2l m 2l m
4
Comparing it with standard equation, we get f x 450 T
w = 100 and k = 20 = = x
f z 300 Tz
w 100
v= = = 5m / s
k 20 Tx 9
æ pö \ = = 2.25.
-4 Tz 4
15. (a) y = 10 sin çè 600t - 2x + ÷ø
3
On comparing with standard equation 20. (35.00)
Given,
y= A sin ( wt - kx + f)
we get w = 600; k=2 Denisty of wire, s = 9 ´ 10 -3 kg cm–3
Velocity of wave Young's modulus of wire, Y = 9 × 1010 Nm–2
w 600 Strain = 4.9 × 10–4
v= = = 300 ms -1
k 2 Stress T / A
16. (b) The frequency of a tuning fork is given by Y= =
Strain Strain
m2 k Y
f= T
4 3 pl 2r \ = Y ´ Strain = 9 × 109 × 4.9 × 10–4
As temperature increases, the length or dimension of the A
prongs increases and also young's modulus increases Also, mass of wire, m = Al s
therefore f decreases.
17. (a) Here, l1 = 17 cm and l2 = 24.5 cm, V = 330 m/s, m
Mass per unit length, m = = As
f=? J
P-228 Physics

Fundamental frequency in the string


T mv 2
Using, v = ´l ÞT =
1 T 1 T m I
f = =
2l m 2l sA T
Again from, Y = DL / L0
A
1 9 ´ 109 ´ 4.9 ´ 10 -4 Tl mv 2 ´ l
= DL = =
2 ´1 9 ´ 103 YA l (YA)
6 ´ 10 –3 ´ 902
1
=
1
49 ´ 109 - 4 -3 = ´ 70 = 35 Hz = 11 –6
= 3 ´ 10 –4 m
2 2 16 ´ 10 ´ 10
= 0.03 mm
21. (106) Given : Vair = 300 m/s, rgas = 2 r air
24. (c) We have given,
B y = 0.03 sin(450 t – 9x)
Using, V =
r Comparing it with standard equation of wave, we get w =
450 k = 9
B w 450
Vgas 2rair \ v= = = 50m/s
= k 9
Vair B Velocity of travelling wave on a stretched string is given
by
rair
T T
v= Þ = 2500
Vair 300 m m
Þ Vgas = = = 150 2m/s
2 2 m = linear mass density
Þ T = 2500 × 5 × 10–3
nv
And fnth harmonic = (in open organ pipe) Þ 12.5 N
2L
(L = 1 metre given) T
25. (b) Wave speed V =
2v v v m
\ f2nd harmonic – ffundamental = – = when car is at rest a = 0
2 ´1 2 ´ 1 2
Mg
\ f2n harmonic – ffundamental = 150 2 = 150 » 106 Hz \ 60 =
2 2 m
22. (b) The velocity of a transverse wave in a stretched wire Similarly when the car is moving with acceleration a,
is given by
( )
12
M g2 + a 2
T 60.5 =
v=
m m
Where, 60.5 g2 + a2
T = Tension in the wire =
m = linear density of wire
60 g2
4
(Q V µ T ) æ 0.5 ö g2 + a2 2
çè1+ ÷ø = 2
=1+
60 g 60
v T
\ 1 = 1 2
v2 T2 Þ g2 + a 2 = g 2 + g 2 ×
60
2 g
v 2.06 ´ 10 4 a =g = [which is closest to g/5]
Þ ´2= 60 30
v T2
26. (a) Fundamental frequency, f = 70 Hz.
4 The fundamental frequency of wire vibrating under
2.06 ´ 10
Þ T2 = = 0.515 ´ 104 N tension T is given by
4
Þ T2 = 5.15 × 103 N 1 T
f =
23. (a) Given, l = 60 cm, m = 6 g, A = 1 mm2, v = 90 m/s and Y 2L m
= 16 × 1011 Nm–2
Waves P-229

Here, µ = mass per unit length of the wire Since rod is clamped at middle fundamental wave shape
L = length of wire is as follow
1 540 l A A
70 = = L Þ l = 2L N
2 L 6 ´ 10-3 2 l/2
Þ L » 2.14 m l = 1.2m (Q L = 60 cm = 0.6m (given)
27. (b) V = f l = f × 2 (l 2 - l1 ) Using v = fl
= 480 × 2(0.70 – 0.30)
= 384 m/s v 5.85 ´103
Þ f= =
3l 4 l 1.2
28. (b) = 2 or l = m = 4.88 × 103 Hz ; 5 KHz
2 3 l
4 34. (a) For first resonance, = l1 + e = 11 cm
Velocity, v = f l = 240 ´ = 320 m/sec 4
3 (Q end correction e = 1 cm given)
240 3l
Also f1 = = 80 Hz For second resonance, = l2 + e
3 4
29. (b) Given, y = 0.3 sin (0.157 x) cos (200 pt) Þ l 2 = 3 ´ 11 - 1 = 32 cm
So k = 0.157 and w = 200p 35. (b) n1 = n2
w 200p T ® Same
or f = 100 Hz, v = = = 4000m/s r ® Same
k 0.157
l ® Same
nv 4v 2v
Now, using f = = = Frequency of vibration
2l 2l l
p T
2v 2 ´ 4000 n=
\l = = = 80m 2lpr 2 r
f 100
30. (c) As there must be node at both ends and at the joint As T, r, and l are same for both the wires
of the wire A and B so n1 = n2
VA uB rB l p1 p2
= = =2= A =
VB u A rA lB r1 r2
Þ l A = 2l B
p
Þ 1 =
1 Q r2 = 4 r1
p2 2
P 1
Þ = 36. (a ) We know that velocity in string is given by
q 2
T
31. (a) If a closed pipe vibration in Nth mode then frequency v= ...(i)
m
( 2N - 1) v = 2N - 1 n
of vibration n = ( ) 1 m mass of string
4l where m = =
(where n1 = fundamental frequency of vibration) l length of string
Hence 20,000 = (2N – 1) × 1500
m
Þ N = 7.1 » 7 The tension T = ´ x´g ..(ii)
\ Number of over tones = (No. of mode of vibration) – 1 l
=7–1=6 From (1) and (2)
32. (d) Velocity of wave on string
T 8 l
V= = ´ 1000 = 40m/s
m 5 T
Here, T = tension and µ = mass/length
x
v 40
Wavelength of wave l = = m
n 100 dx
Separation b/w successive nodes, = gx
dt
l 40 20
= = m = 20 cm x -1/2 dx = g dt
2 2 ´ 100 100
33. (a) In solids, Velocity of wave l l
\ ò x -1/2 dx - g ò dt
Y 9.27 ´ 1010
V= = 0 0
r 2.7 ´ 103 Þ2 l
3
v = 5.85 × 10 m/sec
P-230 Physics

41. (d) Total length of the wire, L = 114 cm


l 20 n1 : n2 : n3 = 1 : 3 : 4
= g´t \ t = 2 =2 =2 2
g 10 Let L1, L2 and L3 be the lengths of the three parts
37. (b)
f 1
As n µ
L
l l 1 1 1
\ L1 : L2 : L3 = : : = 12: 4 : 3
1 3 4
(a) (b)
\ L1 = ç
æ 12 ö
v ´ 114÷ = 72cm
The fundamental frequency in case (a) is f = è 12 + 4 + 3 ø
2l
The fundamental frequency in case (b) is æ 4 ö
L2 = ç ´ 114÷ = 24 cm
v v è 19 ø
f'= = =f
4(l / 2) 2l æ 3 ö
38. (c) Length of pipe = 85 cm = 0.85m and L3 = ç ´ 114÷ = 18 cm
è 19 ø
Frequency of oscillations of air column in closed organ Hence the bridges should be placed at 72 cm and 72 + 24
pipe is given by, = 96 cm from one end.
(2n - 1)u 42. (a) Initially for open organ pipe, fundamental frequency
f =
4L v
n0 = … (i)
(2n - 1)u 2l0
f = £ 1250
4L where l0 is the length of the tube
(2n - 1) ´ 340 v = speed of sound
Þ £ 1250 But when it is half dipped in water, it becomes closed organ
0.85 ´ 4 l
Þ 2n – 1 < 12.5 » 6 pipe of length 0 .
39. (b) Total length of sonometer wire, l = 110 cm = 1.1 m 2
Length of wire is in ratio, 6 : 3 : 2 i.e. 60 cm, 30 cm, 20 cm. Fundamental frequency of closed organ pipe
Tension in the wire, T = 400 N v
Mass per unit length, m = 0.01 kg nc = … (ii)
Minimum common frequency = ? 4lc
As we know, l0
New length, lc =
1 T 1000 2
Frequency, n = = Hz v v
2l m 11 Thus n c = Þ nc = … (iii)
1000 4l0 / 2 2l
Similarly, n1 = Hz From equations (i) and (iii)
6
n 0 = nc
1000
n2 = Hz Thus, nc = f ( Q n0 = f is given)
3
43. (b) Given : Frequency of tuning fork, n = 264 Hz
1000 Length of column L = ?
n3 = Hz
2 For closed organ pipe
Hence common frequency = 1000 Hz v
40. (b) Fundamental frequency, n=
4l
v 1 T 1 T é T mù
f = = = êQ v = and m = ú
Þl=
v 330
2l 2l m 2l Ar ë m lû 4n
=
4 ´ 264
= 0.3125
Tl T Y Dl or, l = 0.3125 × 100 = 31.25 cm
Also, Y = Þ = In case of closed organ pipe only odd harmonics are
ADl A l possible.
1 y Dl Therefore value of l will be (2n – 1) l
Þ f = ....(i) Hence option (b) i.e. 3 × 31.25 = 93.75 cm is correct.
2l lr 44. (d) Two lowest frequencies to which tube will resonates
Dl are 272 Hz and 544 Hz.
l = 1.5 m, = 0.01,
l é æ t ö x ù
r = 7.7 × 103 kg/m3 (given) 45. (d) y = 0.02(m)sin ê2p ç ÷- ú
ë è 0.04( s) ø 0.50(m) û
y = 2.2 × 1011 N/m2 (given)
Comparing it with the standard wave equation
Dl
Putting the value of l, , r and y in eqn. (i) we get, y = a sin(wt - kx )
l we get
2 103
f = ´ 2p
7 3 or f » 178.2 Hz w= rad s–1
0.04
Waves P-231

2p uA v 4l 2
and k =
0.50 \ u = 2l ´ v = 1
B
w 49. (b) To form a node there should be superposition of this
Wave velocity, v = wave with the reflected wave. The reflected wave should
k
travel in opposite direction with a phase change of p. The
2p / 0.04 equation of the reflected wave will be
Þv= = 12.5 m / s
2p / 0.5 y = a sin (wt + kx + p)
Velocity on a string is given by Þ y = – a sin (wt + kx)
50. (a)
T 51. (c) Beat frequency
v=
m = difference in frequencies of two waves
= 11 – 9 = 2 Hz
\ T = v2 ´ m = (12.5)2 × 0.04 = 6.25 N 52. (d)
46. (b) Fundamental frequency for first resonant length 53. (d) According to question, tuning fork gives 1 beat/second
with (N) 3rd normal mode. Therefore, organ pipe will have
v v
n= = (in winter) frequency (256 ± 1) Hz. In open organ pipe, frequency
4l1 4 ´ 18 NV
Fundamental frequency for second resonant length n=
2l
3v ' 3v '
n' = = (in summer) 3 ´ 340
4l 2 4x or, 255 = Þ l = 2 m = 200 cm
2´ l
According to questions, 54. (a) Probable frequencies of tuning fork be n ± 5
v 3v' 1
\ = Frequency of sonometer wire, n µ
4 ×18 4× x l
v' n + 5 100
\ x = 3 ´ 18 ´ \ = Þ 95(n + 5) = 100( n - 5)
v n - 5 95
v' or, 95 n + 475 = 100 n – 500
\ x = 54 ´ cm or, 5 n = 975
v
v' > v because velocity of light is greater in summer as 975
or, n = = 195 Hz
compared to winter (v µ T ) 5
æ 5p ö
\ x > 54cm 55. (a) Given, y (x, t) = 0.5 sin ç x ÷ cos (200 pt),
47. (a) It is given that 315 Hz and 420 Hz are two resonant è 4 ø
frequencies, let these be n th and (n + 1)th harmonies, then comparing with equation – y (x, t) = 2 a sin kx cos wt
5p
we have nv = 315 w = 200 p, k =
2l 4
w 200p
v speed of travelling wave v = = = 160 m/s
and (n + 1) = 420 k 5p 4
2l
56. (b) Since the point x = 0 is a node and reflection is taking
n + 1 420 place from point x = 0. This means that reflection must be
Þ =
n 315 taking place from the fixed end and hence the reflected ray
Þn=3 must suffer an additional phase change of p or a path
v l
v change of .
Hence 3 ´ = 315 Þ = 105 Hz 2
2l 2 l
The lowest resonant frequency is when So, if yincident = a cos ( kx – wt)
n=1 Þ yincident = a cos (– kx – wt + p)
Therefore lowest resonant frequency = – a cos (wt + kx)
= 105 Hz. Hence equation for the other wave
48. (c) The fundamental frequency for tube B closed at one y = a cos(kx + wt + p)
end is given by
57. (d) In case of destructive interference
v é lù Phase difference f = 180° or p
uB = êQl = ú
4l ë 4û So wave pair (i) and (ii) will produce destructive
Where l = length of the tube and v is the velocity of interference.
sound in air. Stationary or standing waves will produce by equations
The fundamental frequency for tube A open with both ends
is given by (iii) & (iv) as two waves travelling along the same line but
lù in opposite direction.
v é
uA = êQl = ú n¢ = n + x
2l ë 2û 58. (d) y = A sin (wt – kx) + A sin (wt + kx)
y = 2A sin wt cos kx
P-232 Physics

This is an equation of standing wave. For position of 65. (a) Let f 1 be the frequency heard by wall,
nodes æ v ö
f1 = ç f0
è v - vc ÷ø
cos kx = 0
2p p
Þ .x = (2n + 1) Here, v = Velocity of sound,
l 2 vc = Velocity of Car,
Þ x=
( 2 n + 1) l
, n = 0,1, 2,3,...........
f0 = actual frequency of car horn
4 Let f2 be the frequency heard by driver after reflection
59. (a) Intensity of a wave from wall.
1 æ v + vc ö æ v + vc ö
I = pw2 A2 v f2 = ç f = f
2 è v ÷ø 1 çè v - v ÷ø 0c
Since, I µ A2w 2
é 345 + vc ù 12 345 + vc
\ I1 µ (2a)2 w 2 Þ 480 = ê ú 440 Þ =
ë 345 - vc û 11 345 - vc
and I 2 µ a 2 (2w ) 2
Þ vc = 54 km/hr
I1 = I 2 66. (a) From the Doppler's effect of sound, frequency
In the same medium, p and v are same. appeared at wall
Intensity depends on amplitude and frequency. 330
60. (b) Maximum number of beats fw = ×f ...(i)
330 - v
= Maximum frequency – Minimum frequency Here, v = speed of bus,
= ( n + 1) – ( n – 1) = 2 Beats per second f = actual frequency of source
61. (d) Frequency of fork 1, no = 200 Hz Frequency heard after reflection from wall (f') is
No. of beats heard when fork 2 is sounded with fork 1 = Dn = 330 + v 330 + v
4 f '= × fw = ×f
330 330 - v
Now on loading (attaching tape) on unknown fork, the 330 + v
mass of tuning fork increases, So the beat frequency Þ 490 = × 420
increases (from 4 to 6 in this case) then the frequency 330 - v
of the unknown fork 2 is given by, 330 ´ 7
Þv= » 25.38 m/s = 91 km/s
n = n 0 – Dn = 200 – 4 = 196 Hz 91
62. (c) It is given that tuning fork of frequency 256 Hz makes 67. (d) Permanent magnets (P) are made of materials with large
5 beats/second with the vibrating string of a piano.
Therefore, possible frequency of the piano are (256 ± 5) retentivity and large coercivity. Transformer cores (T) are
Hz. i.e., either 261Hz or 251 Hz. When the tension in the made of materials with low retentivity and low coercivity.
piano string increases, its frequency will increases. As 68. (c) From Doppler’s effect, frequency of sound heard (f1)
the original frequency was 261Hz, the beat frequency when source is approaching
should decreases, we can conclude that the frequency of c
piano string is 251Hz f1 = f 0
c–v
63. (b) Frequency of unknown fork = known frequency ± Beat Here, c = velocity of sound
frequency = 288 + 4 cps or 288 – 4 cps i.e. 292 cps or 284 v = velocity of source
cps. When a little wax is placed on the unknown fork, it Frequency of sound heard (f2) when source is receding
produces 2 beats/sec. When a little wax is placed on the
unknown fork, its frequency decreases and simultaneously c
the beat frequency decreases confirming that the frequency f2 = f0
c+v
of the unknown fork is 292 cps. Beat frequency = f1 – f2
Note : Had the frequency of unknown fork been 284 cps,
then on placing wax its frequency would have decreased é 1 1 ù
Þ 2 = f1 – f 2 = f 0c ê –
thereby increasing the gap between its frequency and the ë c – v c + v úû
frequency of known fork. This would produce high beat
frequency. 2v
= f0c
64. (b) Frequency heard by the observer é v2 ù
c 2 ê1 – 2 ú
æ vsound ö êë c úû
vobserved = ç ÷ v0 For c>> v
è vsound - v cos q ø
Observer 2c c 350 1
O Þ v= = = = m/s
2 f0 f 0 1400 4
D æ v - vo ö æ 1500 - 5 ö
69. (d) f1 = f ç v - v ÷ = f çè 1500 - 7.5 ÷ø
q è s ø
Source V No reflected signal,
Initially q will be less so cos q more.
\ vobserved more, then it will decrease.
Waves P-233

æ v + vo ö æ 1500 + 7.5 ö é V ù é 340 ù


f2 = f1 ç v + v ÷ = f1 çè 1500 + 5 ÷ø fapp = f0 ê ú Þ fl = f0 ê ú
è s ø ë V – Vs û ë 340 – 34 û
æ 1500 - 5 ö æ 1500 + 7.5 ö é 340 ù
f2 = 500 çè ÷ç ÷
1500 - 7.5 ø è 1500 + 5 ø and, f 2 = f 0 ê ú
502 Hz ë 340 –17 û
f 340 –17 323 f 19
v - v0 v + v0 \ l = = or, 1 =
70. (c) f1 = f and f2 = f f 2 340 – 34 306 f 2 18
v v 75. (a) Frequency of the sound produced by open flute.
u æ v ö 2 ´ 330
S1 L S2 f = 2ç ÷ = = 660 Hz
è 2l ø 2 ´ 0.5
But frequency, 5 25
2v0 Velocity of observer, v0 = 10 ´ = m / s
18 9
f2 – f1 = f ´ As the source is moving towards the observer therefore,
v
2u according to Doppler's effect.
or 10 = 660 ´ \ Frequency detected by observer,
330 é 25 ù
\ u = 2.5 m/s. ê 9 + 330 ú
71. (b) Frequency of sound source (f0) = 500 Hz é v + v0 ù ê ú 660
f'= ê ú f = ê 330 ú
When observer is moving away from the source ë v û
ë û
æ n - n'0 ö
2995
Apparent frequency f1 = 480 = f0 ç n ÷ ....(i) = ´ 660 or, f ' = 665.55 ; 666 Hz
è ø 9 ´ 330
And when observer is moving towards the source 76. (d) nA = 425 Hz, nB = ?
æ n - n"0 ö Beat frequency x = 5 Hz which is decreasing (5 ® 3) after
f 2 = 530 = f0 ç ÷ ....(ii) increasing the tension of the string B.
è n ø Also tension of string B increasing so
From equation (i)
nB­ (Q n µ T)
æ 300 - v '0 ö
480 = 500 ç ÷ Hence nA – nB­= x¯ ¾¾ ® correct
ç 300 ÷ nB­ – nA= x¯ ¾¾ ® incorrect
è ø
v’0 = 12 m/s \ nB = nA – x = 425 – 5 = 420 Hz
From equation (ii) 77. (d) From Doppler's effect
æ v'' ö æ 340 ö
530 = 500 ç1 + 0 ÷ f (direct) = f ç ÷ = f1
è vø è 340 - 5 ø
\ V”0 = 18 m/s æ 340 ö
f (by wall)=f ç ÷ =f
72. (a) When source is moving towards a stationary è 340+5 ø 2
observer, Beats = (f1 – f2)
æ V -0 ö æ 340 340 ö
fapp = f source ç 5=fç -
è V - 50 ÷ø è 340 - 5 340 + 5 ø
÷
æ 350 ö Þ f < 170 Hz.
1000 = fsource ç
è 300 ÷ø 78. (d) We know that the apperent frequency
When source is moving away from observer æ v - v0 ö
f'=ç ÷ f from Doppler's effect
æ 350 ö
f ' = fsource çè ÷ è v - vs ø
350 + 50 ø where v0 = vs = 30 m/s, velocity of observer and source
1000 ´ 300 350 Speed of sound v = 330 m/s
f'= ´ 330 ∗ 30
350 400 \ f '< ≥ 540 = 648 Hz.
f ' » 750 Hz 330 , 30
v , v0 Q Frequency of whistle (f) = 540 Hz.
73. (a) f ' = f
v ∗ vs é v ù 320
79. (d ) f1 = f ê ú= f´ Hz
340 , 20 ë v - vsû 300
or 2000 < f
340 ∗ 20 é v ù 320
\ f = 2250 Hz. f2 = f ê ú= f ´ Hz
74. (b) According to Doppler’s effect, when source is moving ë v + vs û 340
but observer at rest
P-234 Physics

æ f2 ö æ 300 ö æ v ö
æ 340 ö = 506
çè f - 1÷ø ´ 100 = çè 340 - 1÷ø ´ 100 ; 12% apparent frequency h ' = h ç v - v ÷ = 500 ç ÷
1 è s ø è 336 ø
80. (c) According to Doppler’s effect, Hz
æ V +V ö
0 Case 2 : When source is moving away from the stationary
Apparent, frequency f = ç V – V ÷ f 0 listener
è S ø
æ f ö V f æ v ö æ 340 ö
h" = h ç ÷ = 500 ç
0 0
Now, f = ç V – V ÷ V0 + V – V ÷ = 494 Hz
è S ø s è v + vs ø è 344 ø
f0 In case 1 number of beats heard is 6 and in case 2 number of
So, slope = beats heard is 18 therefore frequency of the source at B = 512 Hz
V – VS
Hence, option (c) is the correct answer. H
81. (a) Reflected frequency of sound reaching bat A 0.9 km B I
85. (d) ENGINE L
é V - (-V0 ) ù é V + V0 ù V + 10
= ê úf= ê úf= f C L
ë V - Vs û ëV - Vs û V - 10
Let after 5 sec engine at point C
æ 320 + 10 ö AB BC 0.9 ´ 1000 BC
= ç ÷ ´ 8000 = 8516 Hz t= + 5= +
è 320 - 10 ø 330 330 330 330
82. (b) Given fA = 1800 Hz \ BC = 750 m
vt = v Distance travelled by engine in 5 sec
fB = 2150 Hz = 900 m – 750 m = 150 m
Reflected wave frequency received by A, fA¢ = ? Therefore velocity of engine
Applying doppler’s effect of sound, 150 m
= = 30 m/s
vs f 5sec
f¢ = 86. (c) Bats catch the prey by hearing reflected ultrasonic
vs - v t waves.
æ f ö When the source and the detector (observer) are moving,
here, v t = vs ç1 - A ÷ frequency of reflected waves change. This is according to
è fB ø
Doppler’s effect.
æ 1800 ö 2
= 343 ç 1 - ÷ u=0 a = 2m/s vm
è 2150 ø 87. (a)
vt = 55.8372 m/s Electric s Motor
Now, for the reflected wave, siren cycle
Let the motorcycle has travelled a distances, its velocity
æ vs + v t ö at that point
¢
\ fA = ç v - v ÷ fA 2
è s t ø vm - u 2 = 2as \ v2m = 2 ´ 2 ´ s
æ 343 + 55.83 ö \ vm = 2 s
=ç ÷ ´ 1800
è 343 - 55.83 ø The observed frequency will be
= 2499.44 » 2500Hz é v - vm ù
83. (d) Given: Frequency of sound produced by siren, f = n'= nê ú
800 Hz ë v û
Speed of observer, u = 2 m/s é 330 - 2 s ù
Velocity of sound, v = 320 m/s 0.94n = n ê ú Þ s = 98.01 m
No. of beats heard per second = ? ë 330 û
No. of extra waves received by the observer per second = + 4l é v ù
\ No. of beats/ sec 88. (c) Apparent frequency n' = n ê ú
2 æ 2ö 4 ë v - vs û
= - ç- ÷ = é 300 ù
l è lø l Þ 10000 = 9500 ê ú Þ 300 - v = 300 ´ 0.95
ë 300 - v û
2´ 2 æ Vö
= 320 çQ l = ÷ Þ v = 300 - 285 = 15 ms -1
è f ø
89. (c) Apparent frequency
800
é vù
2 ´ 2 ´ 800 ê v+ ú
= = 10 é v + v0ù = n 5 = n é6ù n' = 6
320 n' = nê ú ê ú ê5ú n 5
84. (c) f = 500 Hz ë v û ê v ú ë û
A 4 m/s C B ë û
The percentage increase in apparent
Listener n'- n 6 - 5
Case 1 : When source is moving towards stationary frequency = ´ 100 = 20%
listener n 5
15
Electric Charges and Fields P-235

Electric Charges
and Fields
Electric Charges and Coulomb's
TOPIC 1
Law +Q
1. Three charges + Q, q, + Q are placed respectively, at
distance, d/2 and d from the origin, on the x-axis. If the net –Q
force experienced by + Q, placed at x = 0, is zero, then
value of q is: [9 Jan. 2019 I]
(a) – Q/4 (b) + Q/2 (c) + Q/4 (d) – Q/2
2. Charge is distributed within a sphere of radius R with a (a) s1 ¹ 0, Q1 = 0 (b) s1 ¹ 0, Q1 = 0
s2 = 0, Q2 = 0 s2 ¹ 0, Q2 = 0
A -2r a (c) s1 = 0, Q1 = 0 (d) s1 ¹ 0, Q1 ¹ 0
volume charge density p(r) = e where A and a
r2 s2 = 0, Q2 = 0 s2 ¹ 0, Q2 ¹ 0
are constants. If Q is the total charge of this charge 5. Two charges, each equal to q, are kept at x = – a and x = a
distribution, the radius R is: [9 Jan. 2019, II] q
on the x-axis. A particle of mass m and charge q 0 = is
2
æ ö placed at the origin. If charge q0 is given a small
æ Q ö a ç 1 ÷ displacement (y <<a) along the y-axis, the net force acting
(a) a log ç 1 - ÷ (b) log ç ÷ on the particle is proportional to [2013]
è 2paA ø 2 çç 1 - Q ÷÷
è 2paA ø 1 1
(a) y (b) –y (c) (d) –
y y
æ ö 6. Two balls of same mass and carrying equal charge are
ç 1 ÷ a æ Q ö hung from a fixed support of length l. At electrostatic
(c) a log ç ÷ (d) log ç 1 - ÷ equilibrium, assuming that angles made by each thread is
çç 1 - Q ÷÷ 2 è 2paA ø small, the separation, x between the balls is proportional
è 2paA ø to : [Online April 9, 2013]
3. Two identical conducting spheres A and B, carry equal (a) l (b) l 2 (c) l 2/3 (d) l 1/3
charge. They are separated by a distance much larger than 7. Two identical charged spheres suspended from a common
their diameter, and the force between them is F. A third point by two massless strings of length l are initially a
distance d(d << l) apart because of their mutual repulsion.
identical conducting sphere, C, is uncharged. Sphere C is
The charge begins to leak from both the spheres at a
first touched to A, then to B, and then removed. As a constant rate. As a result charges approach each other
result, the force between A and B would be equal to with a velocity v. Then as a function of distance x between
[Online April 16, 2018] them, [2011]
3F F 3F (a) v µ x–1 (b) v µ x½ (c) v µ x (d) v µ x–½
(a) (b) (c) F (d) 8. A charge Q is placed at each of the opposite corners of a
4 2 8
square. A charge q is placed at each of the other two
4. Shown in the figure are two point charges +Q and –Q
corners. If the net electrical force on Q is zero, then Q/q
inside the cavity of a spherical shell. The charges are kept
equals: [2009]
near the surface of the cavity on opposite sides of the
1
centre of the shell. If s1 is the surface charge on the inner (a) –1 (b) 1 (c) - (d) -2 2
surface and Q1 net charge on it and s2 the surface charge 2
9. If gE and gM are the accelerations due to gravity on the
on the outer surface and Q2 net charge on it then :
surfaces of the earth and the moon respectively and if
[Online April 10, 2015]
Millikan’s oil drop experiment could be performed on the
P-236 Physics

two surfaces, one will find the ratio [2007] 1 Qq 1 Qq


electronic charge on the moon
(a) F = for r < R (b) > F > 0 for r < R
to be 4pe0 R 2 4pe 0 R 2
electronic charge on the earth
1 Qq 1 Qq
(a) gM / g E (b) 1 (c) 0 (d) g E / g M (c) F = for r > R (d) F = for all r
4pe0 R 2 4pe0 R 2
10. Two spherical conductors B and C having equal radii and 15. Two charged thin infinite plane sheets of uniform surface
carrying equal charges on them repel each other with a
charge density s + and s – , where | s + | > | s – |, intersect
force F when kept apart at some distance. A third spherical
conductor having same radius as that B but uncharged is at right angle. Which of the following best represents the
brought in contact with B, then brought in contact with C electric field lines for this system ? [Sep. 04, 2020 (I)]
s–
and finally removed away from both. The new force of
repulsion between B and C is [2004]
(a) F/8 (b) 3 F/4 (c) F/4 (d) 3 F/8
11. Three charges –q1 , +q2 and –q3 are place as shown in the
figure. The x - component of the force on –q 1 is (a) s+
proportional to [2003]
Y
q3
s–

a
b
(b) s+
q1 +q 2 X
q2 q3 q 2 q3
(a) 2 - 2 cos q (b) 2 + sin q
b a b a2 s–
q q q q
(c) 2 + 3 cos q (d) 2 - 3 sin q
2
a2
2 2
b a b
12. If a charge q is placed at the centre of the line joining two
equal charges Q such that the system is in equilibrium (c) s+
then the value of q is [2002]
(a) Q/2 (b) –Q/2 (c) Q/4 (d) –Q/4

Electric Field and Electric Field s–


TOPIC 2
Lines
13. Charges Q1 and Q2 are at points A and B of a right angle
triangle OAB (see figure). The resultant electric field at (d) s+
point O is perpendicular to the hypotenuse, then Q1/Q2 is
proportional to : [Sep. 06, 2020 (I)]
A
Q1
16. A particle of charge q and mass m is subjected to an electric
x1 field E = E0 (1 – ax2) in the x-direction, where a and E0 are
constants. Initially the particle was at rest at x = 0. Other
Q2 than the initial position the kinetic energy of the particle
O x2 B becomes zero when the distance of the particle from the
origin is : [Sep. 04, 2020 (II)]
x13 x2 x1 x2 2
(a) (b) (c) (d) 2 3 1
x23 x1 x2 x12 (a) a (b) (c) (d)
a a a
14. Consider the force F on a charge ‘q’ due to a uniformly 17. A charged particle (mass m and charge q) moves along X
charged spherical shell of radius R carrying charge Q dis- axis with velocity V0. When it passes through the origin it
tributed uniformly over it. Which one of the following state- r
enters a region having uniform electric field E = - Ejˆ which
ments is true for F, if ‘q’ is placed at distance r from the
extends upto x = d. Equation of path of electron in the
centre of the shell? [Sep. 06, 2020 (II)]
region x > d is : [Sep. 02, 2020 (I)]
Electric Charges and Fields P-237

Y ur
20. An electric dipole of moment p = (iˆ - 3 ˆj + 2kˆ) ´ 10-29 C.m
E is at the origin (0, 0, 0). The electric field due to this dipole
O X r
V0 at r = +iˆ + 3 ˆj + 5kˆ
d r ur
(note that r . p = 0) is parallel to: [9 Jan. 2020, I]

qEd qEd æ d ö (a) (+iˆ - 3 ˆj - 2kˆ) (b) (-iˆ + 3 ˆj - 2kˆ)


(a) y = (x - d ) (b) y = 2 ç
- x÷
mV02 mV0 è 2 ø (c) (+iˆ + 3 ˆj - 2kˆ) (d) (-iˆ - 3 ˆj + 2kˆ)
qEd qEd 2 21. A charged particle of mass ‘m’ and charge ‘q’ moving under
(c) y = x (d) y = x
mV02 mV02 the influence of uniform electric field Eiˆ and a uniform
18. A small point mass carrying some positive charge on it, is r
magnetic field Bk follows a trajectory from point P to Q as
released from the edge of a table. There is a uniform electric shown in figure. The velocities at P and Q are respectively,
field in this region in the horizontal direction. Which of the r r
following options then correctly describe the trajectory of vi and -2vj . Then which of the following statements
the mass ? (Curves are drawn schematically and are not to (A, B, C, D) are the correct? (Trajectory shown is
scale). [Sep. 02, 2020 (II)] schematic and not to scale) [9 Jan. 2020, I]
E Y
x
E
P B
v
y a
y y
O X
2a Q
(a) (b)
2v
x x
3 æ mv 2 ö
y y (A) E = 4 çç qa ÷÷
è ø
(c) (d) 3 æ mv 2 ö
x x (B) Rate of work done by the electric field at P is 4 çç a ÷÷
è ø
19. Consider a sphere of radius R which carries a uniform (C) Rate of work done by both the fields at Q is zero
R (D) The difference between the magnitude of angular
charge density r. If a sphere of radius is carved out of
2 momentum of the particle at P and Q is 2 mav.
ur
EA (a) (A), (C), (D) (b) (B), (C), (D)
it, as shown, the ratio ur of magnitude of electric (c) (A), (B), (C) (d) (A), (B), (C) , (D)
EB
ur ur 22. Three charged particles
field E A and E B , respectively, at points A and B due to y
the remaining portion is: [9 Jan. 2020, I] 2q –4q
B d A
150° d
30° x
O 30°
d
C
–2q

A, B and C with charges – 4q, 2q and –2q are present on


the circumference of a circle of radius d. The charged
particles A, C and centre O of the circle formed an
equilateral triangle as shown in figure. Electric field at O
along x-direction is: [8 Jan. 2020, I]
21 18 17 18 3q 2 3q 3q 3 3q
(a) (b) (c) (d) (a) (b) (c) (d)
34 34 54 54 p Î0 d 2
p Î0 d 2
4p Î0 d 2
4p Î0 d 2
P-238 Physics

23. A particle of mass m and charge q is released from rest in 27. Four point charges –q, +q, + q and –q are placed on y-axis
a uniform electric field. If there is no other force on the at y = –2d, y = –d, y = +d and y = +2d, respectively. The
particle, the dependence of its speed v on the distance x magnitude of the electric field E at a point on the x-axis at
travelled by it is correctly given by (graphs are schematic x = D, with D>> d, will behave as: [9 April 2019, II]
and not drawn to scale) [8 Jan. 2020, II]
1 1 1 1
(a) E µ 3 (b) E µ (c) E µ 4 (d) E µ 2
D D D D
v v
28. The bob of a simple pendulum has mass 2 g and a charge
(a) (b) of 5.0 ¼C. It is at rest in a uniform horizontal electric field
x x of intensity 2000 V/m. At equilibrium, the angle that the
pendulum makes with the vertical is : [8 April 2019 I]
v v (take g = 10 m/s2)
(c) (d) (a) tan–1 (2.0) (b) tan –1 (0.2)
(c) tan–1 (5.0) (d) tan –1 (0.5)
x x
24. Two infinite planes each with uniform surface charge 29. For a uniformly charged ring of radius R, the electric field
density +s are kept in such a way that the angle between on its axis has the largest magnitude at a distance h from
them is 30°. The electric field in the region shown between its centre. Then value of h is: [9 Jan. 2019 I]
them is given by: [7 Jan. 2020, I] R R
(a) (b) (c) R (d) R 2
y 5 2
30. Two point charges q1 ( 10 mC) and q2 (– 25 mC) are
30°
x placed on the x-axis at x = 1 m and x = 4 m respectively.
s é xˆ ù s é æ 3ö xˆ ù The electric field (in V/m) at a point y = 3 m on y-axis
(a) ê (1 + 3) yˆ - ú (b) êç1 + ÷ yˆ + ú is, [9 Jan 2019, II]
2 Î0 ë 2û Î0 ëêè 2 ø 2 ûú
é 1 ù
s éæ 3ö xˆ ù ê take = 9 ´ 109 Nm 2 C -2 ú
(c)
s é
2 Î0 ëê ( ) xˆ ù
1 + 3 yˆ + ú (d)

êç1 -
2 Î0 ëêè ÷
2 ø
yˆ - ú
2 ûú
ë 4p Î0 û

25. A particle of mass m and charge q has an initial velocity (a) (63 î – 27 ĵ ) × 102 (b) (– 63 î + 27 ĵ ) × 102
r r r
v = v0 $j . If an electric field E = E0 i and magnetic field (c) (81 î – 81 ĵ ) × 102 (d) (–81 î + 81 ĵ ) × 102
r 31. A body of mass M and charge q is connected to a spring
B = B0iˆ act on the particle, its speed will double after a of spring constant k. It is oscillating along x-direction about
time: [7 Jan 2020, II] its equilibrium position, taken to be at x = 0, with an
2mv0 3mv0 3mv0 2mv0 amplitude A. An electric field E is applied along the
(a) qE (b) qE (c) (d) qE0 x-direction. Which of the following statements is correct?
0 0 qE0
[Online April 15, 2018]
26. A simple pendulum of length L is placed between the
plates of a parallel plate capacitor having electric field E, 1 1 q2 E2
(a) The total energy of the system is mw2 A2 +
as shown in figure. Its bob has mass m and charge q. The 2 2 k
time period of the pendulum is given by : 2qE
(b) The new equilibrium position is at a distance:
[10 April 2019, II] k
from x = 0
qE
(c) The new equilibrium position is at a distance:
2k
from x = 0
2 2
1 2 2 1q E
(d) The total energy of the system is mw A –
2 2 k
32. A solid ball of radius R has a charge density r given by
æ rö
L L r = r0 ç1 - ÷ for 0 £ r £ R. The electric field outside
2p 2p è Rø
(a) æ qE ö (b) q2 E2
çg+ ÷ g2 - the ball is: [Online April 15, 2018]
è m ø m2
r0 R 3 4r0 R 3 3r 0 R 3 r0 R 3
L L (a) (b) (c) (d)
2p 2p e0 r 2 3e 0 r 2 4e 0 r 2 12e0 r 2
2
(c) æ qE ö (d) æ qE ö
çg- ÷ g2 + ç ÷
è m ø è m ø
Electric Charges and Fields P-239

33. A long cylindrical shell carries positive surface charge s in 37. The magnitude of the average electric field normally
the upper half and negative surface charge - s in the lower present in the atmosphere just above the surface of the
half. The electric field lines around the cylinder will look Earth is about 150 N/C, directed inward towards the center
like figure given in : (figures are schematic and not drawn of the Earth. This gives the total net surface charge carried
to scale) [2015] by the Earth to be: [Online April 9, 2014]
[Given eo = 8.85 × 10–12 C2/N-m2, RE = 6.37 × 106 m]
(a) (b) (a) + 670 kC (b) – 670 kC
(c) – 680 kC (d) + 680 kC
38. The surface charge density of a thin charged disc of radius
(c) (d) C ield Lines R is s. The value of the electric field at the centre of the
s
34. A wire of length L (=20 cm), is bent into a semicircular disc is . With respect to the field at the centre, the
2 Î0
arc. If the two equal halves of the arc were each to be
uniformly charged with charges ± Q, [|Q| = 103e0 electric field along the axis at a distance R from the centre
Coulomb where e0 is the permittivity (in SI units) of free of the disc : [Online April 25, 2013]
space] the net electric field at the centre O of the (a) reduces by 70.7% (b) reduces by 29.3%
semicircular arc would be : [Online April 11, 2015] (c) reduces by 9.7% (d) reduces by 14.6%
Y 39. A liquid drop having 6 excess electrons is kept stationary
under a uniform electric field of 25.5 kVm–1. The density of
liquid is 1.26 × 103 kg m–3. The radius of the drop is (neglect
buoyancy). [Online April 23, 2013]
(a) 4.3 × 10–7 m (b) 7.8 × 10–7 m
(c) 0.078 × 10–7 m (d) 3.4 × 10–7 m
O X O
40. In a uniformly charged sphere of total charge Q and radius
(a) (50 × 10 N/C) $j
3
(b) (50 × 103 N/C) $i R, the electric field E is plotted as function of distance
(c) (25 × 103 N/C) $j (d) (25 × 103 N/C) $i from the centre, The graph which would correspond to the
35. A thin disc of radius b = 2a has a concentric hole of radius above will be: [2012]
‘a’ in it (see figure). It carries uniform surface charge ‘s’
on it. If the electric field on its axis at height ‘h’ (h << a) E(r) E(r)
from its centre is given as ‘Ch’ then value of ‘C’ is :
[Online April 10, 2015] (a) (b)
s
(a) r
4aÎ0 r
s
(b)
8aÎ0
E(r)
s E(r)
(c)
aÎ0 (c) (d)
s
(d) r
2aÎ0 r
36. A spherically symmetric charge distribution is characterised 41. Three positive charges of equal value q are placed at
by a charge density having the following variations: vertices of an equilateral triangle. The resulting lines of
æ rö force should be sketched as in [Online May 26, 2012]
r ( r ) = ro ç1 - ÷ for r < R
è Rø
r(r) = 0 for r ³ R
Where r is the distance from the centre of the charge
distribution ro is a constant. The electric field at an internal (a) (b)
point (r < R) is: [Online April 12, 2014]

ro æ r r 2 ö ro æ r r 2 ö
(a) ç - ÷ (b) ç - ÷
4eo çè 3 4R ÷ø eo çè 3 4R ÷ø
(c) (d)
ro æ r r 2 ö ro æ r r 2 ö
(c) ç - ÷ (d) ç - ÷
3eo çè 3 4R ÷ø 12eo çè 3 4R ÷ø
P-240 Physics

42. A thin semi-circular ring of radius r has a positive charge q E(r)


ur E(r)
distributed uniformly over it. The net field E at the centre
O is [2010]
j (a) (b) r
r O R
O R

E(r) E(r)
i
O
q ˆj q ˆj
(a) 2 2 (b) -
4p e 0 r 4p e 0 r 2
2
(c) r (d)
O R r
q ˆ q ˆj O R
(c) - 2 2 j (d)
2p e 0 r 2p e 0 r 2
2

43. Let there be a spherically symmetric charge distribution 47. Two spherical conductors A and B of radii 1 mm and 2 mm
are separated by a distance of 5 cm and are uniformly
æ5 rö charged. If the spheres are connected by a conducting
with charge density varying as r(r ) = r0 çè - ÷ø upto r
4 R wire then in equilibrium condition, the ratio of the
= R , and r(r ) = 0 for r > R , where r is the distance from magnitude of the electric fields at the surfaces of spheres
A and B is [2006]
the origin. The electric field at a distance r(r < R) from the
(a) 4 : 1 (b) 1 : 2 (c) 2 : 1 (d) 1 : 4
origin is given by [2010]
48. Two point charges + 8q and – 2q are located at
r0 r æ 5 r ö 4pr0 r æ 5 r ö x = 0 and x = L respectively. The location of a point on the
(a) ç - ÷ (b) 3e çè 3 - R ÷ø
4e 0 è 3 R ø 0 x axis at which the net electric field due to these two point
r0 r æ 5 r ö r0 r æ 5 r ö charges is zero is [2005]
(c) ç - ÷ (d) 3ε çè 4 - ÷ø
4ε0 è 4 R ø 0 R L
(a) (b) 2 L (c) 4 L (d) 8 L
44. This question contains Statement-1 and Statement-2. Of 4
the four choices given after the statements, choose the 49. A charged ball B hangs from a silk thread S, which makes
one that best describes the two statements. an angle q with a large charged conducting sheet P, as
Statement-1 : For a charged particle moving from point P
shown in the figure. The surface charge density s of the
to point Q, the net work done by an electrostatic field on
sheet is proportional to [2005]
the particle is independent of the path connecting point P
to point Q.
Statement-2 : The net work done by a conservative force
on an object moving along a closed loop is zero. [2009] P
(a) Statement-1 is true, Statement-2 is true; Statement-2 q
is the correct explanation of Statement-1. S
(b) Statement-1 is true, Statement-2 is true; Statement-2
is not the correct explanation of Statement-1.
(c) Statement-1 is false, Statement-2 is true. B
(d) Statement-1 is true, Statement-2 is false.
Q (a) cot q (b) cos q (c) tan q (d) sin q
45. Let r (r ) = r be the charge density distribution for 50. Four charges equal to -Q are placed at the four corners of
p R4 a square and a charge q is at its centre. If the system is in
a solid sphere of radius R and total charge Q. For a point equilibrium the value of q is [2004]
‘P’ inside the sphere at distance r1 from the centre of the
sphere, the magnitude of electric field is : [2009] Q Q
(a) - (1 + 2 2) (b) (1 + 2 2)
Q Qr1 2 2 4
(a) 2 (b)
4p Î0 r1 4p Î0 R4 Q Q
(c) - (1 + 2 2) (d) (1 + 2 2)
4 2
Qr12
(c) (d) 0 51. A charged oil drop is suspended in a uniform field of 3×104
3p Î0 R4 v/m so that it neither falls nor rises. The charge on the
46. A thin spherical shell of radus R has charge Q spread drop will be (Take the mass of the charge = 9.9×10–15 kg
uniformly over its surface. Which of the following graphs and g = 10 m/s2) [2004]
most closely represents the electric field E(r) produced by (a) 1.6×10–18 C (b) 3.2×10–18 C
the shell in the range 0 £ r < ¥, where r is the distance from (c) 3.3×10–18 C (d) 4.8×10–18 C
the centre of the shell? [2008]
Electric Charges and Fields P-241

(a) surface change density on the inner surface is uniform


Electric Dipole, Electric Flux
TOPIC 3 and Gauss's Law Q/2
and equal to
4 pa 2
52. Two identical electric point dipoles have dipole moments (b) electric field outside the shell is the same as that of a
® ® point charge at the centre of the shell.
P1 = P$i and P2 = - P$i and are held on the x axis at distance (c) surface charge density on the outer surface depends
‘a’ from each other. When released, they move along x- r
on P
axis with the direction of their dipole moments remaining
(d) surface charge density on the inner surface of the
unchanged. If the mass of each dipole is ‘m’, their speed
shell is zero everywhere.-
when they are infinitely far apart is : [Sep. 06, 2020 (II)]
56. Let a total charge 2 Q be distributed in a sphere of radius
P 1 P 1 R, with the charge density given by r(r) = kr, where r is
(a) (b) the distance from the centre. Two charges A and B, of – Q
a pe 0 ma a 2pe 0 ma
each, are placed on diametrically opposite points, at equal
P 2 P 2 distance, a, from the centre. If A and B do not experience
(c) (d)
a pe 0 ma a 2 pe 0 ma any force, then. [12 April 2019, II]
ur 3R
53. An electric field E = 4 xiˆ - ( y 2 + 1) ˆj N/C passes through (a) a = 8–1/4 R (b) a =
the box shown in figure. The flux of the electric field 21/ 4
through surfaces ABCD and BCGF are marked as f1 and (c) a = 2–1/4 R (d)
a = R/ 3
f11 respectively. The difference between (f1 – f11) is (in
57. An electric dipole is formed by two equal and opposite
Nm2/C) _______. [9 Jan 2020, II]
charges q with separation d. The charges have same mass
z
m. It is kept in a uniform electric field E. If it is slightly
A (0, 0, 2) B
(3, 0, 2)
rotated from its equilibrium orientation, then its angular
frequency w is : [8 April 2019, II]
D C
(0, 2, 2)
(3, 2, 2) qE 2qE qE qE
E F (a) (b) (c) 2 (d)
x md md md 2md
(0, 0, 0) (3, 0, 0)
H G
58. An electric field of 1000 V/m is applied to an electric dipole
(0, 2, 0) (3, 2, 0)
at angle of 45°. The value of electric dipole moment is
y 10–29 C.m. What is the potential energy of the electric
54. In finding the electric field using Gauss law the formula dipole? [11 Jan 2019, II]
r q (a) –20 × 10–18 J (b) –7 × 10 –27 J
| E | = enc is applicable. In the formula Î is (c) –10 × 10–29 J (d) – 9 × 10–20 J
Î | A|
0
0

permittivity of free space, A is the area of Gaussian surface 59. Charges – q and + q located at A and B, respectively,
and qenc is charge enclosed by the Gaussian surface. This constitute an electric dipole. Distance AB = 2a, O is the
equation can be used in which of the following situation? mid point of the dipole and OP is perpendicular to AB.
A charge Q is placed at P where OP = y and y >> 2a. The
[8 Jan 2020, I]
charge Q experiences an electrostatic force F. If Q is now
(a) Only when the Gaussian surface is an equipotential moved along the equatorial line to P¢ such that OP¢
surface.
Only when the Gaussian surface is an æ yö æy ö
r = ç ÷ , the force on Q will be close to: ç >> 2a ÷
3
è ø è 3 ø
(b) equipotential surface and | E | is constant on the surface. P [10 Jan 2019, II]
r
(c) Only when | E | = constant on the surface.
(d) For any choice of Gaussian surface.
55. Shown in the figure is a shell made of a conductor. It has
inner radius a and outer radius b, and carries charge Q. At Q P¢
ur
its centre is a dipole p as shown. In this case : O
A B
[12 April 2019, I] –q +q

F
(a) 3 F (b) (c) 9 F (d) 27 F
3
P-242 Physics

60. A charge Q is placed at a distance a/2 above the centre of the through a circular surface of radius 0.02 m parallel to the Y-
square surface of edge a as shown in the figure. The electric Z plane is nearly: [Online April 19, 2014]
flux through the square surface is: (a) 0.125 Nm2/C (b) 0.02 Nm2/C
[Online April 15, 2018] (c) 0.005 Nm /C 2 (d) 3.14 Nm2/C
ur ur
Q 65. Two point dipoles of dipole moment p1 and p 2 are at a
(a) 3e P ur ur
0 distance x from each other and p1 || p 2 . The force between
a/2
Q the dipoles is : [Online April 9, 2013]
(b) 6e 1 4 p1 p2 1 3 p1 p2
0 (a) (b)
4pe0 x 4 4pe0 x3
Q
(c) 2e 1 6 p1 p2 1 8 p1 p2
0 (c) (d)
4pe0 x 4 4pe0 x 4
Q a
66. The flat base of a hemisphere of radius a with no charge
(d) e inside it lies in a horizontal plane. A uniform electric field
0
ur ® p
61. An electric dipole has a fixed dipole moment p , which E is applied at an angle with the vertical direction. The
4
makes angle q with respect to x-axis. When subjected to
uur ur electric flux through the curved surface of the hemisphere
an electric field E1 = Eiˆ , it experiences a torque T1 = t iˆ . is [Online May 19, 2012]
uur
When subjected to another electric field E2 = 3E1 ˆj it 45° ®
uur ur E
experiences torque T2 = -T1 . The angle q is : [2017]
(a) 60° (b) 90° (c) 30° (d) 45°
62. Four closed surfaces and corresponding charge distribu-
tions are shown below. [Online April 9, 2017]

5q
q 8q
2q –q 3q
–2q
q q q q pa2 E
–4q (a) pa2 E (b)
S1 S2 S3 2
S4
Let the respective electric fluxes through the surfaces be pa2 E
( p + 2) pa 2 E
F 1, F 2, F 3, and F 4. Then : (c)
2 2
(d)
2 2 (
2
)
(a) F 1< F2 = F 3 > F4 (b) F 1> F2 > F 3 > F4 67. An electric dipole is placed at an angle of 30° to a non-
(c) F 1= F2 = F 3 = F4 (d) F 1> F 3 ; F 2 < F4 uniform electric field. The dipole will experience [2006]
63. The region between two concentric spheres of radii 'a' and (a) a translational force only in the direction of the field
'b', respectively (see figure), have volume charge density (b) a translational force only in a direction normal to
A the direction of the field
r= , where A is a constant and r is the distance from
r (c) a torque as well as a translational force
the centre. At the centre of the spheres is a point charge (d) a torque only
Q. The value of A such that the electric field in the region
between the spheres will be constant, is: [2016] 68. If the electric flux entering and leaving an enclosed surface
respectively is f1 and f2, the electric charge inside the surface
will be [2003]
(a) (f2 – f1)eo (b) (f1 – f2)/eo
a
(c) (f2 – f1)/eo (d) (f1 – f2)eo
Q
b 69. A charged particle q is placed at the centre O of cube of
length L (A B C D E F G H). Another same charge q is
placed at a distance L from O. Then the electric flux
2Q 2Q through ABCD is E F
[2002]
( )
(a) (b)
p a -b2 2
pa 2 D
c
O
q q
Q Q H

( )
(c) (d) G
2 pa 2 2 p b2 - a 2 A
B
L
64. The electric field in a region of space is given by,
r (a) q /4 p Î0 L (b) zero
E = Eoˆi + 2Eoˆj where Eo = 100 N/C. The flux of the field (c) q/2 p Î0 L (d) q/3 p Î0 L
Electric Charges and Fields P-243

d y
1. (a) Fa Fb
+Q d/2 q d/2 +Q F F
Force due to charge + Q, 5. (a) x
q a a q
KQQ
Fa = 2
d Þ F sin q
Force due to charge q, F sin q
KQq
Fb = 2F cos q
2
ædö Þ Fnet = 2F cosq
ç ÷
è2ø
For equilibrium, æqö
2kq ç ÷
r r è2ø y
Fa + Fb = 0 Fnet = 2
×
æ y2 + a2 ö y + a2
2
kQQ kQq ç ÷
Þ + Q è ø
= 0 \q = -
d2 ( d / 2) 2
4 æqö
2kq ç ÷ y
2
Fnet = 2 è 2 ø 3/2
R
A –2r/a
2. (b) Q = ò rdv = ò
r2
e ( 4pr 2dr ) (y +a )
(Q y << a)
0

kq2 y
R
æ ö
R
ç e –2r/a ÷ Þ So, F µ y
= 4 pA ò e dr = 4 p A ç
–2r/a
÷ a3
0 ç –2 ÷ 6. (d)
è a ø0
æ aö
(
= 4pA ç – ÷ e –2R/a –1
è 2ø
) l
q

Q = 2paA(1–e–2R/a) dr
Tcos q
q
æ ö
ç ÷ r Tsin q
a 1 q Fe
R = log ç ÷ q
ç 1– Q ÷
2 x
è 2paA ø
mg
3. (d) Spheres A and B carry equal charge say 'q'
In equilibrium, Fe = T sin q
kqq
\ Force between them, F = 2 mg = T cos q
r
q
When A and C are touched, charge on both q A = q C = Fe q2
2 tan q = =
Then when B and C are touched, charge on B mg 4p Î0 x 2 ´ mg
q
+q x/2
3q also tan q » sin =
qB = 2 = l
2 4
Now, the force between charge qA and qB x q2
q 3q Hence, 2l =

´ 2 4p Î0 x 2 ´ mg
kq A q B 2 4 = 3 kq = 3 F
F' = = 2q 2 l
r2 r2 8 r2 8 Þ x3 =
4. (c) Inside the cavity net charge is zero. 4p Î0 mg
\ Q1 = 0 and s1 = 0 æ q2l ö
1/3

There is no effect of point charges +Q, –Q and induced \ x = çç ÷÷


charge on inner surface on the outer surface. è 2p Î0 mg ø
\ Q2 = 0 and s2 = 0 Therefore x µ l1/3
P-244 Physics

7. (d) From figure x is distance between the spheres. When third spherical
T cos q = mg ....(i) conductor comes in contact with B charge on B is halved
T sin q = Fe ....(ii) Q Q
i.e., and charge on third sphere becomes . Now it is
2 2
Dividing equation (ii) by (i), we get
touched to C, charge then equally distributes themselves
sin q Fe to make potential same, hence charge on C becomes
Þ cos q = mg Þ Fe = mg tan q
æ Q ö 1 3Q
.
kq 2 x 2mg tan q çè Q + ÷ø =
Þ = mg tan q Þ q2 = 2 2 4
x2 k
Since q is small æ 3Q ö æ Q ö
QC¢ QB¢ ç ÷ç ÷ 3 Q2
4 øè 2 ø
\ tan q » sin q =
x \ Fnew = k =kè = k
2l x2 x2 8 x2
x 3mg 3
\ q2 = Þ q2 µ x3/2 or Fnew = F
2 kl 8
11. (b) Force applied by charge q2 on q1
qq
l q l F12 = k 1 22
Tcosq b
Force applied by charge q3 on q1
T
q Tsinq
Fe qq F12
x F13 = k 1 23
a F13 sin q
mg The X-component of net q
force (Fx) on
dq 3 dx 3 q1 is F12 + F13 sin q
Þ a x = xV
dt 2 dt 2 qq qq
\ Fx = k 1 22 + k 1 22 sin q
dq b a F13
Since = const. F13cos q
dt q q
Þ v µ x–1/2 [Q q2 µ x3] \ Fx µ 22 + 32 sin q
8. (d) Let F be the force between Q and Q. The force b a
between q and Q should be attractive for net force on Q to 12. (d) At equilibrium net force is zero,
be zero. Let F¢ be the force between Q and q. The resultant Q ´Q Qq
of F¢ and F¢ is R. For equilibrium \k 2
+k 2 =0
p(q)
(2 x ) x
A(Q)
x x
l R F¢ Q q Q
Q
Q
Þ q=-
D(q) 4
F¢ C
F kQ2
Net force on Q at C is zero. 13. (c) Electric field due charge Q2, E2 =
r r x22
\ R+ F = 0 Þ 2 F¢ = -F
kQ1
Qq Q2 Electric field due charge Q1, E1 =
Þ 2´k = -k x12
2 2
l ( 2 l) Q1 A
Q
Þ = -2 2
q
x1
9. (b) It is obvious that by charge conservaiton law,
electronic charge does not depend on acceleration due to q
gravity as it is a universal constant. O 90–q q
E2 Q2
So, electronic charge on earth x2 B
= electronic charge on moon q
\ Required ratio = 1. Enet E1
C From figure,
B
10. (d) × E x kQ2 x
r r tan q = 2 = 1 Þ = 1
E1 x2 kQ x2
Q Q x22 ´ 21
Initial force, F = K B 2 C x1
x
Electric Charges and Fields P-245

y
Q2 x12 x1 Q x Q x v0
Þ 2
= Þ 2 = 2 or, 1 = 1 . t=0
Q1 x2 x 2 Q1 x1 Q2 x 2
14. (c) For spherical shell
1 Q
E= (if r ³ R ) (d, –y0)
4pe 0 r 2
=0 (if r < R) P q vx
vy vnet
Force on charge in electried field, F = qE
\F = 0 (For r < R) vy qEt0 æ d ö
tan q = = , çt = ÷
1 Qq vx m × v0 è v0 ø
F= (For r > R)
4pe 0 r 2
15. (c) The electric field produced due to uniformly charged qEd , Slope = - qEd
tan q =
infinite plane is uniform. So option (b) and (d) are wrong. m × v02 mv02
And +ve charge density s+ is bigger in magnitude so its r
field along Y direction will be bigger than field of –ve charge No electric field Þ Fnet = 0, v = const.
density s– in X direction. Hence option (c) is correct.
–s ì qEd ü
E1 ïm = ï
ER ER –s y = mx + c, í mv02 ý
E1 ï(d , - y ) ï
E î 0 þ
E
E2 E2 -qEd qEd 2
+s - y0 = , d + c Þ c = - y0 +
+s mv02 mv02
1
E - qEd qEd 2
E y= x - y0 +
mv02 mv02
16. (c) Given,
2
Electric field, E = E0 (1 - x 2 ) 1 qE æ d ö 1 qEd 2
y0 = × =
2 m çè v0 ÷ø 2 mv02
\ Force, F = qE = qE0 (1 - x2 )
- qEdx 1 qEd 2 qEd 2
y= - +
dv æ dv ö mv02 2 mv02 mv02
Also, F = ma = mv çèQ a = v ÷ø
dx dx
- qEd 1 qEd 2 qEd æ d ö
dv y= + Þy= 2 ç
- x÷
\ mv = qE0 (1 - x 2 ) mv0 2
2 mv0 2
mv0 è 2 ø
dx 18. (d) Net force acting on the particle,
qE0 (1 - x 2 )dx r
Þ v dv = F = qEiˆ + mgjˆ
m Net acceleration of particle is constant, initial velocity is
Integrating both sides we get, zero therefore path is straight line.
v x 2E
qE0 (1 - x 2 )dx ax =
Þ ò v dv = ò
m m
0 0

v 2 qE0 æ 9 x3 ö
Þ = çç x - ÷=0 2
2 m è 3 ÷ø a=
æ 2E ö 2
çè ÷ +g
ay = g mø
3
Þx= æ Rö
a 19. (b) Electric field at A ç R ' = ÷
è 2ø
17. (b) Fx = 0, ax = 0, (v)x = constant
q
d E A .ds =
Time taken to reach at ' P ' = = t0 (let) ...(i) e0
v0 B A
3
4 æRö
1 qE 2 r´ p ç ÷
(Along – y), y0 = 0 + × × t0 ...(ii) r 3 è2ø 3R
2 m Þ EA = R/2
2 2
æRö
e 0 × 4p ç ÷
è2ø
P-246 Physics

r s ( R / 2 ) æ sR ö Electric field due to charge +2q at centre O


Þ EA = =ç ÷
3e0 r 2q é 3iˆ – ˆj ù
è 6e 0 ø E1 =
1
´ 2ê ú
Electric fields at ‘B’ 4pe0 d ë 2 û
3 Electric field due to charge –2q at centre O
4 4 æRö
r k ´ r´ pR 3 k ´ r´ p ç ÷ r
3 3 è2ø 1 2q é 3iˆ – ˆj ù
EB = - E2 = ´ 2ê ú
2 2 4pe 0 d ë 2 û
R æ 3R ö
ç ÷ Electric field due to charge –4q at centre O
è 2 ø
r 1 4q é 3iˆ + ˆj ù
r sR æ 1 ö ( s ) 4p æ R ö
3
E3 = ´ 2ê ú
Þ EB = -ç ÷ ç ÷ 4pe0 d ë 2 û
3e 0 è 4pe0 ø æ 3R ö2 3 è 2 ø
\ Net electric field at point O
ç ÷
è 2 ø r r r r 3q ˆ
r E0 = E1 + E2 + E3 = i
sR sR pe 0 d 2
Þ EB = -
3e0 54e 0 v
23. (b)
17 æ sR ö
Þ EB = ç ÷
54 è e0 ø

E A 1´ 54 æ 9 ö 9 2 18
= =ç ÷= ´ = x
EB 6 ´17 è 17 ø 17 2 34
Using
r r v2 – u2 = 2aS ...(i)
20. (c) Since r × p = 0
Here, u = 0, s = x
r r
E must be antiparallel to p Also, Felectric = ma
qE qE
(
\ Ê is parallel to iˆ + 3 ˆj - 2kˆ ) Þ qE = ma Þ a =
m
Þ a=
m
21. (c) (A) By work energy theorem Substituting the values in (i) we get
1 2 1
2qE
Wmg + Wele = m ( 2v ) - m ( v )
2 v2 = .x
2 2 m

3 mv2 24. (d) E1 y


3
0 + qE0 2a = mv 2 Þ E0 = P +s
2 4 qa 60°
E2 30°
(B) Rate of work done at P = power of electric force x
3 +s
3 mv
= qE0V = From figure,
4 a
r s r s
dw E1 = yˆ and E2 = (– cos 60° xˆ – sin 60° yˆ )
(C) At, Q, = 0 for both the fields 2e 0 2e0
dt
(D) The difference of magnitude of angular momentum s æ 1 3 ö
= çç – xˆ – yˆ ÷
of the particle at P and Q, 2e 0 è 2 2 ÷ø
r
( ) (
DL = - m2v 2akˆ - -mvakˆ ) Electric field in the region shown in figure (P)
r r r r s é 1 æ 3ö ù
DL = 3mva EP = E1 + E2 = ê – xˆ + çç 1– ÷ yˆ ú
2e0 ëê 2 è 2 ÷ø ûú
22. (a) y-axis
r s éæ 3ö xˆ ù
or, E P = êçç1 – ÷÷ yˆ – ú
+ 2q –4q 2e 0 êëè 2 ø 2 úû
B A
150° d 25. (c) In the x direction
30°
Fx = qE
x-axis Þ max = qE
O 30°
E q
Þ ax = 0
C m
–2q
For speed to be double,
Electric Charges and Fields P-247

29. (b) Electric field on the axis of a ring of radius R at a


v02 + v x2 = (2v0 )2 distance h from the centre,
Þ vx = 3 v0 = ax t kQh
E=
(h )
3/2
qE0t 3v0 m 2
+ R2
Þ 3v0 = 0 + Þ t=
m E0 q
26. (d) Time period of the pendulum (T) is given by dE
Condition: for maximum electric field =0
dh
L
T = 2p é ù
geff d ê kQh ú
Þ ú =0
dh ê 2
geff =
(mg )2 + (qE )2 ê R +h
ë ( 2 3/2
ú
û )
m By using the concept of maxima and minima we get,
L R
2 Þ T = 2p h=
æ gE ö 2 2
Þ geff = g 2 + ç ÷ æ qE ö
g2 + ç
è m ø ÷ 30. (a)
è m ø
® ® ® ® ®
27. (d) E = ( E + E ) + ( E + E )
1 2 3 4
y=3
or E = 2E cos a – 2E cos b

–q
d
q E3
d
a (0, 0) x

d D E1 r r
Let E1 and E2 are the vaues of electric field due to charge,
q q1 and q2 respectively
d
magnitude of E = 1 q1
–q 1
4p Î0 r12
2kq D 2 kq D
´ - ´
= ( D2 + d 2 ) 2 2 2
( D + (2 d ) 2 1 10 ´ 10 -6
D +d D + (2d )2
2
=
2 kqD 2kqD (
4p Î0 12 + 32 ) 3
10
= - q1
( D 2 + d 2 ) 3/2 [ D 2 + (2d ) 2 ]3/2 = ( 9 ´ 109 ) ´ 10 ´ 10-7
For d < < D
= 9 10 ´ 10 2
D 1 r r r
Eµ 3
µ \ E1 = 9 10 ´ 10 2 éëcos q1 ( - i ) + sin q1 j ùû
D D2
é 1 3 ˆù
28. (d) At equilibrium resultant force on bob must be zero, so
T cos q = mg ..... (i)
Þ E1 = 9 ´ 10 ´ 10 2 ê
ë 10
-iˆ + ( )
10 úû
j
T sin q = qE ..... (ii) Y
Solving (i) and (ii) we get Þ E1 = 9 ´102 éë -iˆ + 3 ˆj ùû = éë –9iˆ + 27 ˆj ùû102
qE 1 q2
tan q = q Similarly, E2 =
mg q 4 p Î0 r 2
T
qE X
5 ´10-6 ´ 2000 1 9 ´109 ´ ( 25 ) ´ 10 -6
tan q = = q E2 = E2 = 9 × 103 V/m
2 ´ 10-3 ´ 10 2
mg ( 4r + 3 )
2 2

3
[Here, q = 5 × 10–6 C,
E = 2000 v/m, m = 2 × 10–3 kg]
( )
\ E2 = 9 ´ 103 cos q2iˆ - sin q2 ˆj Q tan q2 =
4
r æ4 3 ö
æ1ö
Þ tan -1 ç ÷ (
\ E2 = 9 ´103 ç iˆ - ˆj ÷ = 72iˆ - 54 ˆj ´102
è5 5 ø
)
è2ø
r r r
(
\ E = E1 + E 2 = 63iˆ - 27 ˆj ´ 10 2 V/m )
P-248 Physics

31. (a) Equilibrium position will shift to point where resultant 35. (a) Electric field due to complete disc (R = 2a) at a
force = 0 distance x and on its axis
qE
kxeq = qE Þ xeq = s é ù x s é h ù
k E1 = ê1 – ú E1 = 2e ê1 – ú
2e 0 ê 2 2 ú 0 ëê 4 a 2
+ h 2 ú
û
1 2 2 1 2 ë R +x û
Total energy = mw A + kx eq
2 2 s é hù é here x =h ù
= 1–
1 1q E 2 2 2e0 êë 2a úû êë and, R = 2a úû
Total energy = mw 2 A 2 +
2 2 k 2a
a
æ rö
o
32. (d) Charge density, r = r0 ç1 - ÷
è Rø
dq = rdv

ò
qin = dq = rdv Similarly, electric field due to disc (R = a)
æ rö s æ hö
= r0 ç 1 - ÷ 4pr 2 dr (Q dv = 4pr2dr) E2 = ç 1– ÷
è Rø 2e0 è a ø
Ræ Electric field due to given disc

= 4pr0 ò0 çè1 - R ÷ø r 2dr E = E1 – E2

s é hù s é hù sh
R r2 ê1– 2a ú – 2e ê1– a ú =
= 4pr0 ò0 r 2 dr -
R
dr 2e 0 ë û 0 ë û 4e0 a
s
éé 3 ù R é 4 ù R ù é R3 R 4 ù Hence, c =
r r 4ae 0
= 4pr0 ê ê ú - ê ú ú = 4pr0 ê - ú
êê 3 ú ê 4R ú ú êë 3 4 R úû
ëë û0 ë û0 û 36. (b) Let us consider a spherical shell of radius x and
thickness dx.
é R3 R 3 ù é R3 ù
= 4pr0 ê - ú = 4pr0 ê ú
ëê 3 4 úû ëê 12 ûú
dx
pr R3 x
q= 0 O
3
æ pr R3 ö Shell
E.4pr 2 = ç 0 ÷
è 3Î0 ø
r0 R3 Charge on this shell
\ Electric field outside the ball, E =
12 Î0 r 2 æ xö
dq = r.4px2dx = r0 ç1 - ÷ .4px 2dx
33. (c) Field lines originate perpendicular from positive è Rø
charge and terminate perpendicular at negative charge. \ Total charge in the spherical region from centre
Further this system can be treated as an electric dipole. to r (r < R) is
34. (d) Given: Length of wire L = 20 cm r
æ xö
charge Q = 103e0 q = ò dq = 4pr0 ò ç1 - ÷ x 2dx
è Rø
0
We know, electric field at the centre of the semicircular arc
r
2K l é x3 x 4 ù é r3 r 4 ù 3 é1 r ù
E= = 4pr0 ê - ú = 4pr0 ê - ú = 4pr0r ê - ú
r êë 3 4R úû 0 êë 3 4R úû ë 3 4R û
æ 2Q ö 1 .q
2K ç ÷ \ Electric field at r, E =
or, E= è pr ø é Asl = 2Q ù 4pe 0 r 2
r ê pr úû
ë
1 4 pr0 r3 é 1 r ù r0 é r r 2 ù
4 KQ 4 KQp 2
4pKQ = . ê - ú = ê - ú
= = = = 25 ´ 103 N / Ci$ 4pe 0 r 2 ë 3 4R û e0 êë 3 4R úû
2 2 2
pr pL L
Electric Charges and Fields P-249

37. (c) Given,


Electric field E = 150 N/C
Total surface charge carried by earth q = ? dx
or, q = Î0 E A x
= Î0 E p r2.
= 8.85 × 10–12 × 150 × (6.37 × 106)2.
; 680 Kc
As electric field directed inward hence
q = – 680 Kc é 5 r3 1 r 4 ù 3æ5 r ö
38. (a) Electric field intensity at the centre of the disc. = 4pr0 ê . - . ú = pr0r ç - ÷
êë 4 3 R 4 úû è3 Rø
s Electric field at r,
E= (given) \
2 Î0
1 q
E= . 2
Electric field along the axis at any distance x from the 4p Î0 r
centre of the disc 1 pr0 r 3 æ 5 r ö r0 r æ 5 r ö
æ ö = . ç - ÷= ç - ÷
s x 4pÎ0 r 2 è 3 R ø 4 Î0 è 3 R ø
E' = ç1 - ÷
ç
2 Î0 2 2 ÷ 41. (c) Electric lines of force due to a positive charge is
è x + R ø
From question, x = R (radius of disc) spherically symmetric.
All the charges are positive and equal in magnitude. So
æ ö
\ E ' = s ç1 - R
÷ repulsion takes place. Due to which no lines of force are
2 Î0 çè R + R2
2 ÷
ø present inside the equilateral triangle and the resulting
lines of force obtained as shown:
s æ 2R - R ö
= ç ÷
2 Î0 çè 2R ÷ø +q
4
= E
14
+q +q
\ % reduction in the value of electric field
æ 4 ö
çè E - E÷ø ´ 100
14 1000
= = % ; 70.7%
E 14
42. (c) Let us consider a differential element dl subtending at angle
39. (b) F = qE = mg (q = 6e = 6× 1.6 × 10–19) dQ at the centre Q as shown in the figure. Linear charge density
q
mass m l=
Density (d) = = Qr
volume 4 3
pr j
3
m
or r3 =
+
+

4 + dl
+

pd
3 + dq
+
æ qE ö
Putting the value of d and m ç = + dE q
+
and solving we get r cos q
è g ÷ø i
= 7.8 × 10–7 m O
40. (a) Let us consider a spherical shell of radius x and dE
thickness dx. dE sin q
Charge on this shell æ q ö
æ5 x ö Charge on the element, dq = ç ÷ dl
2
dq = r.4px 2 dx = r0 ç 4 - R ÷ .4px dx è pr ø
è ø q
= (rd q) (Q dl = rdq)
\ Total charge in the spherical region from centre to r (r < R ) is pr
r
æ5 xö æqö
q = ò dq = 4 pr0 ò ç - ÷ x 2 dx = ç ÷dq
0
è4 Rø è pø
P-250 Physics
Electric field at the center O due to dq is Let us consider a spherical shell of thickness dx and radius
1 dq 1 q x. The area of this spherical shell = 4px2.
dE = × 2 = × 2 dq The volume of this spherical shell = 4px2dx. The charge
4 p Î0 r 4 p Î0 pr
enclosed within shell
Resolving dE into two rectangular component, we find
the component dE cos q will be counter balanced by é Q.x ù 4Q
another element on left portion. Hence resultant field at O dq = ê 4ú
[4px2dx] = 4 x3dx
is the resultant of the component dE sin q only. ë pR û R
p The charge enclosed in a sphere of radius r1 can be
q
\ E = ò dE sin q = ò 4 p2 r 2 Î sin qd q calculated by
0 0 r1 r1
q p 4Q é x 4 ù
= [ - cos q] 4Q Q 4
Q = ò dq =
ò
3
0 x dx = ê ú = 4 r1
4p2 r 2 Î0 R4 4 4
R êë úû R
q q 0 0
= (+1 + 1) = \ The electric field at point P inside the sphere at a distance
4p2 r 2 Î0 2p2 r 2 Î0 r1 from the centre of the sphere is
The direction of E is towards negative y-axis.
r q 1 Q
\ E=- ˆj E=
2 2
2p r Î0 4pE r12
43. (a) Let us consider a spherical shell of radius x and é Q 4ù
thickness dx. r
Due to shpherically symmetric charge distribution, the 1 êë R 4 1 úû 1 Q 2
Þ E= = r1
chrge on the spherical surface of radius x is 4p Î0 r12 4p Î0 R 4
æ5 xö 2 46. (a) The electric field inside a thin spherical shell of radius
dq = dVr×4px2dx = r0 ç - ÷ × 4px dx
è 4 R ø R has charge Q spread uniformly over its surface is zero.
\ Total charge in the spherical region from centre to r (r < R) is Q ++ + +
r +

+
æ5 xö R
+
q = ò dq = 4 pr0 ò ç - ÷ x 2 dx

+ + +
+++
0
è4 Rø Q
E=k 2
r
++

+ +
E=0
+

+
dx
+ + +
++
x
Q
Outside the shell the electric field is E = k 2 . These
r
characteristics are represented by graph (a).
+Q +Q2
1
47. (c)
é 5 r3 1 r4 ù 3æ5 r ö r1 r2
= 4 pr0 ê × - × ú = pr0r ç - ÷
ëê 4 3 R 4 ûú è3 Rø A B
\ Electric field intensity at a point on this spherical surface When the two conducting spheres are connected by a
1 q conducting wire, charge will flow from one to other till both
E= × 2
4 p Î0 r acquire same potential.
\ After connection, V1 = V2
1 pr0 r 3 æ 5 r ö r0 r æ 5 r ö Q Q Q Q
= × ç - ÷= - Þk 1 =k 2 Þ 1 = 2
4p Î0 r 2 è 3 R ø 4 Î0 çè 3 R ÷ø r1 r2 r1 r2
44. (a) The ratio of electric fields
Q
45. (b) k 21
R E1 r E Q r2
= 1 Þ 1 = 21 ´ 2
E2 k Q2 E2 r1 Q2
r22
x dx E1 r1 ´ r22 E r 2
Þ = Þ 1 = 2 =
E2 r12 ´ r2 E2 r1 1
Electric Charges and Fields P-251

- K 2q K 8q 52. (b) Let v be the speed of dipole.


48. (b) At P + =0 Using energy conservation
( x - L)2 x2
Ki + U i = K f + U f
1 4
Þ = 2k × p1 1 2 1 2
( x - L) 2 x 2 Þ 0- p2 cos (180°) = mv + mv + 0
1 2 r3 2 2
or =
x-L x æ
Þ x = 2x – 2L or x = 2L ç
çQ Potential energy of interaction between dipole
ç
è
-2 p1 p2 cos q ö
+8q –2q = ÷
4p Î0 r 3 ÷ø
x=0 x=L P
L
2kp1 p2 2kp1 p2
Þ mv 2 = 3
Þv=
x r mr 3
When p1 = p2 = p and r = a
49. (c) P
T cos q p 1
v=
q T a 2p Î0 ma
s
F = Eq = q 53. (–48)
e0K r r
Flux of electric field E through any area A is defined as
T sin q
f = ò E. A cos q
mg Here, q = angle between electric field and area vector of a
surface
T sin q = qE .... (i) For surface ABCD Angle, q = 90°
T cos q = mg .... (ii) \ f1 = ò E. A cos 90° = 0
Dividing (i) by (ii), r uur
qE q æ s ö sq For surface BCGF fn = ò E . dA
tan q = =
mg mg çè e0K ÷ø e0 K . mg \ f11 = é 4 ´ iˆ – ( y 2 + 1) ˆj ù .4iˆ = 16 x
ë û
\ s µ tan q
50. (b) For the system to be equilibrium, net field at A should Nm 2
f11 = 48
be zero C
2 E1 + E2 = E3 f1 – f11= – 48
54. (a)
kQ ´ 2 kQ kq 55. (b) Surface charge density depends only due to Q. Also
\ + =
2 2 2 ® ® q1l
a ( 2 a) æ a ö
çè ÷

Ñò E . d A = e0
E3 –Q –Q B Q 1 Q
or E × 4pr2 = ÞE= ,r³R
E1 e0 4pe 0 r 2
A
E2 ® ® qin
2 E1
56. (a) Ñò E . d A = e 0
E A –Q –Q
2Q
Q 2 Q Q
Þ + = 2q Þ q = (2 2 + 1)
1 2 4
51. (c) Given, Electric field, E = 3 × 104
Mass of the drop, m = 9.9 × 10–15 kg –Q –Q
At equilibrium, coulomb force on drop balances weight of drop. a a
qE = mg
mg 9.9 ´ 10-15 ´ 10
Þ q= Þ q = 4
= 3.3 ´ 10 -18 C
E 3 ´ 10
P-252 Physics

1 KP
or E × 4pr2 = ò S (4pr 2 )dr \ At point P, = + Q
e0 y3
r KPQ
1 At Point P1, F1 = + = 27 F.
(kr )(4 pr 2 )dr
e0 ò
or E × 4pr2 = ( y / 3)3
0
60. (b) When cube is of side a and point charge Q is at the
4pk æ r 4 ö center of the cube then the total electric flux due to this
or E × 4pr2 = ç ÷
e0 çè 4 ÷ø charge will pass evenly through the six faces of the cube.
k 2 So, the electric flux through one face will be equal to 1/6
\ E= r ...(i) of the total electric flux due to this charge.
4e 0
Q
R R Flux through 6 faces =
2 r4 Îo
Also 2Q = ò (kr ) (4pr )dr = 4pk
4 Q
0 0
\ Flux through 1 face, =
pkR 4 6 Îo
Q= ....(ii) 61. (a) T = PE sin q Torque experienced by the dipole in an
2 r r r
From above equations, electric field, T = P ´ E
Qr 2 r
p = p cosq iˆ + p sin q ĵ
E= ....(iii) r r
2pe0 R 4 E1 = Ei
According to given condition
r r r
Q4 T1 = p ´ E1 = (p cos q iˆ + p sin q ĵ ) × E( iˆ )
= EQ ....(iv)
4pe0 (20) 2 t k̂ = pE sinq (– k̂ ) ...(i)
r
From equations (iii) and (iv), we have E2 = 3 E1 j ˆ
r
a = 8–1/4 R. T2 = p cos qiˆ + p sin qˆj ) ´ 3 E1 ˆj
57. (b) t = – PE sin q
or Ia = – PE (q) tkˆ = 3 pE1 cos qkˆ ...(ii)
PE From eqns. (i) and (ii)
a= ( -q ) pE sinq = 3 pE cosq
I
On comparing with tanq = 3 \ q = 60°
a = – w2q
62. (c) The net flux linked with closed surfaces S1, S2, S3 & S4
PE qdE 2qE are
w= = = 1
I 2 md
ædö For surface S1, f1 = (2q)
2m ç ÷ e0
è2ø 1 1
For surface S2, f 2 = (q + q + q - q) = 2q
e0 e0
58. (b) Potential energy of a dipole is given by 1 1
rr For surface S3, f3 = (q + q) = (2q)
U = – P.E e0 e0
= – PE cos q 1 1
For surface S4, f 4 = (8q - 2q - 4q) = (2q)
[Whereq = angle between dipole and perpendicular to the e0 e0
field] Hence, f1 = f2 = f3 = f4 i.e. net electric flux is same for all
= – (10–29) (103) cos 45° surfaces.
= – 0.707 × 10–26 J = – 7 × 10–27J Keep in mind, the electric field due to a charge outside (S3
59. (d) Electric field of equitorial plane of dipole and S4), the Gaussian surface contributes zero net flux
r through the surface, because as many lines due to that
KP charge enter the surface as leave it.
=– 3
r 63. (c) Applying Gauss’s law
r uur Q
Ñò S E × ds = Î0
Q + 2pAr 2 - 2pAa 2
\ E × 4pr2 =
Î0
Electric Charges and Fields P-253

dr 66. (b) We know that,


r=
Ñò E.dS = E Ñò dS
dV r dr
f= cos 45°
Q a Gaussiam
Q = r4pr2 In case of hemisphere
surface
b fcurved = fcircular
r
A
Q= ò 4pr 2dr = 2pA[r2 – a2] 1 E pa 2
r Therefore, fcurved = E pa 2 . =
a 2 2
1 é Q - 2pAa 2 ù 67. (c)
E= ê + 2pA ú +q
4p Î0 ëê r 2
ûú F1
For E to be independent of ‘r’ E1
Q – 2pAa2 = 0
F2
Q E2
\ A=
2pa 2 –q
® As the dipole is placed in non-uniform field, so the force
64. (a) E = E 0 iˆ + 2E 0 ˆj acting on the dipole will not cancel each other. This will
Given, E 0 = 100N / c result in a force as well as torque.
® 68. (a) The electric flux f1 entering an enclosed surface is
So, E = 100iˆ + 200ˆj taken as negative and the electric flux f2 leaving the surface
Radius of circular surface = 0.02 m is taken as positive, by convention. Therefore the net flux
leaving the enclosed surface, f = f2 – f1
2 22 According to Gauss theorem
Area = pr = ´ 0.02 ´ 0.02
7 q
f= Þ q = Î0f = Î0(f2 – f1)
= 1.25 ´10-3 ˆi m 2 [Loop is parallel to Y-Z plane] Î0
Now, flux (f) = EA cosq 69. (None) Electric flux due to charge placed outside is zero.
( ) -3
= 100iˆ + 200ˆj .1.25 ´ 10 ˆi cos q° [q = 0°]
But for the charge inside the cube, flux due to each face is

= 125 × 10–3 Nm2/c 1é q ù


ê ú which is not in option.
= 0.125 Nm2/c 6 ëÎ0 û
65. (c) Force of interaction
C
1 6p1p2 +q +q D
F= .
4p Î0 r 4 .
q
q
.
p1 p2
B
A
–q –q
r
16
P-254 Physics

Electrostatic Potential
and Capacitance
4. A charge Q is distributed over two concentric conducting
Electrostatic Potential and thin spherical shells radii r and R (R > r). If the surface
TOPIC 1 charge densities on the two shells are equal, the electric
Equipotential Surfaces potential at the common centre is : [Sep. 02, 2020 (II)]
1. Ten charges are placed on the circumference of a circle of
radius R with constant angular separation between r
successive charges. Alternate charges 1, 3, 5, 7, 9 have R
charge (+q) each, while 2, 4, 6, 8, 10 have charge (–q)
each. The potential V and the electric field E at the centre 1 (R + r ) 1 (2 R + r )
(a) Q (b) Q
of the circle are respectively : 4pe 0 2( R 2 + r 2 ) 4 pe0 ( R 2 + r 2 )
(Take V = 0 at infinity) [Sep. 05, 2020 (II)] 1 ( R + 2 r )Q 1 (R + r )
(c) (d) Q
10q 4pe 0 2( R + r )
2 2 4pe0 ( R 2 + r 2 )
(a) V = ;E=0
4pe 0 R ur
5. A point dipole = p – po $x kept at the origin. The potential
10 q
(b) V = 0; E = and electric field due to this dipole on the y-axis at a
4pe 0 R 2 distance d are, respectively : (Take V = 0 at infinity)
(c) V = 0; E = 0
[12 April 2019 I]
10 q 10 q ur ur ur
(d) V = ;E= p p -p
4 pe0 R 4 pe 0 R 2 (a) , (b) 0,
4pe 0 d 2 4pe 0 d 3 4pe 0 d 3
2 ur ur ur
2. Two isolated conducting spheres S1 and S2 of radius R p p -p
3 (c) 0, (d) ,
1 4pe 0 d 3 4pe 0 d 2 4pe 0 d 3
and R have 12 mC and –3 mC charges, respectively, and
3 6. A uniformly charged ring of radius 3a and total charge q
are at a large distance from each other. They are now is placed in xy-plane centred at origin. A point charge q is
connected by a conducting wire. A long time after this is moving towards the ring along the z-axis and has speed v
done the charges on S1 and S2 are respectively : at z = 4a. The minimum value of v such that it crosses the
origin is : [10 April 2019 I]
[Sep. 03, 2020 (I)] 1/2 1/2
(a) 4.5 mC on both (b) +4.5 mC and –4.5 mC 2 æ 4 q2 ö 2 æ 1 q2 ö
(a) ç ÷ (b) ç ÷
(c) 3 mC and 6 mC (d) 6 mC and 3 mC m è 15 4pe0a ø m è 5 4pe0a ø
3. Concentric metallic hollow spheres of radii R and 4R hold 1/2 1/2
charges Q1 and Q2 respectively. Given that surface charge 2 æ 2 q2 ö 2 æ 1 q2 ö
(c) ç ÷ (d) ç ÷
densities of the concentric spheres are equal, the potential m è 15 4pe0a ø m è 15 4pe0a ø
difference V(R) – V(4R) is : [Sep. 03, 2020 (II)] 7. A solid conducting sphere, having a charge Q, is
surrounded by an uncharged conducting hollow spherical
3Q1 3Q2 shell. Let the potential difference between the surface of
(a) (b)
16pe 0 R 4pe0 R the solid sphere and that of the outer surface of the hollow
shell be V. If the shell is now given a charge of – 4 Q, the
Q2 3Q1 new potential difference between the same two surfaces
(c) (d) is : [8 April 2019 I]
4pe0 R 4pe0 R
(a) – 2V (b) 2 V (c) 4 V (d) V
Electrostatic Potential and Capacitance P-255

r
8. The electric field in a region is given by E = ( Ax + B ) iˆ , s é a2 - b2 ù s é a 2 - b2 ù
where E is in NC–1 and x is in metres. The values of (a) Î ê a +c ú (b) Î ê b +cú
0 êë ûú 0 êë ûú
constants are A = 20 SI unit and B = 10 SI unit. If the
potential at x = 1 is V1 and that at x = –5 is V2, then
s é b2 - c2 ù s é b 2 - c2 ù
V1 – V2 is : [8 Jan. 2019 II] (c) Î ê b +a ú (d) Î ê c +a ú
(a) 320 V (b) – 48V (c) 180 V (d) – 520 V 0 ëê ûú 0 ëê ûú
9. The given graph shows variation (with distance r from centre ) 14. There is a uniform electrostatic field in a region. The
of : [11 Jan. 2019 I] potential at various points on a small sphere centred at P,
in the region, is found to vary between in the limits 589.0 V
to 589.8 V. What is the potential at a point on the sphere
rO
whose radius vector makes an angle of 60° with the direction
of the field ? [Online April 8, 2017]
(a) 589.5 V (b) 589.2 V (c) 589.4 V (d) 589.6 V
r 15. Within a spherical charge distribution of charge density
rO
r(r), N equipotential surfaces of potential V0, V0 + DV, V0
(a) Electric field of a uniformly charged sphere
+ 2DV, .........V0 + NDV (DV > 0), are drawn and have
(b) Potential of a uniformly charged spherical shell increasing radii r0, r1, r2,......... rN, respectively. If the
(c) Potential of a uniformly charged sphere difference in the radii of the surfaces is constant for all
(d) Electric field of a uniformly charged spherical shell values of V0 and DV then : [Online April 10, 2016]
10. A charge Q is distributed over three concentric spherical 1
shells of radii a, b, c (a < b < c) such that their surface (a) r(r) = constant (b) r(r) µ 2
charge densities are equal to one another. r
The total potential at a point at distance r from their 1
(c) r(r) µ (d) r(r) µ r
common centre, where r < a, would be: r
[10 Jan. 2019 I] 16. The potential (in volts) of a charge distribution is given by
Q ab + bc + ca Q (a 2 + b 2 + c 2 ) V(z) = 30 – 5z2 for |z| £ 1m
(a) 12pÎ (b) V(z) = 35 – 10 |z| for |z| ³ 1 m.
0 abc 4pÎ0 (a 3 + b 3 + c3 ) V(z) does not depend on x and y. If this potential is
Q Q (a + b + c) generated by a constant charge per unit volume r0 (in
(c) 4pÎ (a + b + c) (d) 4pÎ (a 2 + b 2 + c2 ) units of e0) which is spread over a certain region, then
0 0
11. Two electric dipoles, A, B with respective dipole choose the correct statement. [Online April 9, 2016]
r r (a) r0 = 20 e0 in the entire region
moments d A = – 4 qa iˆ and d B = – 2 qa iˆ are placed on
(b) r0 = 10 e0 for |z| £ 1 m and p0 = 0 elsewhere
the x-axis with a separation R, as shown in the figure (c) r0 = 20 e0 for |z| £ 1 m and p0 = 0 elsewhere
(d) r0 = 40 e0 in the entire region
17. A uniformly charged solid sphere of radius R has potential
V0 (measured with respect to ¥) on its surface. For this
The distance from A at which both of them produce the sphere the equipotential surfaces with potentials
same potential is: [10 Jan. 2019 I] 3V0 5V0 3V0 V0
R , , and have radius R1, R2, R3 and R4
2R 2 4 4 4
(a) 2 + 1 (b) respectively. Then [2015]
2 +1
(a) R1 = 0 and R2 < (R4 – R3)
R 2R (b) 2R = R4
(c) 2 -1
(d)
2 -1 (c) R1 = 0 and R2 > (R4 – R3)
12. Consider two charged metallic spheres S1 and S2 of radii (d) R1 ¹ 0 and (R2 – R1) > (R4 – R3)
r
R1 and R2, respectively. The electric fields E1 (on S1) and 18. An electric field E = (25i$ + 30j)NC
$ -1 exists in a region of
E2 (on S2) on their surfaces are such that E1/E2 = R1/R2.
Then the ratio V1(on S1)/V2(on S2) of the electrostatic space. If the potential at the origin is taken to be zero
potentials on each sphere is: [8 Jan. 2019 II] then the potential at x = 2 m, y = 2 m is :
(a) R1/R 2 (b) (R1/R2)2 [Online April 11, 2015]
(a) –110 J (b) –140 J (c) –120 J (d) –130 J
3 r
æR ö 19. Assume that an electric field E = 30x 2 ˆi exists in space.
(c) (R2/R1) (d) ç 1 ÷
è R2 ø Then the potential difference VA - VO , where VO is the
13. Three concentric metal shells A, B and C of respective potential at the origin and VA the potential at x = 2 m is:
radii a, b and c (a < b < c) have surface charge densities (a) 120 J/C (b) –120 J/C [2014]
+s, –s and +s respectively. The potential of shell B is: (c) –80 J/C (d) 80 J/C
[2018]
P-256 Physics

20. Consider a finite insulated, uncharged conductor placed 26. An electric charge 10–3 m C is placed at the origin (0, 0) of
near a finite positively charged conductor. The uncharged X – Y co-ordinate system. Two points A and B are situated
body must have a potential : [Online April 23, 2013] at ( 2, 2) and (2, 0) respectively. The potential
(a) less than the charged conductor and more than at
difference between the points A and B will be [2007]
infinity.
(a) 4.5 volts (b) 9 volts
(b) more than the charged conductor and less than at
(c) Zero (d) 2 volt
infinity.
27. Charges are placed on the vertices of a square as shown.
(c) more than the charged conductor and more than at r
infinity. Let E be the electric field and V the potential at the
(d) less than the charged conductor and less than at centre. If the charges on A and B are interchanged with
infinity. those on D and C respectively, then [2007]
21. Two small equal point charges of magnitude q are q q
suspended from a common point on the ceiling by
A B
insulating mass less strings of equal lengths. They come
to equilibrium with each string making angle q from the
vertical. If the mass of each charge is m, then the
electrostatic potential at the centre of line joining them will
D C
æ 1 ö -q -q
be ç = k÷. [Online April 22, 2013]
è 4p Î0 ø ur
(a) E
ur changes, V remains unchanged
(a) 2 k mg tan q (b) k mg tan q (b) E remains
ur unchanged, V changes
(c) both E and V change
(c) 4 k mg / tan q (d) k mg / tan q ur
(d) E and V remain unchanged
22. A point charge of magnitude + 1 mC is fixed at (0, 0, 0). An 28. The potential at a point x (measured in m m) due to some
isolated uncharged spherical conductor, is fixed with its charges situated on the x-axis is given by V(x) = 20/(x2 – 4)
center at (4, 0, 0). The potential and the induced electric volt. The electric field E at x = 4 m m is given by [2007]
field at the centre of the sphere is :[Online April 22, 2013] (a) (10/9) volt/ m m and in the +ve x direction
(a) 1.8 × 105 V and – 5.625 × 106 V/m
(b) (5/3) volt/ m m and in the –ve x direction
(b) 0 V and 0 V/m
(c) (5/3) volt/ m m and in the +ve x direction
(c) 2.25 × 105 V and – 5.625 × 106 V/m
(d) 2.25 × 105 V and 0 V/m (d) (10/9) volt/ m m and in the –ve x direction
23. A charge of total amount Q is distributed over two 29. Two thin wire rings each having a radius R are placed at a
concentric hollow spheres of radii r and R (R > r) such that distance d apart with their axes coinciding. The charges
the surface charge densities on the two spheres are equal. on the two rings are +q and -q. The potential difference
The electric potential at the common centre is between the centres of the two rings is [2005]
[Online May 19, 2012] é1 ù
q 1 qR
ê - ú (b)
1 ( R - r) Q 1 ( R + r) Q (a)
2 p Î0 êë R R 2 + d 2 úû 4p Î0 d 2
( )
(a) (b)
4pe 0 R2 + r 2
(
4pe 0 2 R 2 + r 2
) q é1 1 ù
(c) ê - ú (d) zero
1 ( R + r) Q 1 ( R - r) Q 4 p Î0 ê R
ë R + d2
2 úû
(c) 4pe
(
0 R2 + r 2 ) (d)
(
4pe 0 2 R 2 + r 2
) 30. A thin spherical conducting shell of radius R has a charge
q. Another charge Q is placed at the centre of the shell.
24. The electric potential V(x) in a region around the origin is
given by V(x) = 4x2 volts. The electric charge enclosed in R
The electrostatic potential at a point P , a distance
a cube of 1 m side with its centre at the origin is (in coulomb) 2
[Online May 7, 2012] from the centre of the shell is [2003]
(a) 8e0 (b) – 4e0 (c) 0 (d) – 8e0
2Q 2Q 2q
25. The electrostatic potential inside a charged spherical ball is (a) 4pe R (b) 4pe R - 4pe R
given by f = ar2 + b where r is the distance from the centre o o o
and a, b are constants. Then the charge density inside the
2Q q (q + Q)2
ball is: [2011] (c) 4pe R + 4pe R (d) 4pe R
(a) –6ae0r (b) –24pae0 o o o
(c) –6ae0 (d) –24pae0r
Electrostatic Potential and Capacitance P-257

33. Hydrogen ion and singly ionized helium atom are


Electric Potential Energy and accelerated, from rest, through the same potential
TOPIC 2 Work Done in Carrying a difference. The ratio of final speeds of hydrogen and
Charge helium ions is close to : [Sep. 03, 2020 (II)]
31. A solid sphere of radius R carries a charge Q + q distributed (a) 1 : 2 (b) 10 : 7
uniformaly over its volume. A very small point like piece of (c) 2 : 1 (d) 5 : 7
it of mass m gets detached from the bottom of the sphere 34. In free space, a particle A of charge 1 mC is held fixed at a
and falls down vertically under gravity. This piece carries point P. Another particle B of the same charge and mass 4
charge q. If it acquires a speed v when it has fallen through mg is kept at a distance of 1 mm from P. If B is released,
a vertical height y (see figure), then : (assume the remaining then its velocity at a distance of 9 mm from P is :
portion to be spherical). é 1 ù
[Sep. 05, 2020 (I)] êTake = 9 ´ 109 Nm 2C -2 ú [10 April 2019 II]
ë 4pe 0 û
(a) 1.0m/s (b) 3.0×104 m/s
3
(c) 2.0×10 m/s (d) 1.5×102 m/s
Q R
35. A system of three charges are placed as shown in the
figure:

q
y
v
If D >> d, the potential energy of the system is best given
é qQ ù by [9 April 2019 I]
(a) v 2 = y ê + g ú
2
êë 4pe 0 R ym úû 1 é - q 2 - qQd ù 1 é - q 2 2qQd ù
(a) ê 2 ú (b) ê + ú
é qQ ù 4p Î0 ë d 2 D û 4p Î0 ë d D2 û
(b) v 2 = y ê + gú
ë 4pe 0 R( R + y )m û 1 é q 2 qQd ù 1 é q 2 qQd ù
é ù (c) ê+ + 2 ú (d) ê- - 2 ú
Qq R 4p Î0 ë d D û 4p Î0 ë d D û
(c) v 2 = 2 y ê 3
+ gú
êë 4pe0 ( R + y ) m úû 36. A positive point charge is released from rest at a distance
é qQ ù r0 from a positive line charge with uniform density. The
2
(d) v = 2 y ê + gú speed (v) of the point charge, as a function of
ë 4 pe 0 R ( R + y ) m û instantaneous distance r from line charge, is proportional
32. A two point charges 4q and –q are fixed on the x-axis at to : [8 April 2019 II]
d d
x=- and x = , respectively. If a third point charge ge
2 2
‘q’ is taken from the origin to x = d along the semicircle as
shown in the figure, the energy of the charge will :
[Sep. 04, 2020 (I)]

ærö
4q –q +r /r ln ç ÷
(a) v µ e 0 (b) v µ
è r0 ø
3q 2 ærö ærö
(a) increase by (c) v µ ln ç ÷ (d) v µ ç ÷
4pe0 d è 0ø
r è r0 ø
2q 2 37. There is a uniform spherically symmetric surface charge
(b) increase by
3pe0 d density at a distance Ro from the origin. The charge
distribution is initially at rest and starts expanding because
q2 of mutual repulsion. The figure that represents best the
(c) decrease by
4pe0 d speed V (R(t)) of the distribution as a function of its
4q 2 instantaneous radius R(t) is: [12 Jan. 2019 I]
(d) decrease by
3pe0 d
P-258 Physics

V(R(t))
V(R(t)) 41. An insulating solid sphere of radius R has a uniformly
positive charge density r. As a result of this uniform charge
distribution there is a finite value of electric potential at
the centre of the sphere, at the surface of the sphere and
also at a point outside the sphere. The electric potential
(a) (b) at infinite is zero. [2012]
R (t)
Statement -1 When a charge q is taken from the centre
Ro Ro R (t) to the surface of the sphere its potential energy changes
V(R(t))
V(R(t)) qr
by .
3e0
Vo
Statement -2 The electric field at a distance r (r <R) from
rr
the centre of the sphere is .
(c) (d) 3e0
(a) Statement 1 is true, Statement 2 is true; Statement 2 is
Ro R (t) Ro R (t) not the correct explanation of statement 1.
38. Three charges Q, + q and + q are placed at the vertices of (b) Statement 1 is true Statement 2 is false.
a right-angle isosceles triangle as shown below. The net (c) Statement 1 is false Statement 2 is true.
electrostatic energy of the configuration is zero, if the value (d) Statement 1 is true, Statement 2 is true, Statement 2 is
of Q is : [11 Jan. 2019 I] the correct explanation of Statement 1
Q 42. Two positive charges of magnitude ‘q’ are placed, at the
ends of a side (side 1) of a square of side ‘2a’. Two negative
charges of the same magnitude are kept at the other corners.
Starting from rest, if a charge Q moves from the middle of
side 1 to the centre of square, its kinetic energy at the
centre of square is [2011 RS]
+q +q 1 2qQ æ 1 ö
(a) zero (b) 1+
- 2q 4pe 0 a çè ÷

(a) + q (b)
2 +1 1 2qQ æ 2 ö 1 2qQ æ 1 ö
(c) 1- (d) 1-
-q 4pe 0 a çè ÷
5ø 4pe 0 a çè ÷

(c) (d) –2q
1+ 2 43. Two points P and Q are maintained at the potentials of 10
39. Four equal point charges Q each are placed in the xy V and – 4 V, respectively. The work done in moving 100
plane at (0, 2), (4, 2), (4, – 2) and (0, – 2). The work electrons from P to Q is: [2009]
required to put a fifth charge Q at the origin of the (a) 9.60 × 10–17J (b) –2.24 × 10–16 J
coordinate system will be: [10 Jan. 2019 II] (c) 2.24 × 10–16 J (d) –9.60× 10–17 J
44. Two insulating plates are both uniformly charged in such
Q2 æ 1 ö Q2 æ 1 ö a way that the potential difference between them is V2 –
(a)
4 pe 0 ç1 + ÷ (b)
4 pe 0 ç1 + ÷
è 3ø è 5ø V1 = 20 V. (i.e., plate 2 is at a higher potential). The plates
are separated by d = 0.1 m and can be treated as infinitely
Q2 Q2 large. An electron is released from rest on the inner surface
(c) (d)
2 2 pe0 4pe 0 of plate 1. What is its speed when it hits plate 2? (e = 1.6 ×
10–19 C, me = 9.11 × 10–31 kg) [2006]
40. Statement 1 : No work is required to be done to move a
test charge between any two points on an equipotential
surface. Y
Statement 2 : Electric lines of force at the equipotential
surfaces are mutually perpendicular to each other. 0.1 m
[Online April 25, 2013]
(a) Statement 1 is true, Statement 2 is true, Statement 2 is X
the correct explanation of Statement 1.
(b) Statement 1 is true, Statement 2 is true, Statement 2 is
not the correct explanation of Statement 1. 1 2
(c) Statement 1 is true, Statement 2 is false. (a) 2.65 × 106 m/s (b) 7.02 × 1012 m/s
(d) Statement 1 is false, Statement 2 is true. (c) 1.87 × 106 m/s (d) 32 × 10–19 m/s
Electrostatic Potential and Capacitance P-259

45. A charged particle ‘q’ is shot towards another charged (a) 450 mC (b) 590 mC
particle ‘Q’ which is fixed, with a speed ‘v’. It approaches (c) 160 mC (d) 650 mC
‘Q’ upto a closest distance r and then returns. If q were
51. A 5 mF capacitor is charged fully by a 220 V supply. It is
given a speed of ‘2v’ the closest distances of approach
would be [2004] then disconnected from the supply and is connected in
(a) r/2 (b) 2 r (c) r (d) r/4 series to another uncharged 2.5 mF capacitor. If the energy
46. On moving a charge of 20 coulomb by 2 cm, 2 J of work X
is done, then the potential difference between the points change during the charge redistribution is J then value
100
is [2002]
(a) 0.1 V (b) 8 V (c) 2 V (d) 0.5 V of X to the nearest integer is ________.
[NA Sep. 02, 2020 (I)]
Capacitors, Grouping of 52. A 10 mF capacitor is fully charged to a potential difference
TOPIC 3 Capacitors and Energy Stored of 50 V. After removing the source voltage it is connected
in a Capacitor to an uncharged capacitor in parallel. Now the potential
47. Two capacitors of capacitances C and 2C are charged to difference across them becomes 20 V. The capacitance of
potential differences V and 2V, respectively. These are then the second capacitor is : [Sep. 02, 2020 (II)]
connected in parallel in such a manner that the positive (a) 15 mF (b) 30 mF
terminal of one is connected to the negative terminal of the (c) 20 mF (d) 10 mF
other. The final energy of this configuration is : 53. Effective capacitance of parallel combination of two
[Sep. 05, 2020 (I)] capacitors C1 and C2 is 10 mF. When these capacitors are
25 2 3 2 individually connected to a voltage source of 1 V, the
(a) CV (b) CV energy stored in the capacitor C2 is 4 times that of C1. If
6 2
9 2 these capacitors are connected in series, their effective
(c) zero (d) CV capacitance will be: [8 Jan. 2020 I]
2
48. In the circuit shown, charge on the 5 mF capacitor is : (a) 4.2 mF (b) 3.2 mF (c) 1.6 mF (d) 8.4 mF
54. A capacitor is made of two square plates each of side ‘a’
[Sep. 05, 2020 (II)]
making a very small angle a between them, as shown in
2 mF 4 mF
figure. The capacitance will be close to: [8 Jan. 2020 II]
V1
a
5 mF
d

O a V2
6V 6V
(a) 18.00 mC (b) 10.90 mC
Î0 a 2 æ aa ö Î0 a 2 æ aa ö
(c) 16.36 mC (d) 5.45 mC (a) çè1 - ÷ø (b) çè1 - ÷ø
d 2d d 4d
49. A capacitor C is fully charged with voltage V0 . After
disconnecting the voltage source, it is connected in parallel Î0 a 2 æ aa ö Î0 a 2 æ 3aa ö
(c) çè1 + ÷ø (d) ç1 - ÷
C d d d è 2d ø
with another uncharged capacitor of capacitance . The 55. A parallel plate capacitor has plates of area A separated
2
energy loss in the process after the charge is distributed by distance ‘d’ between them. It is filled with a dielectric
between the two capacitors is : [Sep. 04, 2020 (II)] which has a dielectric constant that varies as k(x) = K(1 +
1 1 ax) where ‘x’ is the distance measured from one of the
(a) CV02 (b) CV02 plates. If (ad) << l, the total capacitance of the system is
2 3
best given by the expression: [7 Jan. 2020 I]
1 1
(c) CV02 (d) CV02 AK Î0 æ ad ö
4 6 (a) ç1 + ÷
50. In the circuit shown in the figure, the total charge is 750 mC d è 2 ø
A Î0 K æ æ ad ö ö
2
and the voltage across capacitor C2 is 20 V. Then the
(b) ç1 + ç ÷ ÷
charge on capacitor C2 is : [Sep. 03, 2020 (I)] d ç è 2 ø ÷
è ø
C2
C1 = 15 mF æ 2 2ö
A Î0 K ç a d ÷
(c) 1+
d ç 2 ÷
C3 = 8 m F è ø
AK Î0
(d) (1 + ad )
+ – d
V
P-260 Physics

56. A 60 pF capacitor is fully charged by a 20 V supply. It is 60. Figure shows charge (q) versus voltage (V) graph for
then disconnected from the supply and is connected series and parallel combination of two given capacitors.
to another uncharged 60 pF capacitor in parallel. The The capacitances are : [10 April 2019 I]
electrostatic energy that is lost in this process by the
time the charge is redistributed between them is (in nJ)
[NA 7 Jan. 2020 II]
57. The parallel combination of two air filled parallel plate
capacitors of capacitance C and nC is connected to a
battery of voltage, V. When the capacitors are fully
charged, the battery is removed and after that a dielectric
material of dielectric constant K is placed between the
two plates of the first capacitor. The new potential
difference of the combined system is: [9 April 2020 II] (a) 40 mF and 10 mF (b) 60 mF and 40 mF
nV (c) 50 mF and 30 mF (d) 20 mF and 30 mF
(a) (b) V
K+n 61. A capacitor with capacitance 5mF is charged to 5 mC. If
the plates are pulled apart to reduce the capacitance to 2
V (n + 1) V
(c) (d) ¼F, how much work is done? [9 April 2019 I]
K+n (K + n)
(a) 6.25 × 10–6 J (b) 3.75 × 10–6 J
58. Two identical parallel plate capacitors, of capacitance C
(c) 2.16 × 10–6 J (d) 2.55 × 10–6 J
each, have plates of area A, separated by a distance d.
The space between the plates of the two capacitors, is 62. Voltage rating of a parallel plate capacitor is 500 V. Its
filled with three dielectrics, of equal thickness and dielectric dielectric can withstand a maximum electric field of 106 V/
constants K1, K2 and K3. The first capacitors is filled as m. The plate area is 10–4 m2. What is the dielectric constant
shown in Fig. I, and the second one is filled as shown in if the capacitance is 15 pF ? [8 April 2019 I]
Fig. II. –12 2
(given “0 = 8.86 × 10 C /Nm ) 2

If these two modified capacitors are charged by the same (a) 3.8 (b) 8.5 (c) 4.5 (d) 6.2
potential V, the ratio of the energy stored in the two, would 63. A parallel plate capacitor has 1mF capacitance. One of its
be (E1 refers to capacitors (I) and E2 to capacitors (II) : two plates is given + 2mC charge and the other plate,
[12 April 2019 I] +4mC charge. The potential difference developed across
the capacitor is : [8 April 2019 II]
(a) 3 V (b) 1 V (c) 5 V (d) 2 V
64. In the figure shown, after the switch ‘S’ is turned from
position ‘A’ to position ‘B’, the energy dissipated in the
circuit in terms of capacitance ‘C’ and total charge ‘Q’ is:
E1 K1 K 2 K 3
(a) E = ( K + K + K )( K K + K K + K K [12 Jan. 2019 I]
2 1 2 3 2 3 3 1 1 2 A B

E1 ( K1 + K 2 + K3 )( K 2 K3 + K3 K1 + K1 K 2
(b) E = K1 K 2 K 3
S
2

e
E1 9 K1 K 2 K3
(c) E = ( K + K + K )( K K + K K + K K )
C 3C
2 1 2 3 2 3 3 1 1 2

E1 ( K1 + K 2 + K3 )(K 2 K 3 + K 3 K1 + K1 K 2
(d) E = 9 K1 K 2 K3
2
1 Q2 3 Q2 5 Q2 3 Q2
59. In the given circuit, the charge on 4 mF capacitor will be : (a) (b) (c) (d)
8 C 8 C 8 C 4 C
[12 April 2019 II] 65. A parallel plate capacitor with plates of area 1 m2 each, are
at a separation of 0.1 m. If the electric field between the
plates is 100 N/C, the magnitude of charge on each plate is :
C2
(Take Î0 = 8.85 × 10–12 ) [12 Jan. 2019 II]
N – M2
(a) 7.85 × 10–10 C (b) 6.85 × 10–10 C
(a) 5.4 mC (b) 9.6 mC (c) 13.4 mC (d) 24 mC (c) 8.85 × 10–10 C (d) 9.85 × 10–10 C
Electrostatic Potential and Capacitance P-261

66. In the circuit shown, find C if the effective capacitance of 69. A parallel plate capacitor having capacitance 12 pF is
the whole circuit is to be 0.5 µF. All values in the circuit are charged by a battery to a potential difference of 10 V
in µF. [12 Jan. 2019 II] between its plates. The charging battery is now
C 2
A disconnected and a porcelain slab of dielectric constant
2 2 1 6.5 is slipped between the plates. The work done by the
capacitor on the slab is: [10 Jan. 2019 II]
(a) 692 pJ (b) 508 pJ
2
(c) 560 pJ (d) 600 pJ
70. A parallel plate capacitor is of area 6 cm2 and a
2 2 separation 3 mm. The gap is filled with three dielectric
materials of equal thickness (see figure) with dielectric
constants K1 = 10, K2 = 12 and K3 = 1(4) The dielectric
B constant of a material which when fully inserted in
7 6 7 above capacitor, gives same capacitance would be:
(a) µF (b)
µF (c) 4 µF (d) µF [10 Jan. 2019 I]
11 5 10
67. In the figure shown below, the charge on the left plate of
the 10 mF capacitor is –30mC. The charge on the right plate
of the 6mF capacitor is : [11 Jan. 2019 I]
(a) 4 (b) 14 (c) 12 (d) 36
6 mF 71. A parallel plate capacitor is made of two square plates of
10 m F 2 mF
side ‘a’, separated by a distance d (d<<a). The lower
4 mF triangular portion is filled with a dielectric of dielectric
(a) –12 m C (b) +12 m C constant K, as shown in the figure. Capacitance of this
(c) –18 m C (d) +18 m C capacitor is: [9 Jan. 2019 I]
68. Seven capacitors, each of capacitance 2 µF, are to be
connected in a configuration to obtain an effective
d
æ 6ö K
capacitance of ç ÷ µF. Which of the combinations,
è 13 ø
shown in figures below, will achieve the desired value? a
[11 Jan. 2019 II] K Î0 a 2 K Î0 a 2
(a) (b) In K
2d (K + 1) d (K – 1)
K Î0 a 2 1 K Î0 a 2
(a) (c) In K (d)
d 2 d
72. A parallel plate capacitor with square plates is filled with
four dielectrics of dielectric constants K1, K2, K3, K4
arranged as shown in the figure. The effective dielectric
constant K will be: [9 Jan. 2019 II]
(b)
K1 K2 L/2

K3 K4 L/2

d/2 d/2

(c) ( K1 + K3 ) ( K 2 + K 4 )
(a) K =
K1 + K 2 + K 3 + K 4

( K1 + K 2 ) ( K3 + K 4 )
(b) K =
(d) 2(K1 + K 2 + K 3 + K 4 )
P-262 Physics

( K1 + K 2 ) ( K 3 + K 4 ) 78. A capacitance of 2m F is required in an electrical circuit


(c) K = across a potential difference of 1.0 kV. A large number of
K1 + K 2 + K 3 + K 4
1m F capacitors are available which can withstand a
( K1 + K 4 ) ( K 2 + K3 ) potential difference of not more than 300 V. The minimum
(d) K = number of capacitors required to achieve this is [2017]
2(K1 + K 2 + K 3 + K 4 )
(a) 24 (b) 32 (c) 2 (d) 16
73. A parallel plate capacitor of capacitance 90 pF is connected 79. A combination of parallel plate capacitors is maintained at
to a battery of emf 20V. If a dielectric material of dielectric a certain potential difference.
5
constant k = is inserted between the plates, the
3
magnitude of the induced charge will be: [2018] C1 C2 C3
C
(a) 1.2 n C (b) 0.3 n C (c) 2.4 n C (d) 0.9 n C
74. In the following circuit, the switch S is closed at t = 0. The A D E B
charge on the capacitor C1 as a function of time will be
æ CC ö
given by ç Ceq = 1 2 ÷ . [Online April 16, 2018] When a 3 mm thick slab is introduced between all the
è C1 + C2 ø
plates, in order to maintain the same potential difference,
(a) CeqE[1 – exp(–t/RCeq)] C1 C2 the distance between the plates is increased by 2.4 mm.
Find the dielectric constant of the slab.
(b) C1E[1 – exp(–tR/C1)]
S [Online April 9, 2017]
(c) C2E[1 – exp(–t/RC2)] (a) 3 (b) 4 (c) 5 (d) 6
E R
(d) CeqE exp(–t/RCeq) 80. The energy stored in the electric field produced by a metal
75. The equivalent capacitance between A and B in the circuit sphere is 4.5 J. If the sphere contains 4 mC charge, its
given below is:
radius will be : [Take : 1 = 9 ´ 109 N - m 2 / C 2 ]
6 µF 2 µF 4 pe0
A [Online April 8, 2017]
5 µF 5 µF 4 µF (a) 20mm (b) 32mm (c) 28mm (d) 16mm
81. A combination of capacitors is set up as shown in the
2 µF
figure. The magnitude of the electric field, due to a point
B charge Q (having a charge equal to the sum of the charges
[Online April 15, 2018] on the 4 mF and 9 mF capacitors), at a point distance 30 m
from it, would equal : [2016]
(a) 4.9 µF (b) 3.6 µF (c) 5.4 µF (d) 2.4 µF
3m F
76. A parallel plate capacitor with area 200cm2 and separation
4m F
between the plates 1.5cm, is connected across a battery of
9m F
emf V. If the force of attraction between the plates is 25 × 10–
6N, the value of V is approximately: [Online April 15, 2018]
2m F
æ -12 C
2 ö
+ –
ç e 0 = 8.85 ´ 10 ÷
è N.m 2 ø
8V
(a) 150V (b) 100V (c) 250V (d) 300V (a) 420 N/C (b) 480 N/C
(c) 240 N/C (d) 360 N/C
77. A capacitor C1 is charged up to a voltage V = 60V by
82. Figure shows a network of capacitors where the numbers
connecting it to battery B through switch (1), Now C1 is
indicates capacitances in micro Farad. The value of
disconnected from battery and connected to a circuit
capacitance C if the equivalent capacitance between point
consisting of two uncharged capacitors C2 = 3.0mF and C3 =
A and B is to be 1 mF is : [Online April 10, 2016]
6.0mF through a switch (2) as shown in the figure. The sum
C 1
of final charges on C2 and C3 is: [Online April 15, 2018]
A
(2)
(1) 8 6 4
B C2
60 V C1 C3
2 2 12
(a) 36mC (b) 20mC (c) 54mC (d) 40mC
B
Electrostatic Potential and Capacitance P-263

87. The gap between the plates of a parallel plate capacitor of


32 31 33 34
(a) mF (b) mF (c) mF (d) mF area A and distance between plates d, is filled with a
23 23 23 23
83. Three capacitors each of 4 mF are to be connected in such dielectric whose permittivity varies linearly from Î1 at one
a way that the effective capacitance is 6mF. This can be plate to Î2 at the other. The capacitance of capacitor is:
done by connecting them : [Online April 9, 2016] [Online April 19, 2014]
(a) all in series
(a) Î0 (Î1 + Î2 ) A / d
(b) all in parallel
(c) two in parallel and one in series (b) Î0 ( Î2 + Î1 ) A / 2d
(d) two in series and one in parallel (c) Î0 A / éëd ln ( Î2 / Î1 ) ùû
84. In the given circuit, charge Q2 on the 2µF capacitor (d) Î0 ( Î2 - Î1 ) A / ëé d ln ( Î2 / Î1 ) ûù
changes as C is varied from 1µF to 3µF. Q2 as a function of 88. The space between the plates of a parallel plate capacitor
'C' is given properly by: (figures are drawn schematically is filled with a ‘dielectric’ whose ‘dielectric constant’ varies
and are not to scale) [2015] with distance as per the relation:
K(x) = Ko + lx (l = a constant)
1µF The capacitance C, of the capacitor, would be related to its
C vacuum capacitance Co for the relation :
2µF
[Online April 12, 2014]
ld l
(a) C = Co (b) C = C
E ln (1 + K o ld ) d.ln (1 + K o ld ) o
Charge Charge
ld l
(c) C = C (d) C = Co
Q2 Q2 ln (1 + ld / K o ) o d.ln (1 + Ko / ld )
(a) (b) 89. A parallel plate capacitor is made of two plates of length l,
1µF 3µF
C
1µF 3µF
C width w and separated by distance d. A dielectric slab
(dielectric constant K) that fits exactly between the plates
Charge Charge is held near the edge of the plates. It is pulled into the
Q2 ¶U
Q2
capacitor by a force F = - where U is the energy of
(c) (d) ¶x
C C
the capacitor when dielectric is inside the capacitor up to
1µF 3µF 1µF 3µF distance x (See figure). If the charge on the capacitor is Q
85. In figure a system of four capacitors connected across then the force on the dielectric when it is near the edge is:
a 10 V battery is shown. Charge that will flow from [Online April 11, 2014]
switch S when it is closed is : [Online April 11, 2015]
2mF a 3mF
x
l
S

d
3mF b 2mF
Q d2 Q2 w
(a) K (b) ( K - 1)
2wl 2 e o 2dl 2 e0
10 V
Q 2d Q2w
(a) 5 µC from b to a (b) 20 µC from a to b (c) ( K - 1) (d) K
(c) zero (d) 5 µC from a to b 2wl 2 e o 2dl 2 e o
86. A parallel plate capacitor is made of two circular plates 90. Three capacitors, each of 3 mF, are provided. These cannot
separated by a distance 5 mm and with a dielectric of be combined to provide the resultant capacitance of:
dialectric constant 2.2 between them. When the electric [Online April 9, 2014]
field in the dielectric is 3 ´ 104 V m the charge density of (a) 1 mF (b) 2 mF (c) 4.5 mF (d) 6 mF
91. A parallel plate capacitor having a separation between the
the positive plate will be close to: [2014]
plates d, plate area A and material with dielectric constant
(a) 6 ´ 10-7 C m 2 (b) 3 ´10-7 C m2 K has capacitance C0. Now one-third of the material is
(c) 3 ´ 104 C m 2 (d) 6 ´104 C m 2 replaced by another material with dielectric constant 2K,
so that effectively there are two capacitors one with area
P-264 Physics

97. A series combination of n1 capacitors, each of capacity


1 2
A, dielectric constant 2K and another with area A C1 is charged by source of potential difference 4 V. When
3 3 another parallel combination of n2 capacitors each of
and dielectric constant K. If the capacitance of this new capacity C2 is charged by a source of potential difference
C V, it has the same total energy stored in it as the first
capacitor is C then is [Online April 25, 2013] combination has. The value of C2 in terms of C1 is then
C0
[Online May 12, 2012]
4 2 1 n2
(a) 1 (c) (b) (d) 2 C1
3 3 3 (a) 16 n C1 (b) n n
1 1 2
92. To establish an instantaneous current of 2 A through a 1
mF capacitor ; the potential difference across the capacitor n2 16 C1
(c) 2 n C1 (d) n n
plates should be changed at the rate of : 1 1 2
[Online April 22, 2013] 98. Two circuits (a) and (b) have charged capacitors of
(a) 2 × 104 V/s (b) 4 × 106 V/s capacitance C, 2C and 3C with open switches. Charges on
(c) 2 × 106 V/s (d) 4 × 104 V/s each of the capacitor are as shown in the figures. On closing
ur
93. A uniform electric field E exists between the plates of a the switches [Online May 7, 2012]
charged condenser. A charged particle enters the space S S
ur
between the plates and perpendicular to E . The path of
the particle between the plates is a : Q
2Q 2Q Q
[Online April 9, 2013] 2C
3C C 2C
(a) straight line (b) hyperbola
(c) parabola (d) circle
94. The figure shows an experimental plot discharging of a L R L R
capacitor in an RC circuit. The time constant t of this circuit Circuit (a) Circuit (b)
lies between : [2012]
(a) No charge flows in (a) but charge flows from R to L in (b)
(b) Charges flow from L to R in both (a) and (b)
Potential difference

25

(c) Charges flow from R to L in (a) and from L to R in (b)


V in volts

20
(d) No charge flows in (a) but charge flows from L to R in (b)
15
99. Let C be the capacitance of a capacitor discharging through
10 a resistor R. Suppose t1 is the time taken for the energy
5
stored in the capacitor to reduce to half its initial value and
t2 is the time taken for the charge to reduce to one-fourth
0 200 250 300
50 100 150 its initial value. Then the ratio t1/ t2 will be [2010]
Time in seconds
(a) 150 sec and 200 sec (b) 0 sec and 50 sec 1 1
(c) 50 sec and 100 sec (d) 100 sec and 150 sec (a) 1 (b) (c) (d) 2
2 4
95. The capacitor of an oscillatory circuit is enclosed in a
100. A parallel plate capacitor with air between the plates has
container. When the container is evacuated, the resonance
capacitance of 9 pF. The separation between its plates is
frequency of the circuit is 10 kHz. When the container is
‘d’. The space between the plates is now filled with two
filled with a gas, the resonance frequency changes by 50
dielectrics. One of the dielectrics has dielectric constant
Hz. The dielectric constant of the gas is
[Online May 26, 2012] d
k1 = 3 and thickness while the other one has dielectric
(a) 1.001 (b) 2.001 (c) 1.01 (d) 3.01 3
96. Statement 1: It is not possible to make a sphere of 2d
capacity 1 farad using a conducting material. constant k2 = 6 and thickness . Capacitance of the
3
Statement 2: It is possible for earth as its radius is capacitor is now [2008]
6.4 × 106 m. [Online May 26, 2012] (a) 1.8 pF (b) 45 pF (c) 40.5 pF (d) 20.25 pF
(a) Statement 1 is true, Statement 2 is true, Statement 2 is 101. A parallel plate condenser with a dielectric of dielectric
the correct explanation of Statement 1.
constant K between the plates has a capacity C and is
(b) Statement 1 is false, Statement 2 is true. charged to a potential V volt. The dielectric slab is slowly
(c) Statement 1 is true, Statement 2 is true, Statement 2 is removed from between the plates and then reinserted. The
not the correct explanation of Statement 1. net work done by the system in this process is [2007]
(d) Statement 1 is true, Statement 2 is false.
Electrostatic Potential and Capacitance P-265

1 104. A sheet of aluminium foil of negligible thickness is


(a) zero (b) ( K - 1) CV 2 introduced between the plates of a capacitor. The
2
capacitance of the capacitor [2003]
2
(c) CV ( K - 1) (d) ( K - 1) CV 2 (a) decreases (b) remains unchanged
K (c) becomes infinite (d) increases
102. A parallel plate capacitor is made by stacking n equally -18
spaced plates connected alternatively. If the capacitance 105. The work done in placing a charge of 8 ´ 10 coulomb
between any two adjacent plates is ‘C’ then the resultant on a condenser of capacity 100 micro-farad is [2003]
capacitance is [2005] (a) 16 ´ 10 -32 joule (b) 3.1´10 -26 joule
(a) (n + 1) C (b) (n – 1) C
(c) nC (d) C (c) 4 ´10-10 joule (d) 32 ´ 10-32 joule
103. A fully charged capacitor has a capacitance ‘C’. It 106. If there are n capacitors in parallel connected to V volt
is discharged through a small coil of resistance wire source, then the energy stored is equal to [2002]
embedded in a thermally insulated block of specific heat
1 1
capacity ‘s’ and mass ‘m’. If the temperature of the block is (a) CV (b) nCV2 (c) CV2 (d) CV2
2 2n
raised by ‘DT’, the potential difference ‘V’ across the
capacitance is [2005] 107. Capacitance (in F) of a spherical conductor with radius 1 m
is [2002]
mC DT 2mC DT
(a) (b)
s s (a) 1.1´ 10 -10 (b) 10 -6

2ms DT ms DT (c) 9 ´ 10 -9 (d) 10 -3


(c) (d)
C C
P-266 Physics

4. (d) Let s be the surface charge density of the shells.


KQnet
1. (c) Potential at the centre, VC = s
R
QQnet = 0 s
\ VC = 0
Let E be electric field produced by each charge at the centre, C
R r
then resultant electric field will be EC = 0, since equal electric
field vectors are acting at equal angle so their resultant is
equal to zero.
2E
Charge on the inner shell, Q1 = s 4pr 2
Charge on the outer shell, Q2 = s 4pR 2
2E 72° 72° 2E \ Total charge, Q = s 4p (r 2 + R 2 )
72° 72° Q
Þs=
72° 4p ( r + R 2 )
2

Potential at the common centre,


2E 2E
KQ1 KQ2 æ 1 ö
2. (d) Total charge Q1 + Q2 = Q '1 + Q '2 VC = + ç where K = ÷
r R è 4pe0 ø
= 12mC - 3mC = 9mC
Two isolated conducting sphres S1 and S2 are now K s 4pr 2 K s4pR 2
= +
connected by a conducting wire. r R
KQ '1 KQ '2 = K s4p ( r + R )
\ V1 = V2 = = = 12 – 3 = 9 µC
2 R KQ4p(r + R)
R =
3 3 4p(r 2 + R 2 )
Q '1 = 2Q '2 Þ 2Q '2 + Q '2 = 9mC 1 ( r + R )Q
=
\ Q '1 = 6 mC and Q '2 = 3 mC 4pe 0 (r 2 + R 2 )
3. (a) We have given two metallic hollow spheres of radii R 5. (b) The electric potential at the bisector is zero and
and 4R having charges Q1 and Q2 respectively. electric field is antiparallel to the dipole moment.
Potential on the surface of inner sphere (at A) æ ®ö
1 ® ç-P ÷
kQ1 kQ2 \ V = 0 and E = ç d3 ÷
VA = + 4pe0 ç ÷
R 4R è ø
Potential on the surface of outer sphere (at B) 6. (c) Potential at any point of the charged ring
kQ1 kQ2 æ 1 ö Kq
VB = + ç Here, k = ÷ Vp =
4R 4R è 4pe0 ø R 2 + Z2

Q1 Q2

R A
B
4R

Potential difference,
R = 3a
3 kQ 3 Q
DV = VA - VB = × 1 = × 1 Z = 4a
4 R 16p Î0 R
Electrostatic Potential and Capacitance P-267

l = R 2 + Z2 = 5a
The minimum velocity (v0) should just sufficient to reach
a
the point charge at the center, therefore b r
1 10. (d) c P
mv 20 = q [VC - VP ]
2
é Kq Kq ù
= qê -
ë 3a 5a úû kQa kQ b kQ c
Potential at point P, V = + +
4Kq 2 4 1 q2 a b c
v02 = = Since surface charge densities are equal to one another
15ma 15 4pe 0 ma
i.e., sa = sb = sc
1
2 æ 2q 2 ö2
\ Qa : Qb : Qc :: a 2 : b 2 : c 2
Þ v0 = ç ÷
m è 15 ´ 4pe 0 a ø
é a2 ù
7. (d) When charge Q is on inner solid conducting sphere \ Qa = ê 2 2 2 ú Q
ëê a + b + c úû
+Q é b2 ù
Qb = ê 2 2 2 ú Q
–Q êë a + b + c úû
+Q
E é c2 ù
Qc = ê 2 2 2 ú Q
êë a + b + c úû
Electric field between spherical surface
Q é ( a + b + c) ù
r KQ r r \ V= ê ú
E = 2 Soò E.dr = V given 4p Î0 ë a 2 + b2 + c2 û
r 11. (d) Let at a distance ‘x’ from point B, both the dipoles
Now when a charge – 4Q is given to hollow shell
produce same potential
R
+Q
–Q 4qa 2qa
–3Q
4qa 2qa
\ = 2
(R + x) x ( )
Electric field between surface remain unchanged. R
Þ 2x = R + x Þ x =
r KQ 2 –1
E= 2 Therefore distance from A at which both of them produce
r the same potential
as, field inside the hollow spherical shell = 0
R 2R
\ Potential difference between them remain unchanged = +R =
r r 2 –1 2 –1
i.e. ò E.dr = V 12. (b) Electric field at a point outside the sphere is given by
1Q Q
uur E= But r = 4
8. (c) Given, E = ( Ax + B ) iˆ 4p Î0 r 2
pR 3
3
or E = 20x + 10
rR 3
Using V = ò Edx , we have \E =
3 Î0 r 2
1 At surface r = R
V2 – V1 = ò ( 20x + 10) dx = – 180 V rR 3
-5 \E =
3 Î0
or V1 – V2 = 180 V
Let r1 and r2 are the charge densities of two sphere.
9. (b) Electric potential is constant inside a charged spherical rR r R
shell. E1 = 1 and E2 = 2 2
3e 0 3e 0
P-268 Physics

E1 r1 R1 R - dv
Q = = 1 16. (b) S1 = = 10 | z |
E2 r2 R2 R2 dr
This gives r1 = r2 = r - dv
Potential at a point outside the sphere S2 = = 10 (constant : E)
dr
æ ö \ The source is an infinity large non conducting thick
3 ç ÷
1 Q = rR çQ r = Q ÷ plate of thickness 2 m.
V= 3e0 r ç 4 3÷
4pe 0 r ç pR ÷ r×A µ Z
è 3 ø \10Z ×10A =
At surface, r = R e0
rR 2 rR12 rR 2 r 0 = 10e0 for | z | £ 1m .
V= so, V1 = and V2 = 2
3e 0 3e 0 3e 0
Kq
2 17. (a) We know, V0 = = V surface
V1 æ R1 ö R
\ =
V2 çè R2 ÷ø Now, Vi =
Kq
(3R 2 – r 2 ) [For r < R]
2R 3
KQ
13. (b) Potential outside the shell, Voutside = At the centre of sphare r = 0. Here
r
where r is distance of point from the centre of shell 3
V = V0
KQ 2
Potential inside the shell, Vinside =
R 5 Kq Kq
where ‘R” is radius of the shell Now, = (3R 2 – r 2 )
4 R 2R 3
s
C
–s R
B Þ R2 =
s 2
a A
3 Kq Kq
= 3
b c 4 R R
1 Kq Kq
Kq A Kq B KqC =
VB = + + 4 R R4
rb rb rc
R4 = 4R
1 é s 4pa 2 s4 pb 2 s 4pc2 ù
VB = ê - + ú Also, R1 = 0 and R2 < (R4 – R3)
4p Î0 ëê b b c ûú
dv
s é a 2 - b2 ù 18. (a) As we know, E = –
VB = ê + cú dx
Î0 êë b úû Potential at the point x = 2m, y = 2m is given by :
14. (c) Potential gradient is given by,
DV = E.d V 2, 2

0.8 = Ed (max) ò dV = – ò (25dx + 30dy )


DV = Ed cos q = 0.8 × cos 60 = 0.4 0 0
Hence, maximum potential at a point on the sphere on solving we get,
= 589.4 V V = – 110 volt.
19. (c) Potential difference between any two points in an
- dv
15. (c) As we know electric field, E = electric field is given by,
dr r uur
E = constant \ dv and dr same dV = –E × dx
Kf
E= 2 =c VA 2
r ò dV = - ò 30 x 2 dx
VO 0
Þ f µ r2 v0 v0 + v v0 + 2D v

r r r
\ f = ò r4 pr 2 dr V A - VO = -[10 x 3 ]20 = -80 J/ C
0 20. (a) The potential of uncharged body is less than that of
1 the charged conductor and more than at infinity.
Þrµ
r
Electrostatic Potential and Capacitance P-269

21. (c) O By Gauss's theorem


1 q
q Tcosq E= ....(ii)
4 pe 0 r 2
q
q C Tsinq q From (i) and (ii),
x Fe Q = –8 pe0ar3
Þdq = – 24pe0ar2 dr
mg
dq
In equilibrium, Fe = T sin q Charge density, r = = – 6e0a
4pr 2dr
mg = T cos q Y
Fe q2
tan q = = 26. (c) A(Ö2,Ö2)
mg 4p Î0 x 2 ´ mg ®
rA
q2
\ x= O X
4p Î0 tan q mg (0,0) ®
rB B (2,0)
Electric potential at the centre of the line
kq kq
The distance of point A ( )
2, 2 from the origin,
V= + = 4 kmg / tan q
x/2 x/2
rA = ( 2)2 + ( 2 ) 2 = 4 = 2 units.
22. (c) q = 1µC = 1 × 10–6C
r = 4 cm = 4 ×10–2 m The distance of point B(2, 0) from the origin,

kq 9 ´ 109 ´10 -6
rB = (2) 2 + (0) 2 = 2 units.
Potential V = = = 2.25 × 105 V.
r 4 ´ 10-2 Now, potential at A, due to charge q = 10 –3 mC
1 Q
kq VA = ×
Induced electric field E = – 4 p Î0 ( rA )
r2
1 Q
9 ´109 ´1´10-6 Potential at B, due to charge Q = 10–3 QC VB = ×
= = –5.625 × 106 V/m 4 p Î0 ( rB )
16 ´10-4 \ Potential difference between the points A and B is given
23. (c) Let q1 and q2 be charge on two spheres of radius by
'r' and 'R' respectively
1 10 –3 1 10 –3
As, q1 + q2 = Q VA – VB = × - ×
4p Î0 rA 4p Î0 rB
and s1 = s2 [Surface charge density are equal]
10–3 æ 1 1 ö 10 –3 æ 1 1 ö
q1 q2 = ç - ÷= -
\ = 4p Î0 è rA rB ø 4 p Î0 çè 2 2 ÷ø
r pr 2 4pR 2
Qr 2 QR 2 Q
= ´ 0 = 0.
So, q1 = and q2 = 4p Î0
R2 + r 2 R2 + r 2
27. (a) As shown in the figure, the resultant electric fields
1 é q1 q 2 ù
Now, potential, V = ê + ú before and after interchanging the charges will have the
4 pe 0 ë r R û
same magnitude, but opposite directions.
1 é Qr QR ù As potential is a scalar quantity, So the potential will be
+ same in both cases.
4pe 0 êë R 2 + r 2 R 2 + r 2 úû
=
q q
Q( R + r ) 1 A B
= 2 2 4pe0
R +r
24. (c) Charges reside only on the outer surface of a
conductor with cavity.
25. (c) Electric field
®
df E
E=- = – 2ar ....(i) C
dr D
-q -q
P-270 Physics
-q -q Electric potential due to charge q inside the shell is
A ® B 1 q
E V2 =
4pe o R
\ The net electric potential at point P is
1 2Q 1 q
V = V1 + V2 = +
4pe o R 4pe o R
D C
31. (d) By using energy conservation,
q q
DKE + (DPE )Electro + ( DPE )gravitational = 0
20
28. (a) Given, potential V(x) = 2 volt
x -4
dV d æ 20 ö Q R
Electric field E = - =- ç
dx dx è x 2 - 4 ÷ø
40 x
ÞE= + q
( x - 4) 2
2
y
At x = 4 mm , v
40 ´ 4 160 10
E=+ =+
= + volt / mm.
(42 - 4) 2 144 9 1 æ Qq Qq ö
r mV 2 + ç k -k + (- mgy ) = 0
Positive sign indicates that E is in +ve x-direction. 2 è R+ y R ø÷
29. (a) q q 1 æ1 1 ö
Þ mV 2 = mgy + kQq ç -
2 è R R + y ø÷
2kQq y
Þ V 2 = 2 gy +
R
m R( R + y)
R
1 2 é qQ ù
or, V 2 = 2 y ê + gú
d ë 4 pe 0 R ( R + y ) m û
Potential at the center of ring of charge +q = potential due
32. (d) Change in potential energy, Du = q(V f - Vi )
to iteself + potential due to other ring of charge –q.
éq ù Potential of – q is same as initial and final point of the path.
1 q
Þ V1 = ê - ú Y
4pe 0
êë R R 2 + d 2 úû
Potential at the centre of ring of charge –q = potential due q
to itself + potential due to other ring of charge +q.
1 é- q q ù 4q –q X
O
Þ V2 = ê + ú d/2
4pe 0 êë R R2 + d 2 úû d/2 d
DV = V1 – V2
1 éq q ù æ k 4q k 4q ö 4q 2
ê + -
q
-
q
ú Du = q ç - ÷ =-
= è 3d / 2 d / 2 ø 3pe 0 d
4 pe 0 ëê R R R2 + d 2 R 2 + d 2 ûú
–ve sign shows the energy of the charge is decreasing.
1 éq q ù
= ê - ú 33. (c) According to work energy theorem, gain in kinetic
2pe 0êë R R 2 + d 2 úû energy is equal to work done in displacement of charge.
30. (c) Electric potential due to charge Q at point P is 1 2
\ mv = q DV
2
R/2 Here, DV = potential difference between two positions of
Q P charge q.
R q For same q and DV.

1 Q 1 2Q 1
V1 = = vµ
4pe o R / 2 4pe o R m
Electrostatic Potential and Capacitance P-271

Mass of hydrogen ion mH = 1 38. (b) Net electrostatic energy for the system
Mass of helium ion mHe = 4
é q 2 Qq Qq ù
U = Kê + + =0
vH 4 a a a 2ú
\ = = 2 :1. êë úû
vHe 1
Q
é 1 ù
34. (c) Using conservation of energy Þ q = -Q ê1 + ú
2
1 ë û a a 2
U i = U F + mv 2
2 -q 2
ÞQ=
kq1q2 kq1q2 1 2 2+1 a
= + mv +q +q
r1 r2 2
(O,2 ) Q Q(4, + 2)
1 é1 1 ù
Þ mv 2 = kq1q2 ê - ú
2 ë r1 r2 û 39. (b)
2kq1q2 é1 1 ù
v2 = ê - ú Q(4, – 2)
m ë r1 r2 û (O, – 2 ) Q

2 ´ 9 ´ 109 ´ 10-12 é 1 ù Potential at origin


= -6 -3 ê
1 - ú = 4 ´ 10+6
4 ´ 10 ´ 10 ë 9û KQ KQ KQ KQ
v = 2 × 10 m/s
3 v= 2 + 2 + +
20 20
é ù æ 1 ö
1 ê q(- q) qQ ( - q )Q ú and potential at ¥ = 0 =KQ ç 1+ ÷
U= ê + + ú è 5ø
35. (d) 4 p Î0 ê d æ dö æ dö ú
êë çè D + ÷ø çè D - ÷ø ú \ Work required to put a fifth charge Q at origin W =
2 2 û
Q2 æ 1 ö
1 é q 2
qQd ù 2 VQ = 4 ç 1+ ÷
= ê- - 2 ú , Ignoring d pe0 è 5ø
4p Î0 êë d D úû 4 40. (c) The work done in moving a charge along an
36. (b) Using, [K + U]i = [K + U]f equipotential surface is always zero.
or 0 + Vq = mv2 + v’q The direction of electric field is perpendicular to the
or mv2 = (V – V’)q equipotential surface or lines.
r r
41. (c) The potential energy at the centre of the sphere
l lq æ ln 3 ö
- q ò Edr = q ò dr = ç ÷ 3 KQ q
= 2p Î0 r 2p Î0 è r0 ø Uc =
r0 r0 2 R
The potential energy at the surface of the sphere
r
Þ vµ lnr Us =
K qQ
0 R
37. (c) Total energy of charge distribution is constant at any Now change in the energy
instant t.
DU = U c -U s
Uf + Kf = Ui + Ki
K Qq é 3 ù KQq
1 KQ2 KQ2 = -1 =
i.e., mV 2 + = 0+ R êë 2 úû 2R
2 2R 2R 0
4 3
Where Q = r.V = r. pR
1 KQ 2 KQ2 3
\ mV 2 = -
2 2R 0 2R 2K pR 3 rq
DU =
3 R
3
2 KQ 2 æ 1 1ö 2 1 pR rq
\ V= ç - ÷ DU = ´
m 2 è R0 R ø 3 4 p Î0 R
R 2 rq
2
KQ æ 1 1ö 1 1 DU =
\ V= ç - ÷=C - 6 Î0
m è R0 R ø R0 R Using Gauss’s law
Also the slope of V – R curve will go on decreasing.
P-272 Physics

4 1 kqQ r
r uur qen b ´ pR 3 Þ m(2v )2 = Þ r'=
ò E × dA = E0 = E30 2
46. (a) By using
r' 4

b ´ 4 pR 3 W = q(VB – VA)
Þ ò EdA(cos q) = 3E0 2J
\ VB – VA = = 0.1J/C = 0.1V
4 3 1 20C
Þ E(4pR2) = b ´ 3 pR ´ E
0 47. (b) When capacitors C and 2C capacitance are charged
br to V and 2V respectively.
Þ E= (r < R)
3E0
+
C– +
2C–
42. (d) Initial potential of the charge,
2 kq 2 kq V 2V
VA = -
a a 5 Q1 = CV Q2 = 2C ´ 2V = 4CV
1 2q æ 1 ö When connected in parallel
Þ VA = ç 1– ÷ Q1 = CV
4 pE a è 5ø
kq
+ –
(Here potential due to each q = and potential due
a
- kq
to each – q = )
a 5
– +
2a Q2 = 4CV
q q
A By conservation of charge
4CV - CV = (C + 2C )Vcommon
2a 3CV
B Vcommon = =V
3C
Therefore final energy of this configuration,
æ1 1 ö 3
–q –q U f = ç CV 2 + ´ 2CV 2 ÷ = CV 2
è2 2 ø 2
Final potential of the charge
VB = 0 48. (a) +q1 – V0 – +q24 m F
(Q Point B is equidistant from all the four charges)
2 mF
\ Using work energy theorem, + Q
(WAB)electric = Q(VA – VB) – 5 mF

2qQ é 1 ù
= 1-
4pE0 a êë ú
5û 6V 0V
6V

æ 1 ö 2Qq é 1 ù Let q1 and q2 be the charge on the capacitors of 2mF and


= ç 4 pe ÷ a ê1 -
5 úû
4mF. Then charge on capacitor of 5mF
è 0ø ë
Q = q1 + q2
43. (c) Work done, WPQ = q(VQ – VP)
= (–100 × 1.6 × 10–19)(– 4 – 10) Þ 5V0 = 2(6 - V0 ) + 4(6 - V0 )
= +2.24 × 10–16J Þ 5V0 = 12 - 2V0 + 24 - 4V0
44. (a) Gain in kinetic energy = work done by potential 36
difference Þ 11V0 = 36 Þ V0 = V
11
1 2 2eV 180
eV = mv Þ v = Þ Q = 5V0 = mC
2 m 11
49. (d) When two capacitors with capacitance C1 and C2 at
2 ´ 1.6 ´ 10-19 ´ 20
= = 2.65 × 106 m/s potential V1 and V2 connected to each other by wire, charge
9.1 ´ 10 -31 begins to flow from higher to lower potential till they
1 2 kQq acquire common potential. Here, some loss of energy takes
45. (d) mv = place which is given by.
2 r
Electrostatic Potential and Capacitance P-273

C1C2 If C2 be the capacitance of uncharged capacitor, then


Heat loss, H = (V1 - V2 )2 common potential is
2(C1 + C2 )
In the equation, put V2 = 0, V1 = V0 C1V1 + C2V2
V=
C C1 + C2
C1 = C, C2 =
2
10 ´ 50 + 0
Þ 20 = Þ C = 15 mF
C 20 + C

2 (V - 0) 2 = C V 2 53. (c) In parallel combination, Ceq = C1 + C2 = 10 mF
Loss of heat =
æ Cö 0 6
0
When connected across 1 V battery, then
2çC + ÷
è 2ø c1
1 2
H = CV0 æ1 2ö
6 CV
U1 çè 2 1 ÷ø 1 C 1 c2
50. (b) According to question, = = Þ 1 =
U2 æ 1 2ö 4 C2 4
Q = 750mC = q2 + q3 çè C2V ÷ø
2
C2 q2 1v
C1 = 15 mF \ C2 = 8 mF and C1 = 2 mF
Q Now C1 and C2 are connected in series combination,
C3 = 8 mF
750m C q3 CC 2 ´ 8 16
\ Cequivalent = 1 2 = = = 1.6mF
C1 + C2 2 + 8 10
V1 V2 = 20V
54. (a)
dx
Capacitors C2 and C3 are in parallel hence, xtan a
a
Voltage across C2 = voltage across C3 = 20 V x
Change on capacitor C3, d
q3 = C3 ´ V3 = 8 ´ 20 = 160mC x= 0 a
\ q2 = 750mC - 160mC = 590mC Consider an infinitesimal strip of capacitor of thickness
dx at a distance x as shown.
51. (4)
Capacitance of parallel plate capacitor of area A is given
Given, C1 = 5 mF and V1 = 220 Volt
e0 A
When capacitor C1 fully charged it is disconnected from by C =
the supply and connected to uncharged capacitor C2. t
[Here t = seperation between plates]
C2 = 2.5 mF, V2 = 0 So, capacitance of thickness dx will be
Energy change during the charge redistribution,
e 0 adx
1 C1C2 \ dC =
DU = U i - U f = (V1 - V2 ) 2 d + x tan a
2 C1 + C2 Total capacitance of system can be obtained by
1 5 ´ 2.5 integrating with limits x = 0 to x = a
= ´ (220 - 0)2 mJ x=a
2 (5 + 2.5) dx
\ Ceq = ò dC = ae 0 ò
5 x tan a + d
= ´ 22 ´ 22 ´ 100 ´ 10 -6 J x=0
2´3
[By Binomial expansion]
5 ´ 11 ´ 22 55 ´ 22
= ´ 10 -4 J = ´ 10 -4 J a a
3 3 ae æ x tan a ö ae æ x 2 tan a ö
Þ Ceq = 0 ò ç1 – ÷ dx = 0 ç x – ÷
=
1210
´ 10-4 J =
1210
´ 10-3 J ; 4 ´ 10 -2 J d è d ø d è 2d ø
0 0
3 3
x a 2 e 0 æ a tan a ö e0 a 2 æ aa ö
According to questions, = 4 ´ 10 -2 Þ Ceq = = ç1 – ÷= çè1 – ÷ø
100 d è 2d ø d 2d
\x = 4
52. (a) Given, 55. (a) Given, K (x) = K(1 + ax)
Capacitance of capacitor, C1 = 10 mF K e0 A
Potential difference before removing the source voltage, Capacitance of element, Cel =
dx
V1 = 50 V
P-274 Physics

e0 K (1 + ax) A E1 9k1k2 k3
Þ Cel = = E = ( k + k + k )(k k + k k + k k )
dx 2 1 2 3 1 2 2 3 3 1

d 59. (d) V1 + V2 = 10
æ1ö 1 æ dx ö
\ òdç ÷ = = òç ÷ 4µF 6µF
C
è ø C e KA(1 + a x )
el 0è 0 ø
1 1
Þ = [ln(1 + ax )]d0
C e 0 KAa
10V
1 1 and 4V1 = 6V2
Þ = ln(1 + ad )[a d << 1]
C e 0 KAa On solving above equations, we get
V1 = 6 V
1 é a2d 2 ù
= êad - ú Charge on 4 mf,
e0 KAa ëê 2 úû
q = CV1 = 4 × 6 = 24 mC.
1 é ad ù 60. (a) Equivalent capacitance in series combination (C’) is
= 1- given by
e 0 KA êë 2 úû x dx
1 1 1 C1C2
e 0 KA e KA æ ad ö = + Þ C' =
\C = ÞC= 0 ç 1+ ÷ C ' C1 C2 C1 + C2
æ ad ö d è 2 ø
d ç1 - ÷ For parallel combination equivalent capacitance
è 2 ø
C” = C1 + C2
56. (6) In the first condition, electrostatic energy is For parallel combination
1 1 q = 10(C1 + C2)
Ui = CV02 = ´ 60 ´ 10 –12 ´ 400 = 12 ´ 10 –9 J
2 2 q1 = 500 µC
1 500 = 10(C1 + C2)
In the second condition U F = C 'V ' 2 C1 + C2 = 50µF ....(i)
2
For Series Combination–
2
1 æV ö æ V0 ö
Uf = 2C . ç 0 ÷ çèQ C ' = 2C , V ' = 2 ÷ø q 2 = 10
C1C 2
2 è 2ø
( C1 + C2 )
1 C1C 2
= ´ 60 ´ 10 -12 ´ (20) 2 = 6 × 10–9 J 80 = 10 From equation ....(ii)
4 50
Energy lost = Ui – Uf = 12×10–9J –6 × 10–9J = 6 nJ C1C2 = 400 ....(iii)
CV + (nC )V From equation (i) and (ii)
57. (d) V ¢= C1 = 10µF C2 = 40µF
kC + nC
qæ 1 1 ö
(n + 1)V 61. (b) w = w f - vi = 2 çç C - C ÷÷
k+ n è f i ø

1 d /3 d /3 d /3 (5 ´ 10)2 æ 1 1 ö 6
= çè - ÷ø ´ 10
58. (c) C = k e A + k e A + k e A 2 2 5
1 1 0 2 0 3 0
= 3.75 × 10–6J
3k1k2 k3 e 0 A
or C1 = d (k k + k k + k k ) 62. (b) Capacitance of a capacitor with a dielectric of dielectric
1 2 2 3 3 1 constant k is given by
C2
k Î0 A
k1e 0 ( A / 3) k 2 e 0 ( A / 3) k3 e 0 ( A / 3) C=
= + + d
d d d V k Î0 AE
(k1 + k2 + k3 )e 0 A Q E= \ C=
= d V
3d k ´ 8.86 ´ 10 -12
´ 10-4 ´ 106
15 ´ 10-12 =
1 500
C1V 2
U1 2 k = 8.5
=
U2 1
C V2
2 2
Electrostatic Potential and Capacitance P-275

Q 68. (b) As required equivalent capacitance should be


63. (b) V = Q1 Q2
C 6
Ceq = mF
13
æ Q - Q2 ö
=ç 1 Therefore three capacitors must be in parallel and 4 must
è 2C ÷ø
be in series with it.
æ 4 - 2ö Q1 – Q2 (Q1 – Q2 1 é 1 ù é1 1 1 1ù

(
=ç ÷ =1V = + + + +
è 2 ´1ø 2 2 Ceq êë 3C úû êë C C C C úû
64. (b) Energy stored in the system initially 3C 6
1 Ceq = = mF [ as C = 2 m F]
Ui = CE 2 13 13
2 So, desired combination will be as below:
1 Q 2 (CE)2 1 CE 2
Uf = = =
2 Ceq 2 ´ 4C 2 4
[As Q = CE, and Ceq = 4C]
1 3 3 3 Q2
DU = CE 2 ´ = CE 2 =
2 4 8 8 C
69. (b) W = – Du
s Q
65. (c) E = e = Ae ( ce ) 2 - ( ce ) 2
0 0 = ( -1)
\Q = e0. E. A = 8.85 × 10–12 × 100 × 1 2kc 2c
= 8.85 × 10–10C
C 1 e2 c k - 1
66. (a) A =
2 k
= 508 J

4 70. (c) Let dielectric constant of material used be K.


3 k1 Î0 A1 k 2 Î0 A 2 k 3 Î0 A 3 k Î0 A
+ + =
7 B d d d d
C 3 10 Î0 A/3 12 Î0 A/3 14 Î0 A/3 K Î0 A
or + + =
A B d d d d
For series combination Î0 A æ 10 12 14 ö K Î0 A
ç + + ÷=
1 1 1 d è 3 3 3ø d
Ceq
= C +C
1 2 \ K = 12
7C 71. (b)
1
Þ 73 = d
+C 2 y k
3
xa dx
Þ 14 C = 7 + 3 C
a
7
ÞC= mF y d d
11 From figure, = Þy= x
x a a
6mF
d 1 y (d - y)
30mC dy = (dx) Þ = +
67. (d) - + a dc Ke 0 adx e0 adx
- + - + - +
10mF 2mF 1 y æy ö
= çè + d - y÷ø
4mF dc e 0 abx k
As given in the figure, 6µF and 4µF are in parallel. Now e 0 adx
using charge conservation
ò dc = ò y
+d-y
k
6
Charge on 6µF capacitor = ´ 30 = 18µC a
d
dy
6+4

c = e 0 a.
Since charge is asked on right plate therefore is +18µC or, æ1 ö
0 d + y ç - 1÷
èk ø
P-276 Physics

d 74. (a) During charging charge on the capacitor increases


e0a 2 é æ æ 1 ööù with time. Charge on the capacitor C1 as a function of time,
= êl n ç d + y çè - 1÷ø ÷ ú
æ1 ö ë è k ø û0 Q = Q0(1 – e–t/RC) Ceq
çè - 1÷ø d
k
- t RCeq ù
Q = Ceq E éë1 - e û
æ æ1 öö
k Î0 a 2 ç d + d ç k - 1÷ ÷ (Q Q0 = Ceq E)
= lnç è ø÷ R
E
(1 - k ) d ç d ÷ Both capacitor will have charge as they are connected in
ç ÷
è ø series

k Î0 a 2 2
æ 1 ö k Î0 a lnk 75. (d) The simplified circuit of the circuit given in question
= lnç ÷ = as follows:
(1 - k ) d è k ø ( k - 1) d 6m F 2m F
C D E 4m F
k1 k2 A B
k1 k2 L/2 C1 C2
72. (Bonus) Þ k3 k4 5m F 2m F
k3 k4 L/2
C3 C4
C12 5m F
The equivalent capacitance between C & D capacitors of
Þ 2mF, 5mF and 5 mF are in parallel.
Ceq \ CCD= 2 + 5 + 5 = 12 mF (Q In parallel grouping
C34 Ceq = C1 + C2 +.... + Cn)
Similarly equivalent capacitance between E & B CEB
L é L ù
k1 Î0 ´ L k 2 êÎ0 ´ L ú = 4 + 2 = 6mF
. ë û
2 2 Now equivalent capacitance between A & B
C1C2 d/2 d/2
C12 = = 1 1 1 1 5
C1 + C2 é L ù = + + =
êÎ0 2 ´ L ú Ceq 6 12 6 12
(k1 + k 2 ) ê ú
ë d/2 û 12
Þ C eq = = 2.4 mF (Q In series grouping,
5
k1k 2 Î0 L2
C12 = 1 1 1 1
k1 + k 2 d = + + .......... + )
in the same way we get, C eq C1 C 2 Cn
76. (c) Given area of Parallel plate capacitor, A = 200 cm2
k3k 4 Î0 L2
C34 = Separation between the plates, d = 1.5 cm
k3 + k 4 d Force of attraction between the plates, F = 25 × 10–6N
é kk k k ùÎ L2 F = QE
\ C eq = C12 + C34 = ê 1 2 + 3 4 ú 0 .. (i)
ë k1 + k 2 k3 + k 4 û d Q2 s Q
F= (E due to parallel plate = = )
2 A Î0 2 Î0 A2 Î0
k Î0 L2
Now if keq = K, Ceq = ... (ii)
d d = 1.5 cm
Î0 A(V )
on comparing equation (i) to equation (ii), we get But Q = CV =
d
k1k 2 (k3 + k 4 ) + k3 k 4 (k1 + k 2 )
k eq = (Î0 AV )2
(k1 + k 2 )(k 3 + k 4 ) \ F= + –
This does not match with any of the options so this must d 2 ´ 2 A Î0
V
be a bonus.
2 2 2
73. (a) Charge on Capacitor, Qi = CV (Î0 A) ´ V (Î0 A) ´ V
= 2
=
After inserting dielectric of dielectric constant d ´ 2 ´ ( A Î0 ) d2 ´ 2
= K Qf = (kC) V
(8.85 ´ 10-12 ) ´ (200 ´ 10 -4 ) ´ V 2
Induced charges on dielectric or, 25 ´ 10-6 =
Qind = Qf – Qi = KCV – CV 2.25 ´ 10-4 ´ 2

æ5 ö
( K - 1)CV = ç - 1 ÷ × 90 pF × 2V = 1.2nc 25 ´ 10-6 ´ 2.25 ´ 10-4 ´ 2
Þ V= » 250 V
è3 ø 8.85 ´ 10-12 ´ 200 ´ 10-4
Electrostatic Potential and Capacitance P-277

77. (a) The sum of final charges on C2 and C3 is 36 µC.


é æ 12 ö ù
78. (b) To get a capacitance of 2 m F arrangement of Charge on C1 is q1 = ê ç ÷ø ´ 8 ú ´ 4 = 24mC
ë è 4 + 12 û
capacitors of capacitance 1 mF as shown in figure 8
capacitors of 1mF in parallel with four such branches in
4
series i.e., 32 such capacitors are required. The voltage across CP is VP = ×8 = 2V
4 + 12
\ Voltage across 9mF is also 2V
\ Charge on 9mF capacitor = 9 × 2 = 18mC
\ Total charge on 4 mF and 9mF = 42mC

m m m m KQ 42 ´ 10-6
\E = = 9 × 109 × = 420 NC–1
r2 30 ´ 30
82. (a) Capacitors 2mF and 2mF are parallel, their equivalent
= 4 mF
1 1 1 1 1
= + + + \ Ceq = 2 mF 6mF and 12 mF are in series, their equivalent = 4 mF
Ceq 8 8 8 8
3
79. (c) Before introducing a slab capacitance of plates Now 4mF (2 and 2 mF) and 8mF in series = mF
e A 8
C1 = 0 And 4mF (12 & 6 mF) and 4mF in parallel = 4 + 4 = 8mF
3
If a slab of dielectric constant K is introduced between 1 8
plates then 8mF in series with 1mF = + 1 Þ mF
8 9
Ke 0A e A
C= then C1' = 0 8 8 32
d 2.4 Now Ceq = + =
' 9 3 9
C1 and C1 are in series hence, C1'
e A e A 32
e0A k 0 . 0 Ceq of circuit =
3 2.4 9
=
3 e0 A e 0 A 1 1 9 32
k + With C – = + =1Þ C =
3 2.4
3 k = 2.4 k + 3 Slab C eq C 32 23
0.6 k = 3 83. (d) To get effective capacitance of 6 mF two capacitors of
Hence, the dielectric constant of slap is given by, 4 mF each connected in sereies and one of 4 mF capacitor
30 in parallel with them.
k= =5
6 4mF 4mF
Q2
80. (d) Energy of sphere =
2C
16 ´ 10 -12
4.5 =
2C 4mF
16 ´ 10-12 Two capacitances in series
C= = 4pe 0R
9 1 1 1 1 1 1
(capacity of spherical conductor) \ = + = + =
C C1 C2 4 4 2
16 ´ 10-12 1 1 9 1 capacitor in parallel
R= ´ Q 4pe = 9 ´ 10 \ C eq = C3 + C = 4 + 2 = 6 mF
9 4pe 0 0

16 Q1 1mF
= 9 ´ 109 ´ ´ 10-12 = 16 mm 84. (d)
9 Q
3µF
C1 = 4µF 12µF = CP
81. (a) 4µF C
9µF Q2 2mF
Þ
2µF
8V 8V
2 2
From figure, Q2 = Q = Q
2 +1 3
P-278 Physics

æ C ´3 ö s æ ld ö
Q= Eç ÷ = l ln ç1 + K ÷
è C + 3ø è 0ø
2 æ 3CE ö 2CE Now it is given that capacitance of vacuum = C0.
\ Q2 = ç ÷=
3è C + 3ø C +3 Q
Thus, C =
Therefore graph d correctly dipicts. V
s.s
Charge = (Let surface area of plates = s)
v
s.s
=
s æ ld ö
ln ç1 + ÷
l è K0 ø
d 1
= sl. (Q in vacuum e0 =1)
d æ ld ö
C ln ç1 + ÷
1mF 3mF è K0 ø
85. (a) when switch is closed ld æ sö
c= . C0 çè here, C0 = ÷ø
–10 C 15 C æ ld ö d
ln ç 1 + ÷
è K0 ø
89. (c)
90. (d) Possible combination of capacitors
(i) Three capacitors in series combination
5V 5V

3µF 3µF 3µF


–15 C +10 C
When switch is open 1 1 1 1
= + +
Ceq 3 3 3
–12 C +12 C
1
\ = 1mF
Ceq
6V 4V
(ii) Three capacitors in parallel combination

4V 6V 3µF

–12 C +12 C 3µF


Charge of 5mc flows from b to a
3µF
86. (a) Electric field in presence of dielectric between the two Ceq = 3 + 3 + 3 = 9 µF
plates of a parallel plate capaciator is given by, (iii) Two capacitors in parallel and one is in series
s 3µF
E=
Ke 0
Then, charge density
s = Ke0E 3µF
= 2.2 × 8.85 × 10–12 × 3 × 104 3µF
» 6 × 10–7 C/m2 Ceq = 2µF
87. (d) (iv) Two capacitors in series and one is in parallel
88. (c) The value of dielectric constant is given as,
K = K 0 + lx
d d
s
And, V = ò Edr Þ V = ò dx
K
0 0 Ceq = 4.5 µF
d
1 s k Î0 A
= s ò ( K + l x ) dx = éëln ( K 0 + ld ) - ln K 0 ùû 91. (b) C0 =
d
0 0 l
Electrostatic Potential and Capacitance P-279

k Î0 2 2k Î0 A 4 k Î0 A 95. (c) The dielectric constant of the gas is 1.01


C= + = 96. (d) Capacitance of sphere is given by :
3d 3d 3 d
4 k Î0 A C = 4 p Î0 r
C 4 If, C = 1F then radius of sphere needed:
\ = 3 d =
C0 k Î0 A 3 C 1
d r= =
4p Î0 4p´ 8.85 ´10 -12
Q It
92. (c) As, C = = 1012
V V or, r = = 9 × 109 m
V I 2 4p´ 8.85
Þ = =
t C 1 ´ 10 -6 9 × 109 m is very large, it is not possible to obtain such
= 2 × 106 V/s a large sphere. Infact earth has radius 6.4 × 106 m only
93. (c) When charged particle enters perpendicularly in an and capacitance of earth is 711mF.
electric field, it describes a parabolic path
2 97. (d) Equivalent capacitance of n2 number of capacitors
1 æ QE öæ x ö each of capacitance C2 in parallel = n2C2
y= ç ÷ç ÷
2 è m øè 4 ø Equivalent capacitance of n 1 number of capacitors each
This is the equation of parabola. of capacitances C1 in series.
C
y Capacitance of each is C1 = 1
E n1
According to question, total energy stored in both the
combinations are same
P(x, y)
u x 1 æ C1 ö 1
i.e., ç ÷ ( 4V ) 2 = ( n2 C2 ) V 2
2 è n1 ø 2
94. (d) The discharging of a capacitor is given as
16C1
q = q 0 exp [ - t / RC ] \ C2 =
RC = time constant = t n1 n2
98. (c) Charge (or current) always flows from higher potential
q = q 0 e-t/ t to lower potential.
If e is the capacitance of the capacitor
Charge
q = CV and q = CV0 Potential=
Capacitance
Thus, CV = CV0 e t / t
q2
V = V0 e -t / t…(i) 99. (c) Initial energy of capacitor, E1 = 1
2C
From the graph (given in the problem Final energy of capacitor,
when t = 0.5, V = 25 i.e., æ q1 ö
2
V0 = 25 volt. 1 q12
and when t = 200, V = 5 volt E2 = E1 = =ç 2÷
2 4C ç ÷
Thus equation (i) becomes è 2C ø
1
5 = 25e -200/ t
\ t1 = time for the charge to reduce to of its initial
2
Þ 1 / 5 = e -200/ t value
Taking loge on both sides 1
and t2 = time for the charge to reduce to of its initial
1 4
loge = -200 / t Þ – 200 = log e 5 value
5 t æ q2 ö t
200 We have, q2 = q1e-t / CR Þ ln ç q ÷ = -
t= è 1ø CR
log e 5 -
æ 1 ö t1
200 200 \ ln ç ÷= ...(1)
or t = = è 2 ø CR
æ 10 ö log e 10 - log e 2
log e ç ÷ æ 1 ö -t2
è 2ø and ln ç ÷ = ...(2)
è 4 ø CR
200 200
æ 1 ö ln æç ö÷
t= = = 124.300 1
2.302 - 0.693 1.609 ln ç ÷
t1 1 è 2ø 1
= è ø =
By (1) and (2) , 2 =
Which lies between 100 s and 150 s 2 æ 1 ö 4
ln æç ö÷
t2 1 2ln ç ÷
è4ø è2ø
P-280 Physics

103. (c) Applying conservation of energy,


Electric potential energy of capacitor = heat absorbed
1 2m. s. Dt
100. (c) CV 2 = m. s Dt ; V =
2 C
104. (b) The capacitance without aluminium foil is
The capacitance with air between the plates e A
C= 0
d
e0 A Here, d is distance between the plates of a capacitor
C= = 9pF
d A = Area of plates of capacitor
On introducing two dielectric between the plates, the given When an aluminium foil of thickness t is introduced
capacitance is equal to two capacitances connected in between the plates.
series where
e A
k Î A 3Î A Capacitance, C¢ = 0
C1 = 1 0 = 0 d –t
d /3 d /3 If thickness of foil is negligible 50 d – t ~ d. Hence, C = C¢.
3 ´ 3Î0 A 9 Î0 A 105. (d) The work done is stored in the form of potential energy
= = which is given by
d d
and 1 Q2
U=
kÎ A 3k2Î0 A 2 C
C2 = 2 0 =
2d / 3 2d
\ U = 1´
(
8 ´ 10 )
-18 2
= 32 × 10–32 J
3 ´6 Î0 A 9 Î0 A
= = 2 100 ´ 10 -6
2d d
106. (b) In parallel, equivalent capacitance of n capacitor of
The equivalent capacitance Ceq is capacitance C
1 1 1 C¢ = nC
= + Energy stored in this capacitor
Ceq C1 C2
1 1 2
d d 2d E= C V
= + = 2
9 Î0 A 9 Î0 A 9 Î0 A 1 2 1 2
Þ E = (nC )V = nCV
2 2
9 Î0 A 9
\ Ceq = = ´ 9 pF = 40.5 pF C
2 d 2 C
101. (a) The potential energy of a charged capacitor is given n times
C
2
Q
by U = . V
2C V
When a dielectric slab is introduced between the plates Alternatively
Each capacitor has a potential difference of V between the
Q2 plates.
the energy is given by , where K is the dielectric So, energy stored in each capacitor
2KC
constant. 1
= CV 2 .
Again, when the dielectric slab is removed slowly its 2
energy increases to initial potential energy. Thus, work \ Energy stored in n capacitor
done is zero. é1 2ù
102. (b) As n plates are joined alternately positive plate of all = ê CV ú ´ n
(n – 1) capacitor are connected to one point and negative ë2 û
plate of all (n – 1) capacitors are connected to other point. 107. (a) Capacitance of spherical conductor = 4pE0R
It means (n – 1) capacitors joined in parallel. Here, R is radius of conductor
\ Resultant capacitance = (n – 1)C 1
\ C = 4p Î0 R = ´ 1 = 1.1 ´ 10-10 F
9 ´109
17
Current Electricity P-281

Current Electricity
(a) The emf of the battery is 1.5 V and its internal
Electric Current, Drift of resistance is 1.5 W
TOPIC 1 Electrons, Ohm's Law, (b) The value of the resistance R is 1.5 W
Resistance and Resistivity (c) The potential difference across the battery is 1.5 V when
it sends a current of 1000 mA
1. A circuit to verify Ohm’s law uses ammeter and voltmeter (d) The emf of the battery is 1.5 V and the value of R is 1.5 W
in series or parallel connected correctly to the resistor. 4. A current of 5 A passes through a copper conductor
In the circuit : [Sep. 06, 2020 (II)] (resistivity) = 1.7×10 – 8Wm) of radius of cross-section
(a) ammeter is always used in parallel and voltmeter is 5 mm. Find the mobility of the charges if their drift
series velocity is 1.1×10 – 3 m/s. [10 Apr. 2019 I]
(b) Both ammeter and voltmeter must be connected in (a) 1.8m2/Vs (b) 1.5 m2/Vs
parallel (c) 1.3 m2/Vs (d) 1.0m2/Vs
(c) ammeter is always connected in series and voltmeter 5. In an experiment, the resistance of a material is plotted as
in parallel a function of temperature (in some range). As shown in
(d) Both, ammeter and voltmeter must be connected in the figure, it is a straight line. [10 Apr. 2019 I]
series
2. Consider four conducting materials copper, tungsten,
mercury and aluminium with resistivity rC, rT, rM and rA
respectively. Then : [Sep. 02, 2020 (I)]
(a) rC > r A > rT (b) r M > r A > rC
(c) r A > rT > rC (d) r A > rM > rC One may canclude that:
3. To verify Ohm’s law, a student connects the voltmeter R0
across the battery as, shown in the figure. The measured (a) R(T) = 2
(b) R(T) = R 0 e - T0 /T
2

T2
voltage is plotted as a function of the current, and the
- T 2 /T02 2 2
following graph is obtained : [12 Apr. 2019 I] (c) R(T) = R 0 e (d) R(T) = R 0 eT /T0
6. Space between two concentric conducting spheres of radii
a and b (b > a) is filled with a medium of resistivity r. The
resistance between the two spheres will be :
[10 Apr. 2019 II]
r æ1 1ö r æ1 1ö
(a) ç - ÷ (b) ç - ÷
4p è a b ø 2p è a b ø
r æ1 1ö r æ1 1ö
(c) ç + ÷ (d) ç + ÷
2p è a b ø 4p è a b ø
7. In a conductor, if the number of conduction electrons
per unit volume is 8.5 × 1028 m –3 and mean free time is
25 fs (femto second), it’s approximate resistivity is:
(m e = 9.1 × 10–31 kg) [9 Apr. 2019 II]
(a) 10–6 W m (b) 10–7 Wm
If Vo is almost zero, identify the correct statement: (c) 10–8 Wm (d) 10–5 Wm
P-282 Physics

8. A 200 W resistor has a certain color code. If one replaces 16. A uniform wire of length l and radius r has a resistance of
the red color by green in the code, the new resistance will r
100 W. It is recast into a wire of radius . The resistance
be : [8 April 2019 I] 2
(a) 100 W (b) 400 W (c) 300 W (d) 500 W of new wire will be : [Online April 9, 2017]
9. The charge on a capacitor plate in a circuit, as a function of (a) 1600 W (b) 400 W (c) 200 W (d) 100 W
time, is shown in the figure: [12 Jan. 2019 II] 17. When 5V potential difference is applied across a wire of
6 length 0.1 m, the drift speed of electrons is 2.5 × 10–4 ms–1.
5 If the electron density in the wire is 8 × 1028 m–3, the
4 resistivity of the material is close to : [2015]
q(µc) 3 (a) 1.6 × 10–6 Wm (b) 1.6 × 10–5 Wm
2
0 (c) 1.6 × 10–8 Wm (d) 1.6 × 10–7 Wm
2 4 6 8
t(s) 18. Suppose the drift velocity nd in a material varied with the
What is the value of current at t = 4 s ?
applied electric field E as nd µ E . Then V – I graph for
(a) Zero (b) 3 µA (c) 2 µA (d) 1.5 µA
10. A resistance is shown in the figure. Its value and tolerance a wire made of such a material is best given by :
are given respectively by: [9 Jan. 2019 I] [Online April 10, 2015]
V V

(a) (b)
I I
(a) 270 W, 10% (b) 27 kW, 10%
(c) 27 kW, 20% (d) 270 W, 5% V V
11. Drift speed of electrons, when 1.5 A of current flows in a
copper wire of cross section 5 mm2, is v. If the electron (c) (d)
density in copper is 9 × 1028/m3 the value of v in mm/s
close to (Take charge of electron to be = 1.6 × 10–19C) I I
[9 Jan. 2019 I] 19. Correct set up to verify Ohm’s law is :
(a) 0.02 (b) 3 (c) 2 (d) 0.2 [Online April 23, 2013]
12. A copper wire is stretched to make it 0.5% longer. The
A
percentage change in its electrical resistance if its volume
remains unchanged is: [9 Jan. 2019 I]
(a) 2.0% (b) 2.5% (c) 1.0% (d) 0.5% (a) V
13. A carbon resistance has following colour code. What is
the value of the resistance? [9 Jan. 2019 II]

GOY Golden
(a) 530 kW ± 5% (b) 5.3 kW ± 5%
(b) A V
(c) 6.4 MW ± 5%(d) 64 MW ± 10%
14. A heating element has a resistance of 100W at room
temperature. When it is connected to a supply of 220 V,
a steady current of 2 A passes in it and temperature is
500°C more than room temperature. What is the
A
temperature coefficient of resistance of the heating
element? [Online April 16, 2018]
(c)
(a) 1 × 10–4°C–1 (b) 5 × 10–4°C–1
(c) 2 × 10–4°C–1 (d) 0.5 × 10–4°C–1
V
15. A copper rod of cross-sectional area A carries a uniform
current I through it. At temperature T, if the volume charge V
density of the rod is r, how long will the charges take to
travel a distance d ? [Online April 15, 2018]
(d)
2rdA 2rdA rdA rdA
(a) (b) (c) (d)
IT I I IT A
Current Electricity P-283

20. The resistance of a wire is R. It is bent at the middle by 180° 27. The length of a given cylindrical wire is increased by 100%.
and both the ends are twisted together to make a shorter wire. Due to the consequent decrease in diameter the change in
The resistance of the new wire is [Online May 26, 2012] the resistance of the wire will be [2003]
(a) 2 R (b) R/2 (c) R/4 (d) R/8 (a) 200% (b) 100% (c) 50% (d) 300%
21. If a wire is stretched to make it 0.1% longer, its resistance
will : [2011]
TOPIC 2 Combination of Resistances
(a) increase by 0.2% (b) decrease by 0.2%
(c) decrease by 0.05% (d) increase by 0.05%
28. In the given circuit diagram, a wire is joining points B and
DIRECTIONS : Question No. 22 and 23 are based on the
D. The current in this wire is: [9 Jan. 2020 I]
following paragraph.
Consider a block of conducting material of resistivity ‘r’ shown in
the figure. Current ‘I’ enters at ‘A’ and leaves from ‘D’. We apply
superposition principle to find voltage ‘DV’ developed between ‘B’
and ‘C’. The calculation is done in the following steps:
(i) Take current ‘I’ entering from ‘A’ and assume it to spread
over a hemispherical surface in the block.
(a) 0.4A (b) 2A (c) 4A (d) zero
(ii) Calculate field E(r) at distance ‘r’ from A by using Ohm’s
law E = r j, where j is the current per unit area at ‘r’. 29. The series combination of two batteries, both of the same
(iii) From the ‘r’ dependence of E(r), obtain the potential V(r) at r. emf 10 V, but different internal resistance of 20 W and 5
(iv) Repeat (i), (ii) and (iii) for current ‘I’ leaving ‘D’ and W, is connected to the parallel combination of two
superpose results for ‘A’ and ‘D’. resistors 30 W and R W. The voltage difference across
I the battery of internal resistance 20 W is zero, the value
I DV
of R (in W) is _________. [NA. 8 Jan. 2020 II]
30. The current I1 (in A) flowing through 1 W resistor in the
a b a following circuit is: [7 Jan. 2020 I]
A B C D

22. DV measured between B and C is [2008]


rI rI rI rI
(a) – (b) –
pa p(a + b) a (a + b)
rI rI rI (a) 0.4 (b) 0.5 (c) 0.2 (d) 0.25
(c) – (d) 31. A wire of resistance R is bent to form a square ABCD as
2pa 2p(a + b) 2p(a - b)
shown in the figure. The effective resistance between E
23. For current entering at A, the electric field at a distance ‘r’
and C is: (E is mid-point of arm CD) [9 April 2019 I]
from A is [2008]
A B
rI rI rI rI
(a) 2 (b) 2 (c) 2 (d)
8pr r 2pr 4pr 2
24. The resistance of a wire is 5 ohm at 50°C and 6 ohm at
100°C. The resistance of the wire at 0°C will be [2007]
(a) 3 ohm (b) 2 ohm (c) 1 ohm (d) 4 ohm
25. A material 'B' has twice the specific resistance of 'A'. A
circular wire made of 'B' has twice the diameter of a wire D E C
made of 'A'. then for the two wires to have the same
7 3 1
resistance, the ratio lB/lA of their respective lengths must (a) R (b) R (c) R (d) R
be [2006] 64 4 16
1 1 32. A metal wire of resistance 3 W is elongated to make a uniform
(a) 1 (b) (c) (d) 2
2 4 wire of double its previous length. This new wire is now
26. An electric current is passed through a circuit containing bent and the ends joined to make a circle. If two points on
two wires of the same material, connected in parallel. If the the circle make an angle 60° at the centre, the equivalent
4 2 resistance between these two points will be: [9 Apr. 2019 II]
lengths and radii are in the ratio of and , then the ratio
3 3 12 5 5 7
of the current passing through the wires will be [2004] (a) W (b) W (c) W (d) W
(a) 8/9 (b) 1/3 (c) 3 (d) 2 5 2 3 2
P-284 Physics

33. In the figure shown, what is the current (in Ampere) drawn
from the battery? You are given : [8 Apr. 2019 II]
R1 = 15 W, R2 = 10 W, R3 = 20 W, R4 = 5 W, R5 = 25 W, A
R6 = 30 W, E = 15 V
B
5R 5R
(a) 2 R (b) (c) (d) 3 R
2 3
38. In the given circuit diagram when the current reaches steady
state in the circuit, the charge on the capacitor of
capacitance C will be : [2017]
r2
(a) CE +
(r r2 )
r1
(b) CE
(r1 + r)
(a) 13/24 (b) 7/18 (c) 9/32 (d) 20/3
34. A uniform metallic wire has a resistance of 18 W and is (c) CE
bent into an equilateral triangle. Then, the resistance r
(d) CE 1
between any two vertices of the triangle is: (r2 + r)
[10 Jan. 2019 I]
(a) 4 W (b) 8 W (c) 12 W (d) 2 W 39.
35. The actual value of resistance R, shown in the figure is 30
W. This is measured in an experiment as shown using the
V
standard formula R = , where V and I are the reading of
I In the above circuit the current in each resistance is
the voltmeter and ammeter, respectively. If the measured
[2017]
value of R is 5% less, then the internal resistance of the
voltmeter is: [10 Jan. 2019 II] (a) 0.5A (b) 0 A (c) 1 A (d) 0.25 A
1W 1W 1W
V A1
40.
A2 A3
A B 4W ¥
R V
9V 4W 4W
0.5W
1W 1W 1W
(a) 600 W (b) 570 W (c) 35 W (d) 350 W A 9 V battery with internal resistance of 0.5 W is connected
36. In the given circuit the internal resistance of the 18 V cell across an infinite network as shown in the figure. All
is negligible. If R1 = 400W, R3 = 100 W and R4 = 500 W ammeters A1, A2, A3 and voltmeter V are ideal.
and the reading of an ideal voltmeter across R4 is 5 V, Choose correct statement. [Online April 8, 2017]
then the value of R2 will be: [9 Jan. 2019 II] (a) Reading of A1 is 2 A (b) Reading of A1 is 18 A
R3 R4 (c) Reading of V is 9 V (d) Reading of V is 7 V
41. Six equal resistances are connected between points P, Q
and R as shown in figure. Then net resistance will be
R1 maximum between : [Online April 25, 2013]
P
R2

r
18 V r
(a) 300 W (b) 450 W r
r
(c) 550 W (d) 230 W r

37. In the given circuit all resistances are of value R ohm each.
Q R
The equivalent resistance between A and B is : r
[Online April 15, 2018]
(a) P and R (b) P and Q
(c) Q and R (d) Any two points
Current Electricity P-285

42. A letter 'A' is constructed of a uniform wire with resistance


1.0 W per cm, The sides of the letter are 20 cm and the cross Kirchhoff 's Laws, Cells,
TOPIC 3
piece in the middle is 10 cm long. The apex angle is 60 . The Thermo e.m.f. & Electrolysis
resistance between the ends of the legs is close to: 5W
[Online April 9, 2013] 48.
(a) 50.0 W (b) 10 W (c) 36.7 W (d) 26.7 W
10W 10V
43. Two conductors have the same resistance at 0°C but their 20V
temperature coefficients of resistance are a1 and a2. The
respective temperature coefficients of their series and 2W 4W
parallel combinations are nearly [2010] In the figure shown, the current in the 10 V battery is
a1 + a 2 a1 + a 2 close to : [Sep. 06, 2020 (II)]
(a) , a1 + a 2 (b) a1 + a 2 ,
2 2 (a) 0.71 A from positive to negative terminal
a1a 2 a + a 2 , 1 a2
a + (b) 0.42 A from positive to negative terminal
(c) a1 + a 2 , (d) 1
a1 + a 2 2 2 (c) 0.21 A from positive to negative terminal
44. The current I drawn from the 5 volt source will be [2006] (d) 0.36 A from negative to positive terminal
10W 49. In the circuit, given in the figure currents in different branches
and value of one resistor are shown. Then potential at point
B with respect to the point A is : [Sep. 05, 2020 (II)]
5W 10W 20W
D 2V
E B
1A
I 10W 2W
+–
A F
1V C 2A
2A
5 volt
(a) + 2 V (b) – 2 V (c) – 1 V (d) + 1 V
(a) 0.33 A (b) 0.5 A (c) 0.67 A (d) 0.17 A
45. The total current supplied to the circuit by the battery is 50. The value of current i1 flowing from A to C in the circuit
[2004] diagram is : [Sep. 04, 2020 (II)]

2W
6V
6W
8V B
3W
i i
1.5W 2W 2W
5W
4W i1 4W
(a) 4 A (b) 2 A (c) 1 A (d) 6 A A C
46. The resistance of the series combination of two resistances
is S. when they are joined in parallel the total resistance is 2W 2W
P. If S = nP then the minimum possible value of n is
[2004] D
(a) 2 (b) 3 (c) 4 (d) 1 (a) 2 A (b) 4 A (c) 1 A (d) 5 A
47. A 3 volt battery with negligible internal resistance is 51. B
connected in a circuit as shown in the figure. The current 40W 60W
I, in the circuit will be [2003]
A C

3W 90W 110W
3W
3V D
40 V
Four resistances 40 W, 60 W, 90 W and 110 W make the
3W arms of a quadrilateral ABCD. Across AC is a battery of
emf 40 V and internal resistance negligible. The potential
(a) 1 A (b) 1.5 A (c) 2 A (d) 1/3 A
difference across BD in V is __________.
[NA. Sep. 04, 2020 (II)]
P-286 Physics

52. An ideal cell of emf 10 V is connected in circuit shown in 57. In the circuit shown, the potential difference between A
figure. Each resistance is 2 W. The potential difference (in and B is : [11 Jan. 2019 II]
V) across the capacitor when it is fully charged is 1V
1W
__________. [Sep. 02, 2020 (II)]
M

R1 C 5W 1W 2V
10W
R2 R5
R3 A D C B
1W 3V
R4 N

10 V (a) 1 V (b) 2 V (c) 3 V (d) 6 V


58. In the given circuit the cells have zero internal resistance.
53. In the given circuit, an ideal voltmeter connected across The currents (in Amperes) passing through resistance
the 10 W resistance reads 2V. The internal resistance r, of R1 and R2 respectively, are: [10 Jan. 2019 I]
each cell is : [10 Apr. 2019 I]

(a) 1, 2 (b) 2, 2 (c) 0.5, 0 (d) 0, 1


(a) 1 W (b) 0.5 W (c) 1.5 W (d) 0 W 59. When the switch S, in the circuit shown, is closed then
54. For the circuit shown, with R1 = 1.0 W, R2 = 2.0 W, E1 = 2 V the valued of current i will be: [9 Jan. 2019 I]
and E2 = E3 = 4 V, the potential difference between the
points ‘a’ and ‘b’ is approximately (in V) : [8 April 2019 I]

(a) 2.7 (b) 2.3 (c) 3.7 (d) 3.3


55. A cell of internal resistance r drives current through an (a) 3A (b) 5A (c) 4A (d) 2A
external resistance R. The power delivered by the cell to
60. Two batteries with e.m.f. 12 V and 13 V are connected in
the external resistance will be maximum when :
parallel across a load resistor of 10W. The internal
[8 Apr. 2019 II] resistances of the two batteries are 1W and 2W respectively.
(a) R = 0.001 r (b) R = 1000 r
The voltage across the load lies between: [2018]
(c) R = 2r (d) R = r
56. In the given circuit diagram, the currents, I1 = – 0.3 A, I4 = 0.8 (a) 11.6 V and 11.7 V (b) 11.5 V and 11.6 V
A and I5 = 0.4 A, are flowing as shown. The currents I 2, I3 (c) 11.4 V and 11.5 V (d) 11.7 V and 11.8 V
and I6, respectively, are : [12 Jan. 2019 II] 61. In the circuit shown, the current in the 1W resistor is:
P I6 Q [2015]
I3 6V P 2W

1W 9V
I2 I1
I5
S I4 R 3W W 3W
(a) 0.13 A, from Q to P (b) 0.13 A, from P to Q
(a) 1.1 A, – 0.4 A, 0.4 A (b) 1.1 A, 0.4 A, 0.4 A
(c) 1.3A from P to Q (d) 0A
(c) 0.4 A, 1.1 A, 0.4 A (d) –0.4 A, 0.4 A, 1.1 A
Current Electricity P-287

62. In the electric network shown, when no current flows E1 r = 0.5 W


through the 4W resistor in the arm EB, the potential
difference between the points A and D will be :
[Online April 11, 2015]

2W D
F E E2

2V
4W
2W R
4V R
(a) 5.5 W (b) 3.5 W
(c) 4.5 W (d) 2.5 W
A 9V B 3V C 67. A 5V battery with internal resistance 2W and a 2V battery
with internal resistance 1W are connected to a 10W resistor
(a) 6 V (b) 3 V (c) 5 V (d) 4 V as shown in the figure. [2008]
63. The circuit shown here has two batteries of 8.0 V and 16.0 P2
V and three resistors 3 W, 9 W and 9 W and a capacitor of
5.0 mF. [Online April 12, 2014]
I
5V 2V
3W 5 mF 9W 2W 10W 1W

9W
8.0 V 16.0 V

I1 I2 The current in the 10W resistor is


How much is the current I in the circuit in steady state? (a) 0.27 A P2 to P1 (b) 0.03 A P1 to P2
(a) 1.6 A (b) 0.67 A (c) 0.03 A P2 to P1 (d) 0.27 A P1 to P2
(c) 2.5 A (d) 0.25 A 68. A battery is used to charge a parallel plate capacitor till the
64. In the circuit shown, current (in A) through 50 V and 30 V potential difference between the plates becomes equal to
batteries are, respectively. [Online April 11, 2014] the electromotive force of the battery. The ratio of the
5W energy stored in the capacitor and the work done by the
battery will be [2007]
(a) 1/2 (b) 1 (c) 2 (d) 1/4
69. The Kirchhoff's first law (Si = 0) and second law (SiR = SE),
where the symbols have their usual meanings, are
50 V 20 W 10 W 30 V respectively based on [2006]
(a) conservation of charge, conservation of momentum
(b) conservation of energy, conservation of charge
5W
(c) conservation of momentum, conservation of charge
(a) 2.5 and 3 (b) 3.5 and 2 (d) conservation of charge, conservatrion of energy
70. A thermocouple is made from two metals, Antimony and
(c) 4.5 and 1 (d) 3 and 2.5
Bismuth. If one junction of the couple is kept hot and the
65. A d.c. main supply of e.m.f. 220 V is connected across a other is kept cold, then, an electric current will [2006]
storage battery of e.m.f. 200 V through a resistance of 1W. (a) flow from Antimony to Bismuth at the hot junction
The battery terminals are connected to an external resistance
(b) flow from Bismuth to Antimony at the cold junction
‘R’. The minimum value of ‘R’, so that a current passes
through the battery to charge it is: [Online April 9, 2014] (c) now flow through the thermocouple
(a) 7 W (b) 9 W (c) 11 W (d) Zero (d) flow from Antimony to Bismuth at the cold junction
66. A dc source of emf E1 = 100 V and internal resistance r = 0.5 71. Two sources of equal emf are connected to an external
W, a storage battery of emf E2 = 90 V and an external resistance R. The internal resistance of the two sources are
resistance R are connected as shown in figure. For what R1and R2 (R1 > R1). If the potential difference across the
value of R no current will pass through the battery ? source having internal resistance R2 is zero, then
[Online April 22, 2013] [2005]
P-288 Physics

(a) R = R2 - R1
(b) R = R2 ´ ( R1 + R2 ) /( R2 - R1 ) TOPIC 4 Heating Effect of Current
(c) R = R1R2 /( R2 - R1 )
79. An electrical power line, having a total resistance of 2 W,
(d) R = R1R2 /( R1 - R2 ) delivers 1 kW at 220 V. The efficiency of the transmission
72. Two voltameters, one of copper and another of silver, are line is approximately : [Sep. 05, 2020 (I)]
joined in parallel. When a total charge q flows through the (a) 72% (b) 91% (c) 85% (d) 96%
voltameters, equal amount of metals are deposited. If the 80. Model a torch battery of length l to be made up of a thin
electrochemical equivalents of copper and silver are Z1 cylindrical bar of radius ‘a’ and a concentric thin cylindrical
and Z2 respectively the charge which flows through the shell of radius ‘b’ filled in between with an electrolyte of resistivity
silver voltameter is [2005] r (see figure). If the battery is connected to a resistance of
q q Z2 Z value R, the maximum Joule heating in R will take place for :
(a) `(b) (c) q (d) q 1
Z Z Z1 Z2
1+ 2 1+ 1 [Sep. 03, 2020 (I)]
Z1 Z2
73. An energy source will supply a constant current into the
load if its internal resistance is [2005]
l r
(a) very large as compared to the load resistance
(b) equal to the resistance of the load
(c) non-zero but less than the resistance of the load
(d) zero a
74. The thermo emf of a thermocouple varies with the b
temperature q of the hot junction as E = aq + bq2 in volts r æbö r æbö
where the ratio a/b is 700°C. If the cold junction is kept at (a) R= ç ÷ (b) R =
ln ç ÷
2pl è a ø 2pl è a ø
0°C, then the neutral temperature is [2004]
r 2r æ b ö
(c) R = l n æç ö÷
(a) 1400°C b
(d) R = ln ç ÷
(b) 350°C pl è a ø pl è a ø
(c) 700°C 81. In a building there are 15 bulbs of 45 W, 15 bulbs of 100 W,
(d) No neutral temperature is possible for this termocouple. 15 small fans of 10 W and 2 heaters of 1 kW. The voltage
75. The electrochemical equivalent of a metal is 3.35 × 10–7 kg of electric main is 220 V. The minimum fuse capacity (rated
per Coulomb. The mass of the metal liberated at the cathode value) of the building will be: [7 Jan. 2020 II]
when a 3A current is passed for 2 seconds will be (a) 10 A (b) 25 A (c) 15 A (d) 20 A
[2004] 82. The resistive network shown below is connected to a D.C.
(a) 6.6×1057kg (b) 9.9×10–7 kg source of 16 V. The power consumed by the network is 4
(c) 19.8×10–7 kg (d) 1.1×10–7 kg Watt. The value of R is : [12 Apr. 2019 I]
76. The thermo e.m.f. of a thermo-couple is 25 mV/°C at room
temperature. A galvanometer of 40 ohm resistance, capable
of detecting current as low as 10–5 A, is connected with
the thermo couple. The smallest temperature difference that
can be detected by this system is [2003]
(a) 16°C (b) 12°C (c) 8°C (d) 20°C
(a) 6W (b) 8W (c) 1W (d) 16W
77. The negative Zn pole of a Daniell cell, sending a constant o
current through a circuit, decreases in mass by 0.13g in 30 83. One kg of water, at 20 C, is heated in an electric kettle
minutes. If the electeochemical equivalent of Zn and Cu whose heating element has a mean (temperature averaged)
are 32.5 and 31.5 respectively, the increase in the mass of resistance of 20 W. The rms voltage in the mains is 200 V.
the positive Cu pole in this time is [2003] Ignoring heat loss from the kettle, time taken for water to
evaporate fully, is close to :
(a) 0.180 g (b) 0.141g (c) 0.126 g (d) 0.242 g
[Specific heat of water = 4200 J/(kgoC), Latent heat of water
78. The mass of product liberated on anode in an
= 2260 kJ/kg] [12 Apr. 2019 II]
electrochemical cell depends on [2002]
(a) (It)1/2 (b) It (c) I/t (d) I2 t (a) 16 minutes (b) 22 minutes
(where t is the time period for which the current is passed). (c) 3 minutes (d) 3 minutes
Current Electricity P-289
R
84. Two electric bulbs, rated at (25 W, 220 V) and (100 W, 220 V), 91.
R
are connected in series across a 220 V voltage source. If
the 25 W and 100 W bulbs draw powers P1 and P2
respectively, then: [12 Jan. 2019 I]
(a) P1=16 W, P2=4 W (b) P1=16 W, P2=9 W r
(c) P1=9 W, P2=16 W (d) P1=4 W, P2=16 W
85. Two equal resistances when connected in series to a battery,
consume electric power of 60 W. If these resistance are now
connected in parallel combination to the same battery, the In the circuit shown, the resistance r is a variable resistance.
electric power consumed will be : [11 Jan. 2019 I] If for r = fR, the heat generation in r is maximum then the
(a) 60 W (b) 240 W (c) 120 W (d) 30 W value of f is : [Online April 9, 2016]
86. A 2 W carbon resistor is color coded with green, black,
1 1 3
red and brown respectively. The maximum current which (a) (b) 1 (c) (d)
can be passed through this resistor is: [10 Jan. 2019 I] 2 4 4
(a) 20 mA (b) 100 mA (c) 0.4 mA (d) 63 mA 92. In a large building, there are 15 bulbs of 40 W, 5 bulbs of
100 W, 5 fans of 80 W and 1 heater of 1 kW. The voltage
87. A current of 2 mA was passed through an unknown resistor
of electric mains is 220 V. The minimum capacity of the
which dissipated a power of 4.4 W. Dissipated power when
main fuse of the building will be: [2014]
an ideal power supply of 11 V is connected across it is:
(a) 8 A (b) 10 A (c) 12 A (d) 14 A
[10 Jan. 2019 II] 93. Four bulbs B1, B2, B3 and B4 of 100 W each are connected
(a) 11 × 10–5 W (b) 11 × 10–3 W to 220 V main as shown in the figure.
–4
(c) 11 × 10 W (d) 11 × 105 W [Online April 19, 2014]
88. A constant voltage is applied between two ends of a metallic
wire. If the length is halved and the radius of the wire is
220 V B1 B2 B3 B4
doubled, the rate of heat developed in the wire will be:
[Online April 15, 2018]
(a) Increased 8 times (b) Doubled
Ammeter
(c) Halved (d) Unchanged The reading in an ideal ammeter will be:
89. The figure shows three circuits I, II and III which are (a) 0.45 A (b) 0.90 A (c) 1.35 A (d) 1.80 A
connected to a 3V battery. If the powers dissipated by the 94. The supply voltage to room is 120V. The resistance of the
configurations I, II and III are P1, P2 and P3 respectively, lead wires is 6W. A 60 W bulb is already switched on.
then : [Online April 9, 2017] What is the decrease of voltage across the bulb, when a
240 W heater is switched on in parallel to the bulb?[2013]
1W 1W (a) zero (b) 2.9 Volt
1W
(c) 13.3 Volt (d) 10.04Volt
1W 1W
1W
1W
95. Which of the four resistances P, Q, R and S generate the
3V 1W 1W greatest amount of heat when a current flows from A to
3V 1W
1W
1W 1W 3V
B? [Online April 23, 2013]
1W 1W 1W
P = 2W Q = 4W
(II) (III)
(I)
(a) P1 > P2 > P3 (b) P1 > P3 > P2
(c) P2 > P1 > P3 (d) P3 > P2 > P1
90. The resistance of an electrical toaster has a temperature A B
dependence given by R(T) = R0 [1 + a(T – T0)] in its range
of operation. At T0 = 300K, R = 100 W and at T = 500 K, R = R = 1W S = 2W
120 W. The toaster is connected to a voltage source at 200 (a) Q (b) S (c) P (d) R
V and its temperature is raised at a constant rate from 300 96. Two electric bulbs rated 25W – 220 V and 100W – 220V are
to 500 K in 30 s. The total work done in raising the connected in series to a 440 V supply. Which of the bulbs
temperature is : [Online April 10, 2016] will fuse? [2012]
(a) Both (b) 100 W (c) 25 W (d) Neither
5 2
(a) 400ln J (b) 200ln J 97. A 6.0 volt battery is connected to two light bulbs as shown in
6 3 figure. Light bulb 1 has resistance 3 ohm while light bulb 2
1.5 has resistance 6 ohm. Battery has negligible internal
(c) 300 J (d) 400 ln J
resistance. Which bulb will glow brighter?
1.3
[Online May 19, 2012]
P-290 Physics
Bulb 1 (c) metals with low temperature coefficient of resistivity
(d) semiconducting materials having low temperature
coefficient of resistivity
Bulb 2 105. Time taken by a 836 W heater to heat one litre of water
from 10°C to 40°C is [2004]
(a) 150 s (b) 100 s (c) 50 s (d) 200 s
106. A 220 volt, 1000 watt bulb is connected across a 110 volt
+ – mains supply. The power consumed will be [2003]
(a) 750 watt (b) 500 watt
6.0 V (c) 250 watt (d) 1000 watt
(a) Bulb 1 will glow more first and then its brightness will
become less than bulb 2 107. A wire when connected to 220 V mains supply has power
(b) Bulb 1 dissipation P1. Now the wire is cut into two equal pieces
(c) Bulb 2 which are connected in parallel to the same supply. Power
(d) Both glow equally dissipation in this case is P2. Then P2 : P1 is [2002]
98. Three resistors of 4 W, 6 W and 12 W are connected in (a) 1 (b) 4 (c) 2 (d) 3
parallel and the combination is connected in series with a 108. If in the circuit, power dissipation is 150 W, then R is
1.5 V battery of 1 W internal resistance. The rate of Joule [2002]
heating in the 4 W resistor is [Online May 12, 2012] R
(a) 0.55 W (b) 0.33 W (c) 0.25 W (d) 0.86 W
99. This question has Statement 1 and Statement 2. Of the 2W
four choices given after the Statements, choose the one 15 V
that best describes the two Statements.
(a) 2W (b) 6W (c) 5W (d) 4W
Statement 1: The possibility of an electric bulb fusing is
higher at the time of switching ON.
Statement 2: Resistance of an electric bulb when it is not lit Wheatstone Bridge and
up is much smaller than when it is lit up. TOPIC 5 Different Measuring
[Online May 7, 2012] Instruments
(a) Statement 1 is true, Statement 2 is false
(b) Statement 1 is false, Statement 2 is true, Statement 109. Two resistors 400W and 800W are connected in series across
2 is not a correct explanation of Statement 1. a 6 V battery. The potential difference measured by a
(c) Statement 1 is true, Statement 2 is true, Statement 2 voltmeter of 10 kW across 400W resistor is close to:
is a correct explanation of Statement 1. [Sep. 03, 2020 (II)]
(d) Statement 1 is false, Statement 2 is true. (a) 2 V (b) 1.8 V (c) 2.05 V (d) 1.95 V
100. The resistance of a bulb filmanet is 100W at a temperature 110. Which of the following will NOT be observed when a
of 100°C. If its temperature coefficient of resistance be multimeter (operating in resistance measuring mode)
0.005 per °C, its resistance will become 200 W at a probes connected across a component, are just reversed?
temperature of [2006] [Sep. 03, 2020 (II)]
(a) 300°C (b) 400°C (c) 500°C (d) 200°C (a) Multimeter shows an equal deflection in both cases
101. An electric bulb is rated 220 volt - 100 watt. The power i.e. before and after reversing the probes if the chosen
consumed by it when operated on 110 volt will be [2006] component is resistor.
(a) 75 watt (b) 40 watt (c) 25 watt (d) 50 watt (b) Multimeter shows NO deflection in both cases i.e.
102. A heater coil is cut into two equal parts and only one part before and after reversing the probes if the chosen
is now used in the heater. The heat generated will now component is capacitor.
be [2005] (c) Multimeter shows a deflection, accompanied by a
(a) four times (b) doubled splash of light out of connected and NO deflection
(c) halved (d) one fourth on reversing the probes if the chosen component is
103. The resistance of hot tungsten filament is about 10 times LED.
the cold resistance. What will be the resistance of 100 W (d) Multimeter shows NO deflection in both cases i.e.
and 200 V lamp when not in use ? [2005] before and after reversing the probes if the chosen
(a) 20 W (b) 40 W (c) 200 W (d) 400W component is metal wire.
104. The thermistors are usually made of [2004] 111. A potentiometer wire PQ of 1 m length is connected to a
(a) metal oxides with high temperature coefficient of standard cell E 1 . Another cell E 2 of emf 1.02 V is
resistivity connected with a resistance 'r' and switch S (as shown in
(b) metals with high temperature coefficient of figure). With switch S open, the null position is obtained
resistivity at a distance of 49 cm from Q. The potential gradient in
the potentiometer wire is : [Sep. 02, 2020 (II)]
Current Electricity P-291

E1 Sl.No. RW l (cm)
1. 1000 60
P J Q 2. 100 13
r 3. 10 1.5
G
4. 1 1.0
E2 S
(a) 0.02 V/cm (b) 0.01 V/cm Which of the reading is consistent ?
(a) 3 (b) 2 (c) 4 (d) 1
(c) 0.03 V/cm (d) 0.04 V/cm
117. In the circuit shown, a four-wire potentiometer is made of
112. In a meter bridge experiment S is a standard resistance. R a 400 cm long wire, which extends between A and B. The
is a resistance wire. It is found that balancing length is resistance per unit length of the potentiometer wire is
l = 25 cm. If R is replaced by a wire of half length and half r = 0.01 W/cm. If an ideal voltmeter is connected as shown
diameter that of R of same material, then the balancing with jockey J at 50 cm from end A, the expected reading of
distance l¢ (in cm) will now be ____. [NA. 9 Jan. 2020 II] the voltmeter will be : [8 Apr. 2019 II]

113. The length of a potentiometer wire is 1200 cm and it carries


a current of 60 mA. For a cell of emf 5 V and internal (a) 0.50 V (b) 0.75 V (c) 0.25 V (d) 0.20 V
resistance of 20 W, the null point on it is found to be at 118. In a meter bridge, the wire of length 1 m has a non-uniform
1000 cm. The resistance of whole wire is: dR
cross-section such that, the variation of its resistance
[8 Jan. 2020 I] dl
(a) 80 W (b) 120 W (c) 60 W (d) 100 W dR 1
R with length l is µ . Two equal resistances are
114. Four resistances of 15 W, 12 W, 4 W and 10 W respectively dl l
connected as shown in the figure. The galvanometer has
in cyclic order to form Wheatstone’s network. The zero deflection when the jockey is at point P. What is the
resistance that is to be connected in parallel with the length AP? [12 Jan. 2019 I]
resistance of 10 W to balance the network is _____ W.
[NA. 8 Jan. 2020 I] R' R'
115. The balancing length for a cell is 560 cm in a potentiometer
experiment. When an external resistance of 10 W is
connected in parallel to the cell, the balancing length
G
N
changes by 60 cm. If the internal resistance of the cell is P
10
W, where N is an integer then value of N is ________. l 1 l
[NA. 7 Jan. 2020 II] (a) 0.2 m (b) 0.3 m (c) 0.25 m (d) 0.35 m
116. In a meter bridge experiment, the circuit diagram and the 119. An ideal battery of 4 V and resistance R are connected in
corresponding observation table are shown in figure. series in the primary circuit of a potentionmeter of length
[10 Apr. 2019 I] 1 m and resistance 5W. The value of R, to give a potential
difference of 5 mV across 10 cm of potentiometer wire is:
[12 Jan. 2019 I]
(a) 490W (b) 480W (c) 395W (d) 495W
120. The resistance of the meter bridge AB in given figure is 4W.
With a cell of emf e= 0.5 V and rheostat resistance Rh = 2W
the null point is obtained at some point J. When the cell is
replaced by another one of emf e =e2 the same null point
J is found for Rh = 6W. The emf e2 is: [11 Jan. 2019 I]
P-292 Physics
e

A B
J

6V Rh 11 11 13 5
(a) L (b) L (c) L (d) L
(a) 0.4 V (b) 0.3 V (c) 0.6 V (d) 0.5 V 12 24 24 12
121. In a Wheatstone bridge (see fig.), Resistances P and Q are 124. The Wheatstone bridge shown in Fig. here, gets balanced
approximately equal. When R = 400 W, the bridge is balanced. when the carbon resistor used as R1 has the colour code
On interchanging P and Q, the value of R, for balance, is (Orange, Red, Brown). The resistors R2 and R4 are 80 W
405W. The value of X is close to : [11 Jan. 2019 I] and 40 W, respectively.
B Assuming that the colour code for the carbon resistors
gives their accurate values, the colour code for the carbon
P Q resistor, used as R3, would be: [10 Jan. 2019 II]
G
A K2 C R1 R2

R X G

D R3 R4

K1 + –
(a) 401.5 ohm (b) 404.5 ohm
(c) 403.5 ohm (d) 402.5 ohm (a) Brown, Blue, Brown (b) Brown, Blue, Black
122. In the experimental set up of metre bridge shown in the
(c) Red, Green, Brown (d) Grey, Black, Brown
figure, the null point is obtaine data distance of 40 cm from
A. If a 10 W resistor is connected in series with R1, the null 125. In a potentiometer experiment, it is found that no current
point shifts by 10 cm. The resistance that should be passes through the galvanometer when the terminals of
connected in parallel with (R1 + 10) W such that the null the cell are connected across 52 cm of the potentiometer
point shifts back to its initial position is : wire. If the cell is shunted by a resistance of 5 W, a balance
[11 Jan. 2019 II] is found when the cell is connected across 40 cm of the
wire. Find the internal resistance of the cell. [2018]
R1 R2
(a) 1 W (b) 1.5 W (c) 2 W (d) 2.5 W
126. On interchanging the resistances, the balance point of a
G meter bridge shifts to the left by 10 cm. The resistance of
their series combination is 1kW. How much was the resistance
A B on the left slot before interchanging the resistances?
[2018]
(a) 990 W (b) 505 W (c) 550 W (d) 910 W
127. In a meter bridge, as shown in the figure, it is given that
(a) 20 W (b) 40 W (c) 60 W (d) 30 W resistance Y=12.5 W and that the balance is obtained at a
123. A potentiometer wire AB having length L and resistance distance 39.5 cm from end A (by jockey J). After
12 r is joined to a cell D of emf e and internal resistance interchanging the resistances X and Y, a new balance point
r. A cell C having emf e/2 and internal resistance 3r is is found at a distance l2 from end A. What are the values of
connected. The length AJ at which the galvanometer as X and l2 ? [Online April 15, 2018]
shown in fig. shows no deflection is: [10 Jan. 2019 I]
Current Electricity P-293

X Y 131. A 10V battery with internal resistance 1W and a 15V battery


B
with internal resistance 0.6 W are connected in parallel to
G a voltmeter (see figure). The reading in the voltmeter will
A
l1 (100 – l1)
C be close to : [Online April 10, 2015]
39.5 J wire
10V
METER SCALE
1W
15V
Battery Key
(a) 19.15 W and 39.5 cm (b) 8.16 W and 60.5 cm 0.6W
(c) 19.15 W and 60.5 cm (d) 8.16 W and 39.5 cm
128. Which of the following statements is false ? [2017]
V
(a) A rheostat can be used as a potential divider
(b) Kirchhoff's second law represents energy conservation (a) 12.5 V (b) 24.5 V (c) 13.1 V (d) 11.9 V
(c) Wheatstone bridge is the most sensitive when all the 132. In an experiment of potentiometer for measuring the internal
four resistances are of the same order of magnitude resistance of primary cell a balancing length l is obtained on
(d) In a balanced wheatstone bridge if the cell and the the potentiometer wire when the cell is open circuit. Now the
galvanometer are exchanged, the null point is disturbed. cell is short circuited by a resistance R. If R is to be equal to
129. In a meter bridge experiment resistances are connected as the internal resistance of the cell the balancing length on the
shown in the figure. Initially resistance P = 4 W and the potentiometer wire will be [Online May 26, 2012]
neutral point N is at 60 cm from A. Now an unknown resis- (a) l (b) 2l (c) l/2 (d) l/4
tance R is connected in series to P and the new position of 133. It is preferable to measure the e.m.f. of a cell by
the neutral point is at 80 cm from A. The value of unknown potentiometer than by a voltmeter because of the following
resistance R is : [Online April 9, 2017] possible reasons. [Online May 12, 2012]
P Q (i) In case of potentiometer, no current flows through
the cell.
G (ii) The length of the potentiometer allows greater precision.
(iii) Measurement by the potentiometer is quicker.
(iv) The sensitivity of the galvanometer, when using a
A B
N potentiometer is not relevant.
Which of these reasons are correct?
(a) (i), (iii), (iv) (b) (i), (iii), (iv)
( ) (c) (i), (ii) (d) (i), (ii), (iii), (iv)
E Rh K
134. In a sensitive meter bridge apparatus the bridge wire should
33 20 possess [Online May 12, 2012]
(a) W (b) 6 W (c) 7 W (d) W
5 3 (a) high resistivity and low temperature coefficient.
130. A potentiometer PQ is set up to compare two resistances as (b) low resistivity and high temperature coefficient.
shown in the figure. The ammeter A in the circuit reads 1.0 A (c) low resistivity and low temperature coefficient.
when two way key K3 is open. The balance point is at a length (d) high resistivity and high temperature coefficient.
l1 cm from P when two way key K3 is plugged in between 2 135. In a metre bridge experiment null point is obtained at 40 cm
and 1, while the balance point is at a length l2 cm from P when from one end of the wire when resistance X is balanced
key K3 is plugged in between 3 and 1. The ratio of two against another resistance Y. If X < Y, then the new position
of the null point from the same end, if one decides to
resistances R1 , is found to be : [Online April 8, 2017] balance a resistance of 3X against Y, will be close to :
R2
[Online April 9, 2013]
(a) 80 cm (b) 75 cm (c) 67 cm (d) 50 cm
136. The current in the primary circuit of a potentiometer is 0.2
A. The specific resistance and cross-section of the
potentiometer wire are 4 × 10–7 ohm metre and 8 × 10–7 m2,
respectively. The potential gradient will be equal to
[2011 RS]
(a) 1 V /m (b) 0.5 V/m (c) 0.1 V/m (d) 0.2 V/m
137. Shown in the figure below is a meter-bridge set up with null
deflection in the galvanometer.
l1 l2 l1 l1
(a) (b) (c) (d)
l1 + l2 l2 - l1 l1 - l2 l2 - l1
P-294 Physics

55W R 140. In a meter bridge experiment null point is obtained at 20 cm.


from one end of the wire when resistance X is balanced
against another resistance Y. If X < Y, then where will be the
G new position of the null point from the same end, if one
decides to balance a resistance of 4 X against Y [2004]
20 cm
(a) 40 cm (b) 80 cm (c) 50 cm (d) 70 cm
141. The length of a wire of a potentiometer is 100 cm, and the
e. m.f. of its standard cell is E volt. It is employed to measure
the e.m.f. of a battery whose internal resistance is 0.5W. If
the balance point is obtained at l = 30 cm from the positive
The value of the unknown resistor R is [2008]
end, the e.m.f. of the battery is [2003]
(a) 13.75 W (b) 220 W (c) 110 W (d) 55 W
138. In a Wheatstone's bridge, three resistances P, Q and R 30 E 30 E
(a) (b)
connected in the three arms and the fourth arm is formed 100.5 (100 - 0.5)
by two resistances S1 and S2 connected in parallel. The
condition for the bridge to be balanced will be [2006] 30 ( E - 0.5i ) 30 E
(c) (d)
P 2R P R ( S1 + S2 ) 100 100
(a) = (b) = where i is the current in the potentiometer wire.
Q S1 + S2 Q S1 S2
142. An ammeter reads upto 1 ampere. Its internal resistance is
P R ( S1 + S2 ) P R 0.81ohm. To increase the range to 10 A the value of the
(c) = (d) = required shunt is [2003]
Q 2S1S2 Q S1 + S2
(a) 0.03 W (b) 0.3 W (c) 0.9 W (d) 0.09 W
139. In a potentiometer experiment the balancing with a cell is at
143. If an ammeter is to be used in place of a voltmeter, then we
length 240 cm. On shunting the cell with a resistance of 2W,
must connect with the ammeter a [2002]
the balancing length becomes 120 cm. The internal
(a) low resistance in parallel
resistance of the cell is [2005]
(b) high resistance in parallel
(a) 0.5W (b) 1W (c) 2W (d) 4W (c) high resistance in series
(d) low resistance in series.
Current Electricity P-295

1. (c) Ammeter : In series connection, the same current b


r é -1 ù
flows through all the components. It aims at measuring ÞR=
the current flowing through the circuit and hence, it is 4p êë x úû a
connected in series. æ r ö æ1 1ö
Voltmeter : A voltmeter measures voltage change between R = ç ÷ .ç - ÷
è 4p ø è a b ø
two points in a circuit. So we have to place the voltmeter in
parallel with the cicuit component. m
7. (c) r=
2. (b) rM = 98 ´ 10-8 ne 2 t
9.1 ´ 10-31
r A = 2.65 ´ 10-8 =
8.5 ´1028 ´ (1.6 ´10 -19 ) 2 ´ 25 ´ 10 -15
rC = 1.724 ´ 10-8
= 10–8 W-m
rT = 5.65 ´ 10-8 8. (d) Number 2 is associated with the red colour. This
\rM > rT > r A > rC colour is replaced by green.
Q Colour code figure for green is 5
3. (a) When i = 0, V = e = 1.5 volt
4. (d) Charge mobility \ New resistance = 500 W
9. (a) Clearly, from graph
Vd
( m) = [ Where Vd = drift velocity ] Current, I =
dq
= 0at t = 4s [Since q is constant]
E dt
E EA I(r) 10. (b) Color code : Bl, Br, R, O, Y, G, B, V, Gr, W
and resistivity ( r ) = = Þ E=
0, 1, 2, 3, 4, 5, 6, 7, 8 9
j I A
Vd Vd A R = AB × C ± D% where D = tolerance
Þ m= = Dgold = ±5%, Dsilver = ±10%; Dno colour = ±20%
E Ir
Red voilet orange silver

( ) R = 27 × 103 W ± 10% = 27 kW ± 10%


2
1.1 ´10 –3 ´ p ´ 5 ´10 –3
11. (a) Using, I = neAvd
=
5 ´ 1.7 ´10 –8 1
\Drift speed v d =
m 2 neA
m = 1.0 1.5
Vs = 0.02 mms–1
9 ´ 10 ´1.6 ´ 10-19 ´ 5 ´ 10-6
28

5. (b) Equation of straight line from graph rl


12. (c) Resistance, R =
y = – mx + c A
2
æ 1 ö l l rl
Þ l nR = - m ç ÷+c R=r ´ = [?Volume (V) = A?.]
è T2 ø A l V
Since resistivity and volume remains constant therefore
here, m & c are constants
% change in resistance
é æ 1 ö ù æ 1 ö
ê- mç 2 ÷+ cú - mç DR 2Dl
2÷ = = 2 ´ (0.5) = 1%
R = eë è T ø û = e è T ø ´ ec R l
13. (a) Colour code for carbon resistor
-T02
Bl, Br, R, O, Y, G, Blue, V, Gr, W
R(T) = R 0e T2
0 1 2 3 4 5 6 7 8 9
Resistance, R = AB × C ± D
(r)(dx) Bands A and B are the first two significant figures of
6. (a) dR =
4px 2 resistance
B and C indicates the decimal multiplier or the number of
R = ò dR
zeros that follow A and B
b B and D is tolerance: Gold = ± 5%,
dx
ò dR = rò 4 px 2
Silver = ± 10 % No colour = ± 20%
a
R = 53 ´ 104 ± 5% = 530kW ± 5%
P-296 Physics

14. (c) Resistance after temperature increases by 500°C i.e., l


V 220 20. (c) Resistance of wire (R) = r
Rt = = = 110W A
I 2 If wire is bent in the middle then
R0 = 100 (given) temperature coefficient of resistance, a = ? l
using Rt = R0 (1 + at) l¢ = , A¢ = 2 A
110 = 100 (1 + a500) 2

10 \ New resistance, R¢ = r
a= A¢
100 ´ 500 l
r
or, a = 2 ´ 10 -4°C -1 2 rl R
= = = .
charge q 2A 4A 4
15. (c) Charge density r = = Þ q = rAd 21. (a) Resistance of wire
volume Ad
q rAd rl rl 2
Also, q = IT Þ T = = R= = (Q V = Al)
I I A V
16. (a) Given, R1 = 100 W, r' = r/2, R2 = ?
l2
rl Hence, R = r = constant × l2
Resistivity of wire, R = Q Area × length = volume V
A \ Fractional change in resistance
rV
Hence, R = 2 DR Dl
A =2
R l
Since, r ® constant, V ® constant
1 100 ´
DR æ dl ö
Rµ 2 = 200 ´ ç ÷
A R è l ø
1 Q dl/l = 0.1%
or R µ Q A = pr2

R2
r4
= 16 Þ R2 = 16 ´ 100 = 1600 W, Resistance of new wire.
é
\ % change in R = ê 200 ´
ë
0.1 ù
100 úû { }
= 0.2%

R1 \ Resistance will increase by 0.2%.


22. (a) Let j be the current density.
l
17. (b) V = IR = (neAvd )r I
A Then j ´ 2pr 2 = I Þ j =
V 2 pr 2
\ r = V lne rI
d \ E = rj =
Here V = potential difference 2 pr 2
l = length of wire Now, VB – VC
n = no. of electrons per unit volume of conductor.
e = no. of electrons
a r uur a
rI
Placing the value of above parameters we get resistivity
=- ò E × dr = - ò 2 pr 2
dr
a +b a+b
5 a
r= rI é 1 ù rI rI
8 ´ 1028 ´ 1.6 ´ 10-19 ´ 2.5 ´ 10-4 ´ 0.1 =- ê - ú = -
2 p ë r û a + b 2 pa 2 p ( a + b )
= 1.6 × 10–5Wm
On applying superposition as mentioned we get
18. (c) i = neAVd and Vd µ E (Given) rI rI
'
or, i µ E DVBC = 2 ´ DVBC = -
pa p ( a + b )
i2 µ E rI
i2 µ V 23. (c) As shown in Answer (a) E =
Hence graph (c) correctly dipicts the V-I graph for a wire 2pr 2
made of such type of material. 24. (d) Resistance of a metal conductor at temperature t°C
19. (a) In ohm's law, we check V = IR where I is the corrent is given by
flowing through a resistor and V is the potential difference Rt = R0 (1 + at),
across that resistor. Only option (a) fits the above criteria. R0 is the resistance of the wire at 0ºC
Remember that ammeter is connected in series with and a is the temperature coefficient of resistance.
resistance and voltmeter parallel with the resistance. Resistance at 50°C, R50 = R0 (1 + 50a) .. (i)
Resistance at 100°C, R100 = R0 (1 + 100a) ... (ii)
From (i), R50 – R0 = 50aR0 ... (iii)
From (ii), R100 – R0 = 100aR0 ... (iv)
Current Electricity P-297

Dividing (iii) by (iv), we get 28. (b) From circuit diagram,


R50 - R0 1 1 1 1 4
= = + Þ R1 =
R100 - R0 2 R1 1 4 5
Here, R50 = 5W and R100 = 6W 1 1 1 6
5 - R0 1 = + Þ R2 =
\ = R2 2 3 5
6 - R0 2
or, 6 – R0 = 10 – 2 R0 or, R0 = 4W.
25. (d) Let dA and dB are the diameter of wire A and B respectively.
Let rB and rA be the resistivity of wire A and B. We have given
rB = 2rA
dB = 2dA
If both resistances are equal
RB = RA
r B l B r Al A
Þ =
AB AA 4 6
Reff = R1 + R2 = + = 2W
lB rA d B2 rA 4d A2 5 5
\ = ´ = ´ =2
l A rB d A2 2r A d A2 v 20
i= = = 10 A
R1 i1 Reff 2
4i 3i i
26. (b) R2 i2 \ I BC = - = = 2A
5 5 5
29. (30.00)
V
Given,
l1 4 r 2
= and 1 =
l2 3 r2 3
rl rl
R1 = 1 ; R2 = 2
2
pr1 pr22
When wires are in parallel to the circuit potential difference The resistance of 30W is in parallel with R. Their effective
across each wire is same resistance
i1R1 = i2R2
1 1 1
i1 R2 rl pr 2 l 2 r12 = +
\ = = 22 ´ 1 = ´ R ' 30 R
i2 R1 pr2 rr1 l1 r2 2 30 R
R' = ...(i)
3 4 1 30 + R
= ´ =
4 9 3 20 ´ 20
27. (d) Since volume of wire remains unchanged on Also, V = IR Þ 10 =
R '+ 25
increasing length, hence
A × l = = A¢ × l¢ Þ R '+ 25 = 40 Þ R’ = 15
Þ l¢ = 2l 30 R
R ' = 15 = Using (i)
A´ l A´ l A 30 + R
\ A¢ = = =
l¢ 2l 2 Þ 30 + R = 2R
Percentage change in resistance Þ R = 30 W
l¢ l 30. (c)
R f - Ri r -b
= ´100 = A¢ A ´ 100 æ 7R ö æ R ö
Ri l ç ÷ç ÷ 7R
r 8 øè 8 ø
A 31. (b) R eq =è =
R 64
éæ l¢ A ö ù
= êç ´ ÷ - 1ú ´ 100
ëè A¢ l ø û
é æ 2l A ö ù
= ê ç ´ ÷ - 1ú ´ 100 = (4 – 1) × 100
çA ÷
ëê è 2 l ø ûú
= 300%
P-298 Physics

32. (c) When length becomes double its resistance becomes V4


Current, i4= = 0.01A
(R µ l 2 ) R4
R = 4 × 3 = 12 W V3 = i1 R3 = 1V
2 ´ 10 5 V3 + V4 = 6V = V2
Req = = W V1 + V3 + V4 = 18V
12 3
Þ V1 =12 V
33. (c) R3, R4 and R5 are in series so their equivalent V1
R = 20 + 5 + 25 = 50 W i= = 0.03A
R1
This is parallel with R2, and so net resistance of the circuit
i = i1 + i2 Þ i 2 = i – i, = 0.03 – 0.01A = 0.02 A
V2 6
\R 2 = = = 300W
i 2 0.02
37. (a) Rseries = R1 + R2 + ..... + Rn
1 1 1 1
= + + ....... +
R parallel R1 R 2 Rn
R
æ 10 ´ 50 ö 160 R
Req =ç ÷ + 15 + 30 = W R
R
R
è 10 + 50 ø 3 A
C
A C
R C R R
e 15 9
So, i = = = A R R R Req = 2R
Req (100 / 3) 32 R

B B
34. (a) R C C R R C
38. (a) In steady state, flow fo current through capacitor will
be zero.
Current through the circuit,
E
i=
r + r2
Potential difference through capacitor
Q æ E ö
Resistance, R µ l so resistance of each side of the Vc = = E - ir = E - ç r
equilateral triangle = 6 W
C è r + r2 ÷ø
Resistance Req between any two vertices r2
\ Q = CE
1 1 1 r + r2
= + Þ R eq. = 4 W
R eq 12 6 39. (b) The potential difference in each loop is zero.
1 1 1 \ No current will flow or current in each resistance is Zero.
35. (b) Using, R = R + R 40. (a) The given circuit can be redrawn as,
eq 1 2
1W
RRu A
0.95 R = R + Ru (measured value 5% less then internal
xW 4W xW
resistance of voltmeter)
or, 0.95 × 30 = 0.05 Ru B
1W
\ Ru = 19 × 30 = 570 W 1 1 (4 + x)
as 4 W and x W are parallel x' = + =
4 x 4x
36. (a) R3= 100W R4= 500W 4x
x' =
4+x
i i1 & 1 W and 1 W are also parallel x" = 2 W
R1= 400W Now equivalent resistance of circuit
4x 8 + 6x
i2 R2 x= +2 =
4+ x 4+x
18V 4x + x2 = 8 + 6x
x2 – 2x – 8 = 0
Across R4 reading of voltmeter, V4 = 5V
Current Electricity P-299

Here, R0 is the resistance of conductor at 0°C


2 ± 4 - 4(1)(-8) 2 ± 36 2±6
x= = = = 4W In Series, R = R1 + R2
2 2 2
Reading of Ammeter A1 =
V = R0 [2 + ( a1 + a2 ) D t ]
(R + r)
é æ a + a2 ö ù
9 = 2 R0 ê1 + ç 1 ÷ D tú
A1 = = 2 Ampere ë è 2 ø û
4 + 0.5 a + a2
41. (b) Resistance between P and Q \ a eq = 1
2
5
r´ r 1 1 1
æ r rö 6 = 5r In Parallel, = +
rPQ = r P ç + ÷ = R R1 R2
è 3 2ø 5 11
r+ r 1 1
6 +
=
Resistance between Q and R R0 [1 + a1D t ] R0 [1 + a2 D t ]
r 4 1
´ r Þ
r r 4 R0
rQR = P (r + ) = 2 3 = r (1 + aeq D t )
2 3 r 4
+ r 11 2
2 3 1 1
Resistance between P and R = +
R0 (1 + a1D t ) R0 (1 + a2 D t )
r 3 2(1 - aeq Dt ) = (1 - a1Dt )(1 - a 2Dt )
´ r
r ær ö 3 2 3
rPR = P ç + r÷ = = r a1 + a 2
3 2 è ø r 3
+ r 11 \ a eq =
3 2 2
Hence, it is clear that rPQ is maximum 44. (b) The network of resistors is a balanced wheatstone
bridge. Hence, no current will flow through centre resistor.
42. (d) A The equivalent circuit is
10W 20W
60°
xW xW 30W
10W

D E
20 –xW 10 W 20 –x W 15W
5W 10W
B C 5V 5V
1 1 1 10W
For ADE = +
R ' 2x 10
20x
or R'=
10 + 2x
5V
20x 15 ´ 30 V 5
R BC = + 20 - x + 20 - x … (i) Req = = 10 W Þ I = = = 0.5 A
10 + 2x 15 + 30 R 10
20x 6V
or + 40 = 2x
10 + 2x
Solving we get 2W
x = 10 W 1.5W
Putting the value of x = 10 W in equation (i) 45. (a)
We get 6W 3W
20 ´ 10
R BC = + 20 - 10 + 20 - 10 6V 6V
10 + 2 ´ 10
80 3W
= = 26.7 W 3/2W 3/2W
3
43. (d) Let R1 and R2 be the resistances of two conductors, then 3W 3W
R1 = R0 [1 + a1D t ]
6
hence Req = 3/2; \I = = 4A
R2 = R0 [1 + a 2 D t ] 3/ 2
P-300 Physics
46. (c) Let R1 and R2 be the two given resistances (14i1 + 10i2 = 10) ´ 17
Resistance of the series combination,
S = R1 + R2 Þ 238i1 - 170i2 = 170 ...(iv)
Resistance of the parallel combination, On solving equations (iii) and (iv), we get
R1 R2 30
P= -138i1 = 30 Þ i1 = - = -0.217
R1 + R2 138
As per question S = nP i1 is negative it means current flows from positive to
n ( R1R2 ) negative terminal.
Þ R1 + R2 =
( R1 + R2 ) D 2V
49. (d) E B
Þ (R1 + R2)2 = nR1R2 i3
1A
Minimum value of n is 4 for that
(R1 + R2)2 = 4R1R2 2W
Þ (R1 – R2)2 = 0 1A = i1 i i2 = 2A
3
47. (b) In the given circuit, resistance of 3W is in parallel A F
with series combination of two 3W resistance. 1V C 2A
3 ´ 6 18 Let us assume the potential at A = VA = 0
Rp = = = 2W Using Kirchoff's junction rule at C, we get
3+ 6 9
Using ohm’s law V = IR i1 + i3 = i2
V 3 1A + i3 = 2 A Þ i3 = 2 A
ÞI = = = 1.5 A Now using Kirchoff's loop law along ACDB
R 2
VA + 1 + i3 (2) - 2 = VB
3W 3W
3V Þ VA + 1 + i3 (1) - 2 = VB
Þ VB - VA = 3 - 2 = 1 volt
50. (c) The equivalent circuit can be drawn as
3W
2W 2W
I
i2
A i 1 4W 4W C
3V 6W 3V
3W 2W
2W 2W

48. (c) 5W i2
A B E
i1 + i2 i1 8V
Voltage across AC = 8 V
10W 10V
Resistance RAC = 4 + 4 = 8 W
V 8
i1 = = = 1 Amp
RAC 4 + 4
D C F B
20V 2W 4W 51. (2)
Using Kirchoff's loop law in loop ABCD 40W 60W
i1
-5i2 - 10(i1 + i2 ) - 2i2 + 20 = 0 A C
Þ - 10i1 - 17i2 + 20 = 0 ...(i) i2
Using Kirchoff's loop law in loop BEFC 90W 110W
Þ -10 + 4i1 + 10(i1 + i2 ) = 0 D
Þ 14i1 + 10i2 + 10 = 0 ...(ii) 40 V
Multiplying equation (i) by 10, we have 40
Current through AB, i1 = = 0.4
(10i1 + 17i2 = 20) ´ 10 40 + 60
Þ 100 i1 - 170 i 2 = 200 ...(iii) 40 1
Current through AD, i2 = =
Multiplying equation (ii) by 17, we have 90 + 110 5
Using KVL in BAD loop
Current Electricity P-301

VB + i1 (40) - i2 (90) = VD 54. (d) Applying parallel combination of batteries

1 4 E1 E2 E3
Þ VB - VD = (90) - (40) + +
5 10 1+1 2 1 +1
1 1 1
Þ VB - VD = 18 - 16 = 2 V + +
1 +1 2 1+1
52. (08.00)
2W
C A B

i2 2W 2W

i1 2W i2
D F
E i
2W
i
2 4 4
+ +
10 V 2 2 2 = 5´ 2
As capacitor is fully charged no current will flow through it. 1 1 1 3
2W + +
1A 2 2 2
A
10
= =3.3 Volt
1A 2W 3
2W æ e ö
E 55. (d) i = ç ÷
2A
B èR+rø
3A 2W Power delivered to R.
2
10 V æ e ö
P=i R= ç
2
÷ R
We have the current distribution as shown in the figure. è R+r ø
æ 4 ´ 2ö
Equivalent resistance, Req = ç +2
è 4 + 2 ÷ø
10 10 ´ 3
Net current, i = = = 3 Amp
4 10
+2
3
i1 = 2 A and i2 = 1 A
VAEB = 1 × 2 + 3 × 2 = 8 V
53. (b) For the given circuit dP
P to be maximum, =0
dR

d éæ e ö ù
2
or êç ÷ Rú = 0
dR êè R + r ø ú
ë û
or R=r
(I6)0.4A
56. (b)
I3 0.3A
3
i= 0.4A I1
8 + 2r (I5)0.4A I2
Now voltage across AB
0.8A(I4)
3
i´6 = ´6 = 2 From KCL, I3 = 0.8 – 0.4 = 0.4 A
8 + 2r
I2 = 0.4 + 0.4 + 0.3
Þ 9 = 8 + 2r = 1.1 A
and I6 = 0.4 A
1
Þr= W 57. (b) Given, E1 = 1V, E2 = 2V, E3 = 3V, r1 = 1W,
2
r2 = 1W and r 3 = 1W
P-302 Physics
6 = 3 I1 + I1 – I2 ; 4I1 – I2 = 6 ...(1)
E1 E 2 E 3 1 2 3
+ + + + – 9 + 2I2 – (I1 – I2) + 3I2 = 0
r r2 r3 1 1 1
VAB = VCD = 1 = 6 – I1 + 6I2 = 9 ...(2)
1 1 1 1 1 1 = = 2V
+ + + + 3 On solving (1) and (2)
r1 r2 r3 1 1 1
I1 = 0.13A
58. (c) Current passing through resistance R1,
Direction Q to P, since I1 > I2.
v 10 62. (c) As no current flows through arm EB then
i1 = = = 0.5A
R1 20 VD = 0V
VE = 0V
and, i2 = 0
VB = –4V
VA = 5V
59. (b) So, potential difference between the points A and D
VA – VD = 5V
63. (b) line 1 line 2 line 3
5m F I
3W 9W

8.0 V 16.0 V
I1 I2

Let voltage at C = xV In steady state capacitor is fully charged hence no current


From kirchhoff’s current law, will flow through line 2.
KCL : i1 + i2 = i By simplyfing the circuit
20 - x 10 - x x - 0
+ = Þ x = 10
2 4 2 3W 9W
V X 10
\i= = = =5A 16.0 V
R R 2 8.0 V
12V 1W U
60. (b) T Hence resultant potential difference across resistances will
v–12 be 8.0 V.
2W P
S V
v–13 Thus current I =
R
R Q 8.0 8
V 10W 0 = =
3 + 9 12
Using Kirchhoff’s law at P we get
2
V - 12 V - 13 V - 0 or, I = = 0.67 A
+ + =0 3
1 2 10 64. (a) Current through 50 V and 30 V batteries are
[Let potential at P, Q, U = 0 and at R = V respectively 2.5 A and 3 A.
65. (c) Given, emf of cell E = 200 V
V V V 12 13 0 Internal resistance of cells = 1 W
Þ + + = + +
1 2 10 1 2 10 D. C. main supply voltage V = 220 V
External resistance R = ?
10 + 5 + 1 24 + 13 æ 16 ö 37
Þ V= Þ Vç ÷ = æE-Vö
10 2 è 10 ø 2 r =ç ÷R
è V ø
37 ´10 370 æ 20 ö
ÞV= = = 11.56 volt 1=ç
16 ´ 2 32 ÷´ R \ R = 11 W.
è 220 ø
61. (a) From KVL 100 90 R + r 10 0.5 10
– 6 + 3I1 + 1 (Ii – I2) = 0 66. (c) = Þ = Þ 1+ =
R+r R R 9 R 9
6V P 2W 0.5 1
Þ = \ R = 4.5 W
R 9
67. (c) Applying Kirchoff’s second law in AB P2P1A, we get
9V
1W -2i + 5 - 10 i1 = 0
2i + 10i1 = 5 .....(i)
3W q 4W
Current Electricity P-303

B i P2 i–i1 C m = Zq
i1
1 Þ Z1 = q2
10W 2V Þ Zµ .... (i)
1W q Z 2 q1
5V
2W
Also q = q1 + q2 .... (ii)
q q
A P1 D Þ = 1 +1 (Dividing (ii) by q2)
Again applying Kirchoff's second law in P2 CDP1P2 we q2 q2
get,
q
10 i1 + 2 – i + i1= 0 Þ q2 = .... (iii)
2i – 22i1 = 4 ....(ii) q
1+ 1
From (i) and (ii) q2
32i1 = 1 From equation (i) and (iii),
1
Þ i1 = A from P2 to P1 q
32 q2 =
Z
1 2 1+ 2
68. (a) Energy in capacitor = CV Z1
2
Work done by battery = QV = CV2 73. (d) Current is given by
where C = Capacitance of capacitor E
I= ,
V = Potential difference, R+r
e = emf of battery If internal resistance (r) is zero,
1 E
CV 2 I= = constant.
2 1 R
Required ratio = = (Q V = e)
CV 2 2 Thus, energy source will supply a constant current if its
69. (d) Note : Kirchhoff's first law is based on conservation internal resistance is zero.
of charge and Kirchhoff's second law is based on 74. (d) Given E = aq + bq2
conservation of energy. dE
70. (d) At cold junction, current flows from Antimony to Þ = a + 2bq
dq
Bismuth because current flows from metal occurring later At neutral temperature
in the series to metal occurring earlier in the thermoelectric
dE
series. In thermoelectric series, Bismuth comes earlier than q = qn : =0
Antimony so at cold junction, current. Flow from Antimony dq
to Bismuth. -a d2E
Þ qn = = -350 Þ = 2b
71. (a)
R2
2b d q2
R1 E E
I hence no q is possible for E to be maximum no neutral
temperature is possible.
75. (c) From the Faraday’s first law of electrolysis,
m = Zit
Þ m = 3.3 × 10–7 × 3 × 2
= 19.8 × 10–7 kg
76. (a) Let the smallest temperature difference be q°C that
R
can be detected by the thermocouple, then
Thermo emf = (25 × 10–6) q
Let E be the emf of each source of current Let I is the smallest current which can be detected by the
2E galvanometer of resistance R.
Current in the circuit I = Potential difference across galvanometer
R + R1 + R2
IR = 10–5 × 40
Potential difference across cell having internal resistance R2
\ 10–5 × 40 = 25 × 10–6 × q
V = E – iR2 = 0
Þ q = 16°C.
2E 77. (c) According to Faraday’s first law of electrolysis
E– × R2 = 0
R + R1 + R2 m= Z×I×t
When I and t is same, m µ Z
Þ R + R1 + R 2 - 2R 2 = 0
mCu ZCu Þ m = Z Cu ´ m
Þ R + R1 - R 2 = 0 \ = Cu Zn
mZn Z Zn Z Zn
Þ R = R 2 - R1
72. (a) From Faraday’s first law of electrolysis, mass 31.5
Þ mCu = ´ 0.13 = 0.126 g
deposited 32.5
P-304 Physics

78. (b) From the Faraday’s first law of electrolysis


m = ZIt Þ m µ It V2 2202 2202
84. (a) As R = , so R1 = and R 2 =
79. (b) Given : Power, P = 1 kW = 1000 W P 25 100
R = 2W, V = 220 V 220
Current flown i =
P 1000 R1 + R 2
Current, I = =
V 220
2202 2202
2 P1 = i 2 R1 = ´ = 16 W
æ 1000 ö æ 2202 2202 ö 25
Ploss = I 2 R = ç ´2
è 220 ÷ø ç 25 + 100 ÷
è ø
1000 Similarly, P2 = i2R2 = 4 W
\ Efficiency = 1000 + P ´ 100 = 96%. 85. (b) When two resistances are connected in series,
loss
80. (b) Maximum power in external resistance is generated Req = 2R
when it is equal to internal resistance of battery i.e., PR e2 e2
maximum when r = R Power consumed, P = =
R eq 2R
The maximum Joule heating in R will take place for, the In parallel condition, Req = R/2.
resistance of small element
e2
rdr r dr
b New power, P¢ =
2pl òa r
DR = ÞR= (R/2)
2prl or P' = 4P = 240 W(Q P = 60 W)
b 86. (a) Colour code for carbon resistor
a Bl, Br, R, O, Y, G, Blue, V, Gr, W
0 1 2 3 4 5 6 7 8 9
Resistance, R = AB × C ± D
\ Resistance, R = 50 × 102 W
Now using formula, Power, P = i 2R
l r P 2
\i= = = 20mA
R 50 ´102
b 87. (a) Power, P = I2R
R 4.4 = 4 × 10–6 × R
Þ R = 1.1 × 106W
When supply of 11 v is connected
r b
or, R = ln v 2 112 112
2 pl a Power, P’= = ´ ´10 –6
R 1.1 1.1
81. (d) Net Power, P
= 15 × 45 + 15 × 100 + 15 × 10 + 2 × 1000 =11´10 –5 W
= 15 × 155 + 2000 W 88. (a) Rate of heat i.e., Power developed in the wire =
Power, P = VI
V2
P P=
Þ I= R
V
rL rL
15 ´155 + 2000 Resistance of the wire of length, L R1 = =
\ I main = = 19.66 A » 20 A A pr 2
220
82. (b) Equivalent resistance, V2
\ Power, P1 =
4 R ´ 4R 6 R ´ 12 R R1
Req = + R+ +R Resistance of the wire when length is halved i.e., L/2
4R + 4R 6 R + 12 R
= 2R + R + 4R + R L
r
= 8R.
R2 = 2 = rL = R1
V2 162 p(2r )2 p8r 2 8
Using, P = R Þ 4 =
eq 8R V 8V
\ Power, P2 = =
R1 R1
162 8
\ R= =8W
4´8 or, P2 = 8P1 i.e., power increased 8 times of previous
83. (b) or original wire.
Current Electricity P-305

89. (c) From the given circuit, net resistances Voltage across bulb before heater is switched on,
RI = 1 W, RII = 1/2 W, RIII = 3/2 W
240
It is clear that R3 > R1 > R2 V1 = ´ 120 = 117.73 volt
Hence, P3 < P1 < P2 246
Voltage across bulb after heater is switched on,
V2 1
As Power (P) = ÞPµ 48
R R V2 = ´120 = 106.66 volt
54
30
V2 Hence decrease in voltage
90. (None) Work done in 30s, W = ò R
dt
V1 – V2 = 117.073 – 106.66 = 10.04 Volt (approximately)
0
30 30 P=2 W Q=4W
(200)2 (200)2 dt 95. (b)
or, W= ò 20t
dt =
100 ò 20a I1
0 100(1 + a ) 0 1+ t A I2 B
3 3
R=1W S=2W
æ 1 + 20a ö
´ 30 ÷ æ 6ö R1 = P + Q = 2 W + 4 W = 6 W
400 ´ 3 ç
÷ = 60,000 ln
= lnç 3 çè ÷ø
20a 5 R2 = R + S = 1W + 2 W = 3 W
è 1 ø
Q 120 = 100 éë1 + a ( 200 ) ùû I1R1 = I2R2
R2 3 I
1 I1 = I2 = I2 = 2
\ a= R1 6 2
1000
91. (c) Heat energy will be maximum when resistance will be or I2 = 2I1
minimum. Heat flow H = I2 Rt
92. (c) Total power consumed by electrical appliances in
the building, Ptotal = 2500W I22
For Q, H Q = I12 Qt = ´ 4t = I 22 t
Watt = Volt × ampere 4
Þ 2500 = V × I Þ 2500 = 220 I 2 2 2
For S, H S = I 2 St = I2 × 2t = 2I 2 t
2500 \ Greatest amount of heat generated by S.
Þ I= = 11.36 » 12A
220 96. (c) Current capacity of 25 W bulb
(Minimum capacity of main fuse)
W1 25
Power I1 = = Am p
93. (c) Current in each bulb = Voltage V1 220
Current capacity of 100 W bulb
100 W 2 100
= = 0.45A
220 I2 = V = 220 Am p
Current through ammeter = 0.45 × 3 = 1.35 A 2
The current flowing through the circuit
6W B1 B2

(Lead) Bulb
94. (d)
R1 r2

120 V
Power of bulb = 60 W (given) 440V
Resistance of 25 W bulb,
120 ´ 120
Resistance of bulb = = 240W V12 (220)2
60 R1 = = ;
P1 25
é V2 ù
êQ P = ú Resistance of 100 W bulb
ë R û
Power of heater = 240W (given) V22 (220)2
R2 = =
P 100
120 ´120
Resistance of heater = = 60W Reff = R1 + R2
240 Current flowing through circuit
P-306 Physics

440 101. (c) The resistance of the electric bulb is


I=
R eff V 2 (220) 2
R= =
440 P 100
I= 2 The power consumed when operated at 110 V is
(220) (220) 2
+
25 100 V2
P¢ =
440 40 R
= ; I= Am p
é1 1 ù 220 (110) 2 100
(220) 2 ê + Þ P= = = 25 W
ë 25 100 úû (220) /1002 4
102. (b) Heat generated,
æ 25 ö æ 40 ö < I æ = 100 A ö
Q I1 ç = A÷ < I ç = A÷ 2ç ÷
è 220 ø è 220 ø è 200 ø V 2t
Thus the bulb rated 25 W–220 will fuse. H=
R
6´3 After cutting equal length of heater coil will become half.
97. (b) Total resistance = = 2W As R µ l
6+3
R
6 Resistance of half the coil =
Current in circuit = = 3A 2
2
Therefore current through bulb 1 is 2A and bulb 2 is 1A. V 2t
H¢ = = 2H
So bulb 1 will glow more R
98. (c) Resistors 4 W, 6 W and 12 W are connected in parallel, 2
its equivalent resistance (R) is given by \ As R reduces to half, ‘H’ will be doubled.
1 1 1 1
= + + 12 V2
Þ R= = 2W 103. (b) Power, P = Vi =
R 4 6 12 6 R
Again R is connected to 1.5 V battery whose internal \ Resistance of tungsten filament when in use
resistance r = 1 W.
Equivalent resistance now, V 2 200 ´ 200
R¢ = 2W + 1W = 3W Rhot = = = 400 W
P 100
V 1.5 1 Resistance when not in use i.e., cold resistance
Current, Itotal = = = A
R' 3 2 400
Rcold = = 40 W
1 10
Itotal = = 3x + 2x + x = 6x
2 104. (a) Thermistors are usually made of metaloxides with
1 high temperature coefficient of resistivity.
Þx= 105. (a) Heat supplied in time t for heating 1L water from
12
10°C to 40°C
\ Current through 4W resistor = 3x
DQ = mCp × DT
1 1 = 1 × 4180 × (40 – 10) = 4180 × 30
= 3× = A
12 4 But DQ = P × t = 836 × t
Therefore, rate of Joule heating in the 4W resistor
4180 ´ 30
2 Þt= = 150s
æ1ö 1 836
= I2R = ç ÷ ´ 4 = = 0.25W
è4ø 4 106. (c) We know that resistance,
99. (c) 2
Vrated (220) 2
100. (b) Let resistance of bulb filament be R0 at 0°C using R = R= = = 48.4 W
R0 (1 + a Dt) we have Prated 1000
R1 = R0 [1 + a × 100] = 100 ....(1) When this bulb is connected to 110 volt mains supply we
R2 = R0 [1 + a × T] = 200 ....(2) get
On dividing we get V 2 (110)2
P= = = 250W
200 1 + aT 1 + 0.005 T R 48.4
= Þ2= 107. (b) Case 1 Initial power dissipation,
100 1 + 100a 1 + 100 ´ 0.005
Þ T = 400°C R
Note : We may use this expression as an approximation
V2
because the difference in the answers is appreciable. For P1 =
accurate results one should use R = R0eaDT R
V
Current Electricity P-307

Case 2 Balancing length from P = 100 – 49


When wire is cut into two equal pieces, the resistance of 1.02
R \x = = 0.02 volt/cm
each piece is . When they are connected in parallel 100 - 49
2 112. (40) For the given meter bridge
R/2 R
= R l1
Equivalent resistance, Req = = Where, l1 = balancing length
2 4 S 100 – l1
R/2 R/4 R 25 1
R/2 Þ = = ...(i)
S 75 3
=
New resistance,
V V l
r
Power dissipated, l´2 lö
R' = 2 = r æ
2 æV ö 2 A A çQ R = r ÷
V è Aø
P2 = = 4ç = 4 P1
R/4 ç R ÷÷ 4
è ø Þ R¢ = 2R
108. (b) The equivalent resistance of parallel combination of R' l2 2R l2
= Þ =
2W and R is S 100 – l 2 S 100 – l 2
2´R 1 l2
Req = Þ 2´ = Using (i)
2+ R 3 100 – l 2
V2 (15)2 Þ l!
2 = 40 cm
\ Power dissipation P = \ 15 0 =
Re q Req
113. (d)
225 ´ ( R + 2) 2R 3
Þ 150 = Þ =
2R 2+ R 2
Þ 4 R = 6 + 3R Þ R = 6W
109. (d) The voltmeter of resistance 10kW is parallel to the
resistance of 400W. So, their equivalent resistance is
1 1 1 1 1
= + = +
R ' 10 k W 400W 10000 400
1 1 + 25 26 Let R be the resistance of the whole wire
Þ = = Potential gradient for the potentiometer wire
R ' 10000 10000
dV I ´ R é 60 ´ R ù
10000 ' AB ' = - = =ê ú mv / m
Þ R' = W dl l ë l AB û
26
Using Ohm's law, current in the circuit æ dV ö 60 ´ R
V AP = ç lAP = ´ 1000mV
Voltage 6 è d l AB ÷ø 1200
I= = Þ VAP = 50 R mV
Net Resistance 10000 + 800
Also, VAP = 5 V (for balance point at P)
26
Potential difference measured by voltmeter V AP 5
\R = –3
= = 100W
6 10000 50 ´ 10 50 ´ 10 –3
V = IR ' = ´
10000 26 114. (10)
+ 800
26
150
ÞV = = 1.95 volt
77
110. (b) Multimeter shows deflection in both cases i.e. before
and after reversing the probes if the chosen component is
capacitor.
Potential drop
111. (a) Potential gradient, x =
length P S
As per Wheatstone bridge balance condition =
Here, Potential drop = 1.02 Q R
P-308 Physics

Let resistance R’ is connected in parallel with resistance S 118. (c) We have given
of 10W dR 1 dR 1
µ Þ = k´ (where k is constant)
15
\ =
10 R '
Þ 5=
10 R ' dl l dl l
12 10 + R ' 10 + R '
dl
4 dR = k
Þ 50 + 5R’ = 10R’
l
Let R1 and R2 be the resistance of AP and PB respectively.
50 Using wheatstone bridge principle
\ R' = = 10W
5 R ' R1
115. (12) We know that \ = or R1 = R 2
R ' R2
E µ l where l is the balancing length
\ E = k (560) ....(i) dl
Now, ò dR = k ò
When the balancing length changes by 60 cm l
l
E
10 = k (500) ...(ii) \ R1 = k ò l -1 2dl = k.2. l
r + 10
0
1
Dividing (i) by (ii) we get
R 2 = k ò l -1 2dl = k.(2 - 2 l )
r + 10 56 l
Þ = Þ 50 r + 500 = 560 Putting R1 = R2
10 50
k2 l = k(2 - 2 l )
6 N
Þ r = W = W Þ N = 12 \ 2 l =1
5 10 1
l=
116. (c) For a balanced bridge 2
1
R1 l 2 i.e., l = m Þ 0.25 m
= 4
R 2 l1
4v R
R l
So =
X 100 – l i i
119. (c) 5W
Using the above expression
1m
R(100 – l )
X= Current flowing through the circuit (I) is given by
l
æ 4 ö
I=ç A
for observation (1) X =
100 ´ 40 2000
= W è R + 5 ÷ø
60 3 Resistance of length 10 cm of wire
100 ´ 87 8700 10
for observation (2) X = = W =5´ = 0.5W
13 13 100
10 ´ 98.5 1970 According to question,
for observation (3) X = = W æ 4 ö
1.5 3 5 ´ 10 -3 = ç .(0.5)
è R + 5 ÷ø
1 ´ 99
for observation (4) X = = 99W 4
1 \ = 10 -2 or R + 5 = 400 W
Clearly we can see that the value of x calculated in R +5
\ R = 395W
observation (4) is inconsistent than other. 120. (b) Given, Emf of cell, e = 0.5 v
117. (c) The resistance of potentiometer wire
Rheostat resistance, Rh = 2W
R = 0.01 × 400 = 4 W
Potential gradient is
Current in the wire
V 3 1 dv æ 6 ö 4
= ´
i= = = A
RT 4 + 0.5 + 0.57 + 1 2
dL çè 2 + 4 ÷ø L
Let null point be at l cm when cell of emfe = 0.5 v is used.
1
Now V = iRAJ = × (0.01 × 50) = 0.25 V.. æ 6 ö 4
2 thus e1 = 0.5V = ç ´ ´l ... (i)
è 2 + 4 ÷ø L
Current Electricity P-309

For resistance R h = 6 W new potential gradient is On balancing condition


R1(100 – l) = (1000 – R1)l ...(i)
æ 6 ö 4 and at null point
çè ÷´ On Interchanging resistance balance point shifts left by 10 cm
4 + 6ø L R2=1000 – R1 R1
æ 6 ö æ 4ö ... (ii)
çè ÷ ç ÷ ´ l = e2 G
4 + 6ø è L ø
Dividing equation (i) by (ii) we get (l – 10) (100 – l + 10)
0.5 10 =(110 – l)
= thus e2 = 0.3v On balancing condition
e2 6 (1000 – R1) (110 – l) = R1 (l – 10)
121. (d) or, R1 (l – 10) = (1000 – R1) (110 – l) ...(ii)
R1 2 Dividing eqn (i) by (ii)
122. (c) Initially at null deflection = ...(i)
R2 3 100 - l l
=
Finally at null deflection, when null point is shifted l - 10 110 - l
R1 + 10 Þ (100 – l) (110 – l) = l(l – 10)
= 1 Þ R1 + 10 = R2 ...(ii) Þ 11000 – 100l – 110l + l2 = l2 – 10l
R2
Þ 11000 = 200l
Solving equations (i) and (ii) we get or, l = 55
2R 2 Putting the value of ‘l’ in eqn (i)
+ 10 = R 2
3 R1 (100 – 55) = (1000 – R1) 55
R Þ R1 (45) = (1000 – R1) 55
10 = 2 Þ R 2 = 30W Þ R1 (9) = (1000 – R1) 11
3
& R1 = 20W Þ 20 R1 = 11000
Now if required resistance is R then \ R1 = 550KW
30 ´ R 127. (b) For a balanced meter bridge,
30 + R = 2 X
=
Y
Þ Y = 39.5 = X × (100 – 39.5)
30 3 39.5 (100 - 39.5)
R = 60W 12.5 ´ 39.5
123. (c) Let x be the length AJ at which galvanometer shows or, X = = 8.16 W
null deflection current, 60.5
When X and Y are interchanged l1 and (100 – l1) will also
e 3 æx ö e interchange so, l2 = 60.5 cm
i= = or, i ç 12r ÷ = 128. (d) There is no change in null point, if the cell and the
12r + r 13r è L ø 2
galvanometer are exchanged in a balanced wheatstone
e éx ù e e éx ù e
Þ ·12r ú = Þ ·12r ú = bridge.
ê
13r ë L ê
û 2 13r ë L û 2
13L
or, x =
24
124. (a) Given, colour code of resistance,
R1 = Orange, Red and Brown
\ R1= 32 × 10 = 320
using balanced wheatstone bridge principle,
R1 R 2 320 80
= Þ =
R3 R 4 R 3 40
\ R3 = 160 i.e. colour code for R3 Brown, Blue and Brown R1 R2
125. (b) Using formula, internal resistance, On balancing condition =
R3 R4
æl -l ö 52 - 40 ö After exchange
r = ç 1 2 ÷ s = æç ÷ ´ 5 = 1.5W
l
è 2 ø è 40 ø
126. (c) R1 + R2 = 1000
Þ R2 = 1000 – R1
R1 R2 = 1000 – R1

G
R1 R3
On balancing condition =
(l ) 100 – l R2 R
P-310 Physics

P l 133. (c) To measure the emf of a cell we prefer potentiometer


129. (d) In balance position of bridge, = rather than voltmeter because
Q (100 - l )
Initially neutral position is 60 cm. from A, so (i) the length of potentiometer which allows greater
precision.
4 Q 16 8
= ÞQ= = W (ii) in case of potentiometer, no current flows through the
60 40 6 3 cell.
Now, when unknown resistance R is connected in series (iii) of high sensitivity.
to P, neutral point is 80 cm from A then, 134. (a) Bridge wire in a sensitive meter bridge wire should be
4+ R Q of high resistivity and low temperature coefficient.
=
80 20
x 40 2
4+ R 8
= 135. (c) From question, y = 100 - 40 = 3
80 60
64 64 - 24 40 2
R= -4= = W Þx= y
6 6 6 3
20
Hence, the value of unknown resistance R is = W 3x Z
3 Again, =
130. (d) When key is at point (1) y 100 - Z
V1 = iR1 = xl1
2y
When key is at (3) 3´
3 = Z
V2 = i (R1 + R2) = xl2 or
y 100 - Z
R1 l R l
= 1 Þ 1= 1 Solving we get Z = 67 cm
R1 + R 2 l2 R 2 l2 - l1
Therefore new position of null point @ 67 cm
131. (c) As the two cells oppose each other hence, the 136. (c) Potential gradient
effective emf in closed circuit is 15 – 10 = 5 V and net
resistance is 1 + 0.6 = 1.6 W (because in the closed V IR I æ rl ö I r
Þk= = = ç ÷=
circuit the internal resistance of two cells are in series. l l lè A ø A
Current in the circuit,
effective emf 5 0.2 ´ 4 ´ 10 -7 0.8
I= = A k= = = 0.1 V/m
total resistance 1.6 8 ´ 10-7 8

The potential difference across voltmeter will be same as 137. (b) Given,
the terminal voltage of either cell. Balance point from one end, l1 = 20 cm
Since the current is drawn from the cell of 15 V From the condition for balance of metre bridge, we have
\ V1 = E1 – Ir1
55 l1
=
5 R 100 – l1
= 15 – ´ 0.6 = 13.1 V
1.6
132. (c) Balancing length l will give emf of cell 55 20
=
\ E = Kl R 80
Here K is potential gradient. Þ R = 220W
If the cell is short circuited by resistance 'R'
P R
Let balancing length obtained be l¢ then 138. (b) From balanced wheat stone bridge = where
Q S
V = kl¢
S1S 2
æ E -V ö S=
r= ç R
è V ÷ø S1 + S 2
Þ V = E – V [Q r = R given] 139. (c) Initial balancing length, l1 = 240 cm New balancing
Þ 2V = E length, l2 = 120 cm.
or, 2Kl¢ = Kl The internal resistance of the cell,
l
\ l¢ = æl -l ö 240 - 120
2 r= ç 1 2 ÷´R = ´ 2 = 2W
è l 2 ø 120
Current Electricity P-311

140. (c) From the balanced wheat stone bridge


E
R1 l1
=
R2 l 2 i
where l2 = 100 – l1 i
In the first case X = 20 r
Y 80
E'
Y = 4X
In the second case Note : In this arrangement, the internal resistance of the
4X l battery E does not play any role as current is not passing
= through the battery.
Y 100 - l
142. (d) ig × G = (i – ig) S
4X l
Þ = ig ´ G 1 ´ 0.81
4 X 100 – l \ S= = = 0.09W
i - ig 10 - 1
Þ l = 50
141. (d) From the principle of potentiometer, V µ l 143. (c) To use an ammeter in place of voltmeter, we must
If a cell of emF E is employed in the circuit between the connect a high resistance in series with the ammeter.
ends of potentiometer wire of length L, then Connecting high resistance in series makes its resistance
V l much higher.
= ;
E L
El 30E
Þ V= =
L 100
18
P-312 Physics

Moving Charges
and Magnetism
4. A beam of protons with speed 4 × 105 ms–1 enters a uniform
Motion of Charged Particle in magnetic field of 0.3 T at an angle of 60° to the magnetic
TOPIC 1
Magnetic Field field. The pitch of the resulting helical path of protons is
close to : (Mass of the proton = 1.67 × 10–27 kg, charge of
1. An electron is moving along + x direction with a velocity the proton = 1.69 × 10–19 C) [Sep. 02, 2020 (I)]
of 6 × 106 ms–1. It enters a region of uniform electric field of (a) 2 cm (b) 5 cm (c) 12 cm (d) 4 cm
300 V/cm pointing along + y direction. The magnitude and 5. The figure shows a region of length ‘l’ with a uniform
direction of the magnetic field set up in this region such magnetic field of 0.3 T in it and a proton entering the region
that the electron keeps moving along the x direction will with velocity 4 × 105 ms–1 making an angle 60° with the
be : [Sep. 06, 2020 (I)] field. If the proton completes 10 revolution by the time it
cross the region shown, ‘l’ is close to (mass of proton
(a) 3 × 10–4 T, along + z direction = 1.67 × 10–27 kg, charge of the proton = 1.6 × 10–19 C)
(b) 5 × 10–3 T, along – z direction [Sep. 02, 2020 (II)]
(a) 0.11 m B
(c) 5 × 10–3 T, along + z direction
(d) 3 × 10–4 T, along – z direction (b) 0.88 m
2. A particle of charge q and mass m is moving with a 60°
(c) 0.44 m
velocity – v $i (v ¹ 0) towards a large screen placed in the
(d) 0.22 m l
Y-Z plane at a distance d. If there is a magnetic field
ur 6. Proton with kinetic energy of 1 MeV moves from south to
B = B0 k$ , the minimum value of v for which the particle north. It gets an acceleration of 1012 m/s2 by an applied
will not hit the screen is: [Sep. 06, 2020 (I)] magnetic field (west to east). The value of magnetic field:
(Rest mass of proton is 1.6 ´ 10–27 kg) [8 Jan 2020, I]
qdB0 2qdB0
(a) (b) (a) 0.71 mT (b) 7.1 mT
3m m (c) 0.071 mT (d) 71 mT
qdB0 qdB0 7. A particle having the same charge as of electron moves
(c) (d)
m 2m in a circular path of radius 0.5 cm under the influence of
3. A charged particle carrying charge 1 mC is moving with
a magnetic field of 0.5T. If an electric field of 100V/m
velocity (2iˆ + 3 ˆj + 4kˆ) ms–1. If an external magnetic field makes it to move in a straight path then the mass of the
of (5iˆ + 3 ˆj - 6kˆ) ´10-3 T exists in the region where the particle is (Given charge of electron = 1.6 × 10–19C)
particle is moving then the force on the particle is [12 April 2019, I]
ur ur
F ´10-9 N. The vector F is : [Sep. 03, 2020 (I)] (a) 9.1 × 10–31 kg (b) 1.6 × 10–27 kg
(a) - 0.30iˆ + 0.32 ˆj - 0.09kˆ (c) 1.6 × 10–19 kg (d) 2.0 × 10–24 kg
8. An electron, moving along the x-axis with an initial energy
(b) - 30iˆ + 32 ˆj - 9kˆ ur
of 100 eV, enters a region of magnetic field B = (1.5×10–3T) k$
(c) - 300iˆ + 320 ˆj - 90kˆ at S (see figure). The field extends between x = 0 and x =
2 cm. The electron is detected at the point Q on a screen
(d) - 3.0iˆ + 3.2 ˆj - 0.9kˆ
Moving Charges and Magnetism P-313

placed 8 cm away from the point S. The distance d between 13. An electron, a proton and an alpha particle having the same
P and Q (on the screen) is : kinetic energy are moving in circular orbits of radii re,
(Electron’s charge = 1.6 × 10–19 C, mass of electron rp, ra respectively in a uniform magnetic field B. The
= 9.1 × 10–31 kg) [12 April 2019, II] relation between re, rp, ra is : [2018]
(a) re > rp = ra (b) re < rp = ra
(c) re < rp < ra (d) re < ra < rp
14. A negative test charge is moving near a long straight wire
carrying a current. The force acting on the test charge is
parallel to the direction of the current. The motion of the
charge is : [Online April 9, 2017]
(a) away from the wire
(b) towards the wire
(c) parallel to the wire along the current
(d) parallel to the wire opposite to the current
15. In a certain region static electric and magnetic fields exist.
r
The magnetic field is given by B = B0 (iˆ + 2ˆj - 4k)ˆ . If a
(a) 11.65 cm (b) 12.87 cm r
test charge moving with a velocity v = v 0 (3iˆ - ˆj + 2k) ˆ
(c) 1.22 cm (d) 2.25 cm experiences no force in that region, then the electric field
9. A proton, an electron, and a Helium nucleus, have the in the region, in SI units, is : [Online April 8, 2017]
r r
same energy. They are in circular orbits in a plane due to (a) E = -v0B0(3iˆ - 2jˆ - 4k)
ˆ (b) E = -v B (iˆ + ˆj + 7k)
0 0
ˆ
r r
magnetic field perpendicular to the plane. Let rp, re and rHe (c) E = v0 B0 (14jˆ + 7k) ˆ (d) E = -v0 B0 (14jˆ + 7k) ˆ
be their respective radii, then, [10 April 2019, I] 16. Consider a thin metallic sheet perpendicular to the plane
(a) re > rp = rHe (b) re < rp = rHe of the paper moving with speed 'v' in a uniform magnetic
(c) re < rp < rHe (d) re > rp > rHe field B going into the plane of the paper (See figure). If
10. A proton and an α -particle (with their masses in the ratio charge densities s1 and s2 are induced on the left and
of 1 : 4 and charges in the ratio 1 : 2) are accelerated from right surfaces, respectively, of the sheet then (ignore fringe
rest through a potential difference V. If a uniform magnetic effects): [Online April 10, 2016]
field (B) is set up perpendicular to their velocities, the ratio - Î0 vB Î vB v
of the radii rp : ra of the circular paths descrfibed by them (a) s1 = , s2 = 0
2 2
will be: [12 Jan 2019, I]
(b) s1 = Î0 vB, s 2 = - Î0 vB
(a) 1: 2 (b) 1: 2 (c) 1: 3 (d) 1: 3 B
Î vB - Î0 vB
11. In an experiment, electrons are accelerated, from rest, by (c) s1 = 0 , s2 =
applying a voltage of 500 V. Calculate the radius of the 2 2
path if a magnetic field 100 mT is then applied. (d) s1 = s 2 = Î0 vB s1 s2
[Charge of the electron = 1.6 × 10–19 C 17. A proton (mass m) accelerated by a potential difference V
Mass of the electron = 9.1 × 10–31 kg] [11 Jan 2019, I] flies through a uniform transverse magnetic field B. The
field occupies a region of space by width ‘d’. If a be the
(a) 7.5 × 10–3 m (b) 7.5 × 10–2 m angle of deviation of proton from initial direction of motion
(c) 7.5 m (d) 7.5 ×10–4 m (see figure), the value of sin a will be :
12. The region between y = 0 and y = d contains a magnetic [Online April 10, 2015]
r
field B = Bzˆ . A particle of mass m and charge q enters B
the region with a velocity vr = viˆ . if d =
mv
, the a
2qB
acceleration of the charged particle at the point of its
emergence at the other side is : [11 Jan 2019, II] d
qv B æ 1 ˆ 3 ˆö qv B æ 3 ˆ 1 ˆ ö
(a) ç i- j÷ (b) ç i + j÷
qV
Bd B qd
m è2 2 ø m è 2 2 ø (a) (b)
2m 2 mV

qvB æ - ˆj + iˆ ö qvB æ iˆ + ˆj ö B q q
(c) ç ÷ (d) ç ÷ (c) (d) Bd
m è 2 ø m è 2 ø d 2mV 2mV
P-314 Physics

18. A positive charge ‘q’ of mass ‘m’ is moving along the + x (a) 8 N in z-direction (b) 8 N in y-direction
axis. We wish to apply a uniform magnetic field B for (c) 4 N in y-direction (d) 4 N in z-direction
time Dt so that the charge reverses its direction crossing 24. The velocity of certain ions that pass undeflected through
the y axis at a distance d. Then: [Online April 12, 2014] crossed electric field E = 7.7 k V/m and magnetic field
B = 0.14 T is [Online May 7, 2012]
mv pd mv pd
(a) B = and Dt = (b) B = and Dt = (a) 18 km/s (b) 77 km/s
qd v 2qd 2v (c) 55 km/s (d) 1078 km/s
2mv pd 2mv pd r
(c) B = and Dt = (d) B= and Dt = 25. An electric charge +q moves with velocity v = 3iˆ + 4 ˆj + kˆ
qd 2v qd v ur
19. A particle of charge 16 × 10–16 C moving with velocity in an electromagnetic field given by E = 3i$ + $j + 2k$ and
10 ms–1 along x-axis enters a region where magnetic field ur
ur B = iˆ + ˆj - 3kˆ The y - component of the force experienced
of induction B is along the y-axis and an electric field by + q is : [2011 RS]
of magnitude 104 Vm–1 is along the negative z-axis. If
(a) 11 q (b) 5 q (c) 3 q (d) 2 q
the charged particle continues moving along x-axis, the
ur 26. A charged particle with charge q enters a region of
magnitude of B is : [Online April 23, 2013] ur
(a) 16 × 103 Wb m–2 (b) 2 × 103 Wb m–2 constant, uniform and mutually orthogonal fields E and
ur r ur ur
(c) 1 × 103 Wb m–2 (d) 4 × 103 Wb m–2 B with a velocity v perpendicular to both E and B ,
20. Proton, deuteron and alpha particle of same kinetic energy and comes out without any change in magnitude or
are moving in circular trajectories in a constant magnetic r
direction of v . Then [2007]
field. The radii of proton, deuteron and alpha particle are r ur ur 2 r ur ur 2
respectively rp, rd and ra. Which one of the following (a) v = B ´ E / E (b) v = E ´ B / B
relation is correct? [2012] r ur ur 2 r ur ur
(c) v = B ´ E / B (d) v = E ´ B / E 2
(a) ra = rp = rd (b) ra = rp < rd 27. A charged particle moves through a magnetic field
(c) ra > rd > rp (d) ra = rd > rp perpendicular to its direction. Then [2007]
(a) kinetic energy changes but the momentum is
21. This question has Statement 1 and Statement 2. Of the
constant
four choices given after the Statements, choose the one
(b) the momentum changes but the kinetic energy is
that best describes the two Statements.
Statement 1: A charged particle is moving at right angle constant
to a static magnetic field. During the motion the kinetic (c) both momentum and kinetic energy of the particle
energy of the charge remains unchanged. are not constant
Statement 2: Static magnetic field exert force on a moving (d) both momentum and kinetic energy of the particle
charge in the direction perpendicular to the magnetic field. are constant
[Online May 26, 2012] 28. In a region, steady and uniform electric and magnetic fields
(a) Statement 1 is false, Statement 2 is true. are present. These two fields are parallel to each other. A
(b) Statement 1 is true, Statement 2 is true, Statement 2 is charged particle is released from rest in this region. The
not the correct explanation of Statement 1. path of the particle will be a [2006]
(c) Statement 1 is true, Statement 2 is false. (a) helix (b) straight line
(d) Statement 1 is true, Statement 2 is true, Statement 2 is (c) ellipse (d) circle
the correct explanation of Statement 1. 29. A charged particle of mass m and charge q travels on a
22. A proton and a deuteron are both accelerated through the circular path of radius r that is perpendicular to a magnetic
same potential difference and enter in a magnetic field field B. The time taken by the particle to complete one
perpendicular to the direction of the field. If the deuteron revolution is [2005]
follows a path of radius R, assuming the neutron and proton
masses are nearly equal, the radius of the proton’s path 2pq 2 B 2 pmq 2pm 2pqB
(a) (b) (c) (d)
will be [Online May 19, 2012] m B qB m
R 30. A uniform electric field and a uniform magnetic field are
R
(a) 2R (b) (c) (d) R acting along the same direction in a certain region. If an
2 2 electron is projected along the direction of the fields with
23. The magnetic force acting on charged particle of charge 2 a certain velocity then [2005]
mC in magnetic field of 2 T acting in y-direction, when the (a) its velocity will increase
( ) 6 -1
particle velocity is 2iˆ + 3 ˆj ´ 10 ms is (b) Its velocity will decrease
(c) it will turn towards left of direction of motion
[Online May 12, 2012]
(d) it will turn towards right of direction of motion
Moving Charges and Magnetism P-315

31. A particle of mass M and charge Q moving with velocity 36. Magnitude of magnetic field (in SI units) at the centre of
r a hexagonal shape coil of side 10 cm, 50 turns and
v describe a circular path of radius R when subjected to
a uniform transverse magnetic field of induction B. The m0 I
carrying current I (Ampere) in units of is :
work done by the field when the particle completes one p
full circle is [2003] [Sep. 03, 2020 (I)]
æ Mv 2 ö
(a) ç (b) zero (a)250 3 (b) 50 3 (c) 500 3 (d) 5 3
÷ 2 pR
è R ø 37. A long, straight wire of radius a carries a current distribut-
(c) B Q 2 p R (d) B Qv 2p R ed uniformly over its cross-section. The ratio of the
32. If an electron and a proton having same momenta enter a
magnetic fields due to the wire at distance and 2a,
perpendicular to a magnetic field, then [2002] 3
(a) curved path of electron and proton will be same respectively from the axis of the wire is: [9 Jan 2020, I]
(ignoring the sense of revolution)
2 1 3
(b) they will move undeflected (a) (b) 2 (c) (d)
(c) curved path of electron is more curved than that of 3 2 2
the proton 38. An electron gun is placed inside a long solenoid of radius
(d) path of proton is more curved. R on its axis. The solenoid has n turns/length and carries
33. The time period of a charged particle undergoing a circular a current I. The electron gun shoots an electron along the
motion in a uniform magnetic field is independent of its radius of the solenoid with speed v. If the electron does
(a) speed (b) mass [2002] not hit the surface of the solenoid, maximum possible value
(c) charge (d) magnetic induction of v is (all symbols have their standard meaning):
[9 Jan 2020, II]
Magnetic Field Lines,
TOPIC 2 Biot-Savart's
Law and Ampere's Circuital
34. A charged particle going around in a circle can be con-
sidered to be a current loop. A particle of mass m carry-
ing charge q is moving in a plane wit speed v under the
® em0 nIR em0 nIR
influence of magnetic field B . The magnetic moment (a) (b)
m 2m
of this moving particle : [Sep. 06, 2020 (II)]
® ® em0 nIR 2em 0 nIR
mv 2 B mv 2 B (c) (d)
(a) (b) - 4m m
2 B2 2 pB 2 39. A very long wire ABDMNDC is shown in figure carrying
® ® current I. AB and BC parts are straight, long and at
mv 2 B mv 2 B right angle. At D wire forms a circular turn DMND of
(c) - (d) - radius R.
B2 2 B2
35. A wire A, bent in the shape of an arc of a circle, carrying a AB, BC parts are tangential to circular turn at N and D.
current of 2 A and having radius 2 cm and another wire B, Magnetic field at the centre of circle is:
also bent in the shape of arc of a circle, carrying a current [8 Jan 2020, II]
of 3 A and having radius of 4 cm, are placed as shown in
m0 I æ 1 ö
the figure. The ratio of the magnetic fields due to the wires p+
2 pR çè ÷
(a)
A and B at the common centre O is : 2ø
[Sep. 04, 2020 (I)]
m0 I æ 1 ö
p-
2 pR çè ÷
(b)

A B
O m0 I
(c) (p + 1)
2pR
90° m0 I
60° (d)
2R
(a) 4 : 6 (b) 6 : 4
(c) 2 : 5 (d) 6 : 5
P-316 Physics

40. Two very long, straight, and insulated wires are kept at 45. As shown in the figure, two infinitely long, identical wires
90° angle from each other in xy-plane as shown in the are bent by 90º and placed in such a way that the segments
figure. LP and QM are along the x-axis, while segments PS and
QN are parallel to the y-axis. If OP = OQ = 4 cm, and the
magnitude of the magneticf field at O is 10–4 T, and the
two wires carry equal currents (see figure), the magnitude
of the current in each wire and the direction of the magnetic
field at O will be (µ0 = 4p × 10–7 NA–2): [12 Jan 2019, I]
y
S

These wires carry currents of equal magnitude I, whose


directions are shown in the figure. The net magnetic field
at point P will be : [12 April 2019, I] OQ
L P M x
m0 I
(a) Zero (b) – ( xˆ + yˆ )
2 pd
+m 0 I m0 I
(c) ( zˆ ) (d) ( xˆ + yˆ ) N
pd 2 pd
(a) 20 A, perpendicular out of the page
41. A thin ring of 10 cm radius carries a uniformly distributed
(b) 40 A, perpendicular out of the page
charge. The ring rotates at a constant angular speed of 40
À rad s–1 about its axis, perpendicular to its plane. If the (c) 20 A, perpendicular into the page
magnetic field at its centre is 3.8 × 10–9 T, then the charge (d) 40 A, perpendicular into the page
carried by the ring is close to (µ0 = 4p × 10–7 N/A2). 46. A current loop, having two circular arcs joined by two
[12 April 2019, I] radial lines is shown in the figure. It carries a current of 10
(a) 2×10 C (b) 3×10 C (c) 4×10–5C (d) 7×10–6C
–6 –5 A. The magnetic field at point O will be close to:
42. Find the magnetic field at point P due to a straight line [9 Jan. 2019 I]
segment AB of length 6 cm carrying a current of 5 A. (See O
figure) (mo=4p×10–7 N-A–2) [12 April 2019, II]
°
3 cm

3c
5
q =4
m

Q R
2 cm

2 cm

P S

i = 10A
(a) 1.0 × 10–7 T (b) 1.5 × 10–7 T
(c) 1.5 × 10 T–5 (d) 1.0 × 10–5 T
(a) 2.0×10–5T (b) 1.5×10–5T 47. One of the two identical conducting wires of length L is
(c) 3.0×10–5T (d) 2.5×10–5T bent in the form of a circular loop and the other one into
43. The magnitude of the magnetic field at the center of an a circular coil of N identical turns. If the same current is
equilateral triangular loop of side 1 m which is carrying a passed in both, the ratio of the magnetic field at the
current of 10 A is : [10 April 2019, II] central of the loop (B1) to that at the centre of the coil
[Take mo = 4p×10–7 NA–2] B
(BC), i.e., L will be: [9 Jan 2019, II]
(a) 18 mT (b) 9 mT (c) 3 mT (d) 1 mT BC
44. A square loop is carrying a steady current I and the
1
magnitude of its magnetic dipole moment is m. If this (a) N (b)
square loop is changed to a circular loop and it carries the N
same current, the magnitude of the magnetic dipole moment 1
(c) N2 (d)
of circular loop will be : [10 April 2019, II] N2
48. The dipole moment of a circular loop carrying a current I,
m 3m 2m 4m is m and the magnetic field at the centre of the loop is B1.
(a) (b) (c) (d)
p p p p When the dipole moment is doubled by keeping the current
Moving Charges and Magnetism P-317

current constant, the magnetic field at the centre of the 53. Consider two thin identical conducting wires covered
B with very thin insulating material. One of the wires is
loop is B2. The ratio 1 is: [2018] bent into a loop and produces magnetic field B1, at its
B2
centre when a current I passes through it. The ratio B1 :
1 B2 is: [Online April 12, 2014]
(a) 2 (b) 3 (c) 2 (d)
2 (a) 1 : 1 (b) 1 : 3 (c) 1 : 9 (d) 9 : 1
49. A Helmholtz coil has pair of loops, each with N turns and 54. A parallel plate capacitor of area 60 cm2 and separation 3
radius R. They are placed coaxially at distance R and the mm is charged initially to 90 mC. If the medium between the
same current I flows through the loops in the same plate gets slightly conducting and the plate loses the charge
direction. The magnitude of magnetic field at P, midway initially at the rate of 2.5 × 10–8 C/s, then what is the magnetic
between the centres A and C, is given by (Refer to figure): field between the plates ?
[Online April 15, 2018] [Online April 23, 2013]
(a) 2.5 × 10 T –8 (b) 2.0 × 10–7 T
(c) 1.63 × 10–11 T (d) Zero
2R
A P C 55. A current i is flowing in a straight conductor of length L.
The magnetic induction at a point on its axis at a distance
R L
from its centre will be : [Online April 22, 2013]
4
4 N m0 I 8 N m0 I 4 N m0 I 8 N m0 I
(a) (b) (c) (d) m0i
53/ 2 R 53/ 2 R 51/ 2 R 51/ 2 R (a) Zero (b)
2pL
50. A current of 1A is flowing on the sides of an equilateral
triangle of side 4.5 × 10–2m . The magnetic field at the m 0i 4m0i
(c) (d)
centre of the triangle will be: [Online April 15, 2018] 2L 5pL
(a) 4 × 10–5Wb/m2 (b) Zero 56. Choose the correct sketch of the magnetic field lines of a
(c) 2 × 10–5Wb/m2 (d) 8 × 10–5Wb/m2 circular current loop shown by the dot e and the cross
51. Two identical wires A and B, each of length 'l', carry the Ä. [Online April 22, 2013]
same current I. Wire A is bent into a circle of radius R
and wire B is bent to form a square of side 'a'. If BA and
BB are the values of magnetic field at the centres of the
(a) (b)
B
circle and square respectively, then the ratio A is:
BB
[2016]

p2 p2 p2 p2
(a) (b) (c) (d)
16 8 2 8 16 2 (c) (d)
52. Two long current carrying thin wires, both with current I,
are held by insulating threads of length L and are in
equilibrium as shown in the figure, with threads making 57. An electric current is flowing through a circular coil of
an angle 'q' with the vertical. If wires have mass l per unit radius R. The ratio of the magnetic field at the centre of
length then the value of I is : the coil and that at a distance 2 2R from the centre of
(g = gravitational acceleration) [2015] the coil and on its axis is : [Online April 9, 2013]
pgL (a) 2 2 (b) 27 (c) 36 (d) 8
(a) 2 tan q
µ0 58. A charge Q is uniformly distributed over the surface of
non-conducting disc of radius R. The disc rotates about
plgL L an axis perpendicular to its plane and passing through its
(b) tan q
µ0 q centre with an angular velocity w. As a result of this
rotation a magnetic field of induction B is obtained at
plgL
(c) sin q the centre of the disc. If we keep both the amount of charge
µ0 cos q placed on the disc and its angular velocity to be constant
plgL and vary the radius of the disc then the variation of the
(d) 2sin q I I magnetic induction at the centre of the disc will be
µ0 cos q
represented by the figure : [2012]
P-318 Physics

(a) 2.5 × 10–7 T southward


(b) 5 × 10–6 T northward
(c) 5 × 10–6 T southward
B
B (d) 2.5 × 10–7 T northward
(a) (b) 62. A long straight wire of radius a carries a steady current i.
The current is uniformly distributed across its cross
R
R section. The ratio of the magnetic field at a/2 and 2a is
[2007]
(a) 1/2 (b) 1/4 (c) 4 (d) 1
B B 63. A current I flows along the length of an infinitely long,
(c) (d) straight, thin walled pipe. Then [2007]
(a) the magnetic field at all points inside the pipe is the
R R same, but not zero
59. A current I flows in an infinitely long wire with cross section (b) the magnetic field is zero only on the axis of the
in the form of a semi-circular ring of radius R. The magnitude pipe
of the magnetic induction along its axis is: [2011] (c) the magnetic field is different at different points
m0 I m0 I m0 I m0 I inside the pipe
(a) (b) (c) (d) (d) the magnetic field at any point inside the pipe is zero
2p2 R 2pR 4pR p2R
60. Two long parallel wires are at a distance 2d apart. They 64. Two identical conducting wires AOB and COD are placed
carry steady equal currents flowing out of the plane of at right angles to each other. The wire AOB carries an
the paper as shown. The variation of the magnetic field B electric current I1 and COD carries a current I2. The
along the line XX' is given by [2010] magnetic field on a point lying at a distance d from O, in
a direction perpendicular to the plane of the wires AOB
B and COD, will be given by [2007]
1
m0 m 0 æ I1 + I 2 ö 2
(a) X X¢ (a) ( I12 + I 2 2 ) (b) ç ÷
2pd 2p è d ø
1

d d
(c)
m0
2pd 1
(
I 2 + I 22 2 ) (d)
m0
2 pd
( I1 + I 2 )
65. A long solenoid has 200 turns per cm and carries a current
B i. The magnetic field at its centre is 6.28 × 10–2 Weber/m2.
Another long solenoid has 100 turns per cm and it carries
(b) X X¢ i
a current . The value of the magnetic field at its centre is
3
d d [2006]
(a) 1.05 × 10–2 Weber/m2 (b) 1.05 × 10–5 Weber/m2
B
(c) 1.05 × 10–3 Weber/m2 (d) 1.05 × 10–4 Weber/m2
66. Two concentric coils each of radius equal to 2 p cm are
(c) X X¢
placed at right angles to each other. 3 ampere and 4 am-
pere are the currents flowing in each coil respectively.
d d The magnetic induction in Weber/m2 at the centre of the
B coils will be (m0 = 4p ´10-7 Wb / A.m) [2005]

(a) 10 -5 (b) 12 ´ 10 -5
(d) X X¢
(c) 7 ´ 10 -5 (d) 5 ´ 10 -5
67. A current i ampere flows along an infinitely long straight
d d
thin walled tube, then the magnetic induction at any point
61. A horizontal overhead powerline is at height of 4m from inside the tube is [2004]
the ground and carries a current of 100A from east to west. m 0 2i
The magnetic field directly below it on the ground is (a) . tesla (b) zero
4p r
(m0 = 4p×10 –7 Tm A–1) [2008] 2i
(c) infinite (d) tesla
r
Moving Charges and Magnetism P-319

68. A long wire carries a steady current. It is bent into a circle æ ˆj kˆ ö


of one turn and the magnetic field at the centre of the coil (a) abI , along ç + ÷
ç 2 2 ÷ø
is B. It is then bent into a circular loop of n turns. The è
magnetic field at the centre of the coil will be æ ˆj kˆ ö
[2004] (b) 2abI , along ç + ÷
ç 2 2 ÷ø
(a) 2n B (b) n2 B (c) nB (d) 2 n2 B è
69. The magnetic field due to a current carrying circular loop æ ˆj 2kˆ ö
of radius 3 cm at a point on the axis at a distance of 4 cm (c) 2abI , along ç + ÷
ç 5 5 ÷ø
from the centre is 54 mT. What will be its value at the è
centre of loop? [2004] æ ˆj 2kˆ ö
(a) 125 mT (b) 150 mT (c) 250 mT (d) 75 mT (d) abI , along çç + ÷
70. If in a circular coil A of radius R, current I is flowing and in è 5 5 ÷ø
another coil B of radius 2R a current 2I is flowing, then the 74. A small circular loop of conducting wire has radius a and
ratio of the magnetic fields BA and BB, produced by them carries current I. It is placed in a uniform magnetic field B
will be [2002] perpendicular to its plane such that when rotated slightly
(a) 1 (b) 2 (c) 1/2 (d) 4 about its diameter and released, it starts performing simple
harmonic motion of time period T. If the mass of the loop is m
then : [9 Jan 2020, II]
Force and Torque on Current
TOPIC 3 pm
Carrying Conductor (a) T =
2m
(b) T =
IB 2 IB
71. A square loop of side 2a and carrying current I is kept is xz
2pm pm
plane with its centre at origin. A long wire carrying the (c) T = (c) T =
same current I is placed parallel to z-axis and passing IB IB
through point (0, b, 0), (b >> a). The magnitude of torque 75. Two wires A & B are carrying currents I1 and I2 as shown
on the loop about z-axis will be : [Sep. 06, 2020 (II)] in the figure. The separation between them is d. A third
wire C carrying a current I is to be kept parallel to them
2m 0 I 2 a 2 2m 0 I 2 a 2 b at a distance x from A such that the net force acting on it
(a) (b)
pb p(a 2 + b2 ) is zero. The possible values of x are : [10 April 2019, I]
m 0 I2 a 2b m 0 I2 a 2
(c) 2 2 (d)
2 p (a + b ) 2 pb
72. A square loop of side 2a, and carrying current I, is kept in
XZ plane with its centre at origin. A long wire carrying the
same current I is placed parallel to the z-axis and passing
through the point (0, b, 0), (b >> a). The magnitude of the
torque on the loop about z-axis is given by : æ I ö I2
(a) x = ç 1 ÷ d and x = d
[Sep. 05, 2020 (I)] è I1 - I2 ø (I1 + I2 )
m0 I 2a2 m 0 I 2 a3
(a) (b) æ I2 ö æ I2 ö
2 pb 2pb2 x=ç ÷ d and x = ç d
è (I1 - I2 ) ÷ø
(b)
è (I1 + I2 ) ø
2m 0 I 2 a 2 2m 0 I 2 a 3
(c) (d) æ I1 ö æ I2 ö
pb pb2 (c) x = çè (I + I ) ÷ø d and x = çè (I - I ) ÷ø d
73. A wire carrying current I is bent in the shape ABCDEFA as 1 2 1 2
shown, where rectangle ABCDA and ADEFA are
perpendicular to each other. If the sides of the rectangles I1d
(d) x=±
are of lengths a and b, then the magnitude and direction of (I1 - I2 )
magnetic moment of the loop ABCDEFA is :
76. A rectangular coil (Dimension 5 cm × 2.5 cm) with 100
[Sep. 02, 2020 (II)] turns, carrying a current of 3 A in the clock-wise
Z direction, is kept centered at the origin and in the X-Z
E plane. A magnetic field of 1 T is applied along X-axis. If
I
the coil is tilted through 45° about Z-axis, then the torque
F I
C on the coil is: [9 April 2019 I]
D (a) 0.38 Nm (b) 0.55 Nm
Y
O (c) 0.42 Nm (d) 0.27 Nm
b
A a B
X
P-320 Physics

77. A rigid square of loop of side ‘a’ and carrying current I2 is cm carries a current 1 of 12 A. Out of the following differ-
lying on a horizontal surface near a long current I1 carrying ent orientations which one corresponds to stable equilib-
wire in the same plane as shown in figure. The net force rium ? [Online April 9, 2017]
on the loop due to the wire will be: Z Z
[9 April 2019 I]
B
Bd c d
I1 I2 I I c I
(a) a b Y (b) I a Y
a b
X X
a Z Z
m II B
(a) Repulsive and equal to o 1 2 B
2p
mo I1I 2 a I d a I d
(b) Attractive and equal to (c) b Y (d) Y
3p b
I c I c
mo I1I 2 X
(c) Repulsive and equal to X
4p 82. Two coaxial solenoidsuur of different radius carry current I in
(d) Zero the same direction. F1 be the magnetic force on the inner
uur
78. A circular coil having N turns and radius r carries a current solenoid due to the outer one and F2 be the magnetic
I. It is held in the XZ plane in a magnetic field B. The force on the outer solenoid due to the inner one. Then :
torque on the coil due to the magnetic field is : [2015]
[8 April 2019 I] uur uur
(a) F1 is radially inwards and F2 = 0
Br 2 I uur uur
(a) (b) Bpr2I N (b) F1 is radially outwards and F2 = 0
pN uur uur
(c) F1 = F2 = 0
Bpr 2 I uur uur
(c) (d) Zero
N (d) F1 is radially inwards and F2 is radially outwards
79. An infinitely long current carrying wire and a small current 83. A rectangular loop of sides 10 cm and 5 cm carrying a
carrying loop are in the plane of the paper as shown. The current 1 of 12 A is placed in different orientations as shown
redius of the loop is a and distance of its centre from the
in the figures below :
wire is d (d>>a). If the loop applies a force F on the wire
z
then: [9 Jan. 2019 I]
I
B
I I
(A) y
I
d x
z

B
æ aö (B) I
(a) F = 0 (b) F µ çè ÷ø I y
d I
x I
æ a2 ö æ aö
2
z
(c) F µ ç 3 ÷ (d) F µ ç ÷
èd ø è dø B
I
80. A charge q is spread uniformly over an insulated loop of
I
radius r . If it is rotated with an angular velocity w with (C) I y
respect to normal axis then the magnetic moment of the I
loop is [Online April 16, 2018] x
z
1 4 3
(a) qwr 2 (b) qwr 2 (c) qwr 2 (d) qwr 2 B
2 3 2
81. A uniform magnetic field B of 0.3 T is along the positive Z- (D) I
I y
direction. A rectangular loop (abcd) of sides 10 cm × 5 I
x I
Moving Charges and Magnetism P-321

If there is a uniform magnetic field of 0.3 T in the positive


z direction, in which orientations the loop would be in (i) z
stable equilibrium and (ii) unstable equilibrium ?
1.5
[2015] I
(a) (B) and (D), respectively
(b) (B) and (C), respectively
(c) (A) and (B), respectively
B y
(d) (A) and (C), respectively
2.0
84. Two long straight parallel wires, carrying (adjustable)
current I1 and I2, are kept at a distance d apart. If the force x
‘F’ between the two wires is taken as ‘positive’ when the
–1.5
wires repel each other and ‘negative’ when the wires
attract each other, the graph showing the dependence of
‘F’, on the product I1 I2, would be : (a) 1.57 W (b) 2.97 W (c) 14.85 W (d) 29.7 W
[Online April 11, 2015] 87. Three straight parallel current carrying conductors are
F F shown in the figure. The force experienced by the middle
conductor of length 25 cm is: [Online April 11, 2014]

I1 = 30 A I2 = 20 A
(a) (b)
O I1I2 O I1I2

F F 5 cm
3 cm

(c) O (d)
I1I2 O I1I2

I = 10 A
85. A wire carrying current I is tied between points P and
Q and is in the shape of a circular arc of radius R due (a) 3 × 10–4
N toward right
–4
(b) 6 × 10 N toward right
to a uniform magnetic field B (perpendicular to the plane
of the paper, shown by xxx) in the vicinity of the wire. (c) 9 × 10–4 N toward right
If the wire subtends an angle 2q0 at the centre of the (d) Zero
circle (of which it forms an arc) then the tension in the 88. A rectangular loop of wire, supporting a mass m, hangs
wire is : [Online April 11, 2015] r
with one end in a uniform magnetic field B pointing out
IBR of the plane of the paper. A clockwise current is set up
(a)
2sin q0 B such that i > mg/Ba, where a is the width of the loop.
P Q
Then :
IBRq0
(b) [Online April 23, 2013]
sin q0
q0
(c) IBR R

IBR
(d) Q
sin q0 P
y
86. A conductor lies along the z-axis at -1.5 £ z < 1.5 m and
Ii
carries a fixed current of 10.0 A in -â z direction (see figure).
r a x
For a field B = 3.0 ´10-4 e-0.2x aˆ y T, find the power required S R
to move the conductor at constant speed to x = 2.0 m, mg
y = 0 m in 5 ´10-3 s. Assume parallel motion along the (a) The weight rises due to a vertical force caused by
x-axis. [2014] the magnetic field and work is done on the system.
(b) The weight do not rise due to vertical for caused by
the magnetic field and work is done on the system.
P-322 Physics

(c) The weight rises due to a vertical force caused by the mo I é b - a ù


magnetic field but no work is done on the system. (b)
4p êë ab úû
(d) The weight rises due to a vertical force caused by
the magnetic field and work is extracted from the mo I
(c) [2(b - a ) + p / 3(a + b)]
magnetic field. 4p
89. Currents of a 10 ampere and 2 ampere are passed through (d) zero
two parallel thin wires A and B respectively in opposite 92. Due to the presence of the current I1 at the origin:
directions. Wire A is infinitely long and the length of the (a) The forces on AD and BC are zero.
wire B is 2 m. The force acting on the conductor B, which (b) The magnitude of the net force on the loop is given
is situated at 10 cm distance from A will be
I1 I
[Online May 26, 2012] by mo [2(b - a ) + p / 3(a + b] .
(a) 8 × 10–5 N (b) 5 × 10–5 N 4p
(c) 8p × 10–7 N (d) 4p × 10–7 N (c) The magnitude of the net force on the loop is given
90. The circuit in figure consists of wires at the top and bottom m o II1
and identical springs as the left and right sides. The wire by (b - a).
at the bottom has a mass of 10 g and is 5 cm long. The wire 24 ab
is hanging as shown in the figure. The springs stretch 0.5 (d) The forces on AB and DC are zero.
cm under the weight of the wire and the circuit has a total 93. Two long conductors, separated by a distance d carry
resistance of 12 W. When the lower wire is subjected to a current I1 and I2 in the same direction. They exert a force F
static magnetic field, the springs, stretch an additional on each other. Now the current in one of them is increased
0.3 cm. The magnetic field is [Online May 12, 2012] to two times and its direction is reversed. The distance is
24 V also increased to 3d. The new value of the force between
them is [2004]
2F F F
(a) - (b) (c) –2 F (d) -
3 3 3
Magnetic 94. If a current is passed through a spring then the spring
field region
will [2002]
(a) expand (b) compress
5 cm (c) remains same (d) none of these
95. Wires 1 and 2 carrying currents i1 and i2 respectively are
(a) 0.6 T and directed out of page inclined at an angle θ to each other. What is the force on
(b) 1.2 T and directed into the plane of page a small element dl of wire 2 at a distance of r from wire 1 (as
(c) 0.6 T and directed into the plane of page shown in figure) due to the magnetic field of wire 1? [2002]
(d) 1.2 T and directed out of page (a) m 0 i1i2 dl tan q 1 2
Directions : Question numbers 91 and 92 are based on the 2 pr
following paragraph. m0
(b) i1i2 dl sin q i1 r i2
A current loop ABCD is held fixed on the plane of the paper as 2pr
shown in the figure. The arcs BC (radius = b) and DA (radius = m0 q dl
(c) i1i2 dl cos q
a) of the loop are joined by two straight wires AB and CD. A 2 pr
steady current I is flowing in the loop. Angle made by AB and m0
(d) i1i2 dl sin q
CD at the origin O is 30°. Another straight thin wire with steady 4 pr
current I1 flowing out of the plane of the paper is kept at the
origin. [2009] Galvanometer and its
B TOPIC 4 Conversion into Ammeter and
a A Voltmeter
I1 30° I
96. A galvanometer of resistance G is converted into a volt-
O meter of ragne 0 – 1V by connecting a resistance R1 in
D series with it. The additional resistance R1 in series with
b C it. The additional resistance that should be connected in
series with R1 to increase the range of the voltmeter to 0
91. The magnitude of the magnetic field (B) due to the loop – 2V will be : [Sep. 05, 2020 (I)]
ABCD at the origin (O) is :
(a) G (b) R1
m o I (b - a ) (c) R1 – G (d) R1 + G
(a)
24ab
Moving Charges and Magnetism P-323

97. A galvanometer is used in laboratory for detecting the 0 – 5 V. Therefore the value of shunt resistance required
null point in electrical experiments. If, on passing a to convert the above galvanometer into an ammeter of range
current of 6 mA it produces a deflection of 2°, its figure 0 – 10 mA is : [10 April 2019, I]
of merit is close to : [Sep. 05, 2020 (II)] (a) 500 W (b) 100 W (c) 200 W (d) 10 W
(a) 333° A/div. (b) 6 × 10–3 A/div. 102. A moving coil galvanometer has resistance 50 W and it
(c) 666° A/div. (d) 3 × 10–3 A/div. indicates full deflection at 4 mA current. A voltmeter is
98. A galvanometer coil has 500 turns and each turn has an made using this galvanometer and a 5 k W resistance. The
average area of 3 × 10–4 m2. If a torque of 1.5 Nm is required maximum voltage, that can be measured using this
to keep this coil parallel to a magnetic field when a current voltmeter, will be close to: [9 April 2019 I]
of 0.5 A is flowing through it, the strength of the field (in T) (a) 40 V (b) 15 V (c) 20 V (d) 10 V
is __________. [NA Sep. 03, 2020 (II)] 103. A moving coil galvanometer has a coil with 175 turns and
area 1 cm2. It uses a torsion band of torsion constant 10–
99. A galvanometer of resistance 100 W has 50 divisions on 6 N-m/rad. The coil is placed in a magnetic field B parallel
its scale and has sensitivity of 20 µA/division. It is to be
to its plane. The coil deflects by 1° for a current of 1mA.
converted to a voltmeter with three ranges, of 0–2V, 0–10
The value of B (in Tesla) is approximately:
V and 0–20 V. The appropriate circuit to do so is :
[9 April 2019, II]
[12 April 2019, I]
(a) 10–4 (b) 10–2 (c) 10–1 (c) 10–3
104. The resistance of a galvanometer is 50 ohm and the
maximum current which can be passed through it is 0.002
(a)
A. What resistance must be connected to it order to
convert it into an ammeter of range 0 – 0.5 A?
[9 April 2019, II]
(a) 0.5 ohm (b) 0.002 ohm
(b)
(c) 0.02 ohm (d) 0.2 ohm
105. The galvanometer deflection, when key K1 is closed but
K2 is open, equals θ 0 (see figure). On closing K2 also
and adjusting R2 to 5W, the deflection in galvanometer
(c)
θ0
becomes . The resistance of the galvanometer is, then,
5
given by [Neglect the internal resistance of battery]:
(d) [12 Jan 2019, I]
K2 R2
100. A moving coil galvanometer, having a resistance G,
produces full scale deflection when a current Ig flows
R1 = 220 W
through it. This galvanometer can be converted into (i)
an ammeter of range 0 to I0 (I0 > Ig) by connecting a shunt
resistance RA to it and (ii) into a voltmeter of range 0 to V
(V=GI0) by connecting a series resistance Rv to it. Then,
G
[12 April 2019, II]
2
æ I0 - I g
2
ö RA æ I g ö
(a) R A RV = G ç ÷÷ and =ç ÷÷
ç Ig
è ø RV çè I 0 - I g ø
2 K1
R æ Ig ö
(b) RA RV = G and A = çç
2
÷÷ (a) 5W (b) 22W
RV è I 0 - I g ø
2 (c) 25W (d) 12W
æ Ig
2
ö RA æ I 0 - I g ö
106. A galvanometer, whose resistance is 50 ohm, has 25
(c) R A RV = G ç ÷÷ and =ç ÷÷
ç I0 - I g
è ø RV çè I g ø divisions in it. When a current of 4 × 10–4 A passes through
it, its needle (pointer) deflects by one division. To use this
RA Ig
(d) RA RV = G 2 and = galvanometer as a voltmeter of range 2.5 V, it should be
RV ( I0 - I g ) connected to a resistance of :
101. A moving coil galvanometer allows a full scale current [12 Jan 2019, II]
of 10– 4 A. A series resistance of 2 MW is required to (a) 250 ohm (b) 200 ohm
convert the above galvanometer into a voltmeter of range
(c) 6200 ohm (d) 6250 ohm
P-324 Physics

107. A galvanometer having a resistance of 20 W and 30 114. To know the resistance G of a galvanometer by half
division on both sides has figure of merit 0.005 ampere/ deflection method, a battery of emf VE and resistance R
division. The resistance that should be connected in series is used to deflect the galvanometer by angle q. If a shunt
such that it can be used as a voltmeter upto 15 volt, is: of resistance S is needed to get half deflection then G, R
[11 Jan 2019, II] and S related by the equation: [Online April 9, 2016]
(a) 100 W (b) 120 W (c) 80 W (d) 125 W (a) S (R + G) = RG (b) 2S (R + G) = RG
108. A galvanometer having a coil resistance 100 W gives a full (c) 2G = S (d) 2S = G
scale deflection when a current of 1 mA is passed through 115. The AC voltage across a resistance can be measured
it. What is the value of the resistance which can convert using a : [Online April 11, 2015]
this galvanometer into a voltmeter giving full scale (a) hot wire voltmeter
deflection for a potential difference of 10 V? (b) moving coil galvanometer
[8 Jan 2019, II]
(c) potential coil galvanometer
(a) 10 kW (b) 8.9 kW
(d) moving magnet galvanometer
(c) 7.9 kW (d) 9.9 kW
116. In the circuit diagrams (A, B, C and D) shown below, R is
109. In a circuit for finding the resistance of a galvanometer by
a high resistance and S is a resistance of the order of
half deflection method, a 6 V battery and a high resistance
galvanometer r esistance G. The correct circuit,
of 11kW are used. The figure of merit of the galvanometer
corresponding to the half deflection method for finding
60mA/division. In the absence of shunt resistance, the
the resistance and figure of merit of the galvanometer, is
galvanometer produces a deflection of q = 9 divisions when
the circuit labelled as: [Online April 11, 2014]
current flows in the circuit. The value of the shunt K2
R
resistance that can cause the deflection of q/2 , is closest
to [Online April 16, 2018]
(a) 55W (b) 110W (c) 220W (d) 550W S
110. A galvanometer with its coil resistance 25W requires a G
current of 1mA for its full deflection. In order to construct
an ammeter to read up to a current of 2A, the approximate (A)
value of the shunt resistance should be
[Online April 16, 2018]
(a) 2.5 × 10–2W (b) 1.25 × 10–3W K1
K2
(c) 2.5 × 10–3W (d) 1.25 × 10–2W
111. When a current of 5 mA is passed through a galvanometer
having a coil of resistance 15 W , it shows full scale R
deflection. The value of the resistance to be put in series G
with the galvanometer to convert it into to voltmeter of S
range 0 - 10 V is [2017] (B)
(a) 2.535 ´10 W
3 (b) 4.005 ´10 W
3

(c) 1.985 ´ 10 W 3 (d) 2.045 ´ 103 W K1


112. A galvanometer having a coil resistance of 100 W gives a K2
full scale deflection, when a currect of 1 mA is passed
through it. The value of the resistance, which can convert S
this galvanometer into ammeter giving a full scale deflection G
for a current of 10 A, is : [2016]
R
(a) 0.1 W (b) 3W (c) 0.01W (d) 2W
(C)
113. A 50 W resistance is connected to a battery of 5V. A
galvanometer of resistance 100 W is to be used as an
ammeter to measure current through the resistance, for K1
this a resistance rs is connected to the galvanometer. Which
of the following connections should be employed if the S
R K2
measured current is within 1% of the current without the
G
ammeter in the circuit ? [Online April 9, 2016]
(a) rs = 0.5 W in series with the galvanometer (D)
(b) rs = 1 W in series with galvanometer
(c) rs = 1W in parallel with galvanometer
K1
(d) rs = 0.5 W in parallel with the galvanometer.
Moving Charges and Magnetism P-325

RS Ii
(a) Circuit A with G =
( R - S) R1 R2
(b) Circuit B with G = S
(c) Circuit C with G = S
RS G
(d) Circuit D with G =
( R - S)
R3
117. This questions has Statement I and Statement II. Of the (a) 107 W (b) 137 W (c) 107/2 W (d) 77 W
four choices given after the Statements, choose the one 119. A shunt of resistance 1 W is connected across a
that best describes into two Statements. galvanometer of 120 W resistance. A current of 5.5 ampere
Statement-I : Higher the range, greater is the resistance of gives full scale deflection in the galvanometer. The current
ammeter. that will give full scale deflection in the absence of the
shunt is nearly : [Online April 9, 2013]
Statement-II : To increase the range of ammeter, additional
(a) 5.5 ampere (b) 0.5 ampere
shunt needs to be used across it. [2013] (c) 0.004 ampere (d) 0.045 ampere
(a) Statement-I is true, Statement-II is true, Statement-II 120. In the circuit , the galvanometer G shows zero deflection.
is the correct explanation of Statement-I. If the batteries A and B have negligible internal resistance,
(b) Statement-I is true, Statement-II is true, Statement-II the value of the resistor R will be - [2005]
is not the correct explanation of Statement-I. 500 W
G
(c) Statement-I is true, Statement-II is false.
2V
(d) Statement-I is false, Statement-II is true.
12V B R A
118. To find the resistance of a galvanometer by the half
deflection method the following circuit is used with
resistances R1= 9970 W, R2 = 30 W and R3 = 0. The (a) 100 W (b) 200W (c) 1000 W (d) 500 W
deflection in the galvanometer is d. With R3 = 107 W the
121. A moving coil galvanometer has 150 equal divisions. Its
d current sensitivity is 10-divisions per milliampere and
deflection changed to . The galvanometer resistance is voltage sensitivity is 2 divisions per millivolt. In order that
2
each division reads 1 volt, the resistance in ohms needed
approximately : to be connected in series with the coil will be - [2005]
[Online April 22, 2013] (a) 105 (b) 103 (c) 9995 (d) 99995
P-326 Physics

r 2pm
1. (c) E = 300 ˆj V/cm = 3 ´ 104 V/m 4. (d) Pitch = (v cos q)T and T =
r qB
V = 6 ´ 106 iˆ 2pm
\ Pitch = (V cos q)
y qB
E
E = 300 j
2p æ 1.67 ´10-27 ö
V/cm = 3– ´ 104 V/m
x = (4 ´ 105 cos 60°) ç ÷÷ = 4 cm
0.3 çè 1.69 ´10-19 ø
e z
V
V = 6 ´ 106 i$ 5. (c) Time period of one revolution of proton, T =
2pm
r qB
B must be in +z axis. Here, m = mass of proton
r r r
qE + qV ´ B = 0 q = charge of proton
E = VB B = magnetic field.
Linear distance travelled in one revolution,
E 3 ´ 104 p = T(v cos q) (Here, v = velocity of proton)
\ B= = = 5 ´ 10-3 T
V 6 ´ 106
\ Length of region, l = 10 ´ (v cos q)T
Hence, magnetic field B = 5 × 10–3 T along +z direction.
2. (c) In uniform magnetic field particle moves in a circular 2pm
Þ l = 10 ´ v cos 60° ´
path, if the radius of the circular path is 'r', particle will not qB
hit the screen.
20pmv 20 ´ 3.14 ´ 1.67 ´ 10 -27 ´ 4 ´ 105
Þl= =
qB 1.6 ´ 10 -19 ´ 0.3
Þ l = 0.44 m
d

6. (a)
mv é mv 2 ù
r= êQ = qvB0 ú
qB0 ë r û As we know, magnetic force F = qvB = ma
Hence, minimum value of v for which the particle will not r æ qvB ö
\ a =ç
hit the screen. è m ÷ø perpendicular to velocity..
qB0 d
v=
m 2KE 2 ´ e ´ 106
\ Also v = =
3. (a) [Given: q = 1mC = 1 ´ 10-6 C; m m
r
V = (2iˆ + 3 ˆj + 4kˆ) m/s and qvB eB 2 ´ e ´ 106
r \ a= =
B = (5iˆ + 3 ˆj - 6kˆ) ×10 –3 T ] m m m
iˆ ˆj kˆ
r r r
3
-6 -3
F = q(V ´ B) = 10 ´ 10 2 3 4 æ 1.6 ´ 10 –19 ö 2
\ 1012 =ç –27 ÷
. 2 ´ 10 3 B
5 3 -6 è 1.67 ´ 10 ø

= (-30iˆ + 32 ˆj - 9kˆ) ´ 10-9 N \B;


1
´ 10–3 T = 0.71 mT (approx)
r 2
\ F = ( -30iˆ + 32 ˆj - 9kˆ)
Moving Charges and Magnetism P-327

7. (d) As particle is moving along a circular path


9.1
mv ´ 10 -10
\R = ...(i) 0.16 3
qB r= -1
= ´ 10 -4
10 .4
Path is straight line, then
qE = qvB = 7.5 ´ 10 -4
E 12. (BONUS)
E = vB Þ v = ....(ii) Assuming particle enters from (0, d)
B
From equation (i) and (ii) mv r
r= , d=
qB 2
´ ( 0.5) ´ 0.5 ´10 –2
–19 2
qB2 R 1.6 ´ 10
m= =
E 100 (0, d)
\ m = 2.0 × 10–24 kg
8. (b)
mv 2mK.E. (0, 0) 30° V
9. (b) As mvr = qvB Þ r = = Fm
qB qB r/2
r
1
[As : mv2 = K.E. C
2
qVB é - 3i - j ù
Þ m 2 v2 = 2m K.E. a= ê ú
m ë 2 û
Þ mv = 2m K.E.] this option is not given in the all above four choices.
For proton, electron and a-particle, 13. (b) As we know, radius of circular path in magnetic field
mHe = 4mp and mp >> me 2Km
Also aHe = 2qp and qp = qe r=
qB
\ As KE of all the particles is same then,
2Km e
m For electron, re = ....(i)
ra eB
q
\ rHe = rp > re 2Kmp
For proton, rp = ....(ii)
mv eB
10. (a) Radius of the circular path will be r =
qB 2K4m p 2Km p
2Km a
2mKE For a particle, ra = = = ...(iii)
Þ r= (Q p = mv = 2mKE ) qa B 2eB eB
qB \ r e < rp = ra (Q me < mp)
Q KE = qDV
14. (b) The force is parallel to the direction
2mqDV m of current in magnetic field,
\ r= Þrµ
qB q r
hence F = q(v ´ B)
rp 1 According to Fleming's left hand rule,
\ =
ra 2 F
mv
11. (d) Radius of the path (r) is given by r = I v
qB e

r=
2mk
eB
(Q p = mv = 2mk ) BÄ

we have, the direction of motion of charge is towards the


2meV
= (Q k = eV) wire.
eB 15. (d) According to question, as the test charge experiences
no net force in that region i.e., sum of electric force
2m 2 ´ 9.1 ´ 10 -31 r r r
V (500) (Fe = qE) and magnetic forces [Fm = q(v ´ B] will be zero.
e 1.6 ´ 10 -19
r= = Hence, Fe + Fm = 0
B 100 ´ 10-3
P-328 Physics

r r 20. (b) The centripetal force is provided by the magnetic force


Fe = -q(v ´ B)
mv 2 mv m
= - B0 v0 é( 3iˆ - ˆj + 2kˆ ) ´ ( i + 2jˆ - 4kˆ ) ù \ = qvB Þ r =
Bq
\ rµ
q
ë û R
= - B v (14 j + 7k )
ˆ ˆ mp md ma
0 0 \ rp : rd : ra = : :
16. (b) Q F = qE and F = qvB qp qd qa
\ E = vB
s
= 1 : 2 :1
And Gauss's law in Electrostatics E = Thus we have, ra = rp < rd
e0 21. (d) When a charged particle enters the magnetic field
s in perpendicular direction then it experience a force in
E= = vB Þ s = e vB
e0 0 perpendicular direction.
i.e. F = Bqv sinq
σ1= – σ 2
Due to which it moves in a circular path.
17. (d) From figure, sin a = d/R 22. (b) As charge on both proton and deuteron is same i.e. 'e'
Energy acquired by both, E = eV
For Deuteron.
a R
1
Kinetic energy, mV2 = eV
a 2
[V is the potential difference]
d
2eV
v= md
mv 2 But md = 2m
And we know, = qvB
R 2eV eV
mv Therefore, v = =
Þ R= 2m m
qB
mv
dqB Radius of path, R =
\ sin a = eB
mv Substituting value of 'v' we get
q éQ qV = 1 mv 2 ù 2m
ev
sin a = Bd ê ú
2mV ë 2 û m
R=
18. (c) The applied magnetic field provides the required eB
centripetal force to the charge particle, so it can move in ev
m
d R m ...(i)
circular path of radius =
2 2 eB
For proton :
mv 2 1
\ Bqv = mV 2 = eV
d/2 2
2mv 2eV
or, B= V=
qd m
Time interval for which a uniform magnetic field is applied 2eV
m
d mV m
p. Radius of path, R¢ = =
eB eB
Dt = 2
v R
R¢ = 2 ´ [From eq. (i)]
(particle reverses its direction after time Dt by covering 2
semi circle). R
R¢ =
2
pd
Dt = ® æ ® ®ö
2v 23. (a) F = q ç V ´ B ÷
19. (c) Since particle is moving undeflected è ø
-6
= 2 ´10 éë(2iˆ + 3 ˆj ) ´10 ´ 2 ˆj ùû
6
So, q E = qvB

E 104 = 2 ´ 4kˆ = 8 N in Z-direction.


ÞB= = = 103 wb / m 2
V 10
Moving Charges and Magnetism P-329

30. (b) Due to electric field, it experiences force and


E 7.7 ´ 103 accelerates i.e. its velocity decreases.
24. (c) As velocity v = = = 55 km/s
B 0.14 31. (b) The workdone, dW = Fds cosq
25. (a) The charge experiences both electric and magnetic The angle between force and displacement is 90°.
force. Therefore work done is zero.
Electric force, Fe = qE × ×
r r
×

Magnetic force, Fm = q ( v ´ B )
ur ur r ur
\ Net force, F = q é E + v ´ B ù × × ×
ë û
é F
iˆ ˆj kˆ ù
ê ú
= q ê 3$i + $j + 2k$ + 3 4 1 ú ×
S
×

ê 1 1 -3 ú 32. (a) When a moving charged particle is subjected to a


ë û
perpendicular magnetic field, then it describes a circular
= q é3i + j + 2k + i ( -12 - 1) - $j ( -9 - 1) + k ( 3 - 4 )ù
$ $ ˆ $
ë û path of radius.
é ˆ $ $ $ $
= q ë3i + j + 2k - 13i + 10 j - k û $ ù p
r=
qB
= q éë -10i$ + 11$j + k$ ùû where q = Charge of the particle
p = Momentum of the particle
Fy = 11qjˆ B = Magnetic field
Thus, the y component of the force. Here p, q and B are constant for electron and proton, therefore
26. (b) As velocity is not changing, charge particle must go the radius will be same.
undeflected, then 33. (a) The time period of a charged particle of charge q and
qE = qvB 2 pm
mass m moving in a magnetic field (B) is T =
E qB
Þ v=
B Clearly time period is independent of speed of the particle.
Also,
r r
E´B E B sin q
= 34. (d) v
2
B B2
E B sin 90° E r
= 2
= = |v| =v + +q
B B
27. (b) When a charged particle enters a magnetic field at a
direction perpendicular to the direction of motion, the path
of the motion is circular. In circular motion the direction of
velocity changes at every point (the magnitude remains
Length of the circular path, l = 2pr
constant).
Therefore, the tangential momentum will change at every q qv
point. But kinetic energy will remain constant as it is given Current, i = =
T 2 pr
1
by mv 2 and v2 is the square of the magnitude of velocity Magnetic moment M = Current × Area
2
which does not change. qv
= i ´ pr 2 = ´ pr 2
28. (b) The charged particle will move along the lines of 2 pr
electric field (and magnetic field). Magnetic field will exert 1
M = q×v×r
no force. The force by electric field will be along the lines 2
of uniform electric field. Hence the particle will move in a mv
straight line. Radius of circular path in magnetic field, r =
29. (c) Equating magnetic force to centripetal force, qB
1 mv mv 2
mv 2 \M = qv ´ ÞM =
= qvB sin 90º 2 qB 2B
r r r
mv Bqr Direction of M is opposite of B therefore
Þ = Bq Þ v = r
r m r - mv 2 B
Time to complete one revolution, M =
2pr 2pm 2B 2
T= = (By multiplying both numerator and denominator by B).
v qB
P-330 Physics

p 3p Magnetic field at point B (outside)


35. (d) Given : IA = 2 A, RA = 2 cm, q A = 2p - = m 0i
2 2 BB =
p 5p 2p ( 2 a )
IB = 3 A, RB = 4 cm, q B = 2p - =
3 3 m 0i
m Iq BA 6pa = 4 = 2
Using, magnetic field, B = 0 =
4pR \ BB m 0i 6 3
3p 2p (2 a)
2´ ´ 4 38. (b) Magnetic field inside the solenoid is given by
B A I A q A RB 2 6
= ´ = = B = µ0nI .... (i)
BB I B q B RA 5p 5
3´ ´ 2 Here, n = number of turns per unit length
3

36. (c)

30° a
30°

The path of charge particle is circular. The maximum


I
R
3a possible radius of electron =
2 2
Magnetic field due to one side of hexagon mVmax R
\ =
m0 I qB 2
B= (sin 30° + sin 30°)
3a qBR eRm 0 nI
4p Þ Vmax = = (using (i))
2 2m 2m
m0 I æ 1 1 ö m0 I
ÞB= çè + ÷ø =
2 3a 2 2 2 3ap
Now, magnetic field due to one hexagon coil
m0 I 39. (a)
B = 6´
2 3ap
Again magnetic field at the centre of hexagonal shape coil
of 50 turns, B0 = B1 + B2 + B3 + B4
m0 I é 10 ù m I m I m I
= 0 [ sin 90° – sin 45° ] + 0 + 0
B = 50 ´ 6 ´
2 3ap êëQ a = 100 = 0.1 m úû 4 pR 2 R 4pR
[sin45° + sin90°]
150m 0 I m0 I m I æ 1 ö m0 I m0I æ 1 ö
or, B = = 500 3 = – 0 ç1 –
3 ´ 0.1 ´ p p 4 pR è ÷+ + ç1 + ÷
2 ø 2R 4pR è 2ø
37. (a) Let a be the radius of the wire uuur m I æ 1 ö
e
Magnetic field at point A (inside) Be0 = 0 ç p + ÷
2 pR è 2ø
a
m 0 ir m 0 i 3 m 0 i a m 0i ® ®
40. (a) B = B + B
®
BA = = = = 1 2
2 pa 2 2 pa 2 pa 2 6 6 pa
= ( )
µ0 æ i º ˆ i º ˆ ö
. ç .k + -k ÷ = 0
2p è d d ø
41. (b) If q is the charge on the ring, then
Moving Charges and Magnetism P-331

q qw æ 4a2 ö
i= = M1 = ( I )(p) ç 2 ÷
T 2p
è p ø
Magnetic field,
m0 i 4Ia 2
B= M1 =
2R p
æ qw ö 4M
m0 ç ÷ M1 = (Q M = Ia2)
= è 2p ø p
2R 45. (c) Let I be the current in each wire. (directed inwards)
Magnetic field at ‘O’ due to LP and QM will be zero.
æ m 0 ö qw
or 3.8 × 10–9 = çè 4p ÷ø R = 10 ( )
-7 q ´ 40p
0.10
i.e., B0 = BPS + BQN
m 0i m 0i
\ q = 3 × 10–5 C. \ Net magnetic field B0 = +
4 pd 4 pd
µ0 i
42. (b) B = , (sin a + sin b) m0i 2 ´ 10 -7 ´ i
4p r or 10 -4 = +
2 pd 4 ´ 10 -2
Here r = 52 - 32 = 4 cm \ i = 20 A and the direction of magnetic field is
a = b = 37° perpendicular into the plane
-7 5
\ B = 10 ´ 2sin 37° = 1.5 × 10–5 T
4

43. (a) 46. (d)

æ 1ö
r = ç ÷ (a sin 60)
è 3ø There will be no magnetic field at O due to wire
PQ and RS
a 3 æ a ö
r= ´ = Magnetic field at ‘O’ due to arc QR
3 2 çè 2 3 ÷ø
æ pö
.I
ém l ù
B0 = 3 ê 0 (sin 60° + sin 60°) ú
m0 çè 4 ÷ø
=
ë 4pr û 4p r1
3m 0 l æ 3 ö 9 æ m 0l ö Magnetic field at ‘O’ due to are PS
= ´ (2) ç =
æ a ö ç 2 ÷÷ 2 çè pa ÷ø æ pö
è ø .I
4p ç ÷ m çè 4 ÷ø
è2 3ø = 0
4p r2
9 ´ 2 ´ 10 -7 ´ 10 \ Net magnetic field at ‘O’
= = 18 mT
1
é ù
44. (d) Let a be the area of the square and r be the radius of m0 ê 1 1 ú
B = 4p ( p / 4 ) ´ 10 ê -
circular loop.
æ 2a ö
(
êë 3 ´ 10
-2
) (
5 ´ 10 -2 ) ú
úû
2pr = 4a Þ r = ç ÷ r p
è pø
Þ| B |= ´ 10 -5 T » 1 ´ 10 -5 T
For square 3
M = (I) a2 47. (d)
For circular loop r
Loop R Coil
M1 = (I)pr2
L = 2p R L = N × 2pr
P-332 Physics

R
R= Nr Þ r = 51. (b) Case (a) :
N
m0i m 0 Ni m 0 Ni m 0 N i 2
µ0 I µ I
BLoop = Bcoil = = = BA= ´ 2p = 0 ´ 2p (Q 2pR = l)
2R 2r æ Rö 2R 4p R 4 p l / 2p
2ç ÷
è Nø µ0 I
= ´ (2p) 2
B 1 4p l
\ L= 2 Case (b) :
BC N
m0 I
48. (c) Magnetic field at the centre of loop, B1 = 45°
2R a BB
Dipole moment of circular loop is m = IA
a/2
m1 = I.A = I.pR2 {R = Radius of the loop}
If moment is doubled (keeping current constant) R be- µ0 I
comes BB = 4 × [sin 45° + sin 45°]
2R 4p a / 2
µ0 I 2 µ0 I 64 m0 I
( )
2 = 4´ ´ ´ = ´ = 32 2 [4a = l]
m 2 = I.p 2R = 2.IpR 2 = 2m1 4p l / 8 2 4p l 2 4pl
BA p2
m0 I Þ =
B2 = BB 8 2
2 ( 2R ) 52. (d) Let us consider 'l' length of current carrying wire.
At equilibrium
m0 I T cos q = lgl
B 2R
\ 1 = = 2
B2 m0 I q L
2 ( 2R ) T
49. (b) Point P is situated at the mid-point of the line joining T cos q
the centres of the circular wires which have same radii (R).
r Lsin q Lsin q FB
The magnetic fields ( B ) at P due to the currents in the T sin q
wires are in same direction. (ll)g
Magnitude of magnetic field at point, P m0 I ´ Il é FB m0 2 I ´ I ù
and T sin q = êQ =
4p 2l sin q úû
ì ü m0 NIR 2 8m0 NI 2 p 2L sin q ë l
m 0 NIR 2
B = 2 ïï ï
3/ 2 ï
= = pl gL
í æ 2 R2 ö ý 53 / 2 53/ 2 R Therefore, I = 2sin q
ï2ç R + ÷ ï u 0 cos q
4 ø þï 8
îï è m0 nI
53. (b) For loop B =
50. (a) Here, side of the triangle, l = 4.5 × 10–2 m, current, I = 1A 2a
magnetic field at the centre of the triangle ‘O’ B = ? where, a is the radius of loop.
m0 I
1 Then, B1 =
From figure, tan 60° = 3 = 2a
2d m I 2nA
Now, for coil B = 0 .
4p x 3
A
at the centre x = radius of loop
m
10 –2

2
m 0 2 ´ 3 ´ ( I / 3) ´ p ( a / 3 ) m0 .3I
B2 = . =
æ 4.5 ´ 10-2 ö
×

l 4p ( a / 3)3 2a
4.5

Þ d= =ç ÷m 60°
2 3 è 2 3 ø
l=

O B1 m I / 2a
B \ = 0
C B2 m0 .3I / 2a
B1 : B2 = 1: 3
m 0i
Magnetic field, B = (cos q1 + cos q 2 ) 54. (d) Magnetic field between the plates in this case is zero.
4pd 55. (a) Magnetic field at any point lies on axial position of
Putting value of µ = 4p × 10–7 and q1 and q2 we will get B = current carrying conductor B = 0
4 × 10–5 Wb/m2
Moving Charges and Magnetism P-333

56. (a) If magnetic field is perpendicular and into the plane of 62. (d) Since uniform current is flowing through a straight
the paper, it is represented by cross Ä and if the direction wire, current enclosed in the amperean path formed at a
of the magnetic field is perpendicular out of the plane of æ aö
distance r1 ç = ÷ is
the paper it is represented by dot e . è 2ø
57. (b) Given : Radius = R æ p r2 ö a/2
Distance x = 2 2R i = ç 12 ÷ ´ I , P1 P2
ç pa ÷
3/2 3/2 è ø
Bcentre æ x2 ö æ (2 2R)2 ö where I is total current
= çç1 + 2 ÷÷ = çç1 + ÷
Baxis è R ø è R 2 ÷ø Using Ampere circuital law,
uur
= (9)3/2 = 27 Ñò B × dl = m0i
58. (a) The magnetic field due to a disc is given as m0 ´ current enclosed
Þ B1 =
m 0 wQ 1 Path
B= i.e., B µ
2 pR R æ p r12 ö
m0 ´ ç 2 ÷ ´ I
59. (d) Let R be the radius of semicircular ring. Let an ç pa ÷ m ´Ir
Þ B1 = è ø = 0 21
elementary length dl is cut for finding magnetic field. So,
2p r1 2p a
dq
dl = Rdq. Current in a small element, dI = I Now, magnetic field induction at point P2,
p
m I m I
Magnetic field due to the element B2 = 0 × = 0 .
m 2dI m I 2p (2a ) 4pa
dB = 0 = 02
4p R 2p R B1 m 0 Ir1 4pa
\ = ´
The component dB cos q, of the field is cancelled by another B2 2pa 2 m 0 I
opposite component.
Therefore, a

B1 2 r1 2 = 1.
Þ = =
B2 a a
63. (d) There is no current inside the pipe. From Ampere’s
r uur
circuital law Ñò B × dl = m0 I
dB
QI=0
\ B=0
p
m0 I m0I 64. (c) The direction of magnetic field induction due to
Bnet = ò dB sin q =
2p 2
R
ò sin qd q =
p2 R current through AB and CD at P are indicated as B1 and
0
B2. The magnetic fields at a point P, equidistant from AOB
60. (a) The magnetic field varies inversely with the distance and COD will have directions perpendicular to each other,
1 as they are placed normal to each other.
for a long conductor. That is, B µ
d A P D
so, graph in option (a) is the correct one. B1 B2
61. (c) The magnetic field is
I1 d I2
m 2I 2 ´ 100
B= 0 = 10-7 ´ = 5 × 10–6 T
4p r 4 O

W N

100A C B
Magnetic field at P due to current through AB,
m I
B1 = 0 1
4m 2 pd
Magnetic field at P due to current through CD,
E
S m I
Ground B2 = 0 2
2 pd
B
\ Resultant field, B = B12 + B22
Current flows from east to west. Point is below the power
line, using right hand thumb rule, the magnetic field is 2
directed towards south.
æ m ö
\ B= ç 0 ÷
è 2pd ø (I
2
1 + I 22 )
P-334 Physics

( )
m0 2 1/ 2 Magnetic field at the centre of loop is
or, B = I1 + I 22 m i
2 pd B¢ = 0
65. (a) Magnetic field due to long solenoid is given by B = m0nI 2a
In first case B1 = m0n1I1 B × ( x 2 + a 2 )3/2
In second case, B2 = m0n2I2 \ B¢ =
a3
B m ni Put x = 4 & a = 3
\ 2 = 0 22
B1 m 0 n1i1 54(53 )
i Þ B¢ = = 250 µT
100 ´ 3´ 3´ 3
B2 3 70. (a) Magnetic field induction at the centre of current
Þ =
6.28 ´ 10 -2 200 ´ i carrying circular coil of radius r is
6.28 ´ 10 -2 m I
Þ B2 = = 1.05 ´ 10 -2 Wb/m 2 B = 0 ´ 2p
6 4p R
66. (d) m I
(1) Here B A = 0 ´ 2p
4p R
m0 2I
and BB = ´ 2p
(2) 4p 2 R
BA I/R
Þ = =1
BB 2 I / 2 R
The magnetic field due to circular coil (1) is
µi m0i1 m ´ 3 ´ 102 71. (b) y
B1 = 0 1 = -2 = 0
2r 2(2p ´ 10 ) 4p
Magnetic field due to coil (2) (0,b,0)
Total magnetic field
m0 i2 m 0 ´ 4 ´ 10 2
B2 = =
2(2p ´ 10 -2 ) 4p
I
Total magnetic field, B = B12 + B22 x
m
= 0 × 5 × 102 z
4p
y
Þ B = 10–7 × 5 × 102
Þ B = 5 × 10–5 Wb/m2
67. (b) From Ampere’s circuital law
r uur r
ò × dl = m0i
B
b F
Þ B × 2pr = m0i Fcosq
Here i is zero, for r < R, whereas R is the radius r = b2 + a2
\ B=0
a a x
68. (b) Magentic field at the centre of a circular coil of radius F
Fcosq
R carrying current i is B = m 0 i
2R m0 I
Force, F = BI 2 a = I ´ 2a
The circumference of the first loop = 2pR. If it is bent into 2 pr
n circular coil of radius r¢.
n × (2pr¢) = 2pR m0 I 2a
Force, F =
Þ nr¢ = R ...(1) p b2 + a2
n ×m0i
New magnetic field, B¢ = ...(2) Torque, t = F1 ´ Perpendicular distance = F cos q ´ 2a
2 r¢
From (1) and (2),
nm i × n m0 I 2 a b
B¢ = 0 = n2B = ´ ´ 2a
2 pR p b2 + a2 b2 + a 2
69. (c) The magnetic field at a point on the axis of a circular
loop at a distance x from centre is, 2m 0 I 2 a 2b
Þt=
m 0i a 2 p(a 2 + b2 )
B=
2( x 2 + a 2 )3 / 2
Moving Charges and Magnetism P-335

r µ I µ0 I2
2m 0 I 2 a 2 ÞF= 0 1 + =0
If b >> a then t = 2px 2p(d - x)
pb
72. (c) Torque on the loop, µ0 I1 µ0 I2
=
2px 2p(x - d)
t = M ´ B = MB sin q = MB sin 90° I1x – I1d = I2x
m0 I Id
Magnetic field, B = x= 1
2pd I1 - I2
æm I ö Two cases may be possible if I1 > I2 or I2 > I1
\ t = I1 (2a)2 ç 0 2 ÷ sin 90°
è 2pd ø 76. (d) t = MB sin45° = N (iA) B sin 45°

2m I 2 a 2 1
2m0 I1 I 2
= ´ a2 = 0 = 100 ´ 3(5 ´ 2.5) ´ 10-4 ´ 1 ´
pd pd 2
73. (b) Magnetic moment of loop ABCD, = 0.27 N-m
M1 = area of loop × current m 0 æ ii i2 i1i2 ö m ii
r 77. (c) F = 2 çè -
2 a ÷ø
´a = 012
p a 4p
M1 = (abI )( ˆj ) (Here, ab = area of rectangle)
r r r
Magnetic moment of loop DEFA, 78. (b) | t |=| m ´ B | [m = NIA]
r =NIA × B sin 90o [A = pr2]
M 2 = (abI )(i$) Þ t =NIpr2B
Net magnetic moment,
r r r r
M = M1 + M 2 Þ M = abI ($i + $j )
r æ ˆj kˆ ö
Þ |M | = 2abI ç + ÷
è 2 2ø
74. (c) Torque on circular loop, t = MB sin q
where, M = magnetic moment
B = magnetic field 79. (d)
Now, using t = Ia
\ t = MB sin q = Ia
mR 2 a
Þ pR 2 IBq =
2
mR 2
(Q m = IA and moment of inertia of circular loop, I = )
2
mR 2
Þ pR2IB q = wq
2
2pIB 2p 2pIB
Þ w= Þ =
m T m
2pm
Þ T= Force on one pole,
IB
m0 I
F= m´
2p d2 + x 2
Total force, Ftotal = 2F sin q
m 0 Im x
75. (d) = 2´ ´
2 2
2p d + a d + a2
2

m 0 Im x
=
p (d 2 + a 2 )
As net force on the third wire C is zero. Magnetic moment, M = Ipa2 = m × 2
P-336 Physics

m0 I a 2 9 ´ 10-3
or, Total force, Ftotal = = ´ [1 - e -0.4 ]
2(d 2 + a 2 ) 0.2
9×10 –3 ´ (0.33) 2.97×10 –3
m 0 Ia 2 = =
= [Q d >> a ] 2 2
2d 2 Power required to move the conductor is,
W
a2 P=
Clearly Ftotal µ 2 t
d 2.97 ´ 10-3
qv P= = 2.97 W
80. (a) Magnetic moment, m = IA = ( pr 2 ) (0.2) ´ 5 ´ 10-3
2 pr
87. (a) I1 = 30 A I = 10 A I2 = 20 A
qrw 2 1
or, m = (pr ) = qr 2 w
2 pr 2
81. (c) Magnetic moment of current carrying rectangular loop
of area A is given by M = NIA 3 cm
magnetic moment of current carrying coil is a vector and 5 cm
its direction is given by right hand thumb rule, for
r
rectangular loop, B at centre due to current in loop and P Q R
r
M are always parallel. Also given; length of wire Q
= 25 cm = 0.25 m
Force on wire Q due to wire R
B, M B, M
2 ´ 20 ´ 10
e Ä FQR = 10–7 ´ ´ 0.25
Outwards Inwards 0.05
Hence, (c) corresponds to stable equilibrium. = 20 × 10–5 N (Towards left)
r r Force on wire Q due to wire P
82. (c) F1 = F2 = 0
because of action and reaction pair 2 ´ 30 ´ 10
r r FQP = 10–7 × ´ 0.25
83. (a) For stable equilibrium M || B 0.03
r r = 50 × 10–5 N (Towards right)
For unstable equilibrium M || (–B)
84. (a) I1 I2 = Positive Hence, Fnet = FQP – FQR
(attract) F = Negative = 50 × 10–5 N – 20 × 10–5 N
I1 I2= Negative = 3 × 10–4 N towards right
(repell) F = Positive 88. (c)
Hence, option (a) is the correct answer.
85. (c) For small arc length 89. (a) Force acting on conductor B due to conductor A is
2T sin q = BIR 2 q (As F = BIL and L = RZq) given by relation
T = BIR m 0 I1 I 2 l
F=
P Q 2 pr
l-length of conductor B
T T
R r-distance between two conductors
Tsinq Tsinq
q 4p´ 10-7 ´ 10 ´ 2 ´ 2
\F= = 8 × 10–5 N
2 ´ p´ 0.1
90. (a)
86. (b) Work done in moving the conductor is, 91. (a) The magnetic field at O due to current in DA is
2 2
W = ò Fdx = ò 3.0 ´ 10 -4 e -0.2 x ´ 10 ´ 3dx m I p
B1 = o ´ (directed vertically upwards)
0 0
4p a 6
-3 -0.2 x2 The magnetic field at O due to current in BC is
= 9 ´ 10 ò e dx m I p
0 l=3m B2 = o ´ (directed vertically downwards)
9 ´ 10 -3 I = 10 A
z
4p b 6
= [- e-0.2´ 2 + 1] The magnetic field due to current AB and CD at O is zero.
0.2 Therefore the net magnetic field is
x
Moving Charges and Magnetism P-337

B = B1 – B2 (directed vertically upwards) 2 = ig ( R1 + R2 + G) ...(ii)


m I p mo I p
= o - ´
4 p a 6 4p b 6 Dividing eq. (i) by (ii),
m I æ 1 1ö m I 1 G + R1
= o ç - ÷ = o (b - a) Þ =
24 è a b ø 24ab 2 G + R1 + R2
r r r
92. (d) F = I ( l ´ B)
The force on AD and BC due to current I1 is zero. This is Þ G + R1 + R2 = 2G + 2 R1
uur
because the directions of current element I d l and \ R2 = G + R1
r
magnetic field B are parallel.
93. (a) Force acting between two long conductor carrying 97. (d) Given,
current, Current passing through galvanometer, I = 6 mA
m 2I I Deflection, q = 2°
F = 0 1 2 ´l ...(i)
4p d Figure of merit of galvanometer
Where d = distance between the conductors
l = length of conductor I 6 ´10-3
m 2(2 I1 ) I 2 = = = 3 ´10-3 A/div
In second case, F ¢ = - 0 l ..(ii) q 2
4 p 3d 98. (20)
From equation (i) and (ii), we have
F ¢ -2 Given,
\ = Area of galvanometer coil, A = 3 × 10–4 m2
F 3
94. (b) When current is passed through a spring then current Number of turns in the coil, N = 500
flows parallel in the adjacent turns in the same direction. Current in the coil, I = 0.5 A
As a result the various turn attract each other and spring
r r
get compress. Torque t =| M ´ B |= NiAB sin(90°) = NiAB
95. (c) Magnetic field due to current in wire 1 at point P distant
r from the wire is t 1.5
ÞB= = = 20 T
m i NiA 500 ´ 0.5 ´ 3 ´10 -4
B = 0 1 [ cos q + cos q ] q i2
4p r i 99. (c) ig = 20 × 50 = 1000 µA = 1 mA
1
r P
dl Using, V = ig (G + R), we have
m 0 i1 cos q
B= 2 = 10–3 (100 + R1)
2p r q

R1 = 1900 W
This magnetic field is directed perpendicular to the plane of when, V = 10 volt
paper, inwards. 10 = 10–3 (100 + R2 + R1)
The force exerted due to this magnetic field on current
10000 = (100 + R2 + 1900)
element i2 dl is
dF = i2 dl B sin 90° \ R2 = 8000 W
\ dF = i2 dlB 100. (b) In an ammeter,
æ m i cos q ö RA
Þ dF = i2 dl ç 0 1 ÷ ig = i0
è 4p r ø RA + G
m0 and for voltmeter,
= i1 i2 dl cos q
2 pr V = ig (G + RV) = Gi0
96. (d) Galvanometer of resistance (G) converted into a
On solving above equations, we get
voltmeter of range 0-1 V.
RARV = G2
R1
2
G R A æ ig ö
ig =ç
and ÷
RV è i0 - ig ø
V = 1 = ig (G + R1 ) ...(i)
101. (Bonus) v = ig (R + G)
To increase the range of voltmeter 0-2 V Þ 5 = 10–4 (2 × 106 + x)
G x = – 195 × 104W
R1 R2
P-338 Physics

102. (c) V = ig (G + R) = 4 × 10–3 (50 + 5000) = 20V 108. (d) Given,


103. (d) Cq = NBiA sin 90° Resistance of galvanometer, G = 100W
Current, ig = 1 mA
æ p ö
or 10-6 ç = 175B(10-3 ) ´ 10-4 A galvanometer can be converted into voltmeter by
è 180 ÷ø connecting a large resistance R in series with it.
\ B = 10–3 T Total resistance of the combination = G + R
S According to Ohm’s law, V = ig (G + R)
104. (d) Using, ig = i \ 10 = 1 × 10–3 (100 + R0)
S+G
Þ 10000 – 100 = 9900 W = R0
S Þ R0 = 9.9 kW
0.002 = 0.5
S + 50 109. (b) Figure of merit of a galvanometer is the correct required
On solving, we get to produce a deflection of one division in the galvanometer
100 I
S= ; 0.2 W i.e., figure of merit =
q
498
105. (b) When key K1 is closed and key K2 is open e 1
I= G = KW
R+G 9
E
ig = = Cq 0 ... (i) 1 e S 1 eS
220 + R g = ´ Þ =
2 R + GS S+ G 2 R(S + G) + GS
When both the keys are closed G+S
æ E ö 5 Cq0 S
ig = ´ =
ç 5R g ÷ (R g + 5) 5
ç 220 + ÷ 1
G G
è 5 + Rg ø RG ´ I I/2
S= 2
5E Cq 0 (R + G)I R R
Þ = ... (ii) e-
225R g + 1100 5 2
E E
E
= Cq0 ... (i)
220 + R g 1
11 ´ 103 ´ ´ 102 ´ 270 ´ 10 -6
Dividing (i) by (ii), we get S= 2 = 110 W
æ 6ö
225R g + 1100 6-ç ÷
Þ =5 è 2ø
1100 + 5R g
110. (d) According to question, current through galvanometer,
Þ 5500 + 25Rg = 225Rg + 1100 Ig = 1 mA
200Rg = 4400 Current through shunt (I – Ig) = 2 A
Rg = 22W Galvanometer resistance Rg = 25W
Resistance of shunt, S = ? Ig
106. (b) Galvanometer has 25 divisions Ig = 4 × 10–4 × 25 = 10–2 A
I0R0 = (I – Ig)S G
G ig
V = Ig Rnet 10-3 ´ 25
50W R ÞS= I – Ig S
2
v 2.5V S ; 1.25 × 10–2W
v = Ig (G + R) 111. (c) Given : Current through the galvanometer,
2.5 = (50 + R) 10–2 \ R = 200W ig = 5 × 10–3 A
107. (c) Deflection current Galvanometer resistance, G = 15W
= Igmax = nxk =0.005 × 30 Let resistance R to be put in series with the galvanometer
Where, n = Number of divisions = 30 and k = 0.005 amp/ to convert it into a voltmeter.
division V = ig (R + G)
= 15 × 10–2 = 0.15 10 = 5 × 10–3 (R + 15)
v = Ig[20 + R] \ R = 2000 – 15 = 1985 = 1.985 × 103 W
15 = 0.15 [20 + R] 112. (c) Ig G = ( I – Ig)s
100 = 20 + R \ 10–3 × 100 = (10 – 10–3) × S
R = 80 W \ S » 0.01W
Moving Charges and Magnetism P-339

V 5 Þ R(G + S) + GS = 2S(R + G)
113. (d) As we know, I = = = 0.1
R 50 Þ RG + RS + GS = 2S(R + G)
I' = 0.099
Þ RG = 2S(R + G) - S(R + G)
When Galvanometer is connected
100S V \ RG = S(R + G)
R eq = 50 + = 115. (b) To measure AC voltage across a resistance a
100 + S I
moving coil galvanometer is used.
100S 5 116. (d) The correct circuit diagram is D with galvanometer
Þ = - 50
100 + S 0.099 resistance
100S 100S RS
Þ = 50.50 - 50 Þ = 0.5
100 + S 100 + S G=
R-S
Þ 100S = 50 + 0.55 Þ 99.5S = 50
117. (d) Statements I is false and Statement II is true
50
S= = 0.5 W IgG
99.05 For ammeter, shunt resistance, S =
So, shunt of resistance = 0.5W is connected in parallel with I – Ig
the galvanometer. Therefore for I to increase, S should decrease, So additional
V S can be connected across it.
114. (a) According to Ohm's Law, I = 118. (d)
R
V 119. (d) The current that will given full scale deflection in the
Ig = absence of the shunt is nearly equal to the current through
R +G
where, Ig-Galvanometer current, G-Galvonometer resistance the galvanometer when shunt is connected i.e. Ig

R IG IS
As Ig =
G G+S
5.5 ´ 1
V = = 0.045 ampere.
120 + 1
When shunt of resistance S is connected parallel to the
120. (a) 500 W
GS A
Galvanometer then G =
G +S i
2V
V R
\ I= 12V
GS
R+
G +S
IG 10 1
R 2 12 – 2 = (500W)i Þ i = =
G 500 50
12 1
S Again, i = =
500 + R 50
Equal potential difference is given by Þ 500 + R = 600
I'g G = (I - I'g )S Þ R = 100 W
121. (c) Resistance of Galvanometer,
I 'g (G + S) = IS Current sensitivity 10
G= ÞG= = 5W
Ig IS Voltage sensitivity 2
Þ =
2 G +S 150
Here ig = Full scale deflection current = = 15 mA
V V S 10
Þ = ´ V = voltage to be measured = 150 volts
2(R + G ) R + GS G+S
(such that each division reads 1 volt)
G +S
150
1 S ÞR= - 5 = 9995W
Þ = 15 ´ 10 -3
2(R + G) R(G + S) + GS
19
P-340 Physics

Magnetism and
Matter
æm ö M æ m0 ö 2M
Magnetism, Gauss’s Law, 2ç 0÷ ´ qv ç ÷ ´ qv
è 4p ø d
( ) ( )
(d) è 4p ø d 3
3
TOPIC 1 Magnetic Moment, Properties (c)
2 2
of Magnet
4. A magnet of total magnetic moment 10 -2 iˆ A-m2 is
1. A small bar magnet placed with its axis at 30° with an external placed in a time varying magnetic field,
field of 0.06 T experiences a torque of 0.018 Nm. The
B iˆ (coswt)where B = 1 Tesla and w = 0.125 rad/s. The
minimum work required to rotate it from its stable to
work done for reversing the direction of the magnetic
unstable equilibrium position is :
moment at t = 1 second, is: [10 Jan. 2019 I]
[Sep. 04, 2020 (I)] (a) 0.01 J (b) 0.007 J
(a) 6.4 × 10–2 J (b) 9.2 × 10–3 J (c) 0.028 J (d) 0.014 J
(c) 7.2 × 10–2 J (d) 11.7 × 10–3 J 5. A magnetic dipole in a constant magnetic field has :
2. A circular coil has moment of inertia 0.8 kg m2 around any [Online April 8, 2017]
diameter and is carrying current to produce a magnetic (a) maximum potential energy when the torque is maximum
moment of 20 Am2. The coil is kept initially in a vertical (b) zero potential energy when the torque is minimum.
position and it can rotate freely around a horizontal (c) zero potential energy when the torque is maximum.
diameter. When a uniform magnetic field of 4 T is applied (d) minimum potential energy when the torque is maximum.
along the vertical, it starts rotating around its horizontal 6. A magnetic dipole is acted upon by two magnetic fields
diameter. The angular speed the coil acquires after rotating which are inclined to each other at an angle of 75°. One of
the fields has a magnitude of 15 mT. The dipole attains
by 60° will be : [Sep. 04, 2020 (II)]
stable equilibrium at an angle of 30° with this field. The
(a) 10 rad s–1 (b) 10p rad s–1 magntidue of the other field (in mT) is close to :
(c) 20p rad s–1 (d) 20 rad s–1 [Online April 9, 2016]
3. Two magnetic dipoles X and Y are placed at a separation (a) 1 (b) 11 (c) 36 (d) 1060
d, with their axes perpendicular to each other. The dipole 7. A 25 cm long solenoid has radius 2 cm and 500 total number
moment of Y is twice that of X. A particle of charge q is of turns. It carries a current of 15 A. If it is equivalent to a
passing through their midpoint P, at angle q = 45° with the r
horizontal line, as shown in figure. What would be the magnet of the same size and magnetization M (magnetic
uur
magnitude of force on the particle at that instant? (d is moment/volume), then M is : [Online April 10, 2015]
much larger than the dimensions of the dipole)
[8 April 2019 II] (a) 30000p Am–1 (b) 3pAm–1
(c) 30000 Am –1 (d) 300 Am–1
8. A bar magnet of length 6 cm has a magnetic moment of
4 J T–1. Find the strength of magnetic field at a distance of
200 cm from the centre of the magnet along its equatorial
line. [Online May 7, 2012]
(a) 4 × 10–8 tesla (b) 3.5 × 10–8 tesla
(c) 5 × 10–8 tesla (d) 3 × 10–8 tesla
9. A thin circular disc of radius R is uniformly charged with
æ m0 ö M density s > 0 per unit area. The disc rotates about its axis
ç ÷ ´ qv with a uniform angular speed w.The magnetic moment of
( )
(a) è 4p ø d 3
(b) 0
2 the disc is [2011 RS]
Magnetism and Matter P-341

r
4 pR 4 (a) B
(a) pR sw (b) sw (b) zero
2 r r
4 (c) much large than | B | and parallel to B
pR r r
(c) sw (d) 2pR 4 sw (d) much large than | B | but opposite to B
4
10. A magnetic needle is kept in a non-uniform magnetic field. 17. Magnetic materials used for making permanent magnets
It experiences [2005] (P) and magnets in a transformer (T) have different
(a) neither a force nor a torque properties of the following, which property best matches
(b) a torque but not a force for the type of magnet required? [Sep. 02, 2020 (I)]
(c) a force but not a torque
(a) T : Large retentivity, small coercivity
(d) a force and a torque
11. The length of a magnet is large compared to its width and (b) P : Small retentivity, large coercivity
breadth. The time period of its oscillation in a vibration (c) T : Large retentivity, large coercivity
magnetometer is 2s. The magnet is cut along its length (d) P : Large retentivity, large coercivity
into three equal parts and these parts are then placed on 18.
each other with their like poles together. The time period
of this combination will be [2004]
2 2
(a) 2 3 s (b) s (c) 2 s (d) s
3 3
12. A magnetic needle lying parallel to a magnetic field requiers
W units of work to turn it through 600 . The torque needed
to maintain the needle in this position will be [2003]

(a) 3W (b) W (c) 3 (d) 2 W


W
2
13. The magnetic lines of force inside a bar magnet [2003] The figure gives experimentally measured B vs. H variation
(a) are from north-pole to south-pole of the magnet in a ferromagnetic material. The retentivity, co-ercivity
(b) do not exist and saturation, respectively, of the material are:
(c) depend upon the area of cross-section of the bar [7 Jan. 2020 II]
magnet (a) 1.5 T, 50 A/m and 1.0 T
(d) are from south-pole to north-pole of the Magnet (b) 1.5 T, 50 A/m and 1.0 T
(c) 150 A/m, 1.0 T and 1.5 T
The Earth Magnetism, Magnetic (d) 1.0 T, 50 A/m and 1.5 T
TOPIC 2 A paramagnetic material has 1028 atoms/m3. Its magnetic
Materials and their properties 19.
susceptibility at temperature 350 K is 2.8 × 10–4 . Its
14. An iron rod of volume 10–3 m3 and relative permeability susceptibility at 300 K is: [12 Jan. 2019 II]
1000 is placed as core in a solenoid with 10 turns/cm. If a (a) 3.267 × 10–4 (b) 3.672 × 10–4
current of 0.5 A is passed through the solenoid, then the (c) 3.726 × 10–4 (d) 2.672 × 10 –4
magnetic moment of the rod will be : [Sep. 05, 2020 (II)] 20. A paramagnetic substance in the form of a cube with sides
(a) 50 × 102 Am2 (b) 5 × 102 Am2 1 cm has a magnetic dipole moment of 20 × 10–6 J/T when
2 2 a magnetic intensity of 60 × 103 A/m is applied. Its magnetic
(c) 500 × 10 Am (d) 0.5 × 102 Am2
susceptibility is: [11 Jan. 2019 II]
15. A paramagnetic sample shows a net magnetisation of 6 A/m (a) 3.3 × 10–2 (b) 4.3 × 10 –2
when it is placed in an external magnetic field of 0.4 T at a (c) 2.3 × 10–2 (d) 3.3 × 10–4
temperature of 4 K. When the sample is placed in an external 21. At some location on earth the horizontal component of
magnetic field of 0.3 T at a temperature of 24 K, then the earth’s magnetic field is 18 × 10–6 T. At this location,
magnetisation will be : [Sep. 04, 2020 (II)] magnetic needle of length 0.12 m and pole strength 1.8
(a) 1 A/m (b) 4 A/m Am is suspended from its mid-point using a thread, it
(c) 2.25 A/m (d) 0.75 A/m makes 45° angle with horizontal in equilibrium. To keep
this needle horizontal, the vertical force that should be
16. A perfectly diamagnetic sphere has a small spherical cavity
applied at one of its ends is: [10 Jan. 2019 II]
at its centre, which is filled with a paramagnetic substance. (a) 3.6 × 10–5 N (b) 1.8 × 10–5 N
r
The whole system is placed in a uniform magnetic field B. (c) 1.3 × 10–5 N (d) 6.5 × 10–5 N
Then the field inside the paramagnetic substance is : 22. A bar magnet is demagnetized by inserthing it inside a
[Sep. 03, 2020 (II)] solenoid of length 0.2 m, 100 turns, and carrying a current
of 5.2 A. The coercivity of the bar magnet is:
[9 Jan. 2019 I]
P (a) 285 A/m (b) 2600 A/m
(c) 520 A/m (d) 1200 A/m
P-342 Physics

23. The B-H curve for a ferromagnet is shown in the figure. =Horizontal component of earth’s magnetic field = 3.6
The ferromagnet is placed inside a long solenoid with × 10–5 tesla) [Online April 11, 2015]
1000 turns/cm.. The current that should be passed in the (a) 14.6 (b) 19.4 (c) 9.7 (d) 4.9
solenoid to demagnetise the ferromagnet completely is: 27. The coercivity of a small magnet where the ferromagnet
[Online April 15, 2018] gets demagnetized is 3 ´ 103 Am -1. The current required
B(T) to be passed in a solenoid of length 10 cm and number of
turns 100, so that the magnet gets demagnetized when
2, 0
inside the solenoid, is: [2014]
1, 0 H(A/m) (a) 30 mA (b) 60 mA (c) 3 A (d) 6 A
–200 –100 100 200 28. An example of a perfect diamagnet is a superconductor.
–1, 0
This implies that when a superconductor is put in a
–2, 0
magnetic field of intensity B, the magnetic field Bs inside
the superconductor will be such that:
(a) 2 mA (b) 1 mA (c) 40 µA (d) 20 µA [Online April 19, 2014]
24. Hysteresis loops for two magnetic materials A and B are (a) Bs = – B (b) Bs = 0
given below : (c) Bs = B (d) Bs < B but Bs ¹ 0
D B 29. Three identical bars A, B and C are made of different
magnetic materials. When kept in a uniform magnetic field,
the field lines around them look as follows:

H H
A B C

(A) (B)
Make the correspondence of these bars with their
These materials are used to make magnets for elecric
material being diamagnetic (D), ferromagnetic (F) and
generators, transformer core and electromagnet core.
paramagnetic (P): [Online April 11, 2014]
Then it is proper to use : [2016]
(a) A « D, B « P, C « F
(a) A for transformers and B for electric generators.
(b) B for electromagnets and transformers. (b) A « F, B « D, C « P
(c) A for electric generators and trasformers. (c) A « P, B « F, C « D
(d) A for electromagnets and B for electric generators.
(d) A « F, B « P, C « D
25. A fighter plane of length 20 m, wing span (distance from
30. The magnetic field of earth at the equator is approximately
tip of one wing to the tip of the other wing) of 15m and
4 × 10–5 T. The radius of earth is 6.4 × 106 m. Then the
height 5m is lying towards east over Delhi. Its speed is
dipole moment of the earth will be nearly of the order of:
240 ms–1. The earth's magnetic field over Delhi is 5 ×
[Online April 9, 2014]
10–5T with the declination angle ~0° and dip of q such
(a) 1023 A m2 (b) 1020 A m2
2 (c) 1016 A m2 (d) 1010 A m2
that sin q = . If the voltage developed is VB between
3 31. The mid points of two small magnetic dipoles of length d
the lower and upper side of the plane and VW between the in end-on positions, are separated by a distance x, (x > >
tips of the wings then VB and VW are close to : d). The force between them is proportional to x–n where n
[Online April 10, 2016] is: [Online April 9, 2014]
(a) VB = 40 mV; VW = 135 mV with left side of pilot at
N S
higher voltage S N
(b) VB = 45 mV; VW = 120 mV with right side of pilot
x
at higher voltage
(a) 1 (b) 2 (c) 3 (d) 4
(c) VB = 40 mV; VW = 135 mV with right side of pilot
32. Two short bar magnets of length 1 cm each have magnetic
at higher voltage
moments 1.20 Am2 and 1.00 Am2 respectively. They are
(d) VB = 45 mV; VW = 120 mV with left side of pilot at
placed on a horizontal table parallel to each other with
higher voltage
their N poles pointing towards the South. They have a
26. A short bar magnet is placed in the magnetic meridian
common magnetic equator and are separated by a distance
of the earth with north pole pointing north. Neutral
of 20.0 cm. The value of the resultand horizontal magnetic
points are found at a distance of 30 cm from the magnet
induction at the mid-point O of the line joining their
on the East – West line, drawn through the middle point
centres is close to (Horizontal component of earth.s
of the magnet. The magnetic moment of the magnet in
magnetic induction is 3.6× 10.5Wb/m2) [2013]
m0 (a) 3.6 × 10.5 Wb/m2 (b) 2.56 × 10.4 Wb/m2
Am2 is close to: (Given = 10–7 in SI units and BH
4p (c) 3.50 × 10.4 Wb/m2 (d) 5.80 × 10.4 Wb/m2
Magnetism and Matter P-343

33. The earth’s magnetic field lines resemble 38. Curie temperature is the temperature above which
that of a dipole at the centre of the earth. If the magnetic [2003]
moment of this dipole is close to 8 × 1022 Am2, the
(a) a ferromagnetic material becomes paramagnetic
value of earth’s magnetic field near the equator is close
to (radius of the earth = 6.4 × 106 m) (b) a paramagnetic material becomes diamagnetic
[Online April 25, 2013] (c) a ferromagnetic material becomes diamagnetic
(a) 0.6 Gauss (b) 1.2 Gauss (d) a paramagnetic material becomes ferromagnetic
(c) 1.8 Gauss (d) 0.32 Gauss
34. Relative permittivity and permeability of a material e r and
TOPIC 3 Magnetic Equipment
m r , respectively. Which of the following values of these
quantities are allowed for a diamagnetic material? [2008] 39. A ring is hung on a nail. It can oscillate, without slipping
(a) e r = 0.5, m r = 1.5 (b) e r = 1.5, mr = 0.5 or sliding (i) in its plane with a time period T1 and, (ii) back
and forth in a direction perpendicular to its plane, with a
(c) e r = 0.5, m r = 0.5 (d) e r = 1.5, mr = 1.5
T1
35. Needles N1, N2 and N3 are made of a ferromagnetic, a period T2. The ratio will be : [Sep. 05, 2020 (II)]
paramagnetic and a diamagnetic substance respectively. T2
A magnet when brought close to them will [2006] 2 2
(a) (b)
(a) attract N1 and N2 strongly but repel N3 3 3
(b) attract N1 strongly, N2 weakly and repel N3 weakly 3 2
(c) (d)
(c) attract N1 strongly, but repel N2 and N3 weakly 2 3
(d) attract all three of them 40. A magnetic compass needle oscillates 30 times per minute
36. The materials suitable for making electromagnets should at a place where the dip is 45o, and 40 times per minute
have [2004] where the dip is 30o. If B1 and B2 are respectively the total
magnetic field due to the earth and the two places, then
(a) high retentivity and low coercivity
the ratio B1/B2 is best given by :
(b) low retentivity and low coercivity
[12 April 2019 I]
(c) high retentivity and high coercivity (a) 1.8 (b) 0.7 (c) 3.6 (d) 2.2
(d) low retentivity and high coercivity 41. A hoop and a solid cylinder of same mass and radius are
37. A thin rectangular magnet suspended freely has a period made of a permanent magnetic material with their
of oscillation equal to T. Now it is broken into two equal magnetic moment parallel to their respective axes. But
halves (each having half of the original length) and one the magnetic moment of hoop is twice of solid cylinder.
piece is made to oscillate freely in the same field. If its They are placed in a uniform magnetic field in such a
manner that their magnetic moments make a small angle
T'
period of oscillation is T¢, the ratio is [2003] with the field. If the oscillation periods of hoop and
T cylinder are Th and Tc respectively, then:
1 1 [10 Jan. 2019 II]
(a) (b) (a) Th = Tc (b) Th = 2 Tc
2 2 2
(c) Th = 1.5 Tc (d) Th = 0.5 Tc
1 42. A magnetic needle of magnetic moment 6.7 × 10–2 Am2
(c) 2 (d)
4 and moment of inertia 7.5 × 10–6 kg m2 is performing
simple harmonic oscillations in a magnetic field of 0.01
T. Time taken for 10 complete oscillations is : [2017]
(a) 6.98 s (b) 8.76 s
(c) 6.65 s (d) 8. 89 s
P-344 Physics

1. (c) Here, q = 30°, t = 0.018 N-m, B = 0.06 T µ0 2M


and B2 =
Torque on a bar magnet : 4p ( d / 2 ) 3
t = MB sin q
µ0 2M
0.018 = M ´ 0.06 ´ sin 30°
B 4p ( d / 2) 3
1 tan q = 2 =
Þ 0.018 = M ´ 0.06 ´ Þ M = 0.6 A-m 2 \ B1 µ0 2M = 1
2
Position of stable equilibrium (q = 0°) 4p ( d / 2) 3
Position of unstable equilibrium (q = 180°) or q = 45º
Minimum work required to rotate bar magnet from stable uur
The resultant field is 45º from B1. The angle between B
to unstable equilibrium uur
DU = U f - U i = -MB cos180° - (- MB cos0°) and v zero, so force on the particle is zero.
4. (c)
W = 2MB = 2 ´ 0.6 ´ 0.06 Work done, W = 2 m·B
\ W = 7.2 ´ 10-2 J = 2 × 10–2 × 1 cos (0.125)
= 0.02 J
2. (a) Given,
5. (c) Potential energy of dipole,
Moment of inertia of circular coil, I = 0.8 kg m2 U = – pE cos q
Magnetic moment of circular coil, M = 20 Am2 Torque experienced by dipole
Rotational kinetic energy of circular coil, t = pE sin q
Torque will be maximum (tmax) when q = 90° then potential
1 2 energy U = 0
KE = Iw
6. (b) We know that, magnetic dipole moment
2
Here, w = angular speed of coil M = NiA cosθ i.e., M µ cosθ
Potential energy of bar magnet = – MB cos f When two magnetic fields are inclined at an angle of 75°
From energy conservation the equilibrium will be at 30°, so
1
1 2 cos θ < cos(75°, 30°) < cos 45° <
I w = U in - U f = - MB cos 60° - (- MB ) 2
2 x 15
< [ x » 11
MB 1 2 2 2
Þ = Iw r NiA
2 2 7. (c) M (mag. moment/volume) =
Al
20 ´ 4 1 Ni (500)15
Þ = (0.8)w 2 = = = 30000 Am–1
2 2 l 25 ´ 10 –2
Þ 100 = w 2 Þ w = 10 rad 8. (c) Along the equatorial line, magnetic field strength
m M
B= 0
µ
(
4p 2
)
2K 3/2
B1 = 0 r + l2
3. (b) 4 p ( d / 2 )3
Given: M = 4JT–1
r = 200 cm = 2 m
6cm
l= = 3 cm = 3 × 10–2 m
2
4p ´ 10 -7 4
\B= ´
4p 3
é 2
ê 2 + 3 ´ 10
ë
(-2 2 ù 2
ú
û
)
Solving we get, B = 5 × 10–8 tesla
Magnetism and Matter P-345

q Magnetic dipole moment 14. (b) Given,


9. (c) =
2m Angular momentum Volume of iron rod, V = 10-3 m3
Relative permeability, m r = 1000
Number of turns per unit length, n = 10
a Magnetic moment of an iron core solenoid,
M = (m r - 1) ´ NiA
V N
Þ M = (mr -1) ´ Ni Þ M = (mr - 1) ´ iV
l l
\ Magnetic dipole moment (M)
10
q æ mR 2 ö 1 4 Þ M = 999 ´ ´ 0.5 ´ 10 -3 = 499.5 » 500.
M = .ç ÷ .w = s.pR w. 10 -2
2m è 2 ø 4
15. (d) For paramagnetic material. According to curies law
10. (d) A magnetic needle kept in non uniform magnetic field
experience a force and torque due to unequal forces acting 1

on poles. T
11. (b) Initially, time period of magnet
For two temperatures T1 and T2
I 1 2
T = 2p = 25 where I = ml c1T1 = c2T2
MB 12
When the magnet is cut into three pieces the pole strength I
will remain the same and But c =
Moment of inertia of each part, B
1 æ mö æ l ö I I1 I
(I¢) = çè ÷ø çè ÷ø ´ 3 = \ T1 = 2 T2
12 3 3 9 B1 B2
We have, Magnetic moment (M)
= Pole strength (m) × l 6 I 0.3
\ New magnetic moment, Þ ´ 4 = 2 ´ 24 Þ I 2 = = 0.75 A/m
0.4 0.3 0.4
æ lö
M ' = m ´ ç ÷ ´ 3 = ml = M 16. (b) When magnetic field is applied to a diamagnetic
è 3ø
substance, it produces magnetic field in opposite direction
I¢ so net magnetic field inside the cavity of sphere will be
New time period, T¢ = 2p zero. So, field inside the paramagnetic substance kept
M ¢B
inside the cavity is zero.
I T 2
= 2p Þ T¢ = = s. 17. (d) Permanent magnets (P) are made of materials with large
9 MB 9 3 retentivity and large coercivity. Transformer cores (T) are
12. (a) Workdone to turn a magnetic needle from angle q1 to
q2 is given by made of materials with low retentivity and low coercivity.
W = MB (cos q1 - cos q2 )
\ W = MB (cos 0° - cos 60°) 18. (d)

æ 1 ö MB
= MB ç 1 - ÷ =
è 2ø 2
MB
\ Torque, t = MB sin q = MB sin 60° = 3 = 3W
2
13. (d) The magnetif field lines of bar magnet form closed
lines. As shown in the figure, the magnetic lines of force
are directed from south to north inside a bar magnet.
Outside the bar magnet magnetic field lines directed from
north to south pole. 19. (a) According to Curie law for paramagnetic substance,
1 c TC2
cµ T Þ 1 = T
C c2 C1

N S 2.8 ´ 10 –4 300
=
c2 350

2.8 ´ 350 ´10 –4


c2 = = 3.266 × 10–4
300
P-346 Physics

20. (d) Magnetic susceptibility, 28. (b) Magnetic field inside the superconductor is zero.
Diamagnetic substances are repelled in external magnetic
I field.
c=
H 29. (b) Diamagnetic materials are repelled in an external
magnetic field.
Magnetic moment 20 ´10 –6 Bar B represents diamagnetic materials.
where, I = = = 20 N/m2
Volume 10 –6 30. (a) Given, B = 4 × 10–5 T
RE = 6.4 × 106 m
Dipole moment of the earth M = ?
201 m M
Now, c = = ´10 –3 = 3.3 ´10–4
3 B= 0 3
60´10 3 4p d
21. (d) using, MB sinq = F l Sinq (t) 4p´10-7 ´ M
4 ´10-5 =
( )
3
4p´ 6.4 ´ 106
F
B \ M @ 1023 Am2
45°
31. (d) In magnetic dipole
1
Force µ
m r4
In the given question,
Force µ x– n
l Hence, n = 4
MB sin 45° = F sin 45° 32. (b) Given : M1 = 1.20 Am2
2
N
F = 2MB = 2 × 1.8 × 18 × 10–6 = 6.5 × 10–5N BH
B æ Nö B1
22. (b) Corecivity, H = m and B = m 0 ni çè n = ÷ø S S
0 l B2
O
N 100 N N
or, H = i= × 5.2 = 2600 A/m r r
l 0.2
23. (b) Given Number of turns,
n = 1000 turns/cm = 1000 × 100 turns/m S
Coercivity of ferromagnet, H = 100 A/m 20
Current to demagnetise the ferromagnet, I = ? M2 = 1.00 Am2 ; r = cm = 0.1m
2
Using, H = nI Bnet = B1 + B2 + BH
or, 100 = 105 × I
m ( M1 + M 2 )
100 Bnet = 0 + BH
\ I = 5 = 1 mA 4p r3
10
24. (b) Graph [A] is for material used for making permanent 10 -7 (1.2 + 1)
= + 3.6 ´ 10 -5 = 2.56 ´ 10 -4 wb/m2
magnets (high coercivity) (0.1)3
Graph [B] is for making electromagnets and transformers. 33. (a) Given M = 8 × 1022 Am2
25. (d) VB = VBHl = 240 × 5 × 10–5 cos(q) × 5 = 44.7 mv d = Re = 6.4 × 106m
By right hand rule, the charge moves to the left of pilot. m0 2M
Earth’s magnetic field, B = .
26. (c) Here, r = 30cm = 0.3cm 4p d3
m0 M 4 p ´ 10 -7 2 ´ 8 ´ 1022
we know = BH = 3.6 ´ 10 –5 = ´ @ 0.6 Gauss
4 pr 3 4p (6.4 ´ 106 )3
34. (b) For a diamagnetic material, the value of µr is slightly
3.6 ´10–5 less than one. For any material, the value of Îr is always
Þ M = (0.3)3
10 –7 greater than 1.
Hence, M = 9.7 Am2 35. (b) Ferromagnetic substance has magnetic domains
27. (c) Magnetic field in solenoid B = m0ni whereas paramagnetic substances have magnetic dipoles
which get attracted to a magnetic field. Ferromagnetic
B
Þ = ni material magnetised strongly in the direction of magnetism
m0 field, Hence, N1 will be attracted paramagnetic substance
(Where n = number of turns per unit length) attract weekly in the direction of field. Hence, N2 will weakly
attracted. Diamagnetic substances do not have magnetic
B Ni 100i dipole but in the presence of external magnetic field due to
Þ = Þ 3 ´ 103 =
m0 L 10 ´ 10-2 their orbital motion of electrons these substances are
Þ i = 3A repelled. Hence, N3 will be repelled.
Magnetism and Matter P-347

36. (b) Electromagnet should be amenable to magnetisation


& demagnetization. T1 I1 2MR 2 2
\ = = =
\ Materials suitable for making electromagnets should T2 I2 3 2 3
MR
have low retentivity and low coercivity should be low. 2
37. (b) The time period of a rectangular magnet oscillating in
I I
earth’s magnetic field is given by T = 2p 40. (B onus) We have, T = 2p MB
MBH x

where I = Moment of inertia of the rectangular magnet T12 Bx 2


M = Magnetic moment \ =
BH = Horizontal component of the earth’s magnetic field T22 Bx1
Initially, the time period of the magnet
2
æ 2ö B2 cos 45° B2 ´ 2
I 1 or çè ÷ø = =
T = 2p where I = M l2 1.5 B1 cos 30° 2 ´ B1 ´ 3
MBH 12
2
Case 2 æ 4ö B2 2
Magnet is cut into two identical pieces such that each çè ÷ø = ´
3 B1 6
piece has half the original length.
B1 9
I¢ \ = = 0.46
Then T ¢ = 2p B2 8 6
M ¢BH
41. (a) Using, time /oscillation period,
Moment of inertia of each part
2
1 æ M öæ l ö I M I
T = 2p
I¢ = ç ÷ ç ÷ = and M ¢ = MB
12 è 2 ø è 2 ø 8 2
Where, M = magnetic moment, I moment of inertia and B =
T¢ I¢ M I /8 M 1 1 magnetic field
\ = ´ = ´ = =
T M I M /2 I 4 2
38. (a) The temperature above which a ferromagnetic mR 2
substance becomes paramagnetic is called Curie’s Th = 2p
( 2MB)
temperature.
39. (a) Let I1 and I2 be the moment of inertia in first and second
case respectively. 1 / 2mR 2
Tc = 2p
I1 = 2 MR 2 MB
Clearly, Th = Tc
2MR 2 3
I 2 = MR + = MR 2 42. (c) Given : Magnetic moment, M = 6.7 × 10–2 Am2
2 2 Magnetic field, B = 0.01 T
Axis of rotation Moment of inertia, I = 7.5 × 10–6 Kgm2
X
I
Using, T = 2p
MB

7.5 ´ 10-6 2p
= 2p -2
= ´ 1.06 s
T1 6.7 ´ 10 ´ 0.01 10
T2
Time taken for 10 complete oscillations
I t = 10T = 2p × 1.06
Time period, T = 2p
mgd = 6.6568 » 6.65 s
TµI
20
P-348 Physics

Electromagnetic
Induction
3. An elliptical loop having resistance R, of semi major axis a,
Magnetic Flux, Faraday's and semi minor axis b is placed in a magnetic field as shown
TOPIC 1
and Lenz's Law in the figure. If the loop is rotated about the x-axis with
angular frequency w, the average power loss in the loop
1. Two concentric circular coils, C1 and C2, are placed in the due to Joule heating is : [Sep. 03, 2020 (I)]
XY plane. C1 has 500 turns, and a radius of 1 cm. C2 has 200
z B
turns and radius current 20 cm. C2 carries a time dependent
current I(t) = (5t2 – 2t + 3) A Where t is in s. The emf induced
x b x
4 a
in C1 (in mV), at the instant t = 1 s is . The value of x is
x y
______. [NA Sep. 05, 2020 (I)] y
2. A small bar magnet is moved through a coil at constant p 2 a 2 b 2 B 2 w2
speed from one end to the other. Which of the following (a) (b) zero
2R
series of observations will be seen on the galvanometer G
attached across the coil ? [Sep. 04, 2020 (I)] pabBw p 2 a 2 b 2 B 2 w2
(c) (d)
G R R
4. A uniform magnetic field B exists in a direction
perpendicular to the plane of a square loop made of a
a metal wire. The wire has a diameter of 4 mm and a total
magnet
c b length of 30 cm. The magnetic field changes with time at a
Three positions shown describe : (1) the magnet’s entry steady rate dB/dt = 0.032 Ts–1. The induced current in
(2) magnet is completely inside and (3) magnet’s exit. the loop is close to (Resistivity of the metal wire is
(1) (2) (3) 1.23 × 10–8 W m) [Sep. 03, 2020 (II)]
(a) 0.43 A (b) 0.61 A (c) 0.34 A (d) 0.53 A
(a) ® ® 5. A circular coil of radius 10 cm is placed in a uniform magnetic
field of 3.0 × 10–5 T with its plane perpendicular to the field
(1) (2) (3)
initially. It is rotated at constant angular speed about an
axis along the diameter of coil and perpendicular to
magnetic field so that it undergoes half of rotation in 0.2 s.
(b) ® ® The maximum value of EMF induced (in mV) in the coil will
be close to the integer _____. [NA Sep. 02, 2020 (I)]
(1) (2) (3) 6. In a fluorescent lamp choke (a small transformer) 100 V of
reverse voltage is produced when the choke current
changes uniformly from 0.25 A to 0 in a duration of 0.025
(c) ® ®
ms. The self-inductance of the choke (in mH) is estimated
to be ______. [NA 9 Jan. 2020 I]
(1) (2) (3)
7. At time t = 0 magnetic field of 1000 Gauss is passing
perpendicularly through the area defined by the closed
(d) ® ® loop shown in the figure. If the magnetic field reduces
linearly to 500 Gauss, in the next 5 s, then induced EMF
in the loop is: [NA 8 Jan. 2020 I]
Electromagnetic Induction P-349

(c)

(d)

(a) 56 mV (b) 28 mV (c) 48 mV (d) 36 mV


8. Consider a circular coil of wire carrying constant current
I, forming a magnetic dipole. The magnetic flux through
an infinite plane that contains the circular coil and 13. Two coils ‘P’ and ‘Q’ are separated by some distance. When
excluding the circular coil area is given by fi .The a current of 3A flows through coil ‘P’, a magnetic flux of
magnetic flux through the area of the circular coil area is 10–3 Wb passes through ‘Q’. No current is passed through
given by f0. Which of the following option is correct? ‘Q’. When no current passes through ‘P’ and a current of
[7 Jan. 2020 I] 2A passes through ‘Q’, the flux through ‘P’ is:
(a) fi = f0 (b) fi > f0 (c) fi < f0 (d) fi = – f0 [9 Apr. 2019 II]
(a) 6.67 × 10–4 Wb (b) 3.67 × 10–3 Wb
9. A long solenoid of radius R carries a time (t) - dependent
(c) 6.67 × 10–3 Wb (d) 3.67 × 10–4 Wb
current I(t) = I0t(l – t). A ring of radius 2R is placed coaxially
14. The self induced emf of a coil is 25 volts. When the
near its middle. During the time interval 0 £ t £ 1, the
current in it is changed at uniiform rate from 10 A to 25
induced current (IR) and the induced EMF(VR) in the ring
A in 1s, the change in the energy of the inductance is:
change as: [7 Jan. 2020 I]
[9 Jan. 2019 II]
(a) Direction of IR remains unchanged and VR is maximum
(a) 740 J (b) 437.5 J
at t = 0.5
(c) 540 J (d) 637.5 J
(b) At t = 0.25 direction of IR reverses and VR is maximum
15. A conducting circular loop made of a thin wire, has area
(c) Direction of IR remains unchanged and VR is zero at t = 0.25
3.5 × 10 –3m2 and resistance 10W. It is placed perpendicular
(d) At t = 0.5 direction of IR reverses and VR is zero
to a time dependent magnetic field B (t) = (0.4T) sin (50pt).
10. A loop ABCDEFA of straight edges has six corner points The the net charge flowing through the loop during t = 0
A(0, 0, 0), B{5, 0, 0), C(5, 5, 0), D(0, 5, 0), E(0, 5, 5) and s and t = 10 ms is close to: [9 Jan. 2019 I]
F(0, 0, 5). The magnetic field in this region is (a) 14 mC (b) 7 mC (c) 21 mC (d) 6 mC
r
B = ( 3iˆ + 4kˆ )T. The quantity of flux through the loop 16. In a coil of resistance 100 W , a current is induced by
ABCDEFA (in Wb) is _________ . [NA 7 Jan. 2020 I] changing the magnetic flux through it as shown in the
11. A planar loop of wire rotates in a uniform magnetic field. figure. The magnitude of change in flux through the coil is
Initially, at t = 0, the plane of the loop is perpendicular to [2017]
the magnetic field. If it rotates with a period of 10 s about (a) 250 Wb
an axis in its plane then the magnitude of induced emf will
be maximum and minimum, respectively at:[7 Jan. 2020 II] (b) 275 Wb
(a) 2.5 s and 7.5 s (b) 2.5 s and 5.0 s (c) 200 Wb
(c) 5.0 s and 7.5 s (d) 5.0 s and 10.0 s
(d) 225 Wb
12. A very long solenoid of radius R is carrying current
17. A conducting metal circular–wire–loop of radius r is placed
I(t) = kte–at (k >0), as a function of time (t >0). Counter
perpendicular to a magnetic field which varies with time as
clockwise current is taken to be positive. A circular
conducting coil of radius 2R is placed in the equatorial -t
B = B0 e t , where B0 and t are constants, at time t = 0. If
plane of the solenoid and concentric with the solenoid.
The current induced in the outer coil is correctly depicted, the resistance of the loop is R then the heat generated in
as a function of time, by: [9 Apr. 2019 II] the loop after a long time (t ® ¥) is ;
(a) [Online April 10, 2016]

p 2 r 4 B04 p 2 r 4 B02
(a) (b)
2tR 2tR

(b) p 2 r 4 B02 R p 2 r 4 B02


(c) (d)
t tR
P-350 Physics

18. When current in a coil changes from 5 A to 2 A in 0.1 s,


average voltage of 50 V is produced. The self - inductance
of the coil is : [Online April 10, 2015] V (t )
(a) 6 H (b) 0.67 H
(c) 3 H (d) 1.67 H t
19. Figure shows a circular area of radius
®
R where a uniform magnetic field B is
going into the plane of paper and R
increasing in magnitude at a constant (A) I (t ) (B) I (t )
rate. t t

In that case, which of the following graphs, drawn


schematically, correctly shows the variation of the induced
electric field E(r)? [Online April 19, 2014]
(C) I (t ) (D) I (t )

t t
E E
(a) (b) (a) A (b) C
R r R r (c) B (d) D
23. A coil is suspended in a uniform magnetic field, with the
plane of the coil parallel to the magnetic lines of force.
E
When a current is passed through the coil it starts
oscillating; It is very difficult to stop. But if an aluminium
E plate is placed near to the coil, it stops. This is due to :
(c) (d) [2012]
(a) developement of air current when the plate is placed
R r (b) induction of electrical charge on the plate
R r
(c) shielding of magnetic lines of force as aluminium is a
20. A coil of circular cross-section having 1000 turns and 4 paramagnetic material.
(d) electromagnetic induction in the aluminium plate
cm2 face area is placed with its axis parallel to a magnetic
giving rise to electromagnetic damping.
field which decreases by 10–2 Wb m–2 in 0.01 s. The e.m.f.
24. Magnetic flux through a coil of resistance 10 W is changed
induced in the coil is: [Online April 11, 2014] by Df in 0.1 s. The resulting current in the coil varies with
(a) 400 mV (b) 200 mV time as shown in the figure. Then |Df| is equal to (in weber)
(c) 4 mV (d) 0.4 mV [Online May 12, 2012]
21. A circular loop of radius 0.3 cm lies parallel to amuch bigger
i(A)
circular loop of radius 20 cm. The centre of the small loop
is on the axis of the bigger loop. The distance between
their centres is 15 cm. If a current of 2.0 A flows through 4
the smaller loop, then the flux linked with bigger loop is
[2013]
(a) 9.1 × 10–11 weber (b) 6 × 10–11 weber
(c) 3.3 × 10–11 weber (d) 6.6 × 10–9 weber t(s)
22. Two coils, X and Y, are kept in close vicinity of each other. 0.1
When a varying current, I(t), flows through coil X, the (a) 6 (b) 4
induced emf (V(t)) in coil Y, varies in the manner shown (c) 2 (d) 8
here. The variation of I(t), with time, can then be 25. The flux linked with a coil at any instant 't' is given by
represented by the graph labelled as graph : f = 10t2 – 50t + 250. The induced emf at t = 3s is [2006]
[Online April 9, 2013] (a) –190 V (b) –10 V
(c) 10 V (d) 190 V
Electromagnetic Induction P-351

29. A thin strip 10 cm long is on a U shaped wire of negligible


Motional and Static EMI and resistance and it is connected to a spring of spring constant
TOPIC 2 Application of EMI
0.5 Nm–1 (see figure). The assembly is kept in a uniform
26. An infinitely long straight wire carrying current I, one side magnetic field of 0.1 T. If the strip is pulled from its
opened rectangular loop and a conductor C with a sliding equilibrium position and released, the number of
connector are located in the same plane, as shown in the oscillations it performs before its amplitude decreases by
figure. The connector has length l and resistance R. It a factor of e is N. If the mass of strip is 50 grams, its
slides to the right with a velocity v. The resistance of the resistance 10W and air drag negligible, N will be close to :
conductor and the self inductance of the loop are [8 April 2019 I]
negligible. The induced current in the loop, as a function
of separation r, between the connector and the straight
wire is : [Sep. 05, 2020 (II)]

one side opened long (a) 1000 (b) 50000 (c) 5000 (d) 10000
conducting wire loop 30. A 10 m long horizontal wire extends from North East to
C
South West. It is falling with a speed of 5.0 ms–1, at right
v
I R l angles to the horizontal component of the earth’s magnetic
field, of 0.3 × 10–4 Wb/m2. The value of the induced emf in
r wire is : [12 Jan. 2019 II]
(a) 1.5 × 10–3 V (b) 1.1 × 10–3 V
(c) 2.5 × 10–3V (d) 0.3 × 10–3 V
31. There are two long co-axial solenoids of same length l.
The inner and outer coils have radii r1 and r2 and number
m0 Ivl m 0 Ivl of turns per unit length n1 and n2, respectively. The ratio of
(a) (b) mutual inductance to the self-inductance of the inner-coil
4p Rr p Rr
is : [11 Jan. 2019 I]
2m 0 Ivl m0 Ivl n1 n2 r1
(c) (d) ×
p Rr 2p Rr (a)
n2
(b)
n1 r2
27. The figure shows a square loop L of side 5 cm which is
connected to a network of resistances. The whole setup is n2 r22 n2
moving towards right with a constant speed of 1 cm s–1. At (c) × (d)
n1 r12 n1
some instant, a part of L is in a uniform magnetic field of 1
T, perpendicular to the plane of the loop. If the resistance 32. A copper wire is wound on a wooden frame, whose shape
of L is 1.7 &!, the current in the loop at that instant will be is that of an equilateral triangle. If the linear dimension of
close to : [12 Apr. 2019 I] each side of the frame is increased by a factor of 3, keeping
the number of turns of the coil per unit length of the frame
the same, then the self inductance of the coil:
[11 Jan. 2019 II]
(a) decreases by a factor of 9
(b) increases by a factor of 27
(c) increases by a factor of 3
(d) decreases by a factor of 9 3
33. A solid metal cube of edge length 2 cm is moving in a
(a) 60µA (b) 170 µA
(c) 150 µA (d) 115 µA positive y-direction at a constant speed of 6 m/s. There
is a uniform magnetic field of 0.1 T in the positive
28. The total number of turns and cross-section area in a
z-direction. The potential difference between the two
solenoid is fixed. However, its length L is varied by
faces of the cube perpendicular to the x-axis, is:
adjusting the separation between windings. The
[10 Jan. 2019 I]
inductance of solenoid will be proportional to:
(a) 12 mV (b) 6 mV
[9 April 2019 I]
(c) 1 mV (d) 2 mV
(a) L (b) L2 (c) 1/ L2 (d) 1/L
P-352 Physics

34. An insulating thin rod of length l has a linear charge 39. A square frame of side 10 cm and a long straight wire
x carrying current 1 A are in the plate of the paper. Starting
density r(x) = r0 on it. The rod is rotated about an from close to the wire, the frame moves towards the right
l
axis passing through the origin (x = 0) and perpendicular with a constant speed of 10 ms–1 (see figure).
to the rod. If the rod makes n rotations per second, then
the time averaged magnetic moment of the rod is:
I = 1A
[10 Jan. 2019 I]
p
(a) p n r l3 (b) n r l3
3
p x
(c) n r l3 (d) n r l3
4
35. A coil of cross-sectional area A having n turns is placed in v
a uniform magnetic field B. When it is rotated with an
angular velocity w, the maximum e.m.f. induced in the coil
will be [Online April 16, 2018] 10 cm
3
(a) nBAw (b) nBAw
2 The e.m.f induced at the time the left arm of the frame is at
1 x = 10 cm from the wire is: [Online April 19, 2014]
(c) 3nBAw (d) nBAw (a) 2 mV (b) 1 mV
2
36. An ideal capacitor of capacitance 0.2 mF is charged to a (c) 0.75 mV (d) 0.5 mV
potential difference of 10V. The charging battery is then 40. A metallic rod of length ‘l’ is tied to a string of length 2l
disconnected. The capacitor is then connected to an ideal and made to rotate with angular speed w on a horizontal
inductor of self inductance 0.5mH. The current at a time table with one end of the string fixed. If there is a vertical
when the potential difference across the capacitor is 5V, is: magnetic field ‘B’ in the region, the e.m.f. induced across
[Online April 15, 2018] the ends of the rod is [2013]
(a) 0.17A (b) 0.15A (c) 0.34A (d) 0.25A
37. A copper rod of mass m slides under gravity on two smooth
parallel rails, with separation 1 and set at an angle of q
with the horizontal. At the bottom, rails are joined by a
resistance R.There is a uniform magnetic field B normal to
the plane of the rails, as shown in the figure. The terminal
speed of the copper rod is: [Online April 15, 2018]
®
B

l
2 Bwl2 3Bwl 2
R (a) (b)
q 2 2
mgR cos q mgR sin q 4 Bwl2 5Bwl 2
(a) (b) (c) (d)
B2l 2 B2l 2 2 2
mgR tan q mgR cot q 41. A coil of self inductance L is connected at one end of two
(c) (d) rails as shown in figure. A connector of length l, mass m
B2l 2 B2l 2 can slide freely over the two parallel rails. The entire set up
38. At the centre of a fixed large circular coil of radius R, a much is placed in a magnetic field of induction B going into the
smaller circular coil of radius r is placed. The two coils are page. At an instant t = 0 an initial velocity v0 is imparted to
concentric and are in the same plane. The larger coil carries it and as a result of that it starts moving along x-axis. The
a current I. The smaller coil is set to rotate with a constant displacement of the connector is represented by the figure.
angular velocity w about an axis along their common [Online May 19, 2012]
diameter. Calculate the emf induced in the smaller coil after a
time t of its start of rotation. [Online April 15, 2018]
m0 I 2 m0 I B
(a) wr sin wt (b) wpr 2 sin wt
2R 4R L
x-axis
m0 I m0 I 2
(c) wpr 2 sin wt (d) wr sin wt
2R 4R
Electromagnetic Induction P-353

l
P
Displacement

Displacement
(a) (b) RW RW v RW
I
Time Time
I2
I1 Q

Displacement Blv 2 Blv


(a) I1 = - I 2 = , I=
Displacement

6R 6R

(c) (d) Blv 2 Blv


(b) I1 = I 2 = ,I =
3R 3R
Time
Time Blv
(c) I1 = I 2 = I =
42. This question has Statement 1 and Statement 2. Of the R
four choices given after the Statements, choose the one Bl n Bl n
that best describes the two Statements. (d) I1 = I 2 = , I=
6R 3R
Statement 1: Self inductance of a long solenoid of length
L, total number of turns N and radius r is less than 46. Two coaxial solenoids are made by winding thin insulated
wire over a pipe of cross-sectional area A = 10 cm2 and
pm0 N 2 r 2 length = 20 cm. If one of the solenoid has 300 turns and the
. other 400 turns, their mutual inductance is [2008]
L
(m0 = 4p × 10 –7 Tm A–1)
Statement 2: The magnetic induction in the solenoid in
(a) 2.4p × 10–5 H (b) 4.8p × 10–4 H
m0 NI (c) 4.8p × 10–5 H (d) 2.4p × 10–4 H
Statement 1 carrying current I is in the middle of the
L 47. One conducting U tube can slide inside another as shown
solenoid but becomes less as we move towards its ends. in figure, maintaining electrical contacts between the tubes.
[Online May 19, 2012] The magnetic field B is perpendicular to the plane of the
(a) Statement 1 is true, Statement 2 is false. figure . If each tube moves towards the other at a constant
(b) Statement 1 is true, Statement 2 is true, Statement 2 is speed v, then the emf induced in the circuit in terms of B, l
the correct explanation of Statement 1. and v where l is the width of each tube, will be [2005]
(c) Statement 1 is false, Statement 2 is true.
(d) Statement 1 is true, Statement 2 is true, Statement 2 is A
B
not the correct explanation of Statement 1.
43. A boat is moving due east in a region where the earth's v v X
magnetic field is 5.0 × 10–5 NA–1 m–1 due north and
horizontal. The boat carries a vertical aerial 2 m long. If the C
speed of the boat is 1.50 ms–1, the magnitude of the induced
emf in the wire of aerial is: [2011] (a) – Blv (b) Blv
(a) 0.75 mV (b) 0.50 mV (c) 2 Blv (d) zero
(c) 0.15 mV (d) 1mV 48. A metal conductor of length 1 m rotates vertically about one
44. A horizontal straight wire 20 m long extending from east to of its ends at angular velocity 5 radians per second. If the
west falling with a speed of 5.0 m/s, at right angles horizontal component of earth’s magnetic field is 0.2×10–4T,
to the horizontal component of the earth’s magnetic field then the e.m.f. developed between the two ends of the
0.30 × 10–4 Wb/m2. The instantaneous value of the e.m.f. conductor is [2004]
induced in the wire will be [2011 RS] (a) 5 mV (b) 50 mV
(a) 3 mV (b) 4.5 mV (c) 1.5 mV (d) 6.0 mV (c) 5 mV (d) 50mV
45. A rectangular loop has a sliding connector PQ of length l 49. A coil having n turns and resistance RW is connected with
and resistance R W and it is moving with a speed v as a galvanometer of resistance 4RW. This combination is
shown. The set-up is placed in a uniform magnetic field moved in time t seconds from a magnetic field W1 weber to
going into the plane of the paper. The three currents I1, I2 W2 weber. The induced current in the circuit is [2004]
and I are [2010]
P-354 Physics

52. A conducting square loop of side L and resistance R moves


(W2 - W1 ) n (W 2 - W1 )
(a) - (b) - in its plane with a uniform velocity v perpendicular to one
Rnt 5 Rt of its sides. A magnetic induction B constant in time and
space, pointing perpendicular and into the plane at the
(W2 - W1 ) n(W2 - W1 )
(c) - (d) - loop exists everywhere with half the loop outside the field,
5 Rnt Rt as shown in figure. The induced emf is [2002]
50. Two coils are placed close to each other. The mutual
inductance of the pair of coils depends upon [2003]
(a) the rates at which currents are changing in the two
coils L v
(b) relative position and orientation of the two coils
(c) the materials of the wires of the coils
(d) the currents in the two coils
51. When the current changes from +2 A to –2A in 0.05 second,
an e.m.f. of 8 V is induced in a coil. The coefficient of self (a) zero (b) RvB (c) vBL/R (d) vBL
-induction of the coil is [2003]
(a) 0.2 H (b) 0.4 H (c) 0.8 H (d) 0.1 H
Electromagnetic Induction P-355

1. (5) e
For coil C1, No. of turns N1 = 500 and radius, r = 1 cm. Current, i =
R
For coil C2, No. of turns N2 = 200 and radius, R = 20 cm l
But, resistance of wire, R = r
2 dI A
I = (5t - 2t + 3) Þ = (10t - 2)
dt
dB ( A)2 0.032 ´ {p´ 2 ´ 10-3 }2
æ m IN ö \i = = = 0.61 A.
fsmall = BA = ç 0 2 ÷ (pr 2 ) dt rl 1.23 ´10-8 ´ 0.3
è 2R ø
5. (15)
Induced emf in small coil,
Here, B = 3.0 × 10–5 T, R = 10 cm = 0.1 m
d f æ m 0 N 2 ö 2 di æ m 0 N1 N 2 pr 2 ö 2p p
e= = pr N1 = ç ÷ (10t - 2) w= =
dt çè 2r ÷ø dt è 2R ø 2T 0.2
r r
At t = 1 s Flux as a function of time f = B × A = AB cos(wt )

æ m N N pr 2 ö m N N pr 2 -d f
e=ç 0 1 2 8=4 0 1 2 Emf induced, e = = ABw sin(wt )
2R ÷ R
dt
è ø Max. value of Emf = ABw = pR2Bw
4(4p)10-7 ´ 200 10 -4 p
= ´ 500 ´ -2 p = 3.14 ´ 0.1 ´ 0.1 ´ 3 ´ 10 -5 ´
20 10 0.2
= 80 ´ p 2 ´ 10-7 ´ 10 ´ 102 ´ 10-2 = 15 ´ 10 -6 V = 15 mV
4 6. (10) Given dI = 0.25 – 0 = 0.25 A
= 8 ´ 10-4 volt = 0.8 mV= Þ x = 5.
x dt = 0.025 ms
2. (b) Case (a) : When bar magnet is entering with constant Induced voltage
speed, flux (f) will change and an e.m.f. is induced, so Eind = 100 v
galvanometer will deflect in positive direction. Self-inductance, L = ?
Case (b) : When magnet is completely inside, flux (f) will Df L (0.25 – 0)
not change, so galvanometer will show null deflection. Using, Eind = Þ 100 =
Dt .025 ´10-3
Case (c) : When bar magnet is making on exit, again flux Þ L = 10 H = 10 mH
–3
(f) will change and an e.m.f. is induced in opposite direction
7. (a) According to question, dB = 1000 – 500 = 500 gauss
so galvanometer will deflect in negative direction i.e.
= 500 × 10–4T
reverse direction.
Time dt = 5 s
3. (a) As we know, emf e = NABw cos wt , Here N = 1 Using faraday law
Average power,
df dB
Induced EMF , e = – = A
e 2 2 2
A B w cos wt 2 2 A2 B 2 w 2 æ 1 ö dt dt
<P> = < >=< >= çè ÷ø
R R R 2 dB 1000 – 500
= ´ 10 –4 = 10 –2 T/sec
Therefore average power loss in the loop due to Joule dt 5
heating
p 2 a 2 b2 B 2 2
<P>= (w )
2R
4. (b) Given,
Length of wire, l = 30 cm
Radius of wire, r = 2 mm = 2 × 10–3 m
Area, A = ar of X –2 ar of D = (16 × 4 – 2 × Area of triangle) cm2
Resistivity of metal wire, r = 1.23 ´ 10 - 8 W m
æ 1 ö
= ç 64 – 2 ´ ´ 2 ´ 4 ÷ cm 2
d f dB è 2 ø
Emf generated, | e | = = ( A) (Q f = B.A.)
dt dt = 56 × 10–4 m2
P-356 Physics

dB p
\ einduced = A = 56 ´ 10 –4 ´ 10–2 = 56 ´ 10 –6 V = 56mV wt = p Þ t= =5s
dt p
5
8. (d) As magnetic field lines form close loop, hence every
magnetic field line creating magnetic flux through the inner \ Induced emf is zero at t = 5 s
region (fi) must be passing through the outer region. 12. (a) Q = BA
Since flux in two regions are in opposite region. = (m0 ni)A
\ fi = –f0 = m0 n (kt e–at)A
dQ d
e=- = -m 0 nAk (te -a t )
dt dt

9. (d) According to question, = -m0 nAk[t (-1)e- at + e -at ´1]


I(t) = I0t(1 – t)
= - m0 nAk [e -at (1 - t )]
\ I = I0t – I0t2
f = B.A e - m0 nAk -at
f = (m0 nI) × (pR2) i= = [e (1 - t )]
R R
(Q B = m0 nI and A = pR 2 ) At t = 0, i Þ –ve
13. (a) Qcoil = ( NQ) µ i
–df
VR =
dt Q1 i1 3
So, = =
Q2 i2 2
VR = m0 npR 2 ( I0 – 2I 0t )
2 2 -3
Þ VR = 0 at t =
1
s or Q2 = Q1 = ´ 10 = 6.67 × 10–4 Wb
3 3
2
14. (b) According to faraday’s law of electromagnetic induc-
10. (175.00)
-df
tion, e =
dt
di 15 5
L´ = 25 Þ L ´ = 25 or L = H
dt 1 3
Change in the energy of the inductance,
1 1 5
( )
DE = L i12 – i 22 = ´ ´ (252 –10 2 )
2 2 3
Flux through the loop ABCDEFA, 5
r r = ´ 525 = 437.5J
f = B.A = (3iˆ + 4k).(25i
ˆ ˆ + 25k)
ˆ 6
Þ f = (3 × 25) + (4 × 25) = 175 weber 15. [B onus]
11. (b) We have given, time period, T = 10s Df 1 1
Net charge Q = = A(Bf - Bi ) = ´ 3.5 ´10 -3
2p p R 10 10
\ Angular velocity, w = =
æ p ö
10 5
ç 0.4sin - 0 ÷
Magnetic flux, f(t)= BA cos wt è 2 ø
–d f 1
Emf induced, E = = BAw sin wt = BAw sin ( wt ) = (3.5 ´ 10-3 )(0.4 - 0)
dt 10
= 1.4 × 10– 4
p No option matches, So it should be a bonus.
Induced emf, | e | is maximum when wt =
2 16. (a) According to Faraday's law of electromagnetic
p df
Þ t= = 2.5 s induction, e =
2 dt
p Also, e = iR
df
5 \ iR = Þ ò d f = R ò idt
dt
For induced emf to be minimum i.e zero
Electromagnetic Induction P-357

Magnitude of change in flux (df) = R × area under current Df


vs time graph 24. (c) As e = or Ri = Df (Q e = Ri )
Dt Dt
1 1 Þ Df = R(i.Dt)
or, df = 100 ´ ´ ´ 10 = 250 Wb
2 2 = R × area under i – t graph
17. (b) Electric flux is given by 1
f = B.A = 10 × × 4 × 0.1 = 2 weber
2
f = B0 pr 2 e - t / t (Q B = B0e-t/ t ) 25. (b) Electric flux, f = 10t2 – 50t + 250
df
df B0 pr 2 - t / t Induced emf, e = - = - (20t - 50)
Induced E.m.f. e = = e dt
dt t2 et = 3 = –10 V
26. (d) Magnetic field at a distance r from the wire
¥ 2
e p 2 r 4 B02
Heat = ò R 2tR
=
B=
m0 I
0 2 pr
18. (d) According to Faraday’s law of electromagnetic
Magnetic flux for small displacement dr,
induction,
f = B × A = Bldr [Q A = l dr and B.A = BA cos 0°]
Ldi
Induced emf, e =
dt m0 I
Þf= l dr
æ 5–2 ö 2pr
50 = L ç ÷
è 0.1sec ø
50 ´ 0.1 5
Þ L= = = 1.67 H
3 3
I V
19. (a) Inside the sphere field varies linearly i.e., E µ r with
1
distance and outside varies according to E µ r
r2 dr
Hence the variation is shown by curve (a)
d f m 0 Il dr m Ivl
20. (a) Given: No. of turns N = 1000 Emf, e = = × Þe= 0 ×
dt 2 pr dt 2p r
Face area, A = 4 cm2 = 4 × 10–4 m2
e m 0 Ivl
Change in magnetic field, Induce current in the loop, i = = ×
R 2p Rr
DB = 10–2 wbm–2 27. (b) Induced emf,
Time taken, t = 0.01s = 10–2 sec e = Bvl= 1 × 10–2 × 0.05 = 5 × 10–4 V
Emf induced in the coil e = ? Equivalent resistance,
Applying formula, 4´ 2 4
R= + 1.7 = + 1.7 ; 3 W
-d f DB ö 4+2 3
Induced emf, e = = N æç ÷ A cos q
dt è Dt ø e 5 ´10-4
Current, i = = ; 170 m A
1000 ´ 10-2 ´ 4 ´ 10 -4 R 3
= = 400 mV
10-2
m0 N 2 A
21. (a) As we know, Magnetic flux, f = B. A 28. (d) Inductance =
L
m0 (2)(20 ´ 10 -2 ) 2 29. (c) Force on the strip when it is at stretched position x
´ p(0.3 ´ 10 -2 ) 2
2 2 from mean position is
2[(0.2) + (0.15) ]
On solving BIv
= 9.216 × 10–11 = 9.2 × 10–11 weber F = -kx - iIB = -kx - ´ IB
R
22. (a) Induced emf
- di B2 I 2
eµ F = -kx - ´v
dt R
23. (d) Because of the Lenz’s law of conservation of energy. Above expression shows that it is case of damped
Length of straignt wire, l = 20m Earth’s Magneti field, oscillation, so its amplitude can be given by
B = 0.30 × 10–4 Wb/m2.
P-358 Physics

bt 33. (a) Potential difference between two faces


-
Þ A = A0 e 2m perpendicular to x-axis

bt
= l V.B = 2 ´10 –2 (6 ´ 0.1) =12mV
A - A0
Þ 0 = A0 e 2 m [as per question A = ] 34. (c)
e e

2m 2 ´ 50 ´ 10-3 ´ 10
Þ t= =
æ B2I 2 ö 0.01 ´ 0.01 Magnetic moment, M = NIA
çç ÷÷ dQ = r dx
è R ø
dQ
Given, m = 50 × 10–3 kg dI = .w
2p
B = 0.1 T
l = 0.1 m dM = dI × A
R = 10 W l
w r0 r0
k = 0.5 N = . . x p x 2 dx Þ M = n p ò x 3 dx
2p l l
0
m p
Time period, T = 2p ; 2s = . nrl
3
k 4
so, required number of oscillations, df
35. (a) Induced emf in a coil, e = - = NBAsin wt
10000 dt
N= = 5000 Also, e = e0 sin wt
2
\ Maximum emf induced, e0 = nBAw
30. (a) Induced emf, e = Bvl 36. (a) Given: Capacitance, C = 0.2 mF = 0.2 × 10–6 F
= 0.3 × 10–4 × 5 × 10 Inductance L = 0.5 mH = 0.5 × 10–3 H
= 1.5 × 10–3 V Current I = ?
31. (d) The rate of mutual inductance is given by Using energy conservation
M = m0n1n2 pr12 1 1 1
...(i) CV 2 = CV12 + LI 2
2 2 2
The rate of self inductance is given by
1
´ 0.2 ´10 –6 ´10 2 + 0
L = m 0 n12 pr12 ...(ii) 2
M n2 1 –6 2 1 –3 2
Dividing (i) by (ii) Þ = = ´ 0.2 ´10 ´ 5 + ´ 0.5 ´10 I
L n1 2 2
\ I = 3 ´ 10 –1 A = 0.17 A
32. (c) As total length L of the wire will remain constant
37. (b) From Faraday’s law of electomagnetic induction,
L = (3a) N (N = total turns )
d f d ( BA) d ( Bll )
and length of winding = (d) N e= = =
dt lt dt
Bdl ´ l
= = BVl
dt
a a
æ BV ö 2 B 2l 2V
Also, F = ilB = ç (l B ) =
è R ÷ø R
At equilibrium
a
(d = diameter of wire) ®
B
self inductance = m0n2Al
l
æ 3a 2 ö
= m0n2 ç ÷ dN R
è 4 ø q
1
µ a2 N µ a [as N = L/3a Þ Nµ ] B 2lV mgR sin q
a mg sin q = ÞV =
Now ‘a’ increased to ‘3a’
R B 2l 2
So self inductance will become 3 times
Electromagnetic Induction P-359

38. (c) According to Faraday’s law of electromagnetic 45. (b) Due to the movement of resistor R, an emf equal to
induction, Blv will be induced in it as shown in figure clearly,
df
e= - and f = BA cos wt = Bpr 2 cos wt P
dt
l Blv
d 2 2
Þ e = - (pr B cos wt ) = pr B sin wt (w )
dt RW
RW v RW
m0 I æ m0 I ö
\= e pwr 2 sin wt çQ= B ÷ I
2R è 2R ø
39. (b) In the given question, I2
I1 Q
Current flowing through the wire, I = 1A
Speed of the frame, v = 10 ms–1
Side of square loop, l = 10 cm I = I1 + I 2 Also, I1 = I2
Distance of square frame from current carrying wires Solving the circuit,
x = 10 cm.
We have to find, e.m.f induced e = ? Blv
According to Biot-Savart’s law we get I1 = I 2 =
3R
m Idlsin q
B= 0 2 Blv
4p x 2 and I = 2 I1 =
3R
4p´10-7 1´ 10-1 46. (d) Given, Area of cross-section of pipe,
= ´
4p
( )
2
10-1 A = 10 cm2
Length of pipe, l = 20 cm
= 10–6
Induced e.m.f. e = Blv m0 N1 N2 A
M=
= 10–6 × 10–1 × 10 = 1 mv l
40. (d) Here, induced e.m.f.
4p ´ 10-7 ´ 300 ´ 400 ´ 100 ´ 10 -4
w l =
2l 0.2
x
dx m0 N1 N2 A
M=
l
3l
[(3l)2 – (2l)2 ] = 2.4p × 10–4 H
e= ò (wx) Bdx = Bw 2
2l 47. (c) Relative velocity of the tube of width l,
5 Bl 2 w = v – (–v) v = 2v
=
2 \ Induced emf. = B.l (2v)
41. (d)
42. (b) Self inductance of a long solenoid is given by 48. (b) Given, length of conductor l = 1m,
Angular speed, w = 5 rad/s,
m0 N 2 A
L= Magnetic field, B = 0.2 × 10–4 T
l
EmF generated between two ends of conductor
Magnetic field at the centre of solenoid
m0 NI Bwl 2 0.2 ´ 10-4 ´ 5 ´ 1
B= e= = = 50mV
l 2 2
So both the statements are correct and statement 2 is
correct explanation of statement 1 Df (W2 - W1 )
43. (d) As magnetic field lines form close loop, 49. (b) =
Dt t
hence every magnetic field line creating magnetic flux
through the inner region (fi) must be passing through Rtot = ( R + 4 R )W = 5R W
the outer region. Since flux in two regions are in opposite
region. nd f - n(W2 - W1 )
i= =
Rtot dt 5Rt
\ fi = –f0
(Q W2 & W1 are magnetic flux)
44. (a) Induced, emF, e = Bvl
50. (b) Mutual inductance depends on the relative position
= 0.3 × 10–4 × 5 × 20
and orientation of the two coils.
= 3 × 10–3 V = 3 mV.
P-360 Physics

51. (d) Induced emf, r r


-d f d ( B × A) -d ( BA cos0º )
Df -D ( LI ) DI e= =- =
e=- = = -L dt dt dt
Dt Dt Dt
× × × ×
DI A B

\ | e |= L
Dt × × × ×

[2 – (–2)] × l
V
Þ 8= L ´ × × ×
0.05
× × × ×
8 ´ 0.05
Þ L= = 0.1H × ×
D
×
X
×
C

4
52. (d) As the side BC is outside the field, no emf is induced dA d (l ´ x )
\ e = –B = -B
across BC. Further, sides AB and CD are not cutting any dt dt
flux. So, they will not centribute in flux. dx
\ e = - Bl = - Blv
Only side AD is cutting the flux 50 emf will be induced due dt
to AD only.
The induced emf is
21
Alternating Current
( B pr w )2 ( B pr 2 w ) 2
Alternating Current, (a) (b)
TOPIC 1 2R 8R
Voltage and Power
B pr 2 w ( B pr w 2 ) 2
1. An alternating voltage v(t) = 220 sin 100Àt volt is applied (c) (d)
2R 8R
to a purely resistive load of 50W. The time taken for the 6. Alternating current can not be measured by D.C. ammeter
current to rise from half of the peak value to the peak
because [2004]
value is : [8 April 2019 I]
(a) 5 ms (b) 2.2 ms (c) 7.2 ms (d) 3.3 ms (a) Average value of current for complete cycle is zero
2. A small circular loop of wire of radius a is located at the (b) A.C. Changes direction
centre of a much larger circular wire loop of radius b. The (c) A.C. can not pass through D.C. Ammeter
two loops are in the same plane. The outer loop of radius (d) D.C. Ammeter will get damaged.
b carries an alternating current I = Io cos (wt). The emf
induced in the smaller inner loop is nearly : AC Circuit, LCR Circuit,
[Online April 8, 2017] TOPIC 2
Quality and Power Factor
pm0 Io a 2 2
(a) . w sin (wt) (b) pm0 Io . a w cos (wt) 7. A part of a complete circuit is shown in the figure. At some
2 b 2 b instant, the value of current I is 1 A and it is decreasing at a
a2 pm0 I o b 2 rate of 102A s–1. The value of the potential difference VP –VQ,
(c) pm0 Io w sin (wt) (d) w cos ( wt) (in volts) at that instant, is ______.
b a
3. A sinusoidal voltage V(t) = 100 sin (500t) is applied across [NA Sep. 06, 2020 (I)]
a pure inductance of L = 0.02 H. The current through the R=2W
L=50 mH I
coil is: [Online April 12, 2014]
P Q
(a) 10 cos (500 t) (b) – 10 cos (500t) 30 V
(c) 10 sin (500t) (d) – 10 sin (500t) 8. An AC circuit has R = 100 W, C = 2 mF and L = 80 mH,
4. In an a.c. circuit the voltage applied is E = E0 sin wt. The connected in series. The quality factor of the circuit is :
æ pö [Sep. 06, 2020 (I)]
resulting current in the circuit is I = I 0 sin ç wt - ÷ . The (a) 2 (b) 0.5
è 2ø
(c) 20 (d) 400
power consumption in the circuit is given by [2007]
E I 9. In a series LR circuit, power of 400 W is dissipated from a
(a) P = 2 E0 I0 (b) P = 0 0 source of 250 V, 50 Hz. The power factor of the circuit is 0.8. In
2 order to bring the power factor to unity, a capacitor of value C
E0 I 0 is added in series to the L and R. Taking the value C as
(c) P = zero (d) P =
2 æ nö
5. In a uniform magnetic field of induction B a wire in the çè ÷ø mF , then value of n is ______. [NA Sep. 06, 2020 (II)]
3p
form of a semicircle of radius r rotates about the diameter
of the circle with an angular frequency w. The axis of 10. A series L-R circuit is connected to a battery of emf V. If
rotation is perpendicular to the field. If the total resistance the circuit is switched on at t = 0, then the time at which the
of the circuit is R, the mean power generated per period of æ 1ö
rotation is [2004] energy stored in the inductor reaches ç ÷ times of its
è nø
maximum value, is : [Sep. 04, 2020 (II)]
P-362 Physics

L æ n ö L æ n +1ö
(a) R ln ç ÷ (b) ln ç ÷
è n -1 ø R è n -1 ø
L æ n ö L æ n -1 ö
(c) ln ç ÷ (d) ln ç ÷
R è n +1ø R è n ø
2 1
11. A 750 Hz, 20 V (rms) source is connected to a resistance of (a) ln 2 (b) ln 2
100 W, an inductance of 0.1803 H and a capacitance of 10 2
mF all in series. The time in which the resistance (heat (c) 2 ln 2 (d) ln 2
capacity 2 J/°C) will get heated by 10°C. (assume no loss 16. A circuit connected to an ac source of emf e = e0sin(100t)
of heat to the surroundings) is close to : p
with t in seconds, gives a phase difference of between
[Sep. 03, 2020 (I)] 4
(a) 418 s (b) 245 s the emf e and current i. Which of the following circuits
will exhibit this ? [8 April 2019 II]
(c) 365 s (d) 348 s
12. An inductance coil has a reactance of 100 W. When an AC (a) RL circuit with R = 1 kW and L = 10 mH
signal of frequency 1000 Hz is applied to the coil, the applied (b) RL circuit with R = 1 kW and L = 1 mH
voltage leads the current by 45°. The self-inductance of
the coil is : [Sep. 02, 2020 (II)] (d) RC circuit with R = 1 kW and C = 1 mF
(a) 1.1 × 10–2 H (b) 1.1 × 10–1 H (d) RC circuit with R = 1 kW and C = 10 mF.
(c) 5.5 × 10–5 H (d) 6.7 × 10–7 H 17. In the figure shown, a circuit contains two identical
13. Consider the LR circuit shown in the figure. If the switch resistors with resistance R = 5 W and an inductance with L
S is closed at t = 0 then the amount of charge that passes = 2 mH. An ideal battery of 15 V is connected in the circuit.
What will be the current through the battery long after the
L switch is closed? [12 Jan. 2019 I]
through the battery between t = 0 and t = is :
R
[12 April 2019 II] S
L

2.7EL EL
(a) 2 (b) (a) 5.5 A (b) 7.5 A
R 2.7 R 2 (c) 3 A (d) 6 A
I2
7.3EL EL 18.
(c) (d) C R2
R2 7.3R 2
L R1
I1
14. A coil of self inductance 10 mH and resistance 0.1 W is
connected through a switch to a battery of internal
resistance 0.9 W. After the switch is closed, the time taken
for the current to attain 80% of the saturation value is ~
[take ln 5 = 1.6] [10 April 2019 II]
3 3
(a) 0.324 s (b) 0.103 s In the above circuit, C = µF, R2 = 20 W, L = H and
2 10
(c) 0.002 s (d) 0.016 s
R1 = 10 W. Current in L-R1 path is I1 and in C-R2 path it is
15. A 20 Henry inductor coil is connected to a 10 ohm
I2. The voltage of A.C source is given by, V = 200 2 sin
resistance in series as shown in figure. The time at which
rate of dissipation of energy (Joule’s heat) across (100 t) volts. The phase difference between I1 and I2 is :
resistance is equal to the rate at which magnetic energy is [12 Jan. 2019 II]
stored in the inductor, is : [8 April 2019 I] (a) 60° (b) 30°
(c) 90° (d) 0
Alternating Current P-363

19. In the circuit shown, ÎR Î L æ 1ö


(a) (b) ç1 - ÷
R L eL2
R2 è e ø
ÎL ÎR
(c) 2 (d)
R eL2
S2 23. A LCR circuit behaves like a damped harmonic oscillator.
Comparing it with a physical spring-mass damped
S1 oscillator having damping constant ‘b’, the correct
equivalence would be: [7 Jan. 2020 I]
e (a) L « m, C « k, R « b
1 1 1
the switch S1 is closed at time t = 0 and the switch S2 is (b) L « , C « , R «
kept open. At some later time (t0), the switch S1 is opened b m k
(c) L « k, C « b, R « m
and S2 is closed. the behaviour of the current I as a function
of time ‘t’ is given by: [11 Jan. 2019 II] 1
(d) L « m, C « , R « b
k
I I
24. An emf of 20 V is applied at time t = 0 to a circuit containing
in series 10 mH inductor and 5 W resistor. The ratio of the
currents at time t = ¥ and at t = 40 s is close to:
(a) (b) (Take e2 = 7.389) [7 Jan. 2020 II]
(a) 1.06 (b) 1.15
tO t tO t
(c) 1.46 (d) 0.84
I
I 25. In an a.c. circuit, the instantaneous e.m.f. and current are
given by
e = 100 sin 30 t
æ pö
(c) (d) i = 20 sin ç 30 t - ÷
è 4ø
tO t In one cycle of a.c., the average power consumed by the
tO t
circuit and the wattless current are, respectively: [2018]
20. A series AC circuit containing an inductor (20 mH), a 1000
(a) 50W, 10A (b) W, 10A
capacitor (120 mF) and a resistor (60 W) is driven by an 2
AC source of 24 V/50 Hz. The energy dissipated in the 50
(c) W, 0 (d) 50W, 0
circuit in 60 s is: [9 Jan. 2019 I] 2
(a) 5.65 × 102 J (b) 2.26 × 103 J 26. For an RLC circuit driven with voltage of amplitude vm and

(c) 5.17 × 102 J (d) 3.39 × 103 J 1


frequency w0 = the current exhibits resonance. The
LC
21. In LC circuit the inductance L = 40 mH and capacitance C
= 100 mF. If a voltage V(t) = 10 sin(314 t) is applied to the quality factor, Q is given by: [2018]
circuit, the current in the circuit is given as: w0 L w0 R R CR
(a) (b) (c) (d)
[9 Jan. 2019 II] R L (w0C) w0
(a) 0.52 cos 314 t (b) 10 cos 314 t 27. A sinusoidal voltage of peak value 283 V and angular
frequency 320/s is applied to a series LCR circuit. Given
(c) 5.2 cos 314 t (d) 0.52 sin 314 t
that R = 5 W, L= 25 mH and C = 1000 mF. The total impedance,
and phase difference between the voltage across the
22. source and the current will respectively be :
[Online April 9, 2017]

(a) 10 W and tan–1 æç 5 ö÷ (b) 7 W and 45°


As shown in the figure, a battery of emf Î is connected to an è 3ø
inductor L and resistance R in series. The switch is closed at
æ8ö
(c) 10 W and tan –1 ç ÷ (d) 7 W and tan–1 æç ö÷
t = 0. The total charge that flows from the battery, between t 5
= 0 and t = tc (tc is the time constant of the circuit) is: è3ø è 3ø
[8 Jan. 2020 II]
P-364 Physics

28. An arc lamp requires a direct current of 10 A at 80 V to (a) 6.7 mA (b) 0.67 mA
function. If it is connected to a 220 V (rms), 50 Hz AC (c) 100 mA (d) 67 mA
supply, the series inductor needed for it to work is close to : 31. An LCR circuit is equivalent to a damped pendulum. In an
[2016] LCR circuit the capacitor is charged to Q0 and then
(a) 0.044 H (b) 0.065 H connected to the L and R as shown below :
(c) 80 H (d) 0.08 H
29. A series LR circuit is connected to a voltage source with R L
V(t) = V0 sinwt. After very large time, current l(t) behaves
æ Lö
as çè t 0 >> ÷ø : [Online April 9, 2016] C
R
I(t) If a student plots graphs of the square of maximum charge
( )
Q 2Max on the capacitor with time(t) for two different
values L1 and L2 (L1 > L2) of L then which of the following
represents this graph correctly ? (plots are schematic and
(a) not drawn to scale) [2015]
t
t = t0
L1
2 2
I(t) QMax QMax Q0 (For both L1 and L2)
(a) L2 (b)
t t

t
(b) t = t L2
0 2 L1 2
QMax Q
(c) L2 (d) Max L1
t t
I(t)
32. For the LCR circuit, shown here, the current is observed
to lead the applied voltage. An additional capacitor C’,
when joined with the capacitor C present in the circuit,
(c) makes the power factor of the circuit unity. The
t capacitor C’, must have been connected in :
t = t0 [Online April 11, 2015]

I(t) L C R

t
(d) t0

~
V = V0sintw
30. An inductor (L = 0.03 H) and a resistor (R = 0.15 kW) are
connected in series to a battery of 15V emf in a circuit
C
shown below. The key K1 has been kept closed for a long (a) series with C and has a magnitude
2
time. Then at t = 0, K1 is opened and key K2 is closed (w LC –1)
simultaneously. At t = l ms, the current in the circuit will 1 - w2 LC
(b) series with C and has a magnitude
be : ( e5 @ 150 ) [2015] w2 L
0.03 H 0.15 kW 1 - w2 LC
(c) parallel with C and has a magnitude
w2 L
K2 C
(d) parallel with C and has a magnitude 2
(w LC - 1)

15V K1
Alternating Current P-365

33. In the circuits (a) and (b) switches S1 and S2 are closed V
at t = 0 and are kept closed for a long time. The variation of
current in the two circuits for t ³ 0 are roughly shown by
R
figure (figures are schematic and not drawn to scale) :
S1
[Online April 10, 2015]

C S2
C R L R
S1 L
E E (a) Work done by the battery is half of the energy
(a) (b) dissipated in the resistor
(b) At, t = t, q = CV/2
E E
R (b) R (a)
(c) At, t = 2t, q = CV (1 – e–2)
(a)
(a) i (b) i (d) At, t = 2 t, q = CV (1 – e–1)
(b) 37. A series LR circuit is connected to an ac source of
t frequency w and the inductive reactance is equal to 2R. A
t
capacitance of capacitive reactance equal to R is added in
E E series with L and R. The ratio of the new power factor to
R (a) R (b) the old one is : [Online April 25, 2013]
(b)
(c) i (d) i (a)
2 2 3 5
(a) (b) (c) (d)
3 5 2 2
t t
38. When resonance is produced in a series LCR circuit, then
34. In the circuit shown here, the point ‘C’ is kept connected which of the following is not correct ?
to point ‘A’ till the current flowing through the circuit
[Online April 25, 2013]
becomes constant. Afterward, suddenly, point ‘C’ is
disconnected from point ‘A’ and connected to point ‘B’ at (a) Current in the circuit is in phase with the applied
time t = 0. Ratio of the voltage across resistance and the voltage.
inductor at t = L/R will be equal to: [2014] (b) Inductive and capacitive reactances are equal.
(c) If R is reduced, the voltage across capacitor will
A C R increase.
(d) Impedance of the circuit is maximum.
39. The plot given below is of the average power delivered to
L
an LRC circuit versus frequency. The quality factor of the
circuit is : [Online April 23, 2013]
B
average power (microwatts)

e 1- e 1.0
(a) (b) 1 (c) –1 (d)
1- e e
35. When the rms voltages VL, VC and VR are measured 0.5
respectively across the inductor L, the capacitor C and the
resistor R in a series LCR circuit connected to an AC source,
it is found that the ratio VL : VC : VR = 1 : 2 : 3. If the rms 0.0
voltage of the AC sources is 100 V, the VR is close to:
3 4 5 6 7
[Online April 9, 2014] frequency (kHz)
(a) 50 V (b) 70 V (c) 90 V (d) 100 V
36. In an LCR circuit as shown below both switches are open (a) 5.0 (b) 2.0 (c) 2.5 (d) 0.4
initially. Now switch S1 is closed, S2 kept open. (q is charge 40. In a series L-C-R circuit, C = 10 Farad, L = 10–5 Henry
–11

on the capacitor and t = RC is Capacitive time constant). and R = 100 Ohm, when a constant D.C. voltage E is applied
Which of the following statement is correct ? [2013] to the circuit, the capacitor acquires a charge 10 –9 C. The
D.C. source is replaced by a sinusoidal voltage source in
P-366 Physics

which the peak voltage E0 is equal to the constant D.C. 45. A fully charged capacitor C with initial charge q0 is
voltage E. At resonance the peak value of the charge connected to a coil of self inductance L at t = 0. The time at
acquired by the capacitor will be : [Online April 22, 2013] which the energy is stored equally between the electric
and the magnetic fields is: [2011]
(a) 10–15 C (b) 10–6 C (c) 10–10 C (d) 10–8 C
p
41. An LCR circuit as shown in the figure is connected to a (a) LC (b) 2p LC
4
voltage source Vac whose frequency can be varied.
(c) LC (d) p LC
V 46. A resistor ‘R’ and 2µF capacitor in series is connected
through a switch to 200 V direct supply. Across the
capacitor is a neon bulb that lights up at 120 V. Calculate
24 H 2 µF the value of R to make the bulb light up 5 s after the switch
15 W
has been closed. (log10 2.5 = 0.4) [2011]
(a) 1.7 × 105 W (b) 2.7 × 106 W
~
Vac = V0 sin wt (c) 3.3 × 107 W (d) 1.3 × 104 W
The frequency, at which the voltage across the resistor is 47. Combination of two identical capacitors, a resistor R and
maximum, is : [Online April 22, 2013] a dc voltage source of voltage 6V is used in an experiment
(a) 902 Hz (b) 143 Hz (c) 23 Hz (d) 345 Hz on a (C-R) circuit. It is found that for a parallel combination
42. In the circuit shown here, the voltage across E and C are of the capacitor the time in which the voltage of the fully
respectively 300 V and 400 V. The voltage E of the ac source charged combination reduces to half its original voltage is
is : [Online April 9, 2013] 10 second. For series combination the time for needed for
L reducing the voltage of the fully charged series
combination by half is [2011 RS]
(a) 10 second (b) 5 second
C
(c) 2.5 second (d) 20 second
~ E
48. In the circuit shown below, the key K is closed at t = 0. The
current through the battery is [2010]
(a) 400 Volt (b) 500 Volt(c) 100 Volt (d) 700 Volt
43. A resistance R and a capacitance C are connected in series V K

to a battery of negligible internal resistance through a key.


The key is closed at t = 0. If after t sec the voltage across
L R1
the capacitance was seven times the voltage across R, the
value of t is [Online May 12, 2012]
(a) 3 RC ln 2 (b) 2 RC ln 2 R2

(c) 2 RC ln 7 (d) 3 RC ln 7
VR1R2 V
44. In an LCR circuit shown in the following figure, what will (a) at t = 0 and R at t = ¥
2
be the readings of the voltmeter across the resistor and R12 + R22
ammeter if an a.c. source of 220V and 100 Hz is connected V ( R1 + R2 )
V
to it as shown? [Online May 7, 2012] (b) at t = 0 and at t = ¥
R2 R1 R2
L 100 W V VR1R2
C (c) at t = 0 and at t = ¥
R2 R 2 + R2
1 2
V ( R1 + R2 ) V
A (d) at t = 0 and at t = ¥
V V V R1 R2 R2
300V 300 V VR 49. In a series LCR circuit R = 200W and the voltage and the
frequency of the main supply is 220V and 50 Hz
220 V, 100 Hz respectively. On taking out the capacitance from the circuit
the current lags behind the voltage by 30°. On taking out
(a) 800 V, 8 A (b) 110 V, 1.1 A the inductor from the circuit the current leads the voltage
by 30°. The power dissipated in the LCR circuit is [2010]
(c) 300 V, 3 A (d) 220V, 2.2 A
(a) 305 W (b) 210 W (c) Zero W (d) 242 W
Alternating Current P-367

50. E 56. A coil of inductance 300 mH and resistance 2 W is


L connected to a source of voltage 2V. The current reaches
R1 half of its steady state value in [2005]
(a) 0.1 s (b) 0.05 s (c) 0.3 s (d) 0.15 s
R2 57. The self inductance of the motor of an electric fan is 10 H.
S In order to impart maximum power at 50 Hz, it should be
connected to a capacitance of [2005]
An inductor of inductance L = 400 mH and resistors of
resistance R1 = 2W and R2 = 2W are connected to a battery (a) 8 mF (b) 4 mF (c) 2 mF (d) 1 mF
of emf 12 V as shown in the figure. The internal resistance 58. In an LCR series a.c. circuit, the voltage across each of the
of the battery is negligible. The switch S is closed at t = 0. components, L, C and R is 50V. The voltage across the LC
The potential drop across L as a function of time is [2009] combination will be [2004]

(a)
12 -3t
t
e V (
(b) 6 1 - e )
-t / 0.2
V (a) 100 V (b) 50 2 V
(c) 50 V (d) 0 V (zero)
(c) 12e–5t V (d) 6e–5t V
59. In a LCR circuit capacitance is changed from C to 2 C. For
51. In a series resonant LCR circuit, the voltage across R is
the resonant frequency to remain unchanged, the
100 volts and R = 1 kW with C = 2mF. The resonant
inductance should be changed from L to [2004]
frequency w is 200 rad/s. At resonance the voltage across
L is [2006] (a) L/2 (b) 2 L (c) 4 L (d) L/4
–2
(a) 2.5 × 10 V (b) 40 V 60. The power factor of an AC circuit having resistance (R)
and inductance (L) connected in series and an angular
(c) 250 V (d) 4 × 10–3 V
velocity w is [2002]
52. An inductor (L = 100 mH), a resistor (R = 100 W) and a
battery (E = 100 V) are initially connected in series as (a) R/ w L (b) R/(R2 + w 2L2)1/2
shown in the figure. After a long time the battery is (c) w L/R (d) R/(R2 – w 2L2)1/2
disconnected after short circuiting the points A and B.
61. The inductance between A and D is [2002]
The current in the circuit 1 ms after the short circuit is
[2006]

L A 3H 3H
3H D

(a) 3.66 H (b) 9 H


R
(c) 0.66 H (d) 1 H
A B
Transformers and LC
E TOPIC 3
Oscillations
(a) 1/eA (b) eA (c) 0.1 A (d) 1 A
62. For the given input voltage waveform Vin(t), the output
53. In an AC generator, a coil with N turns, all of the same area
voltage waveform Vo(t), across the capacitor is correctly
A and total resistance R, rotates with frequency w in a
depicted by : [Sep. 06, 2020 (I)]
magnetic field B. The maximum value of emf generated in
the coil is [2006] 1kW
(a) N.A.B.R.w (b) N.A.B +5V 5m s
(c) N.A.B.R. (d) N.A.B.w 10nF VO(t)
54. The phase difference between the alternating current and 0V t
m
5

p
s

emf is . Which of the following cannot be the constituent 0


2
of the circuit? [2005] Vo(t)
(a) R, L (b) C alone(c) L alone (d) L, C 3V
55. A circuit has a resistance of 12 ohm and an impedance of
15 ohm. The power factor of the circuit will be [2005] (a) 2V
(a) 0.4 (b) 0.8 (c) 0.125 (d) 1.25
5ms 10ms 15ms t
P-368 Physics

Vo(t) having 4000 turns. The output power is delivered at 230


V by the transformer. If the current in the primary of the
2V
(b) transformer is 5A and its efficiency is 90%, the output
current would be: [9 Jan. 2019 II]

5ms 10ms 15ms t (a) 50 A (b) 45 A (c) 35 A (d) 25 A


Vo(t) 65. A power transmission line feeds input power at 2300 V to a
step down transformer with its primary windings having
4000 turns, giving the output power at 230 V. If the current in
(c) the primary of the transformer is 5 A, and its efficiency is
2V
90%, the output current would be: [Online April 16, 2018]
5ms 10ms 15ms t (a) 20 A (b) 40 A (c) 45 A (d) 25 A
Vo(t) 66. In an oscillating LC circuit the maximum charge on the
capacitor is Q. The charge on the capacitor when the
2V
(d) energy is stored equally between the electric and magnetic
field is [2003]

5ms 10ms 15ms t Q Q Q


(a) (b) (c) (d) Q
63. A transformer consisting of 300 turns in the primary and 2 3 2
150 turns in the secondary gives output power of 2.2kW.
67. The core of any transformer is laminated so as to [2003]
If the current in the secondary coil is 10 A, then the input
(a) reduce the energy loss due to eddy currents
voltage and current in the primary coil are :
(b) make it light weight
[10 April 2019 I] (c) make it robust and strong
(a) 220 V and 20 A (b) 440 V and 20 A (d) increase the secondary voltage
(c) 440 V and 5 A (d) 220 V and 10 A 68. In a transformer, number of turns in the primary coil are 140
64. A power transmission line feeds input power at 2300 V and that in the secondary coil are 280. If current in primary
to a step down transformer with its primary windings coil is 4 A, then that in the secondary coil is [2002]
(a) 4 A (b) 2 A (c) 6 A (d) 10 A.
Alternating Current P-369

1. (d) As V(t) = 220 sin 100 pt æ pö


then I = I0 sin ç wt - ÷
220 è 2ø
so, I(t) = sin 100 pt
50 Now, given v(t) = 100 sin (500 t)
i.e., I = Im = sin (100 pt) E0 100
For I = Im and I0 = = [Q L = 0.02H ]
wL 500 ´ 0.02
p 1 1 æ pö
t1 = ´ = sec. I0 = 10sin ç 500t - ÷
2 100 p 200 è 2ø
Im I0 = -10cos ( 500t )
and for I =
2
4. (c) We know that power consumed in a.c. circuit is given
I p by,
Þ m = I m sin(100 pt 2 ) Þ = 100 pt 2
2 6 P = Erms.Irms cos f
Here, E = E0 sin wt
1 pö
Þ t2 = s æ
600 I = I0 sin ç wt - ÷
è 2ø
p
1 1 2 1 This means the phase difference, is f =
\ treq = - = = s = 3.3 ms 2
200 600 600 300 p
2. (a) For two concentric circular coil, Q cos f = cos = 2
2
p
m 0 pN1N 2 a 2 \ P = Erms .I rms .cos = 0
Mutual Inductance M = 2
2b r r
here, N1 = N2 = 1 5. (b) f = B. A ; f = BA cos wt
m 0 pa 2 df wBA
Hence, M = ..... (i) e=- = wBA sin wt ; i = sin wt
2b dt R
and given I = I0 cos wt ..... (ii) æ wBA ö
2
Pinst = i 2 R = ç ´ R sin 2 wt
Now according to Faraday's second law induced emf
è R ÷ø
dI T T
e = -M 2
dt ò Pinst ´ dt 2 ò sin wtdt
Pavg = 0 =
(wBA) 0 1 ( wBA) 2
T R T =
2 R
a ò dt ò dt
From eq. (ii), 0 0
b
é pr 2 ù
(w B pr 2 ) 2 êA = ú
\ Pavg = 2 ûú
8R ëê
-m 0 pa 2 d 6. (a) D.C. ammeter measure average value of current. In AC
e= (I0 cos w t) current, average value of current in complete cycle is zero.
2b dt
Hence reading will be zero.
m 0 pa 2
e= I0 sin w t (w ) 7. (33)
2b Here, L = 50 mH = 50 × 10–3 H; I = 1 A, R = 2W
pm 0 I0 a 2
e= . w sin w t dl
2 b VP - L - 30 + RI = VQ
dt
3. (b) In a pure inductive circuit current always lags behind
p Þ VP - VQ = 50 ´ 10 -3 ´ 10 2 + 30 - 1 ´ 2
the emf by .
2 = 5 + 30 – 2 = 33 V.
If v ( t ) = v0 sin wt
P-370 Physics

8. (a) Quality factor,


Rt æ n - 1ö
1 L 1 80 ´ 10 -3 Þ- = ln ç ÷
Q= = L è n ø
R C 100 2 ´ 10-6

=
1
40 ´ 103 =
200
=2 L æ n ö
Þt = ln ç
100 100 R è n - 1÷ø
9. (400)
Given: Power P = 400 W, Voltage V = 250 V 11. (d) Here, R = 100, XL = Lw = 0.1803 × 750 × 2p = 850W,

P = Vm × I rms × cos f 1 1
XC = = = 21.23W
C w 10 -5 ´ 2p ´ 750
Þ 400 = 250 ´ I rms ´ 0.8 Þ I rms = 2 A
2
Using P = I rms R
Impedance Z = R 2 + ( X L - X C )2

( I rms )2 × R = P Þ 4 ´ R = 400 = 1002 + (850 - 21.23) 2 = 834.77 ; 835


Þ R = 100W 10mF
100W 0.1803 H
Power factor is,
R
cos f =
R + X L2
2

2 20V/750 Hz
100 æ 100 ö
Þ 0.8 = Þ 100 2 + X L2 = ç
100 2 + X L2 è 0.8 ÷ø 2 æV ö
2
H = irms Rt = ç rms ÷ RT = (ms )Dt
è |Z |ø
2
æ 100 ö 20 20
Þ X L = -1002 + ç Þ X L = 75W Þ ´ ´ 100t = (2) ´ 10
è 0.8 ÷ø 835 835
When power factor is unity, QVrms = 20 V and Dt = 10°C
1 \ Time, t = 348.61 s.
X C = X L = 75 Þ = 75 12. (a) Given,
wC
Reactance of inductance coil, Z = 100W
1 1 Frequency of AC signal, v = 1000 Hz
ÞC= = F
75 ´ 2p ´ 50 7500p Phase angle, f = 45°
XL
æ 106 1ö 400 tan f = = tan 45° = 1
=ç ´ ÷ mF = mF R
è 2500 3p ø 3p
Þ XL = R
N = 400
10. (a) Potential energy stored in the inductor Reactance, Z = 100 = X L2 + R 2

1 2 Þ 100 = R2 + R2
U= LI
2
Þ 2 R = 100 Þ R = 50 2
During growth of current,

(
i = I max 1 - e - Rt / L ) \ X L = 50 2

Þ Lw = 50 2 (Q X L = wL)
U I
For U to be max ; i has to be max
n n 50 2
ÞL= (Q w = 2pv)
2p ´ 1000
I max
\ = I max (1 - e - Rt / L )
n 25 2
= mH
p
1 n -1
Þ e - Rt / L = 1 - = = 1.1 × 10–2 H
n n
Alternating Current P-371

e XC 103
13. (b) We have, i = i0 (1 – e–t/c) = (1 - e - t / c ) tan q1 = R = i
R 2 3
t q1 is close to 90°
Charge, q = ò idt For L-R circuit
0
3 q1
t XL = wL = 100 ´ = 10 3 v
e EL 10
= R ò (1 - e
-t /t
)dt = E t = E ´ ( L / R ) =
0 Re R e 2.7 R 2 R1 = 10
æ – ö
Rt XL v
14. (d) I = I0 ç1 - e L ÷ Here R = RL + r = 1W tan q2 = R q2
çè ÷ø 1

æ – ö
t tan q2 = 3 Þ q2 = tan –1( 3)
0.8I 0 = I0 ç1 - e .01 ÷
i
çè ÷ø q2 =60°
So, phase difference comes out 90° + 60° = 150°
Þ 0.8 = 1 - e -100t If R2 is 20 KW
æ 1ö
Þ e -100t = 0.2 = ç ÷ then phase difference comes out to be 60 + 30 = 90°.
è 5ø
Therefore Ans. is Bonus
1 19. (b)
Þ 100t = ln5 Þ t = ln 5 = 0.016 s I
100
2 æ di ö
15. (c) i R = çè t ÷ø i
dt
di i
Þ =
dt t t
t0
é L 20 ù
Þ t = t ln2 = 2ln2 ê ast = = = 2ú The current will grow for the time t = 0 to
ë R 10 û
16. (d) w = 100 rad/s t = t0 and after that decay of current takes place.
We know that 20. (c) Given: R = 60W, f = 50 Hz, w = 2 pf = 100 p and v = 24v
C = 120 mf = 120 × 10–6f
XC 1
tan f = = 1 1
R wCR xC = = = 26.52W
wC 100p ´ 120 ´ 10 -6
1 xL = wL = 100 p × 20 × 10–3 = 2 pW
or tan45º = xC – xL= 20.24 » 20
wCR
or wCR = 1 R = 60W
f
LHS: wCR = 10 × 10 × 10–6 × 103 = 1
17. (d) Long time after switch is closed, the inductor will be
idle so, the equivalent diagram will be as below
Z
z = R 2 + ( xC – x L )
2
R R
e
z =20 10W
e 2e 2 ´ 15 R 60 3
I= = = = 6A cos f= = =
æ R ´Rö R 5 z 20 10 10
çè ÷
R +Rø
v v2
18. (Bonus) Pavg = VI cos f, I = = cos f = 8.64 watt
z z
Capacitive reactance, Energy dissipated (Q) in time t = 60s is
1 4 2 ´ 104
Q = P.t = 8.64 × 60 = 5.17 × 102J
Xc = = –6 = 21. (a) Given, Inductance, L= 40 mH
wC 10 ´ 3 ´ 100 3
Capacitance, C = 100 mF
Impedance, Z = XC – XL
P-372 Physics

1 æ 1 ö d2 dq q
Þ Z= – wL çèQ X c = and X L = wL÷
ø
L +R + = 0 ...(ii)
wC wC dt 2 dt C
1 Comparing equations (i) & (ii)
= –6
– 314 ´ 40 ´ 10 –3 1
314 ´ 100 ´ 10 L « m, C « , R « b
= 19.28W k
V0 24. (a) The current (I) in LR series circuit is given by
Current, i = sin(wt + p / 2)
Væ – ö
tR
Z
I = ç1 – e L ÷
10 R çè ÷ø
Þ i= cos wt = 0.52 cos (314 t)
19.28 At t = ¥,
22. (a) For series connection of a resistor and inductor, time –¥ ö
æ
variation of current is I = I0 (1– e – t /Tc ) 20
I¥ = ç I – e L/ R ÷ = 4 ...(i)
5 çè ÷ø
At t = 40s,
æ –40 ´ 5 ö –20,000
çè1 – e ÷ = 4(1 – e ) ...(ii)
10 ´ 10 –3 ø
Dividing (i) by (ii) we get
I¥ 1
L Þ = ,
Here, TC = I 40 1– e –20,000
R
Tc
25. (b) As we know, average power Pavg = Vrms Irms cosq
æ V öæ I ö æ 100 ö æ 20 ö
q= ò idt = ç 0 ÷ ç 0 ÷ cos q = ç ÷ç ÷ cos 45° (Q q = 45°)
0 è 2 øè 2 ø è 2 øè 2 ø

ò R (1 – e ) dt
E – t / tc 1000
Þ ò dq = Pavg = watt
2
Îé tc Wattless current I = Irms sin q
Þq= t + tC e – t / tc ù
R ë û 0 I 20
= 0 sin q = sin 45° = 10A
Îé tC ù 2 2
Þ q = êtC + – tC ú w0 w L
Rë e û 26. (a) Quality factor Q = = 0
2Dw R
Î L 27. (b) Given,
Þq=
R Re V0 = 283 volt, w = 320, R = 5 W, L = 25 mH, C = 1000 µF
ÎL xL = wL = 320 × 25 × 10–3 = 8 W
\q =
R 2e 1 1
xC = = = 3.1 W
23. (d) In damped harmonic oscillation, wC 320 ´ 1000 ´ 10 -6
Total impedance of the circuit :
md 2 x
= – kx – bv
dt 2 Z = R 2 + (X L - X C ) 2 = 25 + (4.9) 2 = 7 W
md 2 x dx Phase difference between the voltage and current
Þ +b + kx = 0 ....(i) X - XC
dt 2 dt tan f = L
R
4.9
tan f = » 1 Þ f = 45°
5
28. (b) Here
e e e
i= = =
2
R + X L2 2
R +w L 2 2
R + 4p 2v 2 L2
2

220
10 =
64 + 4p2 (50) 2 L
V 80
–q Ldi [Q R = = = 8]
In LCR circuit, – iR – =0 I 10
C dt On solving we get
L = 0.065 H
Alternating Current P-373

29. (d) 34. (c) Applying Kirchhoff's law of voltage in closed loop
15 ´ 100 V
30. (b) I (0) = = 0.1A –VR –VC = 0 Þ R = -1
0.15 ´ 103 VC
I(¥) = 0 VR
–t A C R
I(t) = [I (0) – I (¥)] e /R
L + i (¥)
–t R
L VL
I(t) = 0.1 e L/ R = 0.1 e L
0.15´1000 B
I(t) = 0.1 = 0.67mA
e 0.03
31. (c) From KVL at any time t 35. (c) Given, VL : VC : VR = 1 : 2 : 3
V = 100 V
di
R L VR = ?
+ dt –
i As we know,

V = VR2 + ( VL - VC )
2

Solving we get, VR ; 90V


+ –
q c 36. (c) Charge on he capacitor at any time t is given
q di by q = CV (1– et/t)
- iR - L = 0 at t = 2t
c dt
q = CV (1 – e–2)
dq q dq Ld 2 q
i=- Þ + R+ =0 37. (d) Power factor (old)
dt c dt dt 2
R R R
= = =
d 2q R dq q 5R
+ + =0 R 2 + XL2 R 2 + (2R)2
2 L dt Lc
dt
Power factor(new)
From damped harmonic oscillator, the amplitude is given
R R R
dt = = =
by A = Ao e - 2 2 2 2 2R
2m R + (X L - XC ) R + (2R - R)
Double differential equation
R
d 2x b dx k New power factor 2R = 5
2
+ + x =0 \ Old power factor =
dt m dt m R 2
Rt Rt 5R
- -
2 2
Qmax = Q oe 2L Þ Q max = Qo e L 38. (d) Impedance (Z) of the series LCR circuit is
Hence damping will be faster for lesser self inductance.
Z = R 2 + (X L - X C ) 2
32. (c) Power factor
R At resonance, X L = XC
cos f = =1
2 Therefore, Zminimum = R
é 1 ù
R 2 + ê wL – ú
ë w(C + C ') û 39. (b)
Pmax
On solving we get,
1.0
1 P Pmax
wL = =P
w(C + C ') 2
0.5
1– w2 LC
C'=
w2 L
Hence option (c) is the correct answer. 0.0
4 5 6
3 7
33. (c) For capacitor circuit, i = i0e–t/RC w1 w0 w2
æ – ö
Rt Quality factor of the circuit
For inductor circuit, i = i0 1– e L ÷
ç w0 5
ç ÷ = w -w = = 2.0
è ø 2 1 2.5
Hence graph (c) correctly depicts i versus t graph.
P-374 Physics

40. (d) -
t2
( RC / 2) V0
1 V = V0 e = ....(2)
1 = 2
41. (c) Frequency f =
2p LC 2 ´ 3.4 24 ´ 2 ´ 10 -6 From (1) and (2)
; 23Hz t1 t2
=
42. (c) Voltage E of the ac source 2 RC ( RC / 2 )
E = VC – VL = 400 V – 300 V = 100 V t 10
43. (a) t = 3 RC ln 2 Þ t2 = 1 = = 2.5 sec.
44. (d) In case of series RLC circuit, 4 4
48. (c) At t = 0, no current will flow through L and R1 as
Equation of voltage is given by inductor will offer infinite resistance.
V 2 = VR2 + (VL - VC )
2 V
\ Current through battery, i =
Here, V = 220 V; VL = VC = 300 V R2
At t = ¥, inductor behave as conducting wire
\ VR = V 2 = 220V RR
Effective resistance, Reff = 1 2
V 220 R1 + R2
Current i = = = 2.2A
R 100 V V ( R1 + R2 )
1 2 \ Current through battery = R =
45. (a) Energy stored in magnetic field = Li eff R1R2
2 49. (d) When only the capacitance is removed phase
1 q2 difference between current and voltage is
Energy stored in electric field =
2 C XL
Energy will be equal when tan f =
R
1 2 1 q2 wL
\ Li = Þ tan f =
2 2 C R
tan wt = 1 1 200
q = q0 cos wt Þ wL = R tan f = 200 ´ =
3 3
1 ( q cos wt )2 When only inductor is removed, phase difference between
Þ L(wq0 sin wt)2 = 0
2 2C current and voltage is
1 p 1
Þw= Þ wt = \ tan f =
LC 4 w CR
p 1 1 200
Þ t= LC Þ = R tan f = 200 ´ =
4 wC 3 3
46. (b) We have, V = V0(1 – e–t/RC) Impedance of the circuit,
Þ 120 = 200(1 – e–t/RC) 2
æ 1 ö
200 – 120 80 Z = R2 + ç - wL ÷
e–t/r = = è w C ø
200 200
t = loge(2.5) 2
æ 200 200 ö
Þ t = RC in (2.5) [Q r = RC] = (200) 2 + ç - ÷ = 200 W
è 3 3ø
Þ R = 2.71 × 106 W
47. (c) Time constant for parallel combination Power dissipated in the circuit = VrmsIrms cos f
= 2RC Vrms R æ R ö V 2rms R
RC = Vrms × × çQ cos f = ÷ =
Time constant for series combination = Z Z è Zø Z2
2
In first case : (220)2 ´ 200 220 ´ 220
( )
t
t V0 – = = = 242 W
V = V0 – Þ = V0 – V0e CR (200)2 200
1 – e CR 2
t 50. (c) Growth in current in branch containing L and R2 when
- 1
V switch is closed is given by
V = V0 e = 0
2 RC ...(1)
2 E
In second case : i= [1 - e - R2t / L ]
C R2
In series grouping, equivalent capacitance = R2t
2 di E R2 - R2t / L E -
Þ = × ×e = e L
dt R2 L L
Alternating Current P-375

Hence, potential drop across L Taking log on both the sides,


Ldi æ E - R2t / L ö Rt
VL = =ç e ÷L - = log1 - log 2
dt è L ø L
2t
- R2t / L
-
400 ´10 -3 L 300 ´ 10 -3
= Ee = 12e = 12e–5tV Þ t= log 2 = ´ 0.69
R 2
V 100
51. (c) Across resistor, I = = = 0.1 A Þ t = 0.1 sec.
R 1000 57. (d) For maximum power, XL = XC, which yields
At resonance,
1 1 1 1
X L = XC = = = 2500 C= =
2 2
wC 200 ´ 2 ´ 10-6 (2 pn ) L 4p ´ 50 ´ 50 ´ 10
Voltage across L is
\ C = 0.1 ´ 10-5 F = 1mF
I X L = 0.1 ´ 2500 = 250 V
52. (a) Initially, when steady state is achieved, 58. (d) In a series LCR circuit voltage across the inductor
E and capacitor are in opposite phase
i=
R \ Net voltage difference across
Let E is short circuited at t = 0. Then
At t = 0 LC = 50 – 50 = 0
E 100 1
Maximum current, i0 = = = 1A 59. (a) Resonant frequency, Fr =
R 100 2p LC
Let during decay of current at any time the current flowing For resonant frequency to remain same
di
is - L - iR = 0 LC = constant
dt \ LC = L' C'
di R Þ LC = L¢ × 2C
Þ = - dt
i L
i t L
di R Þ L' =
Þ ò = ò - dt 2
i L 60. (b) Resistance of the inductor, XL = wL
i0 0
i R The impedance triangle for resistance (R) and inductor (L)
Þ log e =- t connected in series is shown in the figure.
i0 L
2
2
R L
- t 2 +w
Þ i = i0 e L R XL= w L

-100´10 -3 f
R
E - t -3 1 R
Þ i = e L = 1 ´ e 100´10 =
R ur ur e Net impedance of circuit Z = X L2 + R 2
df d ( N B. A)
53. (d) e = - =- R
dt dt Power factor, cos f =
d Z
= - N ( BA cos wt ) = NBAw sin wt
dt R
Þ cos f =
Þ e max = NBAw R + w 2 L2 2
54. (a) Phase difference for R–L circuit lies between 61. (d) All three inductors are connected in parallel. The
æ pö equivalent inductance Lp is given by
ç 0, ÷ but 0 or p/2
è 2ø 1 1 1 1 1 1 1 3
55. (b) Given, Resistance of circuit, R = 12 W = + + = + + = =1
L p L1 L2 L3 3 3 3 3
Imedance of circuit, Z = 15 W
R 12 4 \ Lp = 1
Power factor = cos f = = = = 0.8
62. (a) When first pulse is applied, the potential across
Z 15 5
56. (a) Current in inductor circuit is given by, capacitor
(
i = i0 1 - e
-
Rt
L ) æ 1 ö
V0 (t ) = Vin ç1 - e RC ÷
- -
Rt Rt è ø
i0 1
= i0 (1 - e L ) Þ e L =
2 2 At t = 5ms = 5 ´ 10-6 s
P-376 Physics

Þ Is = 0.9 ´ 50 = 45A
Output current = 45A
65. (c) Given : VP = 2300 V, Vs = 230 V, IP = 5 A, n = 90% = 0.9
Vin 10m F V0(t)
Ps
Efficiency n = 0.9 = Þ Ps = 0.9 Pp
PP

VsIs = 0.9 ×VPIP (Q P = VI)


æ 5´10-6 ö
V0 (t ) = 5 ç 1 - e ´10 ´10 ÷ = 5(1 - e -0.5 ) = 2V
103 -9 0.9 ´ 2300 ´ 5
Is = = 45A
ç ÷ 230
è ø
When no pulse is applied, capacitor will discharge. 66. (c) When the capacitor is completely charged, the total
Now, Vin = 0 means discharging. energy in the LC circuit is with the capacitor and that
energy is given by
1
V0 (t ) = 2e RC = 2e -0.5 = 1.21 V
1 Q2
Umax =
Now for next 5 ms 2 C
1 When half energy is with the capacitor in the form of electric
V0 (t ) = 5 - 3.79e RC field between the plates of the capacitor we get
After 5 ms again, V0 (t ) = 2.79 Volt » 3 V
U max 1 q¢2
=
Hence, graph (a) correctly depicts. 2 2 C
63. (c) Power output (V2I2) = 2.2 kW Here q' is the charge on the plate of capacitor when energy
2.2kW is shared equally.
\ V2 = = 220 volts
(10A ) 1 1 Q 2 1 q¢2 Q
\ ´ = Þ q¢ =
\ Input voltage for step-down transformer 2 2 C 2 C 2
V1 N1 67. (a) Laminated core provide less area of cross-section for
= =2 the current to flow. Because of this, resistance of the core
V2 N 2
increases and current decreases there by decreasing the
Vinput = 2 × Voutput = 2 × 220 energy loss due to eddy current.
= 440 V 68. (b) Number of turns in primary
I1 N 2 Np = 140
Also I = N
2 1 Number of turns in secondary Ns = 280, Ip = 4A, Is = ?

1 Is N p
\ I1 = ´ 10 = 5A Using transformation ratio for a transformer =
2 Ip Ns
P VI
64. (b) Efficiency, h= out = s s
Pin VpIp I s 140
Þ =
4 280
230 ´ Is
Þ 0.9 = Þ Is = 2 A
2300 ´ 5
22
Electromagnetic
Waves
Electromagnetic Waves, E0
(c) ( xˆ - yˆ ) sin(kz - wt )
TOPIC 1 Conduction c
and Displacement Current E
(d) 0 ( xˆ - yˆ ) cos(kz - wt )
c
1. For a plane electromagnetic wave, the magnetic field at a 4. The magnetic field of a plane electromagnetic wave is
ur
point x and time t is B = 3 ´ 10-8 sin[200p ( y + ct )]iT
ˆ
®
B( x, t ) = [1.2 ´ 10-7 sin(0.5 ´ 103®x + 1.5 ´ 1011t )k$ ]T ® where c = 3 × 108 ms–1 is the speed of light.
The instantaneous electric field E corresponding to B The corresponding electric field is : [Sep. 03, 2020 (I)]
is: (speed of light c = 3 × 108 ms–1) [Sep. 06, 2020 (II)] ur
(a) E = 9sin[200p ( y + ct )]kˆ V/m
® V ur
(a) E ( x, t ) = [ -36 sin(0.5 ´ 10 x + 1.5 ´ 10 t ) $j ]
3 11
(b) E = -10-6 sin[200p ( y + ct )]kˆ V/m
m
®
ur
(b) E( x, t ) = [36 sin(1 ´ 103 x + 0.5 ´ 1011 t ) $j ]
V (c) E = 3 ´ 10-8 sin[200p ( y + ct )]kˆ V/m
m ur
® (d) E = -9sin[200p ( y + ct )]kˆ V/m
V
(c) E ( x, t ) = [36 sin(0.5 ´ 103 x + 1.5 ´ 1011 t ) k$ ] 5. The electric field of a plane electromagnetic wave
m
®
propagating along the x direction in vacuum is
(d) E ( x, t ) = [36 sin(1 ´ 103 x + 1.5 ´ 1011 t )$i]
V r r
m E = E0 ˆj cos(wt - kx ) . The magnetic field B, at the
2. An electron is constrained to move along the y-axis with a moment t = 0 is : [Sep. 03, 2020 (II)]
speed of 0.1 c (c is the speed of light) in the presence of r E0
® (a) B = cos(kx)kˆ
electromagnetic wave, whose electric field is E = 30j$ m0 e 0
sin(1.5 × 107t – 5 × 10–2x) V/m. The maximum magnetic force r
(b) B = E0 m0 e0 cos(kx) ˆj
experienced by the electron will be : r
(c) B = E0 m0 e0 cos(kx)kˆ
(given c = 3 × 108 ms–1 & electron charge = 1.6 × 10–19C)
[Sep. 05, 2020 (I)] r E0
(d) B = cos(kx) ˆj
(a) 3.2 × 10–18 N (b) 2.4 × 10–18 N m0 e 0
(c) 4.8 × 10–19 N (d) 1.6 × 10–19 N 6. A plane electromagnetic wave, has frequency of 2.0 × 1010
3. The electric field of a plane electromagnetic wave is given Hz and its energy density is 1.02 × 10–8 J/m3 in vacuum.
r The amplitude of the magnetic field of the wave is close to
by E = E0 ( xˆ + yˆ ) sin(kz - wt )
Its magnetic field will be given by : [Sep. 04, 2020 (II)] 1 Nm 2
( = 9 ´109 2 and speed of light = 3 × 108 ms–1) :
E0 4pe0 C
(a) (- xˆ + yˆ ) sin(kz - wt ) [Sep. 02, 2020 (I)]
c
(a) 150 nT (b) 160 nT
E
(b) 0 ( xˆ + yˆ )sin(kz - wt ) (c) 180 nT (d) 190 nT
c
P-378 Physics

7. In a plane electromagnetic wave, the directions of electric ur


(c) E = (3 ´ l0–8 sin (l.6 ´ l03x + 48 ´ l010t) k̂ v/m)
field and magnetic field are represented by k̂ and 2iˆ - 2 ˆj, ur
(d) E = (3 ´ l0–8sin (l.6 ´ l03x + 48 ´ l010t) k̂ v/m)
respectively. What is the unit vector along direction of 12. The electric field of a plane electromagnetic wave is
propagation of the wave. [Sep. 02, 2020 (II)] given by

(a)
1 ˆ ˆ
(i + j ) (b)
1
( ˆj + kˆ) r E iˆ + ˆj cos(kz + wt )
2 2 E = 0 2
At t = 0, a positively charged particle is at the point
1 ˆ 1 ˆ ˆ
(c) (i + 2 ˆj ) (d) (2i + j ) æ pö
5 5 (x, y, z) = çè 0, 0, ÷ø . If its instantaneous velocity at (t = 0)
k
8. The electric fields of two plane electromagnetic plane
waves in vacuum are given by is v0 kˆ , the force acting on it due to the wave is:
ur ur [7 Jan 2020, II]
E1 = E0 ˆj cos(wt - kx ) and E2 = E 0 kˆ cos(wt - ky ) .
At t = 0, a particle of charge q is at origin with a velocity iˆ + ˆj
r (a) parallel to
2
(b) zero
v = 0.8 cjˆ (c is the speed of light in vacuum). The
instantaneous force experienced by the particle is: iˆ + ˆj
[9 Jan 2020, I] (c) antiparallel to (d) parallel to k̂
2
(a) E0 q(0.8iˆ - ˆj + 0.4k ) (b) E0 q(0.4iˆ - 3 ˆj + 0.8kˆ)
ˆ 13. An electromagnetic wave is represented by the electric
ur
field E = E n$ sin[w t + (6 y - 8z)] . Taking unit vectors in
(c) E0 q(- 0.8iˆ + ˆj + kˆ) (d) E0 q( 0.8iˆ + ˆj + 0.2kˆ) 0

9. A plane electromagnetic wave is propagating along the x, y and z directions to be $i , $j , k$ , the direction of
iˆ + ˆj propogation $s is : [12 April 2019, I]
direction , with its polarization along the direction
2
3$i - 4 $j -4k$ + 3 $j
kˆ. The correct form of the magnetic field of the wave (a) s$ = (b) s$ =
5 5
would be (here B0 is an appropriate constant):
[9 Jan 2020, II] æ -3$j + 4k$ ö 3 $j - 3k$
(c) s$ = ç ÷÷ (d) s$ =
iˆ - ˆj æ iˆ + ˆj ö ç 5 5
cos ç wt - k è ø
(a) B0
2 è 2 ÷ø 14. A plane electromagnetic wave having a frequency
ˆj - iˆ v = 23.9 GHz propagates along the positive z-direction in
æ iˆ + ˆj ö free space. The peak value of the Electric Field is 60 V/m.
(b) B0 cos ç wt + k
2 è 2 ÷ø Which among the following is the acceptable magnetic
field component in the electromagnetic wave ?
ˆ æ iˆ + ˆj ö [12 April 2019, II]
(c) B0 k cos ç wt - k
è 2 ÷ø ur
(a) B = 2 ´ 10 sin(0.5 ´ 10 z + 1.5 ´ 1011 t )$i
7 3

iˆ + ˆj æ iˆ + ˆj ö ur
(d) B0 cos ç wt - k (b) B = 2 ´ 10-7 sin(0.5 ´ 103 z - 1.5 ´ 1011 t )$i
2 è 2 ÷ø ur
10. A plane electromagnetic wave of frequency 25 GHz is (c) B = 60sin(0.5 ´ 103 x + 1.5 ´ 1011 t )k$
ur -7 11 $
propagating in vacuum along the z-direction. At a 2
(d) B = 2 ´ 10 sin(1.5 ´ 10 x + 0.5 ´ 10 t ) j
particular point in space and time, the magnetic field is
r 15. The electric field of a plane electromagnetic wave is given
given by B = 5 ´ 10-8 ˆj T . The corresponding electric ur
r by E = E $i cos(kz) cos(w t)
0
field E is (speed of light c = 3 ´ l08 ms–l) The corresponding magnetic field is then given by :
[8 Jan 2020, II] [10 April 2019, I]
(a) 1.66 ´ 10–16 iˆ V/m (b) – 1.66 ´ 10–16 iˆ V/m ur E 0
(a) B = $j sin (kz) sin (w t)
(c) –15 iˆ V/m (d) 15 iˆ V/m C
11. If the magnetic field in a plane electromagnetic wave is ur E 0
(b) B = $j sin(kz) cos ( w t)
ur C
given by B = 3 ´ 10–8 sin (l.6 ´ 103x + 48 ´ 1010t) ĵ T, then
ur E 0
what will be expression for electric field? (c) B = $j cos (kz) sin (w t)
[7 Jan 2020, I] C
ur ur E 0
(a) E = (60 sin (1.6 ´ l03x + 48 ´ l010t) k̂ v/m) (d) B = k$ sin (kz) cos( w t)
ur C
(b) E = (9 sin (1.6 ´ l03x + 48 ´ l010t) k̂ v/m)
Electromagnetic Waves P-379

16. Light is incident normally on a completely absorbing 23. An electromagnetic wave of intensity 50 Wm–2 enters in a
surface with an energy flux of 25 Wcm–2. If the surface has medium of refractive index ‘n’ without any loss. The ratio
an area of 25 cm2, the momentum transferred to the surface of the magnitudes of electric fields, and the ratio of the
in 40 min time duration will be: [10 April 2019, II] magnitudes of magnetic fields of the wave before and after
(a) 6.3×10–4 Ns (b) 1.4×10–6 Ns entering into the medium are respectively, given by :
(c) 5.0×10–3 Ns (d) 3.5×10–6 Ns [11 Jan 2019, I]
17. The magnetic field of a plane electromagnetic wave is æ 1 1 ö
given by:
r
(a) ç
è n nø
, ÷ (b) (
n, n )
B = B $i [ cos ( kz – w t ) ] + B $j cos ( kz + wt )
0 1 æ 1 ö æ 1 ö
Where B0 = 3 × 10–5 T and B1 = 2 × 10–6 T. (c) ç n , ÷ (d) ç , n÷
The rms value of the force experienced by a stationary è nø è n ø
charge Q = 10–4 C at z = 0 is closest to: [9 April 2019 I] 24. A 27 mW laser beam has a cross-sectional area of 10 mm 2.
(a) 0.6 N (b) 0.1 N The magnitude of the maximum electric field in this
electromagnetic wave is given by :
(c) 0.9 N (d) 3 × 10–2 N [Given permittivity of space Î0 = 9 × 10 –12 SI units, Speed
18. A plane electromagnetic wave of frequency 50 MHz travels of light c = 3 × 108 m/s] [11 Jan 2019, II]
in free space along the positive x-direction. At a particular (a) 2 kV/m (c) 0.7 kV/m
r
point in space and time, E = 6.3 ˆj V / m. The (b) 1 kV/m (d) 1.4 kV/m
r 25. If the magnetic field of a plane electromagnetic wave is
corresponding magnetic field B , at that point will be: given by (The speed of light = 3 × 108 m/s)
[9 April 2019 I]
é 15 æ xö ù
(a) 18.9 × 10–8 k̂T (b) 2.1 × 10–8 k̂T B = 100 × 10–6 sin ê 2 p ´ 2 ´ 10 çè t - ÷ø ú
ë c û
(c) 6.3 × 10–8 k̂T (d) 18.9 × 108 k̂T then the maximum electric field associated with it is:
19. 50 W/m2 energy density of sunlight is normally incident [10 Jan. 2019 I]
on the surface of a solar panel. Some part of incident (a) 6 × 10 N/C4
(b) 3 × 10 N/C4
energy (25%) is reflected from the surface and the rest is
(c) 4 × 104 N/C (d) 4.5 104 N/C
absorbed. The force exerted on 1m2 surface area will be
close to (c = 3 × 108 m/s): [9 April 2019, II] 26. The electric field of a plane polarized electromagnetic
wave in free space at time t = 0 is given by an expression
(a) 15 × 10–8 N (b) 20 × 10–8 N ur
(c) 10 × 10–8 N (d) 35 × 10–8 N E ( x, y ) = 10 ˆj cos [(6 x + 8 z)]
20. A plane electromagnetic wave travels in free space along ur
the x-direction. The electric field component of the wave The magnetic field B ( x, z, t ) is given by: (c is the
at a particular point of space and time is E = 6 Vm –1 along velocity of light) [10 Jan 2019, II]
y-direction. Its corresponding magnetic field component, 1 ˆ
B would be: [8 April 2019 I] (a) (6k + 8i)ˆ cos [ (6 x - 8 z + 10ct ) ]
(a) 2 × 10–8 T along z-direction c
(b) 6 × 10–8 T along x-direction 1 ˆ ˆ cos [ (6 x + 8 z - 10ct ) ]
(b) (6k - 8i)
(c) 6 × 10–8 T along z-direction c
(d) 2 × 10–8 T along y-direction 1 ˆ
21. The magnetic field of an electromagnetic wave is given by: (c) (6k + 8i)ˆ cos [ (6 x + 8 z - 10ct ) ]
c
ur
( )( Wb
B = 1.6 ´ 10 –6 cos 2 ´ 10 7 z + 6 ´ 1015 t 2iˆ + ˆj 2
m
) (d)
1 ˆ
(6k - 8i)ˆ cos [(6 x + 8 z + 10ct ) ]
c
The associated electric field will be : [8 April 2019, II] 27. An EM wave from air enters a medium. The electric fields
ur
(
(a) E = 4.8 × 102 cos(2 × 107 z – 6 × 1015 t) 2iˆ + ˆj ) V
m
r é æ z öù
are E1 = E01 xˆ cos ê 2pv ç - t ÷ú in air and
ë è c øû
ur V r
(b) E = 4.8 ´ 10 2 cos(2 ´ 10 7 z - 6 ´ 1015 t)( -2 $j + $i) E2 = E02 xˆ cos [ k (2 z - ct )] in medium, where the wave
m number k and frequency v refer to their values in air. The
ur medium is nonmagnetic. If Îr1 and Îr refer to relative
(
(c) E = 4.8 × 102 cos(2 × 107 z + 6 × 1015 t) –iˆ + 2 ˆj
V
m
) 2
permittivities of air and medium respectively, which of the
ur following options is correct? [9 Jan 2019, I]
(
(d) E = 4.8 × 102 cos(2 × 107 z + 6 × 1015 t) iˆ – 2 ˆj
V
m
) Îr Îr1
(a) 1 =4 (b) =2
22. The mean intensity of radiation on the surface of the Sun Îr Îr
is about 108 W/m2. The rms value of the corresponding 2 2
magnetic field is closest to : [12 Jan 2019, II] Îr 1 Îr 1
(a) 1 T (b)102 T (c) 10–2 T (d) 10–4 T (c) 1
= (d) 1
=
Îr 4 Îr 2
2 2
P-380 Physics
r
28. The energy associated with electric field is (UE) and with (a) E = B0 csin(kx + wt)kV ˆ /m
magnetic fields is (UB) for an electromagnetic wave in r B0
free space. Then : [9 Jan 2019, II] (b) E = sin(kx + wt)kVˆ /m
r c
UB (c) E = - B0 csin(kx + wt)kV ˆ /m
(a) U E = (b) UE > UB r
2 ˆ
(d) E = B0 csin(kx - wt)kV / m
(c) UE < UB (d) UE = UB 33. Consider an electromagnetic wave propagating in vacuum.
29. A plane electromagnetic wave of wavelength l has an Choose the correct statement : [Online April 10, 2016]
intensity I. It is propagating along the positive Y– (a) For an electromagnetic wave propagating in +y
direction. The allowed expressions for the electric and r 1
magnetic fields are given by [Online April 16, 2018] direction the electric field is E = E yz (x, t)zˆ and
2
ur I é 2p ù r 1 r 1
(a) E = cos ê (y - ct) ú ˆi; B = Ekˆ the magnetic field is B = Bz (x, t)yˆ
e C 0 ël û c 2
(b) For an electromagnetic wave propagating in +y
ur I é 2p ùˆ r 1 r
(b) E = cos ê (y - ct) ú k;B = - Eiˆ 1
e0 C
direction the electric field is E = E yz (x, t)yˆ and
ël û c 2
ur r
2I é 2p ùˆ r 1 the magnetic field is B =
1
B yz (x, t)zˆ
(c) E = cos ê (y - ct) ú k;B = + Eiˆ 2
e0 C ël û c
(c) For an electromagnetic wave propagating in
ur 2I é 2p ùˆ r 1 ˆ r
(d) E = cos ê (y + ct) ú k; B = Ei 1
e0 C
+x direction the electric field is E = E yz (y, z, t)
ël û c 2
30. A monochromatic beam of light has a frequency
( yˆ + zˆ ) and the magnetic field is
3
v= ´ 1012 Hz and is propagating along the direction r 1
2p B= Byz (y, z, t) ( yˆ + zˆ )
2
iˆ + ˆj (d) For an electromagnetic wave propagating in +x
. It is polarized along the k̂ direction. The acceptable
2 r 1
form for the magnetic field is: [Online April 15, 2018] direction the electric field is E = E yz (x, t) ( yˆ - zˆ )
2
E æ iˆ - ˆj ö é 4 æ iˆ - ˆj ö r 12 ù r
(a) k 0 ç ÷ cos ê10 ç ÷ .r - (3 ´ 10 )t ú and the magnetic field is B = 1 B yz (x, t) ( yˆ + zˆ )
C è 2 ø ëê è 2 ø ûú 2
34. For plane electromagnetic waves propagating in the
E0 æ iˆ - ˆj ö é 4 æ iˆ + ˆj ö r 12 ù z-direction, which one of the following combination gives
(b) ç ÷ cos ê10 ç ÷ .r - (3 ´10 )t ú ur ur
C è 2 ø ëê è 2 ø ûú the correct possible direction for E and B field
respectively? [Online April 11, 2015]
E0 ˆ é æ iˆ + ˆj ö r 12 ù
(c) k cos ê104 ç ÷ .r + (3 ´10 )t ú (a) (2$i + 3$j) and ($i + 2$j) (b) (-2$i - 3$j) and (3$i - 2 $j)
C ëê è 2 ø ûú
(c) (3$i + 4 $j) and (4$i - 3$j) (d) ($i + 2$j) and (2$i - $j)
E (iˆ + ˆj + kˆ) é æ iˆ + ˆj ö r 12 ù
(d) 0 cos ê104 ç ÷ .r + (3 ´ 10 )t ú 35. An electromagnetic wave travelling in the x-direction has
C 3 ëê è 2 ø ûú frequency of 2 × 1014 Hz and electric field amplitude of 27
31. The electric field component of a monochromatic Vm–1. From the options given below, which one describes
radiation
ur is given by the magnetic field for this wave ? [Online April 10, 2015]
E = 2 E0 $i cosur kz cos wt r
Its magnetic field B is then given by : (
-8
(a) B ( x, t ) = 3 ´ 10 T ˆj )
[Online April 9, 2017] sin é 2p(1.5 × 10 x – 2 × 1014 t) ù
–8
ur ë û
(a)
2Eo $
c
j sin kz cos wt (b) -
2Eo $
c
j sin kz sin wt (
(b) B ( x,t ) = 9×10 T iˆ
-8
)
2Eo $ 2Eo $ sin é 2p(1.5 × 10 –8 x – 2 × 1014 t) ù
(c) j sin kz sin wt (d) j cos kz cos wt ur ë û

32.
c c
Magnetic field in a plane electromagnetic wave is given by
(
(c) B ( x, t ) = 9 ´10 T ˆj
-8
)
r sin é1.5 × 10 –6 x – 2 × 1014 t) ù
B = B sin(kx + wt)ˆjT ur ë û
( )
0
-8
Expression for corresponding electric field will be : (d) B ( x, t ) = 9 ´ 10 T k
ˆ
Where c is speed of light. [Online April 8, 2017] sin é 2p(1.5 × 10 –6
x – 2 × 1014 t) ù
ë û
Electromagnetic Waves P-381

36. During the propagation of electromagnetic waves in a ®


medium: [2014] æ 2p ö
(b) E = E0 cos çè wt - y ÷ xˆ
ø
(a) Electric energy density is double of the magnetic l
energy density. ®
(b) Electric energy density is half of the magnetic energy æ 2p ö
(c) E = E0 cos ç wt - y÷ zˆ
density. è l ø
(c) Electric energy density is equal to the magnetic energy
® æ 2p ö
density.
(d) E = - E0 cos ç wt + y÷ zˆ
(d) Both electric and magnetic energy densities are zero. è l ø
37. A lamp emits monochromatic green light uniformly in all 44. An electromagnetic wave of frequency v = 3.0 MHz
directions. The lamp is 3% efficient in converting electrical
power to electromagnetic waves and consumes 100 W of passes from vacuum into a dielectric medium with
power. The amplitude of the electric field associated with permittivity Î = 4.0. Then [2004]
the electromagnetic radiation at a distance of 5 m from the (a) wave length is halved and frequency remains
lamp will be nearly: [Online April 12, 2014] unchanged
(a) 1.34 V/m (b) 2.68 V/m (b) wave length is doubled and frequency becomes half
(c) wave length is doubled and the frequency remains
(c) 4.02 V/m (d) 5.36 V/m unchanged
38. An electromagnetic wave of frequency 1 × 1014 hertz is (d) wave length and frequency both remain unchanged.
propagating along z-axis. The amplitude of electric field is 45. Electromagnetic waves are transverse in nature is evident
4 V/m. If e0 = 8.8 × 10–12 C2/N-m2, then average energy by [2002]
density of electric field will be: [Online April 11, 2014] (a) polarization (b) interference
(a) 35.2 × 10–10 J/m3 (b) 35.2 × 10–11 J/m3 (c) reflection (d) diffraction
–12
(c) 35.2 × 10 J/m 3 (d) 35.2 × 10–13 J/m3
39. The magnetic field in a travelling electromagnetic wave
has a peak value of 20 nT. The peak value of electric field TOPIC 2 Electromagnetic Spectrum
strength is : [2013]
(a) 3 V/m (b)6 V/m (c) 9 V/m (d) 12 V/m 46. The correct match between the entries in column I and
40. A plane electromagnetic wave in a non-magnetic dielectric column II are : [Sep. 05, 2020 (II)]
ur ur I II
medium is given by E = E 0 (4 ´ 10 -7 x - 50t ) with
Radiation Wavelength
distance being in meter and time in seconds. The dielectric
constant of the medium is : [Online April 22, 2013] (A) Microwave (i) 100 m
(a) 2.4 (b)5.8 (c) 8.2 (d) 4.8 (B) Gamma rays (ii) 10–15 m
41. Select the correct statement from the following : (C) A.M. radio waves (iii) 10–10 m
[Online April 9, 2013]
(a) Electromagnetic waves cannot travel in vacuum. (D) X-rays (iv) 10–3 m
(b) Electromagnetic waves are longitudinal waves. (a) (A)-(ii), (B)-(i), (C)-(iv), (D)-(iii)
(c) Electromagnetic waves are produced by charges (b) (A)-(i), (B)-(iii), (C)-(iv), (D)-(ii)
moving with uniform velocity.
(d) Electromagnetic waves carry both energy and (c) (A)-(iii), (B)-(ii), (C)-(i), (D)-(iv)
momentum as they propagate through space. (d) (A)-(iv), (B)-(ii), (C)-(i), (D)-(iii)
42. An electromagnetic wave in vacuum has the electric and
r r 47. Chosse the correct option relating wavelengths of different
magnetic field E and B , which are always perpendicular parts of electromagnetic wave spectrum :
r
to each other. The direction of polarization is given by X [Sep. 04, 2020 (I)]
r
and that of wave propagation by k . Then [2012] (a) l visible < l micro waves < l radio waves < l X - rays
r r r r r
(a) X || B and k || B ´ E (b) l radio waves > l micro waves > l visible > l x-rays
r r r r r
(b) X || E and k || E ´ B (c) l x- rays < l micro waves < l radio waves < l visible
r r r r r
(c) X || B and k || E ´ B (d) l visible > l x-rays > l radio waves > l micro waves
r r r r r
(d) X || E and k || B ´ E 48. Given below in the left column are different modes of
43. An electromagnetic wave with frequency w and
communication using the kinds of waves given in the
wavelength l travels in the + y direction. Its magnetic field
is along + x-axis. The vector equation for the associated right column. [10 April 2019, I]
electric field (of amplitude E0) is [Online May 19, 2012] A. Optical Fibre P. Ultrasound
® æ 2p ö Communication
(a) E = - E0 cos ç wt + y ÷ xˆ B. Radar Q. Infrared Light
è l ø
C. Sonar R. Microwaves
D. Mobile Phones S. Radio Waves
P-382 Physics

From the options given below, find the most appropriate


List I List II
match between entries in the left and the right column.
(a) A – Q, B – S, C – R, D – P I Doublet of sodium (A) Visible radiation
(b) A – S, B – Q, C – R, D – P II Wavelength (B) Microwave
(c) A – Q, B – S, C – P, D – R corresponding to
(d) A – R, B – P, C – S, D – Q temperature associated
49. Arrange the following electromagnetic radiations per with the isotropic
quantum in the order of increasing energy : [2016] radiation filling all space
A : Blue light B : Yellow light III Wavelength emitted by (C) Short radio wave
C : X-ray D : Radiowave. atomic hydrogen in
(a) C, A, B, D (b) B, A, D, C
interstellar space
(c) D, B, A, C (d) A, B, D, C
50. Microwave oven acts on the principle of : IV Wavelength of radiation (D) X-rays
[Online April 9, 2016] arising from two close
(a) giving rotational energy to water molecules energy levels in hydrogen
(b) giving translational energy to water molecules
(c) giving vibrational energy to water molecules (a) (I)-(A), (II)-(B), (III)-(B), (IV)-(C)
(d) transferring electrons from lower to higher energy (b) (I)-(A), (II)-(B), (III)-(C), (IV)-(C)
levels in water molecule (c) (I)-(D), (II)-(C), (III)-(A), (IV)-(B)
51. Match List - I (Electromagnetic wave type) with List - II (d) (I)-(B), (II)-(A), (III)-(D), (IV)-(A)
(Its association/application) and select the correct option 54. Match List I (Wavelength range of electromagnetic
from the choices given below the lists: [2014] spectrum) with List II (Method of production of these
List 1 List 2 waves) and select the correct option from the options given
below the lists. [Online April 9, 2014]
1. Infrared waves (i) To treat muscular
strain List I Lis t II
2. Radio waves (ii) For broadcasting (1) 700 nm to (i) Vibration of atoms
3. X-rays (iii) To detect fracture of 1 mm and molecules .
bones
(2) 1 nm to (ii) Inner s hell electrons
4. Ultraviolet rays (iv) Absorbed by the
400 nm in atoms moving
ozone layer of the
from one energy
atmosphere level to a lower level.
1 2 3 4
(a) (iv) (iii) (ii) (i) (3) < 10–3 nm (iii) Radioactive decay of
(b) (i) (ii) (iv) (iii) the nucleus .
(c) (iii) (ii) (i) (iv) (4) 1 mm to (iv) Magnetron valve.
(d) (i) (ii) (iii) (iv) 0.1 m
52. If microwaves, X rays, infrared, gamma rays, ultra-violet, (a) (1)-(iv), (2)-(iii), (3)-(ii), (4)-(i)
radio waves and visible parts of the electromagnetic (b) (1)-(iii), (2)-(iv), (3)-(i), (4)-(ii)
spectrum are denoted by M, X, I, G, U, R and V then which (c) (1)-(ii), (2)-(iii), (3)-(iv), (4)-(i)
of the following is the arrangement in ascending order of (d) (1)-(i), (2)-(ii), (3)-(iii), (4)-(iv)
wavelength ? [Online April 19, 2014] 55. Photons of an electromagnetic radiation has an energy
(a) R, M, I, V, U, X and G 11 keV each. To which region of electromagnetic spectrum
(b) M, R, V, X, U, G and I does it belong ? [Online April 9, 2013]
(c) G, X, U, V, I, M and R (a) X-ray region (b) Ultra violet region
(d) I, M, R, U, V, X and G (c) Infrared region (d) Visible region
53. Match the List-I (Phenomenon associated with 56. The frequency of X-rays; g-rays and ultraviolet rays are
electromagnetic radiation) with List-II (Part of respectively a, b and c then [Online May 26, 2012]
electromagnetic spectrum) and select the correct code from (a) a < b; b > c (b) a > b ; b > c
the choices given below this lists:[Online April 11, 2014] (c) a < b < c (d) a = b = c
Electromagnetic Waves P-383

1. (a) Relation between electric field E0 and magnetic field Directiono f wave propagation
B0 of an electromagnetic wave is given by ( E ´ B ) || C
E
c= 0 (Here, c = Speed of light) Bˆ = iˆ and Cˆ = - ˆj \ Ê = - kˆ
B0 \ E = E0 sin[200p( y + ct )](- kˆ) V/m
Þ E0 = B0 ´ c = 1.2 ´ 10-7 ´ 3 ´ 108 = 36 or, E = -9sin[200p( y + ct )]kˆ V/m
As the wave is propagating along x-direction, magnetic 5. (c) Relation between electric field and magnetic field for
field is along z-direction E0
an electromagnetic wave in vacuum is B0 = .
and ( Eˆ ´ Bˆ ) || Cˆ c
r 1
\ E should be along y-direction. In free space, its speed c =
r r m0 e0
So, electric field E = E0 sin E × ( x, t ) Here, m0 = absolute permeability, e0 = absolute permittivity
V E E0
= [ -36sin (0.5 ´ 103 x + 1.5 ´ 1011t ) ˆj ] \ B0 = 0 = = E0 m 0 e 0
m c 1/ m 0 e 0
As the electromagnetic wave is propagating along x
E0
2. (c) In electromagnetic wave, =C direction and electric field is along y direction.
B0
\ Eˆ ´ Bˆ || Cˆ (Here, Ĉ = direction of propagation of wave)
r
E0 \ B should be in k̂ direction.
\ Maximum value of magnetic field, B0 =
C \ B = E 0 m0 e0 cos (wt – kx) k̂
qV0 E0 At t = 0
Fmax = qVBmax sin 90° =
C B = E 0 m0 e0 cos (kx) k̂
(Given V0 = 0.1 C and E0 = 30)
1 B2
-19 8 6. (b) Energy density =
1.6 ´ 10 ´ 0.1 ´ 3 ´ 10 ´ 30 2 m0
= = 4.8 ´ 10-19 N
3 ´ 108 ÞB= 2 ´ m 0 ´ Energy density
r
3. (a) E = E0 ( xˆ + yˆ ) sin(kz - wt ) 1
m0 = 2
= 4p ´ 10 -7
Direction of propagation of em wave = + k̂ C e0

iˆ + ˆj \ B = 2 ´ 4p ´ 10-7 ´ 1.02 ´ 10 -8 = 160 ´ 10 -9


Unit vector in the direction of electric field, Eˆ =
2 = 160 nT
The direction of electromagnetic wave is perpendicular to 7. (a) Electromagnetic wave will propagate perpendicular to
both electric and magnetic field. the direction of Electric and Magnetic fields

\ kˆ = Eˆ ´ Bˆ Cˆ = Eˆ ´ Bˆ
Here unit vector Ĉ is perpendicular to both Ê and B̂
æ iˆ + ˆj ö ˆ -iˆ + ˆj ur ur
Þ kˆ = ç ÷ ´ B Þ Bˆ = Given, E = k$ , B = 2i$ - 2 $j
è 2 ø 2
iˆ ˆj kˆ
r E 1 iˆ + ˆj
\ B = 0 (- xˆ + yˆ )sin(kz - wt ) \ Cˆ = Eˆ ´ Bˆ = 0 0 1 =
c 2 2
1 -1 0
4. (d) Given : B = 3 ´ 10-8 sin[200p( y + ct )]iT
ˆ
iˆ + ˆj
Þ Cˆ =
-8 2
\ B0 = 3 ´ 10

E0 = CB0 Þ E0 = 3 ´ 108 ´ 3 ´ 10 -8 = 9 V/m


P-384 Physics
r r
8. (d) Given: E1 = E0 ˆj cos ( wt - kx ) 11. (b) Given, B = 3 ´ 10 -8 sin(1.6 ´ 103 x + 48 ´ 1010 t)
r r
i.e., Travelling in +ve x-direction E ´ B should be in x- Using, E0 = B0 ´ C = 3 ´ 10 –8 ´ 3 ´ 108 = 9 V/m
direction
r \ Electric field,
\ B is in K̂ r
E = 9sin(1.6 ´ 103 x + 48 ´ 1010 t ) kˆ V /m
r E æ E0 ö
\ B1 = 0 cos ( wt - kx ) kˆ çèQ B0 = C ÷ø p
C 12. (c) At t = 0, z =
k
r E E
\ E = 0 (iˆ + ˆj ) cos[p ] = – 0 (iˆ + ˆj )
2 2
r r
FE = qE
-(iˆ + ˆj )
Force due to electric field will be in the direction
2
Force due to magnetic field is in direction
r r r r r r
q(v ´ B) and v || k . Therefore, it is parallel to E .
E2 = E0 kˆ cos ( wt - ky )
r E r r r iˆ + ˆj
B2 = 0 iˆ cos ( wt - ky ) Þ Fnet = FE + FB is antiparallel to
2
C
\ Travelling in +ve y-axis
r r ˆ 6 ˆj + 8kˆ -3 ˆj + 4kˆ
E ´ B should be in y-axis 13. (c) S = 2 =
5
r r r r 6 + 82
\ Net force F = qE + q v ´ B ( ) E0
r r r r 60
( ) (
q E1 + E2 + q 0.8cjˆ ´ B1 + B2 ( ) 14. (b) B0 = =
C 3 ´ 108
If t = 0 and x = y = 0 = 20 × 10–8 T = 2 × 10–7 T
r r
E1 = E0 ˆj E2 = E0 kˆ
w 2pf 2p ´ 23.9 ´ 109
r E r E K= = = = 500
B1 = 0 kˆ B2 = 0 iˆ v v 3 ´ 108
c c ®
r Therefore, B = B sin(kz - wt )
( ) E
\ Fnet = qE0 ˆj + kˆ + q ´ 0.8c ´ 0 ˆj ´ kˆ + iˆ
C
( ) 0

= 2 × 10–7 sin(0.5 ´ 103 z - 1.5 ´ 1011 t )i


( ) ˆ
= qE ˆj + k + 0.8 qE iˆ - k
0 0(
ˆ ) 15. (a)

= qE0 ( 0.8iˆ + ˆj + 0.2 kˆ ) E0


=C
B0
9. (a) Direction of polarisation = Ê = kˆ
E0
$ $ Þ B0 =
µ´B µ = i+ j C
Direction of propagation = E
2 r
Given that E = E0 cos(kz) cos(w t) iˆ
r r
But E.B = 0 \ Bˆ = iˆ - j r E
2 E = 0 éëcos ( kz – wt ) ˆi – cos ( kz + wt ) iˆùû
10. (d) Amplitude of electric field (E) and Magnetic field (B) 2
of an electromagnetic wave are related by the relation Correspondingly
r B
E B = 0 éëcos ( kz – wt ) ˆj – cos ( kz + wt ) ˆjùû
=c 2
B
r B
Þ E = Bc B = 0 ´ 2sin kz sin wt
Þ E = 5 × 10–8 × 3 × 108 = 15 N/C 2
r r æE ö
Þ E = 15iˆ V / m B = ç 0 sin kz sin wt ÷ ˆj
è C ø
Electromagnetic Waves P-385

I B20
16. (c) Pressure, P =
C 22. (d) I = ·C
2µ0
F I
Þ =
A C B20 Iµ0
Þ =
IA Dp 2 C
ÞF= =
C Dt Iµ0
Þ Brms =
I C
Þ Dp = ADt
C
108 ´ 4 p ´ 10 -7
(25 ´ 25) ´ 104 ´ 10–4 ´ 40 ´ 60 =
= N-s 3 ´ 108
3 ´ 108 ; 6 × 10–4 T
= 5 × 10–3 N-s Which is closest to 10–4.
17. (a) B0 = B02 + B12 = 302 + 2 2 ´ 10 -6 23. (c) The speed of electromagnetic wave in free space is
given by
= 30 × 10–6T
1
\ E0 = CB = 3 ´ 108 ´ 30 ´ 10 -6 C= ...(i)
m0 Î0
= 9 × 103 V/m
In medium, v = 1 ...(ii)
E0 9
= ´ 103V / m k Î0 m 0
V2 2
Dividing equation (i) by (ii), we get
Force on the charge,
9 C
F = EQ = ´ 103 ´ 10 -4 ; 0.64 N \ = k =n
V
2
18. (b) As we know, 1 1
Î0 E 02C = intensity = Î0 kE 2 v
r 2 2
r |E| 6.3
| B |= = = 2.1 ´ 10-8 T \ E 0C = kE v
2 2
C 3 ´ 108
and Eˆ ´ Bˆ =C ˆ E 20 kV n 2 E 0
Þ = = Þ = n
ˆ = iˆ [Q EM wave travels along +(ve) x-direction.] E2 C n E
Jˆ ´ B similarly
r
ˆ
\ B̂ = kˆ or B = 2.1 ´ 10 –8 kT B20 C B 2 v B0 1
= Þ =
IA 2m0 2m0 B n
19. (b) F = (1 + r )
C 24. (d) EM wave intensity
(1 + 0.25) ´ 50 ´ 1 Power 1
= 8
Þ I= = e 0 E 02 c
3 ´ 10 Area 2
[where E0= maximum electric field]
; 20 ´ 10-8 N
27 ´ 10 –3 1
Þ = ´ 9 ´ 10 –12 ´ E 20 ´ 3 ´ 108
20. (a) The relation between amplitudes of electric and 10 ´10 –6 2
magnetic field in free space is given by
E 6 Þ E 0 = 2 ´103 kV / m =1.4kV / m
B0 = 0 = = 2 ´ 10 -8 T
c 3 ´ 108
25. (b) Using, formula E0 = B0 × C
Propagation direction = Eˆ ´ Bˆ = 100 × 10–6 × 3 × 108
= 3 × 104 N/C
iˆ = ˆj ´ Bˆ
Here we assumed that
Þ B̂ = kˆ B0 = 100 × 10–6 is in tesla (T) units
r
21.
\ The magnetic field component will be along z direction.
(c) E0 = cB0 = 3 × 108 × 1.6 × 10–6 = 4.8 × 102 V/m
ˆ
26. (b) E =10jcos ë ( )( )
é 6iˆ + 8kˆ . xiˆ + zkˆ ù
û
uur uur uur r r
Also S Þ E ´ B ˆ
=10 jcos éë K . r ùû
uur uur r
or - K Þ E ´ (2iˆ + ˆj ) \ K =6iˆ + 8K;ˆ direction of waves travel
uur
(
Therefore direction of E ® - iˆ + 2 ˆj ) i. e. direction of ‘c’.
P-386 Physics

29. (c) If E0 is magnitude of electric field then


E(10 ˆj)
1 2I
e0 E2 ´ C = 1 Þ E0 =
2 Ce 0
E0
E0 =
C
r r
Direction of E ´ B will be along + ĵ .
3iˆ + 4kˆ 30. (c) Eˆ ´ B
ˆ should give the direction of wave propagation
B = ĉ
5
) ˆ ˆ æ ˆ ˆö ˆ ˆ ˆ ˆ ˆ ˆ
Þ K´B ˆ P i´ jÞK ˆ ´ i + j = j - ( - i) = i + j P i + j
–4iˆ + 3kˆ ç ÷
2 è 2ø 2 2 2
Cˆ ´ Eˆ =
5 Option (a), option (b) and option (d) does not satisfy.
r E 10 ˆi + ˆj
B= = Wave propagation vector K̂ should along .
C C 2
r 10 æ –4iˆ + 3kˆ ö æ –8iˆ + 6kˆ ö 31. (c) Given, Electric field component of monochromatic
\ B çç ÷= ç ÷ r
Cè 5 ÷ø çè C ÷ø radiation, (E) = 2E ˆi coskzcos wt
0
r dE dB
1 We know that, =-
or, magnetic field B ( x, z, t ) = dz dt
C
dE dB
( )
6kˆ – 8iˆ cos ( 6x + 8z –10ct ) dz
= -2E 0 k sin kz cos w t = -
dt
1 dB = + 2E0k sin kz cos wt dt ..... (i)
27. (c) Velocity of EM wave is given by v = Integrating eq.n (i), we have

w B = +2E 0 k sin kz ò cos wt dt
Velocity in air = =C
k Magnetic field is given by,
C k
Velocity in medium = = +2E 0 sin kz sin wt
2 w
Here, m1 = m2 = 1 as medium is non-magnetic We also know that,
1 E0 w
= =c
Îr1 Îr1 1 B0 k
C
\ = =2 Þ = Magnetic field vector,
1 æCö Îr2 4
Îr2 çè 2 ÷ø
r 2E
B = 0 ˆj sin kz sin wt
c
28. (d) Average energy density of magnetic field, E0
32. (a) Speed of EM wave in force space (c) =
B02 B0
uB = . r
4m 0 or E = cB0 sin (kx + wt)kˆ
Average energy density of electric field, 33. (d) Wave in X-direction means E and B should be function
of x and t.
e0 E 20 ) ) ) )
uE = y,z ^ y∗z
4 uur ur uur ur
34. (b) As we know, E . B = 0 Q [ E ^ B ]
1 uur ur
Now, E0 = CB0 and C2 = and E ´ B should be along Z direction
m0 Î0

e0 2 As (–2$i – 3 $j ) ´ (3$i – 2 $j ) = 5k$


uE = ´ C 2 B 20 = e0 ´ 1 ´ B02 = B0 = u B Hence option (b) is the correct answer.
4 4 m 0 e0 4m 0
\ uE = uB 35. (d) As we know,
Since energy density of electric and magnetic field is same, E 27
so energy associated with equal volume will be equal i.e., B0 = 0 = 8
= 9 ´10 –8 tesla
C 3 ´ 10
uE = uB
Electromagnetic Waves P-387

Oscillation of B can be only along ĵ or k̂ direction. 40. (b)


41. (d) Electromagnetic waves do not required any medium
w = 2pf = 2p × 2 × 1014 Hz to propagate. They can travel in vacuum. They are
ur
\ B ( x, t ) = (9 ´ 10 –8 T )kˆ sin[2p(1.5 ´ 10 –6 ´ –2 ´104 t )] transverse in nature like light. They carry both energy and
1 momentum.
36. (c) E0 = CB0 and C = A changing electric field produces a changing magnetic
m 0e 0
field and vice-versa. Which gives rise to a transverse wave
1 known as electromagnetic wave.
Electric energy density = e0 E02 = m E
2 42. (b) Q The E.M. wave are transverse in nature i.e.,
r r
1 Bo 2
= mB k ´E r
Magnetic energy density = = =H …(i)
2 m0 m
Thus, mE = mB r
r B
Energy is equally divided between electric and magnetic where H =
field. m
37. (b) Wavelength of monochromatic green light r r
k ´H r
= 5.5 × 10–5 cm and = -E …(ii)
we
Power r r r r
Intensity I =
Area k is ^ H and k is also ^ to E
r r
100 ´ ( 3 /100 ) The direction of wave propagation is parallel to E ´ B.
3
= = Wm -2 The direction of polarization is parallel to electric field.
4p ( 5 )
2 100p
43. (c) In an electromagnetic wave electric field and
Now, half of this intensity (I) belongs to electric field and magnetic field are perpendicular to the direction of
half of that to magnetic field, therefore, propagation of wave. The vector equation for the electric
I 1 field is
= e0 E 02 C r
E = E0 cos æç wt - 2p y ö÷ zˆ
2 4
2I è l ø
or E 0 = 44. (a) Frequency remains unchanged during refraction
e0 C Velocity of EM wave in vacuum
æ 3 ö 1
2´ç p÷ Vvacuum = =C
è 100 ø m0 Î0
=
æ
ç
1

è 4p ´ 9 ´10 ø
ö
(
´ 3 ´108 ) vmed =
1
=
c
µ0 Î0 ´4 2
6
= ´ 30 = 7.2 l med vmed c/2 1
25 = = =
l vacuum v vacuum c 2
\ E 0 = 2.68 V / m
\ Wavelength is halved and frequency remains
38. (c) Given: Amplitude of electric field,
unchanged
E0 = 4 v/m
Absolute permitivity, 45. (a) The phenomenon of polarisation is shown only by
e0 = 8.8 × 10–12 c2/N-m2 transverse waves. The vibration of electromagnetic wave
Average energy density uE = ? are restricted through polarization in a direction
perpendicular to wave propagation.
Applying formula,
46. (d) Energy sequence of radiations is
1 2
Average energy density uE = e0 E Eg -Rays > EX-Rays > Emicrowave > EAM Radiowaves
4
1 -12 2 \ l g -Rays < l X-Rays < l microwave < l AM Radiowaves
Þ uE = ´ 8.8 ´ 10 ´ (4)
4
= 35.2 × 10–12 J/m3 From the above sequence, we have
39. (b) From question, (a) Microwave ® 10 -3 m (iv)
B0 = 20 nT = 20 × 10–9T
(b) Gamma Rays ® 10-15 m (ii)
(Q velocity of light in vacuum C = 3 × 108 ms–1)
r r r
E0 = B0 ´ C (c) AM Radio wave ® 100 m (i)
r r r
| E0 |=| B | ×| C |= 20 ´ 10 -9 ´ 3 ´ 108 (d) X-Rays ® 10-10 m (iii)
= 6 V/m.
P-388 Physics

47. (b) The orderly arrangement of different parts of EM wave 53. (d) Wavelength emitted by atomic hydrogen in interstellar
in decreasing order of wavelength is as follows: space - Part of short radio wave of electromagnetic
l radiowaves > l microwaves > l visible > l X-rays spectrum.
Doublet of sodium - visible radiation.
48. (c) Optical Fibre Communication – Infrared Light
54. (d) Vibration of atoms and molecules 700 nm to 1 mm
Radar – Radio Waves
Radioactive decay of the nucleus < 10–3 nm
Sonar – Ultrasound
Magnetron valve 1 mm to 0.1 m
Mobile Phones – Microwaves
E, Decreases hc hc
49. (c) 55. (a) E = Þ l=
g-rays X-rays uv-rays Visible rays IR rays Radio l E
VIBGYOR Microwaves waves
6.6 ´ 10-34 ´ 3 ´108
Radio wave < yellow light < blue light < X-rays Þ l=
11´1000 ´ 1.6 ´10 -19
(Increasing order of energy)
= 12.4 Å
50. (c) Microwave oven acts on the principle of giving
vibrational energy to water molecules. Increasing order of frequency
51. (d)
(1) Infrared rays are used to treat muscular strain because x-rays u-v rays visible Infrared
these are heat rays. wavelength range of visible region is 4000Å to 7800Å.
(2) Radio waves are used for broadcasting because these
waves have very long wavelength ranging from few 56. (a) Frequency range of g-ray,
centimeters to few hundred kilometers. b = 1018 – 1023 Hz
(3) X-rays are used to detect fracture of bones because Frequency range of X-ray,
they have high penetrating power but they can't penetrate
through denser medium like dones. a = 1016 – 1020 Hz
(4) Ultraviolet rays are absorbed by ozone of the Frequency range of ultraviolet ray,
atmosphere. c = 1015 – 1017 Hz
52. (c) Gamma rays < X-rays < Ultra violet < Visible rays
< Infrared rays < Microwaves < Radio waves. \ a < b; b > c
23
Ray Optics and
Optical Instruments
below. The distance over which the man can see the image
Plane Mirror, Spherical Mirror
TOPIC 1 of the light source in the mirror is: [12 Jan. 2019 I]
and Reflection of Light

1. When an object is kept at a distance of 30 cm from a concave


mirror, the image is formed at a distance of 10 cm from the d
S
mirror. If the object is moved with a speed of 9
cms–1, the speed (in cms–1) with which image moves at L
that instant is ________. [NA Sep. 03, 2020 (II)]
2L
2.
(a) d (b) 2d
Object
d
(c) 3d (d)
2
20 16 12 8 4 5. Two plane mirrors are inclined to each other such that a
(cm) ray of light incident on the first mirror (M1) and parallel to
the second mirror (M2) is finally reflected from the second
mirror (M2) parallel to the first mirror (M1). The angle
between the two mirrors will be: [9 Jan. 2019 II]
A spherical mirror is obtained as shown in the figure from (a) 45° (b) 60°
a hollow glass sphere. If an object is positioned in front of (c) 75° (d) 90°
the mirror, what will be the nature and magnification of the
image of the object? (Figure drawn as schematic and not 6. An object is gradually moving away from the focal point
to scale) [Sep. 02, 2020 (I)] of a concave mirror along the axis of the mirror. The
(a) Inverted, real and magnified graphical representation of the magnitude of linear
magnification (m) versus distance of the object from the
(b) Erect, virtual and magnified
mirror (x) is correctly given by
(c) Erect, virtual and unmagnified
(Graphs are drawn schematically and are not to scale)
(d) Inverted, real and unmagnified
[8 Jan. 2020 II]
3. A concave mirror for face viewing has focal length of 0.4 m.
(a)
The distance at which you hold the mirror from your face
in order to see your image upright with a magnification of
5 is: [9 April 2019 I]
(a) 0.24 m (b) 1.60 m (c) 0.32 m (d) 0.16 m
4. A point source of light, S is placed at a distance L in front
of the centre of plane mirror of width d which is hanging
vertically on a wall. A man walks in front of the mirror
along a line parallel to the mirror, at a distance 2L as shown
P-390 Physics

(b) 10. To get three images of a single object, one should have
two plane mirrors at an angle of [2003]
(a) 60º (b) 90º (c) 120º (d) 30º
11. If two plane mirrors are kept at 60° to each other, then the
number of images formed by them is [2002]
(a) 5 (b) 6 (c) 7 (d) 8

Refraction of Light at Plane


(c) TOPIC 2 Surface and Total Internal
Reflection
12. An observer can see through a small hole on the side of a
jar (radius 15 cm) at a point at height of 15 cm from the
bottom (see figure). The hole is at a height of 45 cm. When
the jar is filled with a liquid up to a height of 30 cm the same
observer can see the edge at the bottom of the jar. If the
refractive index of the liquid is N/100, where N is an integer,
(d) the value of N is ___________. [NA Sep. 03, 2020 (I)]

45 cm
7. A particle is oscillating on the X-axis with an amplitude 15 cm
2 cm about the point x0 = 10 cm with a frequency w. A concave
mirror of focal length 5 cm is placed at the origin (see figure)
15 cm
Identify the correct statements: [Online April 15, 2018]
(A) The image executes periodic motion 13. A light ray enters a solid glass sphere of refractive index
(B) The image executes non-periodic motion m = 3 at an angle of incidence 60°. The ray is both
(C) The turning points of the image are asymmetric w.r.t reflected and refracted at the farther surface of the sphere.
the image of the point at x = 10 cm The angle (in degrees) between the reflected and refracted
(D) The distance between the turning points of the rays at this surface is ___________.
[NA Sep. 02, 2020 (II)]
100 x0 = 10 cm
oscillation of the image is x=0 14. A vessel of depth 2h is half filled with a liquid of refractive
21
index 2 2 and the upper half with another liquid of
(a) (B), (D) (b) (B), (C)
refractive index 2. The liquids are immiscible. The
(c) (A), (C), (D) (d) (A), (D)
apparent depth of the inner surface of the bottom of vessel
8. You are asked to design a shaving mirror assuming that a will be: [9 Jan. 2020 I]
person keeps it 10 cm from his face and views the magnified h h
image of the face at the closest comfortable distance of 25 (a) (b)
2 2( 2 + 1)
cm. The radius of curvature of the mirror would then be :
[Online April 10, 2015] h 3
(c) (d) h 2
(a) 60 cm (b) –24 cm 3 2 4
(c) – 60 cm (d) 24 cm 15. There is a small source of light at some depth below the
4
9. A car is fitted with a convex side-view mirror of focal length surface of water (refractive index = ) in a tank of large
3
20 cm. A second car 2.8 m behind the first car is overtaking cross sectional surface area. Neglecting any reflection from
the first car at a relative speed of 15 m/s. The speed of the the bottom and absorption by water, percentage of light
image of the second car as seen in the mirror of the first that emerges out of surface is (nearly):
one is : [2011] [Use the fact that surface area of a spherical cap of height
1 1 h and radius of curvature r is 2prh] [9 Jan. 2020 II]
(a) m/s (b) 10 m/s (c) 15 m/s (d) m/s (a) 21% (b) 34% (c) 17% (d) 50%
15 10
Ray Optics and Optical Instruments P-391

16. The critical angle of a medium for a specific wavelength, if


the medium has relative permittivity 3 and relative 2 3 2b
(a) + 2b (b) 2a +
a 3
4
permeability for this wavelength, will be: [8 Jan. 2020 I]
3 2b
(a) 15° (b) 30° (c) 2a + (d) 2a + 2b
3
(c) 45° (d) 60° 20. In figure, the optical fiber is l = 2 m long and has a diameter
17. A concave mirror has radius of curvature of 40 cm. It is at of d = 20 mm. If a ray of light is incident on one end of the
the bottom of a glass that has water filled up to 5 cm (see fiber at angle q1 = 40°, the number of reflections it makes
figure). If a small partricle is floating on the surface of before emerging from the other end is close to :
water, its image as seen, from directly above the glass, is at
(refractive index of fiber is 1.31 and sin 40° = 0.64)
a distance d from the surface of water. The value of d is
close to : [12 Apr. 2019 I] [8 April 2019 I]
(Refractive index of water = 1.33)

(a) 6.7 cm (b) 13.4 cm (c) 8.8 cm (d) 11.7 cm (a) 55000 (b) 66000 (c) 45000 (d) 57000
18. A transparent cube of side d, made of a material of refractive 21. A light wave is incident normally on a glass slab of
index m2, is immersed in a liquid of refractive index refractive index 1.5. If 4% of light gets reflected and the
m1(m1< m2). A ray is incident on the face AB at an angle q amplitude of the electric field of the incident light is 30 V/
(shown in the figure). Total internal reflection takes place m, then the amplitude of the electric field for the wave
at point E on the face BC. propogating in the glass medium will be:[12 Jan. 2019 I]
(a) 30 V/m (b) 10 V/m
(c) 24 V/ m (d) 6 V/m
22. Let the refractive index of a denser medium with respect to
a rarer medium be n 12 and its critical angle be qC. At an
angle of incidence A when light is travelling from denser
medium to rarer medium, a part of the light is reflected and
the rest is refracted and the angle between reflected and
Then q must satisfy : [12 Apr. 2019 II] refracted rays is 90°. Angle A is given by :
[Online April 8, 2017]
-1 m1 m 22
(a) q < sin (b) q > sin -1 -1
m2 m12 1 1
(a) (b)
-1 -1
cos (sin qC ) tan (sin qC )
m 22 m1
(c) q < sin -1 -1 (d) q > sin -1 (c) cos–1 (sin qC) (d) tan–1 (sin qC)
m12 m2
23. A diver looking up through the water sees the outside
19. A ray of light AO in vacuum is incident on a glass slab at world contained in a circular horizon. The refractive index
angle 60o and refracted at angle 30o along OB as shown in 4
of water is , and the diver’s eyes are 15 cm below the
the figure. The optical path length of light ray from A to B 3
is : [10 Apr. 2019 I] surface of water. Then the radius of the circle is:
[Online April 9, 2014]
(a) 15 ´ 3 ´ 5 cm (b) 15 ´ 3 7 cm

15 ´ 7 15 ´ 3
(c) cm (d) cm
3 7
P-392 Physics

24. A printed page is pressed by a glass of water. The refractive æ 1ö æ 1 ö


index of the glass and water is 1.5 and 1.33, respectively. If (c) ç1 + ÷ h2 - ç 1 + ÷ h1
the thickness of the bottom of glass is 1 cm and depth of è m1 ø è m2 ø
water is 5 cm, how much the page will appear to be shifted æ 1 ö æ 1 ö
if viewed from the top ? [Online April 25, 2013] (d) ç 1 - ÷ h2 + ç 1 - ÷ h1
è m1 ø è m2 ø
(a) 1.033 cm (b) 3.581 cm 29. A transparent solid cylindrical rod has a refractive index of
(c) 1.3533 cm (d) 1.90 cm 2
. It is surrounded by air. A light ray is incident at the
25. A light ray falls on a square glass slab as shown in the 3
mid-point of one end of the rod as shown in the figure.
diagram. The index of refraction of the glass, if total internal
reflection is to occur at the vertical face, is equal to :
[Online April 23, 2013]
q

45° Incident ray


The incident angle q for which the light ray grazes along
the wall of the rod is : [2009]
æ 2 ö
(a) sin
-1
(
3/2 ) (b) sin ç
-1
è 3ø
÷

æ 1 ö
(c) sin -1 ç ÷ (d) sin -1 (1/ 2 )
è 3ø
30. A fish looking up through the water sees the outside world
contained in a circular horizon. If the refractive index of
5 3 3 4
(a) ( 2 + 1) (b) (c) (d) water is and the fish is 12 cm below the surface, the
2 2 2 2 3
radius of this circle in cm is [2005]
26. Light is incident from a medium into air at two possible
angles of incidence (A) 20° and (B) 40°. In the medium 36
(a) (b) 36 7 (c) 4 5 (d) 36 5
light travels 3.0 cm in 0.2 ns. The ray will : 7
[Online April 9, 2013] 31. Consider telecommunication through optical fibres. Which
(a) suffer total internal reflection in both cases (A) and
of the following statements is not true? [2003]
(B)
(a) Optical fibres can be of graded refractive index
(b) suffer total internal reflection in case (B) only
(b) Optical fibres are subject to electromagnetic
(c) have partial reflection and partial transmission in case
interference from outside
(B)
(d) have 100% transmission in case (A) (c) Optical fibres have extremely low transmission loss
(d) Optical fibres may have homogeneous core with a
27. Let the x-z plane be the boundary between two transparent
suitable cladding.
media. Medium 1 in z ³ 0 has a refractive index of 2 and
32. Which of the following is used in optical fibres? [2002]
medium 2 with z < 0 has a refractive index of 3 . A ray of
r (a) total internal reflection (b) scattering
light in medium 1 given by the vector A = 6 3iˆ + 8 3 ˆj - 10kˆ
(c) diffraction (d) refraction.
is incident on the plane of separation. The angle of
refraction in medium 2 is: [2011] Refraction at Curved Surface
(a) 45° (b) 60° (c) 75° (d) 30° TOPIC 3
Lenses and Power of Lens
28. A beaker contains water up to a height h1 and kerosene
of height h2 above water so that the total height of 33. A point like object is placed at a distance of 1 m in front of
(water + kerosene) is (h1 + h2). Refractive index of water is a convex lens of focal length 0.5 m. A plane mirror is placed
m1and that of kerosene is m2. The apparent shift in the at a distance of 2 m behind the lens. The position and
position of the bottom of the beaker when viewed from nature of the final image formed by the system is :
above is [2011 RS] [Sep. 06, 2020 (I)]
æ 1ö æ 1 ö (a) 2.6 m from the mirror, real
(a) ç1 + m ÷ h1 - ç1 + m ÷ h2 (b) 1 m from the mirror, virtual
è 1ø è 2ø
(c) 1 m from the mirror, real
æ 1 ö æ 1 ö
(b) ç 1 - m ÷ h1 + ç 1 - m ÷ h2 (d) 2.6 m from the mirror, virtual
è 1ø è 2ø
Ray Optics and Optical Instruments P-393

34. A double convex lens has power P and same radii of cur- 39. One plano-convex and one plano-concave lens of same
vature R of both the surfaces. The radius of curvature of a radius of curvature ‘R’ but of different materials are joined
surface of a plano-convex lens made of the same material side by side as shown in the figure. If the refractive index
with power 1.5 P is : [Sep. 06, 2020 (II)] of the material of 1 is m1 and that of 2 is m2, then the focal
R 3R R length of the combination is : [10 Apr. 2019 I]
(a) 2R (b) (c) (d)
2 2 3
35. For a concave lens of focal length f, the relation between
object and image distances u and v, respectively, from its pole
can best be represented by (u = v is the reference line) :
R 2R
[Sep. 05, 2020 (I)] (a) (b)
v m1 - m2 m1 - m2
v
=

f
u

2R R
(c) (d)
2(m1 - m2 ) 2 - (m1 - m2 )
(a)
40. The graph shows how the magnification m produced by a
f u thin lens varies with image distance v. What is the focal
v length of the lens used ? [10 Apr. 2019 II]
v
=

f
u

(b)

f u
v
v
=

f
u

(c)
b2 b2c a b
u (a) (b) (c) (d)
f ac a c c
v
v

41. A convex lens of focal length 20 cm produces images of


=

f
u

the same magnification 2 when an object is kept at two


distances x1 and x2 (x1 > x2) from the lens. The ratio of x1
(d) and x2 is: [9 Apr. 2019 II]
f u (a) 2 : 1 (b) 3 : 1 (c) 5 : 3 (d) 4 : 3
36. The distance between an object and a screen is 100 cm. A 42. A thin convex lens L (refractive index = 1.5) is placed on a
lens can produce real image of the object on the screen for
plane mirror M. When a pin is placed at A, such that OA =
two different positions between the screen and the object.
The distance between these two positions is 40 cm. If the 18 cm, its real inverted image is formed at A itself, as shown
æ N ö in figure. When a liquid of refractive index µi is put between
power of the lens is close to ç D where N is an
è 100 ÷ø the lens and the mirror, the pin has to be moved to A’, such
integer, the value of N is ___________. that OA’ = 27 cm, to get its inverted real image at A’ itself.
[NA Sep. 04, 2020 (II)] The value of µi will be: [9 Apr. 2019 II]
37. A point object in air is in front of the curved surface of a
plano-convex lens. The radius of curvature of the curved
surface is 30 cm and the refractive index of the lens material
is 1.5, then the focal length of the lens (in cm)
is__________. [NA 8 Jan. 2020 I]
38. A thin lens made of glass (refractive index = 1.5) of focal
length f = 16 cm is immersed in a liquid of refractive index
1.42. If its focal length in liquid is fl ,then the ratio fl /f is 4 3
closest to the integer: [7 Jan. 2020 II] (a) (b) (c) 3 (d) 2
3 2
(a) 1 (b) 9 (c) 5 (d) 17
P-394 Physics

43. An upright object is placed at a distance of 40 cm in front R


of a convergent lens of focal length 20 cm. A convergent (a) f1 – f2 (b)
µ2 - µ1
mirror of focal length 10 cm is placed at a distance of 60 cm
on the other side of the lens. The position and size of the 2 f1 f 2
(c) (d) f1 + f2
final image will be : [8 April 2019 I] f1 + f 2
(a) 20 cm from the convergent mirror, same size as the object 48. An object is at a distance of 20 m from a convex lens of
(b) 40 cm from the convergent mirror, same size as the object
focal length 0.3 m. The lens forms an image of the object. If
(c) 40 cm from the convergent lens, twice the size of the
object the object moves away from the lens at a speed of 5m/s,
(d) 20 cm from the convergent mirror, twice the size of the the speed and direction of the image will be :
object [11 Jan. 2019 I]
44. A convex lens (of focal length 20 cm) and a concave mirror, (a) 2.26 × 10–3 m/s away from the lens
having their principal axes along the same lines, are kept (b) 0.92 × 10–3 m/s away from the lens
80 cm apart from each other. The concave mirror is to the (c) 3.22 × 10–3 m/s towards the lens
right of the convex lens. When an object is kept at a (d) 1.16 × 10–3 m/s towards the lens
distance of 30 cm to the left of the convex lens, its image 49. A plano convex lens of refractive index m1 and focal
remains at the same position even if the concave mirror is length f1 is kept in contact with another plano concave
removed. The maximum distance of the object for which lens of refractive index m2 and focal length f2 If the
this concave mirror, by itself would produce a virtual image radius of curvature of their spherical faces is R each
would be : [8 Apr. 2019 II] and f1 = 2f2, then m1 and m2 are related as:
(a) 30 cm (b) 25 cm (c) 10 cm (d) 20 cm [10 Jan. 2019 I]
45. What is the position and nature of image formed by lens (a) m1 + m2 = 3 (b) 2m1 – m2 = 1
combination shown in figure? (f1, f2 are focal lengths) (c) 3m2 – 2m1 = 1 (d) 2m2 – m1 = 1
[12 Jan. 2019 I]
50. The eye can be regarded as a single refracting surface.
2 cm The radius of curvature of this surface is equal to that
of cornea (7.8 mm). This surface separates two media of
A B refractive indices 1 and 1.34. Calculate the distance from
the refracting surface at which a parallel beam of light
will come to focus. [10 Jan. 2019 II]
(a) 1 cm (b) 2 cm
20 cm f1 = + 5 cm f2 = –5 cm (c) 4.0 cm (d) 3.1 cm
(a) 70 cm from point B at left; virtual 51. A convex lens is put 10 cm from a light source and it
(b) 40 cm from point B at right; real makes a sharp image on a screen, kept 10 cm from the lens.
Now a glass block (refractive index 1.5) of 1.5 cm thickness
20
(c) cm from point B at right, real is placed in contact with the light source. To get the sharp
3 image again, the screem is shifted by a distance d. Then d
(d) 70 cm from point B at right; real is: [9 Jan. 2019 I]
46. Formation of real image using a biconvex lens is shown (a) 1.1 cm away from the lens
below : [12 Jan. 2019 II] (b) 0
(c) 0.55 cm towards the lens
screen (d) 0.55 cm away from the lens
2f
52. A planoconvex lens becomes an optical system of 28 cm
2f f f
focal length when its plane surface is silvered and
illuminated from left to right as shown in Fig-A. If the same
lens is instead silvered on the curved surface and
If the whole set up is immersed in water without disturbing
illuminated from other side as in Fig. B, it acts like an optical
the object and the screen positions, what will one observe
system of focal length 10 cm. The refractive index of the
on the screen ?
material of lens if: [Online April 15, 2018]
(a) Image disappears (b) Magnified image
(c) Erect real image (d) No change
47. A plano-convex lens (focal length f2, refractive index µ2,
radius of curvature R) fits exactly into a plano-concave
lens (focal length f 1 , refractive index µ1 , radius of
curvature R). Their plane surfaces are parallel to each other.
Fig. A Fig. B
Then, the focal length of the combination will be :
[12 Jan. 2019 II] (a) 1.50 (b) 1.55 (c) 1.75 (d) 1.51
Ray Optics and Optical Instruments P-395

53. A convergent doublet of separated lenses, corrected for f1 and f2 are close to :
spherical aberration, has resultant focal length of 10cm. (a) f1 = 7.8 cm f2 = 12.7 cm
The separation between the two lenses is 2cm. The focal
(b) f1 = 12.7 cm f2 = 7.8 cm
lengths of the component lenses [Online April 15, 2018]
(a) 18cm, 20cm (b) 10cm, 12cm (c) f1 = 15.6 cm f2 = 25.4 cm
(c) 12cm, 14cm (d) 16cm, 18cm (d) f1 = 7.8 cm f2 = 25.4 cm
54. In an experiment a convex lens of focal length 15 cm is 58. A thin convex lens of focal length ‘f’ is put on a plane
placed coaxially on an optical bench in front of a convex mirror as shown in the figure. When an object is kept at
mirror at a distance of 5 cm from it. It is found that an a distance ‘a’ from the lens - mirror combination, its
object and its image coincide, if the object is placed at a a
distance of 20 cm from the lens. The focal length of the image is formed at a distance in front of the
3
convex mirror is : [Online April 9, 2017] combination. The value of ‘a’ is :
(a) 27.5 cm (b) 20.0 cm (c) 25.0 cm (d) 30.5 cm
[Online April 11, 2015]
55. A hemispherical glass body of radius 10 cm and refractive
index 1.5 is silvered on its curved surface. A small air bubble
is 6 cm below the flat surface inside it along the axis. The
position of the iamge of the air bubble made by the mirror
is seen : [Online April 10, 2016]
10 cm

6cm
O
Silvered
3
(a) 14 cm below flat surface (a) 3f (b) f (c) f (d) 2f
2
(b) 20 cm below flat surface
(c) 16 cm below flat surface
æ 3ö
(d) 30 cm below flat surface 59. A thin convex lens made from crown glass ç m = ÷ has
56. A convex lens, of focal length 30 cm, a concave lens of è 2ø
focal length 120 cm, and a plane mirror are arranged as focal length f. When it is measured in two different
shown. For an object kept at a distance of 60 cm from the 4 5
convex lens, the final image, formed by the combination, is liquids having refractive indices and , it has the focal
3 3
a real image, at a distance of : [Online April 9, 2016]
lengths f1 and f2 respectively. The correct relation between
the focal lengths is: [2014]
(a) f1 = f2 < f
(b) f1 > f and f2 becomes negative
(c) f2 > f and f1 becomes negative
|Focal length| |Focal length|
= 30 cm = 120 cm (d) f1 and f2 both become negative
60cm 20cm
60. The refractive index of the material of a concave lens is m.
It is immersed in a medium of refractive index m1. A parallel
70 cm beam of light is incident on the lens. The path of the
(a) 60 cm from the convex lens emergent rays when m1 > m is:
(b) 60 cm from the concave lens [Online April 12, 2014]
(c) 70 cm from the convex lens m
(d) 70 cm from the concave lens
57. To find the focal length of a convex mirror, a student records
m1
the following data : [Online April 9, 2016] m1
Object Pin Convex Lens Convex Mirror Image Pin
(a)
22.2cm 32.2cm 45.8cm 71.2cm
The focal length of the convex lens is f1 and that of mirror
is f2. Then taking index correction to be negligibly small,
P-396 m Physics

(c) remain same


m1 (d) does not depend on colour of light
m1
66. In an optics experiment, with the position of the object
fixed, a student varies the position of a convex lens and
(b) for each position, the screen is adjusted to get a clear
image of the object. A graph between the object distance u
and the image distance v, from the lens, is plotted using
m the same scale for the two axes. A straight line passing
through the origin and making an angle of 45° with the x-
m1 m1 axis meets the experimental curve at P. The coordinates of
P will be [2009]
æ f fö
(a) ç , ÷ (b) ( f, f )
(c) è 2 2ø
(c) ( 4 f, 4 f ) (d) ( 2 f, 2 f )
67. A student measures the focal length of a convex lens by
putting an object pin at a distance ‘u’ from the lens and
m measuring the distance ‘v’ of the image pin. The graph
between ‘u’ and ‘v’ plotted by the student should look
m1 like [2008]
m1 v(cm)
v(cm)

(d)
(a) (b)
O u(cm) O u(cm)

61. An object is located in a fixed position in front of a screen. v(cm)


Sharp image is obtained on the screen for two positions of v(cm)
a thin lens separated by 10 cm. The size of the images in
two situations are in the ratio 3 : 3. What is the distance
(c) (d)
between the screen and the object? O u(cm) O u(cm)
[Online April 11, 2014]
(a) 124.5 cm (b) 144.5 cm 68. Two lenses of power –15 D and +5 D are in contact with
(c) 65.0 cm (d) 99.0 cm each other. The focal length of the combination is [2007]
62. Diameter of a plano-convex lens is 6 cm and thickness at (a) + 10 cm (b) – 20 cm
the centre is 3 mm. If speed of light in material of lens is (c) – 10 cm (d) + 20 cm
2× 108 m/s, the focal length of the lens is [2013] 69. A thin glass (refractive index 1.5) lens has optical power of
(a) 15 cm (b) 20 cm (c) 30 cm (d) 10 cm – 5 D in air. Its optical power in a liquid medium with
63. The image of an illuminated square is obtained on a screen refractive index 1.6 will be [2005]
with the help of a converging lens. The distance of the (a) – 1D (b) 1 D (c) – 25 D (d) 25 D
square from the lens is 40 cm. The area of the image is 9 70. A plano convex lens of refractive index 1.5 and radius of
times that of the square. The focal length of the lens is : curvature 30 cm. Is silvered at the curved surface. Now this
[Online April 22, 2013] lens has been used to form the image of an object. At what
(a) 36 cm (b) 27 cm (c) 60 cm (d) 30 cm distance from this lens an object be placed in order to have
64. An object at 2.4 m in front of a lens forms a sharp image on a real image of size of the object [2004]
a film 12 cm behind the lens. A glass plate 1 cm thick, of (a) 60 cm (b) 30 cm (c) 20 cm (d) 80 cm
refractive index 1.50 is interposed between lens and film
with its plane faces parallel to film. At what distance TOPIC 4 Prism and Dispersion of Light
(from lens) should object shifted to be in sharp focus of
film? [2012]
71. The surface of a metal is illuminated alternately with
(a) 7.2 m (b) 2.4 m (c) 3.2 m (d) 5.6 m
photons of energies E1 = 4 eV and E2 = 2.5 eV respectively.
65. When monochromatic red light is used instead of blue
The ratio of maximum speeds of the photoelectrons emitted
light in a convex lens, its focal length will [2011 RS]
in the two cases is 2. The work function of the metal in (eV)
(a) increase
is _______________. [NA Sep. 05, 2020 (II)]
(b) decrease
Ray Optics and Optical Instruments P-397

72. The variation of refractive index of a crown glass thin prism 73. A monochromatic light is incident at a certain angle on an
with wavelength of the incident light is shown. Which of equilateral triangular prism and suffers minimum deviation.
the following graphs is the correct one, if Dm is the angle
If the refractive index of the material of the prism is 3,
of minimum deviation ? [11 Jan. 2019, I]
then the angle of incidence is : [11 Jan. 2019 II]
(a) 90° (b) 30°
1.535 (c) 60° (d) 45°
n2 74. A ray of light is incident at an angle of 60° on one face of
1.530 a prism of angle 30°. The emergent ray of light makes an
angle of 30° with incident ray. The angle made by the
1.525 emergent ray with second face of prism will be:
[Online April 16, 2018]
1.520
(a) 30° (b) 90° (c) 0° (d) 45°
75. In an experiment for determination of refractive index of
1.515
glass of a prism by i – d, plot it was found thata ray incident
at angle 35°, suffers a deviation of 40° and that it emerges
1.510 l (nm)
400 500 600 700 at angle 79°. In that case which of the following is closest
to the maximum possible value of the refractive index?
Dm [2016]
(a) 1.7 (b) 1.8 (c) 1.5 (d) 1.6
76. Monochromatic light is incident on a glass prism of angle
A. If the refractive index of the material of the prism is µ,
a ray, incident at an angle q, on the face AB would get
(a) transmitted through the face AC of the prism provided :
[2015]
l (nm)
400 500 600 700
Dm A

(b) B C
é æ æ 1 öù
l (nm) (a) q > cos -1 êµsin ç A + sin -1 ç ÷ ú
400 500 600 700
ëê è è µ ø ûú
Dm
é æ æ 1 öù
(b) q < cos-1 êµsin ç A + sin -1 ç ÷ ú
ëê è è µ ø ûú

-1 é æ -1 æ 1 ö ù
(c) q > sin êµsin ç A - sin ç ÷ ú
(c) ëê è è µ ø ûú

l (nm) -1 é æ -1 æ 1 ö ù
400 500 600 700 (d) q < sin êµsin ç A - sin ç ÷ ú
ëê è è µ ø ûú
Dm
77. The graph between angle of deviation (d) and angle of
incidence (i) for a triangular prism is represented by[2013]

d
(d) (a)

l (nm) o
400 500 600 700
i
P-398 Physics

80. Which of the following processes play a part in the


d formation of a rainbow? [Online May 7, 2012]
(i) Refraction (ii) Total internal reflection
(b)
(iii) Dispersion (iv) Interference
o (a) (i), (ii) and (iii) (b) (i) and (ii)
i (c) (i), (ii) and (iv) (d) (iii) and (iv)
81. The refractive index of a glass is 1.520 for red light and
d 1.525 for blue light. Let D1 and D2 be angles of minimum
deviation for red and blue light respectively in a prism of
(c)
this glass. Then, [2006]
o (a) D1 < D2
i (b) D1 = D2
(c) D1 can be less than or greater than D2 depending
d upon the angle of prism
(d) D1 > D2
(d)
82. A light ray is incident perpendicularly to one face of a 90°
o prism and is totally internally reflected at the glass-air
i interface. If the angle of reflection is 45°, we conclude that
78. A beam of light consisting of red, green and blue colours the refractive index n [2004]
is incident on a right-angled prism on face AB. The
refractive indices of the material for the above red, green
and blue colours are 1.39, 1.44 and 1.47 respectively. A
person looking on surface AC of the prism will see 45°
[Online May 26, 2012] 45°
A
45°

1
(a) n> (b) n> 2
2
45° 1
(c) n< (d) n< 2
C 2
B
(a) no light
(b) green and blue colours TOPIC 5 Optical Instruments
(c) red and green colours
(d) red colour only 83. A compound microscope consists of an objective lens of
79. A glass prism of refractive index 1.5 is immersed in water focal length 1 cm and an eye piece of focal length 5 cm with
4 a separation of 10 cm.
(refractive index ) as shown in figure. A light beam
3 The distance between an object and the objective lens, at
incident normally on the face AB is totally reflected to n
reach the face BC, if [Online May 19, 2012] which the strain on the eye is minimum is cm.
40
B A
The value of n is ______. [NA Sep. 05, 2020 (I)]
q
84. In a compound microscope, the magnified virtual image is
formed at a distance of 25 cm from the eye-piece. The focal
length of its objective lens is 1 cm. If the magnification is
100 and the tube length of the microscope is 20 cm, then the
C focal length of the eye-piece lens (in cm) is __________.
[NA Sep. 04, 2020 (I)]
85. The magnifying power of a telescope with tube length 60
5 2 cm is 5. What is the focal length of its eye piece?
(a) sin q > (b) sin q >
9 3 [8 Jan. 2020 I]
8 1 (a) 20 cm (b) 40 cm
(c) sin q > (d) sin q > (c) 30 cm (d) 10 cm
9 3
Ray Optics and Optical Instruments P-399

86. If we need a magnification of 375 from a compound (c) Statement-1 and statement-2 are true and Statement-
microscope of tube length 150 mm and an objective of 2 is correct explanation for statement-1.
focal length 5 mm, the focal length of the eye-piece, should (d) Statements-1 and statement-2 are true and Statement-
be close to: [7 Jan. 2020 I] 2 is not the correct explanation for statement-1.
(a) 22mm (b) 12mm 93. The focal length of the objective and the eyepiece of a
(c) 2 mm (d) 33mm telescope are 50 cm and 5 cm respectively. If the telescope
87. An observer looks at a distant tree of height 10 m with a is focussed for distinct vision on a scale distant 2 m from
telescope of magnifying power of 20. To the observer the its objective, then its magnifying power will be:
tree appears : [2016] [Online April 22, 2013]
(a) 20 times taller (b) 20 times nearer (a) – 4 (b) – 8 (c) + 8 (d) – 2
(c) 10 times taller (d) 10 times nearer –2
94. A telescope of aperture 3 × 10 m diameter is focused on
88. To determine refractive index of glass slab using a
a window at 80 m distance fitted with a wire mesh of spacing
travelling microscope, minimum number of readings
2 × 10–3 m. Given: l = 5.5 × 10–7 m, which of the following
required are : [Online April 10, 2016]
is true for observing the mesh through the telescope?
(a) Two (b) Four (c) Three (d) Five
[Online May 26, 2012]
89. A telescope has an objective lens of focal length 150 cm
(a) Yes, it is possible with the same aperture size.
and an eyepiece of focal length 5 cm. If a 50 m tall tower at
a distance of 1 km is observed through this telescope in (b) Possible also with an aperture half the present
normal setting, the angle formed by the image of the tower diameter.
is q, then q is close to : [Online April 10, 2015] (c) No, it is not possible.
(a) 30° (b) 15° (c) 60° (d) 1° (d) Given data is not sufficient.
90. In a compound microscope, the focal length of objective 95. We wish to make a microscope with the help of two positive
lens is 1.2 cm and focal length of eye piece is 3.0 cm. When lenses both with a focal length of 20 mm each and the
object is kept at 1.25 cm in front of objective, final image is object is positioned 25 mm from the objective lens. How
formed at infinity. Magnifying power of the compound far apart the lenses should be so that the final image is
microscope should be: [Online April 11, 2014] formed at infinity? [Online May 12, 2012]
(a) 200 (b) 100 (a) 20mm (b) 100 mm
(c) 400 (d) 150 (c) 120 mm (d) 80mm
91. The focal lengths of objective lens and eye lens of a 96. An experment is performed to find the refractive index of
Galilean telescope are respectively 30 cm and 3.0 cm. glass using a travelling microscope. In this experiment
telescope produces virtual, erect image of an object distances are measured by [2008]
situated far away from it at least distance of distinct vision (a) a vernier scale provided on the microscope
from the eye lens. In this condition, the magnifying power
(b) a standard laboratory scale
of the Galilean telescope should be:
(c) a meter scale provided on the microscope
[Online April 9, 2014]
(d) a screw gauge provided on the microscope
(a) + 11.2 (b) – 11.2 (c) – 8.8 (d) + 8.8
92. This question has Statement-1 and Statement-2. Of the 97. The image formed by an objective of a compound
four choices given after the Statements, choose the one microscope is [2003]
that best describes the two Statements. (a) virtual and diminished
Statement 1: Very large size telescopes are reflecting (b) real and diminished
telescopes instead of refracting telescopes. (c) real and enlarged
Statement 2: It is easier to provide mechanical support to (d) virtual and enlarged
large size mirrors than large size lenses. 98. An astronomical telescope has a large aperture to [2002]
[Online April 23, 2013] (a) reduce spherical aberration
(b) have high resolution
(a) Statement-1 is true and Statement-2 is false.
(c) increase span of observation
(b) Statement-1 is false and Statement-2 is true.
(d) have low dispersion
P-400 Physics

1. (1) Let angle between the two mirrors be q.


Distance of object, u = – 30 cm Ray PQ P mirror M1 and Rs P mirror M2
Distance of image, v = 10 cm \ M1Rs = ÐORQ = Ð M1OM2= q
-v (-10) 1 Similarly, ÐM2QP = ÐOQR = Ð M2OM1= q
Magnification, m = = =
u -30 3 \ In DORQ, 3q = 180° Þ q =
180°
= 60°
3
1 6. (c) Using mirror formula, magnification is given by
Speed of image = m2 × speed of object = ´ 9 = 1 cm s–1
9
2. (d) Object is placed beyond radius of curvature (R) of f –1
m= =
concave mirror hence image formed is real, inverted and u – f 1– u
diminished or unmagnified. f
At focus magnification is ¥
And at u = 2f, magnification is 1.
h0 Hence graph (d) correctly depicts ‘m’ versus distance of
O C hi F P object ‘x’ graph.
Image hi < h0

v
3. (c) +5 = -
Þ v = -5u
u
1 1 1
Using + = 7. (c) When object is at 8 cm
v u f
1 1 1 f ´ u 5´8 40
Image V1 = = =- cm
or + = u -f 8-5 3
-5u u 0.4
\ u = 0.32 m When object is at 12 cm
f ´ u 5 ´ 12 60
Image V2 = = =- cm
u - f 12 - 5 7
3d
40 60 100
2 Separation = V1 - V2 = - = cm
3 7 21
L S So A, C and D are correct statements.
4. (c) L
8. (c) Convex morror is used as a shaving mirror.

L 3d
2
O

3d 3d 10 cm 15 cm
Total distance = + = 3d
2 2
5. (b)
M1
From question : v = 15 cm, u = – 10 cm
P Radius of curvature, R = 2f = ?
R q
1 1 1
q Using mirror formula, + =
v u f
1 1 1
+ = Þ f = – 30 cm
15 (-10) f
q q q Therefore radius of curvature,
O M2
Q R = 2f = – 60 cm
Ray Optics and Optical Instruments P-401

9. (a) From mirror formula 13. (90.00)


1 1 1 In the figure, QR is the reflected ray and QS is refracted
+ =
v u f ray. CQ is normal.
Differentiating the above equation, we get
dv v 2 æ du ö Q
=- 2ç ÷
dt u è dt ø e
Also, q
r' S
v f r' R
=
u u– f reflected ray
P r
2
dv æ f ö du
Þ = -ç 60° C
dt è u - f ø÷ dt
2
dv æ 0.2 ö
Þ =ç ÷ ´ 15 m= 3
dt è 2.8 - 0.2 ø
dv 1
Þ = m/s
dt 15
10. (b) The number of images formed is given by Apply Snell's law at P
360 1sin 60° = 3 sin r
n= -1
q
1
360 Þ sin r = Þ r = 30°
Þ -1 = 3 2
q
360° From geometry, CP = CQ
Þ q= = 90° \ r ' = 30°
4
11. (a) When two plane mirrors are inclined at each other at Again apply snell's law at Q,
an angle q then the number of the images (n) of a point
object kept between the plane mirrors is 3 sin r ' = 1sin e
360°
n= - 1, 3
q Þ = sin e Þ e = 60°
2
360°
(if is even integer) From geometry
q
360° r '+ q + e = 180° (As angles lies on a straight line)
\Number of images formed = -1 = 5
60º Þ 30° + q + 60° = 180° Þ q = 90°.
12. (158) 14. (d) Apparent depth,
15
From figure, sin i = and sin r = sin 45° h m1 = 2
15 + 30 2
2

From Snell's law, m ´ sin i = 1 ´ sin r m2 = 2 2 h


15 1
Þm´ = 1 ´ sin 45° = t1 t2 h h 3h 3h 2
2
15 + 30 2 2 D= + = + = =
m1 m2 2 2 2 2 2 4
15. (c) Given,
4
r 15 cm Refractive index, m =
3
45 cm 4
45° 15 cm sin q = 1sin 90°
30 cm i
P
3
15 cm 3
Þ sinq =
4
30 cm
1 7
cos q =
\m = 2 = 158 ´ 10 -2 = N 4
15 100
Solid angle, W = 2p(1– cosq) = 2p(1– 7 / 4)
1125
Hence, value of N ; 158 . Fraction of energy transmitted
P-402 Physics

19. (d) From the given figure


2p(1– cos q) 1 – 7 / 4
= = = 0.17 As sin 60o = m sin 30o
4p 2
Percentage of light emerges out of surface
= 0.17 × 100 = 17%
16. (b) Here, from question, relative permittivity
e
er = = 3 Þ e = 3 e0
e0

m 4 4
Relative permeability m r = m = 3 Þ m = 3 m0
0

\ me = 4m0 e0 sin 60o


Þm= = 3
sin 30o
m 0 e0 v 1 æ 1 ö
= = çQ c = ÷ a
me c 2 çè m 0 e0 ÷ = cos 60o Þ AO = 2a
ø AO
4 b 2b
n = mr er = ´3 = 2 = cos30o Þ BO =
3 BO 3
1 Optical path length = AO + mBO
And n = sin q
c 2b
= 2a + ( 3) = 2a + 2b
1 1 3
Þ sin qc = =
n 2 20. (d) Using Snell’s law of refraction,
\ Critical angle, qc = 30° 1 × sin 40° = 1.31 sin q
17. (c) If v is the distance of image formed by mirror, then 0.64
Þ sin q = = 0.49 » 0.5
1.31
1 1 1
+ = Þ q = 30°
v u f

1 1 1
or + =
v -5 -20
20 x = 20 µm × cot q
\ v= cm
3
2
Distance of this image from water surface \ Number of reflections = -6
20 ´10 ´ cot q
20 35
= + 5 = cm 2 ´ 10 6
3 3 = = 57735 » 57000
20 ´ 3
RD 21. (c) As 4% of light gets reflected, so only (100 – 4 = 96%)
Using, =µ
AD of light comes after refraction so,
RD (35 / 3) 96
\ AD = d = = = 8.8 cm Prefracted = PI
m 1.33 100
96
Þ K 2 A t2 = K1Ai2
µ22 100
18. (c) Using, sin qmax = µ1 µ22 - µ12 = -1
µ12 96
Þ r2 A t2 = r1Ai2
æ 2 ö 100
-1 ç µ2 ÷
or qmax = sin - 1
ç µ12 ÷ 96 1
è ø Þ A 2t = ´ ´ (30) 2
100 3
æ 2 ö 2
-1 ç µ2 ÷
For T1 R, q < sin - 1 64
ç µ12 ÷ At ´ (30)2 = 24
è ø 100
Ray Optics and Optical Instruments P-403

22. (d) Incident ray


denser A A
(µD) 90° 45°
Air
rarer r A
(µR) i1
90° m
i2
m R sin i
From Snell's law, = ..... (i) B
m D sin r
Q Ði = A and Ðr = (90° - A)
m From figure, i1 = 90° - r
We also know that, sin qC = R
mD
sin A 1
\ (sin 90° - r) =
n
From eq (i), sin qC =
sin(90° - A) m
sin A 1
sin q C = Þ cos r = … (ii)
cos A m
sin qC = tan A 1
Now cos r = 1 - sin 2 r = 1 -
or A = tan–1 (sin qC) 2m 2
4
23. (d) Given, µ = R
3 2m 2 - 1
h = 15 cm = … (iii)
R= ? 2m2
C
From eqs (ii) and (iii)
sin 90° h
=m
sin C 1 2m 2 - 1
1 R 3 =
Þ sin C = = = m 2m 2
m 2
R +h 2 4
2 2 2 Squaring both sides and then solving, we get
Þ 16 R = 9 R + 9 h O
or, 7R2 = 9 h2 3
m=
3 3 2
or, R= h= ´ 15cm
7 7 26. (b) Velocity of light in medium
24. (c) Real depth = 5 cm + 1cm = 6 cm
3cm 3 ´ 10-2 m
Vmed = = = 1.5 m/s
Water 0.2ns 0.2 ´ 10-9 s
m = 1.33 Refractive index of the medium
5cm
Vair 3 ´108
m= = = 2 m/s
Vmed 1.5
m = 1.5 1cm
Glass 1
d1 d2 As µ =
+ + .... sin C
Apparent depth =
m1 m2
1 1
5 1 \ sin C = = = 30°
= + m 2
1.33 1.5
Condition of TIR is angle of incidence i must be greater
; 3.8 + 0.7 ; 4.5 cm
than critical angle. Hence ray will suffer TIR in case of (B)
\ Shift = 6 cm – 4.5 cm @ 1.5 cm
(i = 40° > 30°) only.
25. (d) At point A by Snell’s law 27. (a) As refractive index for z > 0 and z £ 0 is different xy
sin 45° 1 plane should be the boundry between two media.
m= Þ sin r = … (i)
sin r m 2 Angle of incidence is given by
At point B, for total internal reflection,

sin i1 =
1
cos (p–i) =
(6 3iˆ + 8 3 ˆj - 10 kˆ .kˆ )
m 20
P-404 Physics

1 Applying Snell’s Law for air and medium inside the cylinder
– cos i = – at P we get
2
Þ Ð i = 60 ° sin q
n=
From Snell's law, sin a
sin i u 2
= Þ sin q = n ´ sin a = n2 - 1 ; [from (i)]
sin r u1 2
æ 2 ö 4 1
\ sin q = ç ÷ - 1 = -1 =
sin i 3 è 3ø 3 3
Þ =
sin r 2 æ 1 ö
or q = sin -1 ç ÷
Þ 2 sin i = 3sin r è 3ø
Þ 30. (a) From the figure it is clear that
2 sin 60° = 3
AB
tan q c =
3 OA
Þ 2´ = 3 sin r
2 Þ R = OA tan qc
ÞÐ r = 45° OA sin qc
Þ R=
28. (b) cos qc
OA sin qc
Þ R=
m2 Kerosene h2
1 - sin 2 qc

m1 h1 R sin qc
Water Þ tan qc = =
12 1 - sin 2 q c
Apparent shift of the bottom due to water, 1 3
Q sin qc = =
é 1ù m 4
Dh1 = h1 ê1 - ú
ë m1 û Þ tan qc =
3 3 R
= =
Apparent shift of the bottom due to kerosene, Dh2 16 - 9 7 12
é 1 ù
= h2 ê1 - ú
ë m2 û A
R
B
Thus, total apparent shift :
= Dh1 + Dh2
qc qc
12 cm
æ 1ö æ 1 ö
= h1 ç1 - ÷ + h2 ç1 - ÷
è m1ø è m 2ø
O
Q
n
90 – a
a 36
29. (c) Þ R= cm
q P 7
31. (b) Optical fibres form a dielectric wave guide and are free
from electromagnetic interference or radio frequency
Applying Snell’s law interference. There is extremely low transmission loss in
for medium inside the cylinder and air at Q we get optical fibre.
32. (a) In an optical fibre, light is sent through the fibre
sin 90° 1 without any loss by the phenomenon of total internal
n= =
sin(90º -a ) cos a reflection. Total internal reflection of light waves confine
the light rays inside the optical fiber.
1
\ cos a = 33. (d) Focal length of the convex lens, f = 0.5 m
n
Object is at 2f so, image (I1) will also be at 2f.
1 n2 - 1 Image of I1 i.e., I2 will be 1 m behind mirror.
\ sin a = 1 - cos 2 a = 1 - = ...(i)
n2 n Now I2 will be object for lens.
Ray Optics and Optical Instruments P-405

1 100 N
Power, P = = =
1m 1m 1m 1m
Object I1 I2 f 21 100
f = 0.5m Mirror \ N = 476.19.
\ u = (-1) + ( -1) + ( -1) = -3 m 37. (60) Given : m = 1.5; Rcurved = 30 cm
Using, Lens-maker formula
1 1 1
Using lens formula,
- = 1 æ 1 1 ö
v u f = (m –1) ç – ÷
f è R1 R2 ø
1 1 1 1 1 3
= + = + or v = = 0.6 m For plano-convex lens
v f u +0.5 -3 5
Hence, distance of image from mirror R1 ® ¥ then R2 = – R
= 2 + 0.6 = 2.6 m and real. R 30
34. (d) Given, using lens maker's formula \f = = = 60 cm
m –1 1.5 –1
1 æ 1 1ö 38. (b) Using lens maker’s formula
= (k - 1) ç - ÷
f è R1 R2 ø
1 æ mg öé 1 1 ù
= çç –1÷÷ ê – ú
Here, R1 = R2 = R (For double convex lens) f è ma R
øë 1 R2û
1 æ1 1 ö Here, mg and ma are the refractive index of glass and air
\ = (m - 1) ç -
f è R - R ÷ø respectively
1 æ 1 1 ö
1 2 Þ = (1.5 –1) ç – ÷ ...(i)
Þ P = = (m - 1) ...(i) f è R1 R2 ø
f R
For plano convex lens, When immersed in liquid

R1 = R ', R2 = ¥ 1 æ mg öæ 1 1 ö
= çç –1÷÷ ç – ÷
Using lens maker's formula again, we have fl è m l ø è R1 R2 ø
[Here, ml = refractive index of liquid]
æ 1 1ö
1.5 P = ( m - 1) ç - ÷ ...(ii)
è R ' ¥ø 1 æ 1.5 öæ 1 1 ö
Þ =ç –1÷ ç – ÷ ...(ii)
fl è 1.42 ø è R1 R2 ø
3 m -1
Þ P= Dividing (i) by (ii)
2 R'
From (i) and (ii), fl (1.5 –1)1.42 1.42 142
Þ = = = »9
3 R' R f 0.08 0.16 16
= Þ R' = 39. (a) Focal length of plano-convex lens-
2 2R 3
35. (d) From lens formula, 1 æ1 1 ö m1 –1
= ( m1 –1) ç – ÷=
1 1 1 uf f1 è ¥ –R ø R
- = Þv=
v u f u+ f
R
Case-I : If v = u Þ f + u = f Þ u = 0 Þ f1 =
( m1 – 1)
Case-II : If u = ¥ then v = f.
Focal length of plano-concave lens -
Hence, correct u versus v graph, that satisfies this
condition is (a). 1 æ 1 1 ö m –1
= ( m 2 –1) ç – ÷= 2
36. (476.19) f2 è –R ¥ ø –R
Given,
–R
Distance between an object and screen, D = 100 cm Þ f2 =
Distance between the two position of lens, d = 40 cm ( m2 – 1)
Focal length of lens, For the combination of two lens-

D 2 - d 2 100 2 - 402 (100 + 40)(100 - 40) 1 1 1 m –1 m2 –1


f = = = = 21 cm = + = 1 –
4D 4(100) 4(100) f eq f1 f 2 R R
P-406 Physics

m1 – m2 when liquid is put between, then


=
R 1 2 2
= +
f2 fl f
R
Þ f eq =
m1 – m2 1 2 2
40. (d) From the equation of line or (27 / 2) = 18 + f
m = k1v + k2 (Q y = mx + c) or f = – 54 cm
v æ vö 1 1
Þ = k1v + k2 çQ m = ÷ Now - = (m1 - 1) ´
u è uø 54 R
1 k æ 1 ö
Þ = k1 + 2 (Dividing both sides by v) = (m1 - 1) ´ ç ÷
u v è -18 ø
k 1 1 4
Þ 2 - – k1 \m1 = +1 =
v u 3 3
1 1 1 40 ´ 20
Comparing with lens formula v - u = f , we get 43. (Bouns) v1 = ( 40 – 20 ) = 40 cm
1 u2 = 60 – 40 = 20 cm
k1 = and k2 = 1
–f
20 ´10
\v2 = = 20 cm
\ f=
1
= –
b ( 20 –10 )
slope of m - v graph c \ Image traces back to object itself as image formed by
lens is a centre of curvature of mirror.
v
41. (b) Using, M < 44. (c) For lens
u
v1
or ,2 < Þ v1 < ,2 x1
x1
1 1 1
We have v , u < f

1 1 1
or , <
,2 x1 x1 20 1 1 1
- =
x1 = 30 cm v u f
1 1 1
And 2 x , x < 20
1 1 1
or - =
2 2 v -30 20
or x2 = – 10 cm \ v = + 60 cm
x1 30 According to the condition, image formed by lens
So, < <3 should be the centre of curvature of the mirror, and so
x2 10
2f’ = 20 or f’ = 10 cm
1 2
42. (a) f = f 45. (d) By lens’s formula, - =
1 1 1
1 l V u f
Here 2f1 = 18 cm or f1 = 9 cm For first lens, [u1 = –20]

1 2 1 1 1 20
= - = Þ VI =
So, or fl = 18 cm V1 -20 5 3
9 fl
Image formed by first lense will behave as an object for
1 æ2ö second lens
Using, = (m - 1) ç ÷
fl èRø 20 14
so, u 2 = -2=
1 æ2ö 3 3
or = (1.5 - 1) ç ÷
18 èRø 1 1 1
- = Þ V2 = 70 cm
\ R = 18 cm V2 14 -5
3
Ray Optics and Optical Instruments P-407

46. (a) According to lens maker's formula, 49. (b) From lens maker’s formula,
1 æ 1 1 ö 1 æ 1 1 ö
= (mrel – 1) ç R – R ÷ = (µ - 1) ç - ÷
f è 1 2ø f R
è 1 R 2ø
Focal length of lens will change due to change in refrective
index mrel. So, image will be formed at new position. Hence 1 æ1 1 ö 1
= (µ1 - 1) ç - ÷=
image disappears f1 è ¥ -R ø 2f2
1 æ +1ö Similarly, for plano-concave lens
47. (b) = (µ2 - 1) ç ÷
f2 è Rø 1 æ 1 1ö
= ( µ2 - 1) ç - ÷
f2 è -R ¥ ø
1 ( -1)
= (µ1 - 1)
f1 R 1 1
Dividing f by f we get,
Now when combined the focal length is given by 1 2

1 1 1
= + (µ1 - 1) (µ2 - 1)
f f1 f2 =
R 2R

= (µ1 - 1)
( -1) + +1 or, 2m1 – µ2 = 1
(µ2 - 1) R
R m2 m1 m 2 – m1
50. (d) using, – =
1 v u R
= éµ 2 - 1 - µ1 + 1 ù
Rë û R = 7.8 mm

µ 2 - µ1
=
R
R
Þ f= m1 = 1 m2 = 1.34
µ2 - µ1
1.34 1 1.34 –1
48. (d) By lens formula Þ – = [Qu = ¥]
V ¥ 7.8
1 1 1
- = \ V = 30.7 mm = 3.07 cm ; 3.1 cm
v u f
1 1 10 51. (d)
- =
v (-20) 3

1 10 1
= -
v 3 20
1 197 60
= ; v=
v 60 197
Magnification of lens (m) is given by

æ 60 ö Using lens formula


ç ÷
æ v ö è 197 ø 1 1 1 1 1 1
m=ç ÷ = - = Þ - = Þ f = 5cm
è uø 20 v u f 10 -10 f
velocity of image wrt. to lens is given by æ 1ö
vI/L = m2vO/L Shift due to slab, = t çè1 - ÷ø in the direction of incident
m
direction of velocity of image is same as that of object
ray
vO/L = 5 m/s
æ 2ö
æ 60 ´ 1 ö
2 or, d = 1.5 ç1 - ÷ = 0.5
v I/L =ç (5) è 3ø
è 197 ´ 20 ÷ø
Now, u = – 9.5
= 1.16 × 10–3 m/s towards the lens 1 1 1
Again using lens formulas - =
v -9.5 5
P-408 Physics

1 1 2 9 The image I gets formed at 60 cm to the right of the lens


Þ = - = and it will be inverted.
v 5 19 95
The rays from the image (I) formed further falls on the
95 convex mirror forms another image. This image should
or, v = = 10.55cm formed in such a way that it coincide with object at the
9
Thus, screen is shifted by a distance same point due to reflection takes place by convex mirror.
d = 10.55 – 10 = 0.55 cm away from the lens. Distance between lens and mirror will be
d = image distance (v) – radius of curvature of convex
52. (b) Case-1 mirror.
+ 5 = 60 – 2f
2f = 60 – 5
1 æ m –1ö f = –28
=ç ÷ 55
f1 è R ø f= = 27.5 cm (convex mirror)
2
55. (b) Given, radius of hemispherical glass R = 10 cm
1 æ m –1 ö
P = 2P1 + P2 Þ = 2ç ÷ 10
28 è R ø \ Focal length f = = –5 cm
2
1
(Q Power, P = & fplane mirror = ¥) u = (10 – 6) = –4 cm.
f
By using mirror formula,
Case-2 1 1 1 1 1 1
+ + = Þ + = Þ v = 20 cm.
v u f v -4 -5
1 æ m –1 ö R m1 1
=ç f2 = – f = –10 cm Apparent height, ha = hr = 30 ´ = 20 cm below
÷ 2
f1 è R ø m2 1.5
1 æ m –1ö 2 flat surface.
P = 2P1 + P2 Þ = 2ç ÷+ 56. (a) Len's formula is given by
10 è 2 ø R
1 1 2 1 1 1
or, = + Þ 2 = 1 - 2 = 18 = -
10 28 R R 10 28 280 f v u
280 For convex lens,
or, R= cm
9 1 1 1 1 1
= + Þ =
30 v 60 60 v
1 æ m –1 ö 5
or, = 2ç ÷9 Þ m–1= Similarly for concave lens
28 è 280 ø 9
5 14 1 1 1 1 1
= - Þ =
\ m = 1+ = = 1.55 - 120 v 40 v 60
9 9
53. (a) For minimum spherical aberration separation, Virtual object 10 cm behind plane mirror.
d = f1 – f2 = 2 cm Hence real image 10 cm infront of mirror or, 60 cm from
Resultant focal length = F = 10 cm convex lens.
57. (a) Taking f2 = 12.07
1 1 1 d
Using = + - and solving, we get f1, f2 18 Using Mirror's formula
F f1 f 2 f1 f 2
1 1 1
cm and 20 cm respectively. = +
f v u
54. (a) Given, focal length of lens (f) = 15 cm
object is placed at a distance (u) = – 20 cm 1 1 1 1 1 1
Þ = + Þ - =
By lens formula, 12.7 25.4 u 12.7 25.4 u
f = 15 cm u = 25.4 = v'
1 1 1 Now using Len's formula
= -
f v u 1 1 1 1 1 1
= - Þ = +
1 1 1 1 1 f v u f1 25.4 + 13.6 10
= + = -
v f u 15 20 1 1 1 390
d = 5 cm Þ = + Þ f1 = = 7.96
1 4 -3 f1 39 10 49
=
v 60 The closest answers is (a) as option (c) and (d) are not
v = 60 cm possible.
Ray Optics and Optical Instruments P-409

58. (d) When object is kept at a distance ‘a’ from thin


1 æ 1 1 ö
covex lens = ( a m l - 1) ç - ÷
fa è R1 R 2 ø
O f m ( a ml - 1)
Þ =
I1 v fa ( m ml - 1)

a If m1 > m, then fm and fa have opposite signs and the


nature of lens changes i.e. a convex lens diverges the light
1 1 1 rays and concave lens converges the light rays. Thus given
By lens formula : v – u = f option (a) is correct.
61. (d) Given: Separation of lens for two of its position,
1 1 1 d = 10 cm
– = Ratio of size of the images in two positions
V (– a) f
I1 3
1 1 1 =
or, = – ...(i) I2 2
v f a
Distance of object from the screen, D = ?
Mirror forms image at equal distance from mirror
Applying formula,

I1 ( D + d )2
=
I 2 ( D - d )2
I1 v v I2
3 ( D + 10) 2
Þ =
Now, again from lens formula 2 ( D - 10) 2

3 D 2 + 100 + 20 D
I3 I2 Þ = 2
2 D + 100 - 20 D
a/3 Þ 3D2 + 300 – 60D = 2D2 + 200 + 40D
v Þ D2 – 100D + 100 = 0
On solving, we get D = 99 cm
3 1 1 Hence the distance between the screen and the object is 99 cm.
– =
a V f
Velocity of light in vacuum
62. (c) \ n =
3 1 1 1 Velocity of light in medium
– + = [From eqn. (i)]
a f a f 3
\ n=
Hence, a = 2f 2 R = 3cm
59. (b) By Lens maker's formula for convex lens 32 + (R – 3mm)2 = R2
Þ 32 + R2 – 2R(3mm) + (3mm)2 = R2 3mm
1 æ m öæ 2 ö Þ R » 15 cm
=ç - 1÷ ç ÷
f è mL øè R ø 1 æ 3 öæ 1 ö
= ç –1 ÷ç ÷ Þ f = 30 cm
4 f è 2 øè 15 ø
for, m L1 = , f1 = 4 R
63. (d) If side of object square = l
3
and side of image square = l¢
5
for m L 2 = , f 2 = -5 R l '2
3 From question, =9
Þ f2 = (–) ve l
60. (a) If a lens of refractive index m is immersed in a medium l'
of refractive index m1, then its focal length in medium is or =3
l
given by
i.e., magnification m = 3
1 æ 1 1 ö u = – 40 cm
= ( m ml- 1) ç - ÷
fm è R1 R 2 ø v = 3 × 40 = 120 cm
If fa is the focal length of lens in air, then f= ?
P-410 Physics

1 1 1 Experimental
From formula, - = 66. (d) |v|
v u f curve
1 1 1
- =
120 -40 f Straight
line
1 1 1 1+ 3
or, = + = \ f = 30 cm (2f, 2f)
f 120 40 120 P
64. (d) The focal length of the lens
45°
1 1 1
= - |u|
f v u
For the graph to intersect y = x line. The value of | v | and |
1 1 u | must be equal.
= +
12 240 From lens formula
20 + 1 21 1 1 1
= = - =
240 240 v u f
240 When u = -2 f , v = 2 f
f = cm
21 f
When glass plate is interposed between lens and film, so Also v =
f
shift produced will be 1+
u
æ 1ö As |u| increases, v decreases for |u| > f. The graph between
Shift = t ç 1 - ÷
è mø |v| and |u| is shown in the figure. A straight line passing
æ 1 ö 1 through the origin and making an angle of 45°with the
1ç 1 - = 1´
è 3 / 2 ÷ø 3 x-axis meets the experimental curve at P (2f, 2f ).
Now image should be form at 1 1 1
67. (c) From the lens formula = -
v' = 12 - 1 = 35 cm f v u
3 3 This graph suggest that when
Now the object distance u. u = – f, v = + µ
Using lens formula again
When u is at – µ , v = f.
1 1 1
< , v (cm)
f v' u
1 1 1
Þ < ,
u v' f
1 3 21 1 é 3 21 ù
Þ = - = -
u 35 240 5 êë 7 48 úû f
1 1 é 48 - 49 ù
Þ = ê
u 5 ë 7 ´ 16 úû –f u (cm)
Þ u = –7 ×16 × 5 = – 560 cm = – 5.6 m
65. (a) From the Cauchy When the object is moved further away from the lens, v
decreases but remains positive.
B C
Formula, µ = A + 2 + 1 68. (c) When two thin lenses are in contact coaxially, power
l l of combination is given by
1 P = P1 + P2
\µµ
l
= (– 15 + 5) D
As, lblue < lred
\ lblue > µred = – 10 D.
From lens maker's formula 1
Also, P =
1 æ1 1ö f
and = (m - 1) ç - ÷ 1 1
f è R1 R2 ø Þ f = = metre
P -10
Þ
1 1 Þ fR > fB.
> æ1 ö
fB fR \ f = - ç ´ 100÷ cm = -10 cm.
è 10 ø
Ray Optics and Optical Instruments P-411

69. (b) According to lens maker's formula in air 71. 2


From the Einstein's photoelectric equation
æ 1 1 ö
1
fa
( )
= a µg -1 ç - ÷
è R1 R2 ø
Energy of photon
= Kinetic energy of photoelectrons + Work function
Þ Kinetic energy = Energy of Photon – Work Function
1 æ 1.5 ö æ 1 1 ö Let f0 be the work function of metal and v1 and v2 be the
Þ =ç - 1÷ ç - ÷ .... (i)
fa è 1 ø è R1 R2 ø velocity of photoelectrons. Using Einstein's photoelectric
equation we have
Using lens maker's formula in liquid medium,
1 2
1 æ mg öæ 1 1 ö mv1 = 4 - f 0 ...(i)
= çç - 1÷÷ ç - ÷ 2
fm è mm ø è R1 R2 ø
1 2
mv2 = 2.5 - f0 ...(ii)
Þ 1 = æ 1.5 - 1ö æ 1 1 ö .... (ii) 2
ç ÷ ç - ÷
f m è 1.6 ø R
è 1 R2ø 1 2
mv1 4 - f0
Dividing (i) by (ii), Þ 2 =
1 2 2.5 - f0
æ ö mv2
2
f m ç 1.5 - 1 ÷
=ç ÷ =–8
f a ç 1.5 ÷ 4 - f0
ç - 1 ÷ Þ (2)2 = Þ 10 - 4f 0 = 4 - f0
è 1.6 ø 2.5 - f 0

1 f0 = 2eV
Pa = - 5 =
fa 72. (a) When angle of prism is small, then angle of deviation
1 is given by Dm = (µ – 1)A
Þ fa = -
5 So, if wavelength of incident light is increased, µ decreases
and hence Dm decreases.
1 8
Þ f m = -8 ´ f a = - 8 ´ - = 73. (c) For minimum deviation:
5 5
A
m 1.6 r1 = r2 = = 30o
Pm = = ´ 5 = 1D 2
fm 8 by Snell’s law m1sin i = m2sin r
70. (c) The focal length(F) of the final mirror is 1 3
1 × sin i = 3 ´ = Þ i = 60
1 2 1 2 2
= + 74. (c) Angle of prism, A = 30°, i = 60°,
F fl f m
angle of deviation, d = 30° 30°
30°
Using lens maker's formula
Using formula, d = i + e – A 60°
1 æ 1 1 ö
Here = ( µ - 1) ç - ÷ Þ e = d +A – i
fl è R1 R2 ø
= 30° + 30° – 60° = 0°
Here, R1 = ¥
\ Emergent ray will be perpendicular to the face
R2 = 30 cm
So it will make angle 90° with the force through which it
é1 1 ù 1 emerges.
= (1.5 - 1) ê - =
ë ¥ -30 úû 60 75. (c) We know that i + e – A = d
35° + 79° – A = 40° \ A = 74°
1 1 1 1
\ = 2´ + = + d
æA m ö æ 74 + d m ö
F 60 30 / 2 10 sin ç ÷ sin ç ÷
\ F = 10 cm è 2 ø= è 2 ø
But m =
sinA / 2 74
Real image will be equal to the size of the object if the sin
object distance 2
5 æ dm ö
u = 2F = 20 cm = sin ç 37° +
3 è 2 ÷ø
P-412 Physics

5 5 1 1
m max can be . That is m max is less than = 1.67 or m< Þm<
3 3 sin q sin 45°
But dm will be less than 40° so Þ m < 2 Þ m < 1.414
5 5 79. (c) For total internal reflection on face AC
m< sin 57° < sin 60° Þ m = 1.5
3 3 q > critical angle (C)
76. (c) When r2 = C, ÐN2RC = 90° and sinq ³ sinC
Where C = critical angle 1
sin q ³
wm g
1
As sin C = = sin r2 4
m
mw
sin q ³ Þ sin q ³ 3
mg 3
A
2
8
N1 \ sin q ³ .
9
q N2 80. (a) Rainbow is formed due to the dispersion of light
Q r1 suffering refraction and total internal reflection (TIR) in
r2 R
P the droplets present in the atmosphere.
81. (a) When angle of prism is small,
B C Angle of deviation, D = (m – 1) A
Applying snell's law at ‘R’ Since lb < lr
µ sin r2 = 1 sin90° ...(i) Þ mr < mb Þ D1 < D2
Applying snell's law at ‘Q’
82. (b) For total internal reflection
1 × sin q = µ sin r 1 ...(ii)
But r1 = A – r2 Incident angle (i) > critical angle (ic),
So, sin q = µ sin (A – r 2) \ sin i > sin ic
sin q = µ sin A cos r 2 – cos A ...(iii) [using (i)]
From (1) 1
Þ sin 45° > sin ic Þ sin ic =
n
2 1
cos r2 = 1 – sin r2 = 1– ...(iv) \ sin 45° >
1
µ2 n
By eq. (iii) and (iv) 1 1
Þ > Þn> 2
1 2 n
sin q = µsin A 1 - - cos A
µ2 83. (50)
on further solving we can show for ray not to transmitted Given : Length of compound microscope, L = 10 cm
through face AC Focal length of objective f0 = 1 cm and of eye-piece,
fe = 5 cm
é –1 æ 1 ö ù u0 = fe = 5 cm
q = sin–1 êm sin(A – sin ç µ ÷ ú
ë è øû Final image formed at infinity ( ¥ ), ve = ¥
So, for transmission through face AC v0 = 10 – 5 = 5
é –1 æ 1 ö ù 1 1 1
q > sin–1 êm sin(A – sin ç µ ÷ ú Using lens formula, - =
ë è øû v u f
77. (c) For the prism as the angle of incidence (i) increases, 1 1 1 1 1 1 5
the angle of deviation (d) first decreases goes to minimum - = Þ - = Þ u0 = - cm
v0 u0 f0 5 u0 1 4
value and then increases.
78. (d) For light to come out through face 'AC', total 5 N
or, =
internal reflection must not take place. 4 40
i.e., q < c Þ sin q < sin c
200
\N = = 50 cm.
1 4
Þ sin q <
m
Ray Optics and Optical Instruments P-413

84. (4.48) 86. (a) According question, M = 375


According to question, final image i.e., v2 = 25 cm, L = 150 mm, f0 = 5 mm and fe = ?
f0 = 1 cm, magnification, m = m1m2 = 100
L æ Dö
fe Using, magnification, M ; f ç 1 + f ÷
0è eø
O object
f0=1cm 25cm I1 image formed by 1st lens
O I1 I2 I2 image formed by 2nd lens 150 æ 250 ö
Þ 375 = ç1 + ÷
x v1 5 è f e ø (Q D = 25 cm = 250 mm)
Objective
Eye-piece 250
20 cm Þ 12.5 = 1 +
fe
Using lens formula,
250
1 1 1 x Þ fe = = 21.7 » 22 mm
For first lens or objective = - = Þ v1 = 11.5
v1 - x 1 x -1
87. (b) A telescope magnifies by making the object appearing
v1 1
closer.
Also magnification | m1 |= = 88. (c) Reading one Þ without slab
u1 x -1
Reading two Þ with slab
For 2nd lens or eye-piece, this is acting as object Reading three Þ with saw dust
æ x ö Minimum three readings are required to determine refractive
\ u2 = -(20 - v1 ) = - ç 20 - ÷ and v2 = – 25 cm index of glass slab using a travelling microscope.
è x -1 ø
89. (c) Magnifying power of telescope,
D 25 b (angle subtended by image at eye piece)
Angular magnification | m A |= = MP =
u2 | u 2 | a (angle subtended by object on objective)

Total magnification m = m1m A = 100 f o 150


Also, MP = = = 30
fe 5
æ ö
æ 1 öç 25 ÷ 50 1
ç ÷ç ÷ = 100 a= = rad
è x - 1 ø ç 20 - x ÷ 1000 20
ç ÷
è x -1 ø
1 3
\ b = q = MP × a = 30 ´ = = 1.5 rad
25 20 2
Þ = 100 Þ 1 = 80( x - 1) - 4 x
20( x - 1) - x 180°
or, b = 1.5 ´ ; 84°
p
81
Þ 76 x = 81 Þ x = 90. (a) Given : f0 = 1.2 cm; fe = 3.0 cm
76
u0 = 1.25 cm; M¥ = ?
æ 81 ö 1 1 1
ç From = -
Þ u2 = - ç 20 - 76 ÷ = -19 f 0 v0 u0
÷
çç 81 ÷ 5
-1÷
è 76 ø 1 1 1
Þ = -
1.2 v0 ( -1.25)
Again using lens formula for eye-piece
1 1 1 25 ´ 19 1 1 1
- = Þ fe = » 4.48cm Þ = -
-25 - 19 f e 106 v0 1.2 1.25
5 Þ v0 = 30 cm
85. (d) For telescope Magnification at infinity,
Tube length (L) = fo + fe = 60 v0 D
f M¥ = - ´
and magnification (m) = o = 5 Þ f 0 = 5 f e u0 f e
fe
\ fo = 50 cm and fe = 10 cm 30 25
= ´
Hence focal length of eye-piece, fe = 10 cm 1.25 3
P-414 Physics

(Q D = 25 cm least distance of distinct vision) = 200 1.22l


Hence the magnifying power of the compound microscope Limit of resolution, Dq =
d
is 200
91. (d) Given, Focal length of objective, f0 = 30 cm 1.22 ´ 5.5 ´ 10-7
= = 2.23 × 10–5 rad.
focal length of eye lens, fe = 3.0 cm 3 ´10-2
Magnifying power, M = ? At a distance of 80 m , the telescope is able to resolve
Magnifying power of the Galilean telescope, between two points which are separated by 2.23 × 10–5
f æ f ö × 80 m
M D = 0 ç1 - e ÷ = 1.78 × 10–3 m
fe è Dø

30 æ 3 ö Eye-Piece
= ç1 - ÷ [Q D = 25 cm]
3 è 25 ø Objective
22
= 10 ´ = 8.8 cm
25
92. (c) One side of mirror is opaque and another side is 95. (c) F0
reflecting this is not in case of lens hence, it is easier to O 20 mm I
provide mechanical support to large size mirrors than large
25 mm V0 fe
size lenses. Reflecting telescopes are based on the same
principle except that the formation of images takes place
by reflection instead of refraction.
93. (d) Given : f0 = 50 cm, f e = 5cm
To obtain final image at infinity, object which is the image
d = 25 cm, u0 = –200 cm formed by objective should be at focal distance of eye-
Magnification M = ? piece.
1 1 1 By lens formula (for objective)
As - =
v0 u 0 f 0 1 1 1
- =
1 1 1 1 1 4 -1 3 v0 u0 f0
Þ = + = - = =
v0 f 0 u 0 50 200 200 200
1 1 1
200 or, - =
or v0 = cm v0 -25 20
3
Now ve = d = –25cm 1 1 1 5- 4 1
Þ = - = mm
v0 20 25 = 100 100
1 1 1
From, - = \ v0 = 100 mm
ve u e fe Therefore the distance between the lenses
= v0 + fe = 100 mm + 20 mm = 120 mm
1 1 1
– = - 96. (a) To find the refractive index of glass using a travelling
u e f e ve microscope, a vernier scale is provided on the microscope
97. (c) A real, inverted and enlarged image of the object is
1 1 6
= + = formed by the objective lens of a compound microscope.
5 25 25
98. (b) The resolving power of a telescope is
-25 D
or, ve = cm R.P =
6 1.22 l
Magnification M = M0 × Me
where D = diameter of the objective lens
v v -200 / 3 -25 l = wavelength of light.
= 0´ e = ´
u0 ue 200 -25 / 6 D
Clearly, R.P µ
1 l
= - ´ 6 = -2 Resolving power of telescope resolution will be high if its
3
objective is of large aperture.
94. (a) Given : d = 3 × 10–2 m
l = 5.5 × 10–7 m
24
Wave Optics

3. Two light waves having the same wavelength l in vacuum


Wavefront, Interference of are in phase initially. Then the first wave travels a path L1
TOPIC 1 Light, Coherent and through a medium of refractive index n1 while the second
Incoherent Sources wave travels a path of length L2 through a medium of
refractive index n2. After this the phase difference between
1. In the figure below, P and Q are two equally intense the two waves is : [Sep. 03, 2020 (II)]
coherent sources emitting radiation of wavelength 20 m.
The separation between P and Q is 5 m and the phase of P 2 p æ L2 L1 ö 2p æ L1 L2 ö
(a) ç - ÷ (b) ç - ÷
is ahead of that of Q by 90º. A, B and C are three distinct l è n1 n2 ø l è n1 n2 ø
points of observation, each equidistant from the midpoint 2p 2p
of PQ. The intensities of radiation at A, B, C will be in the (c) ( n1 L1 - n2 L2 ) (d) (n2 L1 - n1 L2 )
l l
ratio : [Sep. 06, 2020 (I)]
4. In an interference experiment the ratio of amplitudes of
B
a1 1
coherent waves is a = 3 . The ratio of maximum and
2

minimum intensities of fringes will be : [8 April 2019 I]


P Q (a) 2 (b) 18 (c) 4 (d) 9
C A 5. Two coherent sources produce waves of different intensities
(a) 0 : 1 : 4 (b) 2 : 1 : 0 which interfere. After interference, the ratio of the maximum
(c) 0 : 1 : 2 (d) 4 : 1 : 0 intensity to the minimum intensity is 16. The intensity of the
2. Two coherent sources of sound, S1 and S2, produce sound waves are in the ratio: [9 Jan. 2019 I]
waves of the same wavelength, l = 1 m, in phase. S1 and S2 (a) 16 : 9 (b) 25 : 9 (c) 4 : 1 (d) 5 : 3
are placed 1.5 m apart (see fig.). A listener, located at L, 6. On a hot summer night, the refractive index of air is smallest
directly in front of S2 finds that the intensity is at a minimum near the ground and increases with height from the ground.
when he is 2 m away from S2. The listener moves away When a light beam is directed horizontally, the Huygens'
from S1, keeping his distance from S2 fixed. The adjacent principle leads us to conclude that as it travels, the light
maximum of intensity is observed when the listener is at a
beam : [2015]
distance d from S1. Then, d is : [Sep. 05, 2020 (II)]
(a) bends downwards
(b) bends upwards
2m (c) becomes narrower
S2 L (d) goes horizontally without any deflection
2m
7. Interference pattern is observed at ‘P’ due to
superimposition of two rays coming out from a source ‘S’
1.5 m d as shown in the figure. The value of ‘l’ for which maxima is
obtained at ‘P’ is:
(R is perfect reflecting surface) [Online April 12, 2014]
S1
(a) 12 m (b) 5 m (c) 2 m (d) 3 m
P-416 Physics

21 monochromatic light, this film gives an interference pattern


S P due to light reflected from the top (convex) surface and
the bottom (glass plate) surface of the film.
Statement – 1 : When light reflects from the air-glass plate
interface, the reflected wave suffers a phase change of p.
Statement – 2 : The centre of the interference pattern is
30°
R dark. [2011]
(a) Statement – 1 is true, Statement – 2 is true, Statement
2nl ( 2n - 1) l – 2 is the correct explanation of Statement – 1.
l=
(a) l =
3 -1
(b) 2 ( 3 -1 ) (b) Statement – 1 is true, Statement – 2 is true, Statement
– 2 is not the correct explanation of Statement – 1.
( 2n - 1) l 3 ( 2n - 1) l (c) Statement – 1 is false, Statement – 2 is true.
l=
(d) l =
(c)
(
4 2- 3 ) 3 -1
(d) Statement – 1 is true, Statement – 2 is false.
Directions : Questions number 13-15 are based on the following
8. Two monochromatic light beams of intensity 16 and 9 units paragraph.
are interfering. The ratio of intensities of bright and dark An initially parallel cylindrical beam travels in a medium of
parts of the resultant pattern is: [Online April 11, 2014] refractive index m (I) =m0 + m2 I, where m0 and m2 are positive
16 4 7 49 constants and I is the intensity of the light beam. The intensity
(a) (b) (c) (d) of the beam is decreasing with increasing radius
9 3 1 1
9. n identical waves each of intensity I0 interfere with each 13. As the beam enters the medium , it will [2010]
other. The ratio of maximum intensities if the interference (a) diverge
is (i) coherent and (ii) incoherent is : (b) converge
[Online April 23, 2013] (c) diverge near the axis and converge near the periphery
1 1 (d) travel as a cylindrical beam
(a) n2 (b) (c) (d) n
n n2 14. The initial shape of the wavefront of the beam is [2010]
10. A ray of light of intensity I is incident on a parallel glass (a) convex
slab at point A as shown in diagram. It undergoes partial (b) concave
reflection and refraction. At each reflection, 25% of incident (c) convex near the axis and concave near the periphery
energy is reflected. The rays AB and A'B' undergo (d) planar
interference. The ratio of Imax and Imin is : 15. The speed of light in the medium is [2010]
[Online April 9, 2013]
(a) minimum on the axis of the beam
B B' (b) the same everywhere in the beam
A' (c) directly proportional to the intensity I
A (d) maximum on the axis of the beam
16. To demonstrate the phenomenon of interference, we
require two sources which emit radiation [2003]
(a) of nearly the same frequency
C C' (b) of the same frequency
(c) of different wavelengths
(a) 49 : 1 (b) 7 : 1 (c) 4 : 1 (d) 8 : 1 (d) of the same frequency and having a definite phase
11. Two coherent plane light waves of equal amplitude makes relationship
a small angle a (< < 1) with each other. They fall almost
normally on a screen. If l is the wavelength of light waves,
Young's Double Slit
the fringe width Dx of interference patterns of the two sets TOPIC 2
of waves on the screen is [Online May 19, 2012] Experiment
2l l l l 17. A young’s double-slit experiment is performed using
(a) (b) (c) 2a (d)
a a ( ) a monocromatic light of wavelength l. The inntensity of
12. This question has a paragraph followed by two statements, light at a point on the screen, where the path difference is
Statement – 1 and Statement – 2. Of the given four
alternatives after the statements, choose the one that l, is K units. The intensity of light at a point where the
describes the statements. l nK
A thin air film is formed by putting the convex surface of a path difference is is given by , where n is an inte-
6 12
plane-convex lens over a plane glass plate. With ger. The value of n is ______. [NA Sep. 06, 2020 (II)]
Wave Optics P-417

18. In a Young’s double slit experiment, light of 500 nm is used 25. The figure shows a Young’s double slit experimental setup.
to produce an interference pattern. When the distance It is observed that when a thin transparent sheet of
between the slits is 0.05 mm, the angular width (in degree) thickness t and refractive index ¼ is put in front of one of
of the fringes formed on the distance screen is close to : the slits, the central maximum gets shifted by a distance
[Sep. 03, 2020 (I)] equal to n fringe widths. If the wavelength of light used is
(a) 0.17° (b) 0.57° », t will be: [9 April 2019 I]
(c) 1.7° (d) 0.07°
19. Interference fringes are observed on a screen by
illuminating two thin slits 1 mm apart with a light source (l
= 632.8 nm). The distance between the screen and the slits
is 100 cm. If a bright fringe is observed on a screen at a
distance of 1.27 mm from the central bright fringe, then the
path difference between the waves, which are reaching 2nDl nDl
(a) (b)
this point from the slits is close to : [Sep. 02, 2020 (I)] a (m _ 1) a (m _ 1)
(a) 1.27 mm (b) 2.87 nm Dl 2 Dl
(d) 2.05 mm (c) (d)
(c) 2 nm a ( m _ 1) a (m _ 1)
20. In a Young's double slit experiment, 16 fringes are observed
in a certain segment of the screen when light of wavelength 26. In a Young’s double slit experiment, the path difference, at
a certain point on the screen, betwen two interfering waves
700 nm is used. If the wavelength of light is changed to
400 nm, the number of fringes observed in the same 1
is th of wavelength. The ratio of the intensity at this
segment of the screen would be : [Sep. 02, 2020 (II)] 8
(a) 24 (b) 30 point to that at the centre of a bright fringe is close to:
(c) 18 (d) 28 [11 Jan 2019 I]
21. In a Young’s double slit experiment 15 fringes are observed (a) 0.74 (b) 0.85 (c) 0.94 (d) 0.80
on a small portion of the screen when light of wavelength 27. In a Young’s double slit experiment with slit separation
500 nm is used. Ten fringes are observed on the same
1
section of the screen when another light source of 0.1 mm, one observes a bright fringe at angle rad
wavelength l is used. Then the value of l is (in nm) 40
______. [NA 9 Jan 2020 II] by using light of wavelength l1 When the light of
22. In a double-slit experiment, at a certain point on the screen wavelength l2 is used a bright fringe is seen at the
the path difference between the two interfering waves is same angle in the same set up. Given that l1 and l2 are
in visible range (380 nm to 740 nm), their values are:
1 [10 Jan. 2019 I]
th of a wavelength. The ratio of the intensity of light at
8 (a) 625 nm, 500 nm (b) 380 nm, 525 nm
that point to that at the centre of a bright fringe is:
(c) 380 nm, 500 nm (d) 400 nm, 500 nm
[8 Jan 2020 II]
(a) 0.853 (b) 0.672 (c) 0.568 (d) 0.760 28. Consider a Young’s double slit experiment as shown in
23. In a Young’s double slit experiment, the separation figure. What should be the slit separation d in terms of
between the slits is 0.15 mm. In the experiment, a source wavelength l such that the first minima occurs directly
of light of wavelength 589 nm is used and the interference in front of the slit (S1)? [10 Jan 2019 II]
pattern is observed on a screen kept 1.5 m away. The
separation between the successive bright fringes on the
screen is: [7 Jan 2020 II] S1 P st
1 minima
(a) 6.9 mm (b) 3.9 mm (c) 5.9 mm (d) 4.9 mm
24. In a double slit experiment, when a thin film of thickness t d
having refractive index µ is introduced in front of one of Source
the slits, the maximum at the centre of the fringe pattern S2
shifts by one fringe width. The value of t is (» is the
2d Screen
wavelength of the light used) : [12 April 2020 I]
2l l l l
(a) (b)
(a)
(m - 1)
(b)
2(m - 1) 2 ( 5-2 ) ( 5-2 )
l l l l
(c) (d) (c) (d)
(m - 1) (2m - 1)
(
2 5- 2 ) (5 - 2 )
P-418 Physics

29. In a Young's double slit experiment, the slits are placed


0.320 mm apart. Light of wavelength l = 500 nm is
incident on the slits. The total number of bright fringes
that are observed in the angular range – 30° £ q £ 30° s1
is [9 Jan 2019 II] Central fringe
(a) 640 (b) 320 (c) 321 (d) 641
30. In a Young's double slit experiment, slits are separated by (Fringe width = b¢)
0.5 mm, and the screen is placed 150 cm away. A beam of d
light consisting of two wavelengths, 650 nm and 520 nm, s2
is used to obtain interference fringes on the screen. The 2D
least distance from the common central maximum to the
point where the bright fringes due to both the wavelengths
coincide is : [2017] (a) 520 nm (b) 540 nm (c) 560 nm (d) 580 nm
(a) 9.75 mm (b) 15.6 mm (c) 1.56 mm (d) 7.8 mm 34. Two coherent point sources S1 and S2 are separated by a
31. In a Young’s double slit experiment with light of small distance 'd' as shown. The fringes obtained on the
wavelength l the separation of slits is d and distance of screen will be [2013]
screen is D such that D >> d >> l. If the fringe width
is b, the distance from point of maximum intensity to the
point where intensity falls to half of maximum intensity
on either side is: [Online April 11, 2015] d
b b b b
(a) (b) (c) (d) S1 S2 Screen
6 3 4 2
32. In a Young’s double slit experiment, the distance between D
the two identical slits is 6.1 times larger than the slit width.
Then the number of intensity maxima observed within the
(a) points (b) straight lines
central maximum of the single slit diffraction pattern is:
[Online April 19, 2014] (c) semi-circles (d) concentric circles
(a) 3 (b) 6 (c) 12 (d) 24 35. The source that illuminates the double - slit in 'double - slit
33. Using monochromatic light of wavelength l, an interference experiment' emits two distinct monochromatic
experimentalist sets up the Young’s double slit experiment waves of wavelength 500 nm and 600 nm, each of them
in three ways as shown. producing its own pattern on the screen. At the central
If she observes that y = b’, the wavelength of light used point of the pattern when path difference is zero, maxima
is: [Online April 9, 2014] of both the patterns coincide and the resulting interference
pattern is most distinct at the region of zero path difference.
s1 But as one moves out of this central region, the two fringe
Central fringe
systems are gradually out of step such that maximum due
O to on wavelength coincides with the minimum due to the
other and the combined fringe system becomes completely
d (Fringe width b) indistinct. This may happen when path difference in nm
s2 is: [Online April 25, 2013]
D (Screen) (a) 2000 (b) 3000 (c) 1000 (d) 1500
2500
36. A thin glass plate of thickness is l (l is wavelength
3
Transparent
mica sheet of
of light used) and refractive index m = 1.5 is inserted
thickness between one of the slits and the screen in Young’s double
Central fringe slit experiment. At a point on the screen equidistant from
s1 t = 1.8 mm y (Fringe
the slits, the ratio of the intensities before and after the
m = 1.6 width b )
O introduction of the glass plate is :
[Online April 25, 2013]
d (a) 2 : 1 (b) 1 : 4 (c) 4 : 1 (d) 4 : 3
s2 37. This question has Statement-1 and Statement-2. Of the
four choices given after the Statements, choose the one
D (Screen) that best describes the two Statements.
Wave Optics P-419

Statement-1: In Young’s double slit experiment, the number 43. In a Young’s double slit experiment, the two slits act as
of fringes observed in the field of view is small with longer coherent sources of wave of equal amplitude A and
wavelength of light and is large with shorter wavelength wavelength l. In another experiment with the same
of light. arrangement the two slits are made to act as incoherent
Statement-2: In the double slit experiment the fringe width sources of waves of same amplitude and wavelength. If
depends directly on the wavelength of light. the intensity at the middle point of the screen in the first
[Online April 22, 2013] I1
case is I1 and in the second case is I2, then the ratio is
(a) Statement-1 is true, Statement-2 is true and the Statement- I2
2 is correct explanation of the Statement-1. [2011 RS]
(b) Statement-1 is false and the Statement-2 is true. (a) 2 (b) 1 (c) 0.5 (d) 4
(c) Statement-1 is true Statement-2 is true and the 44. A mixture of light, consisting of wavelength 590 nm and an
Statement-2 is not correct explanation of the
unknown wavelength, illuminates Young’s double slit and
Statement-1.
gives rise to two overlapping interference patterns on the
(d) Statement-1 is true and the Statement-2 is false.
screen. The central maximum of both lights coincide.
38. In Young's double slit experiment, one of the slit is wider Further, it is observed that the third bright fringe of known
than other, so that amplitude of the light from one slit is light coincides with the 4th bright fringe of the unknown
double of that other slit. If Im be the maximum intensity,
light. From this data, the wavelength of the unknown light
the resultant intensity I when they interfere at phase
difference f is given by : [2012] is: [2009]
(a) 885.0 nm (b) 442.5 nm
Im Im æ 2 fö
(a) (4 + 5 cos f) (b) ç 1 + 2cos ÷ (c) 776.8 nm (d) 393.4 nm
9 3 è 2ø
45. In a Young’s double slit experiment the intensity at a point
Im æ 2 fö Im æ 2 fö l
(c) ç 1 + 4 cos ÷ (d) ç 1 + 8 cos ÷ where the path difference is (l being the wavelength of
5 è 2ø 9 è 2ø 6
39. In Young’s double slit interference experiment, the slit I
light used) is I. If I0 denotes the maximum intensity, is
widths are in the ratio 1 : 25. Then the ratio of intensity at I0
the maxima and minima in the interference pattern is equal to [2007]
[Online May 26, 2012] 1 1
3
(a) 3 : 2 (b) 1 : 25 (c) 9 : 4 (d) 1 : 5 (a) 3 (b) (c) (d)
4 2 2 2
40. The maximum number of possible interference maxima for 46. A Young’s double slit experiment uses a monochromatic
slit separation equal to 1.8l, where l is the wavelength of
light used, in a Young’s double slit experiment is source. The shape of the interference fringes formed on a
screen is [2005]
[Online May 12, 2012]
(a) zero (b) 3 (c) infinite (d) 5 (a) circle (b) hyperbola
41. In a Young’s double slit experiment with light of wavelength (c) parabola (d) straight line
l, fringe pattern on the screen has fringe width b. When 47. The maximum number of possible interference maxima for
two thin transparent glass (refractive index m) plates of
slit-separation equal to twice the wavelength in Young’s
thickness t1 and t2 (t1 > t2) are placed in the path of the
two beams respectively, the fringe pattern will shift by a double-slit experiment is [2004]
distance [Online May 7, 2012] (a) three (b) five (c) infinite (d) zero
b ( m - 1) æ t1 ö mb t1
(a)
l
ç ÷ (b) l t TOPIC 3 Diffraction, Polarisation of
Light and Resolving Power
è t2 ø 2

b ( m - 1) l
(c) ( t1 - t2 ) (d) ( m - 1) ( t1 + t2 ) 48. A beam of plane polarised light of large cross-sectional
l b area and uniform intensity of 3.3 Wm–2 falls normally on a
42. At two points P and Q on screen in Young’s double slit polariser (cross sectional area 3 × 10–4 m2) which rotates
experiment, waves from slits S1 and S2 have a path about its axis with an angular speed of 31.4 rad/s. The
l energy of light passing through the polariser per
difference of 0 and , respectively. The ratio of intensities
4 revolution, is close to : [Sep. 04, 2020 (I)]
at P and Q will be : [2011 RS] –5
(a) 1.0 × 10 J (b) 1.0 × 10–4 J
(a) 2 : 1 (b) 2 :1 (c) 4 : 1 (d) 3 : 2 (c) 1.5 × 10–4 J (d) 5.0 × 10–4 J
P-420 Physics

49. Orange light of wavelength 6000 × 10–10 m illuminates a


single slit of width 0.6 × 10–4 m. The maximum possible
number of diffraction minima produced on both sides of
the central maximum is _____________.
[NA Sep. 04, 2020 (II)]
50. The aperture diameter of telescope is 5m. The separation
between the moon and the earth is 4 × 105 km. With light
of wavelength of 5500 Å, the minimum separation between 5 3 5 4
(a) (b) (c) (d)
objects on the surface of moon, so that they are just 3 5 3 3
resolved, is close to: [9 Jan. 2020 I] 58. The angular width of the central maximum in a single slit
(a) 60 m (b) 20 m (c) 200 m (d) 600 m diffraction pattern is 60°. The width of the slit is 1 mm. The
51. A polarizer - analyser set is adjusted such that the slit is illuminated by monochromatic plane waves. If
intensity of light coming out of the analyser is just 10% of another slit of same width is made near it, Young's fringes
the original intensity. Assuming that the polarizer - can be observed on a screen placed at a distance 50 cm
from the slits. If the observed fringe width is 1 cm, what is
analyser set does not absorb any light, the angle by which
slit separation distance?
the analyser need to be rotated further to reduce the
(i.e. distance between the centres of each slit.) [2018]
output intensity to be zero, is: [7 Jan. 2020 I] (a) 25 mm (b) 50 mm (c) 75 mm (d) 100 mm
(a) 71.6° (b) 18.4° (c) 90° (d) 45° 59. Unpolarized light of intensity I passes through an ideal
52. The value of numerical aperature of the objective lens of polarizer A. Another indentical polarizer B is placed behind
a microscope is 1.25. If light of wavelength 5000 Å is used, I
A. The intensity of light beyond B is found to be . Now
the minimum separation between two points, to be seen 2
as distinct, will be : [12 April 2019 I] another identical polarizer C is placed between A and B.
(a) 0.24 µm (b) 0.38 µm (c) 0.12 µm (d) 0.48 µm The intensity beyond B is now found to be I . The angle
53. A system of three polarizers P1, P2, P3 is set up such that the 8
between polarizer A and C is: [2018]
pass axis of P3 is crossed with respect to that of P1. The (a) 0° (b) 30° (c) 45° (d) 60°
pass axis of P2 is inclined at 60o to the pass axis of P3. When 60. Light of wavelength 550 nm falls normally on a slit of width
a beam of unpolarized light of intensity Io is incident on P1, 22.0 × 10–5cm. The angular position of the second minima
the intensity of light transmitted by the three polarizers is I. from the central maximum will be (in radians)
The ratio (Io/I) equals (nearly) : [12 April 2019 II] [Online April 15, 2018]
(a) 5.33 (b) 16.00 (c) 10.67 (d) 1.80 p p p p
54. Diameter of the objective lens of a telescope is 250 cm. (a) (b) (c) (d)
8 12 4 6
For light of wavelength 600 nm. Coming from a distant 61. Unpolarized light of intensity I is incident on a system of
object, the limit of resolution of the telescope is close to: two polarizers, A followed by B. The intensity of emergent
[9 April 2019 II] light is I/2. If a third polarizer C is placed between A and B,
the intensity of emergent light is reduced to I/3. The angle
(a) 1.5 ´ 10 -7 rad(b) -7 rad
2.0 ´ 10 between the polarizers A and C is q. Then
(c) 3.0 ´ 10 -7 rad (d) 4.5 ´ 10-7 rad [Online April 16, 2018]
55. Calculate the limit of resolution of a telescope objective 1/ 4 1/ 4
æ2ö æ1ö
having a diameter of 200 cm, if it has to detect light of (a) cos q = ç ÷ (b) cos q = ç ÷
è3ø è3ø
wavelength 500 nm coming from a star. [8 April 2019 II] 1/ 2 1/ 2
(a) 305 × 10–9 radian (b) 610 × 10–9 radian æ1ö æ2ö
(c) cos q = ç ÷ (d) cos q = ç ÷
(c) 152.5 × 10 radian
–9
(d) 457.5 × 10–9 radian è3ø è3ø
56. In a double-slit experiment, green light (5303Å) falls on a 62. A plane polarized light is incident on a polariser with its pass
double slit having a separation of 19.44 µm and a width of axis making angle q with x-axis, as shown in the figure. At four
4.05 µm. The number of bright fringes between the first different values of q, q = 8°, 38°, 188° and 218°, the observed
and the second diffraction minima is : [11 Jan 2019 II] intensities are same. What is the angle between the direction
(a) 10 (b) 05 (c) 04 (d) 09 of polarization and x-axis [Online April 15, 2018]
57. Consider a tank made of glass (refractive index 1.5) with a y
thick bottom. It is filled with a liquid of refractive index m.
A student finds that, irrespective of what the incident q
x
angle i (see figure) is for a beam of light entering the z
liquid, the light reflected from the liquid glass interface is
never completely polarized. For this to happen, the
minimum value of m is: [9 Jan. 2019 I] Pass axis
(a) 203° (b) 45° (c) 98° (d) 128°
Wave Optics P-421

63. An observer is moving with half the speed of light towards 69. Assuming human pupil to have a radius of 0.25 cm and a
a stationary microwave source emitting waves at frequency comfortable viewing distance of 25 cm, the minimum
10 GHz. What is the frequency of the microwave measured separation between two objects that human eye can resolve
by the observer? (speed of light = 3 × 108 ms–1) [2017] at 500 nm wavelength is : [2015]
(a) 17.3 GHz (b) 15.3 GHz (a) 100 µm (b) 300 µm (c) 1 µm (d) 30 µm
(c) 10.1 GHz (d) 12.1 GHz 70. Unpolarized light of intensity I0 is incident on surface of
64. A single slit of width 0.1 mm is illuminated by a parallel a block of glass at Brewster’s angle. In that case, which
beam of light of wavelength 6000 Å and diffraction one of the following statements is true ?
bands are observed on a screen 0.5 m from the slit. The [Online April 11, 2015]
distance of the third dark band from the central bright (a) reflected light is completely polarized with intensity
band is : [Online April 9, 2017] I0
(a) 3 mm (b) 9mm (c) 4.5 mm (d) 1.5 mm less than
2
65. A single slit of width b is illuminated by a coherent
(b) transmitted light is completely polarized with intensity
monochromatic light of wavelength l. If the second and
fourth minima in the diffraction pattern at a distance 1 m I0
less than
from the slit are at 3 cm and 6 cm respectively from the 2
central maximum, what is the width of the central maximum?
(c) transmitted light is partially polarized with intensity
(i.e. distance between first minimum on either side of the
central maximum) [Online April 8, 2017] I0
(a) 1.5 cm (b) 3.0 cm (c) 4.5 cm (d) 6.0 cm 2
66. The box of a pin hole camera, of length L, has a hole of
(d) reflected light is partially polarized with intensity
radius a. It is assumed that when the hole is illuminated by
a parallel beam of light of wavelength l the spread of the I0
spot (obtained on the opposite wall of the camera) is the 2
sum of its geometrical spread and the spread due to 71. Two beams, A and B, of plane polarized light with mutually
diffraction. The spot would then have its minimum size perpendicular planes of polarization are seen through a
(say bmin) when : [2016] polaroid. From the position when the beam A has maximum
(a) a = lL and b min = 4lL intensity (and beam B has zero intensity), a rotation of
polaroid through 30° makes the two beams appear equally
l2 bright. If the initial intensities of the two beams are IA and
(b) a = and b min = 4lL
L
æ 2l2 ö IA
l2 IB respectively, then equals: [2014]
(c) a = and bmin = ç ÷ IB
L ç L ÷
è ø
æ 2l 2 ö 3 1
(d) a = l l and b min = çç ÷
÷
(a) 3 (b) (c) 1 (d)
2 3
è L ø
67. Two stars are 10 light years away from the earth. They are 72. The diameter of the objective lens of microscope makes an
seen through a telescope of objective diameter 30 cm. The angle b at the focus of the microscope. Further, the medium
wavelength of light is 600 nm. To see the stars just resolved between the object and the lens is an oil of refractive index
by the telescope, the minimum distance between them n. Then the resolving power of the microscope
should be (1 light year = 9.46 × 1015 m) of the order of : [Online April 19, 2014]
[Online April 10, 2016] (a) increases with decreasing value of n
(a) 10 km (b) 10 km (c) 1011 km (d) 106 km
8 10 (b) increases with decreasing value of b
68. In Young's double slit experiment, the distance between (c) increases with increasing value of n sin 2b
slits and the screen is 1.0 m and monochromatic light of 1
600 nm is being used. A person standing near the slits is (d) increases with increasing value of
n sin 2b
looking at the fringe pattern. When the separation between
73. A ray of light is incident from a denser to a rarer medium.
the slits is varied, the interference pattern disappears for a
The critical angle for total internal reflection is qiC
particular distance d0 between the slits. If the angular
and Brewster’s angle of incidence is qiB, such that sinqiC/
1° sinqiB = h = 1.28. The relative refractive index of the two
resolution of the eye is , the value of d0 is close to :
60 media is: [Online April 19, 2014]
[Online April 9, 2016] (a) 0.2 (b) 0.4 (c) 0.8 (d) 0.9
(a) 1 mm (b) 3mm (c) 2 mm (d) 4mm
P-422 Physics

74. In an experiment of single slit diffraction pattern, first 78. The first diffraction minimum due to the single slit
minimum for red light coincides with first maximum of some diffraction is seen at q = 30° for a light of wavelength 5000
other wavelength. If wavelength of red light is 6600 Å, Å falling perpendicularly on the slit. The width of the slit
then wavelength of first maximum will be: is [Online May 12, 2012]
[Online April 12, 2014] (a) 2.5 × 10– 5 cm (b) 1.25 × 10–5 cm
(a) 3300Å (b) 4400Å (c) 5500Å (d) 6600Å (c) 10 × 10–5 cm (d) 5 × 10–5 cm
75. Abeam of unpolarised light of intensity I0 is passed 79. Two polaroids have their polarizing directions parallel so
through a polaroidAand then through another polaroid B that the intensity of a transmitted light is maximum. The
which is oriented so that its principal plane makes an angle
angle through which either polaroid must be turned if the
of 45° relative to that of A. The intensity of the emergent
intensity is to drop by one-half is [Online May 7, 2012]
light is [2013]
(a) 135° (b) 90° (c) 120° (d) 180°
(a) I0 (b) I0/2 (c) I0/4 (d) I0/8
80. Statement - 1: On viewing the clear blue portion of the
76. This question has Statement-1 and Statements-2. Of the
sky through a Calcite Crystal, the intensity of transmit-
four choices given after the Statements, choose the one
ted light varies as the crystal is rotated.
that best describes the two Statements.
Statement - 2: The light coming from the sky is polarized
Statement-1 : Out of radio waves and microwaves, the
due to scattering of sun light by particles in the atmo-
radio waves undergo more diffraction.
sphere. The scattering is largest for blue light. [2011 RS]
Statement-2 : Radio waves have greater frequency
compared to microwaves. [Online April 25, 2013] (a) Statement -1 is true, statement-2 is false.
(a) Statement-l is true, Statement-2 is true and (b) Statement-1 is true, statement-2 is true, statement-2 is
Statement-2 is the correct explanation of Statement-1 the correct explanation of statement-1
(b) Statement-1 is false , Statement-2 is true. (c) Statement-1 is true, statement-2 is true, statement-2 is
(c) Statement-1 is true, Statement-2 is false. not the correct explanation of statement-1
(d) Statement-1 is true, Statement-2 is true but Statement- (d) Statement-1 is false, statement-2 is true.
2 is not the correct explanation of Statement-1 81. In an experiment, electrons are made to pass through a narrow
77. A person lives in a high-rise building on the bank of a river slit of width ‘d’ comparable to their de Broglie wavelength.
50 m wide. Across the river is a well lit tower of height 40 m. They are detected on a screen at a distance ‘D’ from the slit
When the person, who is at a height of 10 m, looks through (see figure).
a polarizer at an appropriate angle at light of the tower
reflecting from the river surface, he notes that intensity of
light coming from distance X from his building is the least d
y=0
and this corresponds to the light coming from light bulbs
at height 'Y' on the tower. The values of X and Y are
D
4 Which of the following graphs can be expected to
respectively close to (refractive index of water ; )
3 represent the number of electrons ‘N’ detected as a
[Online April 9, 2013] function of the detector position ‘y’(y = 0 corresponds to
the middle of the slit) [2008]
y y
L

(a) N d (b) N d
40 m

10 m y y
Y
X
50 m
(c) N d (d) N d
(a) 25 m, 10 m (b) 13 m, 27 m
(c) 22 m, 13 m (d) 17 m, 20 m
Wave Optics P-423

82. If I 0 is the intensity of the principal maximum in the single [Take wavelength of light = 500 nm] [2005]
(a) 1 m (b) 5 m (c) 3 m (d) 6 m
slit diffraction pattern, then what will be its intensity when
85. The angle of incidence at which reflected light is totally
the slit width is doubled? [2005]
polarized for reflection from air to glass (refractive index
I0 n), is [2004]
(a) 4 I 0 (b) 2 I 0 (c) (d) I 0
2
(a) tan -1 (1/ n) (b) sin -1 (1/ n)
83. When an unpolarized light of intensity I 0 is incident on a
(c) sin -1 (n) (d) tan -1 (n)
polarizing sheet, the intensity of the light which does not
get transmitted is [2005] 86. Wavelength of light used in an optical instrument are
1 l1 = 4000 Å and l 2 = 5000 Å , then ratio of their
1
(a) I0 (b) I0 (c) I 0 (d) zero
4 2 respective resolving powers (corresponding to l1 and
84. Two point white dots are 1 mm apart on a black paper.
They are viewed by eye of pupil diameter 3 mm. l 2 ) is [2002]
Approximately, what is the maximum distance at which (a) 16 : 25 (b) 9 : 1 (c) 4 : 5 (d) 5 : 4
these dots can be resolved by the eye?
P-424 Physics

1. (b) For (A) 2. (d) Initially, S2 L = 2 m


xP 2
æ3ö 5
S1 L = 22 + ç ÷ = = 2.5 m
P Q xQ è2ø 2
A l
d Path difference, Dx = S1 L - S2 L = 0.5 m =
2
xP - xQ = (d + 2.5) - (d - 2.5) = 5 m
Phase difference Df due to path difference L'
/c
2N
2p 2p p
= ( Dx) = (5) = . S2 L
l 20 2 2 N/c
At A, Q is ahead of P by path, as wave emitted by Q d
reaches before wave emitted by P.
p p S1
\ Total phase difference at A - =0
2 2
When the listner move from L, first maxima will appear if
(due to P being ahead of Q by 90°)
path difference is integral multiple of wavelength.
I A = I1 + I 2 + 2 I1 I 2 cos Df For example
Dx = nl = 1l (n = 1 for first maxima)
= I + I + 2 I I cos(0) = 4 I
\ Dx = l = S1 L '- S 2 L
For C,
Þ1= d - 2 Þ d = 3 m
Path difference, xQ - xP = 5 m
Phase difference Df due to path difference 3. (c) The distance traversed by light in a medium of
refractive index m in time t is given by
2p 2p p
= ( Dx ) = (5) = d = vt ...(i)
l 20 2
where v is velocity of light in the medium. The distance
p p traversed by light in a vacuum in this time,
Total phase difference at C = + =p
2 2
d
I net = I1 + I 2 + 2 I1 I 2 cos(Df) D = ct = c ´ [from equation (i)]
v

= I + I + 2 I I cos(p) = 0 c c
=d = md ...(ii) (Q m = )
For B, v v
This distance is the equvalent distance in vacuum and is
Path difference, xP - xQ = 0
called optical path.
p Optical path for first ray which travels a path L1 through a
Phase difference, Df =
2 medium of refractive index n1 = n1 L1
(due to P being ahead of Q by 90°)
Optical path for second ray which travels a path L2 through
p a medium of refractive index n2 = n2 L2
I B = I + I + 2 I I cos = 2 I
2
Therefore intensities of radiation at A, B and C will be in Path difference = n1 L1 - n2 L2
the ratio Now, phase difference
I A : I B : IC = 4 I : 2 I : 0 = 2 :1: 0. 2p 2p
= ´ path difference = ´ (n1 L1 - n2 L2 )
l l
Wave Optics P-425

a1 3 I 9I
4. (c) Given amplitude ratio of waves is a = 1 From figure I1 = and I2 =
4 64
2
I2 9
2 Þ =
Imax æ a2 + a1 ö I1 16
so, –
Imin çè a2 – a1 ÷ø 2
æ I2 ö
2 ç I + 1÷
æ a2 ö Imax
+1 2 By using =ç 1 ÷
ç a1 ÷ æ 3 + 1ö æ 4ö
2
4 Imin ç I2 ÷
=ç ÷ =ç ÷ =ç ÷ = =4 ç - 1÷
a
ç 2 –1÷ è 3 –1 ø è 2 ø 1 è I1 ø
çè a ÷ø 2
1
æ 9 ö
2 ç 16 + 1÷ 49
Imax æ A1 + A 2 ö = ç ÷ =
5. (b) As we know, =ç ÷ 1
Imin è A1 - A 2 ø ç 9 - 1÷
çè ÷
16 ø
I1 A
and = 1 l
I2 A2 11. (c) Dx =
(2a )
Imax A A + A2 4 12. (b) A phase change of p rad appears when the ray reflects
= 16 Þ max = 4 Þ 1 =
Imin A min A1 - A 2 1 at the glass-air interface. As a result, there will be a
destructive interference at the centre. So, the centre of the
Using componendo and dividendo. interference pattern is dark.
2
13. (b) When light beam is moving and as it enters the medium,
A1 5 I æ5ö 25 the refractive index will decrease from the axis towards the
= Þ 1 =ç ÷ =
A2 3 I2 è 3 ø 9 periphery of the beam.
Therefore, the beam will conver less distance as one move
Plane WF from the axis to the periphery and hence the beam will
µ increases converge.
6. (b) (Light bends
Vel decreases
upwards)
Refracted Decreasing m
WF Axis

7. (c)
8. (d) Intensity µ (amplitude)2
I1 16 a12 14. (d) Initially the parallel beam is cylindrical. Therefore, the
= = wavefront will be planar.
I 2 9 a22 15. (a) The speed of light (v) in a medium of refractive index
Þ a1 = 4; a2 = 3 (m) is given by
Therefore the ratio of intensities of bright and dark parts c
m = , where c is the speed of light in vacuum
v
I Bright (a1 + a2 )2 (4 + 3)2 49 c
= = = c
I Dark (a1 - a2 )2 (4 - 3) 2 1 \v= =
m m0 + m2 ( I )
(Maximum intensity) coherent interference As I is decreasing with increasing radius, it is maximum on
9. (d) the axis of the beam. Therefore, v is minimum on the axis of
(Maximum intensity) in coherent interference
the beam.
n 2 I0 16. (d) To demonstrate the phenomenon of interference we
= =n require two sources of light which emit radiation of same
nI0
frequency and having a definite phase relationship (a phase
10. (a) B B¢ relationship that does not change with time)
I 9 17. (9)
I/4 I
64 In young's double slit experiment, intensity at a point is
A A¢ 3 given by
I
3 64
I 3 f
4 I I = I 0 cos 2
16 ...(i)
2
where, f = phase difference,
P-426 Physics

2p Þ l2= 1.5l1 15l1 = 1.5 × 500 nm


Using phase difference, f = × path difference
l Þ l2 = 750 nm
For path difference l, phase difference f1 = 2p l
22. (a) Given, Path difference, Dx =
l p 8
For path difference, , phase difference f2 = 2p
6 3 Phase differences, Df = Dx
l
Using equation (i),
2p l p
= ´ =
æf ö æ 2p ö l 8 4
cos 2 ç 1 ÷ cos 2 ç ÷
I1 è 2ø è 2ø æ Df ö
= = I = I 0 cos 2 ç ÷
I2 2 æ f2 ö æ pö è 2 ø
cos ç ÷ cos 2 ç ÷
è 2ø è 3ø
æ pö
I ç ÷ æ pö
K 1 4 3K 9 K Þ = cos2 ç 4 ÷ = cos 2 ç ÷
Þ = = Þ I2 = = I0 2 è 8ø
I2 3 3 4 12 ç ÷
è ø
4
I
Þ = 0.853
\ n = 9. I0
18. (b) Given : Wavelength of light, l = 500 nm
23. (c) Given, distance between screen and slits, D = 1.5 m
Distance between the slits, d = 0.05 mm
Seperation between slits, d = 0.15 mm
Angular width of the fringe formed,
Wavelength of source of light, l = 589 nm
l 500 ´ 10-9 Fringe-width,
q= = = 0.01 rad = 0.57°.
d 0.05 ´ 10-3
D 1.5
19. (a) Path difference, DP = d sin q = d q w= l= ´ 589 ´ 10 –9 m
d = distance between slits = 1 mm = 10–3 mm d 0.15 ´ 10–3
D = distance between the slits and screen = 100 cm = 1 m
= 589 × 10–2 mm = 5.89 mm » 5.9 mm
y = distance between central bright fringe and observed
fringe = 1.27 mm 24. (c) Given, D = b
D (m – 1)t Dl
dy 10-3 ´1.270 mm or =
\ DP = = = 1.27 mm d d
D 1m l
20. (d) Let n1 fringes are visible with light of wavelength l1 \ t=
(m - 1)
and n2 with light of wavelength l2. Then 25. (Bonus) Shift = nb (given)
n1 Dl1 n2 Dl 2 æ nlD ö
b= = çèQ b = ÷ (m - 1)t n lD é
Q Shift =
D (m - 1) t ù
d d d ø \ D = ê úû
a a ë a
n 2 l1 nl
Þ = or t =
n1 l 2 (m - 1)

700 l
Þ n2 = ´ 16 = 28 26. (b) Given, path difference, Dx =
400 8
Phase difference (Df) is given by
lD
21. (750) Fringe width, b = where, l = wavelength, D =
d 2p
Df = ( Dx)
distance of screen from slits, d = distance between slits l

ATQ
Df =
( 2p ) l = p
l1D l D l 8 4
15 ´ = 10 ´ 2
d d
For two sources in different phases,
Þ 15l1 = 10l2
Wave Optics P-427

dy l
æ pö = ....(i)
I = I0 cos 2 ç ÷ D 4
è 8ø
lD
I æ pö = b ....(ii)
= cos2 ç ÷ d
I0 è 8ø
Multiplying equation (i) and (ii) we get,
p 1+ 1 b
1 + cos y=
= 4 = 2 = 0.85 4
2 2 32. (c)
27. (a) Path difference = d sinq » dq 33. (b) Given t = 1.8 × 10–6 m
µ = 1.6
1 n = 2 (from figure)
= 0.1 × mm = 2500nm
40 Applying formula (µ – 1) t = nl
For bright fringe, path difference must be integral multiple (1.6 – 1) × 1.8 × 10–6 = 2l
of l. 1.8 ´10-6 ´ 0.6
or, l =
\ 2500 = nl1 = ml2 2
= 540 nm
\ l1 = 625 (for n = 4), l2 = 500 (for m = 5)
34. (d) It will be concentric circles.
28. (a) Here, x1 = 2d and x2 = 5d 35. (d)
36. (c)
l
For, first minima, Dx = D
2
37. (c) Fringe width B = l
l d
\D x = x2 – x1 = 5d – 2d = And number of fringes observed in the field of view is
2
l d
Þd = obtained by
2 ( 5 –2 ) 38.
l
(d) Let a, be the amplitude of light from first slit and a2
29. (d) For 'n' number of maximas be the amplitude of light from second slit.
d sin q = nl a1 = a, Then a2 = 2a
0.32 ´ 10–3 sin 30° = n ´ 500 ´ 10–9 Intensity I µ (amplitude)2
\ I1 = a12 = a2
0.32 ´ 10 -3 1 I2 = a22 = 4a2 = 4I
\ n= -9
´ = 320
500 ´ 10 2 Ir = a12 + a22 + 2a1a2 cos f
Hence total no. of maximas observed in angular range – = I1 + I 2 + 2 I1 I2 cos f
30° £ q £ 30°
= 320 + 1 + 320 = 641 Ir = I1 + 4I1 + 2 4I12 cos f
30. (d) For common maxima, n1l1 = n2l2 Þ Ir = 5I1 + 4I1 cos f …(1)
n1 l 2 520 ´10 4 -9 Now, Imax = (a1 + a2)2 = (a + 2a)2 = 9a2
Þ = = =
n2 l1 650 ´ 10 -9 5 I
Imax = 9I1 Þ I1 = max
For l1 9
Substituting in equation (1)
n1l1D
y= , l1 = 650 nm 5 Imax 4 I max
d Ir = + cos f
4 ´ 650 ´ 10 -9 ´ 1.5 9 9
y= or, y = 7.8 mm
0.5 ´ 10 -3 I
Ir = max [ 5 + 4 cos f]
2 æ Df ö p 9
31. (c) 2I0 = 4I0 cos ç ÷ here, Df = I é f ù
è 2 ø 2 Ir = max ê 5 + 8 cos2 - 4 ú
9 ë 2 û
2p l
But, Df = Dx so, Dx = I é fù
l 4 Ir = max ê1 + 8 cos2 ú
9 ë 2û
P-428 Physics

39. (c) We know that, 43. (a) For coherent sources, intensity at mid point
I1 µ (a + a)2
2
æ w1 ö Þ I1 µ (2a)2
ç w + 1÷ For incoherent sources, intensity of mid point is
I max è 2 ø
= 2
I2 µ 2a2
I min æ w ö
1 -1 I 2
ç w ÷ \ 1 =
è 2 ø I2 1
Imax and Imin are maximum and minium intensity 44. (b) Let l be the wavelength of unknown light. Third bright
fringe of known light coincides with the 4th bright fringe of
the unknown light.
w1 and w2 are widths of two slits
3l1D 4lD
2 \ =
æ 1 ö d d
çç + 1÷÷ æ w1 ö
I max 25 1 3(590) D 4lD
=è çè w = 25 given÷ø
\ ø \ =
I min 2 2 d d
æ 1 ö
çç - 1 ÷÷ 3
è 25 ø Þ l = ´ 590 = 442.5 nm
4
On solving we get, 45. (a) For path difference of l, the phase difference is 2p
36 l
For path difference of , the phase difference is
I max 9 6
= 25 = =9:4
I min 16 4 2p ´ l / 6 p
=
25 l 3
Resultant intensity
nl p
40. (b) As sin q = and sinq cannot be > 1 I = I1 + I 2 + 2 I1 I 2 cos
d 3
nl \ I = I1 + I 2 + I1 I 2
\1=
1.8l For two identical source, I1= I2 = I¢ (say)
or n = 1.8 then I = 3I¢
( )
Hence maximum number of possible interference maximas, 2
Imax = I1 + I 2
0, ±1 i.e. 3
Maximum resultant intensity,
b ( m - 1) b ( m - 1)
( ) = (2 I ¢ )
41. (c) Shift = t1 - t2 2 2
l l = I¢ + I¢ = 4I ¢
b ( m - 1) \ I 3
=
=
l
( t1 - t2 ) I max 4
42. (a) Path difference at P Dx1 = 0
\ Phase difference at P will be The intensity of light at any point of the screen where the
2p phase difference due to light coming from the two slits is f is
Df1 = Dx1 given by
l
2p æ fö
´0 I = I o cos 2 ç ÷
= è 2ø
l Where I0 is the maximum intensity.
= 0°
Resultant Intensity at P NOTE This formula is applicable when I1= I2.
I1 = I 0 + I 0 + 2 I 0 cos 0° = 4 I 0 2p l
Phase difference f = ´ = p/3
Path difference at Q l 6
l 2
Dx2 = I p æ 3ö 3
4 \ = cos2 = ç ÷ =
\ Phase difference at Q I0 6 è 2 ø 4
2p l æ p ö 46. (d) The light passing through the slits interfere and
D= × =ç ÷ produce dark and bright band one screen. The shape of
l 4 è2ø
Resultant intensity at Q. interference fringes formed on a screen in case of a
p monochromatic source is a straight line.
I 2 = I 0 + I 0 + 2 I0 cos = 2 I 0 47. (b) For constructive interference path difference (As sin
2
I1 4 I 0 2 q £ 1)
Thus, = = d sin q = nl
I 2 2I0 1
Wave Optics P-429

Given d = 2l
(1.22)(5500 ´ 10 -10 ) ´ 4 ´ 105 ´ 103
n =
\ 2l sin q = nl Þ sin q = 5
2
n = 0, 1, – 1, 2, – 2 hence five maxima are possible. = 5368 × 10–2 m = 53.68 m » 60 m
48. (d) Given : 51. (b) According to question, the intensity of light coming
Intensity, I0 = 3.3 Wm–2 out of the analyser is just 10% of the original intensity (I0)
Area, A = 3 × 10–4 m2 Using, I = I0cos2q
Angular speed, w = 31.4 rad/s
I0 1
Average energy = I0 A < cos2 q > Þ = I 0 cos2 q Þ = cos 2 q
10 10
1 1
Q < cos2 q > = per revolution Þ cos q = = 0.316 Þ q » 71.6°
2 10
Therefore, the angle by which the analyser need to be
(3.3)(3 ´ 10-4 )
\ Average energy = ; 5 ´ 10- 4 J rotated further to reduced the output intensity to be zero
2
f = 90° – q = 90° – 71.6° = 18.4°
49. (198)
For obtaining secondary minima at a point path difference 1.22l
should be integral multiple of wavelength 52. (a) x =
2µsinq
\ d sin q = nl
1.22 ´ 5000
= = 0.24 µm
nl 2 ´ 1.25
\ sin q =
d
æ I0 ö
For n to be maximum sin q = 1 53. (c) I = çè 2 ÷ø cos2 30° cos2 60°

d 6 ´ 10-5
n= = = 100
l 6 ´ 10-7
Total number of minima on one side = 99
Total number of minima = 198.
I0 3 1
= ´ ´
2 4 4
50. (a)
I 0 32
\ = = 10.67
I 3
1.22l 1.22 ´ 600 ´ 10-9
54. (c) q = =
d 250 ´ 10-2
= 3.0 × 10–7 rad
1.22l 1.22 ´ 500 ´ 10 -9
55. (a) q = = = 305 × 10–9 rad.
d 2
56. (b)
Smallest angular separation between two distant objects 57. (b) According to Brewster’s law, refractive index of ma-
terial (m) is equal to tangent of polarising angle
here moon and earth,
1.5
l Qtan i b = μ =
q = 1.22 μ
a
1 1.5
a = aperture diameter of telescope < (Qsin ic < sin ib )
μ μ + (1.5)
2 2

Distance O1O2 = (q)d


1.5
\sin i b =
Minimum separation between objects on the surface of moon, μ + (1.5)
2 2

æ lö
= ç 1.22 ÷ d or, μ 2 + (1.5 ) <1.5´ μ
2
è aø
P-430 Physics

Þ μ 2 + (1.5) < ( μ×1.5)


2 2
c+v
f = f0 ; V = relative speed of approach
3 c+v
Þμ < i.e. minimum value of m should
5 f0 = 10 GHz
3 c
be c+
5 2 = 10 3 = 17.3 GHz
f = 10
2l c
c-
58. (a) Angular width of central maxima = 2
d
d l´D 64. (b) a = 0.1 mm = 10–4 cm,
or, l = ; Fringe width, b = l = 6000 × 10–10 cm = 6 × 10–7 cm, D = 0.5 m
2 d'
for 3rd dark band, a sin q = 3 l
d 50 ´ 10-2 10 -6 ´ 50 ´10 -2 3l x
10-2 = ´ = or sin q = =
2 d' 2´d' a D
The distance of the third dark band from the central bright
Therefore, slit separation distance, d’ = 25mm
band
59. (c) Axis of transmission of A & B are parallel.
-7
Polariser A Polariser B 3 lD 3 ´ 6 ´ 10 ´ 0.5
x= = = 9 mm
a 10-4
65. (b) For secondary minima,
nl
I I/2 I/2 b sin q = nl Þ sin q =
b
Distance of n th secondary minima x = D sin q
After introducing polariser C between A and B, x
or sin q1 = 1
Polariser A Polariser C Polariser B D
2l
sin q1 =
b
n=4
I/2Cos q
2
I I/2 I/2Cos4 q 4l x 2
sin q2 = =
b D
I I 1 4l 2 l 2 l
cos4 q = Þ cos4 q = x 2 - x1 = - =
2 8 4 b b b
1 2l 2l
Þ cos q = or, q = 45° 3= Þb= ..... (i)
2 b 3
60. (a) If angular position of 2nd maxima from central maxima 2l
Width of central maxima =
is q then b
(2n - 1) l 3l 3 ´ 550 ´ 10 -9 2l
sin q = = = = = 3 cm.
2a 20 2 ´ 22 ´ 10 -7 2l
\ q ; p rad 3
8 .. from eq. (i)
61. (a) Polariser A and B have same alignment of transmission 66. (a) Given geometrical spread = a
axis. l lL
Lets assume polariser c is introduced at q angle Diffraction spread = ´ L =
a a
1 1 lL
cos 2 q ´ cos 2 q = The sum b = a +
2 3 a
For b to be minimum
14
2 æ 2ö db d æ lL ö
or, cos 4 q = Þ cos q = ç ÷ =0 ça + ÷ =0
3 è 3ø da da è a ø
62. (a) a = lL
63. (a) Use relativistic doppler's effect as velocity of b min = lL + lL = 2 lL = 4lL
observer is not small as compared to light
Wave Optics P-431

0.61l l where, qiB is Brewster’s angle of incidence,


67. (a) We know that Dq = =
4 R 1
The minimum distance between them And, m =
sin qic
R 9.46´1015 ´10´ 0.61´ 600´10,9 On solving we get, relative refractive index of the two me-
l< 0.61´l < dia.
9 0.3
74. (b) In a single slit experiment,
= 1.15 × 10 m
11
For diffraction maxima,
Þ 1.115 × 108 km.
68. (c) Given D = 1.0m, wavelength of monochromatic light l
a sin q = ( 2n + 1)
l = 600 nm. 2
p 1 and for diffraction minima,
d : Dθ < 1´ ´
180 60 a sin q = nl
According to question,
d0 < 2´10,3 < 2 mm
l
0.25 1 ( 2 ´1 + 1) = 1´ 6600
69. (d) sin q = = 2
25 100
(Q l R = 6600Å )
6600 ´ 2
0.25cm q l=
3
l = 4400Å
75. (c) Relation between intensities
25cm
1.22l 45° B
I0 (I0/2)
Resolving power = = 30 mm.
2m sin q
(unpolarised) IR
70. (a) When unpolarised light is incident at Brewster’s
angle then reflected light is completely polarized and the A
intensity of the reflected light is less than half of the
incident light. æI ö I 1 I
71. (d) According to malus law, intensity of emerging beam I r = ç 0 ÷ cos2 (45°) = 0 ´ = 0
è 2ø 2 2 4
is given by,
76. (c) Wavelength of radio waves is greater than
I = I0cos2q
microwaves hence frequency of radio waves is less than
Now, IA¢ = IA cos230º microwaves.
IB¢ = IB cos260º The degree of diffraction is greater whose wavelength is
As IA¢ = IB¢ greater.
77. (b)
3 1 IA 1
Þ IA ´ = IB ´ ; = 78. (c) For first minimum,
4 4 IB 3 dsinq = l
72. (c) Resolving power of microscope, l 5000 ´ 10 -8 cm
Þ d= =
2n sin q sin q sin 30°
R.P. =
l 5000 ´10 -8 cm
= = 10 ´ 10 -5 cm
l = Wavelength of light used to illuminate the object 1
2
n = Refractive index of the medium between object and
I0 2 I0
objective 79. (a) For I = and I = I0 cos q =
2 2
q = Angle \ q = 45°
73. (c) Here, sin qic / sin qiB = 1.28 Therefore the angle through which either polaroids turned
As we know, is 135° (= 180° – 45°)
80. (b) When viewed through a polaroid which is rotated
sin qiB
m= then the light from a clear blue portion of the sky shows a
æp ö rise and fall of intensity. The light coming from the sky is
sin ç - qiB ÷
è2 ø polarised due to scattering of sunlight by particles in the
atmosphere.
P-432 Physics

Incident sunlight I0
(unpolarised) \ Intensity of polarized light =
2
Þ Intensity of untransmitted light
Sun I I
= I0 - 0 = 0
Scattered light 2 2
84. (b) y ³ 1.22 l
(polarised)
D d

Þ D£ yd 10 - 3 ´ 3 ´ 10 - 3 = 30
= » 5m
to observer (1.22) l (1.22) ´ 5 ´ 10 - 7 6.1
81. (d) The electron beam will be diffracted and the maxima
is obtained at y = 0. \ Dmax = 5m
Also, the diffraction pattern, should be wider than the slit
85. (d) From the Brewster’s law, angle of incidence for total
width.
polarization is given by tan q = n
2 Þ q = tan–1 n
æ sin f ö p
82. (a) I = I 0 çç ÷÷ and f = ( b sin q ) Where n is the refractive index of the glass.
è f ø l
When the slit width is doubled, the amplitude of the wave 86. (d) The resolving power of an optical instrument is
at the centre of the screen is doubled, so the intensity at inversely proportional to the wavelength of light used.
the centre is increased by a factor 4. ( R.P )1 l 2 5
83. (b) From the law of Malus, I = I0cos2 q = =
( R.P )2 l1 4
When an unpolarised light is converted into plane
polarised light by passing through polariod, its intensity
become hafl.
25
Dual Nature of Radiation and Matter P-433

Dual Nature of
Radiation and Matter
5. A particle moving with kinetic energy E has de Broglie
Matter Waves, Cathode wavelength l. If energy DE is added to its energy, the
TOPIC 1
and Positive Rays l
wavelength become . Value of DE, is: [9 Jan. 2020 I]
1. An electron, a doubly ionized helium ion (He+ +)
and a 2
proton are having the same kinetic energy. The relation (a) E (b) 4E (c) 3E (d) 2E
between their respective de-Broglie wavelengths le, lHe++ 6. An electron of mass m and magnitude of charge |e| initially
and lp is : [Sep. 06, 2020 (I)] at rest gets accelerated by a constant electric field E. The
(a) le > lHe+ + > lp (b) le < lHe+ + = lp rate of change of de-Broglie wavelength of this electron at
time t ignoring relativistic effects is: [9 Jan. 2020 II]
(c) le > lp > lHe+ + (d) le < lp < lHe+ +
2. Assuming the nitrogen molecule is moving with h | e | Et
(a) - (b)
r.m.s.velocity at 400 K, the de-Broglie wavelength of |e|E t h
nitrongen molecule is close to :
h -h
(Given : nitrogen molecule weight : 4.64 × 10–26 kg, Boltzman (c) - (d)
| e | Et | e | Et 2
constant : 1.38 × 10–23 J/K, Planck constant : 6.63 × 10–34 J.s)
r
[Sep. 06, 2020 (II)] 7. An electron (mass m) with initial velocity v = v0iˆ + v0 ˆj is
r
(a) 0.24 Å (b) 0.20 Å in an electric field E = - E0 kˆ. If l 0 is initial de-Broglie
(c) 0.34 Å (d) 0.44 Å wavelength of electron, its de-Broglie wave length at time
m t is given by: [8 Jan. 2020 II]
3. Particle A of mass mA = moving along the x-axis with
2 l0 2
velocity v0 collides elastically with another particle B at l0
(a) (b)
2 2 2
m e E t e2 E02 t 2
rest having mass mB = . If both particles move along 1+ 1+
3 m2 v02 m2 v02
the x-axis after the collision, the change Dl in de-Broglie
wavelength of particle A, in terms of its de-Broglie l0 l0
wavelength (l0) before collision is : [Sep. 04, 2020 (I)] (c) (d)
e2 E 2 t 2 e2 E 2t 2
3 5 1+ 2+
(a) Dl = l0 (b) Dl = l0 2 m2 v02 m 2 v02
2 2
8. A particle ‘P’ is formed due to a completely inelastic collision
(c) Dl = 2l 0 (d) Dl = 4l 0 of particles ‘x’ and ‘y’ having de-Broglie wavelengths ‘gx’
4. A particle is moving 5 times as fast as an electron. The and ‘gy’ respectively. If x and y were moving in opposite
ratio of the de-Broglie wavelength of the particle to that of directions, then the de-Broglie wavelength of ‘P’ is:
the electron is 1.878 × 10–4. The mass of the particle is [9 Apr. 2019 II]
close to : [Sep. 02, 2020 (II)]
g xg y g xg y
(a) 4.8 × 10–27 kg (b) 9.1 × 10–31 kg (a) (b) | g - g |
(c) 1.2 × 10–28 kg (d) 9.7 × 10–28 kg gx +g y x y

(c) g x - g y (d) g x + g y
P-434 Physics

9. Two particles move at right angle to each other. Their de lA 2 lA 1


Broglie wavelengths are l1 and l2 respectively. The particles (a) = (b) =
lB 3 lB 2
suffer perfectly inelastic collision. The de Broglie
wavelength l, of the final particle, is given by : lA 1 lA
[8 April 2019 I] (c) = (d) l = 2
lB 3 B
1 1 1 16. A parallel beam of electrons travelling in
(a) = + (b) l = l1l2
l 2
l12 l 22 x-direction falls on a slit of width d (see figure). If after
passing the slit, an electron acquires momentum py in the
l2 + l2 2 1 1 y-direction then for a majority of electrons passing through
(c) l = (d) l = l + l the slit (h is Planck’s constant) : [Online April 10, 2015]
2 1 2
10. A particle A of mass ‘m’ and charge ‘q’ is accelerated by a
potential difference of 50v Another particle B of mass ‘4m’
and charge‘q’ is accelerated by a potential differnce of Y
lA
2500V. The ratio of de-Broglie wavelength is X
lB

[12 Jan. 2019 I]


(a) 10.00 (b) 0.07
(c) 14.14 (d) 4.47
11. If the deBroglie wavelength of an electron is equal to 10– (a) |Py| d > h (b) |Py| d < h
3 times the wavelength of a photon of frequency 6 × 1014 (c) |Py| d ; h (d) |Py| d > >h
Hz, then the speed of electron is equal to : 17. de-Broglie wavelength of an electron accelerated by a voltage
(Speed of light = 3 × 108 m/s) of 50 V is close to (|e| = 1.6 × 10–19 C, me = 9.1 × 10–31 kg,
Planck’s constant = 6.63 × 10–34J.s h = 6.6 × 10–34 Js) : [Online April 10, 2015]
Mass of electron = 9.1 × 10–31 kg) [11 Jan. 2019 I] (a) 2.4 Å (b) 0.5 Å (c) 1.7 Å (d) 1.2 Å
6
(a) 1.1 ×10 m/s (b) 1.7 ×106 m/s 18. For which of the following particles will it be most difficult
6 to experimentally verify the de-Broglie relationship?
(c) 1.8 ×10 m/s (d) 1.45 ×106 m/s
12. In an electron microscope, the resolution that can be [Online April 9, 2014]
achieved is of the order of the wavelength of electrons (a) an electron (b) a proton
used. To resolve a width of 7.5 × 10–12 m, the minimum (c) an a-particle (d) a dust particle
electron energy required is close to:[10 Jan. 2019 I] 19. Electrons are accelerated through a potential difference V
(a) 500 keV (b) 100 keV (c) 1 keV (d) 25 keV and protons are accelerated through a potential difference
13. Two electrons are moving with non-relativistic speeds 4 V. The de-Broglie wavelengths are le and lp for electrons
perpendicular to each other. If corresponding de Broglie
le
wavelengths are l1 and l2, their de Broglie wavelength in and protons respectively. The ratio of is given by :
the frame of reference attached to their centre of mass is: lp
[Online April 15, 2018] (given me is mass of electron and mp is mass of proton).
[Online April 23, 2013]
1 1 1
(a) lCM = l1 = l2 (b) = +
l1 l1 l 2 le mp le me
(a) = (b) =
lp me lp mp
2l1l 2 æ l + l2 ö
lCM = lCM = ç 1
(c)
l12 + l 22
(d) ÷ le 1 me mp
è 2 ø le
(c) = (d) =2
14. If the de Broglie wavelengths associated with a proton and l p 2 mp lp me
an a-particle are equal, then the ratio of velocities of the
20. If the kinetic energy of a free electron doubles, it’s de-
proton and the a-particle will be:[Online April 15, 2018]
(a) 1 : 4 (b) 1 : 2 (c) 4 : 1 (d) 2 : 1 Broglie wavelength changes by the factor [2005]
15. A particle A of mass m and initial velocity v collides with 1 1
(a) 2 (b) (c) 2 (d)
2 2
m
a particle B of mass which is at rest. The collision is 21. Formation of covalent bonds in compounds exhibits [2002]
2 (a) wave nature of electron
head on, and elastic. The ratio of the de-Broglie (b) particle nature of electron
wavelengths lA to lB after the collision is [2017] (c) both wave and particle nature of electron
(d) none of these
Dual Nature of Radiation and Matter P-435

27. Two sources of light emit X-rays of wavelength 1 nm and


Photon, Photoelectric Effect visible light of wavelength 500 nm, respectively. Both the
TOPIC 2 X-rays and Davisson-Germer sources emit light of the same power 200 W. The ratio of
Experiment the number density of photons of X-rays to the number
density of photons of the visible light of the given
22. A beam of electrons of energy E scatters from a target wavelengths is : [Sep. 03, 2020 (II)]
having atomic spacing of 1Å. The first maximum intensity
1 1
occurs at q = 60º. Then E (in eV) is ______. (a) (b) 250 (c) (d) 500
500 250
(Plank constant h = 6.64 × 10–34 Js, 1 eV = 1.6 × 10–19 J, 28. When radiation of wavelength l is used to illuminate a
electron mass m = 9.1 × 10–31 kg) [NA Sep. 05, 2020 (I)] metallic surface, the stopping potential is V. When the
23. The surface of a metal is illuminated alternately with same surface is illuminated with radiation of wavelength
photons of energies E1 = 4 eV and E2 = 2.5 eV respectively.
V
The ratio of maximum speeds of the photoelectrons emitted 3l, the stopping potential is . If the theshold
in the two cases is 2. The work function of the metal in (eV) 4
is _______________. [NA Sep. 05, 2020 (II)] wavelength for the metallic surface is nl then value of n
24. Given figure shows few data points in a photo electric will be _________. [NA Sep. 02, 2020 (I)]
effect experiment for a certain metal. The minimum energy 29. Radiation, with wavelength 6561 Å falls on a metal surface
for ejection of electron from its surface is : (Plancks constant to produce photoelectrons. The electrons are made to enter
h = 6.62 × 10–34 J.s) [Sep. 04, 2020 (I)] a uniform magnetic field of 3 × 10–4 T. If the radius of the
largest circular path followed by the electrons is 10 mm,
Y the work function of the metal is close to: [9 Jan. 2020 I]
(a) 1.1 ev (b) 0.8 ev
Vstop (V)

(c) 1.6 ev (d) 1.8 ev


C 30. When photon of energy 4.0 eV strikes the surface of a
(6, V) metal A, the ejected photoelectrons have maximum kinetic
B (5.5, 0) energy TA eV and de-Broglie wavelength lA. The maximum
kinetic energy of photoelectrons liberated from another
A metal B by photon of energy 4.50 eV is TB=(TA–1.5)eV. If
X
5 14
f(10 Hz) the de-Broglie wavelength of these photoelectrons lB =
2lA, then the work function of metal B is:[8 Jan. 2020 I]
(a) 2.27 eV (b) 2.59 eV (a) 4 eV (b) 2 eV
(c) 1.5 eV (d) 3 eV
(c) 1.93 eV (d) 2.10 eV
31. A beam of electromagnetic radiation of intensity
25. In a photoelectric effect experiment, the graph of stopping
6.4 × 10–5 W/cm 2 is comprised of wavelength, l = 310
potential V versus reciprocal of wavelength obtained is
nm. It falls normally on a metal (work function j = 2eV)
shown in the figure. As the intensity of incident radiation
of surface area of 1 cm 2. If one in 10 3 photons ejects an
is increased : [Sep. 04, 2020 (II)]
electron, total number of electrons ejected in 1 s is 10x .
V (hc = 1240 eVnm, l eV = 1.6 × 10–19 J), then x is _______.
[NA 7 Jan. 2020 I]
32. The stopping potential V0 (in volt) as a function of frequency
(v) for a sodium emitter, is shown in the figure. The work
q function of sodium, from the data plotted in the figure, will be :
1/l
(Given : Planck’s constant (h) = 6.63 × 10–34 Js, electron
(a) Straight line shifts to right charge e = 1.6 × 10–19 C) [12 Apr. 2019 I]
(b) Slope of the straight line get more steep
(c) Straight line shifts to left
(d) Graph does not change
26. When the wavelength of radiation falling on a metal is
changed from 500 nm to 200 nm, the maximum kinetic energy
of the photoelectrons becomes three times larger. The work
function of the metal is close to : [Sep. 03, 2020 (I)]
(a) 0.81 eV (b) 1.02 eV
(c) 0.52 eV (d) 0.61 eV (a) 1.82 eV (b) 1.66 eV (c) 1.95 eV (d) 2.12 eV
P-436 Physics

33. In a photoelectric effect experiment the threshold 40. Surface of certain metal is first illuminated with light of
wavelength of light is 380 nm. If the wavelength of incident wavelength l1 = 350 nm and then, by light of wavelength
light is 260 nm, the maximum kinetic energy of emitted l2 = 540 nm. It is found that the maximum speed of the
electrons will be: photo electrons in the two cases differ by a factor of (2)
The work function of the metal (in eV) is close to:
Given E (in eV) = [10 Apr. 2019 I]
(a) 1.5 eV (b) 3.0 eV (c) 4.5 eV (d) 15.1 eV 1240
(Energy of photon = l in nm eV ) [9 Jan. 2019 I]
34. A 2 mW laser operates at a wavelength of 500 nm. The ( )
number of photons that will be emitted per second is : (a) 1.8 (b) 2.5 (c) 5.6 (d) 1.4
[Given Planck’s constant h= 6.6×10–34 Js, speed of light 41. The magnetic field associated with a light wave is given
c = 3.0×108 m/s] [10 Apr. 2019 II] at the origin by
(a) 5×1015 (b) 1.5×1016 B = B0 [sin(3.14 × 107)ct + sin(6.28 × 107)ct].
(c) 2×1016 (d) 1×1016 If this light falls on a silver plate having a work function
35. The electric field of light wave is given as of 4.7 eV, what will be the maximum kinetic energy of the
r photoelectrons? [9 Jan. 2019 II]
æ 2p x ö N
E = 103 cos ç -7
- 2p ´ 6 ´ 1014 t ÷ x$ (c = 3 × 108 ms–1, h = 6.6 × 10–34J-s)
è 5 ´10 ø C (a) 6.82 eV (b) 12.5 eV
This light falls on a metal plate of work function 2eV. The (c) 8.52 eV (d) 7.72 eV
stopping potential of the photo-electrons is: 42. An electron beam is accelerated by a potential difference V to
hit a metallic target to produce X-rays. It produces continuous
12375 as well as characteristic X-rays.If lmin is the smallest possible
Given, E (in eV) = [9 April 2019 I] wavelength of X-ray in the spectrum, the variation of log lmin
l ( in Å )
(a) 2.0 V (b) 0.72 V (c) 0.48 V (d) 2.48 V with log V is correctly represented in : [2017]
36. When a certain photosensistive surface is illuminated with
monochromatic light of frequency v, the stopping potential (a) (b)
for the photo current is –V 0/2. When the surface is
illuminated by monochromatic light of frequency v/2, the
stoppoing potential is –V0. The threshold frequency for
photoelectric emission is : [12 Jan. 2019 II] (c) (d)

5v 4 3v
(a) (b) v (c) 2 v (d) 43. A Laser light of wavelength 660 nm is used to weld Retina
3 3 2
detachment. If a Laser pulse of width 60 ms and power 0.5
37. In a Frank-Hertz experiment, an electron of energy 5.6 eV kW is used the approximate number of photons in the
passes through mercury vapour and emerges with an pulse are : [Take Planck’s constant h = 6.62 × 10–34 Js]
energy 0.7 eV. The minimum wavelength of photons emitted [Online April 9, 2017]
by mercury atoms is close to : [12 Jan. 2019 II] (a) 10 20 (b) 1018 (c) 1022 (d) 1019
(a) 1700 nm (b) 2020 nm 44. The maximum velocity of the photoelectrons emitted from
(c) 220 nm (d) 250 nm the surface is v when light of frequency n falls on a metal
38. In a photoelectric experiment, the wavelength of the light surface. If the incident frequency is increased to 3n, the
incident on a metal is changed from 300 nm to 400 nm. The maximum velocity of the ejected photoelectrons will be :
decrease in the stopping potential is close to : [Online April 8, 2017]
[11 Jan. 2019 II]
(a) less than 3v (b) v
æ hc ö
çè = 1240 nm-V÷ø (c) more than 3v (d) equal to 3v
e 45. Radiation of wavelength l, is incident on a photocell. The
(a) 0.5 V (b) 1.5 V fastest emitted electron has speed v. If the wavelength is
(c) 1.0 V (d) 2.0 V 3l
39. A metal plate of area 1 × 10–4 m2 is illuminated by a changed to , the speed of the fastest emitted electron
4
radiation of intensity 16 mW/m2. The work function of the
will be: [2016]
metal is 5 eV. The energy of the incident photons is 10 eV
1 1
and only 10% of it produces photo electrons. The number æ 4 ö2 æ 3 ö2
of emitted photo electrons per second and their maximum (a) = vç ÷ (b) = vç ÷
energy, respectively, will be: è3ø è4ø
1 1
[1 eV = 1.6 × 10–19 J] [10 Jan. 2019 II]
æ 4 ö2 æ 4 ö2
(a) 1014 and 10 eV (b) 1012 and 5 eV (c) > vç ÷ (d) < vç ÷
(c) 1011 and 5 eV (d) 1010 and 5 eV è3ø è3ø
Dual Nature of Radiation and Matter P-437

46. A photoelectric surface is illuminated successively by 50. A photon of wavelength l is scattered from an electron,
which was at rest. The wavelength shift Dl is three times
l
monochromatic light of wavelengths l and . If the of l and the angle of scattering q is 60°. The angle at which
2 the electron recoiled is f. The value of tan f is : (electron
maximum kinetic energy of the emitted photoelectrons in speed is much smaller than the speed of light)
the second case is 3 times that in the first case, the work [Online April 11, 2014]
function of the surface is : [Online April 10, 2016] (a) 0.16 (b) 0.22 (c) 0.25 (d) 0.28
hc hc 51. The anode voltage of a photocell is kept fixed. The
(a) (b)
2l l wavelength l of the light falling on the cathode is gradually
hc 3hc changed. The plate current I of the photocell varies as
(c) (d) follows : [2013]
3l l
47. When photons of wavelength l1 are incident on an isolated
I I
sphere, the corresponding stopping potential is found to
be V. When photons of wavelength l2 are used, the
corresponding stopping potential was thrice that of the (a) (b)
above value. If light of wavelength l3 is used then find the
stopping potential for this case : [Online April 9, 2016] O l O l
hc é 1 1 1ù I I
(a) ê + - ú
e ë l 3 l 2 l1 û
hc é 1 1 1ù (c) (d)
(b) ê + - ú
e ë l3 2l 2 l1 û
O l O l
hc é 1 1 1ù 52. In an experiment on photoelectric effect, a student plots
(c) ê - - ú stopping potential V0 against reciprocal of the wavelength
e ë l 3 l 2 l1 û
l of the incident light for two different metals A and B.
hc é 1 1 3 ù These are shown in the figure. [Online April 25, 2013]
(d) ê + - ú
e ë l3 2l 2 2l1 û
48. Match List - I (Fundamental Experiment) with List - II (its
V0 Metal A
conclusion) and select the correct option from the choices
Metal B
given below the list: [2015]
Lis t-I Lis t-II
A . Fran ck-Hertz (i) Particle n atu re of
Exp eriment ligh t
B. Pho to -electric (ii) Dis crete en erg y 1/l
experimen t levels of atom Looking at the graphs, you can most appropriately say
C. Davis on -Germer (iii) W av e natu re o f that:
experimen t electro n
(a) Work function of metal B is greater than that of metal A
(iv ) Structu re o f ato m (b) For light of certain wavelength falling on both metal,
(a) (A)-(ii); (B)-(i); (C)-(iii) maximum kinetic energy of electrons emitted from A
(b) (A)-(iv); (B)-(iii); (C)-(ii) will be greater than those emitted from B.
(c) (A)-(i); (B)-(iv); (C)-(iii) (c) Work function of metal A is greater than that of metal B
(d) (A)-(ii); (B)-(iv); (C)-(iii) (d) Students data is not correct
49. A beam of light has two wavelengths of 4972Å and 6216Å 53. A copper ball of radius 1 cm and work function 4.47eV is
with a total intensity of 3.6 × 10 –3 Wm –2 equally irradiated with ultraviolet radiation of wavelength 2500 Å.
distributed among the two wavelengths. The beam falls The effect of irradiation results in the emission of electrons
normally on an area of 1 cm2 of a clean metallic surface from the ball. Further the ball will acquire charge and due
of work function 2.3 eV. Assume that there is no loss of to this there will be a finite value of the potential on the ball.
light by reflection and that each capable photon ejects The charge acquired by the ball is :
one electron. The number of photoelectrons liberated in [Online April 25, 2013]
2s is approximately: [Online April 12, 2014] (a) 5.5×10 C –13 (b) 7.5 × 10–13C
(a) 6 × 1011 (b) 9 × 1011 (c) 4.5 × 10 C –12 (d) 2.5 × 10–11C
(c) 11 × 1011 (d) 15 × 1011
P-438 Physics

54. This equation has statement 1 and statement 2. Of the four (a) Statement–1 is true, Statement–2 is true, Statement –
choices given after the statements, choose the one that 2 is the correct explanation of Statement – 1.
describes the two statements. (b) Statement–1 is true, Statement–2 is true, Statement
Statement 1: Davisson-Germer experiment established the – 2 is not the correct explanation of Statement – 1.
wave nature of electrons. (c) Statement – 1 is false, Statement – 2 is true.
Statement 2 : If electrons have wave nature, they can (d) Statement – 1 is true, Statement – 2 is false.
interfere and show diffraction. [2012] 58. Statement -1 : When ultraviolet light is incident on a
(a) Statement 1 is false, Statement 2 is true. photocell, its stopping potential is V0 and the maximum
(b) Statement 1 is true, Statement 2 is false kinetic energy of the photoelectrons is Kmax .When the
(c) Statement 1 is true, Statement 2 is true, Statement ultraviolet light is replaced by X-rays, both V0 and Kmax
2 is the correct explanation of statement 1 increase.
(d) Statement 1 is true, Statement 2 is true, Statement 2 is Statement -2 : Photoelectrons are emitted with speeds
not the correct explanation of Statement 1 ranging from zero to a maximum value because of the
55. Photoelectrons are ejected from a metal when light of range of frequencies present in the incident light. [2010]
frequency u falls on it. Pick out the wrong statement from (a) Statement -1 is true, Statement -2 is true ; Statement
the following. [Online May 26, 2012] -2 is the correct explanation of Statement -1.
(a) No electrons are emitted if u is less than (b) Statement -1 is true, Statement -2 is true; Statement
W/h, where W is the work function of the metal -2 is not the correct explanation of Statement -1
(b) The ejection of the photoelectrons is instantaneous. (c) Statement -1 is false, Statement -2 is true.
(d) Statement -1 is true, Statement -2 is false.
(c) The maximum energy of the photoelectrons is hu.
59. The surface of a metal is illuminted with the light of 400
(d) The maximum energy of the photoelectrons is nm. The kinetic energy of the ejected photoelectrons was
independent of the intensity of the light. found to be 1.68 eV. The work function of the metal is :
56. This question has Statement 1 and Statement 2. Of the (hc = 1240 eV.nm) [2009]
four choices given after the Statements, choose the one (a) 1.41 eV (b) 1.51 eV
that best describes the two Statements. (c) 1.68 eV (d) 3.09 eV
Statement 1: A metallic surface is irradiated by a Directions: Question No. 60 and 61 are based on the following
monochromatic light of frequency u > u0 (the threshold paragraph.
frequency). If the incident frequency is now doubled, the Wave property of electrons implies that they will show diffraction
photocurrent and the maximum kinetic energy are also effects. Davisson and Germer demonstrated this by diffracting
doubled. electrons from crystals. The law governing the diffraction from
Statement 2: The maximum kinetic energy of a crystal is obtained by requiring that electron waves reflected
photoelectrons emitted from a surface is linearly from the planes of atoms in a crystal interfere constructively
dependent on the frequency of the incident light. The (see figure).
photocurrent depends only on the intensity of the incident
light. [Online May 19, 2012]
(a) Statement 1 is true, Statement 2 is true, Statement
Inco m g
2 is the correct explanation of Statement 1. Electr ing i Outgoin s
ons c tro n
(b) Statement 1 is false, Statement 2 is true. Ele
(c) Statement 1 is true, Statement 2 is false. d
(d) Statement 1 is true, Statement 2 is true, Statement
2 is not the correct explanation of Statement 1.
57. This question has Statement – 1 and Statement – 2. Of the Crystal plane
four choices given after the statements, choose the one 60. Electrons accelerated by potential V are diffracted from a
that best describes the two statements. [2011] crystal. If d = 1Å and i = 30°, V should be about [2008]
Statement – 1: A metallic surface is irradiated by a (h = 6.6 × 10–34 Js, me = 9.1 × 10–31 kg, e = 1.6 × 10–19 C)
monochromatic light of frequency v > v0 (the threshold (a) 2000 V (b) 50 V (c) 500 V (d) 1000 V
frequency). The maximum kinetic energy and the stopping
potential are Kmax and V0 respectively. If the frequency 61. If a strong diffraction peak is observed when electrons are
incident on the surface is doubled, both the Kmax and V0 incident at an angle ‘i’ from the normal to the crystal planes
are also doubled. with distance ‘d’ between them (see figure), de Broglie
Statement – 2 : The maximum kinetic energy and the wavelength ldB of electrons can be calculated by the
stopping potential of photoelectrons emitted from a relationship ( n is an integer) [2008]
surface are linearly dependent on the frequency of incident (a) d sin i = nldB (b) 2d cos i = nldB
light.
(c) 2d sin i = nldB (d) d cos i = nldB
Dual Nature of Radiation and Matter P-439

62. Photon of frequency n has a momentum associated with (c) increase by a factor of 2
it. If c is the velocity of light, the momentum is [2007] (d) decrease by a factor of 2
(a) hn / c (b) n /c (c) h n c (d) hn / c2 67. A radiation of energy E falls normally on a perfectly
63. The threshold frequency for a metallic surface corresponds reflecting surface. The momentum transferred to the surface
to an energy of 6.2 eV and the stopping potential for a is [2004]
radiation incident on this surface is 5 V. The incident (a) Ec (b) 2 E / c (c) E / c (d) E / c 2
radiation lies in [2006] 68. According to Einstein’s photoelectric equation, the plot
(a) ultra-violet region (b) infra-red region of the kinetic energy of the emitted photo electrons from a
(c) visible region (d) X-ray region metal Versus the frequency, of the incident radiation gives
64. The time taken by a photoelectron to come out after the a straight line whose slope [2004]
photon strikes is approximately [2006] (a) depends both on the intensity of the radiation and
(a) 10–4 s (b) 10–10 s (c) 10–16 s (d) 10–1 s the metal used
65. The anode voltage of a photocell is kept fixed. The (b) depends on the intensity of the radiation
wavelength l of the light falling on the cathode is gradually (c) depends on the nature of the metal used
changed. The plate current I of the photocell varies as (d) is the same for the all metals and independent of the
follows [2006] intensity of the radiation
69. The work function of a substance is 4.0 eV. The longest
wavelength of light that can cause photoelectron emission
from this substance is approximately [2004]
(a) I (b) I
(a) 310 nm (b) 400 nm (c) 540 nm (d) 220 nm
70. Two identical photocathodes receive light of frequencies
f1 and f2. If the velocites of the photo electrons (of mass
O l O l
m) coming out are respectively v1 and v2, then [2003]
2h
(a) v12 - v2 2 = ( f - f2 )
m 1
I I 1/ 2
é 2h ù
(c) (d) (b) v1 + v2 = ê ( f1 + f 2 )ú
ëm û
O l
(c) v 2 + v 2 = 2 h ( f + f )
O l
1 2
66. A photocell is illuminated by a small bright source placed m 1 2
1/ 2
1 é 2h ù
1 m away. When the same source of light is placed m (d) v1 - v2 = ê ( f1 - f 2 )ú
2 ëm û
away, the number of electrons emitted by photocathode 71. Sodium and copper have work functions 2.3 eV and 4.5 eV
would [2005] respectively. Then the ratio of the wavelengths is nearest
(a) increase by a factor of 4 to [2002]
(b) decrease by a factor of 4 (a) 1 : 2 (b) 4 : 1 (c) 2 : 1 (d) 1 : 4
P-440 Physics

h h h h 10h
1. (c) de-Broglie wavelength, l = = lf = = Þ lf =
P 2 m(KE) mAV0 m V0 mV0
´
1 2 5
\l µ
m 10h 2h
\ Dl = l f - l0 = -
As mHe++ > mP > me mV0 mV0

l He++ > l P > l e or l e > l P > l He++ 8h 2h


Þ Dl = Þ Dl = 4 ´
mv0 mv0
3kT
2. (a) Rms speed of gas molecule, Vrms = \Dl = 4l 0
m
4. (d) de Broglie wavelength
h h h h
de Broglie wavelength, l = = l= Þm=
p 2mk mv lv

h h h 1
\l = = = Clearly, m µ
1 3 3mkT lv
2
2 m ´ mVrms m ´ kT If l and v be the wavelength and velocity of electron and
2 2
l' and v' be the wavelength and velocity of the particle
Substituting the respective values we get then
6.63 ´ 10-34 m' vl 1 1
l= = 0.24Å Þ = = ´ ´ 10 -4
m v ' l ' 5 1.878
3 ´ 4.64 ´ 10 -26 ´ 1.38 ´ 10 -13 ´ 400
3. (d) Þ m = 9.7 ´ 10 -28 kg

(m/2) (m/3) (m/2) 5. (c) As per question, when KE of particle E, wavelength l


VB
V0 and when KE becomes E + DE wavelength becomes l/2
A B (rest) (A) VA (B) (m/3) h
Before collision After collision Using, l =
2mKE
Applying momentum conservation
l h
m m m m =
´ V0 + ´ (0) = VA + VB 2 2m ( KE + DE )
2 3 2 3

V0 VA VB l KE + DE
= = + ...(i) Þ =
2 2 3 l/2 KE
Since, collision is elastic KE + DE
Þ4=
VB - V A KE
e =1 = Þ V0 = VB - VA ...(ii) Þ 4KE – KE = DE
V0
\ DE = 3 KE = 3 E
V0
On solving equations (i) and (ii) : VA = eE
5 6. (d) Acceleration of electron in electric field, a =
m
Now, de-Broglie wavelength of A before collision : Using equation
h h 2h v = u + at
l0 = = Þ l0 =
mAV0 æ m ö mV0 eE
ç ÷ V0 Þ v =0+ t
è2ø m
Final de-Broglie wavelength :
Dual Nature of Radiation and Matter P-441

8. (b) P1 – P2 = (P1 + P2) = P


eEt
Þ v= ..... (i)
m 1
As P µ
De-broglie wavelength l is given by l

h h 1 1 1
l= = [using (i)] or - =
mv æ eEt ö lx l y l
mç ÷
è m ø
l y - lx 1
h or =
Þ l= lxl y l
eEt
Differentiating w.r.t. t 9. (a) From the de-Broglie relation,
h
æ h ö p1 =
dç ÷ dl -h l1
dl eEt ø Þ =
= è dt eEt 2
dt dt h
p2 =
l2
7. (c) Given, Initial velocity, u = v0iˆ + v0 ˆj
Momentum of the final particle (pf) is given by
qE0 eE0
Acceleration, a = = \ pf = p12 + p22
m m
Using v = u + at
h h2 h 2
eE Þ = +
v = v0 iˆ + v0 ˆj + 0 tkˆ l l12 l 22
m

2 1 1 1
r æ eE t ö Þ = +
\ | v | = 2v02 + ç 0 ÷ l 2
l12 l 22
è m ø
10. (c) de Broglie wavelength (l) is given by
h K = qV
de-Broglie wavelength, l = p
l=
h
p
=
h
2mK
=
h
2mqV
Q p = 2mK ( )
h
Þl= (Q p = mv)
mv Substituting the values we get

h lA 2m B q B VB 4m.q.2500
\ = =
Initial wavelength, l 0 = lB 2m A q A VA m.q.50
mv0 2
Final wavelength, = 2 50 = 2 ´ 7.07 = 14.14
h 11. (d) de-Broglie wavelength,
l=
2
m 2v 2 æ eE t ö
0 +ç 0 ÷ h æ 3 ´ 108 ö é cù
= 10-3 ç
è m ø l=
mv ÷
è 6 ´ 1014 ø êëQ l = v úû

l 1
= 6.63 ´ 10-34 ´ 6 ´ 1014
l0 2 v=
æ eE0 t ö 9.1 ´ 10-31 ´ 3 ´ 105
1+ ç ÷÷
ç 2mv
è 0 ø v = 1.45 × 106 m/s

l0 h
Þl= 12. (d) Using, l = {given: l = 7.5 × 10–12}
e 2 E02t 2 p
1+
2m 2 v02 h
ÞP=
l
P-442 Physics

Minimum energy required, m 4


´ u
–34 ü 2
ìï 6.6 ´ 10 ï l A PB 2 3 =2
\ = =
í –12 ý l B PA 4
P2 ( h / l )
2 m´
îï 7.5 ´10 þï 3
KE = = = J = 25keV
2m 2m 2 ´ 9.1 ´ 10 –31 16. (a) From Bragg’s equation
d sin q = l
h ˆ h ˆ
13. (c) Momentum (p) of each electron i and j l
l1 l2 sin q = <1 \ l<d
d
Velocity of centre of mass
h é h ù
h ˆ h ˆ <d
Vcm = i+ j (Q p = mv) | py | êQ l = ú
2ml1 2ml 2 êë | p y | úû
Velocity of 1st particle about centre of mass \ h < | py | d
h ˆ h ˆ 17. (c) de-Broglie wavelength,
V1cm = i- j
2ml1 2ml 2 h h h
l= = =
P mv 2mqV
h 2l1l 2 æ hö
l cm = = çèQ l = p ÷ø 6.6 ´ 10 –34
h2 h2 l12 + l 22 or, l =
+ 2
2
4l1 4l 2 2 ´ 9.1 ´ 10 –31 ´ 1.6 ´10 –19 ´ 50
= 1.7 Å
14. (c) According to question, lp = la
18. (d) Among the given par ticles most difficult to
h h experimentally verify the de-broglie relationship is for a
Using, l = =
p mv dust particle.
19. (d) Energy in joule (E)
h h = charge × potential diff. in volt
So, =
m p ´ v p ma ´ va Eelectron = qeV and Eproton = qp 4V

vp h h
ma 4m p de-Broglie wavelength l = =
Þ = = P 2mE
va mp mp
(Q mass of a-particle is 4 times of mass of proton) h h
le = lP =
vp 4 2me eV and 2mP e4V (Q qe = qP)
So, = ; i.e., 4 :1
va 1 h
m le 2m e eV 2m P e4V mP
15. (d) From question, mA = M; mB = \ = = =2
2 lP h 2me eV me
uA = V uB = 0 2m P e4V
Let after collision velocity of A = V1 and
velocity of B = V2 20. (d) de-Broglie wavelength,
Applying law of conservation of momentum, h h
l= = .... (i)
æ mö p mv
mu = mv1 + ç ÷ v2
è2ø 1 2
but K.E = mv
or, 24 = 2v1 + v2 ....(i) 2
By law of collision
(mv )2
v -v Þ K.E =
e= 2 1 2m
u-0
or, u = v2 – v1 ....(ii) Þ mv = 2m K . E
[Q collision is elastic, e = 1] h
using eqns (i) and (ii) l=
2m K . E
4 4
v1 = and v 2 = u
3 3 1
\ lµ
h K. E
de-Broglie wavelength l =
p l
So, if K.E. is doubled, wavelength becomes
2
Dual Nature of Radiation and Matter P-443

21. (a) Covalent bonds are formed by sharing of electrons 25. (d) According to Einstein's photoelectric equation
with different compounds. Formation of covalent bond is
K max = hv - f0
best explained by molecular orbital theory.
22. (50) hc
Þ eVs = - f0
From Bragg's equation 2d sin q = l and de-Broglie l
h h hc f 0
wavelength, l = = Þ Vs = -
P 2mE le e
where l = wavelength of incident light
f0 = work function
Vs = stopping potential
d
Comparing the above equation with y = mx + c, we get
q q
hc
h slope =
2d sin q = l = e
2mE Increasing the frequency of incident radiation has no effect
on work function and frequency. So, graph will not change.
3 6.6 ´ 10 -34
Þ 2 ´ 10 -10 ´ = hc
2 2mE 26. (d) Using equation, = -f
l
[Q q = 60° and d = 1Å = 1 ´ 10 -10 m ]
hc hc
KEmax = -f= -f ...(1)
1 6.64 ´ 10 2 -48 l 500
\E = ´ ; 50 eV
2 9.1 ´ 10 -31 ´ 3 ´ 1.6 ´ 10 -19 hc
Again, 3 KEmax = -f ...(2)
23. 2 200
From the Einstein's photoelectric equation Dividing equation (2) by (1),
Energy of photon
hc
= Kinetic energy of photoelectrons + Work function 3KEmax 3 200 - f
Þ Kinetic energy = Energy of Photon – Work Function = =
KEmax 1 hc
Let f0 be the work function of metal and v1 and v2 be the -f
500
velocity of photoelectrons. Using Einstein's photoelectric
equation we have Putting the value of hc = 1237.5 and solving we get, work
function, f = 0.61 eV.
1 2 27. (a) Given,
mv1 = 4 - f 0 ...(i)
2
Wavelength of X-rays, l1 = 1 nm = 1 × 10 –9 m
1 2
mv2 = 2.5 - f0 ...(ii)
2 Wavelength of visible light, l 2 = 500 × 10 –9 m
1 2
mv1 4 - f0
The number of photons emitted per second from a source
Þ 2 = of monochromatic radiation of wavelength l and power P
1 2 2.5 - f0
mv2 is given as
2
4 - f0 P P Pl c
Þ (2)2 = Þ 10 - 4f 0 = 4 - f0 n= = = ( QE = hn and n = )
2.5 - f 0 E hn hc l

f0 = 2eV
Þ Clearly n µ l

24. (a) Graph of Vs and f given at B (5.5, 0) n1 l1 1


Þ = =
Minimum energy for ejection of electron n2 l 2 500
= Work function (f). 28. (9)
hV When radiation of wavelength A, lA is used to illuminate,
f = hV joule or f = eV (for V = 0) stopping potential VA = V
e
hc
6.62 ´ 10 -34 ´ 5.5 ´ 1014 = f + eV ...(i)
\f = -19
eV = 2.27 eV l
1.6 ´ 10
P-444 Physics

When radiation of wavelength B, lB is used to illuminate, On solving we get, TA = 2 eV


\ KEB = TA – 1.5 = 2 – 1.5 = 0.5 eV
V
stopping potential, VB = \ Work function of metal B is
4 fB = EB – KEB = 4.5 – 0.5 = 4 eV
hc eV 31. (11.00) Energy of proton
= f+ ...(ii)
3l 4 hc 1240
E= = = 4eV > 2eV [= f]
From eq. (i) – (ii), l 310
hc æ 1 ö 3 (so emission of photoelectron will take place)
ç1 - ÷ = eV = 4 × 1.6 × 10–19 = 6.4 × 10–19 joule
l è 3ø 4
6.4 ´ 10 –5 ´ 1
hc 2 3 8 hc N= = 1014
Þ = eV Þ eV = 4 ´ 6.4 ´ 10 –19
l 3 4 9 l
No. of photoelectrons emitted per second
hc 8 hc
=f+ 1014
l 9 l = = 1011 (Q 1 in 103 photons ejects an electron)
3
10
hc hc \ Value of X = 11.00
\f = = , so, n = 9.
9l nl 32. (b) f0 = 4 × 1014 Hz
29. (a) Using Einstein’s photoelectric equation, W0 = hf0 = 6.63 × 10–34 × (4 × 1014) J
E = w0 + KEmax
(6.63 ´ 10 -34 ) ´ (4 ´ 1014 )
Þ w0 = KEmax – E =
1.6 ´10-19
p2
p = 2mKE Þ KE = = 1.66 eV
2m 33. (a) KEmax= E – f0
p (where E = energy of incident light f0 = work function)
r= Þ p = reB
eB hc hc
= -
l l0
r 2 e2 B 2 12420
K max = KEmax = - w0
2m l é 1 1 ù
= 1237 ê - ú
ë 260 380 û
12420 r 2 eB 2
Þ w0 = - ( In eV ) =
1237 ´120
= 1.5eV
6561 2m
380 ´ 260

= 1.89 ( eV ) -
(10-4 )(1.6 ´10-19 ) 9 ´105 34. (a) Energy of photon (E) is given by
hc
2 ´ 9.07 ´10-31 E=
l

= 1.89 ( eV ) -
(10-4 )(1.6 ´10-19 ) 9 ´ 105 Number of photons of wavelength l emitted in t second
from laser of power P is given by
2 ´ 9.07 ´10-31 Pt l
= (1.89 – 0.79) eV= 1.1 eV n=
hc
30. (a) de-Broglie wavelength (l),
h 2´ l 2 ´ 10 -3 ´ 5 ´ 10-7
Þn=
Momentum, mv = = p = 2m ( KE ) hc
=
2 ´ 10-25
(Q t = 1S)
l
Þ n = 5 × 1015
h 1 35. (c) Here w = 2p ×6 × 1014 or f = 6 × 1014 Hz
\l = Þ lµ
2mKE KE
C 3 ´108
Wavelength l = = = 0.5 ´10-6 m = 5000Ao
lA KB TA –1.5 f 6 ´1014
\ = = (as given)
lB KA TA
12374
Now E = = 2.48 eV
lA 1 5000
Also, l = 2 Using E = w + eVs
B
2.48 = 2 + eVs or Vs = 0.48 V
Dual Nature of Radiation and Matter P-445

36. (BONUS)
4 hc hc 1 æ 4 1ö
Þ - = 3f Þ f = hc ç - ÷
12375 l 2 l1 3 è l 2 l1 ø
37. (d) Using, wavelength, l =
DE
1 æ 4 ´ 350 - 540 ö
= ´ 1240 ç
or, l =
12375
; 250nm 3 è 350 ´ 540 ÷ø = 1.8 eV
4.9
38. (c) Let f = work function of the metal, 41. (d) According to question, there are two EM waves with
different frequency,
hc B1 = B0 sin (p × 107c)t
=f+ eV1 ......(i)
l1 and B2 = B0 sin (2p × 107c)t
To get maximum kinetic energy we take the photon with
higher frequency
hc
=f+ eV2 ......(ii) w
l2 using, B = B0 sin wt and w = 2 pv Þv =
2p
Sutracting (ii) from (i) we get 10 7
B1 = B0sin (p × 107c)t Þ v1 = ´c
æ1 1ö 2
hc ç – ÷ = e(V1 – V2 ) B2 = B0sin (2p × 107c)t Þ v2 = 107c
è l1 l 2 ø where c is speed of light c = 3 × 108 m/s
é ù Clearly, v2 > v1
ê ll = 300nm ú so KE of photoelectron will be maximum for photon of
ê ú higher energy.
hc æ l – l1 ö ê l 2 = 400nm ú v2= 107c Hz
Þ V1 – V2 = ç 2
e è l1·l 2 ÷ø ê hc ú hv = f + KEmax
ê =1240nm – V ú
ëe û energy of photon
Eph = hv = 6.6 × 10–34 × 107 × 3 × 109
æ 100nm ö Eph = 6.6 × 3× 10–19J
= (1240 nm – v) ç ÷ø
è 300 nm ´ 400nm
6.6 ´ 3´10 –19
= 1.03 V » 1V = eV =12.375eV
1.6 ´10 –19
nE KEmax = Eph–f
39. (c) using, intensity I =
At
= 12.375 – 4.7 = 7.675 eV » 7.7 eV
n = no. of photoelectrons
hc
42. (c) In X-ray tube, l min =
æ n ö 10 ´1.6 ´10
–19
n eV
Þ16´10 –3 = ç ÷ ´ or, = 1012
ètø 10–4 t æ hc ö
In l min = In ç ÷ - InV
è eø
So, effective number of photoelectrons ejected per unit
time = 1012 × 10/100 = 1011 Clearly, log lmin versus log V graph
40. (a) From Einstein’s photoelectric equation, slope is negative hence option (c) correctly depicts.
43. (a) Given, l = 660 nm, Power = 0.5 kW, t = 60 ms
hc 1
f + m ( 2v)
2
nhc plt
l1 = 2
....(i)
Power P = Þn=
lt hc
hc 1 2 660 ´ 10-9 ´ 60 ´ 10-3
and l = f + mv ....(ii) = 0.5 ´ 103 ´
2 2 6.6 ´ 10 -34 ´ 3 ´ 108
As per question, maximum speed of photoelectrons in = 100 × 1018 = 1020
two cases differ by a factor 2 44. (c) As the metal surface is same, work function (f) is
same for both the case.
From eqn. (i) & (ii)
Initially KEmax = nh – f ..... (i)
hc After increase
-f KE'max = 3 nh – f .... (ii)
l1 hc 4hc
Þ = 4Þ -f= - 4f
hc l1 l2 For work function f – not to be –ve or zero, v' > 3v
-f
l2
P-446 Physics

45. (c)
c
h
1
– hn0 = mv
2 3.6 ´ 10-3
2 =
l 2
4 hc 1 = 1.8 × 10–3 Wm–2
\ - hn0 = mv¢ 2
3 l 2 work function f = hv
4 4 hc
n - n0 n - n0 =
v¢2 l
\ = 3 \ v¢ = v 3
v2 n - n0 n - n0
=
( 6.62 ´ 10-34 )(3 ´108 )
4 l
\ v¢ > v
3 3
12.4 ´ 10
46. (a) From Einstein's photoelectric equation = ev
l
hc for different wavelengths
K . E .l = - f ...(i)
l
(for monochromatic light of wavelength l) 12.4 ´ 103 12.4 ´ 103
f1 = = = 2.493 eV = 3.984 × 10–19 J
where f is work function l1 4972
hc
K . E .l /2 = -f ...(ii) 12.4 ´ 103 12.4 ´ 103
l/2 f2 = = = 1.994 eV = 3.184 × 10–19
(for monochromatic light of wavelength l/2) l2 6216
From question, J
hc æ hc ö Work function for metallic surface f = 2.3 eV (given)
K . E.l /2 = 3( K . E .l ) Þ - f = 3ç - f÷ f2 < f
l/2 èl ø
Therefore, f2 will not contribute in this process.
2hc hc Now, no. of electrons per m2-s = no. of photons per m2-s
- f = 3 - 3f
l l
1.8 ´ 10-3
hc hc no. of electrons per m2-s = ´ 10-4
-19
Þ 2f = \ f= 3.984 ´ 10
l 2l
47. (None)
From Einstein's photoelectric equation, we have
(Q1 cm 2 -4
= 10 m 2
) = 0.45 × 10 12

So, the number of photo electrons liberated in 2 sec.


hc hc
= + eV ...(1) = 0.45 × 1012 × 2
l1 l 0 = 9 × 1011
hc hc 50. (b)
= + eV ...(2) 51. (d) As l is increased, there will be a value of l above
l 2 l0
which photoelectrons will be cease to come out so
hc hc photocurrent will become zero. Hence (d) is correct answer.
= + 3eV ' ...(3)
l3 l 0 hc
From equation (1) & (2) 52. (d) - f = eV0
l
3 2 1
- = hc f
2l1 2l 2 l 0 v0 = -
el e
hc é 3 1 ù For metal A For metal B
- hc ê - ú = eV '
l1 ë 2l1 2l 2 û fA 1 fB 1
= =
hc l hc l
hc é 1 3 1 ù
ê - + ú=V' 1
e ë l3 2l1 2l 2 û
As the value of (increasing and decreasing) is not
48. (a) Frank-Hertz experiment - Discrete energy levels of l
specified hence we cannot say that which metal has
atom, Photoelectric effect - Particle nature of light. comparatively greater or lesser work function (f).
Davison - Germer experiment - wave nature of electron.
53. (a)
49. (b) Given, l1 = 4972Å 54. (a) Davisson Germer experiment showed that electron
and l 2 = 6216Å beams can undergo diffraction when passed through
atomic crystal. This established wave nature of electron
and I = 3.6 ´ 10 -3 Wm -2 as waves can exhibit interference and diffraction.
Intensity associated with each wavelength
Dual Nature of Radiation and Matter P-447

55. (c) According to photo-electric equation : h h h


K.Emax = hv – hv0 (Work function) l= = =
p 2mK.E 2meV
Some sort of energy is used in ejecting the
photoelectrons. nh
56. (b) The maximum kinetic energy of photoelectrons
\ = 2d cos i
2meV
depends upon frequency on incident light and photo Squaring both side
current depends upon intensity of incident light.
57. (c) By Einstein photoelectric equation, n 2h 2
= 4d 2 cos2 i
Kmax = eV0 = hv – hv0 2meV
For first order interference n = 1
When v is doubled, Kmax and V0 become more than double.
2
58. (d) We know that h
\V =
eV0 = Kmax = hn – f 8med 2 cos 2 i
where, f is the work function. (6.6 ´ 10 -34 )2
=
X-rays have higher frequency (v) than ultraviolet rays. 8 ´ 9.1´ 10 -31
´1.6 ´10 -19 ´ (10 -10 ) 2 ´ cos 2 30
Therefore as v increases K.E and V0 both increases. = 50 V
The kinetic energy ranges from zero to maximum because 61. (b) For constructive interference,
of loss of energy due to subsequent collisions before
2d cos i = nldB
getting ejected.
62. (a) Energy of a photon of frequency n is given by
59. (a) Wavelength of incident light, l = 400 nm hc = 1240
E = hn .
eV.nm
Also, E = mc2, mc2 = hn
K.E = 1.68 eV
hn hn
Using Einstein’s photoelectric equation Þ mc = Þ p=
c c
hc 63. (a) Work function, f = 6.2 eV = 6.2 × 1.6 × 10–19 J
- W = K .E
l Stopping potential, V = 5 volt
hc From the Einstein’s photoelectric equation
ÞW= - K .E
l hc
- f = eV0
1240 l
ÞW= - 1.68 hc
400 Þl=
= 3.1 – 1.68 f + eV0
-34
= 1.41 eV 6.6 ´ 10 ´ 3 ´ 108 -7
= -19
m » 10
60. (b) The path difference between the rays APB and CQD 1.6 ´ 10 (6.2 + 5)
is This range lies in ultra violet range.
Dx = MQ + QN = d cos i + d cos i 64. (b) The photoelectric emission is an instantaneous
Dx = 2d cos i process without any apparent time lag. It is known that
A emission starts in the time of the order of 10–9 second. So,
B the approximate time taken by a photoelectron to come
C D out after the photon strikes is 10–10 second.
65. (b) As l decreases, y increases and hence the speed of
i photoelectron increases. The chances of photo electron
to meet the anode increases and hence photo electric
d P current increases.
M i N
2
Q I ær ö
I1 1
66. (a) I µ 2
; =ç 2÷ =
For constructive interference the path difference is integral r I 2 è r1 ø 4
multiple of wavelength
I 2 ® 4 times I1
\ nl = 2d cos i
When intensity becomes 4 times, no. of photoelectrons
From de-broglie concept emitted would increase by 4 times, since number of electrons
Wavelength, emitted per second is directly proportional to intensity.
P-448 Physics

E 1 2
67. (b) Momentum of photon of energy E is = hf1 - W = mv ....(i)
c 2 1
When a photon hits a perfectly reflecting surface, it reflects Using Einstein’s photo electric equation for another
black in opposite direction with same energy and photodiode we get,
momentum. 1 2
hf 2 - W = mv2 ....(ii)
E æ –E ö 2E 2
\ Change in momentum = –ç ÷ = Subtracting (ii) from (i) we get
C è C ø C
This is equal to momentum transferred to the surface. 1 2 1 2
(hf1 – W) – (hf2 – W) = mv1 - mv2
68. (d) From the Einstein photoelectric equation K.E. = hn – f 2 2
Here, f = work function of metal m 2 2
\ h ( f1 - f 2 ) = ( v1 - v2 )
h = Plank's constant 2
slope of graph of K.E. & n is h (Plank’s constant) which is 2 2 2h
\ v1 - v 2 = ( f1 - f 2 )
same for all metals. m
71. (c) We know that work function,
69. (a) Work function of metal (f) is given by
hC
hc E = hu =
f= l
l where
hc h = Planck’s constant
Þ l= C = velocity of light
f
l = wavelength of light
6.63 ´ 10-34 ´ 3 ´108 E l
Þ l= = 310 nm \ Na = Cu
4 ´ 1.6 ´ 10–19 ECu l Na
70. (a) Let work function be W and v1 and v2 be the velocity
l Na E Cu 4.5 2
of electrons for frequencies f1 and f 2. Þ = = »
l Cu E Na 2.3 1
Using Einstein’s photo electric equation for one
photodiode, we get
26
Atoms P-449

Atoms
4. An a-particle of energy 5 MeV is scattered through 180º by a
Atomic Structure and
TOPIC 1 fixed uranium nucleus. The distance of closest approach is of
Rutherford's Nuclear Model the order of [2004]
1. The graph which depicts the results of Rutherford gold (a) 10–12 cm (b) 10–10 cm
foil experiment with [8 Jan. 2020 I] (c) 10–14 cm (d) 10–15 cm
a-particles is:
q: Scattering angle
Y: Number of scattered a-particles detected
TOPIC 2 Bohr's Model and the Spectra
of the Hydrogen Atom
(Plots are schematic and not to scale)
5. A particle of mass 200 MeV/c2 collides with a hydrogen
atom at rest. Soon after the collision the particle comes
to rest, and the atom recoils and goes to its first excited
state. The initial kinetic energy of the particle (in eV)
(a) (b)
N
is . The value of N is : [NA Sep. 05, 2020 (I)]
4
(Given the mass of the hydrogen atom to be 1 GeV/c2)
6. In the line spectra of hydrogen atom, difference between
the largest and the shortest wavelengths of the Lyman
(c) (d) series is 304 Å. The corresponding difference for the
Paschan series in Å is : __________.
[NA Sep. 04, 2020 (I)]
2. In the Rutherford experiment, a-particles are scattered from 7. In a hydrogen atom the electron makes a transition from (n
a nucleus as shown. Out of the four paths, which path is + 1)th level to the nth level. If n >> 1, the frequency of
not possible? [Online May 7, 2012] radiation emitted is proportional to : [Sep. 02, 2020 (II)]
A 1 1
(a) (b)
n n3
B
1 1
(c) (d) 4
C n2 n
D 8. The energy required to ionise a hydrogen like ion in its
ground state is 9 Rydbergs. What is the wavelength of
(a) D (b) B (c) C (d) A the radiation emitted when the electron in this ion jumps
from the second excited state to the ground state?
1 2 [9 Jan. 2020 II]
3. An alpha nucleus of energy mv bombards a heavy
2 (a) 24.2 nm (b) 11.4 nm
nuclear target of charge Ze. Then the distance of closest (c) 35.8 nm (d) 8.6 nm
approach for the alpha nucleus will be proportional to 9. The first member of the Balmer series of hydrogen atom
[2006]
has a wavelength of 6561 Å. The wavelength of the
1 1 1 second member of the Balmer series (in nm) is ______.
(a) v 2 (b) (c) 2 (d) [NA 8 Jan. 2020 II]
m v Ze
P-450 Physics

10. The time period of revolution of electron in its ground 19. In a hydrogen like atom, when an electron jumps from the
state orbit in a hydrogen atom is 1.6 ´ 10–16 s. The M-shell to the L-shell, the wavelength of emitted radiation
frequency of revolution of the electron in its first excited is l. If an electron jumps from N-shell to the L-shell, the
state (in s–1) is: [7 Jan. 2020 I] wavelength of emitted radiation will be: [11 Jan 2019 II]
(a) 1.6 ´ 1014 (b) 7.8 ´ 1014 27 16 25 20
(a) l (b) l (c) l (d) l
(c) 6.2 ´ 1015 (d) 5.6 ´ 1012 20 25 16 27
11. An excited He+ ion emits two photons in succession, with 20. An electron from various excited states of hydrogen atom
wavelengths 108.5 nm and 30.4 nm, in making a transition emit radiation to come to the ground state. Let ln, lg be
to ground state. The quantum number n, corresponding the de Broglie wavelength of the electron in the nth state
to its initial excited state is (for photon of wavelength », and the ground state respectively. Let L n be the
wavelength of the emitted photon in the transition from
1240eV the nth state to the ground state. For large n, (A, B are
energy E = [12 April 2019 II]
l (innm) constants) [2018]
(a) n = 4 (b) n = 5 (c) n = 7 (d) n = 6 B
12. The electron in a hydrogen atom first jumps from the third (a) L n » A+ 2 (b) Ln » A + Bln
ln
excited state to the second excited state and subsequently
to the first excited state. The ratio of the respective (c) L 2n » A + B l 2n (d) L 2n » l
wavelengths, l1/ l2, of the photons emitted in this process
is : [12 April 2019 II] 21. If the series limit frequency of the Lyman series is v1, then
(a) 20/7 (b) 27/5 (c) 7/5 (d) 9/7 the series limit frequency of the P-fund series is :
13. Consider an electron in a hydrogen atom, revolving in its [2018]
second excited state (having radius 4.65 Å). The de-Broglie (a) 25 nL (b) 16 nL (c) nL/16 (d) nL/25
wavelength of this electron is : [12 April 2019 II] 22. The de-Broglie wavelength (lB) associated with the
(a) 3.5 Å (b) 6.6 Å (c) 12.9 Å (d) 9.7 Å electron orbiting in the second excited state of hydrogen
14. In Li++, electron in first Bohr orbit is excited to a level by a atom is related to that in the ground state (lG) by
radiation of wavelength l. When the ion gets deexcited [Online April 16, 2018]
to the ground state in all possible ways (including (a) lB = lG/3 (b) lB = lG/2
intermediate emissions), a total of six spectral lines are
(c) lB = 2lG (d) lB = 3lG
observed. What is the value of l ? [10 April 2019 II]
23. The energy required to remove the electron from a singly
(Given : h = 6.63×10–34 Js; c = 3 × 108 ms–1)
ionized Helium atom is 2.2 times the energy required to
(a) 11.4 nm (b) 9.4 nm (c) 12.3 nm (d) 10.8 nm
remove an electron from Helium atom. The total energy
15. Taking the wavelength of first Balmer line in hydrogen required to ionize the Helium atom completely is:
spectrum (n = 3 to n = 2) as 660 nm, the wavelength of the
[Online April 15, 2018]
2nd Balmer line (n = 4 to n = 2) will be; [9 April 2019 I]
(a) 889.2 nm (b) 488.9 nm (a) 20 eV (b) 79 eV (c) 109 eV (d) 34 eV
(c) 642.7 nm (d) 388.9 nm 24. Muon (m–1) is negatively charged ( q = e ) with a mass
16. A He+ ion is in its first excited state. Its ionization energy mm = 200me, where me is the mass of the electron and e is
is: [9 April 2019 II] the electronic charge. If m–1 is bound to a proton to form a
(a) 48.36 eV (b) 54.40 eV hydrogen like atom, identify the correct statements
(c) 13.60 eV (d) 6.04 eV [Online April 15, 2018]
17. Radiation coming from transitions n = 2 to n = 1 of (A) Radius of the muonic orbit is 200 times smaller than
hydrogen atoms fall on He+ ions in n = 1 and n = 2 states. that of the electron
The possible transition of helium ions as they absorb
1
energy from the radiation is : [8 April 2019 I] (B) the speed of the m–1 in the nth orbit is times
(a) n = 2 ® n = 3 (b) n = 1 ® n = 4 200
that of the election in the nth orbit
(c) n = 2 ® n = 5 (d) n = 2 ® n = 4
18. A hydrogen atom, initially in the ground state is excited (C) The lonization energy of muonic atom is 200 times
more than that of an hydrogen atom
by absorbing a photon of wavelength 980Å. The radius of
the atom in the excited state, in terms of Bohr radius a0, (D) The momentum of the muon in the nth orbit is 200
times more than that of the electron
will be: [11 Jan 2019 I]
(a) 25a0 (b) 9a0 (c) 16a0 (d) 4a0 (a) (A), (B), (D) (b) (B), (D)
(c) (C), (D) (d) (A), (C), (D)
Atoms P-451

25. Some energy levels of a molecule are shown in the figure. 31. If one were to apply Bohr model to a particle of mass ‘m’
The ratio of the wavelengths r = l1/l2, is given by and charge ‘q’ moving in a plane under the influence of a
[2017] magnetic field ‘B’, the energy of the charged particle in
the nth level will be : [Online April 10, 2015]

æ hqB ö æ hqB ö
(a) n çè ÷ (b) n çè ÷
2pm ø 8pm ø

æ hqB ö
(c) n æç
hqB ö
(d) n ç
è 4pm ÷ø è pm ÷ø
3 1 4 2 32. The radiation corresponding to 3 ® 2 transition of
(a) r = (b) r = (c) r = (d) r =
4 3 3 3 hydrogen atom falls on a metal surface to produce
26. The acceleration of an electron in the first orbit of the photoelectrons. These electrons are made to enter a
hydrogen atom (z = 1) is : [Online April 9, 2017] magnetic field of 3 × 10–4 T. If the radius of the largest
circular path followed by these electrons is 10.0 mm, the
h2 h2 work function of the metal is close to: [2014]
(a) (b)
p 2 m2 r 3 8p 2 m 2 r 3 (a) 1.8 eV (b) 1.1 eV
2 2
h h (c) 0.8 eV (d) 1.6 eV
(c) 2 2 3 (d) 2 3
4p m r 4 pm r 33. Hydrogen (1 H1), Deuterium ( 2
1H ), singly ionised Helium
27. According to Bohr’s theory, the time averaged magnetic
( He4 ) and doubly ionised lithium ( Li 6 )
+ ++
all have
field at the centre (i.e. nucleus) of a hydrogen atom due to 2 3

the motion of electrons in the nth orbit is proportional to : one electron around the nucleus. Consider an electron
(n = principal quantum number) [Online April 8, 2017] transition from n = 2 to n = 1. If the wavelengths of emitted
(a) n –4 (b) n –5 (c) n –3 (d) n –2 radiation are l1, l 2 , l3 and l 4 respectively then
28. A hydrogen atom makes a transition from n = 2 to n = 1 and approximately which one of the following is correct?
emits a photon. This photon strikes a doubly ionized lithium [2014]
atom (z = 3) in excited state and completely removes the
(a) 4l1 = 2l 2 = 2l 3 = l 4
orbiting electron. The least quantum number for the excited
state of the ion for the process is : (b) l1 = 2l 2 = 2l 3 = l 4
[Online April 9, 2016]
(c) l1 = l 2 = 4l3 = 9l4
(a) 2 (b) 4 (c) 5 (d) 3
29. As an electron makes a transition from an excited state to (d) l1 = 2l 2 = 3l3 = 4l4
the ground state of a hydrogen - like atom/ion : 34. Match List - I (Experiment performed) with List-II
[2015] (Phenomena discovered/associated) and select the correct
(a) kinetic energy decreases, potential energy increases option from the options given below the lists:
but total energy remains same [Online April 19, 2014]
(b) kinetic energy and total energy decrease but potential
energy increases List - I List - II
(c) its kinetic energy increases but potential energy and
total energy decrease (1) Davisson and Germer (i) Wave nature of
(d) kinetic energy, potential energy and total energy decrease experiment electrons
30. The de–Broglie wavelength associated with the electron (2) Millikan's oil drop (ii) Charge of an electron
in the n = 4 level is : [Online April 11, 2015] experiment
1 (3) Rutherford (iii) Quantisation of
(a) th of the de–Broglie wavelength of the electron experiment energy levels
4
in the ground state. (4) Franck-Hertz (iv) Existence of nucleus
(b) four times the de–Broglie wavelength of the electron experiment
in the ground state
(c) two times the de–Broglie wavelength of the electron (a) (1)-(i), (2)-(ii), (3)-(iii), (4)-(iv)
in the ground state (b) (1)-(i), (2)-(ii), (3)-(iv), (4)-(iii)
(d) half of the de–Broglie wavelength of the electron in (c) (1)-(iii), (2)-(iv), (3)-(i), (4)-(ii)
the ground state
(d) (1)-(iv), (2)-(iii), (3)-(ii), (4)-(i)
P-452 Physics

35. The binding energy of the electron in a hydrogen atom is 42. A diatomic molecule is made of two masses m1 and m2
13.6 eV, the energy required to remove the electron from which are separated by a distance r. If we calculate its
the first excited state of Li++ is: [Online April 9, 2014] rotational energy by applying Bohr's rule of angular
(a) 122.4 eV (b) 30.6 eV momentum quantization, its energy will be given by : (n is
(c) 13.6 eV (d) 3.4 eV an integer) [2012]
36. Ina hydrogen like atom electron make transition from an (m1 + m2 )2 n2 h 2 n2 h 2
energy level with quantum number n to another with (a) (b)
quantum number (n – 1). If n>>1, the frequency of 2m12 m22 r 2 2(m1 + m2 )r 2
radiation emitted is proportional to : [2013]
2n 2 h 2 (m1 + m2 )n2 h2
1 1 1 1 (c) (d)
(a) (b) (c) (d) (m1 + m2 )r 2 2m1m2 r 2
n n2 n3 n3
2 43. Which of the plots shown in the figure represents speed
37. A 12.5 eV electron beam is used to bombard gaseous (vn) of the electron in a hydrogen atom as a function of the
hydrogen at room temperature. It will emit : principal quantum number (n)? [Online May 26, 2012]
[Online April 25, 2013]
(a) 2 lines in the Lyman series and 1 line in the Balmar
series
(b) 3 lines in the Lyman series
(c) 1 line in the Lyman series and 2 lines in the Balmar series
(d) 3 lines in the Balmer series
38. In the Bohr’s model of hydrogen-like atom the force
between the nucleus and the electron is modified as

e2 æ 1 b ö
F= ç + ÷ , where b is a constant. For this atom,
4pe0 è r 2 r 3 ø
(a) B (b) D (c) C (d) A
the radius of the nth orbit in terms of the Bohr radius
44. A doubly ionised Li atom is excited from its ground state
æ e h2 ö (n = 1) to n = 3 state. The wavelengths of the spectral lines
ç a0 = 0 ÷ is : [Online April 23, 2013] are given by l32, l31 and l21. The ratio l32/l31 and
ç mp e 2 ÷
è ø l21/l31 are, respectively [Online May 12, 2012]
(a) rn = a0n – b (b) rn = a0n2 + b (a) 8.1, 0.67 (b) 8.1, 1.2
(c) rn = a0n – b
2 (d) rn = a0n + b (c) 6.4, 1.2 (d) 6.4, 0.67
39. Orbits of a particle moving in a circle are such that the 45. A hypothetical atom has only three energy levels. The
perimeter of the orbit equals an integer number of de- ground level has energy, E1 = – 8 eV. The two excited
Broglie wavelengths of the particle. For a charged states have energies, E2 = – 6 eV and E3 = – 2 eV. Then
particle moving in a plane perpendicular to a magnetic which of the following wavelengths will not be present in
field, the radius of the nth orbital will therefore be the emission spectrum of this atom?
proportional to : [Online April 22, 2013]
[Online May 12, 2012]
(a) n 2 (b) n (c) n 1/2 (d) n 1/4 (a) 207 nm (b) 465 nm
40. In the Bohr model an electron moves in a circular orbit around (c) 310 nm (d) 620 nm
the proton. Considering the orbiting electron to be a circular
current loop, the magnetic moment of the hydrogen atom, 46. The electron of a hydrogen atom makes a transition from
when the electron is in nth excited state, is : the (n + 1) th orbit to the nth orbit. For large n the
wavelength of the emitted radiation is proportional to
[Online April 9, 2013]
[Online May 7, 2012]
æ e n2 h ö æ e ö nh (a) n (b) n 3 (c) n 4 (d) n 2
(a) ç ÷ (b) ç ÷
ç 2m 2p ÷ è m ø 2p 47. Energy required for the electron excitation in Li++ from the
è ø
2
first to the third Bohr orbit is : [2011]
æ e ö nh æ e ön h (a) 36.3 eV (b) 108.8 eV
(c) ç ÷ (d) ç ÷
è 2m ø 2p è m ø 2p (c) 122.4 eV (d) 12.1 eV
41. Hydrogen atom is excited from ground state to another 48. The transition from the state n = 4 to n = 3 in a hydrogen
state with principal quantum number equal to 4. Then the
like atom results in ultraviolet radiation. Infrared
number of spectral lines in the emission spectra will be :
[2012] radiation will be obtained in the transition from : [2009]
(a) 2 (b) 3 (c) 5 (d) 6 (a) 3 ® 2 (b) 4 ® 2 (c) 5 ® 4 (d) 2 ® 1
Atoms P-453

49. Suppose an electron is attracted towards the origin by a n=4


n =3
k
force where ‘k’ is a constant and ‘r’ is the distance of
r n=2
the electron from the origin. By applying Bohr model to
this system, the radius of the nth orbital of the electron
is found to be ‘rn’ and the kinetic energy of the electron n =1
I II III IV
to be ‘Tn’. Then which of the following is true? [2008]
(a) IV (b) III (c) II (d) I
1 2
(a) Tn µ 2 , rn µ n 52. The wavelengths involved in the spectrum of deuterium
n
(b) Tn independent of n, rn µ n
1
( D)
2
1 are slightly different from that of hydrogen
(c) Tn µ , rn µ n spectrum, because [2003]
n
(a) the size of the two nuclei are different
1 2 (b) the nuclear forces are different in the two cases
(d) Tn µ 3 , rn µ n
n (c) the masses of the two nuclei are different
50. Which of the following transitions in hydrogen atoms emit (d) the attraction between the electron and the nucleus
photons of highest frequency? [2007] is differernt in the two cases
(a) n = 1 to n = 2 (b) n = 2 to n = 6 53. If 13.6 eV energy is required to ionize the hydrogen atom, then
(c) n = 6 to n = 2 (d) n = 2 to n = 1 the energy required to remove an electron from n = 2 is
[2002]
51. The diagram shows the energy levels for an electron in a
certain atom. Which transition shown represents the (a) 10.2 eV (b) 0 eV
emission of a photon with the most energy? [2005] (c) 3.4 eV (d) 6.8 eV
P-454 Physics

1. (c) 6. (10553.14)
2. (c) As a-particles are doubly ionised helium He++ i.e. From Bohr's formula for hydrogen atom,
Positively charged and nucleus is also positively charged
and we know that like charges repel each other. 1 æ 1 1 ö
3. (b) Work done to stop the a particle is equal to K.E. = Rç 2 - 2 ÷
l ç ÷
è n1 n2 ø
1 2 K (Ze) 1 2
\ qV = mv Þ q ´ = mv
2 r 2 R = 1.097 ´ 107 m –1

2(2e) K ( Ze) 4 KZe


2 For Lyman series :
Þr= 2
= 2
mv mv 1
= R (1) = R Q n2 = ¥ and n1 = 1
l min .
1 1
Þrµ
2
and r µ .
v m 1 ì 1 ü 3R
= R í1 - ý = Q n1 = 2, n1 = 1
4. (a) Distance of closest approach l max. î 4þ 4
Ze(2e)
r0 = 4 1 1
æ 1 2ö \ l max. - l min. = - = = 304 (Given)
4pe 0 ç mv ÷ 3R R 3R
è2 ø
For Paschen series :
Energy, E = 5 ´ 106 ´ 1.6 ´ 10-19 J
æ 1ö æ1 1 ö 7R
9 -19 -19 l 'min. = R ç ÷ and l 'max. = R ç - ÷ =
\ r0 = 9 ´ 10 ´ (92 ´ 1.6 ´ 10 ) (2 ´ 1.6 ´ 10 ) è 9ø è 9 16 ø 16 ´ 9
5 ´ 106 ´ 1.6 ´ 10-19
16 ´ 9 9 81
Þ rh = 5.2 ´ 10
- 14
m = 5.3 × 10–12 cm l 'max. - l min.
'
= - =
7R R 7R

5. (51) 81 81 ´ 3 81 ´ 3
or, l 'max. - l 'min. = = = ´ 304
Before collision After collision 7 R 7 ´ 3R 7
Rest Rest æ 1 ö
v v' çèQ = 304Å÷
ø
m 5m m 5m 3R

Particle Hydrogen Particle Hydrogen \ For Pachen series, l 'max. - l min.


'
= 10553.14
From linear momentum conservation, Li = Lf 7. (b) Total energy of electron in nth orbit of hydrogen atom
v Rhc
mV + 0 = 0 + 5mV ' Þ V ' = En = -
5 n2
2
Total energy of electron in (n + 1)th level of hydrogen atom
1 2 1 æ vö
Loss of KE = KEi - KE f = mv - (5m) ç ÷ Rhc
2 2 è 5ø En +1 = -
( n + 1)2
1 2 æ 1 ö 4 æ mv 2 ö When electron makes a transition from (n + 1)th level to nth
= mv ç1 - ÷ = ç
2 è 5 ø 5 è 2 ÷ø level
Change in energy,
4 DE = En +1 - En
=
KEi = 10.2 eV
5
é1 1 ù
[Q Energy in first excited state of atom = 10.2 eV] hn = Rhc × ê 2 - ú (Q E = hn)
ën (n + 1)2 û
N
KEi = 12.75eV= Þ N = 51
4 é (n + 1) 2 - n 2 ù
n = R×c ê 2 2 ú
The value of N = 51. ë n (n + 1) û
Atoms P-455

11. (b) E = E1 + E2
é 1 + 2n ù
n = R×c ê 2 2ú z 2 1240 1240
ë n (n + 1) û 13.6 2 = +
n l1 l2
For n > > 1
é 2n ù 2 RC 13.6(2) 2 æ 1 1 ö 1
Þ n = R×c ê 2 = 3 or = 1240 ç + ÷ø ´ -9
ë n ´ n 2 úû n n 2 è 108.5 30.4 10
On solving, n = 5
1
Þnµ 1 æ 1 1ö
n3 12. (a) = R ç - ÷ = 7R
8. (b) According to Bohr's Theory the wavelength of the l1 è 32 42 ø 16 ´ 9
radiation emitted from hydrogen atom is given by
1 æ 1 1 ö 5R
= Rç -
1 é1 1ù
And
l2 è 22 32 ÷ø = 36
= RZ 2 ê 2 – 2 ú
l ëê n1 n2 úû l1 (5R / 36) 20
Q Z= 3 Now l = =
2 7 R / (16 ´ 9) 7
1 æ 1ö
\ = 9 R ç1 – ÷ c c
l è 9ø 13. (d) v = =
137 n 137 ´ 3
1 1
Þ l= = (R = 10973731.6 m–1) h h h h
8 R 8 ´ 10973731.6 l = p = mv = æ m ´ c ö = mc ´ (3 ´ 137) = 9.7 Å
Þ l = 11.39 nm çè ÷
9. (486.00) 3 ´ 137 ø
The wavelength of the spectral line of hydrogen spectrum 14. (d) Spectral lines obtained on account of transition from
is given by formula
n(n - 1)
æ 1 ö nth orbit to various lower orbits is
1 1 2
= Rç – ÷
l çè n2f ni2 ÷ø n(n - 1)
Þ6=
Where, R = Rydberg constant 2
For the first member of Balmer series nF = 2, ni = 3 Þn=4
1 æ 1 1ö hc –Z 2
\ =Rç 2 – 2÷ ...(i) DE = = 2 (13.6eV )
l è2 3 ø l n
For last member of Balmer series, nf = 2, ni = 4
1 é1 1 ù 1 æ 13.6eV ö æ 1 1ö
=Rê – ú Þ = Z2 ç ÷ ç 2 – 2÷
So,
l'
...(ii) l è hc ø è n2 n1 ø
ë 4 16 û
Dividing (i) by (ii), we get
é 1ù
l¢ 5 ´ 16 = (13.4)(3)2 ê1 - ú eV
Þ = ë 16 û
l 9´ 4´3
1242 ´ 16
5 ´ 4 ´ 656.1 Þl= nm ; 10.8nm
Þ l' = (nm) = 486 nm (13.4) ´ (9)(15)
9´3
10. (b) For first excited state n' = 3 1 æ 1 1 ö 5R
= -R ç - =
n 3
15. (b)
l1 è 2 2 32 ÷ø 36
Time period T µ 2
z 1 æ 1 1 ö 3R
= Rç - =
T2 n ' 3 l2 è 22 42 ÷ø 16
Þ =
T1 n3 l 2 80
\ =
\ T2 = 8T1 = 8 × 1.6 × 10–16s l1 108
1 1
\ Frequency, v = T = 80 80
8 ´ 1.6 ´ 10 –16 l2 = l1 = ´ 660 = 488.9 nm.
2 108 108
» 7.8 × 1014 Hz
P-456 Physics

Since n is very large, using binomial theorem


Z2
16. (c) En = -13.6 2 1 æ 1 ö
n Ln = ç1 + ÷
RZ 2 è n 2 ø
+ -13.6(2) 2
For He , E2 = = -13.60eV 1 1 æ 1 ö
22 Ln = 2
+ ç ÷
RZ RZ 2 è n 2 ø
Ionization energy = 0 – E2 = 13.60 eV
17. (d) Energy released by hydrogen atom for transition 2 pr æ n 2h 2 ö 1
n = 2 to n = 1 As we know, l n = n = 2p çç 2 2 ÷÷
µn
è 4 p mZe ø n
æ1 1ö 3
\ DE1 = 13.6 ´ ç 2 - 2 ÷ = ´ 13.6 eV B
è1 2 ø 4 Ln » A +
l n2
= 10.2 eV
This energy is absorbed by He+ ion in transition from 21. (d) hnL = E ¥ - E1 ...(i)
n = 2 to n = n1 (say) hnf = E ¥ - E5 ...(ii)
æ1 1 ö 2
\ DE2 = 13.6 ´ 4 ´ ç - 2 ÷ = 10.2 eV z2 E5 æ 1 ö 1
ç4 n ÷ Eµ Þ =ç ÷ =
è 1 ø n2 E1 è 5 ø 25
Þ n1 = 4
hn L E1
So, possible transition is n = 2 ® n = 4 Eqn (i) / (ii) Þ =
hn f E 5
hc 12500
18. (3) Energy of photon = = = 12.75eV n L 25 n
l 980 Þ = Þ nf = L
Energy of electron in n th orbit is given by nf 1 25
h
–13.6 é 1 –1 ù 22. (d) de-Broglie wavelength, l =
En = Þ E n – E1 = –13.6 ê 2 2 ú P
n2 ën 1 û
l B Pa mvG
= =
é 1 –1 ù l G PB mv B
Þ12.75 = 13.6 ê 2 2 ú Þ n = 4
ë1 n û z
Speed of electron V µ
\ Electron will excite to n = 4 n
We know that ‘R’ µ n2
lB n B 3
\ Radius of atom will be 16a0 so = = Þ l B = 3l G
lG nG 1
19. (d) When electron jumps from M ® L shell
23. (b) Energy required to remove e– from singly ionized
1 æ 1 1 ö K´5
= Kç 2 - 2 ÷ = ..... (i)
l è2 3 ø 36 (13.6)Z 2
helium atom = = 54.4 eV (Q Z = 2)
When eletron jumps from N ® L shell 12
Energy required to remove e– from helium atom = x eV
1 æ 1 1 ö K´3 According to question, 54.4 eV = 2.2x Þ x = 24.73 eV
=Kç 2 - 2 ÷ = ....(ii)
l' è2 4 ø 16 Therefore, energy required to ionize helium atom
solving equation (i) and (ii) we get = (54.4 + 24.73) eV = 79.12 eV

20 Radius of hydrogen
l' = l 24. (d) (A) Radius of muon =
27 200
20. (a) Wavelength of emitted photon from nth state to the Î0 n2h2
ground state, Radius of H atom = r =
pme2
1 æ1 1 ö
= RZ 2 ç 2 - 2 ÷ Î0 n 2 h 2
Ln è1 n ø Radius of muon = rm =
p ´ 200 me2
-1
1 æ 1 ö r
Ln = 2ç
1- 2 ÷ rm =
RZ è n ø 200
Atoms P-457

(B) Velocity relation given is wrong 27. (d) Magnetic field at the centre of nucleus of H-atom,
(C) Ionization energy in e– H atom m0 I
B= ..... (i)
+ me 4 2r
E= According to Bohr's model, radius of orbit r µ n 2
8 Î20 n 2 h2
from eq. (i) we can also write as B µ n–2
28. (b) A hydrogen atom makes a transition from n = 2 to
200me 4 n=1
Em = = 200 E
8 Î02 n 2 h 2 n2
(D) Momentum of H-atom
nh
mvr = n1
2p
Hence (A), (C), (D) are correct.
é 1 ù 2é 1ù
Then wavelength = Rcz êê 2 , 2 úú < Rc(1) êê1, úú
2 1
hc
25. (b) From energy level diagram, using DE = ëê n1 n 2 ûú ë 4û
l

hc é 3ù
For wavelength l1 DE = – E – (–2E) = l < Rc ê ú ...(1)
l1 êë 4 úû
For ionized lithium
hc
\ l1 = é 1 ù é 1ù
E l < Rc(3) 2 ê ú < Rc 9 ê ú ...(2)
êë n úû
2 êë n 2 úû
æ 4E ö hc
For wavelength l2 DE = – E – ç - ÷ = é 3ù é 1 ù
è 3 ø l2 Rc ê ú = Rc9 ê 2 ú
4
ë û ën û
hc l1 1
\ l2 = \r= = 3 9
æ Eö l2 3 Þ < Þ n < 12 < 2 3
çè ÷ø 4 n2
3
\ The least quantum number must be 4.
26. (c) Speed of electron in first orbit (n = 1) of hydrogen 29. (c) Kinetic energy of electron is
atom (z = 1),
2
æZö
e 2 K.E. µ ç ÷
v= èNø
2e 0 h When the electron makes transition from excited state to
radius of Bohr's first orbit, ground state, then n increases and kinetic energy increases.
Total energy = – KE
h 2e 0 rpme2 \ Total energy also decreases.
r= Þ e0 = ....... (i)
pme 2 h2 Potential energy is lowest for ground state.
Acceleration of electron, h
30. (b) De-Broglie wavelength of electron l =
mV
v2 e4 pme 2
= 2 2´ 2 1
r 4e 0 h h e0 As we know, V µ
n
e 4 ´ pme 2 So, lµn
= ....... (ii)
4h 4 e30 l 4 = 4l1
l1 is the de-Broglie wavelength of the electron in the
eliminating e0 from eq (ii),
ground state.
e4 pme2 h 6 mv 2
= 4 3 3 3 6 from eqn (i) 31. (c) qVB = ....(i)
4h r p m e r
nh
h2 = mvr ....(ii)
= 2 2 3 2p
4p m r
P-458 Physics

Multiplying equation (i) and (ii),


1 æ 1 1 ö
qBnh = Rç - ÷
= m2v 2 l çn 2 n 2 ÷
2p è 1 2 ø
(where Rydberg constant , R = 1.097 × 107)
1
Now multiplying both sides by ,
2m 1 æ1 1 ö
or, = 1.097 ´ 107 ç - ÷
993 ´10 -10 ç 12 n 2 ÷
qBh 1 2 è 2 ø
n = mv
4pm 2 Solving we get n 2 = 3
Spectral lines
é qBh ù Total number of spectral lines = 3
i.e. KE = n ê ú
ë 4pm û Two lines in Lyman series for n 1 = 1, n2 = 2 and n1 = 1,
32. (b) Radius of circular path followed by electron is given by, n2 = 3 and one in Balmer series for n 1 = 2 , n2 = 3
n=3
mu 2meV 1 2m Balmer
r= = = V n=2
qB eB B e Lyman Lyman
n=1
B2 r 2 e
Þ V= = 0.8V
2m mv2 e2 æ 1 b ö
38. (c) As F = = ç + ÷
For transition between 3 to 2. r 4p Î0 è r 2 r 3 ø
æ 1 1 ö 13.6 ´ 5 nh nh
E = 13.6 ç - ÷ = = 1.88eV and mvr = Þv=
è 4 9ø 36 2p 2 pmr
Work function = 1.88 eV – 0.8 eV = 1.08 eV » 1.1eV 2
æ nh ö 1 e2 æ 1 b ö
\ mç ÷ ´ = ç + ÷
1 é1 1ù è 2pmr ø r 4p Î0 è r 2 r 3 ø
33. (c) Wave number = RZ 2 ê 2 - 2 ú
l êë n1 n úû
1 b mn 2 h 2 4p Î0
1 or, + =
Þ lµ r2 r3 4p 2 m 2 e 2 r 3
Z2
\ lZ2 = constant
a n2 1 b æ Î h2 ö
By question n = 1 and n1 = 2 or, 0 = + çQa 0 = 0 Given ÷
Then, l1 = l2 = 4l3 = 9l4 r3 r 2 r3 ç mpe 2 ÷
è ø
34. (b) For nth atom
(1) Davisson and Gemner experiment-wave nature of \ rn = a0n2 – b
electrons. 39. (c) According to the question,
(2) Millikan’s oil drop experiment - charge of an elec-
tron. nh nh
2pr = nl = =
(3) Rutherford experiment - Existance of nucleus. p mv
(4) Frank-Hertz experiment - Quantisation of energy
levels. nh nh
or mvr = or mv =
35. (b) For first excited state, n = 2 and for Li + + Z = 3 2p 2pr
13.6 13.6
En = ´ Z2 = ´ 9 = 30.6 eV mv2 mv nh
n 2 4 F = qv B = or, q B = =
r r 2pr.r
36. (d) DE = hv
DE 1 ù k(2n - 1) nh nh
é 1 or, r2 = or, r =
n= =kê 2
- 2ú = 2 2pqB 2pqB
h ë (n - 1) n û n (n - 1)2
i.e., r µ n1/2
2k 1
» 3 or n µ 3 40. (c) Magnetic moment of the hydrogen atom, when the
n n
electron is in nth excited state, i.e., n ' = (n + 1)
-34
hc hc 6.62 ´10 ´ 3 ´108
37. (a) E= Þl= = = 993Ao As magnetic moment M n = In A = in (prn2 )
l E -19
12.5 ´1.6 ´10
Atoms P-459

Substituting the values we get,


mz 2 e5
i n = eVn = 2.2 ´ 106 1
4e02 n 3 h 3 Vn = m/s or Vn µ
n n
As principal quantum number increases, velocity
n2h2 æ 1 ö decreases.
rn = çk = ÷
2
4p kzme è 2 4p Î0 ø
1 æ 1 1ö
Solving we get magnetic moment of the hydrogen atom 44. (c) = Rç - ÷ where R = Rydberg constant
for nth excited state l è n12 n22 ø

æ e ö nh 1 æ 1 1ö 5 36
Mn ' = ç ÷ =ç - ÷ = l 32 =
è 2m ø 2p l32 è 4 9 ø 36 Þ
5
41. (d) For ground state, the principal quantum no. (n) = 1. Similarly solving for l31 and l21
Principal quantum number 4 belongs to 3rd excited state.
9 4
The possible number of the spectral lines from a state n to l 31 = and l 21 =
ground state is 8 3

n(n - 1) 4(4 - 1) l32 l


= = =6 \ = 6.4 and 21 ; 1.2
2 2 l31 l 31
42. (d) The energy of the system of two atoms of diatomic
45. (b) hc
1 2 E=
molecule E = I w l
2 46. (b) If n1 = n and n2 = n + 1
where I = moment of inertia
n 2 ( n + 1)
2
L Maximum wavelength lmax =
w = Angular velocity = , ( 2n + 1) R
I
L = Angular momentum
Therefore, for large n, l max µ n3
1 47. (b) Energy of excitation (DE) is
I = (m1r12 + m2 r22 )
2
æ1 1 ö
1 2 2 2 DE = 13.6 z2 ç n - n ÷ eV
Thus, E = (m1r1 + m2 r2 )w … (i) è 1 2ø
2
æ1 1 ö
1 L2 Þ DE = 13.6 (3)2 ç 2 - 2 ÷ = 108.8 eV
E= (m1r12 + m2 r2 2 ) 2 è1 3 ø
2 I
48. (c) It is given that transition from the state n = 4 to n = 3
L= nh in a hydrogen like atom result in ultraviolet radiation. For
(According to Bohr's Hypothesis)
æ 1 1 ö
2 infrared radiation ç 2 – 2 ÷ should be less. The only
E=
1
(m1r12 + m2 r22 )
L è n1 n2 ø
2 (m1r1 + m2 r22 )2
2 option is 5 ® 4.
Increasing
Energy
1 L2 n2 h 2 n=5
E= =
2 (m1r12 + m2 r22 ) 8p2 (m1r12 + m2 r22 ) n=4
n=3
(m1 + m2 )n 2 h 2 n=2
E=
8 p2 r 2 m1m2 n=1
49. (b) Given,
43. (a) Velocity of electron in nth orbit of hydrogen atom k
is given by : Centripetal force =
r
2pKZe 2 Then
Vn =
nh k mv 2
=
r r
P-460 Physics

1 2 1
Þ k = mv2 Þ Tn = mv = k é ù
2 2 E will be maximum for the transition for which ê 1 - 1 ú
2 2
Tn is independent of n ëê n1 n2 ûú
Also,
is maximum. Here n2 is the higher energy level.
nh
Angular momentum, L =
2p é 1 1 ù
Clearly, ê - ú is maximum for the third transition,
nh êë n12 n2 2 úû
Þ mvrn = (Q L = mvr)
2p
i.e., 2 ® 1. I transition is showing the absorption of energy.
nh æQ m 2v 2 = km ö
ç ÷ 52. (c) The wavelength of spectrum is given by
Þ rn =
2p km çè or mv = km ÷ø
1 æ 1 1 ö
Clearly, rn µ n = Rz 2 ç 2 - 2 ÷
l ç ÷
50. (d) We have to find the frequency of emitted photons. è n1 n2 ø
For emission of photons electron should makes a transition
from higher energy level to lower energy level. so, option 1.097 ´107
(a) and (b) are incorrect. where R =
m
Frequency of emitted photon is given by 1+
M
æ 1 1 ö
h n = -13.6 ç 2 - 2 ÷ where m = mass of electron
çn ÷
è 2 n1 ø M = mass of nucleus.
For transition from n = 6 to n = 2, Thus, wavelength involved in the spectrum of hudrogen
-13.6 æ 1 1 ö 2 æ 13.6 ö like atom depends upon masses of nucleus. The mass
n1 = ç 2
- 2 ÷ = ´ç ÷ number of hydrogen and deuterium is 1 and 2 respectively,
h è6 2 ø 9 è h ø
so spectrum of deuterium will be different from hydrogen.
For transition from n = 2 to n = 1,
53. (c) The energy required to remove the electron from the
-13.6 æ 1 1 ö 3 æ 13.6 ö . nth orbit of hydrogen is given by
n2 = - = ´ç ÷
h çè 22 12 ÷ø 4 è h ø
13.6
\ n1 < n2 En = 2
eV /atom
n
51. (b) Eenrgy of radiation that corresponds to energy
difference between two energy levels n1 and n2 is given 13.6
as For n = 2, En = = 3.4 eV
4
é 1 1 ù æ 1 1ö Therefore the energy required to remove electron from
E = Rhc ê 2 - 2 ú \ E a ç 2 – 2÷ n = 2 is + 3.4 eV.
êë n1 n2 úû è n1 n2 ø
27
Nuclei P-461

Nuclei
(a) 4.5 × 105 (b) 8 × 106
Composition and Size of the
TOPIC 1 (c) 6.82 × 105 (d) 1.33 × 106
Nuclei
7. Given the masses of various atomic particles mp = 1.0072 u,
1. The radius R of a nucleus of mass number A can be mn = 1.0087 u, me = 0.000548 u, mv = 0, md = 2.0141 u, where
estimated by the formula R = (1.3 × 10–15)A1/3 m. It follows
p º proton, n º neutron, e º electron,
that the mass density of a nucleus is of the order of :
v º antineutrino and d º deuteron. Which of the follow-
( M prot. @ M neut. ; 1.67 ´ 10 -27 kg) [Sep. 03, 2020 (II)] ing process is allowed by momentum and energy conser-
(a) 103 kg m–3 (b) 1010 kg m–3 vation? [Sep. 06, 2020 (II)]
(c) 1024 kg m–3 (d) 1017 kg m–3 (a) n + n ® deuterium atom (electron bound to the nucleus)
2. The ratio of the mass densities of nuclei of 40Ca and 16O is (b) p ® n + e+ + v
close to : [8 April 2019 II] (c) n + p ® d + g
(a) 1 (b) 0.1 (c) 5 (d) 2
(d) e+ + e– ® g
3. An unstable heavy nucleus at rest breaks into two nuclei
which move away with velocities in the ratio of 8:27. The 8. Find the Binding energy per neucleon for 120
50 Sn. Mass of
ratio of the radii of the nuclei (assumed to be spherical ) is: proton mp = 1.00783 U, mass of neutron mn = 1.00867 U and
[Online April 15, 2018] mass of tin nucleus mSn = 119.902199 U.
(a) 8 : 27 (b) 2 : 3 (c) 3 : 2 (d) 4 : 9 (take 1U = 931 MeV) [Sep. 04, 2020 (II)]
4. Which of the following are the constituents of the nucleus? (a) 7.5 MeV (b) 9.0 MeV
[2007] (c) 8.0 MeV (d) 8.5 MeV
(a) Electrons and protons (b) Neutrons and protons 9. In a reactor, 2 kg of 92U235 fuel is fully used up in 30 days.
(c) Electrons and neutrons (d) Neutrons and positrons The energy released per fission is 200 MeV. Given that the
5. If radius of the 27 nucleus is estimated to be 3.6 fermi Avogadro number, N = 6.023 × 1026 per kilo mole and 1 eV
13 Al
= 1.6 × 10–19 J. The power output of the reactor is close to:
then the radius of 125 nucleus be nearly [2005]
52 Te [Sep. 02, 2020 (I)]
(a) 8 fermi (b) 6 fermi (a) 35 MW (b) 60 MW
(c) 5 fermi (d) 4 fermi (c) 125 MW (d) 54 MW
10. Consider the nuclear fission
TOPIC 2 Mass-Energy Equivalence and Ne20 ® 2He4 + C12
Nuclear Reactions
Given that the binding energy/nucleon of Ne20, He4 and
6. You are given that mass of 73 Li = 7.0160 u, C12 are, respectively, 8.03 MeV, 7.07 MeV and 7.86 MeV,
4 identify the correct statement: [10 Jan. 2019 II]
Mass of 2 He = 4.0026 u
(a) energy of 12.4 MeV will be supplied
and Mass of 11 H = 1.0079 u. (b) 8.3 MeV energy will be released
When 20 g of 73 Li is converted into 42 He by proton (c) energy of 3.6 MeV will be released
capture, the energy liberated, (in kWh), is : (d) energy of 11.9 MeV has to be supplied
[Mass of nucleon = 1 GeV/c2] [Sep. 06, 2020 (I)]
P-462 Physics

11. Imagine that a reactor converts all given mass into energy 19. Statement-1: Energy is released when heavy nuclei
and that it operates at a power level of 109 watt. The mass undergo fission or light nuclei undergo fusion and
of the fuel consumed per hour in the reactor will be : Statement-2 : For heavy nuclei, binding energy per
(velocity of light, c is 3 × 108 m/s) [Online April 9, 2017] nucleon increases with increasing Z while for light nuclei
(a) 0.96 gm (b) 0.8 gm it decreases with increasing Z. [2008]
(c) 4 × 10–2 gm (d) 6.6 × 10–5 gm (a) Statement-1 is false, Statement-2 is true
12. Two deuterons undergo nuclear fusion to form a Helium (b) Statement-1 is true, Statement-2 is true; Statement-2
nucleus. Energy released in this process is : (given binding is a correct explanation for Statement-1
energy per nucleon for deuteron=1.1 MeV and for (c) Statement-1 is true, Statement-2 is true; Statement-2
helium=7.0 MeV) [Online April 8, 2017] is not a correct explanation for Statement-1
(a) 30.2 MeV (b) 32.4 MeV (d) Statement-1 is true, Statement-2 is false
(c) 23.6 MeV (d) 25.8 MeV
20. If MO is the mass of an oxygen isotope 8 O17 ,MP and MN
13. When Uranium is bombarded with neutrons, it undergoes
are the masses of a proton and a neutron respectively, the
fission. The fission reaction can be written as :
nuclear binding energy of the isotope is [2007]
235
92 U + 0 n1 ® 56 Ba141 + 36 Kr 92 + 3 x + Q(energy) (a) (MO –17MN)c2 (b) (MO – 8MP)c2
where three particles named x are produced and energy Q (c) (MO– 8MP –9MN)c2 (d) MO c 2
21. When 3Li7 nuclei are bombarded by protons, and the
is released. What is the name of the particle x ?
resultant nuclei are 4Be8, the emitted particles will be
[Online April 9, 2013] [2006]
(a) electron (b) a-particle (a) alpha particles (b) beta particles
(c) neutron (d) neutrino (c) gamma photons (d) neutrons
14. Assume that a neutron breaks into a proton and an
electron. The energy released during this process is : (mass 22. If the binding energy per nucleon in 73 Li and 42 He nuclei
of neutron = 1.6725 × 10–27 kg, mass of proton = 1.6725 × are 5.60 MeV and 7.06 MeV respectively, then in the
10–27 kg, mass of electron = 9 × 10–31 kg). [2012] reaction
(a) 0.51 MeV (b) 7.10 MeV p + 73 Li ¾¾
® 2 42 He
(c) 6.30 MeV (d) 5.4 MeV
15. Ionisation energy of Li (Lithium) atom in ground state is energy of proton must be [2006]
5.4 eV. Binding energy of an electron in Li+ ion in ground (a) 28.24 MeV (b) 17.28 MeV
state is 75.6 eV. Energy required to remove all three (c) 1.46 MeV (d) 39.2 MeV
electrons of Lithium (Li) atom is [Online May 19, 2012] 23. A nuclear transformation is denoted by X (n, a ) 73 Li .
(a) 81.0 eV (b) 135.4 eV Which of the following is the nucleus of element X ?
(c) 203.4 eV (d) 156.6 eV [2005]
16. After absorbing a slowly moving neutron of mass mN
(momentum » 0) a nucleus of mass M breaks into two (a) 10
5 B (b) 12 C 6 (c) 11 4 Be (d) 95 B
nuclei of masses m1 and 5m1 (6m1 = M + mN) respectively. 24. A nucleus disintegrated into two nuclear parts which have
If the de Broglie wavelength of the nucleus with mass m1 their velocities in the ratio of 2 : 1. The ratio of their nuclear
is l, the de Broglie wavelength of the nucleus will be [2011] sizes will be [2004]
(a) 5l (b) l / 5 (c) l (d) 25l (a) 3½ : 1 (b) 1:21/3 (c) 21/3:1 (d) 1:3½
DIRECTIONS: Questions number 17-18 are based on the
following paragraph.
25. The binding energy per nucleon of deuteron 1 H and ( )
2

A nucleus of mass M + Dm is at rest and decays into two daughter


M
( )
4
helium nucleus 2 He is 1.1 MeV and 7 MeV respectively..
nuclei of equal mass each. Speed of light is c. [2010] If two deuteron nuclei react to form a single helium nucleus,
2
then the energy released is [2004]
17. The binding energy per nucleon for the parent nucleus is (a) 23.6 MeV (b) 26.9 MeV
E1 and that for the daughter nuclei is E2. Then (c) 13.9 MeV (d) 19.2 MeV
(a) E2 = 2E1 (b) E1 > E2
26. When a U 238 nucleus originally at rest, decays by emitting
(c) E2 > E1 (d) E1 = 2 E2
an alpha particle having a speed ‘u’, the recoil speed of
18. The speed of daughter nuclei is the residual nucleus is [2003]
Dm 2Dm 4u
(a) c (b) c (a) (b) - 4u
M + Dm M 238 234
4u
Dm Dm (c) (d) - 4u
(c) c (d) c 234 238
M M + Dm
Nuclei P-463

27. In the nuclear fusion reaction 33. Two radioactive substances A and B have decay
2 3 4 constants 5l and l respectively. At t = 0, a sample has the
1 H + 1 H ® 2 He + n same number of the two nuclei. The time taken for the
given that the repulsive potential energy between the two 2
æ 1ö
nuclei is ~ 7.7 ´ 10 -14 J , the temperature at which the ratio of the number of nuclei to become ç ÷ will be :
è eø
gases must be heated to initiate the reaction is nearly
[10 April 2019, II]
[Boltzmann’s Constant k = 1.38 ´ 10 -23 J/K ] [2003] (a) 1/2l (b) 1/4l (c) 1/l (d) 2/l
(a) 10 7 K (b) 10 5 K (c) 10 3 K (d) 10 9 K 34. In a radioactive decay chain, the initial nucleus is 232
90 Th .
At the end there are 6 a-particles and 4 b-particles which
TOPIC 3 Radioactivity are emitted. If the end nucleus is A , A and Z are given by :
ZX

[12 Jan. 2019, II]


28. Acitvities of three radioactive substances A, B and C are
(a) A = 208 ; Z = 80 (b) A = 202 ; Z = 80
represented by the curves A, B and C, in the figure. Then (c) A = 208 ; Z = 82 (d) A = 200; Z = 81
their half-lives T1 (A) : T1 (B) : T1 (C) are in the ratio : 35. Using a nuclear counter the count rate of emitted
2 2 2 [Sep. 05, 2020 (I)] particles from a radioactive source is measured. At t =
0 it was 1600 counts per second and t = 8 seconds it
In R was 100 counts per second. The count rate observed,
6 as counts per second, at t = 6 seconds is close to:
[10 Jan. 2019 I]
4 A (a) 200 (b) 150 (c) 400 (d) 360
2 C B
36. A sample of radioactive material A, that has an activity of
0 10 mCi (1 Ci = 3.7 × 1010 decays/s), has twice the number of
5 10 t nuclei as another sample of a different radioactive material B
(yrs)
which has an acitvity of 20 mCi. The correct choices for half-
(a) 2 : 1 : 1 (b) 3 : 2 : 1 lives of A and B would then be respectively: [9 Jan. 2019 I]
(a) 5 days and 10 days (b) 10 days and 40 days
(c) 2 : 1 : 3 (d) 4 : 3 : 1 (c) 20 days and 5 days (d) 20 days and 10 days
29. A radioactive nucleus decays by two different processes. 37. At a given instant, say t = 0, two radioactive substances
The half life for the first process is 10 s and that for the R
second is 100 s. The effective half life of the nucleus is A and B have equal activities. The ratio B of their
RA
close to : [Sep. 05, 2020 (II)]
activities after time t itself decays with time t as e–3t. If
(a) 9 sec. (b) 6 sec. the half-life of A is ln2, the half-life of B is:
(c) 55 sec. (d) 12 sec. [9 Jan. 2019, II]
30. In a radioactive material, fraction of active material
ln2 ln2
remaining after time t is 9/16. The fraction that was (a) 4ln2 (b) (c) (d) 2ln2
remaining after t/2 is : [Sep. 03, 2020 (I)] 2 4
38. At some instant, a radioactive sample S1 having an activity
4 3 3 7 5mCi has twice the number of nuclei as another sample S2
(a) (b) (c) (d) which has an activity of 10mCi. The half lives of S1 and S2
5 5 4 8
are [Online April 16, 2018]
31. The activity of a radioactive sample falls from 700 s –1 to 500 (a) 10 years and 20 years, respectively
s –1 in 30 minutes. Its half life is close to: [7 Jan. 2020, II] (b) 5 years and 20 years, respectively
(a) 72 min (b) 62 min (c) 20 years and 10 years, respectively
(c) 66 min (d) 52 min (d) 20 years and 5 years, respectively
39. A solution containing active cobalt 27 60Co having activity
32. Two radioactive materials A and B have decay constants
of 0.8 µCi and decay constant l is injected in an animal's
10 l and l, respectively. If initially they have the same
body. If 1cm3 of blood is drawn from the animal's body
number of nuclei, then the ratio of the number of nuclei of
after 10 hrs of injection, the activity found was 300 decays
A to that of B will be 1/e after a time : [10 April 2019, I]
per minute. What is the volume of blood that is flowing in
1 1 11 1 the body? ( 1Ci = 3.7 × 1010 decay per second and at t
(a) (b) (c) (d) = 10 hrs e–lt = 0.84) [Online April 15, 2018]
9l 11l 10l 10l
(a) 6 litres (b) 7 litres (c) 4 litres (d) 5 litres
P-464 Physics

40. A radioactive nucleus A with a half life T, decays into a 47. The counting rate observed from a radioactive source at
nucleus B. At t = 0, there is no nucleus B. At sometime t, the t = 0 was 1600 counts s–1, and t = 8 s, it was 100 counts
ratio of the number of B to that of A is 0.3. Then, t is given
by [2017] s–1. The counting rate observed as counts s–1 at t = 6 s
T will be [Online May 26, 2012]
(a) t = T log (1.3) (b) t = log(1.3) (a) 250 (b) 400 (c) 300 (d) 200
log 2 log1.3 48. The decay constants of a radioactive substance for a and
(c) t = T (d) t = b emission are la and lb respectively. If the substance
log1.3 log 2 emits a and b simultaneously, then the average half life of
41. Half-lives of two radioactive elements A and B are 20 the material will be [Online May 19, 2012]
minutes and 40 minutes, respectively. Initially, the samples
have equal number of nuclei. After 80 minutes, the ratio of 2Ta Tb
decayed number of A and B nuclei will be : [2016] (a) T + T (b) Ta + Tb
a b
(a) 1 : 4 (b) 5 : 4 (c) 1 : 16 (d) 4 : 1
42. Let Nb be the number of b particles emitted by 1 gram Ta Tb
of Na24 radioactive nuclei(half life = 15 hrs) in 7.5 hours,
(c) T + T
a b
(d)
1
2
(Ta + Tb )
Nb is close to (Avogadro number = 6.023 × 1023/g. mole): 49. Which of the following Statements is correct?
[Online April 11, 2015] [Online May 12, 2012]
(a) The rate of radioactive decay cannot be controlled
(a) 6.2 × 1021 (b) 7.5 × 1021
but that of nuclear fission can be controlled.
(c) 1.25 × 1022 (d) 1.75 × 1022 (b) Nuclear forces are short range, attractive and charge
43. A piece of wood from a recently cut tree shows 20 decays dependent.
per minute. A wooden piece of same size placed in a (c) Nuclei of atoms having same number of neutrons are
museum (obtained from a tree cut many years back) shows known as isobars.
(d) Wavelength of matter waves is given by de Broglie
2 decays per minute. If half life of C14 is 5730 years, then formula but that of photons is not given by the same
age of the wooden piece placed in the museum is formula
approximately: [Online April 19, 2014] 50. A sample originally contained 1020 radioactive atoms,
(a) 10439 years (b) 13094 years which emit a-particles. The ratio of a-particles emitted in
the third year to that emitted during the second year is 0.3.
(c) 19039 years (d) 39049 years How many a-particles were emitted in the first year?
44. A piece of bone of an animal from a ruin is found to have [Online May 7, 2012]
14C activity of 12 disintegrations per minute per gm of its (a) 3 × 1018 (b) 3 × 1019
(c) 5 × 1018 (d) 7 × 1019
carbon content. The 14C activity of a living animal is 16 51. The half life of a radioactive substance is 20 minutes. The
disintegrations per minute per gm. How long ago nearly
did the animal die? (Given half life of 14C is t1/2 = 5760 approximate time interval (t2 – t1) between the time t2 when
years) [Online April 12, 2014] 2 1
of it had decayed and time t1 when of it had decayed
(a) 1672 years (b) 2391 years 3 3
(c) 3291 years (d) 4453 years is : [2011]
45. A radioactive nuclei with decay constant 0.5/s is being
produced at a constant rate of 100 nuclei/s. If at t = 0 there (a) 14 min (b) 20 min (c) 28 min (d) 7 min
were no nuclei, the time when there are 50 nuclei is: 52. Statement - 1 : A nucleus having energy E1 decays by
[Online April 11, 2014] b– emission to daughter nucleus having energy E2, but
the b– rays are emitted with a continuous energy spectrum
(b) 2ln æç ö÷ s
4
(a) 1s having end point energy E1 – E2.
è 3ø
Statement - 2 : To conserve energy and momentum in
æ 4ö b– decay at least three particles must take part in the
(c) ln 2 s (d) ln ç ÷ s
è 3ø
transformation. [2011 RS]
46. The half-life of a radioactive element A is the same as the (a) Statement-1 is correct but statement-2 is not correct.
mean-life of another radioactive element B. Initially both (b) Statement-1 and statement-2 both are correct and
substances have the same number of atoms, then : statement-2 is the correct explanation of statement-1.
[Online April 22, 2013]
(c) Statement-1 is correct, statement-2 is correct and
(a) A and B decay at the same rate always.
statement-2 is not the correct explanation of
(b) A and B decay at the same rate initially.
Statement-1
(c) A will decay at a faster rate than B.
(d) Statement-1 is incorrect, statement-2 is correct.
(d) B will decay at a faster rate than A.
Nuclei P-465

53. A radioactive nucleus (initial mass number A and atomic 57. The intensity of gamma radiation from a given source is I.
number Z emits 3 a - particles and 2 positrons. The ratio of I
On passing through 36 mm of lead, it is reduced to . The
number of neutrons to that of protons in the final nucleus 8
will be [2010] I
thickness of lead which will reduce the intensity to will
A- Z -8 A- Z -4 2
(a) (b)
Z -4 Z -8 be [2005]
A - Z - 12 A- Z -4 (a) 9 mm (b) 6mm
(c) (d) (c) 12mm (d) 18mm
Z -4 Z -2 58. Which of the following cannot be emitted by radioactive
54. The half-life period of a radio-active element X is same as substances during their decay? [2003]
the mean life time of another radio-active element Y. Initially (a) Protons (b) Neutrinoes
they have the same number of atoms. Then [2007] (c) Helium nuclei (d) Electrons
(a) X and Y decay at same rate always 59. A nucleus with Z= 92 emits the following in a sequence:
(b) X will decay faster than Y
(c) Y will decay faster than X a, b - , b - a, a, a, a, a, b - , b - , a, b + , b + , a
(d) X and Y have same decay rate initially Then Z of the resulting nucleus is [2003]
55. The energy spectrum of b-particles [number N(E) as (a) 76 (b) 78 (c) 82 (d) 74
a function of b-energy E] emitted from a radioactive source 60. A radioactive sample at any instant has its disintegration
is [2006] rate 5000 disintegrations per minute. After 5 minutes, the
rate is 1250 disintegrations per minute. Then, the decay
constant (per minute) is [2003]
(a) N(E) (b) N(E) (a) 0.4 ln 2 (b) 0.2 ln 2
(c) 0.1 ln 2 (d) 0.8 ln 2
E0
E
E0
E 61. At a specific instant emission of radioactive compound is
deflected in a magnetic field. The compound can emit
(i) electrons (ii) protons
(c) N(E) (d) N(E) (iii) He2+ (iv) neutrons
The emission at instant can be [2002]
E0
E
E0
E (a) i, ii, iii (b) i, ii, iii, iv
(c) iv (d) ii, iii
7 66
56. Starting with a sample of pure Cu ,
of it decays into 62. If N0 is the original mass of the substance of half-life period
8 t1/2 = 5 years, then the amount of substance left after 15
Zn in 15 minutes. The corresponding half life is [2005] years is [2002]
(a) 15 minutes (b) 10 minutes (a) N0/8 (b) N0/16 (c) N0/2 (d) N0/4
1
(c) 7 minutes (d) 5 minutes
2
P-466 Physics

Mass mA Dmc 2
1. (d) Density of nucleus, r = = In use of 1 g Li energy released =
Volume 4 3 mLi
pR
3
Dmc 2
In use of 20 g energy released = ´ 20 g
mA mLi
Þr= (Q R = R0 A1/ 3 )
4 1/ 3 3
p( R0 A ) [(7.016 + 1.0079) - 2 ´ 4.0026]u ´ c 2
3 = ´ 20 g
7.016 ´ 1.6 ´ 10-24
Here m = mass of a nucleon
3 ´1.67 ´ 10-27 æ 0.0187 ´ 1.6 ´ 10 -19 ´ 109 ö
\r = (Given, R0 = 1.3 × 10–15) =ç -24
´ 20÷ = 480 ´ 1010 J
4 ´ 3.14 ´ (1.3 ´10-15 )3 è 7.016 ´ 1.6 ´ 10 ø

Þ r = 2.38 ´1017 kg/m3 Q1 J =2.778×10–7 kWh


\ Energy released = 480 × 1010 × 2.778 × 10–7
2. (a) Nuclear density is independent of atomic number.
3. (c) Let heavy nucleus breaks into two nuclei of mass m1 = 1.33 × 106 kWh
and m 2 and move away with velocities V 1 and V 2 7. (c) For the momentum and energy conservation, mass
respectively. defect (Dm) should be positive. Since some energy is lost
in every process.
V1 8
According to question, = (m p + mn ) > md
V2 27
m1V1 = m2V2 (Law of momentum conservation) 8. (d) Mass defect,

m1 V2 27 Dm = (50m p + 70mn ) - (msn )


Þ = =
m2 V1 8 = (50×1.00783 + 70 × 1.008) – (119.902199)
= 1.096
4
r ´ pR13 æ mass ö
3 çèQ density r = ÷ Binding energy = (Dm)C 2 = ( Dm) ´ 931 = 1020.56
4 volume ø
r ´ pR23
3 Binding energy 1020.5631
= = 8.5 MeV
1

1 Nucleon 120
æ R1 ö æ 27 ö 3 æ 3ö 3 R1 3
Þ çè R ÷ø = çè 8 ÷ø = çè 2 ÷ø \ R =2 9. (b) Power output of the reactor,
2 2
4. (b) energy
P=
5. (b) Radius of a nucleus, time

R = R0 ( A)1/ 3 2 6.023 ´ 1026 ´ 200 ´ 1.6 ´ 10 -19


= ´ ; 60 MW
Here, R0 is a constant 235 30 ´ 24 ´ 60 ´ 60
A = atomic mass number
10. (d)
1/ 3 1/ 3
æA ö
R1 æ 27 ö 3
\ = 1 =ç = E Dmc 2
R2 çè A2 ÷ø è 125 ÷ø 5 11. (c) Power level of reactor, P = =
Dt Dt
5 mass of the fuel consumed per hour in the reactor,
Þ R2 = ´ 3.6 = 6 fermi Dm P 109
3 = 2= 8 2
= 4 ´ 10 -2 gm
Dt c (3 ´ 10 )
6. (d)
7 1
3 Li + 1H ¾¾
®2 ( 4
2 He ) 12. (c) 1H2 + 1H2 ® 2He4
Total binding energy of two deuterium nuclei = 1.1 × 4 =
Dm ® [mLi + mH ] - 2[ M He ] 4.4 MeV
Binding energy of a (2He4) nuclei = 4 × 7 = 28 MeV
Energy released = Dmc 2 Energy released in this process = 28 – 4.4 = 23.6 MeV
Nuclei P-467

13. (c) Nuclear fission equation æMö 2 2


çè ÷ø v1 = Dmc
92 U
135 1
+ 0 n ¾¾
® 56 Ba 141
+ 36 Kr 92 1
+ 3 0 n + Q(energy) 2
Hence particle x is neutron. 2Dmc 2 2 Dm
Þ v12 = Þ V1 = c
14. (a) 1
¾¾ ® 11H + -1e 0 + n + Q M M
0n
The mass defect during the process 19. (d) We know that energy is released when heavy nuclei
undergo fission or light nuclei undergo fusion. Therefore
Dm = mn - mH - me = 1.6725 × 10–27 statement (1) is correct.
– (1.6725 × 10–27+ 9 × 10–31kg) The second statement is false because for heavy nuclei the
–31
= – 9 × 10 kg binding energy per nucleon decreases with increasing Z
The energy released during the process
and for light nuclei, B.E/nucleon increases with increasing
E = Dmc2
Z.
E = 9 × 10–31× 9 × 1016 = 81 × 10–15 Joules
20. (c) Number of protons in oxygen isotope, Z = 8
81 ´ 10-15 Number of neutrons = 17 – 8 = 9
E= = 0.511MeV
1.6 ´ 10 –19 Binding energy
15. (d) = [ZMP + (A – Z)MN – M]c2
= [8MP + (17 – 8)MN – M]c2
16. (c) Initial momentum of system, pi = 0
= [8MP + 9MN – M]c2
Let p1 and p2 be the momentum of broken nuclei of masses
m1 and 5m1 respectively. = [8MP + 9MN – Mo]c2
p f = p1 + p 2 21. (c) 7
+ 11p ¾
¾® 8
+ 0
3 Li 4 Be 0g
From the conservation of momentum We see that both proton number and mass number are
pi = p f equal in both sides, so emitted particle should be massless
0 = p1 + p 2 gamma photons.
22. (b) Given,
p1 = - p 2
Binding energy per nucleon of 73 Li = 5.60 MeV
From de Broglie relation, wavelength
Binding energy per nucleon of 42 He = 7.06 MeV
h h
l1 = l2 = Let E be the energy of proton, then
p1 and p2
E + 7 ´ 5.6 = 2 ´ [4 ´ 7.06]
l1 = l 2 Þ E = 56.48 - 39.2 = 17.28MeV
l1 = l 2 = l. A
23. (a) ZX + 0 n1 ¾¾
® 3 Li 7 + 2 He 4
17. (c) In nuclear fission, the binding energy per nucleon of
Using conservation of mass number
daughter nuclei is always greater than the parent nucleus.
A+1=4+7
é æM M öù Þ A = 10
18. (b) Mass defect, DM = ê( M + Dm ) – çè 2 + 2 ÷ø ú
ë û Using conservation of charge number
Z+0=2+3 ÞZ=5
= [ M + Dm – M ] = Dm
It is boron 5B10
Energy released, Q = DMc2 = Dmc2 ...(i)
From the law of conservation of momentum 24. (b) Given :
v1 2
M M =
(M + Dm) × 0 = v1 – ´ v2 v2 1
2 2 From conservation of momentum m1v1 = m2 v2
Þ v1 = v2
1æMö 2 1æMö 2 1 æ m ö æv ö 1
Þç 1 ÷=ç 2 ÷ =
Now, Q = ç ÷v + ç ÷v –
2è 2 ø 1 2è 2 ø 2 2 è m2 ø è v1 ø 2
(M + Dm) × (0)2 We know that mass of nucleus, m µ A
Nuclear size R µ A1/3µ m1/3
M 2
= v1 (Q v1 = v2 ) ...(ii) 1/3
2 R1 æ m1 ö R
3
1 æ R ö æ 1 ö1/3
=ç ÷ Þ 13 = Þç 1 ÷=ç ÷
From equation (i) and (ii), we get R2 è m2 ø R2 2 è R2 ø è 2 ø
P-468 Physics

25. (a) The chemical reaction of process is 212 H ® 42He ln 2 ln 2 ln 2 æ ln 2 ö


Þ = + çèQ l = ÷
Binding energy of two deuterons, T T1 T2 T ø
4 × 1.1 = 4.4 MeV
1 1 1
Binding energy of helium nucleus = 4 × 7 = 28 MeV Þ = +
T T1 T2
Energy released = 28 – 4.4 = 23.6 MeV
26. (c) Mass of a particle, ma = 4 u 1 1 1 11
Mass of nucleus after fission, mn = 234u From Þ = + = [Given: T1 = 10 s & T2 = 100 s]
T 10 100 100
conservation of linear momentum we have
238 × 0 = 4 u + 234 v 100
ÞT = = 9 sec.
4 11
\ v=- u
234 30. (c) As we know, for first order decay, N (t ) = N0 e -lt
r 4 According to question,
\ Speed =| v |= u
234
27. (d) The average kinetic energy per molecule N (t ) 9
= = e - lt
at temperature T is N0 16
3 After time, t/2;
= kT
2
Where k = Boltzmann's constant N ( t / 2) = N 0 e -l ( t / 2)
This kinetic energy should be able to provide the repulsive
potential energy N ( t / 2) 9
= e-lt =
3 -14
N0 16
\ kT = 7.7 ´ 10
2 3
\ N (t / 2) =
N0
-14 4
2 ´ 7.7 ´ 10 9
ÞT = -23
= 3.7 ´ 10 K 31. (b) We know that
3 ´ 1.38 ´ 10
Activity, A = A0 e – l t
28. (c) Since, R = R0e -lt

ln R = ln R0 + (-l ln t ) æ In2 ö
A = A0 e – tIn 2/T1/ 2 çèQ l = T ÷ø
1/2
ln 2
l= = Slope
t1/ 2 Þ 500 = 700 e -tIn 2/T1/ 2

6 10 7 30 In 2
lA = Þ TA = ln 2 Þ In = (Q t = 30 minute)
10 6 5 T1/2

6 5ln 2 In 2
lB = Þ TB = Þ T1/2 = 30 = 61.8 minute
5 6 In 1.4
2 5ln 2 (Q ln 2 = 0.693 and ln.1.4 = 0.336)
lC = Þ TC =
5 6 Þ T1/2 » 62 minute
32. (a) As, N = N0e–lt
10 5 15
\ T1 : T1 : T1 = : : = 2 :1: 3
6 6 6 NA ( l B -l A ) t = 1 Þ ( l - l ) t = -1
so, N = e
A B C
2 2 2 B A
B e
29. (a) Let l1 and l2 be the decay constants of two process.
Þ (lA – lB) . t = 1
N be the number of nuclei left undecayed after two process.
From the law of radioactive decay we have 1 1 1
Þ t = (l - l ) t = =
dN é dN ù B A 10l - l 9 l
-
dt
= l1 N + l 2 N êëQ - dt = lN úû 33. (a) Let N1 and N2 be the number of radioactive nuclei of
substance at anytime t.
dN
Þ- = ( l1 + l 2 ) N N1 (at t ) = N 0 e -5lt (i)
dt
Þ l eq. = (l1 + l 2 ) N2 (at t ) = N 0 e -lt (ii)
Nuclei P-469

Dividing equation (i) by (ii), we get 38. (b) Given : N1 = 2N2


Activity of radioactive substance = lN
N1 1
= = e -4 lt Þ 4lt = 2
N 2 e2 ln 2 ln 2
Half life period t = or, l =
l T
2 æ 1ö ln 2
Þt = =ç ÷ l1N1 = ´ N1 = 5 mci ..... (i)
4l è 2l ø t1
34. (c) When one a- particle emitted then danghter nuclei ln 2
has 4 unit less mass number (A) and 2 unit less atomic (z) l2 N2 = ´ N 2 = 10 mci ..... (ii)
number (z). t2
Dividing equation (ii) by (i)
90 Th ® 78 Y + 6 2 He
232 208 4

t 2 N1 1
208
Y ® 82
208
X + 4 b praticle ´ =
78
t1 N 2 2
dN t2 1
35. (a) According to question, at t = 0, A0 = =1600 C/s = Þ t1 = 4t 2
dt t1 4
and at t = 8s, A = 100 C/s i.e., Half life of S1 is four times of sample S2.
A 1 Hence 5 years and 20 years.
\ A = 16 in 8s 39. (d) Let initial activity = No = 0.8 m ci
0
0.8 × 3.7 × 104 dps
Therefore half life period, t1/2 = 2s Activity in 1 cm3 of blood at t = 10 hr,
3
æ 1ö 300
\ Activity at t = 6s = 1600 çè ÷ø = 200C/s n= dps = 5 dps
2 60
36. (c) Activity A = l N N = Activity of whole blood at time t = 10 hr.
For material, A 10 = (2 N0)lA N
For material, B 20 = N0lB Total volume of the blood in the person, V =
n
é 0.693 ù N 0 e – lt 0.8 ´ 3.7 ´104 ´ 0.7927
Þ l B = 4l A \T12 A = 4T12 B êQ T12 = @ 5 litres
ë l úû =
n
=
5
40. (d) Let initially there are total N0 number of nuclei
( )
i.e. 20 days half-lives for A and 5 days T1 2 B
For material B. At time t
37. (c) Half life of A = l n2 NB
= 0.3(given)
NA
ln2 Þ NB = 0.3NA
(t1/2 ) A = l N0 = NA + NB = NA + 0.3NA
\ lA= 1 N0
at t = 0 RA = RB \ NA =
1.3
NAe–lAT = NBe–lBT As we know Nt = N0e – lt
NA = NB at t = 0 N0
or, = N0e – lt
1.3
–l t
RB N0 e B 1
At t = t R = –l t 1.3
= e–lt Þ ln(1.3) = lt
A N0 e A
ln(1.3) ln (1.3) ln(1.3) T
or, t = Þ t= = ln(2)
l ln(2)
e ( B A ) = e -3t
- l -l t
T
Þ lB - lA = 3 41. (b) For At½ = 20 min, t = 80 min, number of half lifes n = 4
N0
lB = 3 + lA = 4 \ Nuclei remaining = . Therefore nuclei decayed
24
ln2 ln2 N0
( t1/2 )B = lB
=
4
= N0 -
24
For Bt½ = 40 min., t = 80 min, number of half lifes n = 2
P-470 Physics
N0
\ Nuclei remaining = . Therefore nuclei decayed 2.303 A
22 Then, from, t = log10 0
N0 l A
= N0 -
22 2.303 ´ 5760 16
= log10
N0
1 0.693 12
N0 - 1-
24
16 15 4 5
\Required ratio = N0 =
1 = ´ = 2.303 ´ 5760
N0 - 2 1- 16 3 4 = log10 1.333
2 4 0.693
42. (b) We know that Nb = N0 (1 – e–lt)
2.303 ´ 5760 ´ 0.1249
= = 2390.81 » 2391 years.
6.023 ´ 1023 é ln2 ù 0.693
Nb = ê1 – e 15 ´ 7.5 ú
24 ë û 45. (b) Let N be the number of nuclei at any time t then,
on solving we get,
Nb = 7.4 × 1021 dN N
dN
t

dt
= 100 – lN or ò (100 - lN ) = ò dt
dN0 0 0
43. (c) Given: = 20 decays/min
dt
1
dN
-
l
[ log (100 - lN )]0N = t
= 2 decays/min
dt
log (100 – l N) – log 100 = – lt
T1/2 = 5730 years
As we know, 100 - l N
log = – lt
100
N = N0 e -lt
100 - lN lN
N0 = e -lt 1 - = e -lt
Log = lt 100 100
N

N 100
1
Log 0 N= (1 - e -l t )
\t= l
l N
As, N = 50 and l = 0.5/sec
2.303 ´ T1/ 2 N
= ´ Log10 0
0.693 N 100
\ 50 = (1 – e–0.5t)
0.5
dN 0 Solving we get,
N 20
But dt = 0 = = 10
dN N 2 æ 4ö
t = 2ln ç ÷ sec
dt è 3ø

2.303 ´ 5730 46. (d) ( T1/2 )A = ( t mean )B


\ t= ´1
0.693
0.693 1
= 19039 years Þ l = l Þ l A = 0.693 l B
A B
44. (b) Given, for 14C
A0 = 16 dis min–1 g–1 or lA < lB
A = 12 dis min–1 g–1
Also rate of decay = lN
t1/2 = 5760 years
Initially number of atoms (N) of both are equal but since
0.693 l B > l A , therefore B will decay at a faster rate than A
Now, l =
t1/ 2
47. (d) As we know,

l=
0.693 é N ù é 1 ùn
per year ê ú=ê ú ...(i)
5760
ë N0 û ë 2 û
Nuclei P-471

n = no. of half life 53. (b) When a radioactive nucleus emits 1 a-particle, the
N - no. of atoms left mass number decreases by 4 units and atomic number
decreases by 2 units. When a radioactive nucleus emits 1
N0 - initial no. of atoms
positron the atomic number decreases by 1 unit but mass
By radioactive decay law, number remains constant.
dN \ Mass number of final nucleus = A – 12
= kN
dt Atomic number of final nucleus = Z – 8
k - disintegration constant
\ Number of neutrons, Nn = (A – 12) – (Z – 8) = A – Z – 4
dN
Number of protons, Np = Z – 8
N
\ dt = ...(ii) Nn A- Z -4
dN 0 N0 \ Required ratio = =
dt Np Z -8
From (i) and (ii) we get
54. (c) Let lX and lY be the decay constant of X and Y.
dN
n Half life of X, = average life of Y
dt = é 1 ù
dN 0 êë 2 úû T1 2 = Tav
dt 0.693 1
n 4 n Þ =
é 100 ù é 1 ù é1ù é1 ù lX lY
or, ê ú =ê ú Þ ê ú =ê ú
ë 1600 û ë 2 û ë2û ë2û Þ l X = (0.693). l Y
\ n = 4, Therefore, in 8 seconds 4 half life had occurred
\ lX < lY .
in which counting rate reduces to 100 counts s–1.
Now, the rate of decay is given by
T1 æ dN ö
\ Half life, = 2 sec -ç = l X N0
2 è dt ÷ø x
In 6 sec, 3 half life will occur
æ dN ö
-ç = l y N0
é dN ù è dt ÷ø y
3 dN
ê ú
\ ê dt ú = é 1 ù Þ = 200 counts s–1 As the rate of decay is directly proportional to decay constant,
ë 1600 û êë 2 úû dt Y will decay faster than X.
55. (c) The range of energy of b-particles is from zero to
TaTb some maximum value.
48. (c) Tav = T + T 56. (d) It is given that
a b
7
If a and B are emitted simultaneously. of Cu decays in 15 minutes.
8
49. (a) Radioactive decay is a continuous process. Rate of \ Cu left undecayed is
radioactive decay cannot be controlled. Nuclear fission 3
can be controlled but not of nuclear fusion. 7 1 æ1ö
N=1– = =ç ÷
50. (b) 8 8 è2ø
51. (b) Number of undecayed atom after time t2 ; \ No. of half lifes = 3
N0 t 15
= N 0 e -lt2 ...(i) n= Þ 3=
3 T T
Number of undecayed atom after time t1;
15
2N0 Þ T = half life period = = 5 minutes
= N 0 e -lt1 ...(ii) 3
3 57. (c) Let intensity of gamma radiation from source be I0.
Dividing (ii) by (i), we get
Intensity I = I 0 . e - md ,
2 = el ( 2 1 )
t –t

Þ In 2 = l(t2 – t1) Where d is the thickness of lead.


Þ t2 – t1 = In 2/l Applying logarithm on both sides,
52. (b) Statement-1: A nucleus having energy E1 decays
æ I ö
by b- emission to daughter nucleus having energy E2 then - md = log ç ÷
b- rays are emitted with continuous energy spectrum with è I0 ø
energy E1 – E2.
Statement-2: For energy conservation and momentum I
For d = 36 mm, intensity =
conservation at least three particles, daughter nucleus, b 8
particle and antineutrino are required.
P-472 Physics

Therefore the final atomic number of resulting nucleus


æ I/8ö
- m ´ 36 = logç ÷ ..........(i) = Z –16 + 4 – 2 = Z –14
è I ø
= 92 – 14 = 78
For intensity I/2, thickness = d
60. (a) Initial activity, Ao = 5000 disintegration per minute
æ I / 2ö Activity after 5 min, A = 1250 disintegration per minute
-m ´ d = log ç ...........(ii)
è I ÷ø A = Aoe–lt
Dividing (i) by (ii), A
Þ e–lt = o
A
æ 1ö æ 1ö
log ç ÷ 3log ç ÷ 1 A 1 5000
36 è 8ø è 2ø 36 Þ l = log e o = log e
= = = 3 or d = = 12 mm t A 5 1250
d æ 1ö æ 1ö 3
log ç ÷ log ç ÷
è 2ø è 2ø 2
= log e 2 = 0.4 log e 2
58. (a) The radioactive substances emit a -particles (Helium 5
nucleus), b-particles (electrons) and neutrinoes. Protons 61. (a) Charged particles are deflected in magnetic field.
Electrons, protons and He2+ all are charged species. Hence,
cannot be emitted by radioactive substances during their
correct option is (a).
decay. 62. (a) After every half-life, the mass of the substance
59. (b) The number of a-particles released = 8 reduces to half its initial value.
Decrease in atomic number = 8 × 2 = 16 5 years N 5 years N /2
N 0 ¾¾¾¾ ® 0 ¾¾¾¾ ® 0
The number of b–-particles released = 4 2 2
Increase in atomic number = 4 × 1 = 4 N 0 5years N 0 / 4 N 0
= ¾¾¾¾ ® =
Also the number of b+ particles released is 2, which should 4 2 8
decrease the atomic number by 2.
28
Semiconductor Electronics : Materials, Devices and Simple Circuits P-473

Semiconductor Electronics :
Materials, Devices and
Simple Circuits
V0 6V
Solids, Semiconductors and
TOPIC 1
P-N Junction Diode
(d)
1. With increasing biasing voltage of a photodiode, the pho-
time
tocurrent magnitude : [Sep. 05, 2020 (I)]
3. Take the breakdown voltage of the zener diode used in the
(a) remains constant given circuit as 6V. For the input voltage shown in figure
(b) increases initially and after attaining certain value, it below, the time variation of the output voltage is :
decreases (Graphs drawn are schematic and not to scale)
[Sep. 04, 2020 (I)]
(c) Increases linearly
R1
(d) increases initially and saturates finally 10 V
2. Two Zener diodes (A and B) having breakdown voltages
Vin V0
of 6 V and 4 V respectively, are connected as shown in the V=0
circuit below. The output voltage V0 variation with input
– 10 V
voltage linearly increasing with time, is given by :
V
(Vinput = 0 V at t = 0)
(figures are qualitative) [Sep. 05, 2020 (II)] (a)
t
B
A 6V RL = 400W V
Vin 4V V0
(b)
100W t

V0 V

4V (c)
t
(a)

time V
V0 6V (d)
t
4V
(b) 4. When a diode is forward biased, it has a voltage drop of
0.5 V. The safe limit of current through the diode is 10 mA.
time If a battery of emf 1.5 V is used in the circuit, the value of
V0 6V minimum resistance to be connected in series with the
diode so that the current does not exceed the safe limit is :
4V
[Sep. 03, 2020 (I)]
(c)
(a) 300 W (b) 50 W
time (c) 100 W (d) 200 W
P-474 Physics

5. If a semiconductor photodiode can detect a photon with a 10. In the figure, potential difference between A and B is:
maximum wavelength of 400 nm, then its band gap energy is: [7 Jan. 2020 II]
Planck's constant, h = 6.63 × 10–34 J.s.
Speed of light, c = 3 × 108 m/s [Sep. 03, 2020 (II)]
(a) 1.1 eV (b) 2.0 eV
(c) 1.5 eV (d) 3.1 eV
6. Both the diodes used in the circuit shown are assumed to
be ideal and have negligible resistance when these are
forward biased. Built in potential in each diode is 0.7 V. For (a) 10 V (b) 5 V (c) 15 V (d) zero
the input voltages shown in the figure, the voltage (in 11. Figure shows a DC voltage regulator circuit, with a Zener
Volts) at point A is ________. [NA 9 Jan. 2020 I] diode of breakdown voltage = 6V. If the unregulated input
voltage varies between 10 V to 16 V, then what is the
maximum Zener current ? [12 Apr. 2019 II]

7. The current i in the network is: [9 Jan. 2020 II]

(a) 0.2 A (b) 0.6 A (c) 0.3 A (d) 0 A


(a) 2.5 mA (b) 1.5 mA (c) 7.5 mA (d) 3.5 mA
8. Two identical capacitors A and B, charged to the same
potential 5V are connected in two different circuits as 12. The figure represents a voltage regulator circuit using a
shown below at time t = 0. If the charge on capacitors A Zener diode. The breakdown voltage of the Zener diode is
and B at time t = CR is QA and QB respectively, then (Here 6 V and the load resistance is RL = 4 k. The series resistance
e is the base of natural logarithm) [9 Jan. 2020 II] of the circuit is Ri=1k. If the battery voltage VB varies from
8 V to 16 V, what are the minimum and maximum values of
the current through Zener diode ? [10 Apr. 2019 II]

VC CV
(a) QA = , QB = (b) QA = VC, QB = CV
e 2
VC CV VC (a) 0.5 mA; 6 mA (b) 1 mA; 8.5 mA
(c) QA = VC, QB = (d) QA = , QB =
e 2 e (c) 0.5 mA; 8.5 mA (d) 1.5 mA; 8.5 mA
9. The circuit shown below is working as a 8 V dc regulated 13. The reverse breakdown voltage of a Zener diode is 5.6 V in
voltage source. When 12 V is used as input, the power the given circuit.
dissipated (in mW) in each diode is; (considering both
zener diodes are identical) _____. [NA 9 Jan. 2020 II]

The current Iz through the Zener is : [8 April 2019 I]


(a) 10 mA (b) 17 mA
(c) 15 mA (d) 7 mA
Semiconductor Electronics : Materials, Devices and Simple Circuits P-475

14. In the given circuit the current through (a) 0.8 V (b) 0.6 V
Zener Diode is close to : [11 Jan. 2019 I] (c) 0.2 V (d) 0.4 V
R1 500 W 19. The reading of the ammeter for a silicon diode in the given
circuit is : [2018]
12V

R2 1500 W V2 =10 V R2 200W


(a) 0.0 mA (b) 6.7 mA
(c) 4.0 mA (d) 6.0 mA
15. The circuit shown below contains two ideal diodes, each
with a forward resistance of 50 W. If the battery voltage is
3V
6V, the current through the 100 W resistance (in Amperes)
is : [11 Jan. 2019 II] (a) 0 (b) 15 mA
(c) 11.5 mA (d) 13.5 mA
D1 150 W
20. In the given circuit, the current through zener diode is:
[Online April 16, 2018]
75 W
R1 500 W
D2 15 V Vz = 10 V
100 W 1500 W R2

6V
(a) 2.5mA (b) 3.3mA
(a) 0.036 (b) 0.020
(c) 0.027 (d) 0.030 (c) 5.5mA (d) 6.7mA
16. For the circuit shown below, the current through the 21. What is the conductivity of a semiconductor sample having
Zener diode is: [10 Jan. 2019 II] electron concentration of 5 × 1018 m–3, hole concentration
of 5 × 1019 m–3, electron mobility of 2.0 m2 V–1 s–1 and
5 kW
hole mobility of 0.01 m2 V–1 s–1 ?
[Online April 8, 2017]
(Take charge of electron as 1.6 × 10–19 C)
120 V 50 V 10 kW (a) 1.68 (W – m)–1 (b) 1.83 (W – m)–1
(c) 0.59 (W – m) –1 (d) 1.20 (W – m)–1
22. The V–I characteristic of a diode is shown in the figure.
(a) 9 mA (b) 5 mA The ratio of forward to reverse bias resistance is :
(c) Zero (d) 14 mA [Online April 8, 2017]
17. Mobility of electrons in a semiconductor is defined as the
ratio of their drift velocity to the applied electric field. If, for I(mA)
an n-type semiconductor, the density of electrons is 1019 m –
3
and their mobility is 1.6m2/(V.s) then the resistivity of the 20
semiconductor (since it is an n-type semiconductor
15
contribution of holes is ignored) is close to:[9 Jan. 2019 I]
(a) 2 Wm (b) 4 Wm (c) 0.4 Wm (d) 0.2 Wm 10
18. Ge and Si diodes start conducting at 0.3 V and 0.7 V –10
respectively. In the following figure if Ge diode .7 .8 V (Volt)
connection are reversed, the value of Vo changes by : 1m A
(assume that the Ge diode has large breakdown voltage)
[9 Jan. 2019 II]
Ge
Vo
(a) 10 (b) 10–6
(c) 106 (d) 100
12 V Si 5K
P-476 Physics

23. Identify the semiconductor devices whose characteristics D1 5W


are given below, in the order (i), (ii), (iii), (iv) : [2016]
I I
D2 10 W

V V

A B
(a) 0.4 A and 0.2 A (b) 0.2 A and 0.4 A
(i) (ii) (c) 0.1 A and 0.2 A (d) 0.2 A and 0.1 A
28. In an unbiased n-p junction electrons diffuse from n-region
I dark Resistance
to p-region because : [Online April 10, 2015]
(a) holes in p-region attract them
(b) electrons travel across the junction due to potential
V difference
Intensity (c) only electrons move from n to p region and not the
Illuminated of light vice-versa
(d) electron concentration in n-region is more compared
(iii) (iv) to that in p-region
(a) Solar cell, Light dependent resistance, Zener diode, 29. The forward biased diode connection is: [2014]
simple diode +2V –2V
(a)
(b) Zener diode, Solar cell, simple diode, Light dependent
–3V –3V
resistance (b)
(c) Simple diode, Zener diode, Solar cell, Light dependent (c)
2V 4V
resistance –2V +2V
(d) Zener diode, Simple diode, Light dependent (d)
resistance, Solar cell 30. For LED’s to emit light in visible region of electromagnetic
24. The temperature dependence of resistances of Cu and light, it should have energy band gap in the range of:
undoped Si in the temperature range 300-400 K, is best [Online April 12, 2014]
described by : [2016] (a) 0.1 eV to 0.4 eV (b) 0.5 eV to 0.8 eV
(a) Linear increase for Cu, exponential decrease of Si. (c) 0.9 eV to 1.6 eV (d) 1.7 eV to 3.0 eV
(b) Linear decrease for Cu, linear decrease for Si. 31. A Zener diode is connected to a battery and a load as
(c) Linear increase for Cu, linear increase for Si. show below: [Online April 11, 2014]
(d) Linear increase for Cu, exponential increase for Si.
25. An experiment is performed to determine the 1–V 4 kW A IL
characteristics of a Zener diode, which has a protective
IZ
resistance of R = 100 W, and a maximum power of dissipation I
rating of 1W. The minimum voltage range of the DC source 10 V = VZ RL = 2kW
60 V
in the circuit is : [Online April 9, 2016]
(a) 0 – 5V (b) 0 – 24 V
(c) 0 – 12 V (d) 0 – 8V B
26. A red LED emits light at 0.1 watt uniformly around it. The
The currents, I, IZ and IL are respectively.
amplitude of the electric field of the light at a distance of 1 m
(a) 15 mA, 5 mA, 10 mA
from the diode is : [2015]
(b) 15 mA, 7.5 mA, 7.5 mA
(a) 5.48 V/m (b) 7.75 V/m
(c) 12.5 mA, 5 mA, 7.5 mA
(c) 1.73 V/m (d) 2.45 V/m
27. A 2V battery is connected across AB as shown in the (d) 12.5 mA, 7.5 mA, 5 mA
figure. The value of the current supplied by the battery 32. The I-V characteristic of an LED is [2013]
when in one case battery’s positive terminal is connected
to A and in other case when positive terminal of battery B
is connected to B will respectively be: G
I Y
[Online April 11, 2015] R
(a) (b)

O V O V
Semiconductor Electronics : Materials, Devices and Simple Circuits P-477

V O I

I R (b)
Y I
(c) (d) G
B t
O V
33. Figure shows a circuit in which three identical diodes are
used. Each diode has forward resistance of 20 W and infinite (c)
backward resistance. Resistors R1 = R2 = R3 = 50 W. Battery
voltage is 6 V. The current through R3 is :
[Online April 22, 2013] I

D1 R1 (d)
t
36. If in a p-n junction diode, a square input signal of 10 V is
D2 D3 R2 applied as shown [2007]
5V

+ – RL
6V R3
(a) 50 mA (b) 100 mA -5V
(c) 60 mA (d) 25 mA Then the output signal across RL will be
34. This question has Statement 1 and Statement 2. Of the +5V 10 V
four choices given after the Statements, choose the one
(a) (b)
that best describes the two Statements.
Statement 1: A pure semiconductor has negative
temperature coefficient of resistance.
Statement 2: On raising the temperature, more charge
carriers are released into the conduction band.
(c) (d)
[Online May 12, 2012]
-10 V -5V
(a) Statement 1 is false, Statement 2 is true.
(b) Statement 1 is true, Statement 2 is false. 37. Carbon, silicon and germanium have four valence electrons
(c) Statement 1 is true, Statement 2 is true, Statement 2 is each. At room temperature which one of the following
not a correct explanation of Statement 1. statements is most appropriate ? [2007]
(d) Statement 1 is true, Statement 2 is true, Statement 2 is (a) The number of free electrons for conduction is
the correct explanation of Statement 1. significant only in Si and Ge but small in C.
35. A p-n junction (D) shown in the figure can act as a rectifier. (b) The number of free conduction electrons is significant
An alternating current source (V) is connected in the circuit. in C but small in Si and Ge.
(c) The number of free conduction electrons is negligibly
D small in all the three.
R (d) The number of free electrons for conduction is
significant in all the three.
38. If the lattice constant of this semiconductor is decreased,
then which of the following is correct? [2006]
V ~ conduction
Ec
band width
The current (I) in the resistor (R) can be shown by :[2009] band gap Eg
valence
band width Ev

(a) (a) All Ec, Eg, Ev increase


(b) Ec and Ev increase, but Eg decreases
(c) Ec and Ev decrease, but Eg increases
(d) All Ec, Eg, Ev decrease
P-478 Physics

39. A solid which is not transparent to visible light and whose 44. When p-n junction diode is forward biased then [2004]
conductivity increases with temperature is formed by (a) both the depletion region and barrier height are reduced
[2006] (b) the depletion region is widened and barrier height is
(a) Ionic bonding reduced
(b) Covalent bonding (c) the depletion region is reduced and barrier height is
(c) Vander Waals bonding increased
(d) Metallic bonding (d) Both the depletion region and barrier height are
increased
40. If the ratio of the concentration of electrons to that of
45. A strip of copper and another of germanium are cooled
7
holes in a semiconductor is and the ratio of currents is from room temperature to 80K. The resistance of [2003]
5 (a) each of these decreases
7
, then what is the ratio of their drift velocities? [2006] (b) copper strip increases and that of germanium
4
5 4 5 4 decreases
(a) (b) (c) (d) (c) copper strip decreases and that of germanium increases
8 5 4 7
41. The circuit has two oppositively connected ideal diodes (d) each of these increases
in parallel. What is the current flowing in the circuit?[2006] 46. The difference in the variation of resistance with
4W temeperature in a metal and a semiconductor arises
essentially due to the difference in the [2003]
D1 D2 (a) crystal sturcture
(b) variation of the number of charge carriers with
12V temperature
3W 2W
(c) type of bonding
(d) variation of scattering mechanism with temperature
(a) 1.71 A (b) 2.00 A (c) 2.31 A (d) 1.33 A 47. In the middle of the depletion layer of a reverse- biased
42. In the following, which one of the diodes reverse p-n junction, the [2003]
biased? [2006] (a) electric field is zero
(b) potential is maximum
+10 V
(c) electric field is maximum
(d) potential is zero
48. At absolute zero, Si acts as [2002]
(a) R (a) non-metal (b) metal
+5 V (c) insulator (d) none of these
49. By increasing the temperature, the specific resistance of a
(b) conductor and a semiconductor [2002]
–10 V
R (a) increases for both (b) decreases for both
(c) increases, decreases (d) decreases, increases
–5 V 50. The energy band gap is maximum in [2002]
(a) metals (b) superconductors
(c) (c) insulators (d) semiconductors.
R
TOPIC 2 Junction Transistor
–10 V
51. The output characteristics of a transistor is shown in the
+5 V
figure. When VCE is 10 V and IC = 4.0 mA, then value of bac
R
is ______. [NA Sep. 06, 2020 (II)]
R (IB )
(d)
8 60 mA
50 mA
(IC) in mA

6 40 mA
43. The electrical conductivity of a semiconductor increases 4 30 mA
when electromagnetic radiation of wavelength shorter than 20 mA
2 10 mA
2480 nm is incident on it. The band gap in (eV) for the
semiconductor is [2005] 2 4 6 8 10 12 14
(a) 2.5 eV (b) 1.1 eV (c) 0.7 eV (d) 0.5 eV (VCE) in volts
Semiconductor Electronics : Materials, Devices and Simple Circuits P-479

52. The transfer characteristic curve of a transistor, having


input and output resistance 100 W and 100 k W IC
respectively, is shown in the figure. The Voltage and Power
C RC
gain, are respectively : [12 Apr. 2019 I] RB
B
E VCC ~ v0
56. v1 ~ IE
VBB

In the figure, given that VBB supply can vary from 0 to 5.0
V, VCC = 5 V, bdc = 200, RB = 100 kW, RC = 1 KW and
VBE = 1.0 V. The minimum base current and the input
voltage at which the transistor will go to saturation, will
be, respectively : [12 Jan. 2019 II]
(a) 25 µA and 3.5 V (b) 20 µA and 3.5 V
(c) 25 µA and 2.8 V (d) 20 µA and 2.8 V
57. In a common emitter configuration with suitable bias, it is
given than RL is the load resistance and RBE is small signal
dynamic resistance (input side). Then, voltage gain, current
(a) 2.5×104, 2.5×106 (b) 5×104, 5×106 gain and power gain are given, respectively, by:
(c) 5×104, 5×105 (d) 5×104, 2.5×106 (b is current gain, IB, IC, IE are respectively base, collector
53. An npn transistor operates as a common emitter amplifier, and emitter currents:) [Online April 15, 2018]
with a power gain of 60 dB. The input circuit resistance is RL DI E 2 RL
100W and the output load resistance is 10 kW. The common (a) b , ,b
emitter current gain b is : [10 Apr. 2019 I] RBE DI B RBE
(a) 102 (b) 60 (c) 6×102 (d) 104 RL DI C R
54. An NPN transistor is used in common emitterconfiguration (b) b2 , ,b L
RBE DI B RBE
as an amplifier with 1 k &! load resistance. Signal voltage
of 10 mV is applied across the base-emitter. This produces a RL DIC 2 RL
3 mA change in the collector current and 15 ¼A change in (c) b2 , ,b
RBE DI E RBE
the base current of the amplifier. The input resistance and
voltage gain are: [9 April 2019 I] RL DIC 2 RL
(a) 0.33 k W 1.5 (b) 0.67 k W 300 (d) b , ,b
RBE DI B RBE
(c) 0.67 k W 200 (d) 0.33 k W 300
55. A common emitter amplifier circuit, built using an npn 58. The current gain of a common emitter amplifier is 69. If the
transistor, is shown in the figure. Its dc current gain is 250, emitter current is 7.0 mA, collector current is :
RC = 1 k &! and VCC = 10V. What is the minimum base current [Online April 9, 2017]
for VCE to reach saturation ? [8 Apr. 2019 II] (a) 9.6 mA (b) 6.9 mA
(c) 0.69 mA (d) 69 mA
59. In a common emitter amplifier circuit using an n-p-n
transistor, the phase difference between the input and the
output voltages will be : [Online April 2, 2017]
(a) 135° (b) 180°
(c) 45° (d) 90°
60. For a common emitter configuration, if a and b have their
usual meanings, the incorrect relationship between a and
b is : [2016]
b b2
(a) a = (b) a =
1 +b 1 +b 2
(a) 40 mA (b) 100 mA
(c) 7 mA (d) 10 mA 1 1
(c) = +1 (d) None of these
a b
P-480 Physics

61. A realistic graph depicting the variation of the reciprocal between P and Q. By connecting the common (negative)
of input resistance in an input characteristics measurement terminal of the multimeter to R and the other (positive)
in a common emitter transistor configuration is : terminal to P or Q, some resistance is seen on the multimeter.
[Online April 10, 2016] Which of the following is true for the transistor? [2008]
(a) 1 (a) It is an npn transistor with R as base
(W - 1 )
ri (b) It is a pnp transistor with R as base
0.01
(c) It is a pnp transistor with R as emitter
(d) It is an npn transistor with R as collector
0 0.6
VBE (V)
1 66. In a common base mode of a transistor, the collector current
(b) is 5.488 mA for an emitter current of 5.60 mA. The value of
1 -1
ri
(W ) the base current amplification factor (b) will be [2006]
0.01 (a) 49 (b) 50
(c) 51 (d) 48
0 0.6 1 67. In a common base amplifier, the phase difference between
VBE (V)
the input signal voltage and output voltage is [2005]
p
(c) 1 -1 (a) p (b)
ri
(W ) 4
p
(c) (d) 0
10 2
68. When npn transistor is used as an amplifier [2004]
0 0.6 1
VBE (V) (a) electrons move from collector to base
(b) holes move from emitter to base
(d) (c) electrons move from base to collector
1 -1
(W )
ri (d) holes move from base to emitter
0.1
69. For a transistor amplifier in common emitter configuration
for load impedance of 1k W (h fe = 50 and hoe = 25) the
1
0 0.6
VBE (V)
current gain is [2004]
(a) – 24.8 (b) – 15.7
62. The ratio (R) of output resistance r 0 , and the input (c) – 5.2 (d) – 48.78
resistance r i in measurements of input and output 70. The part of a transistor which is most heavily doped to
characteristics of a transistor is typically in the range : produce large number of majority carriers is [2002]
[Online April 10, 2016] (a) emmiter
(a) R ~ 102 – 103 (b) R ~ 1 – 10 (b) base
(c) R ~ 0.1 – 1.0 (d) R ~0.1 – 0.01 (c) collector
63. An unknown transistor needs to be identified as a npn or (d) can be any of the above three.
pnp type. A multimeter, with +ve and –ve terminals, is used
to measure resistance between different terminals of
transistor. If terminal 2 is the base of the transistor then TOPIC 3 Digital Electronics and Logic
Gates
which of the following is correct for a pnp transistor?
[Online April 9, 2016] 71. Identify the correct output signal Y in the given
(a) +ve terminal 2, –ve terminal 3,resistance low combination of gates (as shown) for the given inputs A
(b) +ve terminal 2, –ve terminal 1, resistane high and B. [Sep. 06, 2020 (I)]
(c) +ve terminal 1, –ve terminal 2, resistance high
B
(d) +ve terminal 3, –ve terminal 2, resistance high
Y
64. An n-p-n transistor has three leads A, B and C. Connecting
A
B and C by moist fingers, A to the positive lead of an
ammeter, and C to the negative lead of the ammeter, one
finds large deflection. Then, A, B and C refer respectively
to: [Online April 9, 2014] A
(a) Emitter, base and collector
(b) Base, emitter and collector B
(c) Base, collector and emitter
(d) Collector, emitter and base. 5 10 15 20 t
65. A working transistor with its three legs marked P, Q and R
is tested using a multimeter. No conduction is found
Semiconductor Electronics : Materials, Devices and Simple Circuits P-481

(a) A+B (b) A + B


(c) A.B (d) A.B
(a)
75. In the given circuit, value of Y is: [8 Jan. 2020 II]
t

(b)
t

(c) t
5 10 15 20

(a) 0
(b) toggles between 0 and 1
(c) will not execute
(d)
(d) 1
t
76. Which of the following gives a reversible operation?
72. Identify the operation performed by the circuit given below: [7 Jan. 2020 I]
[Sep. 04, 2020 (II)]
(a)
A

(b)
B

(c)
C

(a) NAND (b) OR


(c) AND (d) NOT (d)
73. In the following digitial circuit, what will be the output at
‘Z’, when the input (A, B) are (1, 0), (0, 0), (1, 1), (0, 1) : 77. The truth table for the circuit given in the fig. is :
[Sep. 02, 2020 (II)] [9 April 2019 I]

A
Z

éA B Y ù éA B Y ù
B ê0 0 1ú ê0 0 1ú
ê ú ê ú
(a) 0, 0, 1, 0 (b) 1, 0, 1, 1 ê0 1 1ú ê0 1 0ú
(a) ê 1 0 1 ú (b) ê 1 0 0 ú
(c) 1, 1, 0, 1 (d) 0, 1, 0, 0 ê ú ê ú
74. Boolean relation at the output stage-Y for the following êë 1 1 1 úû êë 1 1 0 úû
circuit is: [8 Jan. 2020 I]

éA B Y ù éA B Y ù
ê0 0 1ú ê0 0 0ú
ê ú ê ú
ê0 1 1ú ê0 1 0ú
(c) ê 1 0 0 ú (d) ê 1 0 1 ú
ê ú ê ú
êë 1 1 0 úû êë 1 1 1 úû
P-482 Physics

78. The logic gate equivalent to the given logic circuit is: b
[9 Apr. 2019 II]
a

x
(a) OR (b) NAND
(a) NAND (b) OR (c) NOR (d) AND (c) NOT (d) AND
79. The ouput of the given logic cirfcuit is: [12 Jan. 2019 I] 83. To get an output of 1 from the circuit shown in figure the
A input must be : [Online April 10, 2016]
a
Y
b
Y
B
(a) AB + AB (b) AB + AB c
(c) AB (d) AB (a) a = 0, b = 0, c = 1 (b) a = 1, b = 0, c = 0
80. To get output ‘1’ at R, for the given logic gate circuit (c) a = 1, b = 0, c = 1 (d) a = 0, b = 1, c = 0
the input values must be: [10 Jan. 2019 I] 84. The truth table given in fig. represents :
[Online April 9, 2016]

A B Y
0 0 0
0 1 1
1 0 1
(a) X = 0, Y = 1 (b) X = 1, Y = 1
(c) X = 1, Y = 0 (d) X = 0, Y = 0 1 1 1
81. Truth table for the given circuit will be
[Online April 15, 2018] (a) OR – Gate (b) NAND– Gate
x (c) AND– Gate (d) NOR– Gate

z 85. 50 W
VCC – 6V
y A
50 W
B
x y z x y z Vout
0 0 1 0 0 0
R = 10 kW
(a) 0 1 1 (b) 0 1 0
1 0 1 1 0 0
1 1 0 1 1 1 Given: A and B are input terminals.
Logic 1 = > 5 V
x y z x y z Logic 0 = < 1 V
Which logic gate operation, the above circuit does?
0 0 1 0 0 0
[Online April 19, 2014]
(c) 0 1 1 (d) 0 1 1 (a) AND Gate (b) OR Gate
1 0 1 1 0 1 (c) XOR Gate (d) NOR Gate
1 1 1 1 1 1 86. Identify the gate and match A, B, Y in bracket to check.
[Online April 9, 2014]
82. If a, b, c, d are inputs to a gate and x is its output, then, as
A A.B
per the following time graph, the gate is : [2016]
B
Y
d
(a) AND (A = 1, B = 1, Y = 1)
(b) OR (A = 1, B = 1, Y = 0)
c
(c) NOT (A = 1, B = 1, Y = 1)
(d) XOR (A = 0, B = 0, Y = 0)
Semiconductor Electronics : Materials, Devices and Simple Circuits P-483

87. Which of the following circuits correctly represents the


following truth table ? [Online April 25, 2013] 5V

A B C
0 0 0 C

0 1 0 1
1 0 1 A
1 1 0
2
B

(a) A C
B

A
(b) B (a) A NAND B (b) A OR B
C
(c) A AND B (d) A NOR B
90. Truth table for system of four NAND gates as shown in
figure is : [2012]
A
(c) A
B C

Y
(d) A C
B

88. A system of four gates is set up as shown. The ‘truth B


table’ corresponding to this system is :
[Online April 23, 2013] A B Y A B Y
A 0 0 0 0 0 0
0 1 1 0 1 0
(a) (b)
1 0 1 1 0 1
1 1 0 1 1 1
Y

A B Y A B Y
0 0 1 0 0 1
B 0 1 1 0 1 0
(c) (d)
1 0 0 1 0 1
(a) A B Y (b) A B Y
0 0 1 0 0 0 1 1 0 1 1 1
0 1 0 0 1 0
1 0 0 1 0 1 91. The figure shows a combination of two NOT gates and a
1 1 1 1 1 0 NOR gate. [Online May 26, 2012]

(c) A B Y (d) A B Y
0 0 1 A
0 0 1
0 1 0 0 1 1 Y
1 0 1 1 0 0
1 1 0 1 1 0 B
89. Consider two npn transistors as shown in figure. If 0 Volts
corresponds to false and 5 Volts correspond to true then The combination is equivalent to a
the output at C corresponds to : [Online April 9, 2013] (a) NAND gate (b) NOR gate
(c) AND gate (d) OR gate
P-484 Physics

92. Which one of the following is the Boolean expression for A


NOR gate? [Online May 19, 2012] Y
(a) Y = A+ B (b) Y = A.B B
(c) Y = A.B (d) Y = A Input A
93. Which logic gate with inputs A and B performs the same
operation as that performed by the following circuit?
Input B
A [Online May 7, 2012]
(a) NAND gate B Output is
(b) OR gate (a)
V
(c) NOR gate Lamp
(b)
(d) AND gate
94. The output of an OR gate is connected to both the inputs
of a NAND gate. The combination will serve as a: (c)
[2011 RS]
(a) NOT gate (b) NOR gate (d)
(c) AND gate (d) OR gate
97. In the circuit below, A and B represent two inputs and C
95. The combination of gates shown below yields [2010]
represents the output. [2008]
(a) OR gate A A

(b) NOT gate C


X
(c) XOR gate B
B
(d) NAND gate
96. The logic circuit shown below has the input waveforms The circuit represents
‘ A’ and ‘B’ as shown. Pick out the correct output (a) NOR gate (b) AND gate
waveform. [2009] (c) NAND gate (d) OR gate
Semiconductor Electronics : Materials, Devices and Simple Circuits P-485

1. (d) I-V characteristic of a photodiode is as follows : VR = 1.5 - 0.5 = 1 volt


mA iR = 1 (=VR)
1 1
\ Rmin = = -2 = 100 W
i 10
V 5. (d) Given,
Reverse bias
Wavelength of photon, l = 400 nm
A photodiode can detect a wavelength corresponding to
the energy of band gap. If the signal is having wavelength
mA greater than this value, photodiode cannot detect it.
On increasing the biasing voltage of a photodiode, the hc 1237.5
magnitude of photocurrent first increases and then attains \ Band gap E g = = = 3.09 eV
l 400
a saturation.
6. (12) Right hand diode is reversed biased and left hand
2. (c) Till input voltage reaches 4 V. No zener is in breakdown diode is forward biased.
region so V0 = Vi. Then now when Vi changes between 4 V
Hence Voltage at ‘A’
to 6 V one zener with 4 V will breakdown and P.D. across
VA = 12.7 – 0.7 = 12 volt
this zener will become constant and remaining potential
will dorp across resistance in series with 4 V zener. 7. (c) Both the diodes are reverse biased, so, there is no
flow of current through 5W and 20W resistances.
Now current in circuit increases abruptly and source must
Now, two resistors of 10W and two resistors of 5W are in
have an internal resistance due to which some potential
series.
will get drop across the source also so correct graph
Hence current I through the network = 0.3 A
between V0 and t will be
8. (c) In case I diode is reverse biased, so no current flows
V0
\ QA = CV
6V
In case II, current will flow as diode is forward biased. So,
4V it offers negligible resistance to the flow of current and
thus be replaced by short circuit. Now, the charge of
t capacitor will leak through the resistance and decay
exponentially with time.
3. (c) Here two zener diodes are in reverse polarity so if one
During discharging of capacitor
is in forward bias the other will be in reverse bias and
Potential difference across the capacitor at any instant
above 6V the reverse bias will too be in conduction mode.
Hence when V > 6V the output will be constant. And when t
-
V < 6V it will follow the inut voltage. V ' = Ve CR

4. (c) According to question, when diode is forward biased, But t = CR


Vdiode = 0.5 V V
V ' = Ve -1 =
Safe limit of current, I = 10 mA = 10–2 A e
Rmin = ? CV
\ Charge QB = CV’ =
e
–2 R
10 A 12 – 8
0.5 volt 9. (40) Current in the circuit, I = = 10 –2 A
400
Power dissipited in each diode, P = VI
1.5 V Þ P = 4 × 10–2 = 40 mW
Voltage through resistance
P-486 Physics

10. (a) The given circuit has two 10kW resistances in parallel, 14. (a) Since voltage across zener diode does not reach to
so we can reduce this parallel combination to a single breakdown voltage therefore its resistance will be infinite
equivalent resistance of 5kW. & current through it is 0.
15. (b) As D2 is reversed biased, so no current through 75W
resistor.
now Req= 150 + 50 + 100
= 300 W
Diode is in forward bias. So it will behave like a conducting BatteryVoltage
So, required current I =
wire. 300
30 6
VA – VB = ´ 5 = 10 V I= = 0.02
5 + 10 300
11. (d) Current in load resistance,
6 16. (a) The voltage across zener diode is constant
i1 = 3 = 1.5 × 10–3 A = 1.5 mA
4 ´10
For V = 16 volt, i i –i1
120V 5kW(R1) R2 10kW
(16 - 6)
is = = 5 mA
2 ´ 103
V 50
\ i2 = is – i1 = 5 – 1.5 = 3.5 mA i( R 2 ) = = = 5´10 –3 A
R 10 ´103

12. (c) V 120 – 50 70


i ( R1 ) = = = 14 ´ 10–3 A
R 5 ´10 3
5´10 3

\ izenerdiode= 14 × 10–3 – 5 × 10–3 = 9 × 10–3 A = 9 mA


17. (c) As we know, current density,
j = sE = nevd
vd
s = ne = nem
E
For voltage, V = 8V
1 1
æ 3ö 1 =r= =Resistivity
Current, I1 = ç 8 - 6 - ÷ = = 0.5mA s n e eme
è 2ø 2
For voltage, V = 16V 1
=
æ 3ö 10 ´ 1.6 ´ 1019 - 19 ´ 1.6
19
Current, I 2 = ç16 - 6 - ÷ = 8.5 mA or P = 0.4 Wm
è 2ø
18. (d) Initially Ge and Si are both forward biased so current will
13. (a) effectivily pass through Ge diode \ V° = 12 – 0.3 =11.7 V
And if "Ge" is revesed then current will flow through "Si"
diode
\ V° = 12 – 0.7 =11.3 V
Clearly, V° changes by 11.7 – 11.3 = 0.4 V
P.D. across 800W resistors = 5.6 V 19. (c) Clearly from fig. given in question, Silicon diode is
in forward bias.
5.6
so, I 800 W = A = 7 mA \ Potential barrier across diode
800
DV = 0.7 volts
Now, P.D. across 200W resistors = 9 – 5.6 V = 3.4 V
9 - 5.6 V - DV 3 - 0.7 2.3
so, I 200 W = = 17 mA Current, I = = = = 11.5mA
200 R 200 200
so, current through zener diode = I 2 = 17 – 7 = 10 mA
Semiconductor Electronics : Materials, Devices and Simple Circuits P-487

25. (c) The minimum voltage range of DC source is given by


20. (b) I
V2 = PR Q P = 1 watt, R = 100W
R1 500W = 1×100
15 V IR2 \ V = 10 volt.
IZ
1500W R2 VZ = 10 V 1
26. (d) Using Uav = e 0 E02
2
P
The voltage drop across R2 is VR = VZ = 10V But U av =
2 4p r 2 ´ c
The current through R2 is
P 1
VR 2 \ = e0 E02 ´ c
10V 4p r 2 2
IR 2 = = = 0.667 ´ 10-2 A
R2 1500W
= 6.67 × 10–3 A = 6.67 mA 2P 2 ´ 0.1 ´ 9 ´ 109
E 20 = =
The voltage drop across R1 is 4 pr 2 e 0 c 1 ´ 3 ´ 108

VR1 = 15V - VR 2 = 15V - 10V = 5V \ E0 = 6 = 2.45V/m


The current through R1 is 27. (a) When positive terminal connected to A then diode

VR1 2
IR1 = =
5V
= 10-2 A = 10 ´ 10 -3 A = 10 mA D1 is forward biased, current, I = = 0.4 A
5
R1 500W
When positive terminal connected to B then diode D2
The current through the zener diode is
2
IZ = IR1 - IR 2 = (10 - 6.67)mA = 3.3mA is forward biased, current, I = = 0.2 A
10
21. (a) The conductivity of semiconductor
s = e (heµe + hhµh) 2P 2 ´ 0.1 ´ 9 ´ 109
E 20 = =
= 1.6 × 10 –19(5 × 1018 ×2 + 5 × 1019 × 0.01) 4 pr 2 e 0 c 1 ´ 3 ´ 108
= 1.6 × 1.05 = 1.68
\ E0 = 6 = 2.45V/m
DV 0.1
22. (b) Forward bias resistance = = = 10 W 28. (d) Electrons in an unbiased p-n junction, diffuse from
DI 10 ´ 10-3 n -region i.e. higher electron concentration to p-region
10 i.e. low electron concentration region.
Reverse bias resistance =
-6
= 107 W
10 n
29. (a) P
Forward bias resistance
Ratio of resistances = = 10–6 For forward bias, p-side must be at higher potential than
Reverse bias resistance
n-side. DV = (+ )Ve
23. (c) Graph (p) is for a simple diode.
Graph (q) is showing the V Break down used for zener 30. (d) Energy band gap range is given by,
diode. hc
Eg =
Graph (r) is for solar cell which shows cut-off voltage l
and open circuit current.
Graph (s) shows the variation of resistance h and hence
-7
( -7
For visible region l = 4 ´ 10 : 7 ´ 10 m )
current with intensity of light. 6.6 ´ 10 -34 ´ 3 ´ 108
Eg =
7 ´ 10 -7
r P
24. (a) 19.8 ´ 10-26
=
7 ´ 10 -7

T T 2.8 ´ 10-19
=
Metal (for limited Semiconductor
–Eg
1.6 ´ 10-19
range of temperature) KT B

Eg = 1.75 eV
P-488 Physics

31. (d) Here, R = 4 kW = 4 × 103 W semiconductors (covalent bonding) and insulators (ionic
bonding) increases with increase in temperature.
Vi = 60 V
Solid which is formed by covalent bond is not transparent
Zener voltage Vz = 10 V to visible light and its conductivity increase with
RL = 2 kW = 2 × 103 W temperature.
40. (c) Relation between drift velocity and current is
VZ 10
Load current, IL = = = 5 mA I = nAeVd
RL 2 ´ 103
Ie ne eAve
=
Vi - VZ Ih nh eAvh
Current through R, I =
R 7 7 ve
Þ = ´
60 - 10 50 4 5 vh
= = = 12.5 mA
4 ´ 103 4 ´ 103 ve 5
Þ =
Fom circuit diagram, vh 4
I = IZ + IL 41. (b) D2 is forward biased.
Þ 12.5 = IZ + 5 D1 is reversed biased. So, it will act like an open circuit.
So effective resistance of the circuit
Þ IZ = 12.5 – 5 = 7.5 mA
32. (a) For same value of current higher value of voltage is E 12
R = 4 + 2 = 6W \i = = =2A
required for higher frequency hence (a) is correct answer. R 6
33. (a) Here, diodes D1 and D2 are forward biased and D3 is 42. (d) p-side connected to low potential and n-side is
reverse biased. connected to high potential.
Therefore current through R3 43. (d) Band gap = energy of photon of wavelength 2480
nm. So,
V 6 1
i= = = A = 50 mA
R ' 120 20 hc
Band gap, E g =
34. (d) Temperature coefficient of resistance is negative for l
pure semiconductor. And no. of charge carriers in
æ 6.63 ´ 10 - 34 ´ 3 ´ 108 ö 1
conduction band increases with increase in temperature. = çç ÷´ eV
2480 ´ 10 -9 ÷ -19
35. (b) The given circuit will work as half wave rectifier as it è ø 1.6 ´ 10
conducts during the positive half cycle of input AC. = 0.5 eV
Forward biased in one half cycle and reverse biased in 44. (a) In forward biasing, the p type is connected to positive
the other half cycle]. terminal and n type is connected with negative terminal.
36. (a) The current will flow through RL when the diode is So holes from p region and electron from n region are
forward biased. pushed towards the Junction which reduces the width of
37. (a) Si and Ge are semiconductors but C is an insulator. In depletion layer. Also, distance between diffused holes
Si and Ge at room temperature, the energy band gap is low and electrons decrease, which decrease electric field
due to which electrons in the covalent bonds gains kinetic hence barrier potential.
energy and break the bond and move to conduction band. 45. (c) Copper is a conductor and in conductor resistance
As a result, hole is created in valence band. So, the number decreases with decrease in temperature. Germanium is a
of free electrons is significant in Si and Ge. semicon ductor. In semi-conductor resistance increases
38. (c) A crystal structure is made up of a unit cell with decrease in temperature.
arranged in a particular way; which is periodically 46. (b) When the temperature increases, certain bounded
repeated in three dimensions on a lattice. The spacing electrons become free which tend to promote
between unit cells in various directions is called its conductivity. Simultaneously number of collisions
lattice constants. As lattice constants increases the between electrons and positive kernels increases which
band-gap (E g ), also increases which means more decrease the relaxation time.
energy would be required by electrons to reach the
47. (a) In reverse biasing the width of depletion region
conduction band from the valence band. Automatically
increases, and current flowing through diode is zero. Thus,
E c and E v decreases.
electric field is zero at middle of depletion region.
39. (b) Van der Waal's bonding is attributed to the attractive
forces between molecules of a liquid. The conductivity of
Semiconductor Electronics : Materials, Devices and Simple Circuits P-489

48. (c) Pure silicon, at OK, will contain all the electrons in P0
bounded state. The conduction band will be empty. So Þ = 106
pi
there will be no free electrons (in conduction band) and
holes (in valence band).Therefore no electrons from æR ö
= b2 ç out ÷
valence band are able to shift to conduction band due to è R in ø
thermal agitation. Pure silicon will act as insulator.
æ 10000 ö
49. (c) Specific resistance (resistivity) is given by Þ 106 = b2 ç ÷ [as Rout = 10, 000W Rin = 100W]
è 100 ø
m Þ b = 100
r=
ne 2 t
where n = no. of free electrons per unit volume Dlc 3 ´ 10-3
54. (b) b= = = 200
and t = average relaxation time Dlb 15 ´ 10 -6
For a conductor with rise in temperature n increases.
Increase in temperature results increase in number of V0 R2
We have =b
collision between free electrons due to which relaxation Vi R1
time T decreases. But the decrease in t is more dominant
than increase in n resulting an increase in the value of r . V0 æ 1000 ö
or = 200 ç ÷
For a semiconductor with rise in temperature, n increases Vi è R1 ø
and t decreases. But the increase in n is more dominant
than decrease in t resulting in a decrease in the value of r. V0
If R1 = 0.67kW Þ = 300
Vi
r 2 = r1 (1 + aDT ) 55. (a) Given, b = 250
For conductor a is positive VCC R
Voltage gain, =b C
\ r2 > r 1 for D T positive i.e., increase in temperature. VB RB
For semi conductor a is negative 10 103
\ r2 < r 1 for D T positive. = 250 ´
VB RB
50. (c) In insulators, valence band is completely filled while
VB 1
conduction band is empty. The energy band gap is \ = = 40µA
maximum in insulators. RB 25 ´103
51. (150) 56. (a) At saturation, VCE = 0
At VCE = 10 V and IC = 4 mA VCE = VCC – ICRC

Change in base current, DI B = (30 - 20) = 10 µA VCC


Þ IC = R = 5 × 10–3 A
c
Change in collector current, DIC = (4.5 - 3) = 1.5 mA
Current gain,
æ DI ö 1.5 mA IC
b =ç C ÷ = =150 bdc = I
è DI B ø 10mA B
5 ´ 10 –3
Dic 200 - 100 IB = = 25 mA
200
52. (Bonus) b = Di = 10 - 5 = 20
b At input side
VBB = IBRB + VBE
R2 20 ´ 100 ´103
Voltage gain = b = = 20 ×103 = (25 mA) (100 kW) + 1V
R1 100
VBB = 3.5 V
æ 3ö DIC
2 R2 2 100 ´ 10 57. (d) Curent gain b =
Power gain = b = 20 ç ÷
ç 100 ÷ = 4 × 10
5
R1 IB
è ø
RL
æ P0 ö Voltage gain Av = Current gain × Resistance gain = b R
BE
53. (a) Power gain = 60 = 10log ç p ÷
è iø Power gain Ap = (Current gain)2 × Resistance gain
æP ö RL
Þ 6 = log ç 0 ÷ =b
2
è pi ø RBE
P-490 Physics

58. (b) Given, current gain of CE amplifier b = 69, IE = 7 mA


P P
IC E C
or = 69 n
IB
1 B
b 69 I C
We know that, a = = = 2
1 + b 70 I E
69 69
IC = I E ´ = ´7
70 70 +ve terminal 1, –ve terminal 2, resistance high.
Collector current, IC = 6.9 mA 64. (c) In the given question, A, B and C refer base, collector
59. (b) In common emitter configuration for n-p-n transistor and emitter respectively.
input and output signals are 180° out of phase i.e., phase 65. (b) It is a p-n-p transistor with R as base.
difference between output and input voltage is 180°. 66. (a) Collector current, IC = 5.488 mA,
Ic Emitter current Ie = 5.6 mA
Ic
60. (b, d)We know that a = and b =
Ie Ib Ic 5.488
a= = ,
Also Ie = Ib + Ic Ie 5.6
Ic a
b= = 49
Ic I b 1- a
\ a= = b =
Ib + Ic Ic 1 + b
1+ 67. (d) In common base amplifier circuit, input and output
Ib voltage are in the same phase. So, the phase difference
Option (b) and (d) are therefore incorrect. between input voltage signal and output voltage signal is
zero.
I
68. (c) In npn transistor, electrons moves from emitter to base.
slope = 1/r 69. (d) In common emitter configuration for transistor
amplifier current gain

61. (c) -h f e
Ai =
1 + boe RL
V
Where hfe and hoe are hybrid parameters.
-50
\ Ai =
1 + 25 ´ 10-6 ´ 1 ´ 103
1/r = – 48.78
70. (a) Emitter main function is to supply the majority charge
Þ carrriers towards the collector. Therefore emitter is most
heavily doped.
VBE 71. (a) Boolean expression,

y = A×B = A + B = A+ B
ri
62. (c) For C.B. configuration r ? 0.1ς B
o B

For CE and CC-configuration


A
A
ri
» 1ς .
r0 Truth table :
63. (c) Connecting circuit according to question, it is clear A B Y
0 1 1
1 0 1
0 0 0
1 1 1
Semiconductor Electronics : Materials, Devices and Simple Circuits P-491

1 1 1
74. (d)
A 0 0 0 0 0 0
B 1 1 1 1 1 1
0 0 0
y 1 1 1 0 1 1 0 1 1
y=A+B

OR + NOT ® NOR Gate


72. (c) When two inputs of NAND gate is shorted, it behaves Hence Boolean relation at the output stage – Y for the
like a NOT gate so boolen equation will be circuit,
Y = A + B = A.B
y = A+ B +C
75. (a)
y = A× B × C

A B C
0 0 0 0
1 0 0 0
0 1 0 0
0 0 1 0
Y = AB. A = AB + A = AB + A
1 1 0 0
For A = 1, B = 0
1 0 1 0 Y = (1) × 0 + 0
0 1 1 0 ÞY=0+0=0
1 1 1 1 76. (d) A logic gate is reversible if we can recover input data
Thus, whole arrangement behaves like a AND gate. from the output. Hence NOT gate.
77. (c)
73. (a)
A B (A + B) (A + B). A (A + B).A
A A× B
A 0 0 0 0 1
B A× B x = W + A× B
0 1 1 0 1
1 0 1 1 0
A
A+ B
W = ( A × B ) × ( A + B) 1 1 1 1 0
B
B 78. (b) Truth table ®
The output is of OR-gate
A B A× B A+ B W = ( A × B) × ( A + B) Q = W + A × B Q=x
A B A B A.B
1 0 1 0 0 1 0
0 0 1 1 0
0 1 1 0 0 1 0
0 1 1 0 1
1 1 0 0 0 0 1
1 0 0 1 1
0 0 1 1 1 0 0
1 1 0 0 1
P-492 Physics

A 86. (a)
79. (c) A A A. B
A. AB B Y
A.B
Y = (A. AB). (AB + B)
A.B AB Y = AB.AB = AB + AB = AB + AB = AB
B In this case output Y is equivalent to AND gate.
(AB + B)
B
87. (a) For circuit 1
( )
Y = A × AB + ( AB + B ) A.B = Y+A = C
= A × AB + AB × B A B Y+A = C
= A × ( A + B) + AB × B 0 0 0 1 0
0 1 0 1 0
= AB
1 0 0 0 1
80. (c) From the given logic circuit,
1 1 1 0 0
p= x+y 88. (a) In the given system all four gate is NOR gate
Q = y.x = y + x Truth Table
A B (y ' = A + B) y '' = (A + y ') y ''' = (A + y '') y = y ''+ y '''
Output, R = P + Q
0 0 1 0 0 1
To make output 1 0 1 0 1 0 0
P + Q must be ‘0’ 1 0 0 0 1 0
1 1 0 0 0 1
So, x = 1, y = 0
81. (c) Truth table of the circuit is as follows A B y
i.e.,
0 0 1
x y x a = x. y b = x . y z = a.b
0 1 0
0 0 1 0 0 1
1 0 0
0 1 1 0 1 1
1 1 1
1 0 0 0 0 1
1 1 0 1 0 1 89. (a) The output at C corresponds to A NAND B or

82. (a) In case of an 'OR' gate the input is zero when all inputs A×B = C
are zero. If any one input is ' 1', then the output is '1'. 90. (a)
83. (c) Truth table for given logical circuit A
Y = A.AB B.AB
Y2 = A.AB
a b (a + b ) c Y = (a + b) .c
0 0 0 0 0 Y1 = AB
0 1 1 1 1
B
1 0 1 1 1
Y3 = B.AB
1 1 1 0 0 By expanding this Boolen expression
Output of OR gate must be 1 and c = 1
Y = A.B + B.A
So, a = 1, b = 0 or a = 0, b = 1.
Thus the truth table for this expression should be (a).
84. (a) It represents OR-Gate.
91. (c) Truth table is as shown :
A B A + B =Y
A 0 0 0 A B A B A+B A+B
B Y 0 1 1
1 0 1 0 0 1 1 1 0
1 1 1 0 1 1 0 1 0
85. (a) AND Gate 1 0 0 1 1 0
1 1 0 0 0 1
Semiconductor Electronics : Materials, Devices and Simple Circuits P-493

Thus the combination of two NOT gates and one NOR 95. (a) The final boolean expression of these gates is,
gate is equivalent to a AND gate.
X = ( A . B ) = A + B = A + B Þ OR gate
92. (a) NOR gate is the combination of NOT and OR gate.
Boolean expression for NOR gate is It means OR gate is formed.
96. (d) The final boolean expression
Y = A+ B
A B Y
93. (b) When either of A or B is 1 i.e. closed then lamp will
0 0 0
glow.
0 1 0
In this case, Truth table 1 0 0
1 1 1
Inputs Output

A B Y Y = ( A + B ) = A.B = A × B .
0 0 0 Thus, it is an AND gate for which truth
0 1 1 table is
97. (d) A
1 0 1
1 1 1 C

This represents OR gate. B


94. (b) When both inputs of NAND gate are jointed to form The truth table for the above circuit is :
a single input, it behaves as NOT gate
A B C
OR + NOT = NOR. 1 1 1

( A + B) = NOR gate 1
0
0
1
1
1
A 0 0 0
B (A+B) when either A or B conducts, the gate conducts. It means
(A+B) C = A + B which is for OR gate.
29
P-494 Physics

Communication
Systems
6. The wavelength of the carrier waves in a modern optical
TOPIC 1 Communication Systems fiber communication network is close to :
[8 April 2019 I]
(a) 2400 nm (b) 1500 nm (c) 600 nm (d) 900 nm
1. An amplitude modulated wave is represented by the
7. In a line of sight ratio communication, a distance of about
expression vm = 5(1 + 0.6 cos 6280t ) sin(211´104 t ) volts 50 km is kept between the transmitting and receiving
The minimum and maximum amplitudes of the amplitude antennas. If the height of the receiving antenna is 70m,
modulated wave are, respectively : [Sep. 02, 2020 (I)] then the minimum height of the transmitting antenna
should be : [8 April 2019 II]
3 5 (Radius of the Earth = 6.4 × 106 m).
(a) V, 5 V (b) V, 8 V
2 2 (a) 20 m (b) 51 m
(c) 5 V, 8 V (d) 3 V, 5 V (c) 32 m (d) 40 m
2. In an amplitude modulator circuit, the carrier wave is given 8. A 100 V carrier wave is made to vary between 160 V and
by, C(t) = 4 sin (20000 pt) while modulating signal is given 40 V by a modulating signal. What is the modulation index?
by, m(t) = 2 sin (2000 pt). The values of modulation index [12 Jan. 2019 I]
and lower side band frequency are : [12 April 2019 II] (a) 0.3 (b) 0.5 (c) 0.6 (d) 0.4
(a) 0.5 and 10 kHz (b) 0.4 and 10 kHz 9. To double the covering range of a TV transmittion tower,
(c) 0.3 and 9 kHz (d) 0.5 and 9 kHz its height should be multiplied by: [12 Jan 2019 II]
3. A message signal of frequency 100 MHz and peak 1
(a) (b) 2 (c) 4 (d) 2
voltage100 V is used to execute amplitude modulation on 2
a carrier wave of frequency 300 GHz and peak voltage 400 10. An amplitude modulated signal is given by V(t) = 10[1 +
V. The modulation index and difference between the two 0.3 cos (2.2 × 104t)]sin(5.5 × 105t). Here t is in seconds.
side band frequencies are : [10 April 2019 II] The sideband frequencies (in kHz) are, [Given p = 22/7]
(a) 4 ; 1×108 Hz (b) 4 ; 2×108 Hz [11 Jan 2019 II]
(c) 0.25 ; 2×108 Hz (d) 0.25 ; 1 × 10–8T
(a) 1785 and 1715 (b) 178.5 and 171.5
4. A signal Acoswt is transmitted using v0 sin w0t as carrier
wave. The correct amplitude modulated (AM) signal is: (c) 89.25 and 85.75 (d) 892.5 and 857.5
[9 April 2019 I] 11. An amplitude modulated signal is plotted below :
V (t)
(a) v0 sinw0 t + A sin ( w 0 w ) t + A sin ( w 0 + w ) t 10 V
2 ¯ 2 8V
(b) v0 sin[w0(1 + 0.01 Asinwt)t]
t
(c) v0 sinw0t + Acoswt
(d) (v0 + A) coswt sinw0t 8 ms
5. The physical sizes of the transmitter and receiver antenna 100 ms
in a communication system are: [9 April 2019 II] Which one of the following best describes the above
(a) independent of both carrier and modulation frequency signal? [11 Jan. 2019 II]
(b) inversely proportional to carrier frequency
(c) inversely proportional to modulation frequency (a) (9 + sin (2.5 p × 105 t)) sin (2p × 104t)V
(d) proportional to carrier frequency (b) (1 + 9 sin (2p × 104t)) sin (2.5 p × 105t) V
Communication Systems P-495

(c) (9 + sin (2p× 104t)) sin (2.5 p × 105t)V 20. A signal is to be transmitted through a wave of wave-
length l, using a linear antenna. The length 1 of the an-
(d) (9 + sin (4p × 104t)) sin (5p × 105t) V
tenna and effective power radiated Peff will be given re-
12. A TV transmission tower has a height of 140 m and the
spectively as :
height of the receiving antenna is 40 m. What is the (K is a constant of proportionality)
maximum distance upto which signals can be broadcasted [Online April 9, 2017]
from this tower in LOS (Line of Sight) mode? (Given : 2
æ1ö l æ1ö
radius of earth = 6.4 × 106 m). [10 Jan. 2019 I] (a) l, Peff = K ç ÷ (b) , Peff = K ç ÷
èlø 8 èlø
(a) 65 km (b) 48 km (c) 80 km (d) 40 km 1
l 3 l æ 1 ö2
13. The modulation frequency of an AM radio station is 250 , Peff = K æç ö÷
1
(c) (d) , Peff = K ç ÷
kHz, which is 10% of the carrier wave. If another AM sta- 16 èlø 5 èlø
tion approaches you for license what broadcast frequency 21. A signal of frequency 20 kHz and peak voltage of 5 Volt is
will you allot? [10 Jan. 2019 I] used to modulate a carrier wave of frequency 1.2 MHz and
(a) 2750 kHz (b) 2900 kHz peak voltage 25 Volts. Choose the correct statement.
(c) 2250 kHz (d) 2000 kHz [Online April 8, 2017]
14. In a communication system operating at wavelength 800 (a) Modulation index = 5, side frequency bands are at
nm, only one percent of source frequency is available as 1400 kHz and 1000 kHz
signal bandwidth. The number of channels accomodated (b) Modulation index = 5, side frequency bands are at
for transmitting TV signals of band width 6 MHz are (Take 21.2 kHz and 18.8 kHz
velocity of light c = 3 × 108m/s, h = 6.6 × 10–34J-s) (c) Modulation index=0.8, side frequency bands are at
[9 Jan. 2019 II] 1180 kHz and 1220 kHz
(a) 3.75 × 106 (b) 3.86 × 106 (d) Modulation index=0.2, side frequency bands are at
(c) 6.25 × 105 (d) 4.87 × 105 1220 kHz and 1180 kHz
15. A telephonic communication service is working at carrier
22. Choose the correct statement : [2016]
frequency of 10 GHz. Only10% of it is utilized for
transmission. How many telephonic channels can be (a) In frequency modulation the amplitude of the high
transmitted simultaneously if each channel requires a frequency carrier wave is made to vary in proportion
bandwidth of 5 kHz? [2018] to the amplitude of the audio signal.
(a) 2 × 103 (b) 2 × 104 (c) 2 × 105 (d) 2 × 106 (b) In frequency modulation the amplitude of the high
16. A carrier wave of peak voltage 14 V is used for transmitting frequency carrier wave is made to vary in proportion
a message signal. The peak voltage of modulating signal to the frequency of the audio signal.
given to achieve a modulation index of 80% will be: [2018]
(a) 11.2 V (b) 7 V (c) 22.4 V (d) 28 V (c) In amplitude modulation the amplitude of the high
17. The number of amplitude modulated broadcast stations frequency carrier wave is made to vary in proportion
that can be accommodated in a 300 kHz band width for the to the amplitude of the audio signal.
highest modulating frequency 15 kHz will be: (d) In amplitude modulation the frequency of the high
[Online April 15, 2018] frequency carrier wave is made to vary in proportion
(a) 20 (b) 10 (c) 8 (d) 15 to the amplitude of the audio signal.
18. The carrier frequency of a transmitter is provided by a
tank circuit of a coil of inductance 49mH and a capactiance 23. A modulated signal Cm(t) has the form Cm(t) = 30 sin 300pt
of 2.5nF . It is modulated by an audio signal of 12kHz. The + 10 (cos 200pt – cos 400pt). The carrier frequency fc, the
frequency range occupied by the side bands is: modulating frequency (message frequency) fw and the
[Online April 15, 2018] modulation indix m are respectively given by :
(a) 18kHz – 30kHz (b) 63kHz – 75kHz [Online April 10, 2016]
(c) 442kHz – 466kHz (d) 13482kHz – 13494kHz 1
19. In amplitude modulation, sinusoidal carrier frequency used (a) fc = 200 Hz; fw = 50 Hz; m =
2
is denoted by ωc and the signal frequency is denoted by 2
(b) fc = 150 Hz; fw = 50 Hz; m =
ω m . The bandwidth ( Dωm ) of the signal is such that Dωm 3
< ωc . Which of the following frequencies is not contained 1
(c) fc = 150 Hz; fw = 30 Hz; m =
in the modulated wave ? [2017] 3
(a) ω m + ω c (b) ω c - ω m 1
(d) fc = 200 Hz; fw = 30 Hz; m =
(c) ω m (d) ωc 2
P-496 Physics

24. An audio signal consists of two distinct sounds: one a (b) The critical frequency of an ionospheric layer is the
human speech signal in the frequency band of 200 Hz to highest frequency that will be reflected back by the
2700 Hz, while the other is a high frequency music signal layer when it is vertically incident.
in the frequency band of 10200 Hz to 15200 Hz. The ratio of (c) Electromagnetic waves of frequencies higher than
the AM signal bandwidth required to send both the signals about 30 MHz cannot penetrate the ionosphere.
together to the AM signal bandwidth requried to send
(d) Sky wave signals in the broadcast frequency range
just the human speech is : [Online April 9, 2016]
are stronger at night than in the day time.
(a) 2 (b) 5 (c) 6 (d) 3
32. This question has Statement-1 and Statement-2. Of the
25. A signal of 5 kHz frequency is amplitude modulated on a
four choices given after the Statements, choose the one
carrier wave of frequency 2 MHz. The frequencies of the
that best describes the two Statements.
resultant signal is/are : [2015]
Statement-1: Short wave transmission is achieved due to
(a) 2005 kHz, 2000 kHz and 1995 kHz
the total internal reflection of the e-m wave from an
(b) 2000 kHz and 1995 kHz
appropriate height in the ionosphere.
(c) 2 MHz only
Statement-2: Refractive index of a plasma is independent
(d) 2005 kHz and 1995 kHz
of the frequency of e-m waves. [Online April 22, 2013]
26. Long range radio transmission is possible when the radio
(a) Statement-1 is true, Statement-2 is false.
waves are reflected from the ionosphere. For this to happen
the frequency of the radio waves must be in the range: (b) Statement-1 is false, Statement-2 is true.
[Online April 19, 2014] (c) Statement-1 is true, Statement-2 is true but
Statement -2 is not the correct explanation of
(a) 80 - 150 MHz (b) 8 - 25 MHz
statement-1.
(c) 1 - 3 MHz (d) 150 - 1500 kHz
(d) Statement-1 is true, Statement-2 is true and
27. For sky wave propagation, the radio waves must have a
Statement -2 is the correct explanation of Statement-1.
frequency range in between: [Online April 12, 2014]
33. If a carrier wave c(t) = A sin wct is amplitude modulated by
(a) 1 MHz to 2 MHz (b) 5 MHz to 25 MHz
a modulator signal m(t) = A sin wmt then the equation of
(c) 35 MHz to 40 MHz (d) 45 MHz to 50 MHz modulated signal [Cm(t)] and its modulation index are
28. A transmitting antenna at the top of a tower has height 32 respectively [Online April 9, 2013]
m and height of the receiving antenna is 50 m. What is the (a) Cm (t) = A (1 + sin wm t) sin wc t and 2
maximum distance between them for satisfactory (b) Cm (t) = A (1 + sin wm t) sin wm t and 1
communication in line of sight (LOS) mode? (c) Cm (t) = A (1 + sin wm t) sin wc t and 1
[Online April 9, 2014] (d) Cm (t) = A (1 + sin wc t) sin wm t and 2
(a) 55.4 km (b) 45.5 km (c) 54.5 km (d) 455 km 34. A radar has a power of 1kW and is operating at a frequency
29. A diode detector is used to detect an amplitudemodulated of 10 GHz. It is located on a mountain top of height 500 m.
wave of 60% modulation by using a condenser of capacity The maximum distance upto which it can detect object
250 picofarad in parallel with a load resistance 100 kilo located on the surface of the earth
ohm. Find the maximum modulated frequency which could (Radius of earth = 6.4 × 106m) is : [2012]
be detected by it. [2013] (a) 80 km (b) 16 km (c) 40 km (d) 64 km
D 35. A radio transmitter transmits at 830 kHz. At a certain
distance from the transmitter magnetic field has amplitude
4.82 × 10–11T. The electric field and the wavelength are
Signal C R respectively [Online May 26, 2012]
(a) 0.014 N/C, 36 m (b) 0.14 N/C, 36 m
(c) 0.14 N/C, 360 m (d) 0.014 N/C, 360 m
(a) 10.62 MHz (b) 10.62 kHz
(c) 5.31 MHz (d) 5.31 kHz 36. Given the electric field of a complete amplitude modulated
wave as
30. Which of the following modulated signal has the best
æ ö
noise-tolerance ? [Online April 25, 2013] ˆ c 1 + Em cos w m t cos w c t .
®
E = iE çè ÷ø
(a) Long-wave (b) Short-wave Ec
(c) Medium-wave (d) Amplitude-modulated Where the subscript c stands for the carrier wave and m
31. Which of the following statement is NOT correct? for the modulating signal. The frequencies present in the
[Online April 23, 2013] modulated wave are [Online May 19, 2012]
(a) Ground wave signals are more stable than the sky (a) w c and wc2 + w 2m
wave signals.
(b) wc , wc + wm and wc - wm
Communication Systems P-497

(c) wc and w m (a) Statement–1 is true, Statement–2 is true, Statement–2


is the correct explanation of Statement–1.
(d) wc and wc wm
(b) Statement–1 is true, Statement–2 is true, Statement–2
37. A 10 kW transmitter emits radio waves of wavelength 500 is not the correct explanation of Statement – 1.
m. The number of photons emitted per second by the (c) Statement – 1 is false, Statement – 2 is true.
transmitter is of the order of [Online May 12, 2012]
(d) Statement – 1 is true, Statement – 2 is false.
(a) 1037 (b) 1031 (c) 1025 (d) 1043
39. Which of the following four alternatives is not correct ?
38. This question has Statement – 1 and Statement – 2. Of the We need modulation : [2011 RS]
four choices given after the statements, choose the one
(a) to reduce the time lag between transmission and
that best describes the two statements. [2011]
reception of the information signal
Statement – 1 : Sky wave signals are used for long
(b) to reduce the size of antenna
distance radio communication. These signals are in general,
less stable than ground wave signals. (c) to reduce the fractional band width, that is the ratio of
the signal band width to the centre frequency
Statement – 2 : The state of ionosphere varies from
hour to hour, day to day and season to season. (d) to increase the selectivity
P-498 Physics

1. (b) From the given expression, 10. (c) Equation given


Vm = 5 (1 + 0.6 cos 6280t) sin(211 × 104t)
V (t) = 10 [1 + 0.3 cos (2.2 × 104)]
Modulation index, m = 0.6
sin (5.5 × 105t)
Q Am = mAc
= 10 + 1.5 [sin (57.2 × 104t) + sin (52.8 × 104t)]
Amax + Amin
= Ac = 5 ...(i) wc + ww = 57.2 × 104 = 2pf1
2
Amax - Amin 57.2 ´104
= Am = 3 ...(ii) f1 = = 9.1´104 ; 91KHz
2 æ 22 ö
2´ ç ÷
From equation (i) + (ii), è 7 ø
Maximum amplitude, Amax = 8. wc – ww = 52.8 × 104
From equation (i) – (ii),
52.8 ´10 4
Minimum amplitude Amin = 2. f2 = ; 84 KHz
æ 22 ö
Am 2 2´ ç ÷
2. (d) Modulation index, µ = A = 4 = 0.5 è 7 ø
c
20000 p
Given, fe = = 10000 Hz.
2p
2000p
and fm = = 1000 Hz.
2p
\ LSB = fe – fm = 10000 – 1000 = 9000 Hz.
3. (c) Range of frequency = (fc – fm) to (fc + fm)
\ Band width = 2fm = 2 × 100 × 106 Hz fc–fw fc fc+fw
= 2 × 108 Hz Upper side band frequency (f1) is
A m 100 52.8 ´104
and Modulation index = = = 0.25 f1 = fc – fw = » 85.00 kHz
Ac 400 2p
4. (a)
Lower side band frequency (f2) is
5. (b) Size of antenna,
l C 1 57.2 ´10 4
l = . As l = \l µ f2 = fc + fw = » 90.00 kHz
4 f f 2p
6. (b) Carrier waves of wavelength 1500 nm is used in 11. (c) After analysing the graph we may conclude that
modern optical fiber communication.
(i) Amplitude varies as 8–10 V or 9 ± 1
7. (c) LOS = 2hT R + 2hR R
(ii) Two time period as
or 50 × 103 = 2 hT ´ 6.4 ´ 106 + 2 ´ 70 ´ 6.4 ´ 406
100 ms (signal wave) & 8 ms (carrier wave)
On solving, hT = 32 m
8. (c) Maximum amplitude = Em + Ec = 160 So, equation of AM signal is
Em + 100 = 160
é æ 2 pt ö æ 2 pt ö ù
Em = 160 – 100 = 60 ê9 ± 1sin ç sin ç ú
è T1 ø ÷ è T2 ÷ø û
Modulation index, ë
= [9 ± sin (2p × 104 t)] sin (2.5p × 105 t)V]
E m 60
m= = 12. (a) Maximum distance upto which signal can be
Ec 100 broadcasted
µ = 0.6
d max = 2Rh T + 2Rh R
9. (c) As we know, Range = 2hR where hT and hR are heights of transmission tower and
receiving antenna respectively.
therefore to double the range height ‘h’ should be 4 times.
Communication Systems P-499

Putting values R, hT and hR Therefore frequency range 454.54 ± 12kHz


d max = 2×6.4×106 éë 140 + 40 ùû i.e., 442 kHz – 466 kHz
19. (c) Modulated carrier wave contains frequency wc and
or, d max ; 65 km wc ± w m
20. (a) Length of antenna = comparable to l
13. (d) According to question, modulation frequency, 250
Power radiated by linear antenna inversely depends on
Hz is 10% of carrier wave
the square of wavelength and directly on the length of the
250 antenna. Hence,
f carrier = = 2500KHZ
0.1 2
Power P = m æç ö÷
1
\ Range of signal 2500 ± 250 KHz = 2250 Hz to 2750 Hz For è lø
2000 KHZ here µ = K
f mod = 200 Hz Vm 5
21. (d) Modulation index (m) = = = 0.2
\ Range = 1800 KHZ to 2200 KHZ V0 25
Given, frequency of carrier wave (fc) = 1.2 × 106 Hz
V 3 ´ 108 30 = 1200 kHz.
14. (c) Frequency, f = = = ×1014 Hz
l 8 ´ 10-7 8 Frequency of signal (f0) = 20 kHz.
Side frequency bands = fc ± f0
= 3.75 × 1014 Hz
f1 = 1200 – 20 = 1180 kHz
1% of f = 0.0375 × 1014 Hz
= 3.75 × 1012 Hz = 3.75 × 106 MHz f2 = 1200 + 20 = 1220 kHz
As we know, number of channels accomodated for 22. (c) In amplitude modulation, the amplitude of the high
frequency carrier wave made to vary in proportional to the
transmission = amplitude of audio signal.
Audio signal
total bandwidth of Channel 3.75 ´106
= = 6.25 × 105
bandwidth needed per channel 6
Carrier wave
15. (c) If n = no. of channels
10% of 10 GHz = n × 5 KHz or, Amplitude modulated wave
Þ n = 2 × 105
16. (a) Given : modulation index m = 80% = 0.8
23. (b) Comparing the given equation with standard
Ec = 14 V, Em = ?
mA c
modulated signal wave equation, m = Ac sin wc t +
Em 2
using, m = Þ E m = m ´ E c = 0.8 ´ 14 = 11.2V
Ec mA c
cos (wc – ws) t – cos (wc + ws) t
17. (b) Given, modulating frequency fm = 15 KHz 2
\ Bandwidth of one channel = 2fm = 30 kHz Ac 2
m < 10 Þ m < (modulation index)
300kHz 2 3
\ No of channels accommodate = = 10 Ac = 30
30kHz wc – ws = 200p
18. (c) Given : Inductance, L = 49 µH = 49 × 10–6 H, wc + ws = 400p
capacitance C = 2.5 nF = 2.5 × 10–9 F
Þ f c = 150, fs = 50 Hz .
1 24. (c) Ratio of AM signal Bandwidths
Using w =
LC
15200 , 200 15000
1 1 108 < < < 6.
= = = 2700 , 200 2500
2.5 7 ´ 5 ´ 10-8 7´5
49 ´ 10 -6 ´ ´ 10 -9 25. (a) Amplitude modulated wave consists of three
10
frequencies are wc + wm, w,wc – wm
108 22 i.e. 2005 kHz, 2000kHz, 1995 kHz
or, = 2p ´ f = 2 ´ ´f (Q w = 2pf )
7 ´5 7 26. (b) Frequency of radio waves for sky wave propagation
107 104 is 2 MHZ to 30 MHZ.
or, f = = kHz = 454.54kHz
22 22
P-500 Physics

27. (b) Sky wave propagation is suitable for frequency range 34. (a) Let d is the maximum distance, upto which it can
5 MHz to 25 MHz. detect the objects C
28. (b) Given : hR = 32 m From DAOC
h
hT = 50 m d
2 2 2 B
Maximum distance, dM = ? OC = AC + AO
A q R
Applying, dM = 2Rh T + 2Rh R (h + R)2 = d 2 + R 2 R
O
= 2 ´ 6.4 ´106 ´ 50 + 2 ´ 6.4 ´106 ´ 32 = 45.5 km Þ 2 2 2
d = (h + R) - R
29. (b) Given : Resistance R = 100 kilo ohm
= 100 × 103 W
d = (h + R)2 - R2 ; d = h2 + 2hR
Capacitance C = 250 picofarad
= 250 × 10–12F d = 5002 + 2 ´ 6.4 ´ 10 6 = 80 km
t = RC = 100 × 103 × 250 × 10–12 sec 35. (d) Frequency of EM wave u = 830 KHz
= 2.5 × 107 × 10–12 sec = 830 × 103 Hz.
= 2.5 × 10–5 sec Magnetic field, B = 4.82 × 10–11 T
The higher frequency whcih can be detected with tolerable c
As we know, frequency, u =
distortion is l
1 1 c 3 ´ 108
f = = Hz or l = =
2pma RC 2 p ´ 0.6 ´ 2.5 ´ 10 -5 v 830 ´ 103
l ; 360 m
100 ´ 10 4 4 And, E = BC = 4.82 × 10–11 × 3 × 108
= Hz = ´ 10 4 Hz
25 ´ 1.2p 1.2 p = 0.014 N/C
= 10.61 KHz 36. (b) The frequencies present in amplitude modulated
wave are :
This condition is obtained by applying the condition that
Carrier frequency = wc
rate of decay of capacitor voltage must be equal or less
than the rate of decay modulated singnal voltage for proper Upper side band frequency = wc + wm
detection of mdoulated signal. Lower side band frequency = wc – wm.
30. (b) Short-wave has the best noise tolerance. nhc
37. (b) Power =
31. (c) Above critical frequency (fc), an electromagnetic l
wave penetrates the ionosphere and is not reflected by it. (where, n = no. of photons per second)
32. (a) Effective refractive index of the ionosphere 10 ´ 103 ´ 500
Þ n= ; 1031
1/ 2 -34 8
é 80.5N ù 6.6 ´ 10 ´ 3 ´ 10
n eff = n 0 ê1 -
ë f 2 úû 38. (b) For long distance communication, sky wave signals
where f is the frequency of em waves are used.
33. (c) Modulation index Also, the state of ionosphere varies every time.
Em A So, both statements are correct.
ma = = =1 39. (a) Low frequencies cannot be transmitted to long
Ec A
distances. Therefore, they are super imposed on a high
Equation of modulated signal [Cm(t)] frequency carrier signal by a process known as modulation.
= E(C) + maE(C) sin wmt Speed of electro-magnetic waves will not change due to
= A (1+ sin wCt) sin wmt modulation. So there will be time lag between transmission
(As E(C) = A sinwCt) and reception of the information signal.
Mock Test-1 MT-1

1
PART-I (Multiple Choice Questions) (c) Resistance of both decrease

1. A d.c. supply of 120V is connected to a large resistance X. (d) Resistnance of both increase
A voltmeter of resistance 10kW placed in series in the circuit 4. When white light passes through a dispersive medium, it
reads 4V. What is the value of X? breaks up into various colours. Which of the following
(a) 190 kW (b) 90 kW statements is true?

(c) 290 kW (d) 390 kW (a) Velocity of light for violet is greater than the velocity
of light for red colour.
2. The period of the satellite of the earth orbiting very near to
the surface of the earth is T0. What is the period of the (b) Velocity of light for violet is less than the velocity of
geostationary satellite in terms of T0 light for red.
T0 (c) Velocity of light is the same for all colours
(a) (b) 7T0
7
(d) Velocity of light for differnet colours has nothing to
(c) 7T0 (d) 7 7T0
do with the phenomenon of dispersion
3. In “Al” and “Si”. If temperature is changed from normal 5. An engine has an efficiency of 1/6. When the temperature
temperature to 70 K then
of sink is reduced by 62°C, its efficiency is doubled.
(a) The resistance of Al will increase and that of Si will Temperatures of source and sink are
decrease (a) 99°C, 37°C (b) 124°C, 62°C
(b) The resistance of Al will decrease and that of Si will (c) 37°C, 99°C (d) 62°C, 124°C
increase
MT-2 Mock Test-1

6. Two isothermals are shown in figure at temperatures T1 x(m)


and T2. Which of the following relations is correct?
3

P 0 4 t(s)

(a) 0,0 (b) 0, – 3 kg ms – 1


T1
T2 (c) + 3 kg ms–1, 0 (d) + 3 kg ms– 1, – 3 kg. ms–1
10. In case of a p-n junction diode at high value of reverse
V bias, the current rises sharply. The value of reverse bias is
(a) T1 > T2 (b) T1 < T2 known as
(a) cut off voltage (b) zener voltage
1
(c) T1 = T2 (d) T1 = T (c) inverse voltage (d) critical voltage
2 2
11. A photon materializes into an electron-positron pair. The
7. During the experiment to determine the resistivity of a wire
kinetic energy of the electron is found to be 0.19 MeV.
by a metre bridge, the jockey is moved gently along the
What was the energy of the photon?
wire from left to right to
(a) find a deflection in the galvanometer towards left (a) 0.38 MeV (b) 0.70 MeV
(b) find a deflection in the galvanometer towards right (c) 1.40 MeV (d) None
(c) find no deflection in the galvanometer 12. If 10% of a radioactive material decays in 5 days, then the
(d) find a maximum deflection in the galvanometer amount of the original material left after 20 days is
8. The circular head of a screw gauge is divided into 200 approximately
divisions and move 1 mm ahead in one revolution. If the (a) 60% (b) 65%
same instrument has a zero error of –0.05 mm and the reading (c) 70% (d) 75%
on the main scale in measuring diameter of a wire is 6 mm 13. Lights of two different frequencies, whose photons have
and that on circular scale is 45. The diameter of the wire is energies 1 eV and 2.5 eV respectively, successively
(a) 6.275 mm (b) 6.375 mm (c) 5.75 mm (d) 5.50 mm illuminate a metal whose work function is 0.5 eV. The ratio
9. The adjacent figure shows the position graph of one of the maximum speeds of the emitted electrons will be
dimensional motion of a particle of mass 4 kg. The impulse (a) 1 : 5 (b) 1 : 4
at t =0 s and t = 4 is given respectively as: (c) 1 : 2 (d) 1 : 1
Mock Test-1 MT-3

14. A block of mass 15 kg is placed over a frictionless horizontal


æ 3 ˆ 4 ˆö
surface. Another block of mass 10 kg is placed over it, that (a) ( -3jˆ + 4ˆj) N (b) ç - i + j ÷ N
è 5 5 ø
is connected with a light string passing over two pulleys
fastened to the 15 kg block. A force F = 80N is applied
æ 3ˆ 4 ˆö
horizontally to the free end of the string. Friction coefficient (c) ( 3iˆ - 4ˆj) N (d) ç i - j ÷ N
è5 5 ø
between two blocks is 0.6. The portion of the string
between 10 kg block and the upper pulley is horizontal. 16. A plate of mass (M) is placed on a horizontal frictionless
Pulley, string and connecting rods are massless. (Take g = surface and a body of mass (m) is placed on this plate. The
10 m/s²) coefficient of dynamic friction between this body and the
plate is m. If a force 3mmg is applied to the body of mass (m)
along the horizontal, the acceleration of the plate will be

10kg mm mmg 3mmg 2mmg


(a) g (b) (c) (d)
M M+m M M+m

µ=0.6 17. Two rods of length d1 and d2 and coefficients of thermal


15 kg
conductivities K1 and K2 are kept touching each other.
Both have the same area of cross-section the equivalent
F = 80N of thermal conductivity is
(a) K1 + K2 (b) K1d1 + K2d2
Smooth
d1K1 + d 2 K 2 d1 + d 2
(c) (d) (d / K + d / K )
d1 + d 2 1 1 2 2

The magnitude of acceleration of the 10 kg block is 18. A particle starts S.H.M. from the mean position. Its
(a) 3.2 m/s² (b) 2.0 m/s² (c) 1.6 m/s² (d) 0.8 m/s² amplitude is a and total energy E. At one instant5 its kinetic
15. A body of mass 5 kg under the action of constant force energy is 3 E/4, its displacement at this instant is

® ® a a
( )
F = Fx ˆi + Fy ˆj has velocity at t = 0 s as v = 6iˆ - 2ˆj m/s (a) y =
2
(b) y =
2

®
and at t = 10s as v = +6jˆ m / s . The force ®
F is:
a
(c) y = (d) y = a
3/ 2
MT-4 Mock Test-1

19. Two long parallel wires P and Q are both perpendicular to 22. The radius of curvature of a thin plano-convex lens is 10
the plane of the paper with distance of 5 m between them. cm (of curved surface) and the refractive index is 1.5. If the
If P and Q carry current of 2.5 amp and 5 amp respectively plane surface is silvered, then it behaves like a concave
in the same direction, then the magnetic field at a point mirror of focal length (in cm)
half-way between the wires is 23. Two masses A and B of 10 kg and 5 kg respectively are
connected with a string passing over a frictionless pulley
3m 0 m0
(a) (b) fixed at the corner of a table (as shown in figure). The
2p p
coefficient of friction between the table and the block is
0.2. The minimum mass (in kg) of C that may be placed on
3m 0 m0
(c) (d) A to prevent it from moving is equal to
2p 2p
20. In a circuit L, C and R are connected in series with an C
alternating voltage source of frequency f. The current leads
the voltage by 45°. The value of C is A

1 1
(a) pf (2 pfL - R) (b) 2 pf (2 pfL - R) B

1 1
(c) pf (2 pfL + R ) (d) 2 pf (2 pfL + R) 24. Ammeter and voltmeter readings were recorded as 0.25 A
and 0.5 V during the experiment to determine the resistance
of a given wire using Ohm's law. What is the correct value
PART-II (Numerical Answer Questions)
of the resistance (in ohm)?
21. An insect trapped in a circular groove of radius 12 cm. 25. A wheel is rotating at 900 r.p.m. about its axis. When power
moves along the groove steadily and completes 7 is cut off it comes to rest in 1 minute. The angular retardation
revolutions in 100 sec. What is the linear speed (in m/s) of in (rad/s2) is :
the motion ?
Mock Test-1 MT-5

RESPONSE SHEET

1. a b c d 2. a b c d 3. a b c d 4. a b c d 5. a b c d

6. a b c d 7. a b c d 8. a b c d 9. a b c d 10. a b c d

11. a b c d 12. a b c d 13. a b c d 14. a b c d 15. a b c d

16. a b c d 17. a b c d 18. a b c d 19. a b c d 20. a b c d

21. 22. 23. 24. 25.


MT-6 Solutions Mock Test -1

HINTS & SOLUTIONS


MOCK TEST-1

120V
1. (c) Given (10 kW) I = 4V OR 10 kW . = 4 V OR 8. (a) Pitch = 1 mm
X + 10kW
Number of divisions on circular scale = 200
300 kW = X + 10 kW;
X = 290 kW. Pitch
L.C =
Number of divisions on circular scale
120V X 1 mm
= = 0.005 mm = 0.0005 cm
200
Diameter of the wire = (Main scale reading + Circular
scale reading × L.C.) – zero error
10kW = 6 mm + 45 × 0.005 – (– 0.05)
V
= 6 mm + 0.225 mm + 0.05 mm = 6.275 mm
3
2 3 2 3 9. (d) t < 0, vi = 0 and t > 0, vf = m/s
2. (d) T 2 µ r 3 law; T0 µ R & Ts µ (> 7R) ; 4

æ3 ö
æ Ts ö
2
\ Impulse = m (vf –vi) = 4 ç - 0÷
ç ÷ = 7 3 ; Ts = 7 7 T0 è4 ø
çT ÷
è 0 ø = 3 kg ms –1
3
3. (b) T ¯ (300K to 70K ) Little before 4 second v = m / s, little after 4
4
T ¯ R metal ¯ R ­ semi - conductor second, velocity becomes zero
Therefore, Impulse
(Al) (Si)
æ 3ö
4. (b) As mv > mr therefore, vv < vr. = m(vf - vi ) = 4 ç 0 - ÷ = -3kg ms -1
è 4ø
10. (b) In reverse bias on p-n junction when high voltage is
T2 T2 1 5
5. (a) From h = 1 - , = 1 - h = 1 - = ...(i) applied, electric break down of junction takes place,
T1 T1 6 6
resulting large increase in reverse current. This high
voltage applied is called zener voltage.
T2 - 62 2 2 11. (c) Conservation of Energy,
In 2nd case : = 1 - h' = 1 - = .....(ii)
T1 6 3 Photon ® e + + e - + energy

EP = E +E + K.E. of both e - + e +
2 2 6 4 re + re -
Using (i), T2 – 62 = T1 = ´ T2 = T2
3 3 5 5
We know that rest mass energy of e + + e - is .5 MeV
Hence E P = .5 + .5 + .19 ´ 2 = 1.40MeV
1
or T2 = 62, T2 = 310 K = 310 – 273 = 37°C
5 12. (b) Let initial amount be 100 gm.
disintegrated Left
6 6 5 days 100 ´ 10
T1 = T2 = ´ 310 = 372 K = 372 - 273 = 99°C 100 gm ¾¾ ¾® 10 90
5 5 100
Next 5 days 90 ´ 10
6. (a) As isothermal at T1 is farther from the origin than the 90 ¾¾ ¾ ¾¾® 9 81
100
isothermal at T2, therefore, T1 > T2
Next 5 days81´10
81 ¾¾ ¾¾¾® 8.1 » 73
7. (c) The jockey is moved so as to get the null point (i.e., 100
no deflection in the galvanometer). Next 5 days 73 ´ 10
73 ¾¾ ¾ ¾¾® 7.3 » 65
100
Solutions Mock Test -1 MT-7

13. (c) Einstein equation KEmax = E – Work function; Force, F = ?


1
mv2 = E - W v -u
2 Acceleration, a =
t
Using this concept,

1
6 ˆj - (6iˆ - 2 ˆj ) -3iˆ + 4 ˆj
mV12 max = = m/s2
2 1 - .5 1 V max 1 10 5
= = or 1 =
1 2 2.5 - .5 4 V2 max 2
V2 max Force, F = ma
2

14. (a) First, let us check upto what value of F, both blocks ( -3iˆ + 4 ˆj )
= 5´ = ( -3iˆ + 4 ˆj ) N
move together. Till friction becomes limiting, they will 5
be moving together. Using the FBDs
mmg
16. (a) a =
M
F a1
f
F
f
F Body
15 kg m 3mmg
F fr
M plate
a2 fr

fr = mmg = Ma
10 kg block will not slip over the 15 kg block till
acceleration of 15 kg block becomes maximum as it is 17. (d) When two rods are connected in series
created only by friction force exerted by 10 kg block on
it. A(T1 - T2 )t A(T1 - T2 ) t
Q= =
d1 d 2 (d 1 + d 2 ) / K
a1 > a 2(max) +
K1 K1

F-f f
= for limiting condition as f maximum is 60N.
10 15 d1 + d 2 d1 d 2 (d1 + d 2 )
\ = + ; \K =
K K1 K 2 d1 d 2
F = 100 N +
K1 K 2
Therefore, for F = 80N, both will move together. 1
18. (b) Total energy, E = mw 2 a 2 ;
Their combined acceleration, by applying NLM using 2

both as system, F = 25a 3E 1


K.E. = = mw 2 (a 2 - y 2 ) .
4 2
80
a= = 3.2 m / s 2 3 a2 - y2
25 2 a2 a
So, = or y = or y = .
4 2 4 2
a
15. (a) From question,
19. (a) When current flow in both wire in same direction then
Mass of body, m = 5 kg
magnetic field at half way due to P wire.
Velocity at t = 0,
uur m I m I m
BP = 0 1 = 0 1 = 0 (where I1= 5Amp)
u = (6iˆ - 2 ˆj) m/s 5 p .5 p
2p
2
Velocity at t = 10s,
The direction of Bp is downward
v = + 6 ĵ m/s
MT-8 Solutions Mock Test -1

22. (20) The silvered plano convex lens behaves as a concave


P Q mirror; whose focal length is given by
1 2 1
× = +
5 Amp 2.5 Amp F f1 f m
C
×

5m

Magnetic field at half way due to Q wire


If plane surface is silvered
m0I 2 m0
BQ = = [upward × ]
5 2p R2 ¥
2p fm = = =¥
2 2 2

[where I 2 = 2.5Amp. ] 1 æ 1 1 ö æ 1 1 ö m –1
\ = (m –1) ç – ÷ = (m – 1) ç – ÷ =
Net magnetic field at half way f1 R
è 1 R 2ø èR ¥ø R

m m 3m
B = B P + B Q = 0 + 0 = 0 (downward ) 1 2(m –1) 1 2(m –1) R
p 2p 2p \ = + = ÞF =
F R ¥ R 2(m – 1)
3m 0
Hence net magnetic field at midpoint = Here R = 20 cm, m = 1.5
2p
20
20. (d) From figure, \ F= = 20cm
2(1.5 –1)

23. (15) Let T be the tension in the string; f = frictional force


between block A and table; m’ = minimum mass of C.
For the just motion of block A on table
T = f = mR = m(m + m ' )g = 0.2(10 + m' )g ....(i)

For the just motion of block B, T = 5g ....(ii)


From (i) and (ii), 5g = 0.2 (10 + m’)g
1 5-2
- wL or 5 = 2 + 0.2m’ or m’ = = 15kg
w
tan 45º = C 0.2
R
V 0.5
24. (2) R= = = 2 W.
1 1 I 0.25
Þ - wL = R Þ = R + wL
wC wC
25. (p/2)Angular retardation,
1 1
C= =
w(R + w) 2 pf ( R + 2 pfL) w - w1 2p(n 2 - n1 ) 2p(0 - 900 / 60) p
a= 2 = = = rad / s 2 .
21. (5.3)This of example of uniform circular motion. t t 60 2

2p 7
w= = 2 pn = 2p ´ = 0.44 rad / sec . ;
T 100
V = Rw = 0.44 × 12 = 5.3 cm/sec.
Mock Test-2 MT-9

2
PART-I (Multiple Choice Questions) 3. A current of 4A produces a deflection of 30° in the
galvanometer. The figure of merit is
1. The horizontal component of impulse provided by string
tension in moving from extreme point to lower most (a) 6.5 A/rad (b) 7.6 A/rad
point – (c) 7.5 A/rad (d) 8.0 A/rad
/////////////// 4. A uniform disc of radius R and mass m is connected to a
wall by string of length 2R. The normal reaction of wall is –
q

(a) m 2gh

(b) m 2gh (1 - cos q)


(a) mg (b) mg/2 (c) mg/ 3 (d) 2mg
(c) m 2gh (cos q) 5. For the determination of the focal length of a convex mirror,
(d) can’t be found without knowing tension as a function a convex lens is required because
of time (a) it is not possible to obtain the image produced by a
2. N divisions on the main scale of a vernier callipers coincide convex mirror on the screen
with N + 1 divisions on the vernier scale. If each division (b) a convex lens has high resolving power so it helps to
on the main scale is of a units, determine the least count of measure the focal length correctly
the instrument. (c) a convex mirror always forms a real image which is
a a 2a 2a diminished by the convex lens
(a) (b) (c) (d)
N +1 N -1 N -1 N +1 (d) none of these
MT-10 Mock Test-2

6. A particle moves along x-axis with initial position x = 0. Its 8. A ring of mass M and radius R lies in x-y plane with its
velocity varies with x-coordinate as shown in graph. The centre at origin as shown. The mass distribution of ring is
acceleration ‘a’ of this particle varies with x as – nun-uniform such that at any point P on the ring, the mass
v per unit length is given by l = l0 cos2q (where l0 is a
positive constant). Then the moment of inertia of the ring
about z-axis is –
y

x
M P
R
a a
q x
(a) x (b) x

a a 1 1M 1 M
(a) MR2 (b) MR 2 (c) R (d) R
2 2 l 0 p l0
(c) x (d) x
9. What happens when we do not stir the mixture
continuously in the experiment to determine the specific
heat capacity of a given solid ?
(a) Temperatue of mixture is increased
7. Choose the wrong statement regarding the experiment to (b) Temperature of mixture is not constant
identify a diode, an LED, a resistor and a capacitor (c) Temperature of mixture is constant
(a) when pointer moves in one way when voltage is (d) None of these
applied and doesn't move when reversed and there is 10. An ideal monatomic gas with pressure P, volume V and
light emission, the item is LED temperature T is expanded isothermally to a volume 2V
(b) when pointer moves in one way when voltage is and a final pressure Pi. If the same gas is expanded
applied and doesn't move when reversed and there is adiabatically to a volume 2V, the final pressure is Pa. The
no emission of light, the item is a diode
P
(c) when pointer moves in one way when voltage is ratio a is
applied and not when reversed, the item is a resistor Pi
(d) none of these (a) 2–1/3 (b) 21/3 (c) 22/3 (d) 2–2/3
Mock Test-2 MT-11

11. In a Young’s double slit experiment with light of wavelength


1 ml
l, fringe pattern on the screen has fringe width b. When (d) is proportional to but ¹ p
two thin transparent glass (refractive index m) plates of E 3qE
thickness t1 and t2 (t1 > t2) are placed in the path of the 14. A sinusoidal wave is propagating in negative x-direction
two beams respectively, the fringe pattern will shift by a in a string stretched along x-axis. A particle of string at
distance x = 2m is found at its mean position and it is moving in
b ( m - 1) æ t1 ö positive y-direction at t = 1 sec. The amplitude of the wave,
mb t1
the wavelength and the angular frequency of the wave are
(a) l çè t ÷ø
2
(b) l t2
0.1 meter, p/2 meter and 2p rad/sec respectively.
b ( m - 1) l The equation of the wave is
(c) ( t1 - t2 ) (d) ( m - 1) ( t1 + t2 ) (a) y = 0.1 sin (4p (t – 1) + 8 (x – 2))
l b
r uuur (b) y = 0.1 sin ((t – 1) – (x – 2))
q
12. Consider Gauss’s law Ñò E.dA = Î0 . Which of the (c) y = 0.1 sin (2p (t – 1) + 4 (x – 2))
following is true ? (d) None of these
15. In an experiment, a small steel ball falls through a liquid at
a constant speed of 10 cm/s. If the steel ball is pulled
+ – + upward with a force equal to twice its effective weight,
how fast will it move upward ?
(a) 5 cm/s (b) Zero (c) 10 cm/s (d) 20 cm/s
r 16. A ball of mass 160 g is th rown up at an an gle
(a) E must be the electric field due to the enclosed charge of 60° to the horizontal at a speed of 10 ms–1. The angular
r momentum of the ball at the highest point of the trajectory
(b) If net charge inside the Gaussian surface = 0, then E
must be zero everywhere over the Gaussian surface. with respect to the point from which the ball is thrown is
(c) If the only charge inside the Gaussian surface is an nearly (g = 10 ms–2)
electric dipole, then the integral is zero. (a) 1.73 kg m2/s (b) 3.0 kg m2/s
r uuur (c) 3.46 kg m /s2 (d) 6.0 kg m2/s
(d) E is parallel to dA everywhere over the Gaussian surface. 17. A block of mass m is attached with massless spring of
13. A dipole consisting of two charges +q and –q joined by a force constant k. The block is placed over a fixed rough
massless rod of length l, is seen oscillating with a small inclined surface for which the coefficient of friction is
amplitude in a uniform electric field of magnitude E. The µ = 3/4. The block of mass m is initially at rest. The block of
period of oscillation mass M is released from rest with spring in unstretched
(a) is proportional to E state. The minimum value of M required to move the block
(b) is proportional to 1/E up the plane is (neglect mass of string and pulley and
ml friction in pulley.)
(c) is p
3qE
MT-12 Mock Test-2

(a) –10 cm. (b) –10/4 cm. (c) –10/3 cm. (d) None
PART-II (Numerical Answer Questions)
21. The activity of a radioactive substance drops to 1/32 of its
m M initial value in 7.5 h. Find the half life (in hours).
22. Six resistors of 10W each are connected as shown. The
37°
///////////////////////////////////////// equivalent resistance (in ohms) between points X and Y is –
3 4 6 3
(a) m (b) m (c) m (d) m X
5 5 5 2
18. A charged particle of mass m and having a charge Q is
placed in an electric field E which varies with time as E = E0
sin wt. What is the amplitude of the S.H.M. executed by Y
the particle?
QE 0 1 QE 0 23. The electric potential V is given as a function of distance x
(a) 2 (b)
2 mw 2 by V = (5x2 + 10x – 9) volt. Value of electric field (in v/m) at
mw
x = 1m is
2QE 0
(c) (d) None of these 24. Three moles of an ideal monoatomic gas perform a cycle
mw 2 shown in figure. The gas temperatures in different states
19. In a photoelectric experiment, with light of wavelength l, are T1 = 200K, T2 = 400K, T3 = 1600 K, and T4 = 800K. The
the fastest electron has speed v. If the exciting wavelength work done (in kJ) by the gas during the cycle is
is changed to 3l/4, the speed of the fastest emitted electron
(Take R = 25/3 J/mol-K)
will become –
P
3 4 2 3
(a) v (b) v
4 3
4 4
(c) less than v (d) greater than v
3 3
20. A thin convex lens of focal length 10cm and refractive 1
index 1.5 is cut vertically into two equal pieces. They are 4
placed as shown with a liquid of refractive index 3 between
them. What is the focal length of the combination ? T
25. A body is executing simple harmonic motion. At a
displacement x from mean position, its potential energy is
E1 = 2J and at a displacement y from mean position, its
potential energy is E2 = 8J. The potential energy E (in
Joule) at a displacement (x + y) from mean position is
MT-14 Solutions Mock Test -2

HINTS & SOLUTIONS

MOCK TEST-2

1. (a) Dp = mv = m 2gh
a
2. (a) Least count of vernier callipers
Hence the graph relating a to x is x
= value of one division of main scale – value of one
division of vernier scale
Now, N × a = (N + 1) a'
(a' = value of one division of vernier scale) 7. (c) For a resistor the pointer moves in both ways when
voltage is applied.
N
a¢ = a 8. (a) Divide the ring into infinitely small lengths of mass
N +1
dm1. Even though mass distribution is non-uniform,
a each mass dm1 is at same distance R from origin.
\ Least count = a - a ¢ =
N +1 \ MI of ring about z-axis is
3. (b) Here I = 4A = dm1R2 + dm2 R2 + ..............+ dmnR2 = MR2
9. (b) If temp of mixture (T) is not constant then according
c
æ 30 ´ p ö pc to calculation we find wrong value of specific heat. If
q =30° = ç ÷ =
è 180 ø 6 we stir continuously then temperature of whole mixture
becomes same.
I 4 4´6´7 2´6´7 10. (d) For isothermal process :
Now, k = = = =
q æ öp 22 11 PV = Pi .2V
ç ÷
è6ø P = 2Pi ...(i)
For adiabatic process
84
= = 7.6 A / rad PVg = Pa (2V)g
11
4. (c) (Q for monatomic gas g= 5 3 )

mg = T cos 30° 30° T 5 5


R or, [From (i)]
R N 2Pi V 3 = Pa (2V) 3
N = T sin 30°
-2
mg Pa 2 P
Þ N= Þ = 5 Þ a =2 3
3 mg Pi Pi
23

b ( m - 1) b ( m - 1)
5. (a) A convex mirror always forms a virtual image which 11. (c) Shift = t1 - t2
l l
can't be cast on a screen. Hence, a convex lens is
used to find the focal lenght of a convex mirror.
b ( m - 1)
6. (a) The linear relationship between v and x is =
l
( t1 - t2 )
v = – mx + C where m and C are positive constants.
Q net
dv 12. (c) f= For dipole Qnet = 0 Þ f = 0
\ Acceleration, a = v = -m ( -mx + C) Î0
dx

\ a = m2x – mC
Solutions Mock Test -2 MT-15

ml 2 v2 sin 2 q é v2 sin 2 q ù
13. (c) I= ; cos q êH = ú
12 = 2g úû
2g êë

d 2q
Restoring troque = qElsin q » qEl q = 102 ´ sin 2 60°´ cos 60°
dt 2 =
2 ´10

qEl qEl ´12 2p = 3.46 kg m2/s


\w = = =
I ml 2 T 17. (a) As long as the block of mass m remains stationary,
the block of mass M released from rest comes down
ml 2Mg
\T = p by (before coming it rest momentarily again).
3qE K
Thus the maximum extension in the spring is
14. (c) The equation of wave moving in negative x-direction,
assuming origin of position at x = 2 and origin of time 2Mg
x= .......... (1)
(i.e. initial time) at t = 1 sec. k
y = 0.1 sin (2pt + 4x) For block of mass m to just move up the incline

Shifting the origin of position to left by 2m, that is, to kx = mg sin q + µ mg cos q .......... (2)
x = 0. Also shifting the origin of time backwards by 1 3 3 4 3
sec, that is to t = 0 sec. 2Mg = mg ´ + mg ´ or M= m
5 4 5 5
y = 0.1 sin (2p (t – 1) + 4 (x – 2)) 18. (a) For a particle undergoing SHM with an amplitude A
15. (c) Weight of the body and angular frequency w, the maximum acceleration =
T (upthrust) w2 A
4 3 F Here the maximum force on the particle = QE0
W = mg = pr rg (viscous r
3
force) QE 0
\ maximum acceleration = = w2A
4 3 m
T= pr sg
3 W(weight) QE 0
\ A=
mw 2
and F = 6phvr
1 hc
When the body attains terminal velocity net force 19. (d) mv2 = -f
2 l
acting on the body is zero. i.e.,
1 hc 4hc 4
mn '2 = -f= - f . Clearly, v ¢ > v
W–T –F=0 2 (3l / 4) 3l 3

2 r 2 (r - s)g 1 æ 1 1 ö
= (m - 1) ç -
And terminal velocity v = 20. (c) f è R1 R 2 ÷ø
9 h
R = 10 cm.
As in case of upward motion upward force is twice its
effective weight, therefore, it will move with same æ 1 1ö 10
f ¢ = (3 - 1) ç - =-
speed 10 cm/s è -10 10 ÷ø 4
16. (c) Given : m = 0.160 kg
q = 60°
v = 10 m/s
® ®
Angular momentum L = r ´ m v
f=20 f' f=20
= H mv cos q
MT-16 Solutions Mock Test -2

-dV
1 1 4 1 2 4 23. (–20) E = V = 5x2 + 10x –9,
= - + = - dx
f eq 20 10 20 20 10
-d
E= (5x2 + 10x – 9) = – (10x + 10)
10 dx
Þ f eq = - cm
3 On putting value of x in it we get,

t /T 7.5/T E = –(10 × 1 + 10) = –20 V/m


A æ 1ö 1 æ 1ö
21. (1.5)Using =ç ÷ or =ç ÷
A0 è 2ø 32 è 2 ø 24. (15) Process1®2 and Process3®4 are isochoric processes.
W12 = 0, W34 = 0, W23 = nR (T3 – T2) = 3R (1600 –
5 7.5/T 400)
æ 1ö æ 1ö 7.5
or ç ÷ = ç ÷ or 5 = i.e., T = 1.5 hours
è 2ø è 2ø T = 3600 R
W41 = nR (T1 – T4) = 3R (200 – 800) = –1800 R
22. (5) D
W = (3600 – 1800) R = 1800R = 15 kJ

X Y 1 2 1 2
25. (18) E1 = kx , E 2 = ky ,
2 2
C
1
E= k(x + y) 2 = E1 + E 2 + 2 E1E 2
2
Equivalent resistance = 5W
= 2 + 8 + 2 16 = 18J

You might also like